MA8251 Notes 1

You might also like

Download as pdf or txt
Download as pdf or txt
You are on page 1of 828

Downloaded From : www.EasyEngineering.

net

ww
w.E
a syE
ngi
nee
rin
g.n
et

**Note: Other Websites/Blogs Owners Please do not Copy (or) Republish


this Materials, Students & Graduates if You Find the Same Materials with
EasyEngineering.net Watermarks or Logo, Kindly report us to
easyengineeringnet@gmail.com
Downloaded From : www.EasyEngineering.net
Downloaded From : www.EasyEngineering.net

ww
w.E
UNIT 1 asy
En
gin
ee
By EasyEngineering.net rin
g.n
et

Downloaded From : www.EasyEngineering.net


Downloaded From : www.EasyEngineering.net

ww
w.E
asy
En
gin
ee rin
g.n
et

Downloaded From : www.EasyEngineering.net


Downloaded From : www.EasyEngineering.net

1 Matrices

1.1 Introduction

ww The theory of matrices was first introduced by the well known

w.E
mathematician, Arthur Cayley in the second half of the nineteenth century. For
more than 65 years after its introduction, the application of it was never felt by

asy
any of the mathematicians. It’s validity was first felt by mathematicians when
Heisenberg used the theory of matrices in quantum mechanics. Its applications

En
are plenty in the fields of almost all branches of science and engineering. One can

gin
notice the use of matrices in Astronomy, Mechanics, Nuclear physics, Electrical
circuits, Differential equations, Computer graphics, Graph theory, Optimization
techniques and so on.
ee rin
Many research articles are noticed on the applications of the inverse of
the matrices and eigen value properties on the study of equilibrium analysis of

g.n
industrial organizations. A recent research paper in the year 2010 in one of the

et
most popular research journals reveals a special class of Mk matrices which arise
from strategic behaviour in industrial organizations and by suggesting a new
inverse, lead directly to closed form expressions of strategic equilibria for
arbitrary coalition structures in linear oligopolies, which include four classes of
oligopoly equilibria, Bertrand equilibria for an arbitrary coalition structure,
Cournot equilibria for an arbitrary coalition structure, Bertrand and Cournot
equilibria with multi product firms which are previously unknown and now it will
be useful to scholars in industrial organization.

Downloaded From : www.EasyEngineering.net


Downloaded From : www.EasyEngineering.net

2 Engineering Mathematics - II

Many applications of matrices can be cited on recent studies, for example


in environmental analysis, Thermosetting composite fibres and Polyester via
matrices. As matrices are widely used by engineers in industry, it is essential for
every engineering graduate to have a thorough knowledge about matrices. As you
are exposed already to the basic concepts of operations on matrices like addition,
subtraction, scalar multiplication, multiplication of the matrices, let us now
concentrate on further analysis on matrices. In the lower classes you are aware of
the concepts of finding the inverse of a square matrix, solution of simultaneous

ww equations using matrices, rank of a matrix. Let us concentrate on the evaluation


of eigen values and its applications and move further to know more on matrix

w.E
theory.

asy
1.2 Eigen values and Eigen vectors of a Real Matrix

Definition En
gin
Let A be a square matrix of order n. A number λ is called an eigen value

ee
of A if there exists a nonzero column matrix X such that AX = λX. Then X is
called an eigen vector of A corresponding to λ.
Note
rin
g.n
If λ is an eigen value and X is an eigen vector corresponding to λ, then
AX = λX ⇒ AX − λX = 0 ⇒ (A − λI)X = 0.

et
This is a homogeneous system of equations. It will have a nontrivial
solution if |A − λI| = 0. This equation is called the characteristic equation.
|A − λI| is called the characteristic polynomial of A. It is an nth degree polynomial
in λ. The roots of the characteristic equation are called the eigen values of A.
(Eigen in German means characteristic)
The following are simple ways to find the characteristic equation of a given
square matrix.

Downloaded From : www.EasyEngineering.net


Downloaded From : www.EasyEngineering.net

Matrices 3

Results

1. If A is a 2 × 2 matrix, then the characteristic equation takes the form


λ2 − s1 λ + s2 = 0 where s1 = sum of the diagonal elements of A = trace of A
(tr(A)) and s2 = |A|.

2. If A is a square matrix of order 3, then the characteristic equation


|A − λI| = 0 takes the form λ3 − s1 λ2 + s2 λ − s3 = 0 where
s1 = sum of the main diagonal elements of A.

ww s2 = sum of the minors of the elements of the main diagonal.


s3 = |A|.

w.E 3. Sum of the eigen values of a matrix A is equal to the trace of the matrix.

asy
4. Product of the eigen values = |A|.
✎ ☞

En ✍
Worked Examples

gin
 
 1 −2
Example 1.1. Find the eigen values and eigen vectors of the matrix   .

−5 4 

Solution. Let A = 
−5 4 
Step 1. To find the eigen values

 1 −2
.
 ee rin
Since A is a 2 × 2 matrix, the characteristic equation takes the form
g.n
where s1 = sum
λ2 − s1 λ + s2 = 0
et
of the diagonal elements of A = 1 + 4 = 5.
1 −2
s2 = |A| = = 4 − 10 = −6.
−5 4
∴ (1) becomes
λ2 − 5λ − 6 = 0 ⇒ (λ − 6)(λ + 1) = 0 ⇒ λ = −1, λ = 6.
∴ The eigen values are −1 and 6.

Downloaded From : www.EasyEngineering.net


Downloaded From : www.EasyEngineering.net

4 Engineering Mathematics - II

Step 2. To find the eigen vectors


 
 x1 
Let X =   be the eigen vector of A corresponding to λ.
x2

∴ (A − λI)X =0.
  
1 − λ −2   x1 


   =0.
  (1)
−5 4 − λ  x2 

ww when λ = −1, (1) becomes

w.E
  
 2 −2  x1 


   = 0
  (2)
−5 5   x2 

asy 2x1 − 2x2 = 0 ⇒ x1 − x2 = 0.

−5x1 + 5x2 = 0 ⇒ x1 − x2 = 0.

En
Both the equations are same. They are reduced to one single equation namely
x1 = x2 =⇒
x1
1
=
x2
1
gin
=⇒ x1 = 1, x2 = 1.
 
1

 ee
∴ The eigen vector X1 corresponding to λ = −1 is  .
 
−5 −2  x1 
When λ = 6, (1) becomes     = 0.
1

rin
g.n
 
−5 −2  x2 
−5x1 − 2x2 = 0 ⇒ 5x1 + 2x2 = 0

−5x1 − 2x2 = 0 ⇒ 5x1 + 2x2 = 0.

We have the same situation as above. Both the equations are reduced to 5x1 = −2x2
et
x1 x2
=⇒ =
2 −5
=⇒ x1 = 2, x2 = −5.
 
 2 
Hence, the eigen vector X2 corresponding to the eigen value λ = 6 is  .
−5

Downloaded From : www.EasyEngineering.net


Downloaded From : www.EasyEngineering.net

Matrices 5

 
4 1
Example 1.2. Find the eigen values and eigen vectors of the matrix  .

3 2
  [Jan 1997]
4 1
Solution. Let A =  .

3 2
Step 1. To find the eigen values
Since A is a 2 × 2 matrix, the characteristic equation takes the form

λ2 − s1 λ + s2 = 0

ww where, s1 = sum of the main diagonal elements of A = 4 + 2 = 6.


4 1

w.E s2 = |A| = = 8 − 3 = 5.
3 2
The characteristic equation is λ2 − 6λ + 5 = 0 ⇒ (λ − 1)(λ − 5) = 0 ⇒ λ = 1, λ = 5.

 
 x1  asy
Step 2. To find the eigen vectors

En
Let X =   be the eigen vector of A corresponding to λ.
x2

gin ∴ (A − λI)X =0

ee
  
4 − λ 1   x1 
=⇒      =0 (1)

rin
3 2 − λ  x2 

  
3 1  x1 
When λ = 1, the equation (1) becomes     = 0 g.n
et

 
3 1  x2 
x2 x1
i.e., 3x1 + x2 = 0 ⇒ x2 = −3x1 ⇒ =
−3 1
=⇒ x1 = 1, x2 = −3.
 
 1 
∴ The eigen vector corresponding to λ = 1 is X1 =  .
−3
  
−1 1   x1 
When λ = 5,(1) becomes     = 0
 
3 −3  x2 

Downloaded From : www.EasyEngineering.net


Downloaded From : www.EasyEngineering.net

6 Engineering Mathematics - II

i.e., −x1 + x2 = 0

⇒ x1 = x2 = 1
 
1
Hence, the eigen vector X2 corresponding to the eigen value λ = 5 is X2 =  .
1

Note. In the above example in case  (i), if we take  x1 = k, we get x2 = −3k.

ww
 k   1 
The corresponding eigen vector is   = k  .
−3k −3
i.e., we get infinite number of eigen vectors corresponding to an eigen value.

w.E i.e., the eigen vector corresponding to an eigen value is not unique.

asy
Type-I: Evaluation of eigen vectors when all the eigen values are different.

En 
3 −4 4

gin
 
Example 1.3. Find the eigen values and eigen vectors of the matrix 1 −2 4.
 
 
1 −1 3
 

3 −4 4

Solution. Let A = 1 −2 4.






ee rin
1 −1 3

g.n
 
Step 1. To find the eigen values
Since A is a 3 × 3 matrix, the characteristic equation takes the form

λ3 − s1 λ2 + s2 λ − s3 = 0
et
s1 = tr(A) = 3 − 2 + 3 = 4.
s2 = sum of the minors of the main diagonal elements of A.

−2 4 3 4 3 −4
= + +
−1 3 1 3 1 −2
s2 = −6 + 4 + 9 − 4 − 6 + 4 = −2 + 5 − 2 = 1.

Downloaded From : www.EasyEngineering.net


Downloaded From : www.EasyEngineering.net

Matrices 7


3 −4 4

s3 = |A| = 1 −2 4 = 3(−2) + 4(−1) + 4 = −6 − 4 + 4 = −6.

1 −1 3
The characteristic equation is λ3 − s1 λ2 + s2 λ − s3 = 0.
i.e., λ3 − 4λ2 + λ + 6 = 0.
λ = −1 is a root.
By synthetic division,
−1 1 −4 1 6

ww 0
1
−1
−5
5
6
−6
0

w.E λ2 − 5λ + 6 = 0 ⇒ (λ − 2)(λ − 3) = 0 ⇒ λ = 2, λ = 3.

The eigen values are −1, 2, 3.

asy
Step 2. To find the eigen vectors

En
 
 x1 
 
Let X =  x2  be an eigen vector corresponding to λ.
 
 
x3
 
gin

3 − λ

ee −4
(A − λI)X = 0

4
 
  x1 
rin
g.n
   
−2 − λ 4   x2  = 0. (1)
  
 1   

1 −1 3 − λ x3
 

When λ = −1, (1) becomes




 
4 −4 4  x1 
  
et
1 −1 4  x2  = 0
  
   
1 −1 4 x3
The equations are
4x1 − 4x2 + 4x3 = 0

x1 − x2 + 4x3 = 0

Downloaded From : www.EasyEngineering.net


Downloaded From : www.EasyEngineering.net

8 Engineering Mathematics - II

x1 − x2 + 4x3 = 0.

Taking the first two equations and on solving by the method of cross
multiplication, we obtain
x1 x2 x3
−4 4 4 −4
−1 4 1 −1

ww x1
=
−12 −12
x1
x2

x2
=
x3
x3
0

w.E 1
=
1
=
0
⇒ x1 = 1, x2 = 1, x3 = 0.

asy
 
1
 
⇒ X1 = 1 .
 

En
 
0
 

When λ = 2, (1) becomes


gin   
1 −4 4  x1 

ee
   
1 −4 4  x2  = 0
   

rin
   
1 −1 1 x3
The equations are
x1 − 4x2 + 4x3 = 0
g.n
x1 − 4x2 + 4x3 = 0

x1 − x2 + x3 = 0.
et
Taking the last two equations and on solving by the method of cross multiplication,
we obtain
x1 x2 x3
−4 4 1 −4
−1 1 1 −1

Downloaded From : www.EasyEngineering.net


Downloaded From : www.EasyEngineering.net

Matrices 9

x1 x2 x3
= =
0 3 3
x1 x2 x3
= =
0 1 1
⇒ x1 = 0, x2 = 1, x3 = 1.
 
0
 
X2 = 1 .
 
1

ww When λ = 3, (1) becomes


  
0 −4 4  x1 

w.E
   
1 −5 4  x2  = 0
  
   
1 −1 0 x3

asy −4x2 + 4x3 = 0 ⇒ x2 = x3

En
x1 − 5x2 + 4x3 = 0 ⇒ x1 − x2 = 0

x1 = x2 ⇒ x1 = x2 = x3 = 1.

gin  
1
 

ee ∴ X3 = 1
 
 
1

rin
 
     
1 0 1
     
∴ The eigen vectors are 1 , 1 and 1.
   
      g.n
0 1 1
et
     

 
 1 2 1 
 
Example 1.4. Find the eigen values and eigen vectors of the matrix  6 −1 0 .
 
 
−1 −2 −1
 

  [Jan 2013]
 1 2 1 

 
Solution. Let A =  6 −1 0 .

 
−1 −2 −1
 

Downloaded From : www.EasyEngineering.net


Downloaded From : www.EasyEngineering.net

10 Engineering Mathematics - II

Step 1. To find the eigen values


Since A is a 3 × 3 matrix, the characteristic equation takes the form

λ3 − s1 λ2 + s2 λ − s3 = 0

where s1 = sum of the main diagonal elements of A


s1 = 1 − 1 − 1 = −1.
s2 = sum of the minors of the main diagonal elements

−1 0 1 1 1 2
= + +

ww

−2 −1 −1 −1 6 −1

= 1 − 0 + (−1) + 1 + (−1 − 12)

w.E = 1 + 0 − 13

asy
= −12.

1 2 1

s3 = |A| = 6 −1 0

−1 −2 −1
En

gin
= 1(1 − 0) − 2(−6 − 0) + 1(−12 − 1)

= 1 + 12 − 13

= 0.
The characteristic equation is
ee rin
3 2
λ + λ − 12λ = 0
g.n
λ(λ2 + λ − 12) = 0

λ(λ + 4)(λ − 3) = 0

λ = 0, −4, 3.
et
The eigen values are 0, −4, 3.
Step 2. To find the eigen vectors
 
 x1 
 
Let X =  x2  be the eigen vector corresponding to the eigen value λ.
 
x3

Downloaded From : www.EasyEngineering.net


Downloaded From : www.EasyEngineering.net

Matrices 11

∴ (A − λI)X = 0
  
1 − λ 2 1   x1 
   
i.e.,  6 −1 − λ 0   x2  = 0. (1)
  
   
−1 −2 −1 − λ x3
  

When λ = 0, (1) becomes


  
 1 2 1   x1 
   
  x2  = 0
   
6 −1 0

ww

   
−1 −2 −1 x3

w.E i.e., x1 + 2x2 + x3 = 0 (2)

asy
6x1 − x2 = 0 (3)

−x1 − 2x2 − x3 = 0. (4)

(3) can be written as


En
gin 6x1 = x2
x2

ee x1 =

⇒ x1 = 1, x2 = 6.
6

rin
Substituting in (2) we obtain, 1 + 12 + x3 = 0
g.n

 1 

∴ The eigen vector is X1 =  6 .




⇒ x3 = −13.

et
 
−13
 
When λ = −4, (1) becomes
  
 5 2 1  x1 
   
  x2  = 0
   
 6 3 0
   
−1 −2 3 x3

Downloaded From : www.EasyEngineering.net


Downloaded From : www.EasyEngineering.net

12 Engineering Mathematics - II

i.e., 5x1 + 2x2 + x3 = 0 (5)

6x1 + 3x2 = 0 (6)

−x1 − 2x2 + 3x3 = 0. (7)

(6) can be written as


6x1 = −3x2
x1 x2
=
−3 6

ww x1
1
=
x2
−2
∴ x1 = 1, x2 = −2.

w.E
Substituting in (5) we obtain

asy 5 − 4 + x3 = 0

⇒ x3 = −1.
 
 1 
En
gin
 
∴ The eigen vector is X2 = −2.
 
 
−1
When λ = 3, (1) becomes

−2 2


ee 1
 
  x1 
  
 6 −4 0   x2  = 0.
  
rin

−1 −2 −4 x3
  
g.n
i.e., − 2x1 + 2x2 + x3 = 0

6x1 − 4x2 = 0
et (8)

(9)

−x1 − 2x2 − 4x3 = 0. (10)

(9) can be written as


6x1 = 4x2

⇒ 3x1 = 2x2

Downloaded From : www.EasyEngineering.net


Downloaded From : www.EasyEngineering.net

Matrices 13

x1 x2
⇒ =
2 3
∴ x1 = 2, x2 = 3.

Substituting in (8) we obtain


−4 + 6 + x3 = 0

2 + x3 = 0

⇒ x3 = −2.

ww  
 2 
 

w.E
∴ The eigen vector is X3 =  3 .
 
 
−2
 
     

asy
 1   1   2 
     
∴ The eigen vectors are  6  , −2 and  3 .
 

En
     
−13 −1 −2

gin
Type-II: Evaluation of Eigen vectors when two of the eigen values are equal.

ee
 
−2 2 −3
 
Example 1.5. Find the eigen values and eigen vectors of  2 1 −6.[Jan 2009]
rin

 
−1 −2 0
 

−2 2 −3



g.n
Solution. Let A =  2 1 −6.

et
 
 
−1 −2 0
 
Step 1. To find the eigen values
Since A is a 3 × 3 matrix, the characteristic equation takes the form

λ3 − s1 λ2 + s2 λ − s3 = 0

where s1 = tr(A) = −2 + 1 + 0 = −1.


s2 = sum of the minors of the main diagonal elements of A.

Downloaded From : www.EasyEngineering.net


Downloaded From : www.EasyEngineering.net

14 Engineering Mathematics - II


1 −6 −2 −3 −2 2
= + +
−2 0 −1 0 2 1

s2 = 0 − 12 + 0 − 3 − 2 − 4 = −21.

−2 2 −3

s3 = |A| = 2 1 −6

−1 −2 0

= −2(0 − 12) − 2(0 − 6) − 3(−4 + 1) = 24 + 12 + 9 = 45.

ww The characteristic equation is λ3 + λ2 − 21λ − 45 = 0.


λ = −3 is a root. By synthetic division,

w.E −3 1
0
1
−3
−21
6
−45
45

asy 1 −2 −15 0

En =⇒ (λ + 3)(λ2 − 2λ − 15) = 0

gin
(λ + 3)(λ − 5)(λ + 3) = 0 ⇒ λ = −3, λ = −3, λ = 5.

The eigen values are −3, −3, 5.


Step 2. To find the eigen vectors
 
 x1 
 
ee rin
g.n
Let X =  x2  be an eigen vector corresponding to λ.
 
x3
 


−2 − λ


2
(A − λI)X = 0.

−3
 
  x1 
  
et
1 − λ −6   x2  = 0. (1)
  
 2   

−1 −2 0 − λ x3
 
  
 1 2 −3   x1 
   
When λ = −3, (1) becomes  2 4 −6  x2  = 0.
  
   
−1 −2 3 x3
  

Downloaded From : www.EasyEngineering.net


Downloaded From : www.EasyEngineering.net

Matrices 15

The above three equations are reduced to

x1 + 2x2 − 3x3 = 0. (2)

Since two of the eigen values are equal, from (2) we have to get two eigen vectors
by assigning arbitrary values for two variables. First, choosing x3 = 0, we obtain
x1 x2
x1 + 2x2 = 0 which implies x1 = −2x2 ⇒ = ⇒ x1 = 2, x2 = −1
2 −1
 
 2 

ww
 
∴ X1 = −1 .
 
 
0
 

w.E
Also in (2) assign x2 = 0, we obtain x1 − 3x3 = 0,
x1 x3

asy
which implies x1 = 3x3 ⇒ = ⇒ x1 = 3, x3 = 1.
3 1
 

En
3
 
X2 = 0 .

gin
 
1
 

ee
  
−7 2 −3  x1 
   
When λ = 5, (1)=⇒  2 −4 −6  x2  = 0.
rin
  
   
−1 −2 −5 x3
  
The equations are
−7x1 + 2x2 − 3x3 = 0 

 g.n
et





(A)

2x1 − 4x2 − 6x3 = 0 



−x1 − 2x2 − 5x3 = 0. 

Taking the first two equations and on solving by the method of cross
multiplication, we obtain
x1 x2 x3
2 −3 −7 2
−4 −6 2 −4

Downloaded From : www.EasyEngineering.net


Downloaded From : www.EasyEngineering.net

16 Engineering Mathematics - II

x1 x2 x3
= =
−12 − 12 −6 − 42 28 − 4
x1 x2 x3
= =
−24 −48 24
x1 x2 x3
= = .
1 2 −1

=⇒ x1 = 1, x2 = 2, x3 = −1.

ww The values of x1 , x2 , x3 satisfy the last equation in (A).

w.E
 
 1 
 
∴ X3 =  2  .
 

asy
 
−1
 

En
     
 2  3  1 
     

gin
∴ The eigen vectors are −1 , 0 and  2 .
     
     
0 1 −1
     

ee 
2 2 1

Example 1.6. Find the eigen values and eigen vectors of 1 3 1.




rin
g.n
 
1 2 2
 
[Jun 2010, Jan 2007]

et
 
2 2 1
 
Solution. Let A = 1 3 1.
 
 
1 2 2
 
Step 1. To find the eigen values
Since A is a 3 × 3 matrix, the characteristic equation takes the form
λ3 − s1 λ2 + s2 λ − s3 = 0.
Now, s1 = sum of the main diagonal elements of A
= 2 + 3 + 2 = 7.

Downloaded From : www.EasyEngineering.net


Downloaded From : www.EasyEngineering.net

Matrices 17

s2 = sum of the minors of the main diagonal elements of A.



3 1 2 1 2 2
= + +
2 2 1 2 1 3

= (6 − 2) + (4 − 1) + (6 − 2)

=4+3+4

= 11.

2 2 1

ww

s3 = |A| = 1 3 1

1 2 2

w.E = 2(6 − 2) − 2(2 − 1) + 1(2 − 3)

=8−2−1

= 5. asy
En
The characterstic equation is λ3 − 7λ2 + 11λ − 5 = 0.
λ = 1 is a root.
By synthetic division we have
gin
1 1
0
ee −7
1
11
−6
−5
5
rin
1 −6 5
λ3 − 6λ + 5 = 0
0

g.n
(λ − 1)(λ − 5) = 0

λ = 1, 5.
et
The given eigen values are 1, 1, 5.
Step 2. To find the eigen vectors
 
 x1 
 
Let X =  x2  be the eigen vector corresponding to an eigen value λ.
 
x3

Downloaded From : www.EasyEngineering.net


Downloaded From : www.EasyEngineering.net

18 Engineering Mathematics - II

∴ (A − λI)X = 0
  
2 − λ 2 1   x1 
   
3−λ 1   x2  = 0. (1)

 1
   
1 2 2 − λ x3

When λ = 1, (1) becomes


  
1 2 1  x1 
   

ww 1 2 1  x2  = 0.


   
   
1 2 1 x3

w.E
All the three equations are reduced to one single equation
x1 + 2x2 + x3 = 0 (2)

asy
Since two of the eigen values are equal, from(2), we have to get two eigen vectors
by assigning arbitrary values for two variables.
First, choosing x3 = 0, we obtain
En x1 + 2x2 = 0

gin x1 = −2x2

ee x1
−2
=
x2
1
⇒ x1 = −2, x2 = 1
rin
 
−2
  g.n
et
∴ One eigen vector is X1 =  1 .
 
0
Assign x2 = 0, (2) becomes
x1 + x3 = 0

x1 = −x3
x1 x3
=
1 −1
⇒ x1 = 1, x3 = −1.

Downloaded From : www.EasyEngineering.net


Downloaded From : www.EasyEngineering.net

Matrices 19

 
 1 
 
∴ The second eigen vector is X2 =  0 .
 
 
−1
 
When λ = 5, (1) becomes
  
−3 2 1   x1 
   
 1 −2 1   x2  = 0.

   
1 2 −3 x3
The equations become

ww

−3x1 + 2x2 + x3 = 0 






(A)

x1 − 2x2 + x3 =0 

w.E




x1 + 2x2 − 3x3 = 0. 

All the three equations are different.

asy
Taking the first two equations and applying the rule of cross multiplication we
obtain

En x1 x2 x3

−2
2
gin 1
1
−3
1
2
−2

ee x1
=
x2
2+2 1+3 6−2
=
x3
rin
x1
4
=
x2
4
=
x3
4 g.n
 
1
 
⇒ x1 = 1, x2 = 1, x3 = 1.
et
The eigen vector is X3 = 1.
 
 
1
 
     
−2  1  1
     
∴ The eigen vectors are  1  ,  0  , 1.
     
     
0 −1 1
 

Downloaded From : www.EasyEngineering.net


Downloaded From : www.EasyEngineering.net

20 Engineering Mathematics - II

 
 3 10 5 
 
Example 1.7. Find the eigen values and eigen vectors of A = −2 −3 −4.
 
 
3 5 7
 
[May 2000]
 
 3 10 5 
 
Solution. Given A = −2 −3 −4.
 
 
3 5 7
 
Step 1. To find the eigen values

ww Since A is a 3 × 3 matrix, the characteristic equation takes the form


λ3 − S 1 λ2 + S 2 λ − S 3 = 0.

w.E Now, S 1 = sum of the main diagonal elements

asy
= 3 − 3 + 7 = 7.

S 2 = sum of the minors of the main diagonal elements of A.



= En

+

+

−3 −4 3 5 3 10

gin

5 7 3 7 −2 −3

= −21 + 20 + 21 − 15 + (−9 + 20)

= −1 + 6 + 11

= 16.
ee rin
S 3 = |A|
g.n
et

3 10 5

= −2 −3 −4

3 5 7

= 3(−21 + 20) − 10(−14 + 12) + 5(−10 + 9)

= −3 + 20 + (−5)

= 20 − 8

= 12.

Downloaded From : www.EasyEngineering.net


Downloaded From : www.EasyEngineering.net

Matrices 21

The characteristic equation is λ3 − 7λ2 + 16λ − 12 = 0.


λ = 2 is a root.
By synthetic division, we have

2 1 −7 16 −12
0 2 −10 12
1 −5 6 0

λ2 − 5λ + 6 = 0

ww (λ − 2)(λ − 3) = 0

w.E
The eigen values are 2, 2, 3.
λ = 2, 3.

 
 x1  asy
Step 2. To find the eigen vectors

En
 
Let X =  x2  be the eigen vector corresponding to an eigen value λ.
 
x3

gin
 

∴ (A − λI)X = 0.

3 − λ
 ee 10 5
 
  x1 
  
 −2 −3 − λ −4   x2  = 0.
   
rin (1)

g.n
   
3 5 7 − λ x3
When λ = 2, (1) becomes

et
  
 1 10 5   x1 
   
−2 −5 −4  x2  = 0.
  
   
3 5 5 x3
The equations becomes
x1 + 10x2 + 5x3 = 0

2x1 + 5x2 + 4x3 = 0

3x1 + 5x2 + 5x3 = 0.

Downloaded From : www.EasyEngineering.net


Downloaded From : www.EasyEngineering.net

22 Engineering Mathematics - II

Since all the three equations are different, we will get only one eigen
vector. Since we need two eigen vectors corresponding to the twice repeated eigen
value 2, the two eigen vectors are the same.
Taking the first two equations and using the rule of cross multiplication
we obtain
x1 x2 x3
10 5 1 10
5 4 2 5

ww x1

x1
=
x2

x2
=
40 − 25 10 − 4 5 − 20
x3
x3

w.E 15
x1
=

=
6
x2
=

=
−15
x3

asy 5
∴ x1 = 5, x2 = 2, x3 = −5
2 −5

En
   
 5   5 
   
The two eigen vectors are X1 =  2  , X2 =  2 .

gin
 
   
−5 −5
 
When λ = 3, (1) becomes

 ee
 0 10 5   x1 
 
  
−2 −6 −4  x2  = 0.
   
rin
g.n
   
3 5 4 x3
The equations are
10x2 + 5x3 = 0

2x1 + 6x2 + 4x3 = 0


et
3x1 + 5x2 + 4x3 = 0.

From 10x2 + 5x3 = 0 we obtain


10x2 = −5x3

2x2 = −x3

Downloaded From : www.EasyEngineering.net


Downloaded From : www.EasyEngineering.net

Matrices 23

x2 x3
=
1 −2
∴ x2 = 1, x3 = −2.

Substituting this in the second equation we obtain

2x1 + 6 − 8 = 0

2x1 − 2 = 0

ww 2x1 = 2

w.E
x1 = 1.
 
 1 
 

asy
∴ The eigen vector is X3 =  1 .
 
 
−2
 

En
     
 5   5   1 
     
The eigen vectors are  2  ,  2  ,  1 .

gin
     
     
−5 −5 −2
     

ee
Type-III: Evaluation of eigen vectors when all the eigen values are equal.

 6 −6 5 

rin

Example 1.8. Find the eigen values and eigen vectors of A = 14 −13 10.



 g.n
7 −6 4
et
 

  [Dec 1998]
 6 −6 5 
 
Solution. Given A = 14 −13 10.
 
7 −6 4
 
Step 1. To find the eigen values
Since A is a 3 × 3 matrix, the characteristic equation takes the form

λ3 − s1 λ2 + s2 λ − s3 = 0.

Downloaded From : www.EasyEngineering.net


Downloaded From : www.EasyEngineering.net

24 Engineering Mathematics - II

Now, s1 = sum of the main diagonal elements


= 6 − 13 + 4
= −3.
s2 = sum of the minors of the main diagonal elements

−13 10 6 5 6 −6
= + +
−6 4 7 4 14 −13

= −52 + 60 + 24 − 35 + (−78 + 84)

ww
= 8 − 11 + 6

= 3.

w.E

6 −6 5

s3 = |A| = 14 −13 10

asy
7 −6 4

= 6(−52 + 60) + 6(56 − 70) + 5(−84 + 91)

En
= 6(8) + 6(−14) + 5(7) = 48 − 84 + 35

= 83 − 84
gin
ee
= −1.
The characteristic equation is

λ3 + 3λ2 + 3λ + 1 = 0.
rin
(λ + 1)3 = 0
g.n
The eigen values are −1, −1, −1.
∴ λ = −1, −1, −1.
et
Step 2. To find the eigen vectors
 
 x1 
 
Let X =  x2  be the eigen vector corresponding to an eigen value λ.
 
 
x3
 

∴ (A − λI)X = 0

Downloaded From : www.EasyEngineering.net


Downloaded From : www.EasyEngineering.net

Matrices 25

  
6 − λ −6 5   x1 
   
 14 −13 − λ 10   x2  = 0 (1)
   
   
7 −6 4 − λ x3
when λ = −1, (1) becomes
  
 7 −6 5   x1 
   
14 −12 10  x  = 0.
   2 
   
7 −6 5 x3
The above three equations are reduced to one single equation

ww 7x1 − 6x2 + 5x3 = 0.

w.E
With this single equation, we must get three different eigen vectors, which can be
achieved by assigning arbitrary values to x1 , x2 & x3 .

asy x2 x3
 
0
 

En
x1 = 0 ⇒ −6x2 + 5x3 = 0 ⇒ = ∴ X1 = 5 .
 
5 6  
6
 

gin  
 5 

ee x1 x3
 
x2 = 0 ⇒ 7x1 + 5x3 = 0 ⇒ = ∴ X2 =  0  .
 
−5 7

rin
 
−7
 

g.n
 
6
x1 x2
 
x3 = 0 ⇒ 7x1 − 6x2 = 0 ⇒ = ∴ X3 = 7 .
 
6 7

et
 
0
 

 
−5 −5 −9
 
Example 1.9. Find the eigen values and eigen vectors of A =  8 9 18 .
 
 
−2 −3 −7
 
 
−5 −5 −9
 
Solution. Given A =  8 9 18 .

 
−2 −3 −7

Downloaded From : www.EasyEngineering.net


Downloaded From : www.EasyEngineering.net

26 Engineering Mathematics - II

Step 1. To find the eigen values


Since A is a 3 × 3 matrix, the characteristic equation takes the form
λ3 − s1 λ2 + s2 λ − s3 = 0.
Now, s1 = sum of the main diagonal elements.

= −5 + 9 − 7

= 9 − 12 = −3.
s2 = sum of the minors of the main diagonal elements.

ww
9 18 −5 −9 −5 −5
= + +
−3 −7 −2 −7 8 9

w.E = −63 + 54 + 35 − 18 + (−45 + 40)

= −9 + 17 − 5 = 3.

asy

−5 −5 −9

s3 = |A| = 8 9 18

En

−2 −3 −7

gin
= −5(−63 + 54) + 5(−56 + 36) − 9(−24 + 18)

= −5(−9) + 5(−20) − 9(−6)

= 45 − 100 + 54 = −1.
The characteristic equation is
ee rin
λ3 − s1 λ2 + s2 λ − s3 = 0

λ3 + 3λ2 + 3λ + 1 = 0 g.n
(λ + 1)3 = 0

λ = −1, −1, − 1.
et
The eigen values are −1, −1, −1.
Step 2. To find the eigen vectors
 
 x1 
 
Let X =  x2  be the eigen vector corresponding to an eigen value λ.
 
 
x3
 

Downloaded From : www.EasyEngineering.net


Downloaded From : www.EasyEngineering.net

Matrices 27

∴ (A − λI)X = 0.
  
−5 − λ −5 −9   x1 
   
9−λ 18   x2  = 0. (1)

 8   

−2 −3 −7 − λ x3
 

When λ = −1, (1) becomes


  
−4 −5 −9  x1 

ww
   
 8 10 18   x2  = 0
  
   
−2 −3 −6 x3

w.E
The equations are
−4x1 − 5x2 − 9x3 = 0

asy 8x1 + 10x2 + 18x3 = 0

En −2x1 − 3x2 − 6x3 = 0.

gin
The above three equations are reduced to two equations

ee4x1 + 5x2 + 9x3 = 0

rin
2x1 + 3x2 + 6x3 = 0.
g.n
Using the rule of cross multiplication we get

5
x1
9
x2
4
x3
5
et
3 6 2 3

x1 x2 x3
= =
30 − 27 18 − 24 12 − 10
x1 x2 x3
= =
3 −6 2

Downloaded From : www.EasyEngineering.net


Downloaded From : www.EasyEngineering.net

28 Engineering Mathematics - II

 
 3 
 
The only eigen vector is X = −6.
 
 
2
 
 
 3 
 
In this case X1 = X2 = X3 = −6.
 
 
2
 

ww Exercise I(A)

w.E
I. Find the characteristic equation, eigen values and eigen vectors of the following
matrices.

asy  

En
  1 1 
 1 −2 2.  .

1.   . 3 −1

gin

−5 4 


ee
II. Find the eigen values and eigen vectors of the following matrices.

 6 −2 2 
 
 1 −1 0 

rin 
 1 1 2

g.n
 
 8 −6 2       
3. −2 3 −1 6.  1 9.  0 2 2
     
  2 1 
1. −6 7 −4
       
2 −1 3 −1 2 −1

et −1 1 3
       
2 −4 3
 
     
−2 2 −3 2 0 −1 2 1 2 
     
4.  2 7. 0 2 −2 10. 0 2 −1
     
1 −6
     
−1 −2 0 1 −1 2 0 0 2
     

       
1 0 −1 2 1 0  3 −1 3 4 2 3
       
2. 1 2 1  5. 0 2 1 8. −1 5 1 11. 0 4 0.
    
       
2 2 3 0 0 2 3 1 5 0 5 4
       

Downloaded From : www.EasyEngineering.net


Downloaded From : www.EasyEngineering.net

Matrices 29

1.3 Properties of Eigen values and Eigen vectors

Property 1.1. A square matrix A and its transpose AT have the same eigen values.

Proof. The eigen values of the matrix A are the roots of its characteristic equation
|A − λI| = 0.
Now, (A − λI)T = AT − (λI)T = AT − λI T = AT − λI.
Also, |(A − λI)T | = |A − λI| ⇒ |AT − λI| = |A − λI|.

ww This implies that the characteristic polynomial of A and AT are equal.


⇒ The characteristic equations of A and AT are equal.

w.E
⇒ A and AT have the same eigen values.

✎ ☞


asy Worked Examples


✍ ✌

En
 
−1 2 −2
√ √  
Example 1.10. 1, 5 and − 5 are the eigen values of the matrix  1 1 .

2

gin
 
−1 −1 0
 
 

ee
−1 1 −1
 
Write down the eigen values of the matrix  2 2 −1.
 


−2 1 0


rin
g.n
   
−1 2 −2 −1 1 −1
   
Solution. Let A =  1 1  and B =  2 2 −1.
   
2


−1 −1 0
B is the transpose of A.




−2 1 0

By the above property, A & B have the same eigen values.




et
√ √
∴ The eigen values of B are 1, 5 and − 5.
 
 2 2 0 
 
Example 1.11. If 1, 3 and −4 are the eigen values of the matrix  2 1 1 , what
 
 
−7 2 −3
 

Downloaded From : www.EasyEngineering.net


Downloaded From : www.EasyEngineering.net

30 Engineering Mathematics - II

 
2 2 −7
 
are the eigen values of the matrix 2 1 2 ?
 
 
0 1 −3
 
   
 2 2 0  2 2 −7
   
Solution. Let A =  2 1 1  and B = 2 1 2 .
   
   
−7 2 −3 0 1 −3
   
B is the transpose of A.
By the above property, A and B have the same eigen values.

ww ∴ The eigen values of B are 1, 3 and −4.

w.E
Property 1.2. Sum of the eigen values of a square matrix A is equal to the sum of
the elements on its main diagonal.

asy
Proof. Let A be a square matrix of order n.
The characteristic equation is |A − λI| = 0.

En
It is of the form λn − s1 λn−1 + s2 λn−2 + · · · + (−1)n sn = 0 (1)

gin
where, s1 = sum of the elements of the leading diagonal.
Let λ1 , λ2 , . . . , λn be the roots of (1).
From the theory of equations,
Sum of the roots = −
coeff. of λn−1
coeff. of λn
. ee
i.e., λ1 + λ2 + · · · + λn = −(−s1 ) = s1 = tr(A). rin
i.e., Sum of the eigen values = sum of the main diagonal elements of A.
g.n 

Property 1.3. Product of the eigen values of a square matrix A is equal to |A|.

Proof. For the square matrix A , the characteristic equation is |A − λI| = 0.


et
It is of the form λn − s1 λn−1 + s2 λn−2 + · · · + (−1)n sn = 0, (1)
where sn = |A|.
Let λ1 , λ2 , . . . , λn be the roots of (1).
From the theory of equations, we have
constant term
Product of the roots = (−1)n .
coeff. of λn

Downloaded From : www.EasyEngineering.net


Downloaded From : www.EasyEngineering.net

Matrices 31

i.e., λ1 , λ2 , . . . , λn = (−1)n (−1)n sn = (−1)2n |A|.


i.e., Product of the eigen values = |A|. 
✎ ☞
Worked Examples
✍ ✌
Example 1.12. Find the sum and product of the eigen values of the matrix
 
−2 2 −3
 
1 −6. [Dec 1999]
 
 2 

−1 −2 0

ww Solution. Sum of the eigen values = tr(A) = −2 + 1 + 0 = −1.


−2 2 −3

w.E
Product of the eigen values = 2 1 −6 = −2(−12) − 2(−6) − 3(−3) = 45.

−1 −2 0

asy
Example 1.13. Find the sum and product of the eigen values of the matrix
 
−1 1 1 
 
 1 −1 1 .

En [March 1996]
 
 
1 1 −1

gin
Solution. Sum of the eigen values = Sum of the elements along the main

ee
= −1 − 1 − 1 = −3.
diagonal

rin
g.n

−1 1 1

Product of the eigen values = |A| = 1 −1 1

et

1 1 −1

= −1(1 − 1) − 1(−1 − 1) + 1(1 + 1)

= −1 × 0 + 2 + 2 = 4.
 
 6 −2 2 
 
Example 1.14. The product of the two eigen values of the matrix A = −2 3 −1
 
 
2 −1 3
 
is 16, find the third eigen value. [Jan 2003,2012]

Downloaded From : www.EasyEngineering.net


Downloaded From : www.EasyEngineering.net

32 Engineering Mathematics - II

Solution. Let λ1 , λ2 , λ3 be the eigen values.


Given λ1 λ2 = 16.

6 −2 2

But, product of eigen values = −2 3 −1

2 −1 3

i.e., λ1 λ2 λ3 = 6(8) + 2(−4) + 2(−4)

16λ3 = 48 − 8 − 8 = 32

ww ∴ λ3 = 2.

2 0 1

w.E
 
Example 1.15. If 2 and 3 are the eigen values of 0 2 0, find the value of x.
 
 
x 0 2
 

asy
2 0 1

Solution. Let A = 0 2 0.



En

 
x 0 2
 

gin
Let λ1 , λ2 , λ3 be the eigen values of A.
We know that, λ1 + λ2 + λ3 = tr(A) = 6.

ee2 + 3 + λ3 = 6

λ3 = 1.
rin
Also, product of the eigen values = |A|.
g.n
et

2 0 1

λ1 λ2 λ3 = 0 2 0

x 0 2

2.3.1 = 2(4) − 0(0) + 1(0 − 2x)

6 = 8 − 2x

2x = 2

x = 1.

Downloaded From : www.EasyEngineering.net


Downloaded From : www.EasyEngineering.net

Matrices 33

 
a 4
Example 1.16. Find the values of a and b, such that the matrix   has 3 and

1 b
−2 its eigen values. [May 2011]
Solution. Sum of the eigen values = tr(A) = a + b
3−2=a+b
⇒a+b=1 (1)
Product of the eigen values = |A|
a 4
i.e., 3 × (−2) = = ab − 4

ww 1 b

−6 = ab − 4

w.E ab = −6 + 4 = −2

asy 2
b=− .
a
Substituting in (1) we get
En a−
2
=1

gin a
a2 − 2 = a

ee a2 − a − 2 = 0

(a − 2)(a + 1) = 0 rin
a = 2, −1.
g.n
When a = 2, b = −1.
When a = −1, b = 2.
et
Example 1.17. If the sum of two eigen values and trace of a 3 × 3 matrix A are
equal, find the value of |A|.
Solution. Let the eigen values be λ1 , λ2 , λ3 .
We know that λ1 + λ2 + λ3 = tr(A).
But, given that λ1 + λ2 = tr(A).

Downloaded From : www.EasyEngineering.net


Downloaded From : www.EasyEngineering.net

34 Engineering Mathematics - II

∴ tr(A) + λ3 = tr(A).
=⇒ λ3 = 0.
Now, |A| = product of the eigen values = λ1 × λ2 × 0 = 0.

Property 1.4. If λ1 , λ2 , . . . , λn are the non zero eigen values of a square matrix A of
1 1 1
order n, then , , . . . , are the eigen values of A−1 .
λ1 λ2 λn
Proof. Let λ be a non zero eigen value of A. Then, there exists a non zero column
matrix X such that

ww AX = λX. (1)

w.E
Since all the eigen values are non zero, A is non-singular. Hence, A−1 exists.
Premultiplying both sides of (1) by A−1 we get,

asy A−1 (AX) = A−1 λX

En (A−1 A)X = λ(A−1 X)

gin IX = λA−1 X
1


1
λ
is an eigen value of A−1 .
ee λ
X = A−1 X.

rin
This is true for all eigen values of A.

1 1
, ,...,
λ1 λ2
1
λn
are the eigen values of A−1 . g.n 

Property 1.5. If λ1 , λ2 , . . . , λn are the eigen values of A, then


(i) cλ1 , cλ2 , . . . , cλn are the eigen values of cA where c , 0.
et
(ii) λm m m m m
1 , λ2 , λ3 , . . . , λn are the eigen values of A where m is a positive integer.

Proof. Let λ be an eigen value of A.


Then, there exists a column matrix X such that

AX = λX. (1)

Downloaded From : www.EasyEngineering.net


Downloaded From : www.EasyEngineering.net

Matrices 35

(i). Let c , 0.
Multiply (1) by c we get, c(AX) = c(λX)

(cA)X = (cλ)X.

⇒ cλ is an eigen value of cA.


This is true for all eigen values of A.
∴ cλ1 , cλ2 , . . . , cλn are the eigen values of cA.
(ii) We have

ww AX = λX

w.E =⇒ A(AX) = A(λX)

=⇒ (AA)X = λ(AX)

asy A2 X = λ(λX)

En =⇒ A2 X = λ2 X.

=⇒ λ2 is an eigen value of A2 .
gin
ee
A(A2 X) = A(λ2 X)

=⇒ (AA2 )X = λ2 (AX)
rin
=⇒ A3 X = λ2 λX = λ3 X.
g.n
⇒ λm is an eigen value of Am .
This is true for all eigen values of A.
et
=⇒ λ3 is an eigen value of A3 . Proceeding like this, we arrive at Am X = λm X.

∴ λm m m m
1 , λ2 , . . . , λn are the eigen values of A . 

Downloaded From : www.EasyEngineering.net


Downloaded From : www.EasyEngineering.net

36 Engineering Mathematics - II

✎ ☞
Worked Examples
✍ ✌
 
−1 0 0
 
Example 1.18. Given  2 −3 0. Find the eigen values of A2 . [Jan 2010]

 
1 4 2
 
Solution. The given matrix is triangular.
Hence, the eigen values are the elements along the leading diagonal.
The Eigen values of A are −1, −3, 2.
∴ The Eigen values of A2 are (−1)2 , (−3)2 , 22 .

ww i.e., 1, 9, 4.

w.E
 
 3 10 5 
 
Example 1.19. If 2 and 3 are the eigen values of A = −2 −3 −4, find the eigen
 
 

asy 3 5 7
 

values of A−1 and A3 . [Dec 2000]

En
Solution. Let λ be the third eigen value.
we know that, sum of the eigen values = tr(A).

gin
2+3+λ=3−3+7

The Eigen values of A are 2, 2, 3.


ee
5 + λ = 7 ⇒ λ = 2.

rin
1 1 1
The Eigen values of A−1 are , , .
2 2 3 g.n
The Eigen values of A3 are 23 , 23 , 33 (i.e) 8, 8, 27.

2 2 1


 et
Example 1.20. Two of the eigen values of the matrix A = 1 3 1 are equal to 1

 
1 2 2
 

each. Find the eigen values of A−1 . [Dec.2002]


Solution. Let λ1 , λ2 , λ3 be the eigen values of A.
Given λ1 = λ2 = 1.
∴ λ1 + λ2 + λ3 = sum of the elements along the main diagonal

Downloaded From : www.EasyEngineering.net


Downloaded From : www.EasyEngineering.net

Matrices 37

1 + 1 + λ3 = 2 + 3 + 2
2 + λ3 = 7
λ3 = 5.
∴ The eigen values of A are 1,1,5
1
∴ Eigen values of A−1 are 1, 1, .
5
 
 3 −1 1 
 
Example 1.21. Two of the eigen values of the matrix A = −1 5 −1 are 3 and
 

ww
 
1 −1 3
 

6. Find the eigen values of A−1 . [May 2002]

w.E
Solution. Let λ1 , λ2 andλ3 be the eigen values of the given matrix A.
∴ λ1 + λ2 + λ3 = sum of the elements along the main diagonal

asy
3 + 6 + λ3 = 3 + 5 + 3

9 + λ3 = 11

λ3 = 2.
En
The eigen values of A are 3, 6, 2.
1 1 1
∴ The eigen values of A−1 are , , .
3 6 2gin
ee  
 1 −2
Example 1.22. Find the eigen values of the matrix 
−5 4  rin
. Hence, find the

matrix whose eigen values are 16 and −1.


 
 1 −2 g.n
[Jan 2001]

Solution. Let A = 


−5 4 
.

The characteristic equation is |A − λI| = 0.


et
This is of the form λ2 − s1 λ + s2 = 0.

s1 = tr(A) = 1 + 4 = 5

1 −2
s2 = = 4 − 10 = −6.
−5 4

Downloaded From : www.EasyEngineering.net


Downloaded From : www.EasyEngineering.net

38 Engineering Mathematics - II

The characteristic equation is λ2 − 5λ − 6 = 0 ⇒ (λ − 6)(λ + 1) = 0 ⇒ λ = −1, 6.


1 1
We know that, the matrix whose eigen values are and is A−1 .
−1 6
 
1 T −1 4 2
∴ A−1 = A = .
|A| c

6 5 1
 
3 2 1
 
Example 1.23. Find the eigen values of ad j(A) if A = 0 4 2

 
0 0 1
 

ww 
3 2 1

Solution. Given A = 0 4 2




w.E 

0 0 1


Since A is triangular, the eigen values are the elements along the main diagonal.

We have A−1 = asy


∴ The eigen values are 3, 4, 1.
1
ad j(A)
⇒ ad j(A) = |A| A −1
En
|A|

1 1

gin
The eigen values of A−1 are , , 1
3 4
1 1
∴ The eigen values of ad j(A) are |A| × , |A| × , |A| × 1

3 2 1


ee
Now, |A| = 0 4 2 = 3 × 4 × 1 = 12.
3 4

rin
g.n

0 0 1
1 1
∴ Eigen values of ad j(A) are 12 × , 12 × , 12 × 1
i.e., 4, 3, 12.
3 4


2 0 1



et
Example 1.24. Find the eigen values of ad j(A) if A = 0 2 0.
 
 
1 0 2
 
Step 1. To find the eigen values of A
Since A is a 3 × 3 matrix, the characteristic equation is of the form

λ3 − s1 λ2 + s2 λ − s3 = 0.

Downloaded From : www.EasyEngineering.net


Downloaded From : www.EasyEngineering.net

Matrices 39

Now, s1 = sum of the main diagonal elements

= 2 + 2 + 2 = 6.

s2 = sum of the minors of the main diagonal elements.



2 0 2 1 2 0
= + +
0 2 1 2 0 2

=4−0+4−1+4−0

= 4 + 3 + 4 = 11.

ww s3 = |A|

w.E
2 0 1

= 0 2 0

1 0 2

asy
= 2(4 − 0) − 0 + 1(0 − 2) = 8 − 2 = 6.
The characteristic equation is

En
gin
λ3 − 6λ2 + 11λ − 6 = 0.

By synthetic division, we have ee


λ = 1 is a root.

rin
1 1
0
−6
1
11
−5
−6
6 g.n
1 −5 6

λ2 − 5λ + 6 = 0
0
et
(λ − 3)(λ − 2) = 0

λ = 2, 3.

The eigen values of A are 1, 2, 3.


We know that ad j(A) = |A| A−1 .

Downloaded From : www.EasyEngineering.net


Downloaded From : www.EasyEngineering.net

40 Engineering Mathematics - II

1 1 1
∴ The eigen values of ad j(A) are |A| × , |A| × , |A| ×
1 2 3
1 1
i.e.,6 × 1, 6 × , 6 ×
2 3
i.e.,6, 3, 2

Property 1.6. If λ1 , λ2 , . . . , λn are the eigen values of A, then


(i) λ1 − k, λ2 − k, . . . , λn − k are the eigen values of A − kI.
(ii) a0 λ21 + a1 λ1 + a2 , a0 λ22 + a1 λ2 + a2 , . . . , a0 λ2n + a1 λn + a2 are the eigen values of
a0 A2 + a1 A + a2 I.

ww Proof. (i) let λ be an eigen value of A.

w.E
Then, there exists a column matrix X such that AX = λX.
Since X , 0 is a column matrix, we have

asy
AX − kX = λX − kX
⇒ (A − kI)X = (λ − k)X.

En
⇒ λ − k is an eigen value of A − kI.
This is true for all eigen values of A.

gin
∴ λ1 − k, λ2 − k, . . . , λn − k are the eigen values of A − kI.
(ii) We have

ee
AX = λX ⇒ A2 X = λ2 X ⇒ a0 (A2 X) = a0 (λ2 X)

=⇒ (a0 A2 )X = (a0 λ2 )X. rin


a1 (AX) = a1 (λX) ⇒ (a1 A)X = (a1 λ)X. g.n
Add a2 X both sides we get,
(a0 A2 )X + (a1 A)X = (a0 λ2 )X + (a1 λ)X.
et
(a0 A2 + a1 A + a2 I)X = (a0 λ2 + a1 λ + a2 )X

⇒ a0 λ2 + a1 λ + a2 is an eigen value of a0 A2 + a1 A + a2 I.
This is true for all eigen values of A.
∴ a0 λ21 + a1 λ1 + a2 , a0 λ22 + a1 λ2 + a2 , . . . are the eigen values of a0 A2 + a1 A + a2 I. 

Downloaded From : www.EasyEngineering.net


Downloaded From : www.EasyEngineering.net

Matrices 41

✎ ☞
Worked Examples
✍ ✌
 
3 1 4
 
Example 1.25. If A = 0 2 6, find the eigen values of A2 − 2A + I.
 
 
0 0 5
 
Solution. Since the given matrix is triangular, the eigen values are 3, 2, 5.
Therefore, eigen values of A2 − 2A + I are 32 − 2(3) + 1, 22 − 2(2) + 1 and 52 − 2(5) + 1.
i.e., 4, 1, 16.

ww Example 1.26. Form the matrix whose eigen values are α − 5, β − 5 and γ − 5 where

−1 −2 −3




w.E
α, β and γ are the eigen values of A =  4


5 −6.
7 −8 9


[Jan 2001]

Solution. Using (i) of Property 1.6, we obtain the matrix whose eigen values are

asy
α − 5, β − 5 and γ − 5 is A − 5I.
∴ The required matrix is

En

gin
−1 −2 −3



  
1 0 0 −6 −2 −3
  



ee 0 −6 .
A − 5I =  4 5 −6 − 5 0 1 0 =  4
     
     
7 −8 9 0 0 1 7 −8 4

rin
     

 
1 7 5

Example 1.27. What is the sum of the squares of the eigen values of 0 2 9.




 g.n
0 0 5
et
 
[Jan 2001]
Solution. The given matrix is triangular.
∴ Eigen values are 1, 2, 5.
Sum of squares of eigen values = 12 + 22 + 52 = 30.
 
1 1 −1
 
Example 1.28. Find the eigen values of the matrix. A = 0 −1 4 . Find also

 
0 0 2
 

Downloaded From : www.EasyEngineering.net


Downloaded From : www.EasyEngineering.net

42 Engineering Mathematics - II

the sum of the cubes of the eigen values.


Solution. Since A is triangular, the eigen values are the elements along the
leading diagonal.
∴ The eigen values are 1, −1 and 2.
Sum of the cubes of the eigen values = 13 + (−1)3 + 23

= 1 − 1 + 8 = 8.

Example 1.29. Find the sum of the fourth powers of the eigen values of the matrix

ww
 
 1 0 0 
 
A = −1 2 0 .

w.E
 
0 3 −2
 
Solution. Since the matrix A is triangular, the eigen values are the elements
along the leading diagonal.

asy
∴ The eigen values are 1, 2 and − 2.

En
Sum of the fourth powers of the eigen values = 14 + 24 + (−2)4

= 1 + 16 + 16

gin = 33.
 
4
ee
Example 1.30. If   is an eigen vector of the matrix 

eigen value.
1

1 2

5 4

rin
, find the corresponding

Solution. Let A = 


 
5 4
 
4
g.n
 and X =  . Let λ be the corresponding eigen value.
1 2

et
1

Now, (A − λI)X = 0.
  
5 − λ 4  4
i.e.,      = 0
1 2 − λ 1
4(5 − λ) + 4 = 0 ⇒ 5 − λ = −1 ⇒ λ = 6.

Also, 4 + 2 − λ = 0 ⇒ λ = 6.

Hence, the eigen value corresponding to the given eigen vector is 6.

Downloaded From : www.EasyEngineering.net


Downloaded From : www.EasyEngineering.net

Matrices 43

     
 1  1 1 1 
Example 1.31. If   and   are the eigen vectors of the matrix A =  , find

−3 1 3 −1
the corresponding eigen values.  
 1 
Solution. Let λ1 be the eigen value corresponding to the eigen vector X1 =  .
−3

∴ (A − λ1 I)X1 = 0.

  
1 − λ1 1   1 

ww
    = 0.
  
3 −1 − λ1  −3

w.E Also the second equation is


i.e., 1 − λ1 − 3 = 0 ⇒ λ1 = −2.

asy
3 − 3(−1 − λ1 ) = 0

En
3 + 3 + 3λ1 = 0

3λ1 = −6

ginλ1 = −2.

ee
 
1
Let λ2 be the eigen value corresponding to the eigen vector X2 =  .
1
rin
∴ (A − λ2 I)X2 = 0.
g.n
et
  
1 − λ2 1  1
    = 0
 
 3 −1 − λ2  1

⇒ 1 − λ2 + 1 = 0

λ2 = 2.

Also 3 − 1 − λ2 = 0 ⇒ λ2 = 2
∴ The eigen values of A are −2 and 2.

Downloaded From : www.EasyEngineering.net


Downloaded From : www.EasyEngineering.net

44 Engineering Mathematics - II

Definition [Linearly dependent and independent vectors]


(i) The vectors X1 , X2 , . . . , Xn are said to be linearly dependent if there exist real
numbers c1 , c2 , . . . , cn not all zero such that c1 X1 + c2 X2 + · · · + cn Xn = 0.
(ii) The vectors X1 , X2 , . . . , Xn are said to be linearly independent if they are not
linearly dependent.

Results
(i) If the vectors X1 , X2 , . . . , Xn are linearly independent and satisfying the

ww relation c1 X1 + c2 X2 + · · · + cn Xn = 0, then c1 = c2 = · · · = cn = 0.
(ii) If the vectors X1 , X2 , . . . , Xn are linearly dependent, then any one vector can

w.Ebe written as the linear combination of all the other vectors.

Note
asy 
1 0 0

En 
(i) The eigen values of the unit matrix 0 1 0 are 1, 1, 1, and the



gin
 
0 0 1
 
     
1 0 0

ee
     
corresponding eigen vectors are 0 , 1 , 0. These vectors are linearly
     

rin
     
0 0 1
     
independent.

λ1 a b 
 g.n 

(ii) The eigen values of the triangular matrix  0 λ2 c  are λ1 , λ2 , λ3 .


i.e., In a triangular matrix, the eigen values are the elements in the main
0 0 λ3 et 

diagonal.
(iii) The eigen values of A, A2 , . . . , Am are λ, λ2 , . . . , λm . which are all different.
But they all have the same eigen vector X.
(iv) λ and a0 λ2 +a1 λ+a2 are the eigen values of A and a0 A2 +a1 A+a2 I respectively.
But they have the same eigen vector X.

Downloaded From : www.EasyEngineering.net


Downloaded From : www.EasyEngineering.net

Matrices 45

Exercise I(B)
 
 1 2 −2 
 
1. Find the sum and product of the eigen values of the matrix  1 3 .
 
0
 
−2 −1 −3
 

 
 3 −1
2. If α and β are the eigen values of  , form the matrix whose eigen

−1 5 
values are α3 and β3 .

ww  
1 2
3. Prove that 
2 1
 and −3A−1 have the same eigen values.

w.E 
 8 −6 2 



4.

asy
Two of the eigen values of the matrix −6 7 −4 are 3 and 15. Find the
 
 
2 −4 3
 
third eigen value.
En  

gin
2 2 1
 
5. Two eigen values of the matrix A = 1 3 1 are equal to unity each. Find
 

ee
 
1 2 2
 

the eigen values of A−1 .

rin 
1 0 0

g.n

 
6. If the product of two of the eigen values of the matrix A = 0 3 −1is 2,
 

et 0 −1 3
 
find the third eigen value.
 
a 4
7. Find the constants a and b such that the matrix   has 3 and −2 as eigen
1 b
values.
 
 4 6 6 
 
8. Two eigen values of A =  1 3 2  are equal and are double the third.
 
−1 −5 −2

Downloaded From : www.EasyEngineering.net


Downloaded From : www.EasyEngineering.net

46 Engineering Mathematics - II

Find the eigen values of A2 .


   
1 1 3 −1
   
9. If the matrix A = 1 5 1 has an eigen vector  0 , find the corresponding
  
   
3 1 1 1
   
eigen value of A.
 
1 1 3
 
10. If the eigen values of the matrix A = 1 5 1 are −2, 3, 6, find the eigen
 
 

ww 3 1 1
 
values of AT .

w.E
1.4 Cayley Hamilton Theorem

asy
Statement. Every square matrix satisfies its own characteristic equation.

En
Example. If the characteristic equation of a square matrix A is λ3 − 6λ2 + 5λ + 3 = 0,
then we have, A3 − 6A2 + 5A + 3I = 0.

gin
Results. Cayley Hamilton theorem is useful for
(i) finding the inverse of a non-singular matrix,and
(ii) finding the higher powers of A.
ee
The above results are demonstrated in the following examples.
rin
g.n
✎ ☞
Worked Examples
✍ ✌

et
 
 1 0 0 0 
 

 
 2 + i −1 0 0 
Example 1.32. If the matrix A =   where i = −1, then using
 −3 2i i 0 
 
 
4 −i 1 −i
Cayley Hamilton theorem, prove that A4 = I. [Jun 2011]
Solution. A is a triangular matrix.
The eigen values of A are 1, −1, i, −i.
The characteristic equation is

Downloaded From : www.EasyEngineering.net


Downloaded From : www.EasyEngineering.net

Matrices 47

(λ − 1)(λ + 1)(λ − i)(λ + i) = 0


(λ2 − 1)(λ2 − i2 ) = 0
(λ2 − 1)(λ2 + 1) = 0
λ4 − 1 = 0
By Cayley Hamilton theorem, A4 − I = 0 ⇒ A4 = I.
 
1 0
Example 1.33. If A =  , express A3 interms of A and I, using Cayley Hamilton
4 5
theorem. [Jan 2001]

ww Solution. Since A is a 2 × 2 matrix, the characteristic equation is of the form

λ2 − s1 λ + s2 = 0.

w.E Now, s1 = sum of the main diagonal elements = 1 + 5 = 6.

asy

1 0
s2 = |A| = = 5 − 0 = 5.
4 5

En
The characteristic equation is λ2 − 6λ + 5 = 0.
By Cayley Hamilton theorem we get

gin
A2 − 6A + 5I = 0. (1)
Premultiplying by A, we get

ee
A3 − 6A2 + 5A = 0

A3 = 6A2 − 5A rin
= 6(6A − 5I) − 5A [ by (1)]
g.n
= 36A − 30I − 5A

= 31A − 30I.
 
et
1 2 
Example 1.34. Find the value of A4 if A =  , using Cayley Hamilton

2 −1
theorem. [Jan 2003]
Solution. Since A is a 2 × 2 matrix, the characteristic equation is of the form

λ2 − s1 λ + s2 = 0.

Downloaded From : www.EasyEngineering.net


Downloaded From : www.EasyEngineering.net

48 Engineering Mathematics - II

Now, s1 = sum of the main diagonal elements

= 1 + (−1) = 0.

1 2
s2 = |A| = = −1 − 4 = −5.
2 −1
The characteristic equation is
λ2 − 5 = 0.

By Cayley Hamilton theorem, A2 − 5I = 0.

ww i.e., A2 = 5I

Now, A4 = A2 .A2

w.E 
= 5I.5I = 25I = 25 
0 1
 
1 0 25 0 
 = 
 

 .
0 25

asy  
1 4

En
Example 1.35. Using Cayley Hamilton theorem, find the inverse of  .

2 3
[Jan 2003]
 
1 4
Solution. Let A =  .
gin
ee

2 3
Since A is a 2 × 2 matrix, the characteristic equation is of the form
λ2 − s1 λ + s2 = 0.
rin
Now, s1 = sum of the main diagonal elements = 1 + 3 = 4.
s2 = |A| = 3 − 8 = −5. g.n
The characteristic equation is λ2 − 4λ − 5 = 0.
By Cayley Hamilton theorem, we have
A2 − 4A − 5I = 0.
et
Multiplying by A−1 we get,

A−1 A2 − 4A−1 A − 5A−1 I = 0

A − 4I − 5A−1 = 0

5A−1 = A − 4I

Downloaded From : www.EasyEngineering.net


Downloaded From : www.EasyEngineering.net

Matrices 49

1
A−1 = [A − 4I]
5
     
1 1 4 1 0 1 −3 4 
 . =   .
=   − 4     
5 2 3 0 1   5  2 −1

 
1 4
Example 1.36. Verify Cayley Hamilton theorem for the matrix A =   and

2 3
find its inverse. Also express A5 − 4A4 − 7A3 + 11A2 − A − 10I as a linear polynomial
in A. [Jun 2009]

ww Step 1. To find the characteristic equation


 
1 4

w.E
Let A =  .

2 3
Since A is a 2 × 2 matrix, the characteristic equation is of the form

asy λ2 − s1 λ + s2 = 0.
Now, s1 = sum of the main diagonal elements = 1 + 3 = 4.

En
s2 = | A | = 3 − 8 = −5.
The characteristic equation is
gin
ee λ2 − 4λ − 5 = 0.

Step 2. Verification of Cayley Hamilton theorem


rin
By cayley Hamilton theorem, we have

A2 − 4A − 5I = 0.
g.n
Now, A2
= A.A = 



 
1 4 1
4 9 16
 = 
 
2 3 2

3 8 17

.
et
     
9 16 4
16 5 0
A2 − 4A − 5I =   − 
   −  
8 17 8 12 0 5
 
0 0
=   = 0.
0 0
∴ Cayley Hamilton theorem is verified.

Downloaded From : www.EasyEngineering.net


Downloaded From : www.EasyEngineering.net

50 Engineering Mathematics - II

Step 3. To find A−1


we have

A2 − 4A − 5I = 0.

Multiplying by A−1 we get,

A−1 A2 − 4A−1 A − 5A−1 I = 0

A − 4I − 5A−1 = 0

ww 5A−1 = A − 4I
     

w.E
1 1 1 4 1 0 1 −3 4 
A−1 = [A − 4I] =   − 4   =   .
5 5 2 3 0 1
 5 
2 −1

asy
Step 4. To find A5 − 4A4 − 7A3 + 11A2 − A − 10I
Consider the polynomial λ5 − 4λ4 − 7λ3 + 11λ2 − λ − 10.
Divide by λ2 − 4λ − 5 we get,
En
gin
λ5 − 4λ4 − 7λ3 + 11λ2 − λ − 10 = (λ2 − 4λ − 5)(λ3 − 2λ + 3) + λ + 5.
Replacing λ by A we get,

ee
A5 − 4A4 − 7A3 + 11A2 − A − 10I = (A2 − 4A − 5I)(A3 − 2A + 3I) + A + 5I
rin
[ Using Cayley Hamilton theorem]

g.n
= 0 + A + 5I

= A + 5I.


 2 −1 1 

Example 1.37. Verify Cayley Hamilton theorem for the matrix A = −1 2 −1.




et
 
1 −1 2
 

Find A−1 and A4 . [Jan 2009]


Solution.
Step 1. To find the characteristic equation
Since A is a 3 × 3 matrix, the characteristic equation is of the form

Downloaded From : www.EasyEngineering.net


Downloaded From : www.EasyEngineering.net

Matrices 51

λ3 − s1 λ2 + s2 λ − s3 = 0. (1)

Now, s1 = sum of the main diagonal elements

= 2 + 2 + 2 = 6.

s2 = sum of the minors of the elements of the main diagonal



2 −1 2 1 2 −1
= + +
−1 2 1 2 −1 2

=4−1+4−1+4−1

ww =3+3+3

w.E
= 9.

2 −1 1

asy

s3 = |A| −1 2 −1 = 2(3) + 1(−1) + 1(−1) = 6 − 1 − 1 = 4.

1 −1 2

En
The characteristic equation is λ3 − 6λ2 + 9λ − 4 = 0.


 2 −1 1  gin
Step 2. Verification of Cayley Hamilton theorem.


A = −1 2 −1


 ee 1 −1 2




rin
g.n
  
 2 −1 1   2 −1 1 
   
2
A = A · A = −1 2 −1 −1 2 −1
   




1 −1 2
 4 + 1 + 1 −2 − 2 − 1 2 + 1 + 2   6 −5 5 

 

1 −1 2


 
 
et


= −2 − 2 − 1 1 + 4 + 1 −1 − 2 − 2 = −5 6 −5
  
   
2 + 1 + 2 −1 − 2 − 2 1 + 1 + 4 5 −5 6
   
  
 6 −5 5   2 −1 1 
   
A3 = A2 × A = −5 6 −5 −1 2 −1

   
5 −5 6 1 −1 2

Downloaded From : www.EasyEngineering.net


Downloaded From : www.EasyEngineering.net

52 Engineering Mathematics - II

   
 12 + 5 + 5 −6 − 10 − 5 6 + 5 + 10   22 −21 21 
   
= −10 − 6 − 5 5 + 12 + 5 −5 − 6 − 10 = −21
  
22 −21
   
10 + 5 + 6 −5 − 10 − 6 5 + 5 + 12 21 −21 22
   
       
 22 −21 21   6 −5 5   2 −1 1  1 0 0
       
3 2
A − 6A + 9A − 4I = −21 22 −21 − 6 −5 6 −5 + 9 −1 2 −1 − 4 0 1 0
     
  
       
21 −21 22 5 −5 6 1 −1 2 0 0 1
       
 
0 0 0
 

ww = 0 0 0 = 0.
 
 
0 0 0
 

w.E
∴ Cayley Hamilton theorem is verified.
Step 3. To find A−1

asy
By Cayley Hamilton theorem, we have A3 − 6A2 + 9A − 4I = 0.
Multiplying by A−1 we get,

En
ginA2 − 6A + 9I − 4A−1 = 0.

ee
⇒ 4A−1 = A2 − 6A + 9I.

1
⇒ A−1 = [A2 − 6A + 9I].
4
rin
   
 2 −1 1   2 −1 1   6 −5 5 
    

 g.n
A2 = −1 2 −1 −1 2 −1 = −5 6 −5
et
 
     
1 −1 2 1 −1 2 5 −5 6
    

       
 6 −5 5   12 −6 6  9 0 0  3 1 −1
1  1 
       
∴ A−1 = −5 6 −5 − −6 12 −6 + 0 9 0 .A−1 =  1 3 1  .
     
4       4  
5 −5 6 6 −6 12 0 0 9 −1 1 3
       

Step 4. To find A4
Consider λ4 . Dividing λ4 by λ3 − 6λ2 + 9λ − 4 we get,

Downloaded From : www.EasyEngineering.net


Downloaded From : www.EasyEngineering.net

Matrices 53

λ4 = (λ + 6)(λ3 − 6λ2 + 9λ − 4) + 27λ2 − 50λ + 24.


Replacing λ by A we get,

A4 = (A + 6I)(A3 − 6A2 + 9A − 4I) + 27A2 − 50A + 24I.

= 27A2 − 50A + 24I [ By Cayley Hamilton theorem].


 
 86 −85 85 
 
= −85 86 −85 .
 
85 −85 86

ww Example 1.38. If A = −1 3



 1

2 −2



0 , verify Cayley Hamilton theorem and hence

w.E
find A−1 .


0 −2 1


[Jan 2005]
Solution.
asy
Step 1. To find the characteristic equation

En
Since A is 3 × 3 matrix, the characteristic equation is of the form

gin
λ3 − s1 λ2 + s2 λ − s3 = 0.

Now, s1 = sum of the main diagonal elements

=1+3+1=5 ee rin
s2 = sum of the minors of the elements of the main diagonal

=

3 0 1 −2 1 2
+ +

g.n
et
−2 1 0 1 −1 3

=3−0+1−0+3+2

=3+1+5=9

1 2 −2

s3 = |A| = −1 3 0

0 −2 1

= 1(3 − 0) − 2(−1 − 0) − 2(2 − 0) = 3 + 2 − 4 = 1.

Downloaded From : www.EasyEngineering.net


Downloaded From : www.EasyEngineering.net

54 Engineering Mathematics - II

The characteristic equation is λ3 − 5λ2 + 9λ − 1 = 0.


By Cayley Hamilton theorem, we have A3 − 5A2 + 9A − I = 0.
Step2. Verification of Cayley Hamilton theorem

    
 1 2 −2  1 2 −2 −1 12 −4
     
A2 = A.A = −1 3 0  = −4 7

0  −1 3 2 
     
0 −2 1 0 −2 1 2 −8 1
    

    
−1 12 −4  1 2 −2 −13 42 −2

ww A3 = A2 .A = −4 7


 

2  −1 3
 
  
0  = −11 9
 

10 


w.E
2 −8 1 0 −2 1 10 −22 −3
    

       
−13 42 −2 −1 12 −4  1 2 −2  1 0 0
      

asy

Now A3 − 5A2 + 9A − I = −11 9 10  − 5 −4 2  + 9 −1 3 0  − 0 1 0
     
7
       
10 −22 −3 2 −8 1 0 −2 1 0 0 1
     

En
 
0 0 0
 

gin
= 0 0 0 = 0.
 
 
0 0 0
 
Hence, Cayley Hamilton theorem is verified.
Step 3. To find A−1 ee
By Cayley Hamilton theorem, we have A3 − 5A2 + 9A − I = 0.
rin
Multiplying by A−1 we get
A2 − 5A + 9I − A−1 = 0 g.n

3 2 6

∴ A−1 = A2 − 5A + 9I = 1 1 2 .






et
2 2 5
 

Example 1.39. Find the matrix A8 − 5A7 + 7A6 − 3A5 + 8A4 − 5A3 + 8A2 − 2A + I if
 
2 1 1
 
A = 0 1 0, using Cayley Hamilton theorem. [Jun 2009]

 
1 1 2
 

Downloaded From : www.EasyEngineering.net


Downloaded From : www.EasyEngineering.net

Matrices 55

Solution.
Step 1. To find the characteristic equation
Since A is a 3 × 3 matrix, the characteristic equation takes the form
λ3 − s1 λ2 + s2 λ − s3 = 0

Now, s1 = sum of the main diagonal elements

=2+1+2=5

s2 = sum of the minors of the main diagonal elements

ww

1 0 2 1 2 1
= + + = 2 + 3 + 2 = 7.
1 2 1 2 0 1

w.E
2 1 1



s3 = |A| = 0 1 0 = 2(2 − 0) − 1(0 − 0) + 1(0 − 1) = 4 − 1 = 3.

asy
1 1 2

En
The characteristic equation is λ3 − 5λ2 + 7λ − 3 = 0.
By Cayley Hamilton theorem we get A3 − 5A2 + 7A − 3I = 0.

gin
Step 2. To find the value of given matrix expression

ee
Consider the polynomial λ8 − 5λ7 + 7λ6 − 3λ5 + 8λ4 − 5λ3 + 8λ2 − 2λ + 1.
Divide this polynomial by λ3 − 5λ2 + 7λ − 3 we get

rin
λ8 − 5λ7 + 7λ6 − 3λ5 + 8λ4 − 5λ3 + 8λ2 − 2λ + 1 = (λ5 + 8λ + 35)(λ3 − 5λ2 + 7λ − 3)

Replacing λ by A we get
+127λ2
g.n
− 223λ + 106.

A8 − 5A7 + 7A6 −3A5 + 8A4 − 5A3 + 8A2 − 2A + I et


= (A5 + 8A + 35I)(A3 − 5A2 + 7A − 3I) + 127A2 − 223A + 106I

= 127A2 − 223A + 106I [ By Cayley Hamilton theorem].


 
295 285 285
 
=  0 0  .
 
10
 
285 285 295
 

Downloaded From : www.EasyEngineering.net


Downloaded From : www.EasyEngineering.net

56 Engineering Mathematics - II

 
1 0 0
 
Example 1.40. If A = 1 0 1 then show that An = An−2 + A2 − I for n ≥ 3. Hence
 
 
0 1 0
 

find A50 . [Jan 2006]


Solution. Since A is a 3 × 3 matrix, the characteristic equation is of the form

λ3 − s1 λ2 + s2 λ − s3 = 0.
Now, s1 = sum of the main diagonal elements

ww =1+0+0=1

s2 = sum of the minors of the main diagonal elements

w.E

0 1 1 0 1 0
= + +
1 0 0 0 1 0

asy
= 0 − 1 − 0 + 0 = −1.

En
1 0 0

s3 = |A| = 1 0 1

gin

0 1 0

= 1(0 − 1) = −1.

ee
∴ The characteristic equation is λ3 − λ2 − λ − 1 = 0.
By Cayley Hamilton theorem we have,
rin
3 2
A −A −A+I =0

A3 − A2 = A − I
g.n
A4 − A3 = A2 − A

A5 − A4 = A3 − A2
et
············

An − An−1 = An−2 − An−3

Adding all these equations we get, An − A2 = An−2 − I.

∴ An = An−2 + A2 − I

Downloaded From : www.EasyEngineering.net


Downloaded From : www.EasyEngineering.net

Matrices 57

An−2 = An−4 + A2 − I.

∴ An = An−4 + 2(A2 − I).

An−4 = An−8 + 2(A2 − I).

∴ An = An−8 + 4(A2 − I).

An−8 = An−16 + 4(A2 − I).

An = An−16 + 8(A2 − I).


n−2 2

ww If n is even, An = An−(n−2) +
n−2 2
= A2 + (A − I)
2
(A − I)

w.E
2
n−2 2 n−2
= A2 + A − I
2 2
n n−2

When n = 50, we have asy An = A2 −


2 2
I.

A50 = 25A2 − 24I.


En
Now, A2 = A · A

1 0 0 1 0 0 1 0 0

gin   



0 1 0 0 1 0
 
= 1 0 1 1 0 1 = 1 1 0
 
 
 ee  
 
 
 
1 0 1




rin
g.n
   
1 0 0 1 0 0
   
50
A = 25 1 1 0 − 24 0 1 0
   




25 0 0  24 0 0 




1 0 1


 
 


0 0 1




et
= 25 25 0  −  0 24 0 
  
   
25 0 25 0 0 24
   
 
 1 0 0
 
= 25 1 0 .
 
25 0 1

Downloaded From : www.EasyEngineering.net


Downloaded From : www.EasyEngineering.net

58 Engineering Mathematics - II

 
1 2
Example 1.41. If n is a positive integer, find An for the matrix A =   using

4 3
Cayley Hamilton theorem. [Jan 2005]
Solution. Since A is a 2 × 2 matrix, the characteristic equation is of the form
λ2 − s1 λ + s2 = 0.
Now, s1 = sum of the main diagonal elements

= 1 + 3 = 4.

1 2

ww s2 = |A| =
4 3

w.E = 3 − 8 = −5.
The characteristic equation is
λ2 − 4λ − 5 = 0.

asy
(λ − 5)(λ + 1) = 0.
The eigen values are −1, 5.

En
By Cayley Hamilton theorem we get, A2 − 4A − 5I = 0.

gin
Dividing λn by λ2 − 4λ − 5, let Q(λ) be the quotient and R(λ) be the remainder.
Since we are dividing by λ2 − 4λ − 5, R(λ) is of degree atmost 1.
∴ Let R(λ) = aλ + b.
Hence, we can write λn as
ee rin
n 2
λ = (λ − 4λ − 5)Q(λ) + aλ + b.
g.n (A)

When λ = −1, we get

When λ = 5, we get
−a + b = (−1)n et (1)

5a + b = 5n (2)
1
(2) − (1) ⇒ 6a = 5n − (−1)n . ⇒ a = [5n − (−1)n ].
6
Substituting in (1) we get
1 1
b = a + (−1)n = [5n − (−1)n ] + (−1)n = [5n + 5(−1)n ].
6 6

Downloaded From : www.EasyEngineering.net


Downloaded From : www.EasyEngineering.net

Matrices 59

Replacing λ by A in (A) we get

An = (A2 − 4A − 5I)Q(A) + aA + bI

= aA + bI [ by C.H Theorem]
1 1
= [5n − (−1)n ]A + [5n + 5(−1)n ]I.
6 6
 
1 2 
Example 1.42. If A =   find An in terms of A and I. [Jun 2003]

2 −1

ww Solution. Since A is a 2 × 2 matrix, the characteristic equation is of the form


λ2 − s1 λ + s2 = 0.

w.E Now, s1 = sum of the main diagonal elements

= 1 − 1 = 0.

asy

1 2
s2 = |A| =
2 −1

= −1 − 4 = −5.
En
gin
The characteristic equation is λ2 − 5 = 0

ee i.e., λ2 − 5 = 0

=⇒ λ = ± 5.
rin
By Cayley Hamilton theorem we get A2 − 5I = 0.
Dividing λn by λ2 − 5, let Q(λ) be the quotient and R(λ) be the remainder. g.n
Since we are dividing λn by λ2 − 5, R(λ) is of degree atmost 1.
et
∴ R(λ) = aλ + b.

∴ λn = (λ2 − 5)Q(λ) + aλ + b. (1)


√ √ √
When λ = 5, we get( 5)n = a 5 + b. (2)
√ √ √
When λ = − 5, we get (− 5)n = − 5a + b (3)

Downloaded From : www.EasyEngineering.net


Downloaded From : www.EasyEngineering.net

60 Engineering Mathematics - II

√ √ √
(2) − (3) ⇒ 2 5a = ( 5)n − (− 5)n

= ( 5)n [1 − (−1)n ]
√ [1 − (−1)n ]
a = ( 5)n−1 .
2
substituting in (2) we get
√ √
b = ( 5)n − a( 5)
√ √ [1 − (−1)n ]
= ( 5)n − ( 5)n
2
√ n [2 − 1 − +(−1)]n
= ( 5)
2
√ n [1 + (−1)n ]

ww n 2
b = ( 5)

∴ λ = (λ − 5)φ(λ) +
√ n−1 2
5
.
n
[1 − (−1) ]λ +
√ √
( 5)n + (− 5)n
.

w.E
2 2
Replacing λ by A we get

( 5)n−1 √ [1 + (−1)n ]

asy
n
A = [1 − (−1)n ]A + ( 5)n .
2 2

Exercise I(C)
En
gin 
 13 −3 5 

1. Verify Cayley Hamilton theorem find the inverse of A =  0


0 .

ee
 
4
 
−15 9 −7
 


 2 2 0 

rin





g.n
2. Verify Cayley Hamilton theorem for the matrix −2 1 1 . Hence find its
−7 2 −3

et
 
inverse.
 
2 1 
3. Using Cayley Hamilton theorem find the inverse of A =  .

1 −5

4. Using Cayley Hamilton theorem, find the inverse of the following matrices.
 
  2 0 −1
5 4  
(i)   (ii)  
  0 2 2 
1 3  
1 −1 2

Downloaded From : www.EasyEngineering.net


Downloaded From : www.EasyEngineering.net

Matrices 61

 
 0 c −b 
 
5. Show that the matrix −c 0 a  satisfies Cayley Hamilton theorem.
 
 
b −a 0
 

 
−1 0 3 
 
6. Verify Cayley Hamilton theorem for the matrix A =  8 1 −7.
 
 
−3 0 8
 

 
 2 −1 2 

ww
 
7. Using Cayley Hamilton theorem find A4 for the matrix A = −1 2 −1.
 
 
1 −1 2
 

w.E 8. Find the characteristic equation of the matrix A and hence Compute

1 0 2

asy
 
2A8 − 3A5 + A4 − 4I where A is 0 −1 1.
 
 

En
0 1 0
 

 

gin
7 3
9. If A =   , find An interms of A and I using Cayley Hamilton theorem.

2 6
Hence find the value of A3 .

1 2
10. If A = 
0 2

ee
, find An interms of A and I.
rin
 
1 3
11. If A = 
 
1 3
, prove that An = 7n−1  . g.n

2 6

3 −4
12. If A = 






2 6


1 + 2n −4n 
, prove that An =  .

et
1 −1  n 1 − 2n
 
 0 0 2 
 
13. If A =  2 1 0, using Cayley Hamilton theorem obtain the value of
 
 
−1 −1 3
 

A6 − 25A2 + 122A.

Downloaded From : www.EasyEngineering.net


Downloaded From : www.EasyEngineering.net

62 Engineering Mathematics - II

1.5 Diagonalization of Matrices - Similarity Transformation

Similar matrices. Let A and B be square matrices of order n. A is similar to B if


there exists a nonsingular matrix P such that A = P−1 BP.
The transformation which transforms B into A is called a similarity
transformation. The matrix P is called a similarity matrix.
Results.
1. If A and B are similar then |A| = |B|.

ww 2. If A and B are similar, then they have the same eigen values.

Diagonalisation of a square matrix

w.E
A square matrix A is said to be diagonalizable, if there exists a nonsingular matrix
P such that P−1 AP = D where D is a diagonal matrix. The matrix P is called the

asy
modal matrix of A. D is also called as the spectral matrix of A.
Working rule to diagonalize a matrix

En
Step 1. Find the eigen values λ1 , λ2 , . . . , λn .

gin
Step 2. Find the linearly independent eigen vectors X1 , X2 , . . . , Xn , corresponding to
λ1 , λ2 , . . . , λn .
ee
Step 3. Form the modal matrix P = [X1 , X2 , . . . , Xn ].
rin
Step 4. Find P−1 .
g.n
et
 
λ1 0 . . . 0 
 
 
−1  0 λ2 . . . 0 
Step 5. Find P AP = D =  . .. .. .
 .. . . 
 
0 0 . . . λn 


Note. P is possible only when A has n linearly independent eigen vectors.

Downloaded From : www.EasyEngineering.net


Downloaded From : www.EasyEngineering.net

Matrices 63

Computation of powers of a square matrix using similarity


transformation
we have D = P−1 AP

PDP−1 = P(P−1 AP)P−1

= (PP−1 )A(PP−1 )

= IAI = A

∴ A = PDP−1

ww A2 = A · A

= (PDP−1 )(PDP−1 )

w.E = PD(P−1 P)DP−1

= PDIDP−1

In the same way we get


asy
A2 = PD2 P−1
A3 = PD3 P−1 . 

En λn 0 . . . 0 
 1



gin n
 
n n −1
 0 λ2 . . . 0 
In general A = PD P , where D =  . .. .. 

 .. . .

ee
 

0 0 . . . λnn 


Properties of eigen values of similar matrices


rin
Property 1.7. If A and B are similar matrices, they have the same characteristic
equation. g.n
Proof. Since A and B are similar, there exists a matrix P such that
B = P−1 AP
et
∴ B − λI = P−1 AP − λI

= P−1 AP − P−1 λIP

= P−1 (AP − λIP)

= P−1 (A − λI)P

Downloaded From : www.EasyEngineering.net


Downloaded From : www.EasyEngineering.net

64 Engineering Mathematics - II

Hence, | B − λI | = |P−1 (A − λI)P |

= | P−1 | | A − λI | | P |

= | P−1 | | P | | A − λI |

= | P−1 P | | A − λI |

= | I | | A − λI |

= | A − λI |.
The characteristic equations of A and B are respectively |A − λI| = 0 and

ww |B − λI| = 0. Hence, they are equal.

Corollary. Two similar matrices have the same eigen values.




w.E
Property 1.8. If A and B are n × n matrices and B is a non singular matrix, then

asy
A and B−1 AB have the same eigen values.

Proof. The characteristic polynomial of B−1 AB is

En
| B−1 AB − λI | = | B−1 AB − B−1 (λI)B |

gin
= | B−1 (A − λI)B |

= | B−1 | | A − λI | | B |

ee
= | B−1 | | B | | A − λI |

= | B−1 B | | A − λI | rin
= | I | | A − λI |
g.n
⇒ B−1 AB
= | A − λI |.

= Characteristic polynomial of A.
and A have the same characteristic equation.
et
⇒ They have the same eigen values. 

Result
If we are asked to find f (A), then f (A) can be evaluated by f (A) = P f (D)P−1

Downloaded From : www.EasyEngineering.net


Downloaded From : www.EasyEngineering.net

Matrices 65

✎ ☞
Worked Examples
✍ ✌
 
−19 7 
Example 1.43. Reduce the matrix A =   to the diagonal form.

−42 16
 
−19 7 
Solution. Let the matrix be A =  .
−42 16
Step 1. To find the eigen values

s1 = tr(A) = −19 + 16 = −3.

ww
s2 = |A| =
−19 7

w.E
−42 16

= −304 + 294 = −10.

asy
The characteristic equation is

En λ2 − s1 λ + s2 = 0

gin i.e., λ2 + 3λ − 10 = 0

ee
(λ + 5)(λ − 2) = 0

The eigen values are λ1 = 2, λ2 = −5.


λ = 2, −5.

rin
Step 2. To find the eigen vectors
g.n
et
 
 x1 
Let X =   be the eigen vector corresponding to an eigen value λ.
x2

∴ (A − λI)X = 0
  
−19 − λ 7   x1 
i.e.     = 0
 
−42 16 − λ  x2 

(−19 − λ)x1 + 7x2 = 0  

(A)


−42x1 + (16 − λ)x2 = 0. 

Downloaded From : www.EasyEngineering.net


Downloaded From : www.EasyEngineering.net

66 Engineering Mathematics - II

case(i) When λ = 2, (A) become

−21x1 + 7x2 = 0

−42x1 + 14x2 = 0.

Both the equations are reduced to one single equation

3x1 = x2
x1 x2
= .

ww 1 3
 
1
∴ The eigen vector corresponding to λ = 2 is X1 =  .

w.E
case(ii) When λ = −5, (A) reduces to
3

asy −14x1 + 7x2 = 0

En −42x1 + 21x2 = 0.

gin
Both the equations are reduced to one single equation

ee −2x1 = −x2
x1
1
=
x2
2
rin
g.n
 
1
∴ The eigen vector corresponding to λ = −5 is X2 =  .
2
Step 3. To form the modal matrix P
The modal matrix P is obtained with X1 , X2 as columns.


 1 1

et
 .
 
∴ P = X1 X2 = 
3 2

Step 4. To find P−1


   
a b 1  d −b
−1
We know that if A =   , then A =
 
 .
c d  ad − bc  −c a 

Downloaded From : www.EasyEngineering.net


Downloaded From : www.EasyEngineering.net

Matrices 67

   
1  2 −1 −2 1 
Hence P−1 =   =  .
−1 −3 1   3 −1
Step 5. To find P−1 AP
   
−2 1  −19 7  1 1
P−1 AP =   
 
 
 


3 −1 −42 16 3 2
     
−2 1  −19 + 21 −19 + 14 −2 1  2 −5 
=   

 = 
 
 



3 −1 −42 + 48 −42 + 32  3 −1 6 −10

ww
   
−4 + 6 10 − 10  2 0
=   = 
 


6 − 6 −15 + 10 0 5

w.E
which is the required diagonal form of A.

asy
 
 1 2 −2
 
Example 1.44. Reduce the matrix A =  1 1  to the diagonal form. Hence

2

En
 
−1 −1 0
 

find A4 .

Solution. gin
Step 1. To find the eigen values
ee
s1 = tr(A) = sum of the elements along the main diagonal = 1 + 2 + 0 = 3.
rin
s2 = sum of the minors of the elements of the main diagonal
g.n
et
2 1 1 −2 1 2
= + + = 0 + 1 + 0 − 2 + 2 − 2 = −1.
−1 0 −1 0 1 2

1 2 −2

s3 = |A| = 1 2 1 = 1(0 + 1) − 2(0 + 1) − 2(−1 + 2) = 1 − 2 − 2 = −3.

−1 −1 0

The characteristic equation is

λ3 − s1 λ2 + s2 λ + s3 = 0

Downloaded From : www.EasyEngineering.net


Downloaded From : www.EasyEngineering.net

68 Engineering Mathematics - II

i.e., λ3 − 3λ2 − λ + 3 = 0

λ = 1 is a root.

By synthetic division we have

1 1 −3 −1 3
0 1 −2 −3
1 −2 −3 0

ww ∴ The characteristic equation becomes

(λ − 1)(λ2 − 2λ − 3) = 0

w.E (λ − 1)(λ − 3)(λ + 1) = 0

asy λ = −1, 1, 3.

En
The eigen values are λ1 = −1, λ2 = 1, λ3 = 3.
Step 2. To find the eigen vectors
 
 x1 
 
gin
Let X =  x2  be the eigen vector corresponding to an eigen value λ.
 
 
x3
 
ee ∴ (A − λI)X = 0
rin

1 − λ
 2
 
−2  x1 
g.n
et
  
i.e.  1 2 − λ 1   x2  = 0.

   
−1 −1 −λ x3
  

(1 − λ)x1 + 2x2 − 2x3 = 0 






x1 + (2 − λ)x2 + x3 (A)

=0  



−x1 − x2 − λx3 =0 

case(i) When λ = −1, (A) become

2x1 + 2x2 − 2x3 = 0. (1)

Downloaded From : www.EasyEngineering.net


Downloaded From : www.EasyEngineering.net

Matrices 69

x1 + 3x2 + x3 = 0. (2)

−x1 − x2 + x3 = 0. (3)

The above three equations are reduced to the following two equations.

x1 + x2 − x3 = 0

x1 + 3x2 + x3 = 0.

By the rule of cross multiplication we have,

ww x1 x2 x3

w.E
1 −1 1 1
3 1 1 3
x1 x2 x3

asy
= =
1 + 3 −1 − 1 3 − 1
x1 x2 x3
= =

En 4
x1
=
−2
x2 x3
= .
2

gin
2 −1 1
 
 2 

ee
 
∴ X1 = −1 .
 
 
1
rin
 

case(ii) When λ = 1, (A) become

2x2 − 2x3 = 0 g.n (4)

x1 + x2 + x3 = 0 et (5)

−x1 − x2 − x3 = 0. (6)

The above three equations are reduced to

x2 = x3

and x1 + x2 + x3 = 0.

Downloaded From : www.EasyEngineering.net


Downloaded From : www.EasyEngineering.net

70 Engineering Mathematics - II

let x3 = 1 ⇒ x2 = 1.

Hence x1 = −2
 
−2
 
∴ X2 =  1  .
 
 
1
 

case(iii) When λ = 3, (A) become

−2x1 + 2x2 − 2x3 = 0 (7)

ww x1 − x2 + x3 = 0 (8)

w.E −x1 − x2 − 3x3 = 0. (9)

asy
The above three equations are reduced to the following two equations.

x1 − x2 + x3 = 0

En x1 + x2 + 3x3 = 0.

gin
By the rule of cross multiplication we have

−1
1
x1
ee 1
3
x2
1
1
x3
−1
1 rin
g.n
x1
=
x2
−3 − 1 1 − 3 1 + 1
x1
=
x2
=
=
x3
x3
et
−4 −2 2
x1 x2 x3
= = .
2 1 −1
 
 2 
 
∴ X3 =  1  .
 
−1

Downloaded From : www.EasyEngineering.net


Downloaded From : www.EasyEngineering.net

Matrices 71

Step 3. To form the modal matrix P


The modal matrix P is formed with the eigen vectors X1 , X2 , X3 as columns.
 
 2 −2 2 
   
i.e, P = X1 X2 X3 = −1 1 1  .
 
 
1 1 −1

Step 4. To find P−1



2 −2 2

ww

|P| = −1 1 1

1 1 −1

w.E = 2(−1 − 1) + 2(1 − 1) + 2(−1 − 1)

= 2(−2) + 2(0) + 2(−2) = −4 + 0 − 4 = −8.

cofactor of 2 = +

asy
1 1

= −1 − 1 = −2.

En
1 −1

−1 1

cofactor of −2 = −
gin = −(1 − 1) = 0.
1 −1

ee

−1 1
cofactor of 2 = + = −1 − 1 = −2.

rin
1 1

−2 2
cofactor of −1 = −

g.n
= −(2 − 2) = 0.
1 −1

et
2 2
cofactor of 1 = + = −2 − 2 = −4.
1 −1

2 −2
cofactor of 1 = − = −(2 + 2) = −4.
1 1

−2 2
cofactor of 1 = + = −2 − 2 = −4.
1 1

2 2
cofactor of 1 = − = −(2 + 2) = −4.
−1 1

Downloaded From : www.EasyEngineering.net


Downloaded From : www.EasyEngineering.net

72 Engineering Mathematics - II


2 −2
cofactor of −1 = + = 2 − 2 = 0.
−1 1
 
−2 0 −2
 
∴ Pc =  0 −4 −4 .
 
 
−4 −4 0
 

 
−2 0 −4
1 T −1 
 
P−1 = P =  0 −4 −4 .

|P| c

ww

8  
−2 −4 0

w.E
Step 5. To find P−1 AP

−2 0 −4  1

2 −2

  2 −2 2 

asy
−1 
     
P−1 AP =  0 −4 −4  1  
2 1  −1 1 1 
8      
−2 −4 0 −1 −1 0 1 1 −1
   

En
−2 + 0 + 4 −4 + 0 + 4 4 + 0 + 0  2 −2 2 
 

gin
−1 
   
=  0 − 4 + 4  
0 − 8 + 4 0 − 4 + 0 −1 1 1 
8    
−2 − 4 + 0 −4 − 8 + 0 4 − 4 + 0 1 1 −1
  

=
−1 
8 

 ee
 2 0
 0 −4 −4 −1 1
4

  2 −2 2 
 
 
1 




rin
−6 −12 0 1 1 −1
g.n
  
 
 4 + 0 + 4 −4 + 0 + 4 4 + 0 − 4

et

−1 
 
=  0 + 4 − 4 
0−4−4 0 − 4 + 4 
8  
−12 + 12 + 0 12 − 12 + 0 −12 − 12 + 0
 
   
8 0 0  −1 0 0
−1 
   
= 0 −8 0  =  0 1 0
8    
0 0 −24 0 0 3
   

which is the required diagonal form.

Downloaded From : www.EasyEngineering.net


Downloaded From : www.EasyEngineering.net

Matrices 73

Step 6. To find A4
By similarity transformation, D = P−1 AP.

 
−1 0 0
 
∴ D =  0 1 0
 
 
0 0 3
 

Hence  
1 0 0 
 

ww
4
D = 0 1 0 
 
 
0 0 81
 

w.E Now A4 = PD4 P−1


   

asy
 2 −2 2  1 0 0  −2 0 −4
−1 
     
= −1 1     
1  0 1 0   0 −4 −4
8      

En 1 1 −1 0 0 81 −2 −4 0
   
  
 2 + 0 + 0 0 − 2 + 0 0 + 0 + 162 −2 0 −4
=
−1 
8 

gin
−1 + 0 + 0 0 + 1 + 0 0 + 0 + 81   0 −4 −4

 
 


ee
  
1 + 0 + 0 0 + 1 + 0 0 + 0 − 81 −2 −4 0

rin
  
 2 −2 162  −2 0 −4
−1 
   
= −1 1   
81   0 −4 −4
 
8  

1 1 −81 −2 −4 0
 
  

g.n
et
 
−4 + 0 − 324 0 + 8 − 648 −8 + 8 + 0
−1 
 
=  2 + 0 − 162 0 − 4 − 324 4 − 4 + 0 
8  
−2 + 0 + 162 0 − 4 + 324 −4 − 4 + 0

   
−328 −640 0   41 80 0 
−1 
   
= −160 −328 0  =  20 
41 0
8    
160 320 −8 −20 −40 1
   

Result. A simple way to find the inverse of a 3 × 3 matrix.

Downloaded From : www.EasyEngineering.net


Downloaded From : www.EasyEngineering.net

74 Engineering Mathematics - II

 
a1 a2 a3 
 
Let A = b1 b2 b3 
 
 
c1 c2 c3
 
Step1. Rearrange the elements in the following way

b2 b3 b1
c2 c3 c1
a2 a3 a1

ww Step2. Attach the first column as the 4th column

w.E b2 b3 b1 b2

asy c2 c3 c1 c2

En
a2 a3 a1 a2

gin
Step3. Attach the first row as the 4th row

ee
b2 b3 b1 b2
c2 c3 c1 c2
rin
a2 a3 a1 a2
b2 b3 b1 b2 g.n
Step4. The matrix Ac formed by the Cofactors of the elements of A is given by et
 
 A1 A2 A3 
 
Ac =  B1 B2 B3 .
 
 
C1 C2 C3
 

Ais , Bis and Cis are calculated as follows, similar to the rule of Cross multiplication.

Downloaded From : www.EasyEngineering.net


Downloaded From : www.EasyEngineering.net

Matrices 75

b2 b3 b1 b2

A1 A2 A3

c2 c3 c1 c2

B1 B2 B3

a2 a3 a1 a2

C1 C2 C3

ww b2 b3 b1 b2

w.E
 
 b2 c3 − b3 c2 b3 c1 − b1 c3 b1 c2 − b2 c1 
 
Ac =  c2 a3 − c3 a2 c3 a1 − c1 a3 c1 a2 − c2 a1 

 

asy a2 b3 − a3 b2 a3 b1 − a1 b3 a1 b2 − a2 b1
 

1 T
Now A−1 =

En
A .
|A| c
Example. Consider the previous example.
To find P−1 we have 
 2 −2 2  gin
ee
 
P = −1 1
 
1 

rin
 
1 1 −1
 

1 1 −1 1

g.n
1

−2
−1

2
1

2
1

−2
et
1 1 −1 1
 
−2 0 −2
 
Pc =  0 −4 −4

 
−4 −4 0
 

Downloaded From : www.EasyEngineering.net


Downloaded From : www.EasyEngineering.net

76 Engineering Mathematics - II

1 T
P−1 = P .
|P| c
 
 2 0 −4
1
 
= −  0 −4 −4

8  
−2 −4 0
 
Note. We shall apply this procedure to find the inverse of any 3 × 3 matrix here
after.
 
2 −2 3 
 
Example 1.45. Diagonalize the matrix A = 1 1

ww

1 
 
1 3 −1
 

w.E
 
2 −2 3 
 
Solution. Given A = 1 1
 
1 
 

asy
1 3 −1
 
Step 1. To find the eigen values

En
Since A is a 3 × 3 matrix, the characteristic equation is of the form

λ3 − s1 λ2 + s2 λ − s3 = 0.

gin
Now, s1 = sum of the main diagonal elements


= 2 + 1 − 1 = 2.


ee
s2 = sum of the minors of the main diagonal elements

rin
g.n
1 1 2 3 2 −2
= + +
3 −1 1 −1 1 1

= −4 − 5 + 4 = −5.

2 −2 3

s3 = |A| = 1 1


et
1
1 3 −1

= 2(−1 − 3) + 2(−1 − 1) + 3(3 − 1)

= −8 − 4 + 6

= −6.

Downloaded From : www.EasyEngineering.net


Downloaded From : www.EasyEngineering.net

Matrices 77

The characteristic equation is

λ3 − 2λ2 − 5λ + 6 = 0.

λ = 1 is a root.

By synthetic division we have

1 1 −2 −5 6
0 1 −1 −6

ww 1 −1

λ2 − λ − 6 = 0
−6 0

w.E (λ − 3)(λ + 2) = 0

asy λ = −2, 3.

En
The eigen values are 1, −2, 3.
Step 2. To find the eigen vectors

gin
 
 x1 
 
Let X =  x2  be the eigen vector corresponding to an eigen value λ.
 
 
x3
 
ee ∴ (A − λI)X = 0.
rin

2 − λ −2
 3
 
  x1 
   g.n
1−λ 1   x2  = 0.
et (1)
  
 1   

1 3 −1 − λ x3
 

When λ = 1, (1) becomes


  
1 −2 3   x1 
   
1   x2  = 0.
  
1 0   

1 3 −2 x3
 

The equations become


x1 − 2x2 + 3x3 = 0 (2)

Downloaded From : www.EasyEngineering.net


Downloaded From : www.EasyEngineering.net

78 Engineering Mathematics - II

x1 + x3 = 0 (3)

x1 + 3x2 − 2x3 = 0. (4)

(3) ⇒ x1 = −x3
x1 x3
=
−1 1
⇒ x1 = −1, x3 = 1.
Substituting in (2) we get
−1 − 2x2 + 3 = 0

ww 2 = 2x2
x2 = 1.  

w.E −1
 
∴ X1 =  1 .
 
1

asy
 
When λ = −2 (1) becomes   

En
4 −2 3  x1 
   
1 3 1  x  = 0.
  2 

gin

   
1 3 1 x3

ee
The above equations are reduced to
4x1 − 2x2 + 3x3 = 0
x1 + 3x2 + x3 = 0.
rin
By the rule of cross multiplication, we obtain
x1 x2 x3 g.n
−2
3
3
1
4
1
−2
3
et
x1 x2 x3
= =
−2 − 9 3 − 4 12 + 2
x1 x2 x3
= =
−11 −1 14
∴ x1 = 11, x2 = 1, x3 = −14

Downloaded From : www.EasyEngineering.net


Downloaded From : www.EasyEngineering.net

Matrices 79

 
 11 
 
∴ X2 =  1  .
 
 
−14
 

When λ = 3, (1) becomes


  
−1 −2 3   x1 
   
 1 −2 1   x2  = 0.
   
   
1 3 −4 x3

ww The above equations are reduced to

w.E
−x1 − 2x2 + 3x3 = 0 (5)

x1 − 2x2 + x3 = 0 (6)

asy x1 + 3x2 − 4x3 = 0. (7)

En
All the three equations are different.

gin
Taking (5) & (6) and by the rule of cross multiplication we obtain

−2
−2
x1

ee 3
1
x2
−1
1
x3
−2
−2
rin
g.n
x1
=
x2

x1
4
=

=
x3
−2 + 6 3 + 1 2 + 2
x2
4
=
x3
4
et
∴ x1 = x2 = x3 = 1
 
1
 
X3 = 1 .
 
1

Downloaded From : www.EasyEngineering.net


Downloaded From : www.EasyEngineering.net

80 Engineering Mathematics - II

Step 3. To form the modal matrix P


The modal matrix P is formed with the eigen vectors X1 , X2 , X3 as columns.
 
−1 11 1
   
i.e., P = X1 X2 X3 =  1 1 1 .
 
 
1 −14 1
 

Step 4. To find P−1



−1 11 1

ww |P| = 1 1 1

1 −14 1

w.E = −1(1 + 14) − 11(1 − 1) + 1(−14 − 1)

= −15 + 0 − 15

= −30.
To find Pc asy
1 1 1 1
En
−14 1
11 1 −1
1 −14
11 gin
1 1

 15

1 1

0 −15
Pc = −25 −2 −3 

ee rin
g.n

 
10 2 −12
 

P−1 =
1 T
P
|P| c

1 

 15 −25 10 


et
= −  0 −2 2 
30  
−15 −3 −12
 
 
−15 25 −10
1 
 
=  0 
2 −2 
30  
15 3 12

Downloaded From : www.EasyEngineering.net


Downloaded From : www.EasyEngineering.net

Matrices 81

Step 5. To find P−1 AP


   
−15 25 −10 2 −2 3  −1 11 1
1 
     
−1
P AP =
    
 0
 2 −2  1 1 1   1 1 1
30   
   
  
15 3 12 1 3 −1 1 −14 1
 
  
−15 25 −10 −1 11 1
1 
   
=  0 −4 4   1 
1 1
30   
 
45 9 36 1 −14 1
  
 

ww
30 0 0 
1 
 
=  0 −60 0 
30  

w.E 0 0 90
 
 
1 0 0
 
= 0 −2 0 .

asy
 
 
0 0 3
 
which is the required diagonal form.

En 
 6 −2 2 

Example 1.46. Diagonalize the matrix −2 3 −1.


gin 






ee 2 −1 3
 

rin
 
 6 −2 2 
 
Solution. Let A = −2 3 −1.

g.n
 
 
2 −1 3
 
Step 1. To find the eigen values

s1 = tr(A) = 6 + 3 + 3 = 12.

s2 = sum of the minors of the main diogonal elements.


et

3 −1 6 2 6 −2
= + +
−1 3 2 3 −2 3

= 9 − 1 + 18 − 4 + 18 − 4

= 8 + 14 + 14 = 36.

Downloaded From : www.EasyEngineering.net


Downloaded From : www.EasyEngineering.net

82 Engineering Mathematics - II


6 −2 2

s3 = |A| = −2 3 −1

2 −1 3

= 6(8) + 2(−4) + 2(−4)

= 48 − 8 − 8 = 32.

The characteristic equation is

ww λ3 − s1 λ2 + s2 λ − s3 = 0

i.e., λ3 − 12λ2 + 36λ − 32 = 0.

w.E λ = 2 is a root.

asy
By synthetic division we have

En 2 1 −12 36 −32

gin
0 2 −20 32
1 −10 16 0

ee
Hence, the characteristic equation becomes

(λ − 2)(λ2 − 10λ + 16) = 0


rin
(λ − 2)(λ − 2)(λ − 8) = 0
g.n
∴ The eigen values are λ1 = 2, λ2 = 2, λ3 = 8.
λ = 2, 2, 8.
et
Step 2. To find the eigen vectors
 
 x1 
 
Let X =  x2  be the eigen vector corresponding to an eigen value λ.
 
x3
 

∴ (A − λI)X = 0.

Downloaded From : www.EasyEngineering.net


Downloaded From : www.EasyEngineering.net

Matrices 83

  
6 − λ −2 2   x1 
   
 −2 3 − λ −1   x2  = 0.
   
   
2 −1 3 − λ x3

(6 − λ)x1 − 2x2 + 2x3 = 0 






−2x1 + (3 − λ)x2 − x3 = 0  . (A)





2x1 − x2 + (3 − λ)x3 = 0 

case(i) when λ = 2, (A) reduces to

ww 4x1 − 2x2 + 2x3 = 0

w.E −2x1 + x2 − x3 = 0

asy 2x1 − x2 + x3 = 0.

The above three equations are reduced to the single equation

En 2x1 − x2 + x3 = 0.

gin
Since λ = 2 is a repeated root, we have to find two eigen vectors by assigning a

Assigning x3 = 0, we obtain
ee
particular value to two variables, one at a time.

rin
2x1 = x2
x1 x2 g.n
et
=
1 2
 
1
 
∴ X1 = 2 .
 
 
0
 

Assigning x2 = 0, we obtain
2x1 = −x3
x1 x3
= .
−1 2

Downloaded From : www.EasyEngineering.net


Downloaded From : www.EasyEngineering.net

84 Engineering Mathematics - II

 
−1
 
∴ X2 =  0  .
 
 
2
 

case(ii) when λ = 8, (A) reduces to

−2x1 − 2x2 + 2x3 = 0 (1)

−2x1 − 5x2 − x3 = 0 (2)

2x1 − x2 − 5x3 = 0. (3)

ww Since all the three equations are different, we can consider the the first two
equations and solve for x1 , x2 , &, x3 by the method of cross multiplication

w.E −2
x1
2
x2
−2
x3
−2

asy
−5 −1 −2 −5

En x1
=
x2
=
x3

gin
2 + 10 −4 − 2 10 − 4
x1
=
12 −6
x2
=
x3
6

eex1
2
=
x2
−1
=
x3
1
∴ x1 = 2, x2 = −1, x3 = 1.
rin
x1 , x2 , x3 satisfy (3).
  g.n
et
 2 
 
∴ X3 = −1 .
 
 
1
 

Step 3. To form the modal matrix P


P can be obtained by considering X1 , X2 , X3 as columns.
 
1 −1 2 
 
∴ P = 2 0 −1

 
0 2 1

Downloaded From : www.EasyEngineering.net


Downloaded From : www.EasyEngineering.net

Matrices 85

Step 4. To find P−1


1 −1 2

|P| = 2 0 −1

0 2 1

= 1(0 + 2) + 1(2 − 0) + 2(4 − 0) = 2 + 2 + 8 = 12.

To find Pc

ww 0 −1 2 0

w.E
2 1 0 2
−1 2 1 −1
0 −1 2 0

asy 
2 −2 4 

En
 
Pc = 5 1 −2
 
 
1 5 2
 

ginP−1 =
1 T
P .
|P| c

ee =

 2
1 
12 

5 1

−2 1 5 .

rin
4 −2 2
g.n
 

Step 5. To find P−1 AP

1 

 2
 5 1
 
  6 −2 2  1 −1 2 
   


et
P−1 AP = −2 1 5 −2 3 −1 2 0 −1
12      
4 −2 2 2 −1 3 0 2 1
   
  
 12 − 10 + 2 −4 + 15 − 1 4 − 5 + 3  1 −1 2 
1 
   
= −12 − 2 + 10 4 + 3 − 5 −4 − 1 + 15 2 0 −1
12    
24 + 4 + 4 −8 − 6 − 2 8+2+6 0 2 1
  

Downloaded From : www.EasyEngineering.net


Downloaded From : www.EasyEngineering.net

86 Engineering Mathematics - II

  
 4 10 2  1 −1 2 
1 
   
= −4 2 10 2 0 −1
12    
32 −16 16 0 2 1
  
 
 4 + 20 + 0 −4 + 0 + 4 8 − 10 + 2 
1 
 
=

 −4 + 4 + 0
 4 + 0 + 20 −8 − 2 + 10 
12  
32 − 32 + 0 −32 + 0 + 32 64 + 16 + 16
 
   
24 0 0  2 0 0
1 
   

ww =  0 24 0  = 0 2 0



 

12    
0 0 96 0 0 8
   

w.E
which is the required diagonal form.
 
3 1

asy
Example 1.47. Find eA and 4A if A =  .

1 3

Solution.
Step 1. To find the eigen values En
gin
s1 = sum of the main diagonal elements.

= 3 + 3 = 6.

s2 = |A| =
ee
3 1

= 9 − 1 = 8.
rin
g.n
1 3

The characteristic equation is

λ2 − s1 λ + s2 = 0

λ2 − 6λ + 8 = 0
et
(λ − 2)(λ − 4) = 0

λ = 2, 4.

The eigen values are λ1 = 2, λ2 = 4.

Downloaded From : www.EasyEngineering.net


Downloaded From : www.EasyEngineering.net

Matrices 87

Step 2. To find the eigen vectors


 
 x1 
Let X =   be the eigen vector corresponding to an eigen value λ.
x2

∴ (A − λI)X = 0
  
3 − λ 1   x1 
i.e.     = 0.
 
1 3 − λ  x2 

(3 − λ)x1 + x2 = 0 

ww


(A)


x1 + (3 − λ)x2 = 0. 

w.E
case(i) when λ = 2, (A) reduces to one single equation

x1 + x2 = 0

asy i.e.x1 = −x2

En x1
−1
= .
x2
1

gin  
−1
∴ X1 =   .

case(ii) when λ = 4, (A) reduces to ee 1

rin
−x1 + x2 = 0
g.n
x1 − x2 = 0

Both the equations are reduced to one single equation et


x1 = x2

x1 x2
= .
1 1
 
1
∴ X2 =   .
1

Downloaded From : www.EasyEngineering.net


Downloaded From : www.EasyEngineering.net

88 Engineering Mathematics - II

Step 3. To form the modal matrix P


P is obtained by taking X1 , X2 as columns.
 
−1 1
∴ P =   .

1 1

Step 4. To find P−1



−1 1
|P| = = −1 − 1 = −2.

ww   
1 1

w.E
1 1  1 −1 1 −1 1
P−1 = PT =   =  .
|P| c −2 −1 −1 2  1 1
Step 5. To find P−1 AP

asy
P−1 AP =
   
1 −1 1 3 1 −1 1

En  
2  1 1 1 3  1 1
 

gin
  
1 −3 + 1 −1 + 3 −1 1
=    
2  3 + 1 1 + 3   1 1

= ee 


1 −2 2 −1 1

2  4 4  1 1

rin
g.n
 
1  2 + 2 −2 + 2
=  
2 −4 + 4 4 + 4 

=


 
1 4 0 2 0
=
2 0 8 0 4

. et
Step 6. To find eA and 4A
By similarity transformation

 
2 0
−1  .
D = P AP =  

0 4

Downloaded From : www.EasyEngineering.net


Downloaded From : www.EasyEngineering.net

Matrices 89

 
e2 0 
Let f (A) = eA , then f (D) = eD =  .
4
0 e 

Now, eA = P f (D)P−1
    
−1 1 e2 0  1 −1 1
=      
1 1  0 e4  2  1 1
  
  
1 −e2 e4  −1 1
=   
2 e2 e4   1 1

ww eA = 
1

 e2 + e4 −e2 + e4 
2 −e + e

2 4 2 4

 .

w.E

e +e 

Replacing e by 4, we get

asy
 
1  42 + 44 −42 + 44 
A
4 =  
2 −42 + 44 42 + 44 

En 
1  16 + 256 −16 + 256
= 

gin

2 −16 + 256 16 + 256 
   
= 
ee
1 272 240 136 120
=
2 240 272 120 136
.

rin
Exercise I(D)
g.n
1. Find a nonsingular matrix P such that P−1 AP is in a diagonal form where A
is given by
 
−2 2 
 
0 1

0 0 1



et
i.  ii.  iii. 0 1 0
   
 
2 −5 1 0  
1 0 0
 
     
0 2 0 2 0 2  4 6 6 
     
iv. 2 0 2 v. 0 2 0 vi.  1
  
3 2 
     
0 2 0 2 0 2 −2 −4 −3
     

Downloaded From : www.EasyEngineering.net


Downloaded From : www.EasyEngineering.net

90 Engineering Mathematics - II

2. Reduce the following matrices to the diagonal form using similarity


transformation.
   √   
−1 2 −2  −1 − 3 0  7 −2 −2
   √   
i. 1 1  ii. − 3 0 iii. −2 1
   
2 1 4 
     
−1 −1 0 0 0 0 −2 4 1
     
     
 2 2 0  −15 4 3    2 −1 0 
     1 −2  
iv.  2 1 1  v.  10 −12 6 vi.   vii.  9 6 .
     
4

 

    −5 4 
 
−7 2 −3 20 −4 2 −8 0 −3
    

ww 3. Reduce the matrix A = 



5 3

 to the diagonal form and hence evaluate An and

w.E

1 3
A4 .
 

asy
 3 −1 1 
 
4. Find A4 if A = −1 5 −1
 

En
 
1 −1 3

gin
 
1 −1 2 
 
5. Reduce the matrix A = 0 2 −1 to the diagonal form by similarity

transformation. Hence find A3 .




0 0 3


ee rin
1.6 Diagonlization by orthogonal transformation g.n
A real square matrix A is said to be orthogonal if AAT = AT A = I. et
Result. A real square matrix A is orthogonal if AT = A−1 .

Normalized eigen vector


 
a
 
Let X = b be an eigen vector corresponding to an eigen value λ. Then the
 
c
 

Downloaded From : www.EasyEngineering.net


Downloaded From : www.EasyEngineering.net

Matrices 91

X
normalized eigen vector of X is defined as √ .
a2 + b2 + c2
Example
   
 1   √1 
   6 
If X = −1 is an eigen vector, then its normalised eigen vector is  −1 √ .
   
   6 
2
   √2 
6
Diagonalization by orthogonal transformation
A real square matrix A is said to be orthogonally diagonalizable, if there exists an
orthogonal matrix N such that N −1 AN = D ⇒ N T AN = D. This transformation

ww which transforms A into D is called an orthogonal transformation or orthogonal


reduction.

w.E
Symmetric Matrices
A real square matrix A is said to be symmetric if AT = A.

asy
Working rule for orthogonal reduction

En
Step 1. Find the eigen values λ1 , λ2 , . . . , λn .

gin
Step 2. Find eigen vectors X1 , X2 , . . . , Xn which are pairwise orthogonal.

as columns. ee
Step 3. Form the normalised modal matrix N with the normalised eigen vectors

rin
g.n
 
λ1 0 . . . 0 
 
 0 λ . . . 0 
2
Step 4. Find N T and N T AN = D =  . .
et
 
. .
 .. .. .. 


 0 0 . . . λ 
n

Definition 1.1. If Xi , X j , i, j = 1, 2, . . . , n are orthogonal, then XiT X j = 0, i , j.

Properties of eigen values of orthogonal matrices

Property 1.9. The eigen values of a real symmetric matrix are real.

Downloaded From : www.EasyEngineering.net


Downloaded From : www.EasyEngineering.net

92 Engineering Mathematics - II

Proof. Let λ (real or complex) be an eigen value of A.


Let X be the eigen vector corresponding to the eigen value λ.

∴ AX = λX. (1)

Let X̄ be the Complex conjugate of X.


Premultiply (1) by X̄ T we get
X̄ T AX = λX̄ T X

X̄ T AX = λX̄ T X

ww i.e., X T ĀX̄ = λ̄X T X̄.


Since A is a real symmetric matrix, Ā = A & AT = A.

w.E ∴ X T AX̄ = λ̄X T X̄.

T  T asy
Taking transpose on both sides we get

En

X T AX̄ = λ̄X T X̄

X̄ T AT X = X̄ T X λ̄T = λ̄X̄ T X

X̄ T AX = λ̄X̄ T X
gin
X̄ T λX = λ̄X̄ T X

λX̄ T X = λ̄X̄ T X.
ee
Now X is an n × 1 matrix and X̄ T is a 1 × n matrix. rin
Hence, X̄ T X is a 1 × 1 matrix, which is a positive real.
g.n
⇒ λ = λ̄.
⇒ λ is a real number. et 

Property 1.10. The eigen vectors corresponding to distinct eigen values of a real
symmetric matrix are orthogonal. [Dec.2002]

Proof. By property 1.9, we know that, if A is a real symmetric matrix, then its
eigen values are real.
Let λ1 and λ2 be two real eigen values such that λ1 , λ2 .

Downloaded From : www.EasyEngineering.net


Downloaded From : www.EasyEngineering.net

Matrices 93

Let X1 and X2 be the eigen vectors corresponding to the eigen values λ1


and λ2 respectively.
∴ AX1 = λ1 X1 (1)

AX2 = λ2 X2 . (2)

Premultiplying (1) by X2T we get

X2T AX1 = X2T λ1 X1 = λ1 X2T X1 .

ww Taking transpose both sides we get


 T 
X2T AX1 = λ1 X2T X1
T

w.E ⇒ X1T AT X2 = λ1 X1T X2

i.e., X1T AX2 = λ1 X1T X2 [since, A is symmetric, AT = A].

asy
(3)
Premultiplying (2) by X1T we get

EnX1T AX2 = X1T λ2 X2 = λ2 X1T X2 . (4)

From (3) and (4) we get


gin
ee
λ1 X1T X2 = λ2 X1T X2

i.e., λ1 X1T X2 − λ2 X1T X2 = 0


rin
(λ1 − λ2 )X1T X2 = 0.
g.n
Since λ1 , λ2 , λ1 − λ2 , 0.

∴ X1T X2 = 0
et
⇒ X1 and X2 are orthogonal.

1
Property 1.11. If λ is an eigen value of an orthogonal matrix, then is also its
λ
eigen value.

Downloaded From : www.EasyEngineering.net


Downloaded From : www.EasyEngineering.net

94 Engineering Mathematics - II

Proof. Let A be an orthogonal matrix.

⇒ AT = A−1 . (1)

We know that A and AT have the same eigen values.


1
Also we know that if λ is an eigen value of A, then is an eigen value of A−1 .
λ
1
From (1), we get is an eigen value of AT .
λ
1
⇒ is an eigen value of A. 
λ

ww Results

w.E 1. Diagonalization of orthogonal transformation is possible only for a real


symmetric martix.

asy
2. If A is a real symmetric matrix, then the eigen vectors of A will be not only
linearly independent but also pairwise orthogonal.

En
3. If A is orthogonal, then AT is orthogonal.

gin
4. If A is an orthogonal matrix, then |A| = ±1.


ee
5. The eigen values of an orthogonal matrix are of magnitude 1.

Worked Examples


✌ rin
Example 1.48. Prove that A = 

 cos θ sin θ 

 is orthogonal. g.n [Jan 2009]

 
− sin θ cos θ
Solution. |A| = cos2 θ + sin2 θ = 1, which is nonsingular.

 cos θ sin θ  cos θ − sin θ



et
AAT =    
− sin θ cos θ sin θ cos θ 
 
   
 cos2 θ + sin2 θ − sin θ cos θ + sin θ cos θ 1 0
=   0 1 = I
  =  
− sin θ cos θ + cos θ sin θ sin2 θ + cos2 θ

∴ A is orthogonal.

Downloaded From : www.EasyEngineering.net


Downloaded From : www.EasyEngineering.net

Matrices 95

 
 1 2 2 
1 
 
Example 1.49. Show that A =  2 1 −2 is orthogonal.
3  
−2 2 −1
 
 
 1 2 2 
1 
 
Solution. Given A =  2 1 −2
3  
−2 2 −1
 
 
1 2 −2
1 
 
AT = 2 1

2 

ww 3 

2 −2 −1


w.E
  
 1 2 2  1 2 −2
1 
   
T
AA =  2 1 −2 2 1
  
 2 
9    
−2 2 −1 2 −2 −1

asy
  
   
9 0 0 1 0 0
1 

En
   
= 0 9 0 = 0 1 0 = I

 
 
9   
   
0 0 9

gin
0 0 1
 
∴ A is orthogonal.

Solution. Since A is orthogonal, AT = A−1 .


ee
Example 1.50. If A is an orthogonal matrix, prove that A−1 is also orthogonal.

rin
Let AT = A−1 = B.
Now BT = (A−1 )T = (AT )−1 = B−1 . g.n
∴ B is orthogonal.
i.e., A−1 is orthogonal. et
Example 1.51. Prove that if A and B are orthogonal matrices, then AB is
orthogonal.
Solution. Given A & B are orthogonal.
∴ AT = A−1 and BT = B−1 .
Now, (AB)T = BT AT = B−1 A−1 = (AB)−1 ⇒ AB is orthogonal.

Downloaded From : www.EasyEngineering.net


Downloaded From : www.EasyEngineering.net

96 Engineering Mathematics - II

 
 2 1 −1 
 
Example 1.52. Diagonalise the symmetric matrix A =  1 1 −2 by an
 
 
−1 −2 1
 
orthogonal transformation. [Jan 2004]
Solution. A is a real symmetric matrix.
Step 1. To find the eigen values
Since A is a 3 × 3 matrix, the characteristic equation is λ3 − s1 λ2 + s2 λ − s3 = 0.
Now s1 = sum of the main diagonal elements

ww = 2 + 1 + 1 = 4.

s2 = sum of the minors of the main diagonal elements.

w.E
=

1 −2 2 −1 2 1
+ +


−2 1 −1 1 1 1

=1−4+2−1+2−1 asy
= −3 + 1 + 1 = −1.

En

gin
2 1 −1

s3 = |A| = 1 1 −2

ee

−1 −2 1

= 2(−3) − 1(−1) − 1(−1)


rin
= −6 + 1 + 1 = −4.
The characteristic equation is λ3 − 4λ2 − λ + 4 = 0.
g.n
By synthetic division we get,
λ = 1 is a root.
et
1 1 −4 −1 4
0 1 −3 −4
1 −3 −4 0

λ2 − 3λ − 4 = 0.

Downloaded From : www.EasyEngineering.net


Downloaded From : www.EasyEngineering.net

Matrices 97

(λ + 1)(λ − 4) = 0.

∴ The characteristic equation becomes (λ − 1)(λ + 1)(λ − 4) = 0.


∴ The eigen values are −1, 1, 4.
Step 2. To find the eigen vectors
 
 x1 
 
Let X =  x2  be the eigen vector corresponding to λ. Then,
 
 
x3
 

ww 
2 − λ
(A − λI)X = 0.

1 −1
   
  x1  0

w.E
     
1 − λ −2   x2  = 0 . (1)

 1
     
−1 −2 1 − λ x3 0
When λ = −1, (1) becomes
asy 
 3 1 −1
 
  x1 

En
   
2 −2  x2  = 0.
  
 1   

gin

−1 −2 2 x3
 

The equations are reduced to two equations


3x1 + x2 − x3 = 0.
x1 + 2x2 − 2x3 = 0.
By the rule of cross multiplication we get,
ee rin
x1 x2 x3
g.n
1
2
−1
−2
3
1
1
2 et
x1 x2 x3
= = =⇒ x1 = 0, x2 = 1, x3 = 1.
0 5 5
 
0
 
∴ X1 = 1.
 
1

Downloaded From : www.EasyEngineering.net


Downloaded From : www.EasyEngineering.net

98 Engineering Mathematics - II

When λ = 1 (1) become   


 1 1 −1  x1 
   
0 −2  x2  = 0.

 1   

−1 −2 0 x3
 

i.e., x1 + x2 − x3 = 0.
x1 x3
x1 − 2x3 = 0 ⇒ x1 = 2x3 ⇒ = =⇒ x1 = 2, x3 = 1.
2 1
x1 x2
−x1 − 2x2 = 0 ⇒ x1 = −2x2 ⇒ = =⇒ x1 = 2, x2 = −1.
2 −1

ww  
 2 
∴ x1 = 2, x2 = −1, x3 = 1

w.E  


∴ X2 = −1.
 
 
1
When λ = 4, (1) become
asy   
−2 1 −1  x1 

En
   
 1 −3 −2  x2  = 0.
   

gin
   
−1 −2 −3 x3
i.e., −2x1 + x2 − x3 = 0 ⇒ 2x1 − x2 + x3 = 0.
x1 − 3x2 − 2x3 = 0.
ee
−x1 − 2x2 − 3x3 = 0 ⇒ x1 + 2x2 + 3x3 = 0.
All the three equations are different. rin
Taking the first two equations and solving by the rule of cross multiplication we
g.n
get

−1
x1
1
x2
2
x3
−1
et
−3 −2 1 −3

x1 x2 x3
i.e., = = .
5 5 −5
=⇒ x1 = 1, x2 = 1, x3 = −1.

Downloaded From : www.EasyEngineering.net


Downloaded From : www.EasyEngineering.net

Matrices 99

 
 1 
 
∴ X3 =  1 .
 
 
−1
 
Since A is symmetric and all the eigen values are different X1 , X2 , X3 are pairwise
orthogonal.
Step 3. To form the modal matrix N
     
 0   √2   √1 
   6   3 
The normalized eigen vectors are  √1  ,  −1
√ 
 and  √1 .

ww
 2   6   3 
 
 √1   √1  −1

2 6 3
The modal matrix N is formed with the normalized eigen vectors as columns.

w.E
 
 0 √2 √1 

 6 3
 1
i.e., N =  √ −1 √1 

.
 2 6 3

asy

√1 √1 −1 


2 6 3
Step 4. To find T
N AN


 0En √1 √1 

  2 1

−1  0 √2 √1 

gin

 2 2     6 3 
N T AN =  √2 −1 √1  1 −2  √12 −1 √1 
  
  1
√ √
 
 6 6 6   6 3 

ee
 √1 √1 −1

 
−1 −2 1  √1
 √1 −1


3 3 3 2 6 3


−1 0 0

rin
g.n
 
=  0 1 0 = D.
 
 
0 0 4
 


1 1 3



Example 1.53. Diagonalise the matrix 1 5 1 by an orthogonal reduction.

et
 
3 1 1
 
 
1 1 3
 
Solution. Let A = 1 5 1.
 
 
3 1 1
 
A is a real symmetric matrix.

Downloaded From : www.EasyEngineering.net


Downloaded From : www.EasyEngineering.net

100 Engineering Mathematics - II

Step 1. To find the eigen values


Since A is a 3 × 3 matrix, the characteristic equation is of the form

λ3 − s1 λ2 + s2 λ − s3 = 0.

Now, s1 = sum of the main diagonal elements of A.

= 1 + 5 + 1 = 7.

s2 = sum of the minors of the main diagonal elements



5 1 1 3 1 1

ww = + +
1 1 3 1 1 5

w.E =5−1+1−9+5−1

= 4 − 8 + 4 = 0.

asy

1 1 3

s3 = |A| = 1 5 1

En

3 1 1

= 1(5 − 1) − 1(1 − 3) + 3(1 − 15)

= 4 + 2 − 42 gin
= −36.
ee
The characteristic equation is λ3 − 7λ2 + 36 = 0.
rin
λ = −2 is a root.
By synthetic division we have g.n
−2 1
0
−7
−2
0
18
36
−36
et
1 −9 18 0

λ2 − 9λ + 18 = 0
(λ − 3)(λ − 6) = 0
λ = 3, λ = 6.
The eigen values are −2, 3, 6.

Downloaded From : www.EasyEngineering.net


Downloaded From : www.EasyEngineering.net

Matrices 101

Step 2. To find the eigen vectors


 
 x1 
 
Let X =  x2  be the eigen vector corresponding to an eigen value λ.
 
 
x3
 

∴ (A − λI)X = 0
  
1 − λ 1 3   x1 
   
5−λ 1   x2  = 0. (1)
  
 1

ww    


3 1 1 − λ x3
 

When λ = −2, (1) becomes

w.E 

 
3 1 3  x1 
  
1 7 1  x2  = 0.
  

asy
   
3 1 3 x3
The above equations are reduced to

En
3x1 + x2 + 3x3 = 0

gin
x1 + 7x2 + x3 = 0.
By the rule of cross multiplication we obtain

1
ee
x1
3
x2
3
x3
1
rin
7 1 1 7

g.n
x1
=
x2
=
1 − 21 3 − 3 21 − 1
x1
=
x2
=
x3
x3
et
−20 0 20
x1 = 1, x2 = 0, x3 = −1.
 
 1 
 
∴ X1 =  0  .
 
−1

Downloaded From : www.EasyEngineering.net


Downloaded From : www.EasyEngineering.net

102 Engineering Mathematics - II

When λ = 3, (1) becomes


  
−2 1 3   x1 
   
 1 2 1   x  = 0.
   2 
   
3 1 −2 x3
The above equations become
−2x1 + x2 + 3x3 = 0
x1 + 2x2 + x3 = 0
3x1 + x2 − 2x3 = 0.

ww Taking the first two equations and applying the rule of cross multiplication we get
x1 x2 x3

w.E 1
2
3
1
−2
1
1
2

asy x1 x2 x3

En =
1 − 6 3 + 2 −4 − 1
x1 x2
=
x3

gin
= =
−5 5 −5
∴ x1 = 1, x2 = −1, x3 = 1.

ee  
 1 
 
∴ X2 = −1 .
 
rin
g.n
1
 

When λ = 6, (1) becomes

et
  
−5 1 3   x1 
   
 1 −1 1   x2  = 0.
   
   
3 1 −5 x3
The above equations become
−5x1 + x2 + 3x3 = 0
x1 − x2 + x3 = 0
3x1 + x2 − 5x3 = 0.
Taking the first two equations and applying rule of cross multiplication we get

Downloaded From : www.EasyEngineering.net


Downloaded From : www.EasyEngineering.net

Matrices 103

x1 x2 x3
1 3 −5 1
−1 1 1 −1
x1 x2 x3
= =
1+3 3+5 5−1
x1 x2 x3
= =
4 8 4
∴ x1 = 1, x2 = 2, x3 = 1.
 
1
 

ww ∴ X3 = 2 .
 
 
1
 

w.E
Since A is symmetric and all the eigen values are different x1 , x2 , x3 are pairwise
orthogonal.
Step 3. To form the modal matrix N
asy
    
 √1   √1   √1 
 2   3   6 

En
The normalized eigen vectors are  0  , − √1   √2 .
    
   3   6 

gin
 −1
√   √1   √1 
2 3 6
The modal matrix N is formed with the normalized eigen vectors as columns.

 √1
 2
∴ N =  0 



√1


ee
3
1
3
√2

√1 

6

6



rin
g.n
− √1 √1 √1

2 3 6

Step 4. To find N T AN

T

 √1
 2
N AN =  √1

0
− √1
2
√1 

 

− √1  1 1 3  √1
  2
 1 5 1  0
 
 − √1
√1
3
√1 
6
√2 




et
 3 3 3  
  
  3 6 

 √1 √2 √1 3 1 1 − √1  √1 √1
6 6 6 2 3 6
 √ √   1   
− 2 0 2   √ √1 √1  −2 0 0
3
√   2 6
 √ 

   
1 2 
=  3 − 3 3  0 −   0 3 0 .
=
  
√ √
  

 √ 3 6
√ √   1   
   
6 2 6 6 −√ √1 √1 0 0 6
 
2 3 6

Downloaded From : www.EasyEngineering.net


Downloaded From : www.EasyEngineering.net

104 Engineering Mathematics - II

 
3 1 1 
 
Example 1.54. Diagonalise the matrix A = 1 3 −1 by means of an orthogonal
 
 
1 −1 3
 
transformation. [Jan 2004]
Solution. A is a symmetric matrix.
Step 1. To find the eigen values
Since A is a 3 × 3 matrix, the characteristic equation is of the form
λ3 − s1 λ2 + s2 λ − s3 = 0.

ww Nows1 = sum of the main diagonal elements

= 3 + 3 + 3 = 9.

w.E s2 = sum of the minors of the main diagonal elements



3 −1 3 1 3 1

= +

asy +
−1 3 1 3 1 3

3 1 En
= 9 − 1 + 9 − 1 + 9 − 1 = 24.

1


s3 = |A| = 1 3 −1
gin

ee
1 −1 3

= 3(9 − 1) − 1(3 + 1) + 1(−1 − 3)

= 24 − 4 − 4 = 16. rin
∴ The characteristic equation is λ3 − 9λ2 + 24λ − 16 = 0.
g.n
By synthetic division we have
λ = 1 is a root.
et
1 1 −9 24 −16
0 1 −8 16
1 −8 16 0

=⇒ λ2 − 8λ + 16 = 0.

Downloaded From : www.EasyEngineering.net


Downloaded From : www.EasyEngineering.net

Matrices 105

(λ − 4)(λ − 4) = 0.

The characteristic equation becomes (λ − 1)(λ − 4)(λ − 4) = 0.


∴ The eigen values are 1, 4, 4.
Step 2. To find the eigen vectors
 
 x1 
 
Let X =  x2  be the eigen vector corresponding to λ. Therefore,
 
 
x3
 

ww 
3 − λ
(A − λI)X = 0.

1
 
1   x1 

w.E 

 1

1 −1 3 − λ x3
  
3 − λ −1   x2  = 0.
  
 
(1)

When λ = 1, (1) becomes


asy 
2 1 1
 
  x1 

En    


1 2 −1  x2  = 0.
  

gin
   
1 −1 2 x3
The above equations become

ee 2x1 + x2 + x3 = 0.

x1 + 2x2 − x3 = 0.
rin
x1 − x2 + 2x3 = 0.
g.n
Taking the first two equations and solving for x1 , x2 , x3 by the rule of cross
multiplication we get
x1 x2 x3
et
1 1 2 1
2 −1 1 2

x1 x2 x3
= =
−1 − 2 1 + 2 4 − 1

Downloaded From : www.EasyEngineering.net


Downloaded From : www.EasyEngineering.net

106 Engineering Mathematics - II

x1 x2 x3
=⇒ = = .
−3 3 3
=⇒ x1 = −1, x2 = 1, x3 = 1.
 
−1
 
∴ X1 =  1 .
 
 
1
 
When λ = 4, (1) becomes
  
−1 1 1   x1 
   
 1 −1 −1  x2  = 0.


ww 
1 −1 −1 x3
The above equations are reduced to one single equation
  

w.E x1 − x2 − x3 = 0. (1)

 
1 asy
Put x3 = 0 ⇒ x1 − x2 = 0 ⇒ x1 = x2 =⇒ x1 = 1, x2 = 1.
 
∴ X2 = 1.
 
En
gin
 
0
 
 
a

ee
 
Let X3 = b be orthogonal to X2 .
 

rin
 
c
 
By the condition of orthogonality, X3T X2 = 0.
 
 1
  g.n
et

a b c 1 = 0
 
 
0

=⇒ a + b = 0 ⇒ a = −b.

Also X3 should satisfy (1).

∴ a − b − c = 0 ⇒ −2b − c = 0 ⇒ 2b = −c
b c
⇒ = ⇒ b = −1, c = 2, a = 1.
−1 2

Downloaded From : www.EasyEngineering.net


Downloaded From : www.EasyEngineering.net

Matrices 107

 
 1 
 
∴ X3 = −1.
 
 
2
 
Step 3. To find the modal matrix N
The normalised eigen vectors are
     
 −1 √   √1   √1 
 3   2   6 
 √1  ,  √1  ,  −1
 3   2   √6  .

     
√1 √2

ww 0 3

The normalised modal matrix is formed by the normalised eigen vectors as


6

w.E
columns. 
 −1
 3
√ √1
2
√1 
6




 −1
 3
√ √1
3
√1 

3

asy
∴ N =  √
 1
√1 −1
√ 
 , N T =  √
 1
√1

0 
 3 2 6   2 2 
 √1 0 √2   √1 −1
√ √2

3 6 6 6 6

Step 4. To find N T AN
En

 −1
 3

gin √1
3
√1 

 3
3
  −1
  √
1
  3
1 √1
2
√1 
6


ee
  
N T AN =  √1 1
0  1 3 −1  √1 1 −1
  
√ √ √ 

 2 2     3 2 6 

rin
 √1 −1
√ 2

 
1 −1 3  √1 0 √2 
6 6 6 3 6

g.n
    
 −1 √ √1 √1    −1√ √4 √4  1 0 0
 3 3 3   3 2 6   
=  √1 √1 0   √1 √4 −4 = 0 4 0 .
 
√ 

 2 2   3 2 6

et
  
 √1 −1
√ √2
  √1 0 √8
 
0 0 4

6 6 6 3 6
which is the required diagonal form.

Computation of powers of a square matrix


Let A be the given square matrix. Diagonalising A, we obtain,

D = N T AN = N −1 AN.

D2 = D.D = (N −1 AN)(N −1 AN) = N −1 A(NN −1 )AN

Downloaded From : www.EasyEngineering.net


Downloaded From : www.EasyEngineering.net

108 Engineering Mathematics - II

= N −1 AAN = N −1 A2 N

D3 = N −1 A3 N

In general, Dr = N −1 Ar N
 
λr 0 0 . . . 0 
 1 
r r
Where D =  0 λ2 0 . . . 0 .
 
 
 r
0 0 0 . . . λn

Premultiplying by N and postmultiplying by N −1 we get,

ww Ar = NDr N −1 .

w.E
 
2 0 1
 
Example 1.55. Diagonalise the matrix A = 0 3 0. Hence find A3 . [Jan 2006]
 
 

asy 1 0 2
 
Solution. A is a symmetric matrix.
Step 1. To find the eigen values

En
Since A is a 3 × 3 matrix, the characteristic equation is of the form

ginλ3 − s1 λ2 + s2 λ − s3 = 0.

ee
Now s1 = sum of the main diagonal elements
rin
g.n
= 2 + 3 + 2 = 7.

s2 = sum of the minors of the main diagonal elements

et

3 0 2 1 2 0
= + +
0 2 1 2 0 3

=6−0+4−1+6−0

= 6 + 3 + 6 = 15.

2 0 1

s3 = |A| = 0 3 0

1 0 2

Downloaded From : www.EasyEngineering.net


Downloaded From : www.EasyEngineering.net

Matrices 109

= 2(6 − 0) + 1(0 − 3)

= 12 − 3 = 9.

The characteristic equation is λ3 − 7λ2 + 15λ − 9 = 0.

λ = 1 is a root.

By synthetic division, we get

ww 1 1
0
−7
1
15
−6
−9
9

w.E 1 −6

λ2 − 6λ + 9 = 0.
9 0

asy (λ − 3)(λ − 3) = 0

En ⇒ λ = 3, 3.

The eigen values are 1, 3, 3.


Step 2. To find the eigen vectorsgin
 
 x1 
  ee
Let X =  x2  be the eigen vector corresponding to an eigen value λ.
 
 
rin
x3
g.n
 

et
∴ (A − λI)X = 0
  
2 − λ 0 1   x1 
   
i.e.,  0 3−λ 0   x2  = 0. (1)
 
   
1 0 2 − λ x3
  

When λ = 1, (1) becomes


  
1 0 1  x1 
   
0 2 0  x  = 0.
   2 
   
1 0 1 x3

Downloaded From : www.EasyEngineering.net


Downloaded From : www.EasyEngineering.net

110 Engineering Mathematics - II

The above equations become


x1 + x3 = 0
2x2 = 0.
From the last equation we get x2 = 0.
Also x1 = −x3
x1 x3
=
1 −1
∴ x1 = 1, x3 = −1.
 
 1 

ww
 
∴ X1 =  0 .
 
 
−1
 

w.E
When λ = 3, (1) becomes


 
−1 0 1   x1 
  

asy  0 0 0   x2  = 0.


   
   
1 0 −1 x3

En
The above equations are reduced to one single equation

gin x1 − x3 = 0

⇒ x1 = x3
(2)

 
1
ee
i.e., x1 = 1, x3 = 1 and x2 = any value = 0 (say)

rin
g.n
 
∴ X2 = 0.
 
 
1
 
 
a
 
Let X3 = b be orthogonal to X2 .
 
et
c
 

∴ X3T X2 = 0.
 
 1
 

i.e., a b c 0 = 0 =⇒ a + c = 0. (3)
 
 
1

Downloaded From : www.EasyEngineering.net


Downloaded From : www.EasyEngineering.net

Matrices 111

i.e., Also X3 will satisfy by (2).


Solving (2) & (3) we get a = 0, c = 0. Choose b = 1.
 
0
 
We get the corresponding eigen vector X3 = 1.
 
 
0
 

Step 3. To find the modal matrix N


     
 √1   √1  0
 2   2   
The normalised eigen vectors are  0  ,  0  and 1.
   

ww
 
 −1   1   
0
√ √
 
2 2
The modal matrix N is formed with the normalised eigen vectors as columns.

w.E 
 √1
 2
√1
2
0




asy 0 1 .
∴ N =  0 
 
 −1
√ √1 0
2 2

Step 4. To find

N T AN
En
  

gin
 √1 0 − √1  2 0 1  √1 √1 0
 2 2     2 2 
T
N AN =  √1 0 √1  0 3 0  0

0 1
 
 2 2 

ee
    
1 0 2 − √1 √1

0 1 0 0
2 2

rin
    
 √1 0 − √1   √1 √1 0 1 0 0
 2 2   2 2   
=  √3

g.n
3 
0 1 = 0 3 0 = D.
 
0 √   0
 
 2 2  
  1   
1  0 0 3

0 3 0 − √ √ 0
2 2
Step 5. To find A3

1 0 0 

D3 = 0 27 0 


et
 
0 0 27
    
 √1 √1 0 1 0 0   √1 0 −1
√  14 0 13
 2 2     2 2  
3 3 T
Now, A = ND N =  0 1 0 27 0   √1 √1  =  0 27 0  .
    
0 0
 
 
    2
 2   
 −1 √1
 
0 0 0 27  0 1 0 13 0 14
√   
2 2

Downloaded From : www.EasyEngineering.net


Downloaded From : www.EasyEngineering.net

112 Engineering Mathematics - II

 
 8 −6 2 
 
Example 1.56. Diagonalise the matrix A = −6 7 −4. Hence find the value of
 
 
2 −4 3
 

A4 . [Jan 2013]
Solution. A is a symmetric matrix.
Step 1. To find the eigen values
Since A is a 3 × 3 matrix, the characteristic equation is of the form

λ3 − s1 λ2 + s2 λ − s3 = 0.

ww Now, s1 = sum of the main diagonal elements.

w.E = 8 + 7 + 3 = 18.

s2 = sum of the minors of the main diagonal elements

asy

7 −4 8 2 8 −6
= + +
−4 3 2 3 −6 7

En
= 21 − 16 + 24 − 4 + 56 − 36

gin
= 5 + 20 + 20 = 45.

ee
8 −6 2

rin
s3 = |A| = −6 7 −4

2 −4 3

= 8(21 − 16) + 6(−18 + 8) + 2(24 − 14)


g.n
= 40 − 60 + 20 = 0
∴ The characteristic equation is λ3 − 18λ2 + 45λ = 0

λ(λ2 − 18λ + 45) = 0


et
λ(λ − 3)(λ − 15) = 0

λ = 0, 3, 15.

The eigen values are 0, 3, 15

Downloaded From : www.EasyEngineering.net


Downloaded From : www.EasyEngineering.net

Matrices 113

Step 2. To find the eigen vectors


 
 x1 
 
Let X =  x2  be the eigen vector corresponding to an eigen value λ.
 
 
x3
 

∴ (A − λI)X = 0
  
8 − λ −6 2   x1 
   
λ   x  = 0. (1)

 −6 7 − −4   2 

ww
   
2 −4 3 − λ x3
When λ = 0, (1) becomes

w.E
  
 8 −6 2   x1 
   
−6 7 −4  x2  = 0.
  
   

The above equations becomeasy 2 −4 3 x3

En
8x1 − 6x2 + 2x3 = 0

gin
−6x1 + 7x2 − 4x3 = 0
2x1 − 4x2 + 3x3 = 0.

−6
ee
Taking the first two equations and applying the rule of cross multiplication we get
x1
2
x2
8
x3
−6 rin
7 −4 −6 7
g.n
x1
=
x1
x2

x2
=
x3
24 − 14 −12 + 32 56 − 36
x3
et
= =
10 20 20
x1 = 1, x2 = 2, x3 = 2.
 
1
 
∴ X1 = 2 .
 
2

Downloaded From : www.EasyEngineering.net


Downloaded From : www.EasyEngineering.net

114 Engineering Mathematics - II

When λ = 3, (1) becomes


  
 5 −6 2   x1 
   
−6 4 −4  x2  = 0.
  
   
2 −4 0 x3
The equations become
5x1 − 6x2 + 2x3 = 0
−6x1 + 4x2 − 4x3 = 0
2x1 − 4x2 = 0

ww i.e., x1 = 2x2
x1
=
x2

w.E 2 1
∴ x1 = 2, x2 = 1.
Substituting in the first equation we get
10 − 6 + 2x3 = 0

4 + 2x3 = 0
asy
2x3 = −4
En
x3 = −2.
gin
ee
 
 2 
 

rin
∴ X2 =  1  .
 
−2
When λ = 15, (1) becomes
  
−7 −6 2   x1  g.n
et
   
−6 −8 −4   x2  = 0.
   
   
2 −4 −12 x3
The above equations become
−7x1 − 6x2 + 2x3 = 0

−6x1 − 8x2 − 4x3 = 0

2x1 − 4x2 − 12x3 = 0.

Downloaded From : www.EasyEngineering.net


Downloaded From : www.EasyEngineering.net

Matrices 115

Taking the first two equations and applying the rule of cross multiplication, we
get
x1 x2 x3
−6 2 −7 −6
8 4 6 8

x1 x2 x3
= =
−24 − 16 12 + 28 −56 + 36

ww x1
=
x2
−40 40 −20
=
x3

w.E
x1 x2 x3
= =
2 −2 1
∴ x1 = 2, x2 = −2, x3 = 1.
 
 2 
 
∴ X3 = −2.
asy
En
 
 
1
 

gin
Since A is a symmetric matrix and all the eigen values are different, X1 , X2 & X3
are pairwise orthogonal.
Step 3. To find the modal matrix N
   ee
 1   2   2 
 3   3   3 
The normalized eigen vectors are  23 , 13 , −2 rin
g.n
     
     3 
 2   −2   1 
3 3 3
The modal matrix N is formed with the normalized eigen vectors as columns.

 1
 3
∴ N =  32
2
3
1
2 
3 

−2 



et
 3 3  
2 −2 1 

3 3 3

 
1 2 2 
1 
 
= 2 1 −2

3  
2 −2 1
 

Downloaded From : www.EasyEngineering.net


Downloaded From : www.EasyEngineering.net

116 Engineering Mathematics - II

Step 4. To find N T AN
   
1 2 2   8 −6 2  1 2 2 
1
     
N T AN = 2 1 −2 −6 7 −4 2 1 −2
9      
2 −2 1 2 −4 3 2 −2 1
   
  
 0 0 0  1 2 2 
1 
   
=  6 3 −6 2 1 −2
9    
30 −30 15 2 −2 1
  
   

ww
0 0 0  0 0 0 
1 
   
= 0 27 0  = 0 3 0  = D
9    

w.E 0 0 135 0 0 15
 

Step 5. To find A 4
 

asy
0 0 0 
 
D = 0 3 0 .
 
 
0 0 15
En
 
   
0 0 0  0 0 0

gin

   
D4 = 0 34 0  = 0 81 0 .
   
0 0 154 0 0 50625

ee
   

Now A4 = ND4 N T

1 2 2

 0 0 0

 1 2 2

rin
g.n

1 
     
= 2 1 −2 0 81
  
0  2 1 −2
9      
2 −2 1 0 0 50625 2 −2 1

et
   
  
0 162 101250  1 2 2 
1
   
= 0 81 −101250 2 1 −2
 
9    
0 −162 50625 2 −2 1
  
   
 202824 −202338 100926   22536 −22482 11214 
1 
   
= −202338 202581 −101412 = −22482 22509 −11268 .
   
9   
   
100926 −101412 50949 11214 −11268 5661
 

Downloaded From : www.EasyEngineering.net


Downloaded From : www.EasyEngineering.net

Matrices 117

Example 1.57. The eigen vectors of a 3 × 3 real symmetric matrix A corresponding


to the eigen values 2, 3, 6 are [1 0 − 1]T , [1 1 1]T , [−1 2 − 1]T respectively. Find A.
[Jan 2012]
Solution. Given, A is symmetric and the eigen values are different.
∴ The eigen vectors are pairwise disjoint.
 
 √1 √1 −1
√  
 2 3 6 
√1 √2 
Now, N =  0
  .
 3 6 

ww √1
 −1
√ −1


2 3 6

∴ By the orthogonal transformation, we obtain

w.E 
2 0 0

N T AN = D = 0 3 0 .



asy

 
0 0 6
 

En
   1 
 √1 √1 −1
√   2 0 0  √2 0 −1
√  
 2 3 6     2 
T

gin
1 2
Now, A = NDN =  0 3 0  √13 √1 √1 
  
 0
√ √ 
 
 3 6   3 3 
√1 √2
 −1
√ −1

 
0 0 6  −1
 √ −1


2 3 6 6 6 6

ee
 3 −1 1 

= −1 5 −1 .






rin
1 −1 3

g.n
 

Exercise I(E)



 cos θ sin θ 0

1. Prove that the matrix B = − sin θ cos θ 0 is orthogonal.
 
et
 
0 0 1
 

 
 6 −3 2
 
2. Show that the matrix 17 −3 −2 6 is orthogonal.
 
 
2 6 3
 

Downloaded From : www.EasyEngineering.net


Downloaded From : www.EasyEngineering.net

118 Engineering Mathematics - II

3. Prove that the following matrices are orthogonal.


   
0 1 0 −1 2 2 
  1  
(i) 1 0 0 (ii)  2 −1 2 .
  9  
0 0 1 2 2 −1
   

 
 7 −2 0 
 
4. Diagonalise the matrix −2 6 −2.
 
 
0 −2 5
 

ww 
 8 −6 2 




w.E
5. Reduce A = −6 7 −4 to the diagonal form by an orthogonal reduction.
 
 
2 −4 3
 

asy
 10 −2 −5

6. Reduce the matrix A = −2 2


3  to the diagonal form.

En 

−5 3 5


gin 
 6 −2 2 

ee
 
7. By an orthogonal transformation, diagonalise the matrix A = −2 3 −1.
 

rin
 
2 −1 3
 


 2 −1 1 



8. Reduce the matrix A = −1 2 −1 to diagonal form by an orthogonal
  g.n
reduction. Hence find A3 .


1 −1 2


et
9. The eigen vectors corresponding to the eigen values 1, 2, 3 of the symmetric
matrix A are [1 − 1 0]T , [0 0 1]T and [1 1 0]T respectively. Find A.

10. [1 − 1]T and [1 1]T are the eigen vectors of the symmetric matrix A
corresponding to the eigen values 0 and 2 respectively. Find A.

Downloaded From : www.EasyEngineering.net


Downloaded From : www.EasyEngineering.net

Matrices 119

1.7 Quadratic Form

Definition. A homogeneous polynomial of second degree in any number of


variables is called a quadratic form.

Example.
(1) x2 + 2xy + 2y2 is a quadratic form in 2 variables.
(2) ax2 + 2hxy + by2 + cz2 + 2gyz + 2 f zx is a quadratic form in 3 variables.
(3) ax2 + by2 + cz2 + dw2 + 2hxy + 2gyz + 2 f zx + 2ℓxw + 2myw + 2nzw is a quadratic form

ww in 4 variables.
Definition.
n P n
The general quadratic form in n variables x1 , x2 , . . . , xn is

w.E
P
ai j xi x j where ai j ’s are real numbers such that ai j = a ji for all i, j =
j=1 i=1
1, 2, 3, . . . , n.

asy
Notation. Usually the quadratic form is denoted by Q.

En
n X
X n
∴ Q= ai j xi x j .
j=1 i=1

Matrix form  of Q gin


ee

 x1  a11 a12 . . . a1n 
   

rin
   
 x2  a21 a22 . . . a2n 
Let X =  .  , A =  . .. .. ..  where ai j = a ji . Then, A is a symmetric matrix.
 ..   .. . . . 
 
x 
n

a
n1 an2 . . . ann
n P n


g.n
et
ai j xi x j can be written as Q = X T AX. A is called
P
Now the quadratic form Q =
j=1 i=1
the matrix of the quadratic form.
Example. Consider the quadratic form x2 + 4xy + y2 .
i.e., x2 + 2xy + 2yx + y2 .   
1 2  x
The quadratic form is [x y]    .
 
2 1 y
   
 x 1 2
Hence X =   , A = 
   .

y 2 1

Downloaded From : www.EasyEngineering.net


Downloaded From : www.EasyEngineering.net

120 Engineering Mathematics - II

1.8 Canonical form of Q

A quadratic form of Q which contains only the square terms of the variables is
said to be in canonical form.
Example. x2 + 2y2 , x2 − y2 , x2 + y2 + z2 , x12 + x22 + x32 + 4x42 are in canonical forms.
Reduction of Q to canonical form by orthogonal transformation
Let Q = X T AX be a quadratic form in n variables x1 , x2 , . . . , xn and A = [ai j ] be the
symmetric matrix of order n of the quadratic form. We shall reduce A to the

ww diagonal form by an orthogonal transformation. Let X = NY where


λ1 0

N is the
. . . 0 


w.E

 0 λ2 . . . 0 
normalized modal matrix of A so that N T AN = D, where D =  . .. .. 

 .. . . 
 

asy . . . λn 

0 0
where λ1 , λ2 , . . . , λn are the eigen values of A.

En
We have Q = X T AX = (NY)T A(NY) = (yT N Y )A(NY)

gin
= Y T (N T AN)Y = Y T DY.

ee
    
y1  λ1 0 . . . 0  y1 
     

rin
    
y2   0 λ2 . . . 0  y2 
If Y =  .  then Q = [y1 y2 . . . yn ]  . .. ..   .. 
 
 ..   .. . .   . 
 
 
yn


0 0 . . . λn  yn 
  

g.n
which is the required canonical form.
= λ1 y21 + λ2 y22 + · · · + λn y2n
et
Note
(i) In the above canonical form, some λi ’s may be positive or negative or zero.
(ii) In the canonical form, the coefficients are the eigen values of the matrix A.
(iii) A quadratic form is said to be real if the elements of the symmetric matrix
are real.

Downloaded From : www.EasyEngineering.net


Downloaded From : www.EasyEngineering.net

Matrices 121

Definition
If A is the matrix of the quadratic form Q in the variables x1 , x2 , . . . , xn ,
then the rank of Q is equal to the rank of A. The rank of A is denoted by ρ(A).
If rank of A < n, where n is the number of variables or order of A, then |A| = 0
and Q is called a singular form.

Definition
Let Q = X T AX be a quadratic form in n variables x1 , x2 , . . . , xn , where
X = [x1 x2 . . . xn ]T and A is the matrix of the quadratic form.

ww (i) The number of positive and negative eigen values of A is called the rank of

w.Ethe quadratic form. It is denoted by r.


(ii) The number of positive eigen values of A is defined as the index of the

asy
quadratic form. It is also denoted by p. It is equal to the number of positive
terms in the canonical form.

En
(iii) The difference between the number of positive and negative eigen values
of A is called the signature of the quadratic form. It is denoted by s. It is also

gin
equal to the difference between the number of positive and negative terms in

ee
the canonical form. i.e., s = p − (r − p) = 2p − r where ρ(A) = r.
(iv) Q is said to be positive definite if all the n eigen values of A are positive.
i.e., r = n, p = r.
rin
Ex. y21 + y22 + · · · + y2n is positive definite.
g.n
(v) Q is said to be negative definite if all the n eigen values of A are negative.
i.e.,r = n, p = 0.
Ex. −y21 − y22 − · · · − y2n is negative definite.
et
(vi) Q is said to be positive semi definite if all the n eigen values are ≥ 0 with
atleast one eigen value = 0.
i.e., if r < n and p = r.
Ex. y21 + y22 + y24 + · · · + y2r is positive semi definite.

Downloaded From : www.EasyEngineering.net


Downloaded From : www.EasyEngineering.net

122 Engineering Mathematics - II

(vii) Q is said to be negative semi definite if all the n eigen values of A are ≤ 0
with atleast one value = 0.
i.e., if r < n and p = 0.
Ex. −y21 − y22 − y25 − · · · − y2r , (r < n) is negative semi definite.
(viii) Q is said to be indefinite if A has positive and negative eigen values.
Ex. y21 − y22 + y23 − y24 − y25 + · · · + y2n is indefinite.

Result. The nature of the quadratic form can also be found without finding

ww the eigen values of A or without reducing to canonical form but by using the
principal minors of A.

w.EDefinition. Let Q = X T AX be a quadratic form in n variables x1 , x2 , . . . , xn and


let the matrix of the form A be

asy 
a11 a12 . . . a1n 


a21 a22 . . . a2n 



En A =  .
 .. ..
.
..
.
..  .
.



gin
 
an1 an2 . . . ann 


ee

a11 a12 a13
a11 a12

rin

Let D1 = |a11 |, D2 = , D3 = a a a23 etc.

a 21 22
21 a22


a31 a32 a33
Finally Dn = |A|. The determinants D1 , D2 , . . . , Dn are called the principal minors
of A. The quadratic form Q is said to be g.n
(i) Positive definite if Di > 0 for all i = 1, 2, . . . , n.
(ii) Negative definite if (−1)i Di > 0 for all i = 1, 2, . . . , n. i.e.,D1 , D3 , D5 , . . . are
et
negative and D2 , D4 , D6 , . . . are positive.
(iii) Positive semidefinite if Di ≥ 0 for all i = 1, 2, 3, . . . , n and atleast one Di = 0.
(iv) Negative semidefinite if (−1)i Di ≥ 0 for all i = 1, 2, 3, . . . , n and atleast one
Di = 0.
(v) Indefinite in all other cases.

Downloaded From : www.EasyEngineering.net


Downloaded From : www.EasyEngineering.net

Matrices 123

Law of inertia of a quadratic form


The index of a real quadratic form is invariant under a real non singular
transformation. This property is called the law of inertia of the quadratic form.
✎ ☞
Worked Examples
✍ ✌

Example 1.58. Write down the matrix of the quadratic form 2x2 + 3y2 + 6xy.
[Jan 2001]
Solution. The given quadratic form is in two variables.

ww Hence, the matrix of the quadratic form is a 2 × 2 symmetric matrix. Let A be the
required matrix. Then

w.E
 
a11 a12 
A =   .
a21 a22 

a11 = coefficient of x2 = 2.
a12 = a21 = 1 asy
coefficient of xy = 1
× 6 = 3.

En
2 2
a22 = coefficient
  of y2 = 3.

gin
2 3
∴ A =  

3 3

ee
Example 1.59. Write down the matrix of the quadratic form
2x12 − 2x22 + 4x32 + 2x1 x2 − 6x1 x3 + 6x2 x3 .
rin [Jan 2002]
Solution. The given quadratic form contains 3 variables. Hence, the matrix A of
the quadratic form is a 3 × 3 symmetric matrix. The matrix of the quadratic form g.n
et
 
a11 a12 a13 
 
is A = a21 a22 a23 .
 
 
a31 a32 a33
 

a11 = Coeff. of x12 = 2.


a22 = Coeff. of x22 = −2.
a33 = Coeff. of x32 = 4.
1
a12 = a21 = 2 Coeff. of x1 x2 = 21 2 = 1.
1
a13 = a31 = 2 Coeff. of x1 x3 = 21 (−6) = −3.

Downloaded From : www.EasyEngineering.net


Downloaded From : www.EasyEngineering.net

124 Engineering Mathematics - II

a23 = a32 = 21 Coeff. of x2 x3 = 21 6 = 3.


 
 2 1 −3
 
∴ A =  1 −2 3 .
 
 
−3 3 4
 

Example 1.60. Write down the matrix of the quadratic form 2x2 +8z2 +4xy+10xz−2yz
[Jun 2013, Dec 2001].
Solution. The given quadratic form contain 3 variables. Hence the matrix A of
the quadratic form is a 3 × 3 matrix.

ww 
a11 a12 a13 

Hence A = a21 a22 a23 




w.E 

a31 a32 a33


Now, a11 = Coeff.of x2 = 2

asy
a12 = a21 =
1
2
1
× Coeff.of xy = × 4 = 2.
2

En
a13 = a31
1 1
= × Coeff.of xz = × 10 = 5.
2 2

gin
a22 = Coeff.of y2 = 0
1 1


a23 = a32 =

ee
2
× Coeff.of yz = × (−2) = −1.

a33 = Coeff.of z2 = 8.

2

rin
g.n
2 2 5 
 
∴ A = 2 0 −1 .

et
 
5 −1 8
 

 
2 4 5
 
Example 1.61. Write down the quadratic form of the matrix 4 3 1.[Jan 2003]

 
5 1 1
 
Solution. The given matrix is a 3 × 3 symmetric matrix. Hence, the quadratic
form is in 3 variables. The required quadratic form must be of the form

Q = a11 x2 + 2a12 xy + 2a13 xz + a22 y2 + 2a23 yz + a3 z2 .

Downloaded From : www.EasyEngineering.net


Downloaded From : www.EasyEngineering.net

Matrices 125

 
a11 a12 a13 
 
Comparing the given matrix with a21 a22 a23  we get,
 
 
a31 a32 a33
 
a11 = 2, a12 = 4, a13 = 5, a22 = 3, a23 = 1, a33 = 1.
∴ Q = 2x2 + 8xy + 10xz + 3y2 + 2yz + z2 .
= 2x2 + 3y2 + z2 + 8xy + 10xz + 2yz.

Example 1.62. Write down the quadratic form corresponding to the matrix
 
2 2 5 

ww
 
2 0 −1.

 
5 −1 8

w.E Solution. Since the given matrix is a 3 × 3 symmetric matrix, the quadratic form
must be in 3 variables x, y, z. Let Q be of the required quadratic form

asy
∴ Q = a11 x2 + a22 y2 + a33 z2 + 2a12 xy + 2a13 xz + 2a23 yz.

a11 a12 a13 

En
 
Comparing the given matrix with we have
 
a21 a22 a23 
 

gin
a31 a32 a33
a11 = 2, a22 = 0, a33 = 8, a12 = 2, a13 = 5, a23 = −1.
∴ Q = 2x2 + 8z2 + 4xy − 2yz + 10xz.

ee
Example 1.63. Write down the quadratic form corresponding to the matrix

 2 0 −2

rin
g.n
 
 0 2 1 . [Jan 2013]
 
 
−2 1 −2
Solution. Since the given matrix is a 3 × 3 symmetric matrix, the quadratic form
must be in 3 variables x, y, z.
∴ Q = a11 x2 + a22 y2 + a33 z2
Let Q be the required quadratic form.
+ 2a12 xy + 2a13 xz + 2a23 yz.
et
 
a11 a12 a13 
 
Comparing the given matrix with a21 a22 a23  we obtain
 
a31 a32 a33
 
a11 = 2, a12 = 0, a13 = −2, a22 = 2, a23 = 1, a33 = −2.
∴ Q = 2x2 + 2y2 − 2z2 − 4xz + 2yz.

Downloaded From : www.EasyEngineering.net


Downloaded From : www.EasyEngineering.net

126 Engineering Mathematics - II

Example 1.64. Determine the nature of the quadratic form f (x1 , x2 , x3 ) = x12 + 2x22 .
[Jan 2003]
Solution. Since it contains only square terms it is of the canonical form. The
canonical form contains 3 variables but the RHS expression contains only two
terms. The coefficients of the canonical form are the eigen values of the matrix of
the quadratic form. Hence, λ1 = 1, λ2 = 2 and λ3 = 0. Hence, the given quadratic
form is positive semi-definite.

ww Example 1.65. Find the nature of the quadratic form 2x2 + 2xy + 3y2 .
Solution. Let us form the matrix A of the given quadratic form. Since the Q.F

w.E
contains  only two
∴ A = 
 variables, A is a 2 × 2 matrix.
a11 a12 
. From the given Q.E, we have
21 a a 
22

asy
a11 = Coeff.of x2 = 2.

En
1 1
a12 = a21 = × Coeff.of xy = × 2 = 1
2 2
a22 = Coeff.of y2 = 3.
 
2 1
∴ A =  gin


ee

1 3
Let us analyse the nature of the Q.F with the help of the principal minors.

rin
g.n
D1 = |2| = 2.

D2 =
2
1
1
3
= 6 − 1 = 5.

Since all Di > 0, the quadratic form is positive definite.


et
Example 1.66. Find the nature of the quadratic form 6x2 + 3y2 + 3z2 − 4xy − 2yz + 4zx.
[Jan 2010]
Solution. Since the given Q.F is in 3 variables, the matrix A of the Q.E is a 3 × 3
matrix.

Downloaded From : www.EasyEngineering.net


Downloaded From : www.EasyEngineering.net

Matrices 127

 
a11 a12 a13 
 
∴ A = a21 a22 a23 .
 
 
a31 a32 a33
 
From the given Q.F we have
a11 = Coeff.of x2 = 6.
1 1
a12 = a21 = × Coeff.of xy = × (−4) = −2
2 2
1 1
a13 = a31 = × Coeff.of xz = × 4 = 2
2 2

ww a22 = Coeff.of y2 = 3.

a23 = a32 =
1 1
× Coeff.of yz = × (−2) = −1.

w.E 2
a33 = Coeff.of z2 = 3
 
2

asy
 6 −2 2 
 
∴ A = −2 3 −1.
 
 
2 −1 3
En
 
The principal minors are D1 = 6 > 0.

gin

6 −2
D2 = = 18 − 4 = 14 > 0.
−2 3

ee
D3 = |A| = 6(8) + 2(−4) + 2(−4) = 48 − 8 − 8 = 32 > 0.
Since D1 , D2 , D3 are positive, the Q.F is positive definite.
rin
Example 1.67. Determine λ so that the quadratic form λ(x2 +y2 +z2 )+2xy−2yz+2zxg.n
is positive definite.

et
a11 a12 a13 

Solution. If A is the matrix of the given quadratic form, then A = a21 a22 a23 


 
a31 a32 a33
 

Now, a11 = Coeff.of x2 = λ


1 1
a12 = a21 = × Coeff.of xy = × 2 = 1
2 2
1 1
a13 = a31 = × Coeff.of xz = × 2 = 1
2 2

Downloaded From : www.EasyEngineering.net


Downloaded From : www.EasyEngineering.net

128 Engineering Mathematics - II

a22 = Coeff.of y2 = λ
1 1
a23 = a32 = × Coeff.of yz = × (−2) = −1
2 2
a33 = Coeff.of z2 = λ
 
λ 1 1 
 
∴ A = 1 λ −1.

 
1 −1 λ
 
Let us find out the principal minors D1 , D2 and D3 .

ww D1 = λ.

D2 = λ2 − 1 = (λ + 1)(λ − 1).

w.E D3 = λ(λ2 − 1) − 1(λ + 1) + 1(−1 − λ)

asy
= λ(λ − 1)(λ + 1) − (λ + 1) − (λ + 1) = (λ + 1)(λ2 − λ − 1 − 1)

= (λ + 1)(λ2 − λ − 2) = (λ + 1)(λ − 2)(λ + 1) = (λ + 1)2 (λ − 2).

En
Since the Q.F is positive definite, we have D1 > 0, D2 > 0 and D3 > 0.

ginD1 > 0 =⇒ λ > 0

ee
D2 > 0 =⇒ (λ + 1)(λ − 1) > 0.
rin
=⇒ λ + 1 > 0 and λ − 1 > 0.
g.n
et
=⇒ λ > −1 and λ > 1.

OR

λ + 1 < 0 and λ − 1 < 0

=⇒ λ < −1 and λ < 1.

D3 > 0 =⇒ (λ + 1)2 (λ − 2) > 0

=⇒ λ > 2 [ since (λ + 1)2 is always > 0].

Downloaded From : www.EasyEngineering.net


Downloaded From : www.EasyEngineering.net

Matrices 129

We have to consider the following cases.


case(i). λ > 0, λ > −1, λ > 1, λ > 2.
or
case(ii). λ > 0, λ < −1, λ < 1, λ > 2. In the above cases the value of λ that satisfies
the above conditions is λ > 2.

Example 1.68. Show that the Q.F ax12 − 2bx1 x2 + cx22 is positive definite if a > 0
and ac − b2 > 0.

ww Solution. The given Q.F. is in 2 variables. Hence, the matrix A of the Q.F is a
2 × 2 matrix. 

w.E
a11 a12 
Let A be  .
a21 a22 
Now, a11 = Coeff.of x12 = a.

a12 = a21 =
1
asy 1
× Coeff.of x1 x2 = × (−2b) = −b.

En
2 2
a22 = Coeff.of x22 = c.

gin 
∴ A =  

 a −b
 .

ee
−b c 

We have D1 = a, D2 = ac − b2 .
Given a > 0 and ac − b2 > 0. rin
=⇒ D1 > 0 and D2 > 0.
∴ The Q.F is positive definite. g.n
Example 1.69. Find the index, signature, rank and nature of the quadratic form
in 3 variables x2 + 2y2 − 3z2 .
et[May 2011]
Solution. Since the given Q.F contains only square terms, it is in the canonical
form in 3 variables.
The eigen values are the coefficients of x2 , y2 and z2 .

∴ λ1 = 1, λ2 = 2, λ3 = −3

Downloaded From : www.EasyEngineering.net


Downloaded From : www.EasyEngineering.net

130 Engineering Mathematics - II

No. of positive eigen values = 2.


∴ Index = 2.
No. of negative eigen values = 1.
Signature = Difference of the number of positive and negative eigen values
= 2 − 1 = 1.
Rank = 3 = Number of positive and negative eigen values.
Nature of the Q.F is indefinite.

Example 1.70. If the quadratic form ax2 +2bxy+cy2 is positive definite (or negative

ww definite), then prove that the quadratic equation ax2 + 2bx + c = 0 has imaginary

w.E
roots.
Solution. Since the given Q.F contains two variables, the matrix of the Q.F. A is

asy
a 2 × 2 matrix.
 
a11 a12 
Let A be  .

En
a 21a  22

Now, a11 = Coeff.of x2 = a.

a12 = a21 =
1
2 gin 1
× Coeff.of xy = × 2b = b.
2


a22 = Coeff.of y2 = c.

a b
∴ A = 
b c
.

ee rin
The principal minors are D1 = a, D2 = ac − b2 .
g.n
If the quadratic form is positive definite then D1 > 0 and D2 > 0.

=⇒ a > 0 and ac − b2 > 0. et


=⇒ a > 0 and b2 − ac < 0. (1)

If the quadratic form is negative definite, then (−1)i Di > 0 for all i.

=⇒ −D1 > 0 and D2 > 0

=⇒ −a > 0 and ac − b2 > 0.

Downloaded From : www.EasyEngineering.net


Downloaded From : www.EasyEngineering.net

Matrices 131

=⇒ a < 0 and b2 − ac < 0. (2)

Consider the quadratic equation

ax2 + 2bx + c = 0. (3)

The discriminant of the quadratic equation is

∆ = 4b2 − 4ac = 4(b2 − ac) < 0.

ww ∴ The roots of (3) are imaginary.

Example 1.71. Reduce the quadratic form x12 + 5x22 + x32 + 2x1 x2 + 2x2 x3 + 6x3 x1 to

w.E
canonical form through an orthogonal transformation.
Solution.
[Jan 2006]

asy
Step 1. To form the matrix of the Q.F
Let A be the matrix of the Q.F.

En
Since the given Q.F is in 3 variables, A must be a 3 × 3 matrix.

gin
 
a11 a12 a13 
 
Let A = a21 a22 a23 .
 


a31 a32 a33
Now, a11 = Coeff.of x12 = 1


ee rin
a12 = a21
1
2
1
1
= × Coeff.of x1 x2 = × 2 = 1
2
1 g.n
a13 = a31 = × Coeff.of x1 x3 = × 6 = 3
2
a22 = Coeff.of x22 = 5.
1
2

1
et
a23 = a32 = × Coeff.of x2 x3 = × 2 = 1.
2 2
a33 = Coeff.of x32 = 1.
 
1 1 3
 
∴ A = 1 5 1.

 
3 1 1
 

Downloaded From : www.EasyEngineering.net


Downloaded From : www.EasyEngineering.net

132 Engineering Mathematics - II

Step 2. To find the eigen values


Since A is a 3 × 3 matrix, the characteristic equation is of the form

λ3 − s1 λ2 + s2 λ − s3 = 0.

Now, s1 = sum of the main diagonal elements

= 1 + 5 + 1 = 7.

s2 = sum of the minors of the main diagonal elements.



5 1 1 3 1 1

ww = + +
1 1 3 1 1 5

w.E =5−1+1−9+5−1

= 4 − 8 + 4 = 0.

asy
1 1 3

s3 = |A| = 1 5 1

En

3 1 1

gin
= 1(5 − 1) − 1(1 − 3) + 3(1 − 15)

= 4 + 2 − 42

= −36.
ee
∴ The characteristic equation is λ3 − 7λ2 + 36 = 0.
λ = −2 is a root. rin
By synthetic division, we get
g.n
−2 1
0
1
−7
−2
−9
0
18
18
36
−36
0
et
λ2 − 9λ + 18 = 0.

(λ − 3)(λ − 6) = 0.

The characteristic equation becomes (λ + 2)(λ − 3)(λ − 6) = 0.


The eigen values are −2, 3, 6.

Downloaded From : www.EasyEngineering.net


Downloaded From : www.EasyEngineering.net

Matrices 133

Step 3.To find the eigen vectors


 
 x1 
 
Let X =  x2  be an eigen vector corresponding to the eigen value λ.
 
 
x3
 

∴ (A − λI)X = 0.
  
1 − λ 1 3   x1 
   
5−λ 1   x2  = 0. (1)

 1
  

ww

3 1 1 − λ x3
When λ = −2, (1) becomes

w.E
  
3 1 3  x1 
   
1 7 1  x2  = 0.
   

asy
   
3 1 3 x3
The above two equations are reduced to two equations

En 3x1 + x2 + 3x3 = 0

gin x1 + 7x2 + x3 = 0.

1
ee
Solving the two equations by the rule of cross multiplication we get
x1
3
x2
3
x3
1 rin
7 1 1 7
g.n
x1

x1
=
x2

x2
=
1 − 21 3 − 3 21 − 1
x3
x3 et
= =
−20 0 20
=⇒ x1 = 1, x2 = 0, x3 = −1.
 
 1 
 
∴ X1 =  0  .
 
−1

Downloaded From : www.EasyEngineering.net


Downloaded From : www.EasyEngineering.net

134 Engineering Mathematics - II

  
−2 1 3   x1 
   
When λ = 3, (1) become  1 2 1   x2  = 0.
  
   
3 1 −2 x3
  
The above equations are reduced to

−2x1 + x2 + 3x3 = 0

x1 + 2x2 + x3 = 0

3x1 + x2 − 2x3 = 0.

ww From the first two equations by the rule of cross multiplication we get
x1 x2 x3

w.E 1
2
3
1
−2
1
1
2

asy
En x1
=
1 − 6 3 + 2 −4 − 1
x2
=
x3

gin
x1 x2 x3
= = =⇒ x1 = 1, x2 = −1, x3 = 1.
−5 5 −5
 

ee
 1 
 
∴ X2 = −1 .
 
 
1
 
rin

−5 1
 3
 
  x1 
   g.n
When λ = 6, (1) becomes  1 −1 1   x2  = 0.

et

   
3 1 −5 x3

The above equations are reduced to

−5x1 + x2 + 3x3 = 0

x1 − x2 + x3 = 0

3x1 + x2 − 5x3 = 0.

By the rule of cross multiplication, from the first two equations we get

Downloaded From : www.EasyEngineering.net


Downloaded From : www.EasyEngineering.net

Matrices 135

x1 x2 x3
1 3 −5 1
−1 1 1 −1

x1 x2 x3
= =
1+3 3+5 5−1

ww x1
4
=
x2
8
=
x3
4
=⇒ x1 = 1, x2 = 2, x3 = 1.

w.E  
1
 
X3 = 2 .
 

asy  


1
 

Step 4. To form the modal matrix


En
gin
Since the eigen values are different, the eigen vectors are mutually orthogonal.
     
 √1   √1   √1 
 2   3   6 
  

ee
The normalized eigen vectors are  0 , − √1 ,  √2 .
  
 −1   √
2
3
  
  6 
√1
  
3
The modal matrix N is formed with columns as the normalized eigen vectors.
√1
6
rin
g.n
et
 
 √1 √1 √1 
 2 3 6 
−1 √2 

∴ N =  0 √

 3 6 
 −1
√ √1 √1 
2 3 3

Step 5. To find N T AN
   
 √1 0 − √1  1 1 3  √1 √1 √1 
 2 2     2 3 6 
N T AN =  √1 − √1 1
− √1 √2 
  
 1 5 1  0
√   
 
 3 3 3  3 6 
 √1 2
√ √1
 
3 1 1 − √1
 
√1 √1 
6 6 3 2 3 3

Downloaded From : www.EasyEngineering.net


Downloaded From : www.EasyEngineering.net

136 Engineering Mathematics - II

 √ √   1 
− 2 0 2  √2 √1 √1 
 √ 3 6
√ √  
 
 
=  3 − 3 3  0 − √1 √2 
   
 √ 3 6
√ √  
  √1 1 1


6 2 6 6 −
 √ √
2 3 3
 
−2 0 0
 
=  0 3 0 = D.

 
0 0 6
 
Step 6. Reduction to Canonical form
 
y1 

ww Let X = NY, where Y = y2 .


 
 
 

w.E
y3
 

The given Q.F is Q = X T AX.

asy
= (NY)T A(NY)

= Y T N T ANY

En
= Y T DY

gin
  
−2 0 0 y1 
     
= y1 y2
  
y3  0 3 0 y2 

ee
   
0 0 6 y3
  

= −2y21 + 3y22 + 6y23


rin
which is canonical.

g.n
Example 1.72. Reduce the quadratic form 8x2 + 7y2 + 3z2 − 12xy + 4xz − 8yz to the
canonical form by an orthogonal transformation. Find one set of values of x, y, z
et
(not all zero) which will make the Quadratic form zero. [Jan 2012]
Solution.
Step 1. To find the matrix of the Q.F
Since the given Q.F contains 3 variables, the matrix of the Q.F A is a 3 × 3
matrix.

Downloaded From : www.EasyEngineering.net


Downloaded From : www.EasyEngineering.net

Matrices 137

 
a11 a12 a13 
 
∴ A = a21 a22 a23 
 
 
a31 a32 a33
 

Now, a11 = Coeff.of x12 = 8


1 1
a12 = a21 = × Coeff.of xy = × (−12) = −6
2 2
1 1
a13 = a31 = × Coeff.of xz = × 4 = 2
2 2
a22 = Coeff.of y2 = 7.

ww a23
1 1
= a32 = × Coeff.of yz = × (−8) = −4.
2 2

w.E a33 = Coeff.of z2 = 3.



 8 −6 2 



∴ A = −6 7 −4.
asy

 
2 −4 3
Step 2. To find the eigen values
En
Since A is a 3 × 3 matrix, the characteristic equation is of the form

gin
λ3 − s1 λ2 + s2 λ − s3 = 0.

= 8 + 7 + 3 = 18. ee
Now, s1 = sum of the main diagonal elements

s2 = sum of the minors of the main diagonal elements.rin



=

7 −4 8 2 8 −6
+ +

g.n
et

−4 3 2 3 −6 7

= 21 − 16 + 24 − 4 + 56 − 36

= 5 + 20 + 20 = 45.

8 −6 2

s3 = |A| = −6 7 −4

2 −4 3

= 8(21 − 16) + 6(−18 + 8) + 2(24 − 14)

Downloaded From : www.EasyEngineering.net


Downloaded From : www.EasyEngineering.net

138 Engineering Mathematics - II

= 40 − 60 + 20 = 0.

The characteristic equation is λ3 − 18λ2 + 45λ = 0

λ(λ2 − 18λ + 45) = 0

λ(λ − 3)(λ − 15) = 0

λ = 0, 3, 15.

ww Step 3. To find the eigen vectors


 

w.E
 x1 
 
Let X =  x2  be the eigen vector corresponding to the eigen value λ.
 
 
x3
 

asy ∴ (A − λI)X = 0.

En
8 − λ −6
 
2   x1 

gin
   
 −6 7 − λ −4   x2  = 0. (1)

   
2 −4 3 − λ x3

When λ = 0, (1) becomes


 ee

 
 8 −6 2   x1 
  
rin
  x2  = 0.
g.n
   
 −6 7 −4
   
2 −4 3 x3

The equations become


8x1 − 6x2 + 2x3 = 0
et
−6x1 + 7x2 − 4x3 = 0

2x1 − 4x2 + 3x3 = 0.

Taking the first two equations and applying the rule of cross multiplication, we
obtain

Downloaded From : www.EasyEngineering.net


Downloaded From : www.EasyEngineering.net

Matrices 139

x1 x2 x3
−6 2 8 −6
7 −4 −6 7

x1 x2 x3
= =
24 − 14 −12 + 32 56 − 36
x1 x2 x3
= =
10 20 20
=⇒ x1 = 1, x2 = 2, x3 = 2.

ww  
1
 
∴ X1 = 2 .

w.E   
 
2
 

asy
 5 −6 2   x1 
   
When λ = 3, (1) become −6 4 −4  x2  = 0.
  
   

En
2 −4 0 x3
  
The above equations become

gin 5x1 − 6x2 + 2x3 = 0

ee
−6x1 + 4x2 − 4x3 = 0

2x1 − 4x2 = 0.
rin
From the last equation we get
2x1 = 4x2 g.n
x1
4
=
x2
2
⇒ x1 = 2, x2 = 1.
et
Substituting in the first equation we get
10 − 6 + 2x3 = 0

4 + 2x3 = 0

2x3 = −4

x3 = −2.

Downloaded From : www.EasyEngineering.net


Downloaded From : www.EasyEngineering.net

140 Engineering Mathematics - II

 
 2 
 
∴ X2 =  1  .
 
 
−2
 
  
−7 −6 2   x1 
   
When λ = 15, (1) becomes −6 −8 −4   x2  = 0.
  
   
2 −4 −12 x3
  
The above equations become

ww −7x1 − 6x2 + 2x3 = 0

w.E −6x1 − 8x2 − 4x3 = 0

2x1 − 4x2 − 12x3 = 0.

asy
Taking the first two equations and applying the rule of cross multiplication, we
get

En
gin
x1 x2 x3
−6 2 −7 −6
−8

ee−4 −6 −8

rin
g.n
x1 x2 x3
= =
24 + 16 −12 − 28 56 − 36

et
x1 x2 x3
= = =⇒ x1 = 2, x2 = −2, x3 = 1.
40 −40 20
 
 2 
 
X3 = −2 .
 
1
 

Step 4. To form the modal matrix


Since all the eigen values are different, X1 , X2 , X3 are pairwise orthogonal.
The modal matrix N is formed with the normalized eigen vectors are columns.

Downloaded From : www.EasyEngineering.net


Downloaded From : www.EasyEngineering.net

Matrices 141

     
 1   2   2 
 3   3   3 
The normalized eigen vectors are  23 ,  13 ,  −2 .
     
     3 
 2   −2   1 
3 3 3
 
1 2 2 
1 
 
∴ N = 2 1 −2 .

3  
2 −2 1
 

Step 5. To find N T AN

ww
   
1 2 2   8 −6 2  1 2 2 
1
     
N T AN = 2 1 −2 −6 7 −4 2 1 −2
9 

w.E
    
2 −2 1 2 −4 3 2 −2 1
   
  
 0 0 0  1 2 2 

asy
1 
   
=  6
  
3 −6 2 1 −2
9    
30 −30 15 2 −2 1
  

0 0 0
En  
 0 0 0 

gin
1 
   
= 0 27 0  = 0 3 0  = D.

9   
 
0 0 135 0 0 15
   
Step 6. Reduction to Canonical form
 
y1 
Let X = NY, where Y = y2 .
 
ee rin
 
y3
g.n
The given Q.F is Q = X T AX.

= (NY)T A(NY)
et
= Y T N T ANY

= Y T DY
  
0 0 0  y1 
     
= y1 y2
  
y3 0 3 0  y2 
   
0 0 15 y3
  

Downloaded From : www.EasyEngineering.net


Downloaded From : www.EasyEngineering.net

142 Engineering Mathematics - II

= 3y22 + 15y23 .

which is canonical.
we have
 X = NY   
 x 1 2 2  y1 
  1 
     
y = 2 1 −2 y2 
  3    
z 2 −2 1 y3
1
x = (y1 + 2y2 + 2y3 )
3

ww 1
y = (2y1 + y2 − 2y3 )
3
1

w.E z = (2y1 − 2y2 + y3 ).


3
Equating the quadratic form to zero we get

asy 3y22 + 15y23 = 0.

En
⇒ y2 = 0, y3 = 0, and assume y1 = 3.

gin
Substituting these values in x, y, z we obtain x = 1, y = 2, z = 2.

ee
These set of values will make the quadratic form zero.

Example 1.73. Reduce the quadratic form x12 + 2x22 + x32 − 2x1 x2 + 2x2 x3 to the
rin
canonical form through an orthogonal transformation and hence show that it is

g.n
positive semidefinite. Also give a nonzero set of values (x1 , x2 , x3 ) which makes the
quadratic form to zero.
Solution.
Step 1. To find the matrix of the Q.F
et[Jan 2009]

Since the given Q.F contains 3 variables, the matrix of the Q.F A is a 3 × 3
matrix.  
a11 a12 a13 
 
Let A = a21 a22 a23 

 
a31 a32 a33
 

Downloaded From : www.EasyEngineering.net


Downloaded From : www.EasyEngineering.net

Matrices 143

Now, a11 = Coeff.of x12 = 1


1 1
a12 = a21 = × Coeff.of x1 x2 = × (−2) = −1
2 2
1 1
a13 = a31 = × Coeff.of x1 x3 = × 0 = 0
2 2
a22 = Coeff.of x22 = 2.
1 1
a23 = a32 = × Coeff.of x2 x3 = × 2 = 1
2 2
a33 = Coeff.of x32 = 1.

ww
 
 1 −1 0
 
∴ A = −1 2 1.
 

w.E
 
0 1 1
 
Step 2. To find the eigen values

asy
Since A is a 3 × 3 matrix, the characteristic equation is of the form

λ3 − s1 λ2 + s2 λ − s3 = 0.

En
gin
Now, s1 = sum of the main diagonal elements

= 1 + 2 + 1 = 4.

ee
s2 = sum of the minors of the main diagonal elements.

2 1 1 0 1 −1

rin
=

g.n
+ +
1 1 0 1 −1 2

=2−1+1−0+2−1

= 3.

1 −1 0

et

s3 = |A| = −1 2 1

0 1 1

= 1(2 − 1) + 1(−1 − 0) + 0

= 1 − 1 = 0.

Downloaded From : www.EasyEngineering.net


Downloaded From : www.EasyEngineering.net

144 Engineering Mathematics - II

∴ The characteristic equation is λ3 − 4λ2 + 3λ = 0

λ(λ2 − 4λ + 3) = 0

λ(λ − 1)(λ − 3) = 0

λ = 0, 1, 3.

The eigen values are 0, 1, 3.

ww Step 3. To find the eigen vectors


 
 x1 
 

w.E
Let X =  x2  be the eigen vector corresponding to the eigen value λ.
 
x3
 

asy ∴ (A − λI)X = 0.

En
  
1 − λ −1 0   x1 
   
 −1 2 − λ 1   x2  = 0. (1)
  

gin 
0 1 1 − λ x3
  
 

When λ = 0, (1) becomes


ee
  
 1 −1 0  x1 
   
−1 2 1  x2  = 0.
  
rin

0 1 1 x3
  

g.n
The equations are
x1 − x2 = 0 et
−x1 + 2x2 + x3 = 0

x2 + x3 = 0.

From the first equation we get

x1 = x2 =⇒ x1 = 1, x2 = 1.

Downloaded From : www.EasyEngineering.net


Downloaded From : www.EasyEngineering.net

Matrices 145

From the last equation we get


x2 = −x3
⇒ x3 = −1.
 
 1 
 
∴ X1 =  1 .
 
 
−1
 
When λ = 1, (1) becomes
  
 0 −1 0  x1 
   

ww −1 1 1  x2  = 0.


  
   
0 1 0 x3

w.E
The equations are
x2 = 0
−x1 + x2 + x3 = 0
asy
Substituting x2 = 0, the second equation becomes

En
x1 = x3 ⇒ x1 = 1, x3 = 1.
 
1 gin
ee
 
∴ X2 = 0.

rin
 
1
  

g.n
−2 −1 0   x1 
   
When λ = 3, (1) become −1 −1 1   x2  = 0.
  

et
   
0 1 −2 x3
  
The equations are
2x1 + x2 = 0
x1 + x2 − x3 = 0
x2 − 2x3 = 0.
From the first equation we have
2x1 = −x2
x1 x2
=
−1 2

Downloaded From : www.EasyEngineering.net


Downloaded From : www.EasyEngineering.net

146 Engineering Mathematics - II

⇒ x1 = −1, x2 = 2.
From the last equation we have
x2 = 2x3
x2 x3
=
2 1
=⇒ x2 = 2, x3 = 1.
 
−1
 
∴ X3 =  2 
 
 
1
 

ww Step 4. To find the modal matrix


Since all the eigen values are different, X1 , X2 , X3 are pairwise orthogonal.

w.E
     
 √1   √1   −1
√ 
 3   2   6 
 1     2 
The normalized eigen vectors are  √ ,  0 ,  √ .
 3     6 

asy
   1   −1
√ √ √1
3 2 6
The modal matrix N is formed with the normalized eigen vectors as columns.

En

 √1
 3
√1
2
−1
√ 


6

gin
∴ N =  √1 2   .
 0 √
 3 6 
√1 √1
 −1

ee
3 2 6

Step 5. To find N T AN

 √1 √1 −1
  1
√1 −1 rin

g.n
√    1 −1 0  √3 √  
 3 3 3     2 6 
N T AN =  √1 0 √1 
 
 −1 2 1  √13
 0 √2 

 2 2 6 

et
 
√2 1 √1 √1
 −1
√ √
 
0 1 1  −1
 √ 
6 6 6 3 2 6
  
 0
 0 0   √13 √1
2
−1
√ 
6

  
=  √1 √1    √1 √2 

0 0 
 2 2   3 6 
 −3 √6 3    −1 √1 √1

√ √ √
6 6 6 3 2 6
 
0 0 0
 
= 0 1 0 = D.
 
 
0 0 3
 

Downloaded From : www.EasyEngineering.net


Downloaded From : www.EasyEngineering.net

Matrices 147

Step 6. Reduction to Canonical form


 
y1 
 
Let X = NY, where Y = y2 .
 
 
y3
 

The given Q.F is Q = X T AX.

= (NY)T A(NY)

= Y T N T ANY

ww = Y T DY
  
0 0 0 y1 

w.E
     
= y1 y2
  
y3 0 1 0 y2 
   
0 0 3 y3
  

which is canonical. asy


= y22 + 3y23 .

En
Step 7. To find the nature of the Q.F

gin
Since two eigen values are positive and one eigen value is 0, the given quadratic
form is positive semidefinite.

ee
Step 8. To find the value of x1 , x2 , x3 for which the Q.F = 0
Taking y22 + 3y23 = 0 =⇒ y2 = 0, y3 = 0.
Now X = NY is reduced to the equations. rin
1 1 1
x1 = √ y1 + √ y2 − √ y3 g.n
3

1
2

2
x2 = √ y1 + √ y3
6
et
3 6

−1 1 1
x3 = √ y1 + √ y2 + √ y3 .
3 2 6

Taking y1 = 3, we get x1 = 1, x2 = 1, x3 = −1.
∴ x1 = 1, x2 = 1, x3 = −1 make the given quadratic form zero.

Downloaded From : www.EasyEngineering.net


Downloaded From : www.EasyEngineering.net

148 Engineering Mathematics - II

Example 1.74. Reduce the Q.F 2x12 + 6x22 + 2x32 + 8x1 x3 to canonical form. State the
type of the canonical form. [Jan 2008]
Solution.
Step 1. To find the matrix of the Q.F
Let A be the matrix of the Q.F.
Since the given Q.F contains 3 variables, A must be a 3 × 3 matrix.
 
a11 a12 a13 
 
A = a21 a22 a23 .
 

ww
 
a31 a32 a33
 

Now, a11 = Coeff.of x12 = 2.

w.E a12 = a21 =


1
2
1
× Coeff.of x1 x2 = 0.
1

a22 2 asy
a13 = a31 = × Coeff.of x1 x3 = × 8 = 4
2
= Coeff.of x2 = 6.
2

a23 = a32 =
1
2 En 1
× Coeff.of x2 x3 = × 0 = 0.
2
a33 = Coeff.of x32 = 2.
  gin
ee
2 0 4
 
∴ A = 0 6 0.
 


4 0 2


rin
Step 2. To find the eigen values
Since A is a 3 × 3 matrix, the characteristic equation is of the form g.n
λ3 − s1 λ2 + s2 λ − s3 = 0.
et
Now, s1 = sum of the main diagonal elements

= 2 + 6 + 2 = 10.

s2 = sum of the minors of the main diagonal elements.



6 0 2 4 2 0
= + +
0 2 4 2 0 6

Downloaded From : www.EasyEngineering.net


Downloaded From : www.EasyEngineering.net

Matrices 149

= 12 − 0 + 4 − 16 + 12 − 0

= 12 − 12 + 12 = 12.

2 0 4

s3 = |A| = 0 6 0

4 0 2

= 2(12 − 0) − 0 + 4(0 − 24)

= 24 − 96

ww = −72.

w.E
∴ The characteristic equation is λ3 − 10λ2 + 12λ + 72 = 0

λ = −2 is a root.

asy
By synthetic division we get

En −2 1 −10 12 72

gin 0 −2 24 −72

ee 1 −12 36

λ2 − 12λ + 36 = 0
0

rin
(λ − 6)(λ − 6) = 0
g.n
The eigen values are −2, 6, 6.
λ = 6, 6.
et
Step 3. To find the eigen vectors
 
 x1 
 
Let X =  x2  be the eigen vector corresponding to the eigen value λ.
 
 
x3
 

∴ (A − λI)X = 0.

Downloaded From : www.EasyEngineering.net


Downloaded From : www.EasyEngineering.net

150 Engineering Mathematics - II

  
2 − λ 0 4   x1 
   
6−λ 0   x2  = 0. (1)
  
 0   

4 0 2 − λ x3
 

When λ = −2, (1) becomes


  
4 0 4  x1 
   
0 8 0  x2  = 0.
   
   
4 0 4 x3
The equations are reduced to

ww 4x1 + 4x3 = 0

w.E 8x2 = 0

∴ x2 = 0.

asy and 4x1 = −4x3

En x1
4
=
x3
−4

gin ⇒ x1 = 1, x3 = −1.

ee
 
 1 
 
∴ X1 =  0 .

rin
 
 
−1
 
When λ = 6, (1) becomes
  
−4 0 4   x1 
g.n
et
   
 0 0 0   x  = 0.
   2 
   
4 0 −4 x3
The above equations are reduced to one single equation

−4x1 + 4x3 = 0

i.e., 4x1 = 4x3

x1 = x3 .

Downloaded From : www.EasyEngineering.net


Downloaded From : www.EasyEngineering.net

Matrices 151

⇒ x1 = 1, x3 = 1, x2 = any value. Take x2 = 0.

 
1
 
X2 = 0.
 
 
1
 
 
a
 
Let X3 = b orthogonal to X2 .
 
 
c
 

ww T
∴ X3 X2 = 0
 
 1
 

w.E

a b c 0 = 0
 
 
1
 
a+c=0

asy
X3 also satisfy
(2)

−4a + 4c = 0
i.e., a − c = 0 En (3)
Solving (2) and (3) we get
gin
  Take b = 1.
0
 
ee
a = 0, c = 0 and b =any value.

rin
∴ X3 = 1.
 
0 g.n
Step 4. To find the modal matrix
     
 √1   √1  0
 2   2   
The normalized eigen vectors are  0 ,  0 , 1.
 
et
     
√   √1  0
 −1  
2 2
The modal matrix is formed with the normalised eigen vectors as columns.
 
 √1 1
 2 √2 0

0 1.
 
∴ N =  0
 
 −1
√ √1 0
2 2

Downloaded From : www.EasyEngineering.net


Downloaded From : www.EasyEngineering.net

152 Engineering Mathematics - II

Step 5. To find N T AN
   1 
 √1 0 −1 √  2 0 4  √ 1
0
 2 √2
 

 2 2
 
  
 
N T AN =  √1 0 √1  0 6 0  0
    
0 1

 2 2    
0 1 0 4 0 2 √2 √12
  −1
0
 √ √   1 
− 2 0 2   √ √1 0 
 √  2 2
√   
 
 
=  3 2 0 3 2  0
   0 1
   
0  −1 √1
 √
0 6 0

2 2

ww
 
−2 0 0
 
=  0 6 0 = D.

w.E
 
0 0 6

Step 6. Reduction to Canonical form


 
y1  asy
 
Let X = NY, where Y = y2 .
 
En
gin
 
y3
 

The given Q.F is Q = X T AX.

ee
= (NY)T A(NY)

= Y T N T ANY
rin
= Y DY T

g.n
et
  
−2 0 0 y1 
     
= y1 y2
  
y3  0 6 0 y2 
   
0 0 6 y3
  

= −2y21 + 6y22 + 6y23


which is canonical.
Step 7. To find the nature
Since two of the eigen values are positive and one eigen value is negative,
the given Q.F is indefinite.

Downloaded From : www.EasyEngineering.net


Downloaded From : www.EasyEngineering.net

Matrices 153

Example 1.75. Reduce 6x2 + 3y2 + 3z2 − 4xy − 2yz + 4xz into a canonical form by
an orthogonal reduction and find the rank, signature, index and nature of the
quadratic form. [Jan 2015, Jan 2005]
Solution.
Step 1. To find the matrix of the Q.F
Since the given Q.F contains 3 variables, the matrix of the Q.F A is a 3 × 3 matrix.
 
a11 a12 a13 
 
A = a21 a22 a23 .
 

ww
 
a31 a32 a33
 

Now, a11 = Coeff.of x2 = 6

w.E a12 = a21 =


1
2
1
1
× Coeff.of xy = × (−4) = −2.
2
1
a13 = a31
2
asy
= × Coeff.of xz = × 4 = 2

a22 = Coeff.of y2 = 3.
2

a23 = a32 =
1
En 1
× Coeff.of yz = × (−2) = −1.

gin
2 2
a33 = Coeff.of z2 = 3.
 

ee
 6 −2 2 
 
∴ A = −2 3 −1.
 


2 −1 3


rin
Step 2. To find the eigen values
Since A is a 3 × 3 matrix, the characteristic equation is of the form g.n
λ3 − s1 λ2 + s2 λ − s3 = 0.
et
Now, s1 = sum of the main diagonal elements

= 6 + 3 + 3 = 12.

s2 = sum of the minors of the main diagonal elements.



3 −1 6 2 6 −2
= + +
−1 3 2 3 −2 3

Downloaded From : www.EasyEngineering.net


Downloaded From : www.EasyEngineering.net

154 Engineering Mathematics - II

= 9 − 1 + 18 − 4 + 18 − 4

= 8 + 14 + 14 = 36.

6 −2 2

s3 = |A| = −2 3 −1

2 −1 3

= 6(9 − 1) + 2(−6 + 2) + 2(2 − 6)

= 48 − 8 − 8

ww = 32.

w.E
∴ The characteristic equation is λ3 − 12λ2 + 36λ − 32 = 0.
λ = 2 is a root. By synthetic division we get

asy 2 1
0
−12
2
36
−20
−32
32

En 1 −10 16 0

gin λ2 − 10λ + 16 = 0

ee (λ − 2)(λ − 8) = 0.

The characteristic equation becomes (λ − 2)(λ − 2)(λ − 8) = 0.


rin
The eigen values are 2, 2, 8.
Step 3. To find the eigen vectors g.n
et
 
 x1 
 
Let X =  x2  be an eigen vector corresponding to the eigen value λ.
 
x3
 

∴ (A − λI)X = 0.
  
6 − λ −2 2   x1 
   

−2 3 − λ −1   x  = 0.

   2 
  
2 −1 3 − λ x3

Downloaded From : www.EasyEngineering.net


Downloaded From : www.EasyEngineering.net

Matrices 155


(6 − λ)x1 − 2x2 + 2x3 = 0 






−2x1 + (3 − λ)x2 − x3 = 0  (1)





2x1 − x2 + (3 − λ)x3 = 0. 

When λ = 8 (1) ⇒ −2x1 − 2x2 + 2x3 = 0 ⇒ x1 + x2 − x3 = 0.

−2x1 − 5x2 − x3 = 0 ⇒ 2x1 + 5x2 + x3 = 0.

2x1 − x2 − 5x3 = 0 ⇒ 2x1 − x2 − 5x3 = 0.

ww From the first two equations using the rule of cross multiplication we get
x1
=
x2
=
x3
⇒ x1 = 2, x2 = −1, x3 = 1.

w.E 6 −3 3
 
 2 

asy
 
∴ X1 = −1 .
 
1

En
When λ = 2, (1) is reduced to a single equation, 2x1 − x2 + x3 = 0.

gin
x1 x2
Choose x3 = 0 ⇒ 2x1 − x2 = 0 ⇒ 2x1 = x2 ⇒ = ⇒ x1 = 1, x2 = 2.
1 2

ee
 
1
 
∴ X2 = 2 .

rin
 
 
0
 

 
a g.n
et
 
Let X3 = b be orthogonal to X2 .
 
 
c
 

=⇒ X2T X3 = 0 ⇒ a + 2b = 0. (2)

X3 also satisfy 2a − b + c = 0. (3)


a b
(2) ⇒ a = −2b =⇒ = .
−2 1
b c
(3) ⇒ −4b − b + c = 0 ⇒ 5b = c ⇒ = .
1 5

Downloaded From : www.EasyEngineering.net


Downloaded From : www.EasyEngineering.net

156 Engineering Mathematics - II

a b c
Combining the above two equations we obtain = = .
−2 1 5

Hence, b = 1, c = 5, a = −2.
 
−2
 
∴ X3 =  1  .
 
5

Step 4. To find the modal matrix

ww The normalised eigen vectors are


h 2 −1 1 iT h 1 2 iT h −2
√ √ √ , √ √ 0 , √
6 6 6 5 5

1

5 iT
30 30 30

w.E 
 √2
 6
√1
5
−2 
√ 
30 

asy

∴ N =  −1 √2 √1 


 .
 6 5 30  
 √1 0 √5


En
6 30

Step5. To find N T AN

T

 √2
 6
−1

6
√1
gin
6

  6 −2 2   √2
 

  6
 
√1
5
−2 
√ 
30 

ee
1 2 2 √1 
0  −2 3 −1  −1
  
N AN =  √ 
 √  √ √

 5 5     6 5 30  
 √−2 1 √5
 2 −1 3  √1  5 

rin
0

  √
30 30 30 6 30
  
 √ 16 −8
√ √8    √2 √1 −2 

g.n

 6 6 6    6 5 30  
=  √2 −8
0   −1 √2 √1 

√ √
  
 5 5   6 5 30 

et

√2 √10 √1 √5
 √−4   0
30 30 30 6 30
 
8 0 0
 
= 0 2 0 = D.

 
0 0 2
 
Step 6. Reduction to canonical form
 
y1 
 
Let X = NY, where Y = y2 
 
y3

Downloaded From : www.EasyEngineering.net


Downloaded From : www.EasyEngineering.net

Matrices 157

The given Q.F. is Q = X T AX

= (NY)T ANY

= Y T N T ANY

= Y T DY
  
8 0 0 y1 
   
T
Y DY = [y1 y2 y3 ] 0 2 0 y2 
  
   
0 0 2 y3
  

ww  

w.E
8y1 
 
= [y1 y2 y3 ] 2y2 
 
 
2y3
 

asy
En = 8y21 + 2y22 + 2y23

which is the canonical form.


Rank of the Q.F = 3. gin
Index = 3.
Signature = 3.
ee rin
Since all the eigen values are positive, the Q.F is positive definite.

g.n
Example 1.76. Reduce the quadratic form 2x1 x2 + 2x2 x3 + 2x3 x1 in to canonical
form. Discuss also its nature.
Solution.
et[Jan 2013].

Step 1. To find the matrix of the quadratic form


Since given Q.F. contains 3 variables, the matrix of the Q.F. A is a 3 × 3 matrix.
 
a11 a12 a13 
 
A = a21 a22 a23 .
 
a31 a32 a33
 

Downloaded From : www.EasyEngineering.net


Downloaded From : www.EasyEngineering.net

2 ANNA UNIVERSITY SOLVED


QUESTION PAPERS

ww
w.E
2.1 Unit I MATRICES

a syE
2.1.1 APRIL/ MAY 2015 [R 2013]

Part A
ngi 
 8 −6 2 

nee 
1. Find the sum and product of all the eigen values of −6 7 −4.



rin 2 −4 3

g.n
Solution. Sum of the eigen values = trace of A
= 8 + 7 + 3 = 18.
et
Product of the eigen values = |A|

8 −6 2

= −6 7 −4

2 −4 3

= 8(21 − 16) + 6(−18 + 8) + 2(24 − 14)


= 40 − 60 + 20 = 0.

Downloaded From : www.EasyEngineering.net


37
38
Downloaded Supplement to Engineering Mathematics - II
From : www.EasyEngineering.net

2. Give the nature of the quadratic form whose matrix is


 
−1 0 0 
 
 
 0 −1 0 .
 
0 0 −2
Solution. The given matrix is a diagonal matrix whose elements
along the leading diagonal are the eigen values. We notice that all
the eigen values are negative. Hence, the given quadratic form is
negative definite.
Part B  
 6 −2 2 
ww 
11. (a) (i) Find the eigen values and eigen vectors of −2 3


−1.


w.E 
 6 −2 2 

2 −1 3

a 

syE
Solution. Let A = −2 3 −1.

2 −1 3




Step 1. To find the eigen values


ngi
nee
s1 = tr(A) = 6 + 3 + 3 = 12. rin
s2 = sum of the minors of the main diogonal elements. g.n

=

3 −1 6 2 6 −2
+ +


−1 3 2 3 −2 3
et
= 9 − 1 + 18 − 4 + 18 − 4
= 8 + 14 + 14 = 36.

6 −2 2

s3 = |A| = −2 3 −1

2 −1 3

= 6(8) + 2(−4) + 2(−4)


= 48 − 8 − 8 = 32.

Downloaded From : www.EasyEngineering.net


———-
Anna University
Downloaded Solved Question Papers
From : www.EasyEngineering.net 39

The characteristic equation is

λ3 − s1 λ2 + s2 λ − s3 = 0

i.e., λ3 − 12λ2 + 36λ − 32 = 0.

λ = 2 is a root.

By synthetic division we have

2 1 −12 36 −32

ww 0
1
2
−10
−20
16
32
0

w.E
Hence, the characteristic equation becomes

a syE
(λ − 2)(λ2 − 10λ + 16) = 0

ngi
(λ − 2)(λ − 2)(λ − 8) = 0

λ = 2, 2, 8.
nee
∴ The eigen values are λ1 = 2, λ2 = 2, λ3 = 8.
rin
Step 2. To find the eigen vectors
 
 x1  g.n
 
Let X =  x2  be the eigen vector corresponding to an eigen value λ.
 
x3
et
∴ (A − λI)X = 0.
  
6 − λ 2   x1 
−2
   

3 − λ −1   x2  = 0.
 −2
   
−1 3 − λ x3
2

(6 − λ)x1 − 2x2 + 2x3 = 0  





−2x1 + (3 − λ)x2 − x3 = 0   . (A)




2x1 − x2 + (3 − λ)x3 = 0 

Downloaded From : www.EasyEngineering.net


———-
40
Downloaded Supplement to Engineering Mathematics - II
From : www.EasyEngineering.net

case(i) when λ = 2, (A) reduces to

4x1 − 2x2 + 2x3 = 0


−2x1 + x2 − x3 = 0
2x1 − x2 + x3 = 0.

The above three equations are reduced to the single equation


2x1 − x2 + x3 = 0.
Since λ = 2 is a repeated root, we have to find two eigen vectors by
assigning a particular value to two variables, one at a time.
ww
Assigning x3 = 0, we obtain

w.E 2x1 = x2

a syE
x1
1
=
 
x2
2

ngi1
 
∴ X1 = 2 .
 
0 nee
Assigning x2 = 0, we obtain
rin
2x1 = −x3 g.n
x1
−1
= .
x3
2
et
 
−1
 
∴ X2 =  0  .
 
2
case(ii) when λ = 8, (A) reduces to

−2x1 − 2x2 + 2x3 = 0 (1)

−2x1 − 5x2 − x3 = 0 (2)

2x1 − x2 − 5x3 = 0. (3)

Downloaded From : www.EasyEngineering.net


———-
Anna University
Downloaded Solved Question Papers
From : www.EasyEngineering.net 41

Since all the three equations are different, we can consider the the
first two equations and solve for x1 , x2 , &, x3 by the method of cross
multiplication
x1 x2 x3
-2 2 -2 -2

-5 -1 -2 -5

x1 x2 x3
= =
2 + 10 −4 − 2 10 − 4
ww x1
=
12 −6
x2
=
x3
6

w.E x1
2
=
x2
−1
=
x3
1

a ∴ x1 = 2, x2 = −1, x3 = 1.
syEx1 , x2 , x3 satisfy (3).

ngi
 
 2 
 
∴ X3 = −1 .
  nee
1
rin

 3 −1 1 


 g.n
11. (a) (ii) If A = −1 5 −1, verify Cayley-Hamilton theorem

1 −1 3

et
and hence find A−1 .
Solution.
Step 1. To find the characteristic equation
Since A is a 3 × 3 matrix, the characteristic equation is of the form
λ3 − s1 λ2 + s2 λ − s3 = 0. (1)

Now, s1 = sum of the diagonal elements


= 3 + 5 + 3 = 11.
s2 = sum of the minors of the elements of the main diagonal

Downloaded From : www.EasyEngineering.net


———-
42
Downloaded Supplement to Engineering Mathematics - II
From : www.EasyEngineering.net

5 −1 3 1 3 −1
= + +
−1 3 1 3 −1 5

= 15 − 1 + 9 − 1 + 15 − 1 = 14 + 8 + 14 = 36.

3 −1 1

s3 = |A| −1 5 −1 = 3(15 − 1) + 1(−3 + 1) + 1(1 − 5) = 3 × 14 − 2 − 4

1 −1 3
= 42 − 6 = 36.
The characteristic equation is λ3 − 11λ2 + 36λ − 36 = 0.

ww
Step 2. Verification of Cayley Hamilton theorem.
 
 3

A = −1 w.E
−1 1 
5 −1


A2 = A · A

1
a
−1 3


syE
 
 3 −1 1   3 −1 1 

ngi
  
= −1 5 −1 −1 5 −1
  

 nee

1 −1 3 1 −1 3
  rin 
 9 + 1 + 1 −3 − 5 − 1 3 + 1 + 3   11
  
= −3 − 5 − 1 1 + 25 + 1 −1 − 5 − 3 = −9
−9 7 
 g.n

3 + 1 + 3 −1 − 5 − 3 1 + 1 + 9
 
7
27 −9
−9 11

et
A3 = A2 × A
  
 11 −9 7   3 −1 1 
   
= −9 27 −9 −1 5 −1
   
7 −9 11 1 −1 3
   
 33 + 9 + 7 −11 − 45 − 7 11 + 9 + 21   49 −63 41 
   
= −27 − 27 − 9 9 + 135 + 9 −9 − 27 − 27 = −63 153 −63
   
21 + 9 + 11 −7 − 45 − 11 7 + 9 + 33 41 −63 49

A3 − 11A2 + 36A − 36I

Downloaded From : www.EasyEngineering.net


———-
Anna University
Downloaded Solved Question Papers
From : www.EasyEngineering.net 43

     
 49 −63 41   121 −99 77   108 −36 36 
     
= −63 153 −63 − −99 297 −99 + −36 180 −36
     
41 −63 49 77 −99 121 36 −36 108
   
36 0 0  0 0 0
   
−  0 36 0  = 0 0 0 = 0.
   
0 0 36 0 0 0

∴ Cayley Hamilton theorem is verified.


Step 3. To find A−1

ww
By Cayley Hamilton theorem, we have A3 − 11A2 + 36A − 36I = 0.
Multiplying by A−1 we get,

w.E
A2 − 11A + 36I − 36A−1 = 0.

a
⇒ 36A−1 = A2 − 11A + 36I.
1 2 syE
⇒ A−1 =
36
[A − 11A + 36I].
 ngi
 
 11 −9 7   33 −11 11  36 0
  
0 
∴ A−1 =
1   
nee  
−9 27 −9 − −11 55 −11 +  0 36

0  .
36 
7 −9 11
 
11 −11 33
rin  
0 0 36


1 

11 − 33 + 36 −9 + 11 + 0 7 − 11 + 0 

 g.n
=
36 
 −9 + 11 + 0 27 − 55 + 36 −9 + 11 + 0 
7 − 11 + 0 −9 + 11 + 0 11 − 33 + 36
 et
 
 14 2 −4
1  
A−1 =  2 8 2  .
36  
−4 2 14

11. (b) Reduce the quadratic form x2 + 5y2 + z2 + 2xy + 2yz + 6zx into
canonical form and hence find its rank.
Solution.
Step 1. To form the matrix of the Q.F
Let A be the matrix of the Q.F.

Downloaded From : www.EasyEngineering.net


———-
44
Downloaded Supplement to Engineering Mathematics - II
From : www.EasyEngineering.net

Since the given Q.F is in 3 variables, A must be a 3 × 3


matrix.  
a11 a12 a13 
 
Let A = a21 a22 a23 .
 
a31 a32 a33
Now, a11 = Coeff.of x2 = 1
1 1
a12 = a21 = × Coeff.of xy = × 2 = 1
2 2
1 1
a13 = a31 = × Coeff.of xz = × 6 = 3
2 2

ww a22 = Coeff.of y2 = 5.
1 1
w.E a23 = a32 =
2
× Coeff.of yz = × 2 = 1.

a33 = Coeff.of z2 = 1.
2


a
1 1 3

∴ A = 1 5 1.

 syE

3 1 1

ngi
Step 2. To find the eigen values
nee
rin
Since A is a 3 × 3 matrix, the characteristic equation is of the form
λ3 − s1 λ2 + s2 λ − s3 = 0.
g.n
Now, s1 = sum of the main diagonal elements et
= 1 + 5 + 1 = 7.
s2 = sum of the minors of the main diagonal elements.

5 1 1 3 1 1
= + +
1 1 3 1 1 5

= 5 − 1 + 1 − 9 + 5 − 1 = 4 − 8 + 4 = 0.

1 1 3

s3 = |A| = 1 5 1

3 1 1

Downloaded From : www.EasyEngineering.net


———-
Anna University
Downloaded Solved Question Papers
From : www.EasyEngineering.net 45

= 1(5 − 1) − 1(1 − 3) + 3(1 − 15) = 4 + 2 − 42 = −36.

∴ The characteristic equation is λ3 − 7λ2 + 36 = 0.


λ = −2 is a root.
By synthetic division, we get

−2 1 −7 0 36
0 −2 18 −36
1 −9 18 0

ww λ2 − 9λ + 18 = 0.

w.E (λ − 3)(λ − 6) = 0.

The characteristic equation becomes (λ + 2)(λ − 3)(λ − 6) = 0.

a syE
The eigen values are −2, 3, 6.
Step 3.To find the eigen vectors
 
 x1 
  ngi
nee
Let X =  x2  be an eigen vector corresponding to the eigen value λ.
 
x3
rin

∴ (A − λI)X = 0.
  g.n
1 − λ


 1 5−λ
13   x1 
  
1   x2  = 0.
  
et
(1)

3 1 1 − λ x3
When λ = −2, (1) becomes
  
3 1 3  x1 
   
1 7 1  x2  = 0.
   

3 1 3 x3

The above two equations are reduced to two equations

3x1 + x2 + 3x3 = 0

Downloaded From : www.EasyEngineering.net


———-
46
Downloaded Supplement to Engineering Mathematics - II
From : www.EasyEngineering.net

x1 + 7x2 + x3 = 0.

Solving the two equations by the rule of cross multiplication we get

x1 x2 x3
1 3 3 1

7 1 1 7

x1 x2 x3
= =
1 − 21 3 − 3 21 − 1
x1 x2 x3

ww −20
=
0
=
20

w.E =⇒ x1 = 1, x2 = 0, x3 = −1.
 
 1 

a syE
 
∴ X1 =  0  .
 
−1
 ngi
 
−2 1 3   x1 
   
nee
When λ = 3, (1) become  1 2 1   x2  = 0.
   
3 1 −2 x3
rin
The above equations are reduced to
g.n
−2x1 + x2 + 3x3 = 0
et
x1 + 2x2 + x3 = 0

3x1 + x2 − 2x3 = 0.

From the first two equations by the rule of cross multiplication we


get
x1 x2 x3
1 3 -2 1

2 1 1 2

Downloaded From : www.EasyEngineering.net


———-
Anna University
Downloaded Solved Question Papers
From : www.EasyEngineering.net 47

x1 x2 x3
= =
1 − 6 3 + 2 −4 − 1
x1 x2 x3
= = =⇒ x1 = 1, x2 = −1, x3 = 1.
−5 5 −5
 
 1 
 
∴ X2 = −1 .
 
1
  
−5 1 3   x1 
   
When λ = 6, (1) becomes  1 −1 1   x2  = 0.
   

ww
The above equations are reduced to
3 1 −5 x3

w.E −5x1 + x2 + 3x3 = 0

a syE x1 − x2 + x3 = 0

ngi
3x1 + x2 − 5x3 = 0.

nee
By the rule of cross multiplication, from the first two equations we
get
x1 x2 x3rin
1 3 -5 1 g.n
-1 1 1 -1 et
x1 x2 x3
= =
1+3 3+5 5−1
x1 x2 x3
= = =⇒ x1 = 1, x2 = 2, x3 = 1.
4 8 4
 
1
 
X3 = 2 .
 
1
Step 4. To form the modal matrix
Since the eigen values are different, the eigen vectors are mutually

Downloaded From : www.EasyEngineering.net


———-
48
Downloaded Supplement to Engineering Mathematics - II
From : www.EasyEngineering.net

orthogonal.
     
 √1   √1   √1 
 2   3   6 
The normalized eigen vectors are  0 , − √1 ,  √2 .
   3   6 
 −1

  √1   √1 
2 3 6
The modal matrix N is formed with columns as the normalized
eigen vectors.

 
 √1 √1 √1 
 2 3 6 
∴ N =  0 −1
√ √2  
 3 6 
 −1 √1 √1

ww
Step 5. To find N AN T

2 3 3

w.E    
a  √1
 2
N T AN =  √1
 3
0
syE − √13
− √12  1 1 3  √12
√1 
3 
 

 
 
 1 5 1  0


  −
√1
3
√1
3
√1 
6
√ 
2
6




 3 1 1 − √1
 √1
 √
6
√2
6
√1
3
ngi
√  1
2

√1
3
√1
3

=  3 − 3
− 2
 √


0
√ √ 



3  0
nee
2  √2


√1
3
√1 

− √3 √6 
1 2
6


 √
6 2 6
√ √   1
6 −√ rin √1 √1



−2 0 0

2 3 3
g.n

=  0 3 0 = D.

0 0 6

 et
Step 6. Reduction to Canonical form
 
y1 
 
Let X = NY, where Y = y2 .
 
y3

The given Q.F is Q = X T AX.


= (NY)T A(NY)
= Y T N T ANY

Downloaded From : www.EasyEngineering.net


———-
Anna University
Downloaded Solved Question Papers
From : www.EasyEngineering.net 49

= Y T DY
  
 0 0 y1 
  −2   

= y1 y2 y3  0 3 0 y2 
   
0 0 6 y3

= −2y21 + 3y22 + 6y23

which is canonical.
Since two of the eigen values are positive and one eigen value is
negative, the rank = 3.

ww
w.E
2.1.2 NOV/DEC 2014 [R 2013]

Part A
a syE
1.If 2, 3 are the eigen values of 0

2


0 1

2 0, then find the value of b.

bngi 0 2


2 0 1

Solution. Let A = 0 2 0.

 nee
 
rin
b 0 2
Let λ1 , λ2 , λ3 be the eigen values of A. g.n
We know that, λ1 + λ2 + λ3 = tr(A) = 6.
et
2 + 3 + λ3 = 6

λ3 = 1.

Also, product of the eigen values = |A|.



2 0 1

λ1 λ2 λ3 = 0 2 0

b 0 2

2.3.1 = 2(4) − 0(0) + 1(0 − 2b)

Downloaded From : www.EasyEngineering.net


———-
50
Downloaded Supplement to Engineering Mathematics - II
From : www.EasyEngineering.net

6 = 8 − 2b
2b = 2
b = 1.

2. State Cayley-Hamilton theorem.


Solution. Every square matrix satisfies its own characteristic
equation.
Part B
11. (a) (i) Using Cayley-Hamilton theorem find A4 for the matrix
ww  
 2 −1 2 
 

w.E
A = −1 2 −1.

1 −1 2


Solution.
a syE
Step 1. To find the characteristic equation

ngi
Since A is a 3 × 3 matrix, the characteristic equation is of the form

nee
λ3 − s1 λ2 + s2 λ − s3 = 0. (1)

Now, s1 = sum of the main diagonal elements rin


= 2 + 2 + 2 = 6. g.n
s2 = sum of the minors of the elements of the main diagonal

2 −1 2 1 2 −1
et
= + +
−1 2 1 2 −1 2

=4−1+4−1+4−1
= 3 + 3 + 3 = 9.

2 −1 1

s3 = |A| = −1 2 −1

1 −1 2

= 2(3) + 1(−1) + 1(−1) = 6 − 1 − 1 = 4.

Downloaded From : www.EasyEngineering.net


———-
Anna University
Downloaded Solved Question Papers
From : www.EasyEngineering.net 51

The characteristic equation is λ3 − 6λ2 + 9λ − 4 = 0.


Step 2. Verification of Cayley Hamilton theorem.
 
 2 −1 1 
 
A = −1 2 −1
 
1 −1 2
  
 2 −1 1   2 −1 1 
   
A2 = A · A = −1 2 −1 −1 2 −1
   
1 −1 2 1 −1 2
   
ww  4 + 1 + 1 −2 − 2 − 1 2 + 1 + 2   6 −5

= −2 − 2 − 1 1 + 4 + 1 −1 − 2 − 2 = −5 6
  5 

−5
w.E 
2 + 1 + 2 −1 − 2 − 2 1 + 1 + 4
 
5 −5 6


a
A3 = A2 × A
 syE 
 6 −5 5   2 −1 1 

  
= −5 6 −5 −1 2 −1
   ngi 



5 −5 6 1 −1 2
 
nee 
 12 + 5 + 5 −6 − 10 − 5 6 + 5 + 10   22
  
= −10 − 6 − 5 5 + 12 + 5 −5 − 6 − 10 = −21
rin −21 21 


10 + 5 + 6 −5 − 10 − 6 5 + 5 + 12
 
21 g.n 22
−21
−21
22


A3 − 6A2 + 9A − 4I
     
et
 22 −21 21   6 −5 5   2 −1 1 
     
= −21 22 −21 − 6 −5 6 −5 + 9 −1
   
2 −1
     
21 −21 22 5 −5 6 1 −1 2
   
1 0 0 0 0 0
   
− 4 0 1 0 = 0 0 0 = 0.
   
0 0 1 0 0 0

∴ Cayley Hamilton theorem is verified.

Downloaded From : www.EasyEngineering.net


———-
52
Downloaded Supplement to Engineering Mathematics - II
From : www.EasyEngineering.net

Step 3. To find A4
Consider λ4 . Dividing λ4 by λ3 − 6λ2 + 9λ − 4 we get,
λ4 = (λ + 6)(λ3 − 6λ2 + 9λ − 4) + 27λ2 − 50λ + 24.
Replacing λ by A we get,

A4 = (A + 6I)(A3 − 6A2 + 9A − 4I) + 27A2 − 50A + 24I.


= 27A2 − 50A + 24I [ By Cayley Hamilton theorem].
 
 86 −85 85 
 
= −85 86 −85 .
 
ww 85 −85 86

w.E 
 7 −2

11. (a) (ii) Find the eigenvalues and eigen vectors of −2 6

0 

−2.
a  syE
 7 −2 0 


0 −2 5



Solution. Given A = −2 6 −2.ngi


0 −2 5

nee
Step 1. To find the eigen values
rin
Since A is a 3 × 3 matrix, the characteristic equation takes the form
λ3 − s1 λ2 + s2 λ − s3 = 0. g.n
Now, s1 =sum of the main diagonal elements
= 7 + 6 + 5 = 18
et
s2 = sum of the minors of the main diagonal elements

6 −2 7 0 7 −2
= + +
−2 5 0 5 −2 6

= (30 − 4) + (35 − 0) + (42 − 4) = 99.



7 −2 0

s3 = |A| = −2 6 −2

0 −2 5

= 7(30 − 4) + 2(−10 − 0) + 0 = 182 − 20 = 162.

Downloaded From : www.EasyEngineering.net


———-
Anna University
Downloaded Solved Question Papers
From : www.EasyEngineering.net 53

∴ The characteristic equation is λ3 − 18λ2 + 99λ − 162 = 0.


λ = 3 is a root.
By synthetic division we get

3 1 −18 99 −162
0 3 −45 162
1 −15 54 0

The remaining factor is

ww λ2 − 15λ + 54 = 0
(λ − 6)(λ − 9) = 0

w.Eλ = 6, 9.
∴ The eigen values are 3, 6, 9.
a
Step 3. To find the eigen vectors
 
 x1 
syE
 
ngi
Let X =  x2  be an eigen vector corresponding to the eigen value λ.
 
x3 nee
∴ (A − λI)X = 0.
rin

7 − λ

−2
 
0   x1 
   g.n
 −2


0
6 − λ −2   x2  = 0.
−2 5 − λ x3
   et
(1)

When λ = 3 (1) becomes


  
 4 −2 0   x1 
   
    = 0.
 −2 3 −2   x2 
   
0 −2 2 x3
The equations become
4x1 − 2x2 = 0 ⇒ 2x1 − x2 = 0 (2)
−2x1 + 3x2 − 2x3 = 0 (3)
−2x2 + 2x3 = 0 ⇒ −x2 + x3 = 0. (4)
From (2) and (4) we get 2x1 = x2 = x3 .

Downloaded From : www.EasyEngineering.net


———-
54
Downloaded Supplement to Engineering Mathematics - II
From : www.EasyEngineering.net

If we set x1 = 1, then x2 = 2, x3 = 2.
 
1
 
∴ One eigen vector is X1 = 2 .
 
2
When λ = 6 (1) becomes
  
 1 −2 0   x1 
   
  
−2 0 −2  x2  = 0.
   
0 −2 −1 x3
The equations become
x1 − 2x2 = 0 ⇒ x1 = 2x2 (5)

ww −2x1 − 2x3 = 0 ⇒ x1 = x3 (6)

w.E−2x1 − 2x3 = 0 ⇒ x3 = −2x2 .


From (5), (6) and (7) we obtain x1 = 2x2 = −x3 .
(7)

a
When x2 = 1, x1 = 2, x3 = −2
syE  
 2 
 
∴ The second eigen vector is X2 =  1 .
ngi
 

When λ = 9 (1) becomes


  
−2
nee
−2 −2 0   x1 


  
   = 0. rin


−2 −3 −2
0 −2 −4 x3
  x2 
  g.n
The equations become
−2x1 − 2x2 = 0 ⇒ x1 = −x2
et
(8)
−2x1 − 3x2 − 2x3 = 0 (9)
−2x2 − 4x3 = 0 ⇒ x2 = −2x3 . (10)
From (8), (9) and (10) we obtain x1 = −x2 = 2x3 .
Let x3 = 1. Then x1 = 2, x2 = −2
 
 2 
 
∴ The third eigen vector is X2 = −2.
 
1

Downloaded From : www.EasyEngineering.net


———-
Anna University
Downloaded Solved Question Papers
From : www.EasyEngineering.net 55

11. (b) (i) Reduce the quadratic form 3x2 + 5y2 + 3z2 − 2yz + 2zx − 2xy
to the canonical form through orthogonal transformation.
Solution.
Step 1. To find the matrix of the Q.F
Since the given Q.F contains 3 variables, the matrix A of the Q.F is
a 3 × 3 matrix.  
a11 a12 a13 
 
Let A = a21 a22 a23 .
 
a31 a32 a33

ww Now, a11 = Coeff.of x2 = 3.

w.E a12 = a21 =


1
2
1
× Coeff.of xy = × (−2) = −1.
2

a a13 = a31
1

syE
1
= × Coeff.of xz = × 2 = 1.
2
a22 = Coeff.of y2 = 5.
2

a23 = a32 =
1 ngi 1
× Coeff.of yz = × (−2) = −1.
2
a33 = Coeff.of z2 = 3.
2
nee
  rin
 3 −1 1 

∴ A = −1 5 −1.

g.n

1 −1 3
Step 2. To find the eigen values

et
Since A is a 3 × 3 matrix, the characteristic equation is of the form
λ3 − s1 λ2 + s2 λ − s3 = 0, where

s1 = sum of the main diagonal elements = 3 + 5 + 3 = 11.


s2 = sum of the minors of the main diagonal elements.

5 −1 3 1 3 −1
= + +
−1 3 1 3 −1 5

= 14 + 8 + 14 = 36.

Downloaded From : www.EasyEngineering.net


———-
56
Downloaded Supplement to Engineering Mathematics - II
From : www.EasyEngineering.net


3 −1 1

s3 = |A| = −1 5 −1

1 −1 3

= 3(15 − 1) + 1(−3 + 1) + 1(1 − 5)


= 3 × 14 − 2 − 4 = 42 − 6 = 36.

∴ The characteristic equation is λ3 − 11λ2 + 36λ − 36 = 0.


λ = 2 is a root.
By synthetic division we get

ww 2 1 −11 36 −36

w.E 0
1
2
−9
−18
18
36
0

aλ2 − 9λ + 18 = 0
syE
(λ − 3)(λ − 6) = 0 ngi
λ = 3, 6.
nee
∴ The eigen values are 2, 3, 6.
Step 3. To find the eigen vectors rin
 
 x1  g.n
 
Let X =  x2  be an eigen vector corresponding to the eigen value λ.
 
x3
et
∴ (A − λI)X = 0.
  
3 − λ −1 1   x1 
   

 −1 5 − λ −1   x2  = 0. (1)
   
1 −1 3 − λ x3
When λ = 2 (1) becomes
  
 1 −1 1   x1 
   
    = 0.
 −1 3 −1   x2 
   
1 −1 1 x3

Downloaded From : www.EasyEngineering.net


———-
Anna University
Downloaded Solved Question Papers
From : www.EasyEngineering.net 57

The equations become


x1 − x2 + x3 = 0
−x1 + 3x2 − x3 = 0
x1 − x2 + x3 = 0.
From the last two equations, using the rule of cross multiplication
we get

ww
w.E
a x1
=

=
syE
x2
x2

=
=
3 − 1 −1 + 1 1 − 3
x1 x3

x3

x1
=
x2
=
x3
2 0 −2 1
ngi
0
⇒ x1 = 1, x2 = 0, x3 = −1.
−1

nee
 
 1 
  rin
∴ X1 =  0  .
  g.n
−1
et
When λ = 3 (1) becomes
  
 0 −1 1   x1 
   
    = 0.
−1 2 −1   x2 
   
1 −1 0 x3
The equations become
−x2 + x3 = 0
−x1 + 2x2 − x3 = 0
x1 − x2 = 0.
From the first and the last equations we get

Downloaded From : www.EasyEngineering.net


———-
58
Downloaded Supplement to Engineering Mathematics - II
From : www.EasyEngineering.net

x1 = x2 = x3 = 1(say)
 
1
 
∴ X2 = 1 .
 
1

When λ = 6 (1) becomes


  
−3 −1 1   x1 
   
    = 0.
 −1 −1 −1   x2 
  
1 −1 −3 x3
The equations become

ww −3x1 − x2 + x3 = 0
−x1 − x2 − x3 = 0

w.Ex1 − x2 − 3x3 = 0.

we get a
From the last two equations using the rule of cross multiplication,

syE
ngi
nee
rin
x1
=
x2
=
3 − 1 −1 − 3 1 + 1
x3
g.n
x1
2
=
x2
−4
=
x3
2
⇒⇒
x1
1
=
x2
−2
=
x3
1
et
⇒ x1 = 1, x2 = −2, x3 = 1.
 
 1 
 
∴ X3 = −2 .
 
1
     
 1  1  1 
     
∴ The eigen vectors are  0  , 1 , −2.
     
−1 1 1
Step 4. To find the modal matrix

Downloaded From : www.EasyEngineering.net


———-
Anna University
Downloaded Solved Question Papers
From : www.EasyEngineering.net 59

The normalised eigen vectors are


h 1 −1 iT h 1 1 1 iT h 1 −2 1 iT
√ 0 √ , √ √ √ , √ √ √ .
2 2 3 3 3 6 6 6
 
 √1 √1 √1 

 2 3 6
∴ N =  0  √1 √ 
−2  .
 3 6 
 −1
√ √1 √1
2 3 6
T
Step5. To find N AN
     
 √1 √ 
−1
  3 −1 1   √1 √1 √1  2 0 0
 2 0 

2 
  
  2 3 6

  
  −1 5 −1  0  = 0 3 0 = D.
N T AN =  √1 √1 √1  √1 √ 
−2
      
ww  3
 √1
6
−2

3

6
√1
3

6
1 −1 3  −1


2
√1
3

3
√1
6

6
0 0 6

w.E
Step 6. Reduction to canonical form
 
y1 
 
a
Let X = NY, where Y = y2 .
syE  
y3
The given Q.F. is Q = X AX = (NY)T ANY = Y T N T ANY = Y T DY

T

 ngi  
2 0 0 y1 
    nee 2y1 
 
Y DY = [y1 y2 y3 ] 0 3 0 y2  = [y1 y2 y3 ] 3y2  = 2y21 + 3y22 + 6y23 ,
T

0 0 6 y3
  
rin  
6y3

which is in the canonical form. g.n


11. (b) (ii) If β is a eigenvalue of a matrix, then prove that
eigen value of A . −1
1
β
is the et
Solution. Given β is a non zero eigen value of A. Then, there exists
a non zero column matrix X such that

AX = βX. (1)

Since all the eigen values are non zero, A is non-singular. Hence, A−1
exists.
Premultiplying both sides of (1) by A−1 we get,

A−1 (AX) = A−1 βX

Downloaded From : www.EasyEngineering.net


———-
60
Downloaded Supplement to Engineering Mathematics - II
From : www.EasyEngineering.net

(A−1 A)X = β(A−1 X)

IX = βA−1 X
1
X = A−1 X.
β
1
⇒ is an eigen value of A−1 .
β

2.1.3 MAY/JUNE 2014 [R 2013]

ww
Part A 
2 1


0

w.E
1. Find the Eigen values of the inverse of the matrix A = 0 3

0 0
4.
4


a
Solution. A is a triangular matrix.
syE
The eigen values of A are 2, 3, 4.
1 1 1
∴ Eigen values of A−1 are , , .
2 3 4 ngi
nee
2. If 2, −1, −3 are the Eigen values of the matrix A, then find the
Eigen values of the matrix A2 − 2I.
rin
Solution. If λ1 , λ2 , λ3 are the eigen values of A, then the eigen
values of g.n
A2 − 2I are λ21 − 2, λ22 − 2, λ23 − 2.
Here λ1 = 2, λ2 = −1, λ3 = −3.
et
∴ The required eigen values are 22 − 2, (−1)2 − 2, (−3)2 − 2.
i.e., 2, −1, 7.
Part B
11. (a) (i) Verify Cayley Hamilton theorem for the matrix
 
1 0 3 
 
A = 2 1 −1, hence find A−1 . (8)
 
1 −1 1
Solution.
Step 1. To find the characteristic equation

Downloaded From : www.EasyEngineering.net


———-
Anna University
Downloaded Solved Question Papers
From : www.EasyEngineering.net 61

Since A is a 3 × 3 matrix, the characteristic equation is of the form


λ3 − s1 λ2 + s2 λ − s3 = 0.

Now, s1 = sum of the main diagonal elements.


= 1 + 1 + 1 = 3.
s2 = sum of the minors of the main diagonal elements.

1 −1 1 3 1 0
= + +
−1 1 1 1 2 1

= 1 − 1 + 1 − 3 + 1 − 0 = −1.

ww
1 0


3

w.E
s3 = |A| = 2 1 −1

1 −1 1

a syE
= 1(1 − 1) + 3(−2 − 1) = −9.

The characteristic equation is λ3 − 3λ2 − λ + 9 = 0.


ngi
Step 2. Verification of Cayley Hamilton theorem
By Cayley Hamilton theorem, A3 − 3A2 − A + 9I = 0. nee

1 0 3

 1 0 3 rin 

2

Now, A = A × A = 2 1 −1 2 1 −1

 
 
 

 g.n

1 −1 1 1 −1 1
1 + 0 + 3 0 + 0 − 3 3 + 0 + 3 4 −3 6

 
 


et
= 2 + 2 − 1 0 + 1 + 1 6 − 1 − 1 = 3 2 4
   
1−2+1 0−1−1 3+1+1 0 −2 5
  
4 −3 6 1 0 3 

  
 
A3 = A2 × A = 3 2 4 2 1 −1
  
   
0 −2 5 1 −1 1
   
4 − 6 + 6 0 − 3 − 6 12 + 3 + 6  4 −9 21
   
= 3 + 4 + 4 0 + 2 − 4 9 − 2 + 4  = 11 −2 11
   
0−4+5 0−2−5 0+2+5 1 −7 7

Downloaded From : www.EasyEngineering.net


———-
62
Downloaded Supplement to Engineering Mathematics - II
From : www.EasyEngineering.net

A3 − 3A2 − A + 9I
       
 4 −9 21 4 −3 6 1 0 3  1 0 0
       
= 11 −2 11 − 3 3 2 4 − 2 1 −1 + 9 0 1 0
       
1 −7 7 0 −2 5 1 −1 1 0 0 1
 
4 − 12 − 1 + 9 −9 + 9 − 0 + 0 21 − 18 − 3 + 0
 
= 11 − 9 − 2 + 0 −2 − 6 − 1 + 9 11 − 12 + 1 + 0
 
1 + 0 − 1 + 0 −7 + 6 + 1 + 0 7 − 15 − 1 + 9
 
0 0 0
 
= 0 0 0 = 0.
ww 
0 0 0


w.E
a syE
Hence, Cayley Hamilton theorem is verified.
Step 3. To find A−1
ngi
By Cayley Hamilton theorem we have A3 −3A2 − A+9I = 0.
Premultiply by A−1 we get, A2 − 3A − I + 9A−1 = 0. nee
rin
g.n
9A−1 = I + 3A − A2

1 0 0




1 0

 
3  4 −3
 

6

et
= 0 1 0 + 3 2 1 −1 − 3 2 4
     
0 0 1 1 −1 1 0 −2 5
   
1 + 3 − 4 0 + 0 + 3 0 + 9 − 6 0 3 3 
   
= 0 + 6 − 3 1 + 3 − 2 0 − 3 − 4 = 3 2 −7
   
0+3−0 0−3+2 1+3−5 3 −1 −1
 
0 3 3 
1  
A−1 = 3 2 −7
9  
3 −1 −1

Downloaded From : www.EasyEngineering.net


———-
Anna University
Downloaded Solved Question Papers
From : www.EasyEngineering.net 63

 
−2 2 −3
 
11. (a) (ii) Find the Eigen values and Eigen vectors of  2 1 −6.

 
−1 −2 0
 
−2 2 −3
 
Solution. Let A =  2 1 −6.

 
−1 −2 0
Step 1. To find the eigen values
Since A is a 3×3 matrix, the characteristic equation takes
the form

ww λ3 − s1 λ2 + s2 λ − s3 = 0
where s1 = tr(A) = −2 + 1 + 0 = −1.
w.E
s2 = sum of the minors of the main diagonal elements of A.

=
a
syE
1 −6 −2 −3 −2 2
+ +


−2 0 −1 0 2 1
ngi
s2 = 0 − 12 + 0 − 3 − 2 − 4 = −21.

−2 2 −3
nee

s3 = |A| = 2

1 −6 rin

−1 −2 0

g.n
= −2(0 − 12) − 2(0 − 6) − 3(−4 + 1) = 24 + 12 + 9 = 45. et
The characteristic equation is λ3 + λ2 − 21λ − 45 = 0.
λ = −3 is a root. By synthetic division,

−3 1 1 −21 −45
0 −3 6 45
1 −2 −15 0

=⇒ (λ + 3)(λ2 − 2λ − 15) = 0

(λ + 3)(λ − 5)(λ + 3) = 0 ⇒ λ = −3, λ = −3, λ = 5.

Downloaded From : www.EasyEngineering.net


———-
64
Downloaded Supplement to Engineering Mathematics - II
From : www.EasyEngineering.net

The eigen values are −3, −3, 5.


Step 2. To find the eigen vectors
 
 x1 
 
Let X =  x2  be an eigen vector corresponding to λ.
 
x3

∴ (A − λI)X = 0.
  
−2 − λ 2 −3   x1 
   

 2 1−λ −6   x2  = 0. (1)
   

ww 
−1

 1
−2 0 − λ x3
 
2 −3  x1 
w.E 
When λ = −3, (1) becomes  2

  
4 −6  x2  = 0.
  

a −1 −2 3 x3
syE
The above three equations are reduced to

ngi
x1 + 2x2 − 3x3 = 0. (2)

nee
Since two of the eigen values are equal, from (2) we have to get two

rin
eigen vectors by assigning arbitrary values for two variables. First,
choosing x3
x1
= 0, we obtain x1 + 2x2
x2
=
g.n
0 which implies
x1 = −2x2 ⇒
2
=
−1
⇒ x1 = 2, x2 = −1

 
 2 
et
 
∴ X1 = −1 .
 
0

Also in (2) assign x2 = 0, we obtain x1 − 3x3 = 0,


x1 x3
which implies x1 = 3x3 ⇒ = ⇒ x1 = 3, x3 = 1.
3 1
 
3
 
X2 = 0 .
 
1

Downloaded From : www.EasyEngineering.net


———-
Anna University
Downloaded Solved Question Papers
From : www.EasyEngineering.net 65

  
−7 2 −3  x1 
   
When λ = 5, (1)=⇒  2 −4 −6  x2  = 0.
   
−1 −2 −5 x3
The equations are

−7x1 + 2x2 − 3x3 = 0 





2x1 − 4x2 − 6x3 = 0  (A)




−x1 − 2x2 − 5x3 = 0. 

Taking the first two equations and on solving by the method of cross

ww
multiplication, we obtain
x1 x2 x3
w.E 2 -3 -7 2

a -4
syE -6 2 -4
x1
=
x2
ngi
=
−12 − 12 −6 − 42 28 − 4
x3

x1
=
x2
−24 −48 24
=
x3
nee
x1
=
x2
=
x3
. rin
1 2 −1
=⇒ x1 = 1, x2 = 2, x3 = −1. g.n
The values of x1 , x2 , x3 satisfy the last equation in (A). et
 
 1 
 
∴ X3 =  2  .
 
−1
     
 2  3  1 
     
∴ The eigen vectors are −1 , 0 and  2 .
   
     
0 1 −1
11. (b) (i) Reduce the quadratic form x12 + 5x22 + x32 + 2x1 x2 + 2x2 x3 + 6x3 x1
to the canonical form through orthogonal transformation and find

Downloaded From : www.EasyEngineering.net


———-
66
Downloaded Supplement to Engineering Mathematics - II
From : www.EasyEngineering.net

its nature.
Solution.
Step 1. To form the matrix of the Q.F
Let A be the matrix of the Q.F.
Since the given Q.F is in 3 variables, A must be a 3 × 3
matrix.  
a11 a12 a13 
 
Let A = a21 a22 a23 .
 
a31 a32 a33

ww Now, a11 = Coeff.of x12 = 1


1 1

w.E a12 = a21 =

a13 = a31
2
1
× Coeff.of x1 x2 = × 2 = 1
2
1
= × Coeff.of x1 x3 = × 6 = 3
a 2
syE
a22 = Coeff.of x22 = 5.
2

a23 = a32 =
1
2 ngi 1
× Coeff.of x2 x3 = × 2 = 1.
2


a33 = Coeff.of = 1.

x32
nee
1 1 3

∴ A = 1 5 1.

rin

3 1 1

g.n
Step 2. To find the eigen values
Since A is a 3 × 3 matrix, the characteristic equation is of
et
the form
λ3 − s1 λ2 + s2 λ − s3 = 0.

Now, s1 = sum of the main diagonal elements


= 1 + 5 + 1 = 7.
s2 = sum of the minors of the main diagonal elements.

5 1 1 3 1 1
= + +
1 1 3 1 1 5

Downloaded From : www.EasyEngineering.net


———-
Anna University
Downloaded Solved Question Papers
From : www.EasyEngineering.net 67

= 5 − 1 + 1 − 9 + 5 − 1 = 4 − 8 + 4 = 0.

1 1 3

s3 = |A| = 1 5 1

3 1 1

= 1(5 − 1) − 1(1 − 3) + 3(1 − 15)


= 4 + 2 − 42 = −36.

∴ The characteristic equation is λ3 − 7λ2 + 36 = 0.


λ = −2 is a root.
ww
By synthetic division, we get

w.E −2 1 −7 0 36

a syE 0
1
−2
−9
18
18
−36
0

ngi
λ2 − 9λ + 18 = 0.
nee
(λ − 3)(λ − 6) = 0.
rin
The characteristic equation becomes (λ + 2)(λ − 3)(λ − 6) = 0. g.n
The eigen values are −2, 3, 6.
Step 3.To find the eigen vectors
et
 
 x1 
 
Let X =  x2  be an eigen vector corresponding to the eigen value λ.
 
x3

∴ (A − λI)X = 0.

  
1 − λ 1 3   x1 
   

 1 5−λ 1   x2  = 0. (1)
   
3 1 1 − λ x3

Downloaded From : www.EasyEngineering.net


———-
68
Downloaded Supplement to Engineering Mathematics - II
From : www.EasyEngineering.net

When λ = −2, (1) becomes


  
3 1 3  x1 
   

1 7 1  x2  = 0.
   
3 1 3 x3
The above two equations are reduced to two equations

3x1 + x2 + 3x3 = 0

x1 + 7x2 + x3 = 0.

Solving the two equations by the rule of cross multiplication we get

ww x1 x2 x3

w.E 1

1
3 3
1
1

7
a 7
syE
x1
1 − 21 3 − 3 21 − 1
=
x2
=
x3

x1
−20
=
x2
0 ngi =
x3
20
=⇒ x1 = 1, x2 = 0, x3 = −1.
  nee
 1 
 
∴ X1 =  0  .
rin
 
−1 g.n
  
−2 1 3   x1 
   
et
When λ = 3, (1) become  1 2 1   x2  = 0.
   
3 1 −2 x3
The above equations are reduced to

−2x1 + x2 + 3x3 = 0

x1 + 2x2 + x3 = 0

3x1 + x2 − 2x3 = 0.

From the first two equations by the rule of cross multiplication we


get

Downloaded From : www.EasyEngineering.net


———-
Anna University
Downloaded Solved Question Papers
From : www.EasyEngineering.net 69

x1 x2 x3
1 3 -2 1

2 1 1 2
x1 x2 x3
= =
1 − 6 3 + 2 −4 − 1
x1 x2 x3
= = =⇒ x1 = 1, x2 = −1, x3 = 1.
−5 5 −5
 
 1 
 
∴ X2 = −1 .
 

ww 
−5 1
1
 
3   x1 
w.E    
When λ = 6, (1) becomes  1 −1 1   x2  = 0.
   

a
The above equations are reduced to
syE
3 1 −5 x3

ngi
−5x1 + x2 + 3x3 = 0

x1 − x2 + x3 = 0
nee
3x1 + x2 − 5x3 = 0.
rin
By the rule of cross multiplication, from the first two equations we
g.n
get
x1 x2 x3 et
1 3 -5 1

-1 1 1 -1
x1 x2 x3
= =
1+3 3+5 5−1
x1 x2 x3
= = =⇒ x1 = 1, x2 = 2, x3 = 1.
4 8 4
 
1
 
X3 = 2 .
 
1

Downloaded From : www.EasyEngineering.net


———-
70
Downloaded Supplement to Engineering Mathematics - II
From : www.EasyEngineering.net

Step 4. To form the modal matrix


Since the eigen values are different, the eigen vectors are mutually
orthogonal.
     
 √1   √1   √1 
 2   3   6 
     
The normalized eigen vectors are  0 , − √1 ,  √2 .
   3   6 
−1
√ √1 √1
2 3 6
The modal matrix N is formed with columns as the normalized
eigen vectors.

 
 √1 √1 √1 
ww  2
∴ N =  0

−1

3

3
2
6
√ 
6



w.E  −1

2
√1
3
√1
3

a
Step 5. To find N AN


T

syE   
 √1
 2
0
ngi
− √12  1 1 3  √12
   
 1 5 1  0 
√1
3
√1 
6


N T AN =  √1
 3
 √1
− √13
√2
√1 

√1
3    nee  
3 1 1 − √1

− √1

√1
3
√ 


2

1
6



 √
− 2
6 6 3
√  1
2  √2 rin √1
2

√1 


3 3

=  3 − 3
 √


0
√ √ 




3  0 
3
− √3 √6 
1 2
6 
g.n

 √

−2 0 0
6 2 6


√   1
6 −√
2
√1
3
√1
3


et
 
=  0 3 0 = D.
 
0 0 6

Step 6. Reduction to Canonical form


 
y1 
 
Let X = NY, where Y = y2 .
 
y3

The given Q.F is Q = X T AX.

Downloaded From : www.EasyEngineering.net


———-
Anna University
Downloaded Solved Question Papers
From : www.EasyEngineering.net 71

= (NY)T A(NY)
= Y T N T ANY
= Y T DY
  
 0 0 y1 
  −2   

= y1 y2 y3  0 3 0 y2 
   
0 0 6 y3

= −2y21 + 3y22 + 6y23

which is canonical.
ww
Since two of the eigen values are positive and one eigen value is

w.E
negative, the given Q.F is indefinite.
11. (b) (ii) Prove that the Eigen values of a real symmetric matrix
are real. a syE
Solution. Let λ (real or complex) be an eigen value of A.
ngi
Let X be the eigen vector corresponding to the eigen value λ.
∴ AX = λX.
nee (1)
Let X̄ be the Complex conjugate of X.
Premultiply (1) by X̄ T we get rin
X̄ T AX = λX̄ T X g.n
X̄ T AX = λX̄ T X
T T
i.e., X ĀX̄ = λ̄X X̄.
et
Since A is a real symmetric matrix, Ā = A & AT = A.

∴ X T AX̄ = λ̄X T X̄.

Taking transpose on both sides we get


 T  T
X T AX̄ = λ̄X T X̄
X̄ T AT X = X̄ T X λ̄T = λ̄X̄ T X
X̄ T AX = λ̄X̄ T X

Downloaded From : www.EasyEngineering.net


———-
72
Downloaded Supplement to Engineering Mathematics - II
From : www.EasyEngineering.net

X̄ T λX = λ̄X̄ T X
λX̄ T X = λ̄X̄ T X.

Now X is an n × 1 matrix and X̄ T is a 1 × n matrix.


Hence, X̄ T X is a 1 × 1 matrix, which is a positive real.
⇒ λ = λ̄.
⇒ λ is a real number.

2.1.4 JANUARY 2014 [R 2013]

ww
Part A

w.E
1. If the eigen values of the matrix A of order 3 × 3 are 2, 3, 1 then find
the eigen values of adjoint of A.

a
Solution. Eigen values of A are 2, 3, 1.
syE
|A| = Product of eigen values
= 2 × 3 × 1 = 6.
ngi
We know that ad j(A) = |A| A−1 .
1 1
Eigen values of A−1 are , , 1. nee
2 3
1
rin
1
∴ The eigen values of ad j(A) are 6 × , 6 × , 6 × 1
i.e., 3, 2, 6.
2 3
g.n
2. If λ is the eigen value of the matrix A ,then prove that λ2 is the
eigen value of A2 .
et
Solution. Since λ is an eigen value of A, there exists a column
matrix X such that
AX = λX
A(AX) = A(λX)
(AA)X = λ(AX)
A2 X = λ(λX)
A2 X = λ2 X.
⇒ λ2 is an eigen value of A2 .

Downloaded From : www.EasyEngineering.net


———-
Anna University
Downloaded Solved Question Papers
From : www.EasyEngineering.net 73

Part B
11. (a)
(i) Find the eigen values and eigen vectors of the matrix
 
2 1 2
 

3 1
1
 
2 2 1
 
2 2 1
 
Solution. Let A = 1 3 1.
 
1 2 2
Step 1. To find the eigen values

ww
Since A is a 3 × 3 matrix, the characteristic equation takes the form
λ3 − s1 λ2 + s2 λ − s3 = 0, where

w.E
s1 = sum of the main diagonal elements of A

a
= 2 + 3 + 2 = 7.
syE
s2 = sum of the minors of the main diagonal elements of A.

ngi
3 1 2 1 2 2

= + +
2 2 1 2 1 3 nee
rin
= (6 − 2) + (4 − 1) + (6 − 2) = 4 + 3 + 4 = 11.

2 2 1
g.n
s3 = |A| = 1 3 1 = 2(6 − 2) − 2(2 − 1) + 1(2 − 3) = 8 − 2 − 1 = 5.

1 2 2
et
The characteristic equation is λ3 − 7λ2 + 11λ − 5 = 0.
λ = 1 is a root.
By synthetic division we have

1 1 −7 11 −5
0 1 −6 5
1 −6 5 0

λ2 − 6λ + 5 = 0

Downloaded From : www.EasyEngineering.net


———-
74
Downloaded Supplement to Engineering Mathematics - II
From : www.EasyEngineering.net

(λ − 1)(λ − 5) = 0 ⇒ λ = 1, 5.

∴ The eigen values are 1, 1, 5.


Step 2. To find the eigen vectors
 
 x1 
 
Let X =  x2  be the eigen vector corresponding to an eigen value λ.
 
x3

∴ (A − λI)X = 0
  
2 − λ 2 1   x1 

ww 
 1

3−λ
  
1   x2  = 0.
  
(1)

w.E
When λ = 1, (1) becomes
1 2 2 − λ x3

a syE 
1


 
2 1  x1 
  
2 1  x2  = 0.
1

1 2 1 x3ngi
  

All the three equations are reduced to one single equation nee
x1 + 2x2 + x3 = 0. rin (2)
Since two of the eigen values are equal, from(2), we have to get two
g.n
eigen vectors by assigning arbitrary values for two variables.
First, choosing x3 = 0, we obtain x1 + 2x2 = 0 ⇒ x1 = −2x2 et
x1 x2
i.e.,
= ⇒ x1 = −2, x2 = 1.
−2 1
 
−2
 
∴ One eigen vector is X1 =  1 .
 
0
Assign x2 = 0, (2) becomes

x1 + x3 = 0
x1 = −x3

Downloaded From : www.EasyEngineering.net


———-
Anna University
Downloaded Solved Question Papers
From : www.EasyEngineering.net 75

x1 x3
=
1 −1
⇒ x1 = 1, x3 = −1.

 
 1 
 
∴ The second eigen vector is X2 =  0 .
 
−1
When λ = 5, (1) becomes
  
−3 2 1   x1 
   

ww  1


1
−2 1   x2  = 0.
2 −3 x3
  

The equations become


w.E
−3x1 + 2x2 + x3
a =0  syE







x1 − 2x2 + x3 =0  




= 0. 
ngi (A)
x1 + 2x2 − 3x3
nee
All the three equations are different.
rin
Taking the first two equations and applying the rule of cross
multiplication we obtain g.n
x1 x2 x3 et
2 1 -3 2
-2 1 1 -2

x1 x2 x3
= =
2+2 1+3 6−2

x1 x2 x3
= =
4 4 4

⇒ x1 = 1, x2 = 1, x3 = 1.

Downloaded From : www.EasyEngineering.net


———-
76
Downloaded Supplement to Engineering Mathematics - II
From : www.EasyEngineering.net

 
1
 
The third eigen vector is X3 = 1.
 
1
     
−2  1  1
     
∴ The eigen vectors are  1  ,  0  , 1.
     
0 −1 1
11. (a) (ii) Using Cayley-Hamilton theorem find A−1 and A4 ,
 
 1 2 −2
 
if A = −1 3 0 .
 

ww
Solution.
0 −2 1

w.E
Step 1. To find the characteristic equation
Since A is 3 × 3 matrix, the characteristic equation is of the form
a syE
λ3 − s1 λ2 + s2 λ − s3 = 0.

ngi
Now, s1 = sum of the main diagonal elements
= 1 + 3 + 1 = 5.
nee
s2 = sum of the minors of the elements
of the main diagonal rin

3 0 1 −2 1

2 g.n
= + +
−2 1 0 1 −1 3

et
=3−0+1−0+3+2
=3+1+5
= 9.

1 2 −2

s3 = |A| = −1 3 0

0 −2 1

= 1(3 − 0) − 2(−1 − 0) − 2(2 − 0)


= 3 + 2 − 4 = 1.

Downloaded From : www.EasyEngineering.net


———-
Anna University
Downloaded Solved Question Papers
From : www.EasyEngineering.net 77

The characteristic equation is λ3 − 5λ2 + 9λ − 1 = 0.


By Cayley Hamilton theorem, we have

A3 − 5A2 + 9A − I = 0.

Step2. Verification of Cayley Hamilton theorem

    
 1 2 −2  1 2 −2 −1 12 −4
     
A2 = A.A = −1 3 
0  −1 3 0  = −4 7 2 
     
0 −2 1 0 −2 1 2 −8 1

ww 
−1
 12

−4  1
 
2
 
−2 −13 42 −2
 


3 2

w.E
A = A .A = −4

2 −8
7 2  −1 3
1
 
0 −2 1
0  = −11
 
9
10 −22 −3
10 


∴ A3 − 5A2 + 9A − I

a  syE
     
−13 42 −2

= −11
9


−1

10  − 5 −4

12
7 ngi
−4

 1

2  + 9 −1

3
2 −2 1
 
0  − 0
0 0

1 0

10 −22 −3
 
2 −8 1

nee

0 −2 1
 
0 0 1



0 0 0


 rin
= 0 0 0 = 0.
  g.n
0 0 0

Hence, Cayley Hamilton theorem is verified.


et
Step 3. To find A−1
By Cayley Hamilton theorem, we have A3 − 5A2 + 9A − I = 0.
Multiplying by A−1 we get

A2 − 5A + 9I − A−1 = 0
 
3 2 6
 
∴ A−1 = A2 − 5A + 9I = 1 1 2 .
 
2 2 5

Downloaded From : www.EasyEngineering.net


———-
78
Downloaded Supplement to Engineering Mathematics - II
From : www.EasyEngineering.net

Step 4. To find A4
By Cayley Hamilton theorem, we have

A3 − 5A2 + 9A − I = 0.
A3 = 5A2 − 9A + I.

Multiplying by A we get

A4 = 5A3 − 9A2 + A

ww 
−13 42 −2




−1 12 −4  1

 
  2 −2




w.E
= 5 −11

9
10 −22 −3

10  − 9 −4 7



2 −8 1
 
2  + −1 3
 
0 −2 1
0 



= −55
a 
 syE
−65 210 −10  9
45
 
 
 
50  +  36
 
−108 36   1
 
−63 −18 + −1 3
2 −2


0 

50 −110 −15 ngi
 
−18 72 −9
 
0 −2 1



 −65 + 9 + 1 210 − 108 + 2 −10 + 36 − 2
 nee 

= −55 + 36 − 1

45 − 63 + 3
rin
50 − 18 + 0 




50 − 18 + 0 −110 + 72 − 2 −15 − 9 + 1
 g.n
−55 104 24 

= −20 −15 32  .




et
32 −40 −23

11. (b) Reduce the quadratic form 6x2 + 3y2 + 3z2 − 4xy − 2yz + 4xz into
a canonical form by an orthogonal reduction.Hence find its rank and
nature.
Solution.
Step 1. To find the matrix of the Q.F
Since the given Q.F contains 3 variables, the matrix of the Q.F A is
a 3 × 3 matrix.

Downloaded From : www.EasyEngineering.net


———-
Anna University
Downloaded Solved Question Papers
From : www.EasyEngineering.net 79

 
a11 a12 a13 
 
Let A = a21 a22 a23 .
 
a31 a32 a33

Now, a11 = Coeff.of x2 = 6


1 1
a12 = a21 = × Coeff.of xy = × (−4) = −2.
2 2
1 1
a13 = a31 = × Coeff.of xz = × 4 = 2
2 2
a22 = Coeff.of y2 = 3.

ww a23 = a32 =
1
2
1
× Coeff.of yz = × (−2) = −1.
2

w.E a33 = Coeff.of z2 = 3.

a 
 6 −2 2 

∴ A = −2 3 −1.

syE


2 −1 3

ngi
Step 2. To find the eigen values
nee
rin
Since A is a 3 × 3 matrix, the characteristic equation is of the form
λ3 − s1 λ2 + s2 λ − s3 = 0.
s1 = sum of the main diagonal elements g.n
= 6 + 3 + 3 = 12.
s2 = sum of the minors of the main diagonal elements.
et

3 −1 6 2 6 −2
= + +
−1 3 2 3 −2 3

= 9 − 1 + 18 − 4 + 18 − 4 = 8 + 14 + 14 = 36.

6 −2 2

s3 = |A| = −2 3 −1

2 −1 3

= 6(9 − 1) + 2(−6 + 2) + 2(2 − 6) = 48 − 8 − 8 = 32.

Downloaded From : www.EasyEngineering.net


———-
80
Downloaded Supplement to Engineering Mathematics - II
From : www.EasyEngineering.net

∴ The characteristic equation is λ3 − 12λ2 + 36λ − 32 = 0.


λ = 2 is a root. By synthetic division, we get

2 1 −12 36 −32
0 2 −20 32
1 −10 16 0

λ2 − 10λ + 16 = 0

(λ − 2)(λ − 8) = 0.

ww
The characteristic equation becomes (λ − 2)(λ − 2)(λ − 8) = 0.
The eigen values are 2, 2, 8.

w.E
Step 3. To find the eigen vectors
 
 x1 
a
 
syE
Let X =  x2  be an eigen vector corresponding to the eigen value λ.
 
x3
ngi

∴ (A − λI)X = 0.
  nee
6 − λ

 −2
−2
rin
2   x1 
  
−1   x2  = 0.


2
3−λ
−1 3 − λ x3
  
g.n
(1)

When λ = 8 (1) becomes


  
−2 −2 2   x1 
et
   
−2 −5 −1  x2  = 0.
   
2 −1 −5 x3
The equations are −2x1 − 2x2 + 2x3 = 0 ⇒ x1 + x2 − x3 = 0.

−2x1 − 5x2 − x3 = 0 ⇒ 2x1 + 5x2 + x3 = 0.

2x1 − x2 − 5x3 = 0 ⇒ 2x1 − x2 − 5x3 = 0.

From the first two equations, using the rule of crossmultiplication


we get,

Downloaded From : www.EasyEngineering.net


———-
Anna University
Downloaded Solved Question Papers
From : www.EasyEngineering.net 81

x1 x2 x3
= =
1 + 5 −2 − 1 5 − 2
x1 x2 x3
= = ⇒ x1 = 2, x2 = −1, x3 = 1.
6 −3 3
 
 2 
 
∴ X1 = −1 .
 

ww
When λ = 2 (1) becomes
  
1

w.E
 4 −2 2   x1 

 −2 1 −1
  
   = 0.
  x2 
a
2 −1 1 x3
syE
 

ngi
i.e., 4x1 − 2x2 + 2x3 = 0 ⇒ 2x1 − x2 + x3 = 0

nee
−2x1 + x2 − x3 = 0 ⇒ 2x1 − x2 + x3 = 0
2x1 − x2 + x3 = 0.
rin
The above three equations are reduced to one single equation
2x1 − x2 + x3 = 0. Choose x3 = 0. ⇒ 2x1 − x2 = 0 ⇒ 2x1 = x2 ⇒
x1
=g.n
x2

⇒ x1 = 1, x2 = 2.
 
1
1 2
et
 
∴ X2 = 2 .
 
0
 
a
 
Let X3 = b be orthogonal to X2 .
 
c
 
 
  a
 
⇒ X2 X3 = 0 ⇒ 1 2 0 b ⇒ a + 2b = 0.
T
(2)
 
c

Downloaded From : www.EasyEngineering.net


———-
82
Downloaded Supplement to Engineering Mathematics - II
From : www.EasyEngineering.net

X3 also satisfy 2a − b + c = 0. (3)


a b
(2) ⇒ a = −2b =⇒ = .
−2 1
b c
(3) ⇒ −4b − b + c = 0 ⇒ 5b = c ⇒ = .
1 5
a b c
Combining the above two equations we obtain, = = .
−2 1 5

Hence, a = −2, b = 1, c = 5.

 
−2
 
∴ X3 =  1  .
 
ww 5

w.E
Step 4. To find the modal matrix
The normalised eigen vectors are

a
h 2 −1 1 iT h 1 2 iT h −2
√ √ √ , √ √ 0 , √
6 6 6 5 5

1

5 iT
30 30 30 syE

 √2
 6
√1
5
−2 
√ 
30 


ngi
∴ N =  −1
√
 6
 √1
√2
5
√ 
1
30 
 .
 nee
6
0 √5
30
rin
Step5. To find N T AN
g.n

 √2
 6

−1

6
√1
6

  6
 

−2

2   √26
 
√1
5
−2 
√ 
30 


et
N T AN =  √1 √2 0  −2 3 −1  −1 √ √2 √ 
1
30 

 5 5     6 5 
 √−2 √1 √5 2 −1 3  √1 0 √5
30 30 30 6 30
  
 √16 −8
√ √8    √26 √1 −2
√  
 6 6 6    5 30  
 
=  √2 −8
√ 0   −1 √ √2 √ 
1

 5 5   6 5 30  
 √−4 √2 √10 √1 0 √5
30 30 30 6 30
 
8 0 0
 
= 0 2 0 = D.
 
0 0 2

Downloaded From : www.EasyEngineering.net


———-
Anna University
Downloaded Solved Question Papers
From : www.EasyEngineering.net 83

Step 6. Reduction to canonical form


 
y1 
 
Let X = NY, where Y = y2 
 
y3
The given Q.F. is
Q = X T AX = (NY)T ANY = Y T N T ANY = Y T DY

    
8 0 0 y1  8y1 
     
Y T DY = [y1 y2 y3 ] 0  
2 0 y2  = [y1 y2 y3 ] 2y2  = 8y21 + 2y22 + 2y23 ,
     

ww 0 0 2 y3 2y3

w.E
which is in the canonical form.
Rank of the Q.F = 3.
Index = 3.
Signature = 3.
a syE
ngi
Since all the eigen values are positive, the Q.F is positive definite.

nee
2.1.5 JANUARY 2014 [R 2008] rin
g.n
Part A 
 6
 −2

2 

et
1. The product of two eigen values of the matrix A = −2 3 −1 is
 
2 −1 3
16. Find the third eigenvalue.
Solution. Let λ1 , λ2 , λ3 be the eigen values of A.
Given λ1 λ2 = 16.
6 −2 2

|A| = −2 3 −1

2 −1 3

= 6(9 − 1) + 2(−6 + 2) + 2(2 − 6) = 48 − 8 − 8 = 32.

Downloaded From : www.EasyEngineering.net


———-
84
Downloaded Supplement to Engineering Mathematics - II
From : www.EasyEngineering.net

We know that λ1 λ2 λ3 = 32
16λ3 = 32
λ3 = 2.
∴ The third eigen value is 2.
2. Discuss the nature of the quadratic form 2x2 + 3y2 + 2z2 + 2xy.
 
2 1 0
 
Solution. The matrix of the quadratic form is A = 1 3 0.
 
0 0 2
The principal minors are

ww D1 = 2 > 0.

w.E
D2 =
2 1
1 3

= 6 − 1 = 5 > 0.

a
syE
2 1 0




0 0 2
ngi
D3 = 1 3 0 = 2(6 − 0) − 1(2 − 0) = 12 − 2 = 10 > 0.

nee
rin
Since all Di ’s are positive, the given quadratic form is positive
definite.
Part B g.n
11. (a) (i) Find the eigenvalues and eigenvectors of the matrix



 8 −6 2 

et
A = −6 7 −4.
 
2 −4 3
Solution.
Step 1. To find the characteristic equation
Since A is 3 × 3 matrix, the characteristic equation is of the form
λ3 − s1 λ2 + s2 λ − s3 = 0, where

s1 = sum of the main diagonal elements of A


= 8 + 7 + 3 = 18.

Downloaded From : www.EasyEngineering.net


———-
Anna University
Downloaded Solved Question Papers
From : www.EasyEngineering.net 85

s2 = sum of the minors of the main diagonal elements of A



7 −4 8 2 8 −6
= + +
−4 3 2 3 −6 7

= (21 − 16) + (24 − 4) + (56 − 36)


= 5 + 20 + 20 = 45.

8 −6 2

s3 = |A| = −6 7 −4

2 −4 3

ww = 8(21 − 16) + 6(−18 + 8) + 2(24 − 14)

w.E
= 40 − 60 + 20 = 0.

a
The characteristic equation is

syE
λ3 − 18λ2 + 45λ = 0
λ(λ2 − 18λ + 45) = 0
λ(λ − 3)(λ − 15) = 0 ngi
λ = 0, 3, 15. nee
Step 3. To find the eigen vectors
  rin
 x1 
 
g.n
Let X =  x2  be the eigen vector corresponding to the eigen value λ.
 
x3 et
∴ (A − λI)X = 0.
  
8 − λ −6 2   x1 
   
 −6 7 − λ −4   x2  = 0. (1)
   

2 −4 3 − λ x3
When λ = 0 (1) becomes
  
 8 −6 2   x1 
   

−6 7 −4  x2  = 0.
   
2 −4 3 x3

Downloaded From : www.EasyEngineering.net


———-
86
Downloaded Supplement to Engineering Mathematics - II
From : www.EasyEngineering.net

The equations become


8x1 − 6x2 + 2x3 = 0
−6x1 + 7x2 − 4x3 = 0
2x1 − 4x2 + 3x3 = 0.
Taking the first two equations and applying the rule of cross
multiplication, we get

ww x1 x2 x3

w.E =
x1
=
x2
=
=
24 − 14 −12 + 32 56 − 36
x3

  a
1
  syE
10 20 20

∴ X1 = 2.
 
2 ngi
When λ = 3 (1) becomes nee

 5

 
−6 2   x1 
   rin

−6

4 −4  x2  = 0.
   g.n
The equations become
2 −4 0 x3
et
5x1 − 6x2 + 2x3 = 0
−6x1 + 4x2 − 4x3 = 0
2x1 − 4x2 + 0x3 = 0.
Taking the last two equations and applying the rule of cross
multiplication we get,

Downloaded From : www.EasyEngineering.net


———-
Anna University
Downloaded Solved Question Papers
From : www.EasyEngineering.net 87

x1 x2 x3
= =
0 − 16 −8 24 − 8
x1 x2 x3
= =
−16 −8 16
 
 2 
 
∴ X2 =  1 .
 
−2
When λ = 15 (1) becomes
  
−7 −6 2   x1 
   

−6 −8 −4   x2  = 0.
  
ww 
2 −4 −12 x3

w.E
The equations become
−7x1 − 6x2 + 2x3 = 0

a
−6x1 − 8x2 − 4x3 = 0
syE
2x1 − 4x2 − 12x3 = 0.
ngi
Taking the first two equations and applying the rule of cross
multiplication we get,
nee
rin
g.n
x1
=
x2
=
x3
et
24 + 16 −12 − 28 56 − 36
x1 x2 x3
= =
40 −40 20
 
 2 
 
∴ X3 = −2.
 
1
     
1  2   2 
     
∴ The eigen vectors are 2 ,  1 and −2.
     
2 −2 1

Downloaded From : www.EasyEngineering.net


———-
88
Downloaded Supplement to Engineering Mathematics - II
From : www.EasyEngineering.net

11. (a) (ii) Using Cayley-Hamilton theorem find A−1 for the matrix
 
1 0 3 
 
A = 2 1 −1.
 
1 −1 1
 
1 0 3 
 
Solution. A = 2 1 −1
 
1 −1 1
The characteristic equation is λ3 − s1 λ2 + s2 λ − s3 = 0, where

s1 = sum of the main diagonal elements


ww= 1 + 1 + 1 = 3.

w.E
s2 = sum of the minors of the main diagonal elements

1 −1 1 3 1 0

= a + +
syE
−1 1 1 1 2 1

= (1 − 1) + (1 − 3) + (1 − 0) = −1.
ngi
1 0

3
nee
s3 = |A| = 2 1 −1 = 1(1 − 1) − 0(2 + 1) + 3(−2 − 1) = 0 − 0 − 9 = −9.

1 −1 1

rin
The characteristic equation is λ3 − 3λ2 − λ + 9 = 0.
g.n
By Cayley-Hamilton theorem A3 − 3A2 − A + 9I = 0. et
Premultiply byA−1 ,we have A2 − 3A − I + 9A−1 = 0.

−9A−1 = I + 3A − A2 .
    
1 0 3  1 0 3  4 −3 6

     
A2 = 2 1 −1 2 1

−1 = 3 2 4
     
1 −1 1 1 −1 1 0 −2 5
     
1 0 0 3 0 9  4 −3 6
     
−9A−1 = 0 1 0 + 6 3 −3 − 3 2 4
     
0 0 1 3 −3 3 0 −2 5

Downloaded From : www.EasyEngineering.net


———-
Anna University
Downloaded Solved Question Papers
From : www.EasyEngineering.net 89

   
0 3 3  0 3 3 
  1  
−9A−1 = 3 2 −7 ⇒ ∴ A−1 = − 3 2 −7
  9  
3 −1 −1 3 −1 −1

11. (b) Reduce the quadratic form Q = 3x2 − 3y2 − 5z2 − 2xy − 6yz − 6xz
to its canonical form using orthogonal transformation. Also find its
rank, index and signature.
Solution.
Step 1. To find the matrix of the Q.F

ww
Since the given Q.F contains 3 variables, the matrix of the Q.F A is
a 3 × 3 matrix.  

w.E a11

Let A = a21
a12
a22
a13 

a23 .

a 
a31 a32
syE a33


Now, a11 = Coeff.of x2 = 3


ngi
a12 = a21 =
1
2
1
nee
× Coeff.of xy = × (−2) = −1.
2
a13 = a31
1 1
rin
= × Coeff.of xz = × (−6) = −3.
2 2
a22 = Coeff.of y2 = −3.
g.n
a23 = a32
1
2
2
1
= × Coeff.of yz = × (−6) = −3.
2 et
a33 = Coeff.of z = −5.
 
 3 −1 −3
 
∴ A = −1 −3 −3.
 
−3 −3 −5
Step 2. To find the eigen values
Since A is a 3 × 3 matrix, the characteristic equation is of the form
λ3 − s1 λ2 + s2 λ − s3 = 0, where

s1 = sum of the main diagonal elements

Downloaded From : www.EasyEngineering.net


———-
90
Downloaded Supplement to Engineering Mathematics - II
From : www.EasyEngineering.net

= 3 − 3 − 5 = −5.
s2 = sum of the minors of the main diagonal elements.

−3 −3 3 −3 3 −1
= + +
−3 −5 −3 −5 −1 −3

= (15 − 9) + (−15 − 9) + (−9 − 1) = 6 − 10 − 24 = −28.



3 −1 −3

s3 = |A| = −1 −3 −3

−3 −3 −5

ww = 3(15 − 9) + 1(5 − 9) − 3(3 − 9) = 18 − 4 + 18 = 32.

w.E
∴ The characteristic equation is λ3 + 5λ2 − 28λ − 32 = 0.

a
λ = −1 is a root. By synthetic division we get

−1 syE 1 5 − 28 − 32
0 −1
ngi−4 32
1 4 − 32
nee 0

λ2 + 4λ − 32 = 0
rin
(λ − 4)(λ + 8) = 0.
g.n
The characteristic equation becomes (λ − 4)(λ + 1)(λ + 8) = 0.
The eigen values are 4, −1, −8.
et
Step 3. To find the eigen vectors
 
 x1 
 
Let X =  x2  be an eigen vector corresponding to the eigen value λ.
 
x3

∴ (A − λI)X = 0.
  
3 − λ −1 −3   x1 
   

 −1 −3 − λ −3   x2  = 0. (1)
   
−3 −3 −5 − λ x3

Downloaded From : www.EasyEngineering.net


———-
Anna University
Downloaded Solved Question Papers
From : www.EasyEngineering.net 91

When λ = 4 (1) becomes


  
−1 −1 −3  x1 
   
−1 −7 −3  x2  = 0.
   
  
−3 −3 −9 x3
The equations are −x1 − x2 − 3x3 = 0
−x1 − 7x2 − 3x3 = 0
−3x1 − 3x2 − 9x3 = 0 ⇒ −x1 − x2 − 3x3 = 0.
From the first two equations, using the rule of crossmultiplication
we get,

ww
w.E
a syEx1
=
x2
3 − 21 3 − 3 7 − 1
=
x3

x1
= ngi
x2
=
x3
−18
x1 x2
0

= .
nee
6
x3
−3
=
0 1
rin
 
−3
 
∴ X1 =  0  .
g.n
 
1
et
When λ = −1 (1) becomes
  
 4 −1 −3  x1 
   
    = 0.
 −1 −2 −3   x2 
  
−3 −3 −4 x3
The equations are 4x1 − x2 − 3x3 = 0
−x1 − 2x2 − 3x3 = 0
−3x1 − 3x2 − 4x3 = 0.
From the first two equations, using the rule of crossmultiplication
we get,

Downloaded From : www.EasyEngineering.net


———-
92
Downloaded Supplement to Engineering Mathematics - II
From : www.EasyEngineering.net

x1 x2 x3
= =
3 − 6 3 + 12 −8 − 1
x1 x2 x3
= =
−3 15 −9
x1 x2 x3
= = .
1 −5 3

ww  
 1 
 
∴ X2 = −5 .
w.E  
3

a
When λ = −8 (1) becomes


syE
 
 11 −1 −3  x1 
  


  
  ngi
−1 5 −3  x2  = 0.

−3 −3 3 x3
The equations are 11x1 − x2 − 3x3 = 0 nee
−x1 + 5x2 − 3x3 = 0
rin
−3x1 − 3x2 + 3x3 = 0.
g.n
From the last two equations, using the rule of crossmultiplication,
we get et

x1 x2 x3
= =
15 − 9 9 + 3 3 + 15
x1 x2 x3
= =
6 12 18
x1 x2 x3
= =
1 2 3

Downloaded From : www.EasyEngineering.net


———-
Anna University
Downloaded Solved Question Papers
From : www.EasyEngineering.net 93

 
1
 
∴ X3 = 2 .
 
3
Since all the eigen values are different, the eigen vectors are
pairwise orthogonal.
Step 4. To find the modal matrix
The normalised eigen vectors are
h −3 1 iT h 1 −5 3 iT h 1 2 3 iT
√ 0 √ , √ √ √ , √ √ √
10 10 35 35 35 14 14 14

ww 
 √−3
 10

√1
35
√1 

√ 


14 

w.E
∴ N =  0

 √1
−5

√3
35

2

3
14 
 .


a
10 35 14

Step5. To find N T AN
syE

 √−3
 10
0 √1 

  3
10   ngi
−1

−3  √−310
 
√1
35
√1  
14 

N T AN =  √1
 35
 √1
−5

35
√3 
35 

 −1
 

−3
nee
−3  0
−5  √1
 
−5

35
√ 
2
14 


√2 √3

rin √3 3
−3 −3 √
14 14 14 10 35 14
    
 −12
 10
=  √−1
√ 0
√5
√4 

−3 
  √−310
10  
  0
√1

−5
35
√1 

√2 
 4
14  
 = 0
0 
0

−1 0  = D.
g.n
et
√ √
 35 35 35    35 14    
 √−8   1  0
−16
√ −24
√ √ √3 √3 0 −8
14 14 14 10 35 14

Step 6. Reduction to canonical form


 
y1 
 
Let X = NY, where Y = y2 
 
y3
The given Q.F. is Q = X AX = (NY)T ANY = Y T N T ANY = Y T DY
T

    
4 0 0  y1   4y1 
    
 
Y DY = [y1 y2 y3 ] 0 −1
T 
0  y2  = [y1 y2 y3 ]  −y2  = 4y21 − y22 − 8y23 ,
     
0 0 −8 y3 −8y3

Downloaded From : www.EasyEngineering.net


———-
94
Downloaded Supplement to Engineering Mathematics - II
From : www.EasyEngineering.net

which is the canonical form.


Rank of the Q.F = 3, Index = 1, Signature = −1.

2.1.6 JANUARY 2013 [R 2008]


Part A
1. Find the symmetric matrix  A whose  eigenvalues are 1 and 3 with
 1  1
corresponding eigenvectors   and  .
−1 1
Solution.  

ww
Let the required symmetric matrix be A = 
a b

b c
.

w.E
The eigen values are  λ1 = 1 and
 1 
  λ2 = 3, with corresponding eigen
1

a
vectors are X1 =   and X2 =  .
−1
By definition, AX1 = λ1 X1 syE
1

  
a b  1 

   
 1   1 
   = 1   =  
ngi
b c −1 −1 −1
nee
which yields the equations rin
a − b = 1. g.n
(1)

b − c = −1.
et
(2)

Also, AX2 = λ2 X2    


a b 1 1 3
i.e.,     = 3   =   which yields the equations
b c  1   1  3 

a+b=3 (3)

b+c=3 (4)

(1) + (3) =⇒ 2a = 4 =⇒ a = 2.

(2) + (4) =⇒ 2b = 2 =⇒ b = 1.

Downloaded From : www.EasyEngineering.net


———-
Anna University
Downloaded Solved Question Papers
From : www.EasyEngineering.net 95

(4) =⇒ 1 + c = 3 =⇒ c = 2.

 
2 1
∴ The required matrix is A =  .

1 2
2. Write down the quadratic form corresponding to the matrix
 
 2 0 −2
 
 .
 0 2 1 
 
−2 1 −2
Solution.
From the matrix of the quadratic form we observe that
ww
a11 = 2, a12 = 0, a13 = −2, a22 = 2, a23 = 1, a33 = −2.

w.E
The required quadratic form is
a11 x2 + a22 y2 + a33 z2 + 2a12 xy + 2a13 xz + 2a23 yz

Part B
a syE
= 2x2 + 2y2 − 2z2 − 4xz + 2yz.

 
ngi  1


11. (a) (i) Find the eigenvalues and eigenvectors of  6 −1
2 1 

0 .
nee 
−1 −2 −1


 1
 2


1  rin
Solution. Let A =  6 −1 0 .




 g.n
−1 −2 −1
Step 1. To find the eigen values et
Since A is a 3 × 3 matrix, the characteristic equation takes the form
λ3 − s1 λ2 + s2 λ − s3 = 0, where

s1 = sum of the main diagonal elements of A.


= 1 − 1 − 1 = −1.
s2 = sum of the minors of the main diagonal elements.

−1 0 1 1 1 2
= + +
−2 −1 −1 −1 6 −1

= 1 − 0 + (−1) + 1 + (−1 − 12) = 1 + 0 − 13 = −12.

Downloaded From : www.EasyEngineering.net


———-
96
Downloaded Supplement to Engineering Mathematics - II
From : www.EasyEngineering.net


1 2 1

s3 = |A| = 6 −1 0

−1 −2 −1

= 1(1 − 0) − 2(−6 − 0) + 1(−12 − 1) = 1 + 12 − 13 = 0.

The characteristic equation is


λ3 + λ2 − 12λ = 0
λ(λ2 + λ − 12) = 0
λ(λ + 4)(λ − 3) = 0

ww λ = 0, −4, 3.

w.E
∴ The eigen values are 0, −4, 3.
Step 2. To find the eigen vectors
 
a  x1 
 
syE
Let X =  x2  be the eigen vector corresponding to the eigen value λ.
 

∴ (A − λI)X = 0
x3
ngi

1 − λ
 2 nee
 
1   x1 
  
i.e.,  6

−1 − λ
rin
0   x2  = 0.
  
(1)

When λ = 0, (1) becomes


−1 −2 −1 − λ x3
g.n

 1

2 1
 
  x1 
  
 6 −1 0   x2  = 0
et
   
−1 −2 −1 x3

i.e., x1 + 2x2 + x3 = 0 (2)


6x1 − x2 = 0 (3)
−x1 − 2x2 − x3 = 0. (4)

(3) can be written as


6x1 = x2

Downloaded From : www.EasyEngineering.net


———-
Anna University
Downloaded Solved Question Papers
From : www.EasyEngineering.net 97

x1 x2
=
1 6

⇒ x1 = 1, x2 = 6.

Substituting in (2) we obtain, 1 + 12 + x3 = 0

⇒ x3 = −13.
 
 1 
 
∴ The eigen vector is X1 =  6 .
 
−13
ww
When λ = −4, (1) becomes

w.E 
 5


2
 
1  x1 
  

a syE
 6

−1
3 0  x2  = 0
−2 3 x3
  

ngi
i.e., 5x1 + 2x2 + x3 = 0
nee (5)
6x1 + 3x2 = 0
rin (6)
−x1 − 2x2 + 3x3 = 0.
g.n (7)

(6) can be written as


6x1 = −3x2
et
x1 x2
=
−3 6
x1 x2
=
1 −2

∴ x1 = 1, x2 = −2.

Substituting in (5) we obtain

5 − 4 + x3 = 0 ⇒ x3 = −1.

Downloaded From : www.EasyEngineering.net


———-
98
Downloaded Supplement to Engineering Mathematics - II
From : www.EasyEngineering.net

 
 1 
 
∴ The second eigen vector is X2 = −2.
 
−1
When λ = 3, (1) becomes
  
−2 2 1   x1 
   
  
 6 −4 0   x2  = 0.
  
−1 −2 −4 x3

i.e., − 2x1 + 2x2 + x3 = 0 (8)

ww 6x1 − 4x2 = 0 (9)

w.E −x1 − 2x2 − 4x3 = 0. (10)

a
(9) can be written as
syE 6x1 = 4x2

ngi
⇒ 3x1 = 2x2


x1
2
=
x2
3 nee
∴ x1 = 2, x2 = 3. rin
Substituting in (8) we obtain g.n
−4 + 6 + x3 = 0 et
2 + x3 = 0

⇒ x3 = −2.
 
 2 
 
∴ The third eigen vector is X3 =  3 .
 
−2
     
 1   1   2 
     
∴ The eigen vectors are  6  , −2 and  3 .
   
     
−13 −1 −2

Downloaded From : www.EasyEngineering.net


———-
Anna University
Downloaded Solved Question Papers
From : www.EasyEngineering.net 99

 
 8 −6 2 
 
11. (a) (ii) If the eigenvalues of A = −6 7 −4 are 0, 3, 15 find the
 
2 −4 3
eigenvectors of A and diagonalize the matrix A.
Solution.
The given eigen values are 0,3,15.
Step 1. To find the eigen vectors
 
 x1 
 
Let X =  x2  be the eigen vector corresponding to the eigen value λ.
 
ww x3

w.E 
∴ (A − λI)X = 0.
 

a 8 − λ

syE

 −6

−6 2   x1 
  
7 − λ −4   x2  = 0.
  
(1)
2
ngi
−4 3 − λ x3

When λ = 0 (1) becomes


   nee
 8 −6


2   x1 
   rin
−4  x2  = 0.
−6 7

2 −4 3 x3
  
g.n
The equations become
et
8x1 − 6x2 + 2x3 = 0
−6x1 + 7x2 − 4x3 = 0
2x1 − 4x2 + 3x3 = 0.
Taking the first two equations and applying the rule of cross
multiplication we get,

Downloaded From : www.EasyEngineering.net


———-
100
Downloaded Supplement to Engineering Mathematics - II
From : www.EasyEngineering.net

x1 x2 x3 x1 x2 x3
= = ⇒ = =
24 − 14 −12 + 32 56 − 36 10 20 20
 
1
 
∴ X1 = 2.
 
2
When λ = 3 (1) becomes
  
 5 −6 2   x1 
   

−6 4 −4  x2  = 0.
   
2 −4 0 x3

ww
The equations become

w.E
5x1 − 6x2 + 2x3 = 0
−6x1 + 4x2 − 4x3 = 0

a syE
2x1 − 4x2 + 0x3 = 0.
Taking the last two equations and applying the rule of cross
multiplication we get,
ngi
nee
rin
g.n
x1

x1
=
x2

x2
=
0 − 16 −8 24 − 8
x3

x3
et
= =
−16 −8 16
 
 2 
 
∴ X2 =  1 .
 
−2
When λ = 15 (1) becomes
  
−7 −6 2   x1 
   

−6 −8 −4   x2  = 0.
   
2 −4 −12 x3

Downloaded From : www.EasyEngineering.net


———-
Anna University
Downloaded Solved Question Papers
From : www.EasyEngineering.net 101

The equations become


−7x1 − 6x2 + 2x3 = 0
−6x1 − 8x2 − 4x3 = 0
2x1 − 4x2 − 12x3 = 0.
Taking the first two equations and applying the rule of cross
multiplication we get,

ww
w.E
a syE
x1
=
x2
=
x3
24 + 16 −12 − 28 56 − 36

ngi
x1
=
x2
=
40 −40 20
x3
nee
  rin
 2 
 
∴ X3 = −2.
g.n
 
1
     
et
1  2   2 
     
∴ The eigen vectors are 2 ,  1 and −2.
   
     
2 −2 1
Step 2. To find the modal matrix
The normalised eigen vectors are
h 1 2 2 iT h 2 1 −2 iT h 2 −2 1 iT
, ,
3 3 3 3 3 3  3 3 3
 1 2 2 
 3 3 3 
 
∴ N =  3 3 2 1 −2
3
.
 
2 −2 1 
3 3 3

Downloaded From : www.EasyEngineering.net


———-
102
Downloaded Supplement to Engineering Mathematics - II
From : www.EasyEngineering.net

Step 3. To find N T AN
   
 1 2 2 
2   31 32 2 
 3 3 3   8 −6 3 

 
N T AN =  32 −2    −2  
1
3 3   −6 7 −4  23 31 3  
  
2 1  1 
3
−2
3 3 2 −4 3 23 −2 3 3
    
 0 0   13 2 2 
 0 0 0 
0 3 
   3

=  2 −2   = 
1 −2  23 1
3 3   0 3 0  = D.
   
1  
10 −10 5 23 −2
3 3 0 0 15

11. (b) (i) Reduce the quadratic form 2x1 x2 + 2x2 x3 + 2x3 x1 into

ww
canonical form.
Solution.
w.E
Step 1. To find the matrix of the quadratic form

3 × 3 matrix.
a 
syE
Since given Q.F. contains 3 variables, the matrix of the Q.F. A is a


a11

Let A = a21
a12 a13 

a23 .
ngi

a31
a22
a32 a33

nee
Now, a11 = Coeff.of x12 = 0
rin
a12 = a21 =
1
2
1
× Coeff.of x1 x2 = × 2 = 1.
2 g.n
a13 = a31
1 1
= × Coeff.of x1 x3 = × 2 = 1
2 2
et
a22 = Coeff.of x22 = 0.
1 1
a23 = a32 = × Coeff.of x2 x3 = × 2 = 1.
2 2
a33 = Coeff.of x32 = 0.
 
0 1 1
 
∴ A = 1 0 1.
 
1 1 0

Downloaded From : www.EasyEngineering.net


———-
Anna University
Downloaded Solved Question Papers
From : www.EasyEngineering.net 103

Step 2. To find the eigen values


Since A is a 3 × 3 matrix, the characteristic equation is of the form
λ3 − s1 λ2 + s2 λ − s3 = 0, where

s1 = sum of the main diagonal elements


= 0 + 0 + 0 = 0.
s2 = sum of the minors of the main diagonal elements.

0 1 0 1 0 1
= + + = 0 − 1 + 0 − 1 + 0 − 1 = −3.
1 0 1 0 1 0

ww
0 1 1



w.E
s3 = |A| = 1 0 1 = 0 − 1(0 − 1) + 1(1 − 0) = 1 + 1 = 2.

1 1 0

a syE
∴ The characteristic equation is λ3 − 3λ − 2 = 0.

ngi
λ = −1 is a root. By synthetic division we get

−1 1 0 −3
nee−2
0
1
−1
−1
1
−2 rin
2
0
g.n
λ2 − λ − 2 = 0

(λ − 2)(λ + 1) = 0.
et
λ = 2, −1.

The eigen values are −1, −1, 2..


Step 3. To find the eigen vectors
 
 x1 
 
Let X =  x2  be the eigen vector corresponding to the eigen value λ.
 
x3

∴ (A − λI)X = 0.

Downloaded From : www.EasyEngineering.net


———-
104
Downloaded Supplement to Engineering Mathematics - II
From : www.EasyEngineering.net

  
−λ 1 1   x1 
   
 1 −λ 1   x2  = 0. (1)
   

1 1 −λ x3

When λ = 2 (1) becomes


  
−2 1 1   x1 
   

 1 −2 1   x2  = 0.
   
1 1 −2 x3

The equations become


ww −2x1 + x2 + x3 = 0

w.E
x1 − 2x2 + x3 = 0
x1 + x2 − 2x3 = 0.
a syE
Taking the first two equations and applying the rule of cross
multiplication we get

x1 ngi
x2 x3
1 1 -2 nee 1
-2 1 1 rin -2
x1 x2 x3 g.n
= =
1+2 1+2 4−1
x1 x2 x3
et
= =
3 3 3

=⇒ x1 = x2 = x3 = 1.
 
1
 
∴ X1 = 1.
 
1
  
1 1 1  x1 
   
When λ = −1, (1) becomes 1 1 1  x2  = 0.
   
1 1 1 x3

Downloaded From : www.EasyEngineering.net


———-
Anna University
Downloaded Solved Question Papers
From : www.EasyEngineering.net 105

The above three equations are equivalent to one equation

x1 + x2 + x3 = 0
Let x3 = 0.
∴ x1 = −x2
x1 x2
=
1 −1
⇒ x1 = 1, x2 = −1
 
 1 
 
∴ X2 = −1 .
ww  
0

w.E
 
a

a
 

syE
Let X3 = b orthogonal to X2
 
c


 
 
  1  ngi
∴ X3T X2 = 0 ⇒ a b c −1 = 0
 
  nee
0
rin
⇒ a−b=0
g.n
(2)

X3 also satisfy a + b + c = 0 et
(3)

From (2) we have a = b ⇒ a = b = 1.


Substituting in (3) we get 1 + 1 + c = 0 ⇒ c = −2.
 
 1 
 
∴ X3 =  1 .
 
−2
Step 4. To find the modal matrix
     
 √1   √1   √1 
 3   2   6 
The normalised eigen vectors are  √1 ,  −1√  ,  √1 .
 3   2   6 
 √1     −2 
3
0 √
6

Downloaded From : www.EasyEngineering.net


———-
106
Downloaded Supplement to Engineering Mathematics - II
From : www.EasyEngineering.net

The modal matrix N is formed with the normalised eigen vectors as


columns
 
 √1 √1 √1 
 3 2 6 

∴ N =  √1 −1
√ √1   .
 3 2 6 
 √1 0 −2


3 6

Step 5. To find N T AN
   
 √1 √1 √1 
 0 1 1  √13 √1 √1 
 3 3    6

3
  2

N T AN =  √1 −1
√ 0  1 0 1  √13 −1
√ √ 
1
6

 2    
ww
2 2 
 √1 √1 −2

 1 1 0  √1 0−2

6 6 6 3 6
    
w.E  √2
 3
=  −1
√
√2

√1
3
√2 
3
  √13
 
0   √13
√1

−1

2
√1 
 2 0
6
√1 
   0 

 = 0 −1 0  = D.

a  2
 −1
6
√ −1

2

6
syE √2
6
 
3
√1 0
2
−2

6

6

 
0 0 −1


Step 6. Reduction to canonical form


 
y1  ngi
 
Let X = NY, where Y = y2 
  nee
y3
rin
The given Q.F. is
Q = X T AX = (NY)T ANY = Y T N T ANY = Y T DY g.n

2 0

 
0  y1 

 
 

 
y1 
 
et
Y T DY = [y1 y2 y3 ] 0 −1 0  y2  = [y1 y2 y3 ] y2  = 2y21 − y22 − y23 .
     
0 0 −1 y3 y3

which is the required canonical form.


Step 7. To find the nature of the Q.F
Since the Q.F contains both positive and negative eigen values, it is
indefinite.  
1 −1 1
 
11. (b) (ii) Show that the matrix 0 1 0 satisfies its own
 
2 0 3

Downloaded From : www.EasyEngineering.net


———-
Anna University
Downloaded Solved Question Papers
From : www.EasyEngineering.net 107

characteristic equation. Find also its inverse.


Solution.
Step 1. To find the characteristic equation
 
1 −1 1
 
Let A = 0 1 0
 
2 0 3
Since A is a 3 × 3 matrix, the characteristic equation is of the form
λ3 − s1 λ2 + s2 λ − s3 = 0 where

s1 = sum of the main diagonal elemnets.


ww = 1 + 1 + 3 = 5.

w.E
s2 = sum of the minors of the main diagonal elments.

1 0 1 1 1 −1

= a + +
0 3 2 3 0 1 syE
= (3 − 0) + (3 − 2) + (1 − 0) = 3 + 1 + 1 = 5.

ngi
1 −1 1

s3 = |A| = 0 1 0 = 1(3 − 0) + 1(0 − 0) + 1(0 − 2) = 3 − 2 = 1.
nee

2 0 3

rin
g.n
Therefore the characteristic equation of A is λ3 − 5λ2 + 5λ − 1 = 0.
By Cayley-Hamilton theorem we get A3 − 5A2 + 5A − I = 0. et
Step 2. To verify Cayley Hamilton theorem

    
1 −1 1 1 −1 1 3 −2 4 
     
A = A × A = 0
2
1 0 0 1 0 = 0 1 0 
     
2 0 3 2 0 3 8 −2 11
    
1 −1 1 3 −2 4  11 −5 15
     
A3 = A × A2 = 0 1 0 0 1 0  =  0 1 0 
     
2 0 3 8 −2 11 30 −10 41

Downloaded From : www.EasyEngineering.net


———-
108
Downloaded Supplement to Engineering Mathematics - II
From : www.EasyEngineering.net

Now, A3 − 5A2 + 5A − I
       
11 −5 15 3 −2 4  1 −1 1 1 0 0
    
 
   
=  0 1

0  − 5 0 1

0  + 5 0 1 0 − 0 1 0

       
30 −10 41 8 −2 11 2 0 3 0 0 1
       
11 −5 15 −15 10 −20  5 −5 5  1 0 0
       
=  0 1 0  +  0 −5 0  +  0 5 0  − 0 1 0
       
30 −10 41 −40 10 −55 10 0 15 0 0 1
 
0 0 0
 
= 0 0 0 = 0
ww 
0 0 0


w.E
Hence Cayley Hamilton theorem verified.
Step 3. To find A−1
a syE
By Cayley Hamilton theorem, we have A3 − 5A2 + 5A − I = 0
Multiply by A−1 , we get A2 − 5A + 5I − A−1 = 0
ngi
∴A−1 = A2 − 5A + 5I nee

3
 −2 4 



1 −1 1






1 0
 rin 
0

= 0

1 0  − 5 0 1 0 + 5 0 1
  


0
 g.n

3

8 −2 11
 
−2 4   −5
 
2 0 3

5
 
−5  5
 
0 0 1
 
0 0  3
 
3 −1


et
= 0 1 0  +  0 −5 0  + 0 5 0 =  0 1 0 
       
8 −2 11 −10 0 −15 0 0 5 −2 −2 1

2.1.7 JUNE 2013 [R 2008]


Part A  
3 1 4
 
1. Find the eigenvalues of A−1 where A = 0 2 6.
 
0 0 5
Solution. Since A is triangular, the eigenvalues of A are the

Downloaded From : www.EasyEngineering.net


———-
Anna University
Downloaded Solved Question Papers
From : www.EasyEngineering.net 109

elements in the principal diagonal.


Hence, the eigen values of A are 3, 2, 5.
1 1 1
∴ The eigenvalues of A−1 are , , .
3 2 5
2. Write down matrix of the quadratic form
2x2 + 8z2 + 4xy + 10xz − 2yz.
Solution. If A is the matrix of the quadratic form, then
 
2 2 5 
 
A = 2 0 −1.
 
5 −1 8

ww
Part B
11. (a) (i)

Find the eigenvalues and eigenvectors of the matrix

2 0

A = 0 2
w.E 1

0.

Solution.

1 0 a2


syE
Step 1. To find the eigenvalues
ngi
nee
Since A is a 3 × 3 matrix, the characteristic equation is of the form
λ3 − s1 λ2 + s2 λ − s3 = 0, where
rin
s1 = sum of the main diagonal elements = 2 + 2 + 2 = 6.
g.n
s2 = sum of the minors of the main diagonal elements

=

2 0 2 1 2 0
+ +

= 4 + (4 − 1) + 4 = 11.
et
0 2 1 2 0 2

s3 = |A| = 2(4 − 0) + 1(0 − 2) = 8 − 2 = 6.

The characteristic equation is λ3 − 6λ2 + 11λ − 6 = 0


λ = 1 is a root. By synthetic division we have

1 1 −6 11 −6
0 1 −5 6
1 −5 6 0

Downloaded From : www.EasyEngineering.net


———-
110
Downloaded Supplement to Engineering Mathematics - II
From : www.EasyEngineering.net

λ2 − 5λ + 6 = 0
(λ − 2)(λ − 3) = 0
⇒ λ = 2, 3.
∴ The eigen values are λ = 1, 2, 3.
Step 2. To find the eigenvectors
 
 x1 
 
Let X =  x2  be the eigen vector corresponding to an eigen value λ.
 
x3
∴ [A − λI] X = 0
    
2 − λ 1   x1  0
ww 

 0 2−λ
0
    
0   x2  = 0.
    
(1)

w.E
When λ = 1 equation (1) becomes

1 0 2 − λ x3 0

a
1 0 1  x1 

 

syE
  
0 1 0  x2  = 0.
   

1 0 1 x3
 
ngi
The equations become
x1 + x3 = 0 ⇒ x1 = −x3 . nee
x2 = 0. rin
x1 + x3 = 0
g.n
If x1 = 1 then x3 = −1.
   
 x1   1 
   
et
∴ X1 =  x2  =  0 .
   
x3 −1
When λ = 3 equation (1) becomes
  
−1 0 1   x1 
  
 0 −1 0   x2  = 0.
   
  
1 0 −1 x3
The equations become
−x1 + x3 = 0.
−x2 = 0.

Downloaded From : www.EasyEngineering.net


———-
Anna University
Downloaded Solved Question Papers
From : www.EasyEngineering.net 111

x1 − x3 = 0.
If x1 = 1 then x3 = 1.
 
1
 
∴ X2 = 0.
 
1
When λ = 2 equation (1) becomes
  
0 0 1  x1 
  
0 0 0  x2  = 0.
   
  
1 0 0 x3
The equations become

ww x3 = 0.

w.E
x1 = 0.
x2 = any value say = 1.
 
a 0
 
∴ X3 = 1.
 
syE
0
    ngi
 
0
 1  1
    
nee
 .
∴ The eigen vectors are  0  , 0 and
1
   
−1 1
 
0 rin

 2 −1


2 
 g.n
11. (a) (ii) Show that the matrix A = −1 2

1 −1 2
 et
−1 satisfies its own

characteristic equation. Find also its inverse.


Solution.
Step 1. To find the characteristics equation
Since A is a 3 × 3 matrix, the characteristic equation is of the form
λ3 − s1 λ2 + s2 λ − s3 = 0, where

s1 = sum of its leading diagonal elements


= 2 + 2 + 2 = 6.
s2 = sum of the minors of its leading diagonal elements

Downloaded From : www.EasyEngineering.net


———-
112
Downloaded Supplement to Engineering Mathematics - II
From : www.EasyEngineering.net

2 −1 2 2 2 −1
= + +
−1 2 1 2 −1 2

= (4 − 1) + (4 − 2) + (4 − 1) = 3 + 2 + 3 = 8.

2 −1 2

s3 = |A| = −1 2 −1

1 −1 2

= 2(4 − 1) + 1(−2 + 1) + 2(1 − 2)


= 2(3) + 1(−1) + 2(−1) = 6 − 1 − 2 = 3.

ww
∴ The characteristic equation of A is λ3 − 6λ2 + 8λ − 3 = 0.

w.E
By Cayley Hamilton theorem, we have A3 − 6A2 + 8A − 3I = 0.
Step 2. Verification of Cayley Hamilton theorem.

 2

a −1 syE 
2   2 −1
 
 
2   7 −6 9 
 


A2 = A × A = −1 −1 −1 2 −1 = −5 6 −6

1 −1
2
2
 
ngi
1 −1 2
 
5 −5 7



 2
 −1

2   7 −6 nee
 
9   29 −28 38 

A = A × A = −1
3 2

2
 
−1 −5 6
  rin
 
−6 = −22 23 −28
 


1 −1 2 5 −5 7 22 −22 29
g.n
Now, A3 − 6A2 + 8A − 3I

 29 −28 38 




 7
 −6

9 

 2 −1 2 




1
 0 0
et

   
= −22 23 −28 − 6 −5
 
6 −6 + 8 −1 2 −1 − 3 0

1 0
       
22 −22 29 5 −5 7 1 −1 2 0 0 1
       
 29 −28 38   42 −36 54   16 −8 16  3 0 0
       
= −22 23 −28 − −30 36 −36 + −8 16 −8 − 0 3 0
       
22 −22 29 30 −30 42 8 −8 16 0 0 3
 
0 0 0
 
= 0 0 0 = 0
 
0 0 0

Downloaded From : www.EasyEngineering.net


———-
Anna University
Downloaded Solved Question Papers
From : www.EasyEngineering.net 113

∴ Cayley Hamilton theorem is verified.


Step 3. To find A−1
By Cayley Hamilton theorem we have A3 − 6A2 + 8A − 3I = 0.
Multiply by A−1 we get

A2 − 6A + 8I − 3A−1 = 0
3A−1 = A2 − 6A + 8I
     
 7 −6 9   2 −1 2  1 0 0
     
3A−1 = −5 6 −6 − 6 −1 2
 
−1 + 8 0

1 0
     

ww 
5 −5 7
 
1 −1 2
 
0 0 1

w.E  7 −6 9  −12





= −5 6 −6 +  6

 


 

 −12
6 −12 8
 
6  + 0
 
0 0
8 0


a 
 3 0 −3 syE
5 −5 7

−6

 3
6 −12

0 −3

0 0 8

3A−1

=  1 2 0  ⇒ A =  1


 ngi
−1 1
3 

2 0  .


−1 1 3
nee
−1 1 3

11. (b) Reduce the quadratic form 3x + 5y2 + 3z2 − 2xy − 2yz + 2zx into 2
rin
canonical form.
g.n
Solution. Refer question no. 11. (b) (i) of Nov/Dec. 2014 [2.1.2]
et
2.2 Unit II Vector Calculus

2.2.1 May/June 2016 [R 2013]


Part A

1. Evaluate ∇2 log r.
Solution. We have ~r = x~i + y~j + z~k
p
r = |~r| = x2 + y2 + z2 .
∴ r2 = x2 + y2 + z2 .

Downloaded From : www.EasyEngineering.net


———-
Downloaded From : www.EasyEngineering.net

ww
w.E
UNIT 2 asy
En
gin
ee
By EasyEngineering.net rin
g.n
et

Downloaded From : www.EasyEngineering.net


Downloaded From : www.EasyEngineering.net

2 Vector Calculus

2.1 Introduction

ww Scalar point function

w.E Let P be a point with position vector ~r in a region R. To each point P(~r) of the
region R in space, if there is a unique scalar or real number denoted by φ(~r), then

asy
φ is called a scalar point function in R. The region R is called a scalar field.
Vector point function

En
To each point P(~r) of a region R in space, if there exists a unique vector denoted
by F(~
field. gin
~ r), then F~ is called a vector point function in R. The region R is called a vector

Derivative of a vector function


ee
The derivative of a vector function f~(t) at t denoted by
d f~ f~(t + ∆t) − f~(t)
d f~
dt
is defined as
rin
= lim
dt ∆t→0 ∆t
if the limit exists. It is also denoted by f~′ (t).
g.n
Results
(i) If f~(t) = f1 (t)~i + f2 (t)~j + f3 (t)~k, then ~f ′ (t) = f1′ (t)~i + f2′ (t)~j + f3′ (t)~k.

(ii) The higher derivatives of f~ are 2 , 3 , . . .


d2 f~ d3 f~
et
dt dt
d~c ~
(iii) If ~c is a constant vector, then = 0.
dt

Downloaded From : www.EasyEngineering.net


Downloaded From : www.EasyEngineering.net

166 Engineering Mathematics - II

Geometrical meaning of the derivative

Q
Let P be a point on a curve with
→ →
position vector ~r. Let Q be an adjacent → ∆r ∆r
r +

point with position vector ~r + ∆~r. Then,


d~r ∆~r
= lim , which is along the tangent O

P
dt ∆t→0 ∆t r
at P.

ww ∴
d~r
dt
denotes the tangent vector at P.
d~r
dt

w.E
The unit tangent vector is d~r
.
| dt |

Differentiation formulae

asy
Let f~ and ~g be two differentiable vector functions. Then the following are easy
to derive.
(i)
d ~  d f~ d~g
f ± ~g = ± . En
dt dt dt

gin
(ii) If φ is a scalar point function, then
d ~
dt
φf = φ
d f~ dφ ~
dt
+
dt
f.

(iii)

(iv)
d ~ 
dt
d ~ 
f · ~g =

f × ~g =
f~ ·

f~ ×
d~g d f~
dt
+

+
dt
d~g d f~
· ~g.
ee
× ~g. rin
g.n
dt dt dt
d  ~ ~ d ~   d f~ ~   ~ d~g ~   ~ d~h 
(v) f ~g h = f · ~g × ~h = ~g h + f h + f ~g .
dt dt dt dt dt
(vi) If f~ is a function of the scalar s and s is a function of t then
Level Surfaces
d f~ d f~ ds
dt
=
ds dt
.
et
Let φ be a continuous scalar point function defined in the region R. The set of all
points satisfying the equation φ(x, y, z) = c defines a surface called the level surface.

Downloaded From : www.EasyEngineering.net


Downloaded From : www.EasyEngineering.net

Vector Calculus 167

2.2 Gradient of a scalar point function

The definition of ∇
The Hamiltonian operator or the vector differential operator denoted by ∇ is
∂ ∂ ∂
defined as ∇ = ~i + ~j + ~k .
∂x ∂y ∂z
If φ(x, y, z) is a scalar point function, continuously differentiable in a given region
R, then the gradient of φ is defined as

∂φ ~ ∂φ ~ ∂φ

ww grad φ = ∇φ = ~i
∂x
+j
∂y
+k .
∂z

w.E
Geometrical meaning of ∇φ
Let ~r = x~i + y~j + z~k be the position vector of any point P on the level surface

asy
φ(x, y, z) = c. Now,

d~r = dx~i + dy~j + dz~k,

En
which is a tangent to the surface at P. Also,
 ∂φ
∇φ · d~r = ~i
∂x
+ ~j
gin
∂φ ~ ∂φ   ~
∂y
+k
∂z
· dxi + dy~j + dz~k


=
∂φ
∂x
dx +

= dφ = 0
∂φ
∂y
dy +

∂φ
∂zee
dz

Since φ = c .

rin
∴ ∇φ is perpendicular to d~r. g.n
But d~r is the tangent vector at P. Hence ∇φ is normal at P.

∇φ
|∇φ|
is the unit normal vector at P.
et
2.3 Directional derivative

The directional derivative of a scalar point function φ in a given direction ~a


is the rate of change of φ in the direction of ~a. It is the component of ∇φ in the

Downloaded From : www.EasyEngineering.net


Downloaded From : www.EasyEngineering.net

168 Engineering Mathematics - II

∇φ · ~a
direction of ~a, which is given by .
|~a|
∇φ · ~a ~a
= ∇φ ·
|~a| |~a|

~a
= |∇φ| × × cos θ, where θ is the angle between ∇φ and ~a.
|~a|

= |∇φ| cos θ.

Note

ww 1. The directional derivative at a given point is maximum when cos θ is

w.E maximum. The maximum value of cos θ is 1. i.e. when θ = 0.


Therefore, Maximum directional derivative = |∇φ|.

asy
2. The directional derivative is minimum when cos θ = −1. i.e. when θ = π.

En
Therefore, Minimum directional derivative = −|∇φ|.

gin
Equation of the tangent plane
Let φ(x, y, z) = c be a level surface and A be a given point with position vector ~r0 .

ee
Let P be any point on the tangent plane at A with position vector ~r. Let

~r = x~i + y~j + z~k.


rin
~r0 = x0~i + y0 ~j + z0~k.
~ = ~r − ~r0 = (x − x0 )~i + (y − y0 )~j + (z − z0 )~k.
Now, AP g.n
~ = ~r − ~r0 lies in the tangent plane at A.
Since ∇φ is normal at A, AP
Therefore, (~r − ~r0 ) · ∇φ = 0.
et
∂φ ∂φ ∂φ
i.e., (x − x0 ) + (y − y0 ) + (z − z0 ) = 0, which is the equation of the tangent
∂x ∂y ∂z
plane at A(~r0 ). The partial derivatives are evaluated at (x0 , y0 , z0 ).
Equation of the normal
If P(~r) is any point on the normal at A, then ∇φ and ~r − ~r0 are parallel.
Therefore, the equation of the normal at A is (~r − ~r0 ) × ∇φ = ~0.

Downloaded From : www.EasyEngineering.net


Downloaded From : www.EasyEngineering.net

Vector Calculus 169

x − x0 y − y0 z − z0
It’s cartesian equation is ∂φ
= ∂φ
= ∂φ
, the partial derivatives are to be
∂x ∂y ∂z
evaluated at A(x0 , y0 , z0 ).
Angle between two surfaces
Let f (x, y, z) = c1 & g(x, y, z) = c2 be two surfaces. Let P be a common point. The
angle between the two surfaces is equal to the angle between the normals at P. If
θ is the angle between the normals, then

∇ f · ∇g
cos θ = .

ww |∇ f ||∇g|

If the surfaces are perpendicular or orthogonal, then ∇ f · ∇g = 0.

w.E
The surfaces touch each other when θ = 0.
Properties of ∇φ

asy
Let f and g be two scalar point functions and c is a constant. Then
(i) ∇c = ~0.
(ii) ∇(c f ) = c∇ f .
En
(iii) ∇( f ± g) = ∇ f ± ∇g.
(iv) ∇( f g)
! = f ∇g + g∇ f . gin
(v) ∇
Proof.
f
g
=

∂c
g∇ f − f ∇g
g2

∂c ∂c
, g , 0.
ee "
∂c ∂c ∂c
#
rin
(i) ∇c = ~i + ~j + ~k = ~0. ∵ c is a constant,
∂x
(ii) ∇c f = ~i
∂(c f
∂y
)
+ ~j
∂(c
∂z
f )
+ ~k
∂(c f )
= c~i
∂ f
+ c~j
∂ f
=
∂x ∂y ∂z
+ c~k
∂f
= =0 .

g.n
= c ~i

∂x
f
∂x
+ ~j


∂y
f
∂y !
+ ~k

∂z
f
∂z
= c∇ f .

(iii)∇( f ± g) = ~i ( f ± g) + ~j ( f ± g) + ~k ( f ± g)
∂x


∂y ∂z

et
∂x ∂y ∂z
" # " # " #
~ ∂ f ∂g ~ ∂ f ∂g ~ ∂ f ∂g
=i ± +j ± +k ±
∂x ∂x ∂y ∂y ∂z ∂z
!
∂ f ∂ f ∂ f ∂g ∂g ∂g
= ~i + ~j + ~k ± ~i + ~j + ~k
∂x ∂y ∂z ∂x ∂y ∂z
= ∇ f ± ∇g.

Downloaded From : www.EasyEngineering.net


Downloaded From : www.EasyEngineering.net

170 Engineering Mathematics - II

∂ ∂ ∂
(iv)∇( f g) = ~i ( f g) + ~j ( f g) + ~k ( f g)
∂x ∂y ∂z
" # " # " #
~ ∂g ∂f ~ ∂g ∂f ~ ∂g ∂f
=i f +g +j f +g +k f +g
∂x ∂x ∂y ∂y ∂z ∂z
! !
∂g ∂g ∂g ∂ f ∂ f ∂ f
= f ~i + ~j + ~k + g ~i + ~j + ~k
∂x ∂y ∂z ∂x ∂y ∂z
= f ∇g + g∇ f.
! ! ! !
f ~ ∂ f ~ ∂ f ~ ∂ f
(v)∇ =i +j +k
g ∂x g ∂y g ∂z g
 ∂f
 g − f ∂g 
 ∂f
 g ∂z − f ∂g
 ∂f 
∂g

ww
 
 g − f  ∂y ∂y ∂z

= ~i  ∂x 2 ∂x  + ~j   + ~k  
g g2 g2
 
 

w.E g ~i ∂∂xf + ~j ∂∂yf + ~k ∂∂zf f ~i ∂g ~ ∂g ~ ∂g


h i h i
∂x + j ∂y + k ∂z
= −
g2 g2
g∇ f f ∇g
= 2 − 2
g
g∇ f − f ∇g
g
.
asy
En
=
g2 ✎ ☞
Worked Examples
✍ ✌

gin
Example 2.1. Find |∇φ| if φ = 2xz4 − x2 y at (2, −2, 1). [Jan 2009]
Solution. Given φ = 2xz4 − x2 y.
∂φ
∂x
∂φ
= 2z4 − 2xy. ee rin
= −x2 .
∂y
∂φ
= 8xz3 . g.n
∂z

∇φ = ~i
∂φ ~ ∂φ ~ ∂φ ~ 4
∂x
+j
∂y
+k
∂z
= i(2z − 2xy) + ~j(−x2 ) + ~k(8xz3 ). et
(∇φ)(2,−2,1) = ~i(2(1) − 2(2)(−2)) − 4~j + ~k8(2)1 = 10~i − 4~j + 16~k
√ √
|∇φ| = 100 + 16 + 256 = 372.

Example 2.2. Find grad φ at (1, −2, 1) if φ = 3x2 y − y3 z2 . [Jan 2006]


Solution. Given φ = 3x2 y − y3 z2 .

Downloaded From : www.EasyEngineering.net


Downloaded From : www.EasyEngineering.net

Vector Calculus 171

∂φ
= 6xy.
∂x
∂φ
= 3x2 − 3y2 z2 .
∂y
∂φ
= −2y3 z.
∂z
∂φ ~ ∂φ ~ ∂φ ~
grad φ = ~i +j +k = i(6xy) + ~j(3x2 − 3y2 z2 ) + ~k(−2y3 z).
∂x ∂y ∂z

(grad φ)(1,−2,1) = −12~i + ~j(−9) + 16~k = −12~i − 9~j + 16~k.

ww Example 2.3.
Solution. Given φ = + +
If φ = x2 + y2 + z2 , find ▽φ at (1,1,-1).
x2 y2 z2 .
[Jan 2005]

w.E We have ▽φ = ~i
∂φ ~ ∂φ ~ ∂φ
∂x
+j
∂y
+k
∂z
= 2x~i + 2y~j + 2z~k.

asy
(▽φ)(1,1,−1) = 2~i + 2~j − 2~k.

En
Example 2.4. Find the directional derivative of φ = xyz at (1, 1, 1) in the direction
of ~i + ~j + ~k.
Solution. Given φ = xyz. gin [Jun 2013]

∇φ = ~i
∂φ ~ ∂φ ~ ∂φ
∂x
+j
∂y
+k
∂z
ee rin
= yz~i + xz~j + xy~k.

(∇φ)(1,1,1) = ~i + ~j + ~k. g.n


Let ~a = ~i + ~j + ~k.

|~a| = 1 + 1 + 1 = 3.
√ et
~a
Directional derivative of φ in the direction of ~a = ∇φ ·
|~a|
(~i + ~j + ~k)
= (~i + ~j + ~k) · √
3
1+1+1 3 √
= √ = √ = 3.
3 3

Downloaded From : www.EasyEngineering.net


Downloaded From : www.EasyEngineering.net

172 Engineering Mathematics - II

Example 2.5. Find the directional derivative of φ = xy + yz + zx in the direction of


the vector ~i + 2~j + 2~k at (1, 2, 0). [Jun 2009]
Solution. Given φ = xy + yz + zx.
∂φ ~ ∂φ ~ ∂φ
∇φ = ~i +j +k
∂x ∂y ∂z
= (y + z)i + (x + z)~j + (y + x)~k.
~
(∇φ)(1,2,0) = 2~i + ~j + 3~k.
Let ~a = ~i + 2~j + 2~k.
√ √
|~a| = 1 + 4 + 4 = 9 = 3.

ww The directional derivative of φ in the direction of ~a = ∇φ.


~a
|~a|

w.E = (2~i + ~j + 3~k).


(~i + 2~j + 2~k) 2 + 2 + 6 10
3
=
3
= .
3

asy
Example 2.6. Find the directional derivative of φ = xy + yz + zx at the point (1,2,3)
in the direction 3~i + 4~j + 5~k. [Apr 2003]
Solution. Given φ = xy + yz + zx.
En
▽φ = ~i
∂φ ~ ∂φ ~ ∂φ
∂x
+j
∂y
+k
∂z
gin
ee
= (y + z)~i + (x + z)~j + (x + y)~k.

(▽φ)(1,2,3) = (2 + 3)~i + (1 + 3)~j + (1 + 2)~k

= 5~i + 4~j + 3~k. rin


~a = 3~i + 4~j + 5~k.
g.n
et
√ √ √
~a = 9 + 16 + 25 = 50 = 5 2.
~a
Directional derivative in the direction of ~a = ▽φ.
~a
(3~i + 4~j + 5~k)
= (5~i + 4~j + 3~k). √
5 2
15 + 16 + 15
= √
5 2
46
= √ .
5 2

Downloaded From : www.EasyEngineering.net


Downloaded From : www.EasyEngineering.net

Vector Calculus 173

Example 2.7. Find the directional derivative of φ(x, y, z) = x2 yz + 4xz2 at the point
(1, −2, −1) in the direction of the vector 2~i − ~j − 2~k.
Solution. Given φ(x, y, z) = x2 yz + 4xz2 .
∂φ ~ ∂φ ~ ∂φ
∇φ = ~i +j +k
∂x ∂y ∂z
= (2xyz + 4z2 )~i + x2 z~j + (x2 y + 8xz)~k.

(∇φ)(1,−2,−1) = (2(1)(−2)(−1) + 4)~i − ~j + (−2 − 8)~k

= 8~i − ~j − 10~k.

ww Let ~a = 2~i − ~j − 2~k.


w.E |~a| = 4 + 1 + 4 = 3.
~a 1
= (2~i − ~j − 2~k).
|~a| 3

asy
The directional derivative of φ in the direction of ~a = ∇φ ·
~a 1 37
= (16 + 1 + 20) = .

En |~a| 3
Example 2.8. Find the directional derivative of φ = x2 yz + 4xz2 at the point
3

Solution. Given φ = x2 yz + 4xz2 . gin


P(1, −2, −1) in the direction of PQ where Q is (3, −3, −3). [Jun 2008]

 ∂φ 
ee
∂φ
∂x
= 2xyz + 4z2 .
rin
∂x (1,−2,−1)
∂φ
= 2 · 1 · (−2)(−1) + 4(−1)2 = 4 + 4 = 8.

= x2 z g.n
∂φ
 ∂φ 
∂y
∂y

(1,−2,−1)
= 1(−1) = −1. et
= x2 y + 8xz
∂z
 ∂φ 
= 1(−2) + 8 · 1(−1) = −10.
∂z (1,−2,−1)
 ∂φ ∂φ ~ ∂φ 
(∇φ)(1,−2,−1) = ~i + ~j +k
∂x ∂y ∂z (1,−2,−1)
= 8~i − ~j − 10~k.

Downloaded From : www.EasyEngineering.net


Downloaded From : www.EasyEngineering.net

174 Engineering Mathematics - II

−−→
~a = PQ = 3~i − 3~j − 3~k − (~i − 2~j − ~k)

= 2~i − ~j − 2~k.

|~a| = 4 + 1 + 4 = 3.
~a 1
= (2~i − ~j − 2~k).
|~a| 3
~a 1 37
Directional derivative = ∇φ · = (16 + 1 + 20) = .
|~a| 3 3

Example 2.9. Find the maximum value of the directional derivative of φ = x3 yz at

ww (1, 4, 1).
Solution. Given φ = x3 yz.

w.E ∂φ
∂x
= 3x2 yz.

∂x asy
 ∂φ 
(1,4,1)
∂φ
= 3 · 1 · 4 · 1 = 12.

 ∂φ  En
∂y
= x3 z.

∂y
gin
(1,4,1)
∂φ
= 1 · 1 = 1.

 ∂φ 
∂z
∂z

(1,4,1)
ee
= x3 y.

= 1 · 4 = 4.
rin
g.n
∂φ ~ ∂φ ~ ∂φ
(∇φ)(1,4,1) = ~i +j +k = 12~i + ~j + 4~k.
∂x ∂y ∂z

Maximum value of directional derivative = |∇φ| =


√ √
144 + 1 + 16 = 161.

Example 2.10. In what direction from the point (1, 1, −2) is the directional
et
derivative of φ = x2 − 2y2 + 4z2 maximum. Also find the maximum directional
derivative.
Solution. Given φ = x2 − 2y2 + 4z2 .
∂φ ~ ∂φ ~ ∂φ
∇φ = ~i +j +k = 2x~i − 4y~j + 8z~k.
∂x ∂y ∂z
(∇φ)(1,1,−2) = 2~i − 4~j − 16~k.

Downloaded From : www.EasyEngineering.net


Downloaded From : www.EasyEngineering.net

Vector Calculus 175

The directional derivative is maximum in the direction of ∇φ = 2~i − 4~j − 16~k.

Maximum value of the directional derivative = |∇φ|



= 4 + 16 + 256

= 276

= 4 × 69

= 2 69.

ww Example 2.11. In what direction from the point (1,-1,-2) is the directional
derivative of φ = x3 y3 z3 a maximum? What is the magnitude of this maximum.

w.E
Solution. Given φ = x3 y3 z3 .
[May 2005]

▽φ = ~i
∂x
+j
∂y
asy
∂φ ~ ∂φ ~ ∂φ
+k
∂z

En
= 3x2 y3 z3~i + 3x3 y2 z3 ~j + 3x3 y3 z2~k.
(▽φ)(1,−1,−2) = 3 × 1 × (−1) × (−8)~i + 3 × 1 × 1 × (−8)~j + 3 × 1 × (−1) × 4~k

gin
= 24~i − 24~j − 12~k.

ee
p
|▽φ| = (24)2 + (−24)2 + (−12)2

= 576 + 576 + 144

rin
= 1296

= 36. g.n
et
The directional derivative is maximum in the direction of 24~i − 24~j − 12~k and its
maximum value is 36.

Example 2.12. Find a unit normal vector to the surface x3 +y3 +3xyz = 3 at (1, 2, −1).
Solution. Given φ = x3 + y3 + 3xyz − 3.

∂φ
= 3x2 + 3yz.
∂x
 ∂φ 
= 3 − 6 = −3.
∂x (1,2,−1)

Downloaded From : www.EasyEngineering.net


Downloaded From : www.EasyEngineering.net

176 Engineering Mathematics - II

∂φ
= 3xz + 3y2 .
∂y
 ∂φ 
= −3 + 12 = 9.
∂y (1,2,−1)
∂φ
= 3xy.
∂z
 ∂φ 
= 6.
∂z (1,2,−1)
∂φ ~ ∂φ ~ ∂φ
(∇φ) = ~i +j +k
∂x ∂y ∂z

ww = −3~i + 9~j + 6~k.



|∇φ| = 9 + 81 + 36

w.E Unit normal vector =


√ √ √
= 126 = 9 × 14 = 3 14.
∇φ 3(−~i + 3~j + 2~k)

asy
= √
|∇φ| 3 14
−~i + 3~j + 2~k
.

En
= √
14

gin
Example 2.13. Find a unit normal vector to the surface x2 y + 2xz2 = 8 at (1, 0, 2).
Solution. Given φ = x2 y + 2xz2 − 8.

(∇φ) = ~i
∂φ ~ ∂φ ~ ∂φ
∂x
+j
∂y
+k .
∂zee
= (2xy + 2z2 )~i + x2 ~j + 4xz~k rin
(∇φ)(1,0,2) = 8~i + ~j + 8~k.
g.n

|∇φ| = 64 + 1 + 64 = 129.

Unit normal vector =


∇φ
|∇φ|

et
8~i + ~j + 8~k
= √ .
129

Example 2.14. Find the angle between the surfaces x2 +y2 +z2 = 9 and x2 +y2 −z = 3
at the point (2, −1, 2).
Solution. Let the given surfaces be f and g.

Downloaded From : www.EasyEngineering.net


Downloaded From : www.EasyEngineering.net

Vector Calculus 177

∴ f = x2 + y2 + z2 − 9 and g = x2 + y2 − z − 3.

∂ f ~∂ f ~ ∂ f
∇ f = ~i +j +k = 2x~i + 2y~j + 2z~k.
∂x ∂y ∂z
(∇ f )(2,−1,2) = 4~i − 2~j + 4~k.
√ √
|∇ f | = 16 + 4 + 16 = 36 = 6.
∂g ∂g ∂g
∇g = ~i + ~j + ~k = 2x~i + 2y~j − ~k.
∂x ∂y ∂z
(∇g)(2,−1,2) = 4~i − 2~j − ~k.

ww √
|∇g| = 16 + 4 + 1 = 21.

w.E
Let θ be the angle between the surfaces. Then

∇ f · ∇g 16 + 4 − 4 8

asy
cos θ =
|∇ f ||∇g|
=
 8 
θ = cos−1 √ .

6 21
= √ .
3 21

En 3 21

Example 2.15.
at the point (1,1,1). gin
Find the angle between the surfaces x log z = y2 − 1 and x2 y = 2 − z
[May 2001]

Given φ1 = x log z − y2 + 1.
ee
Solution. Let φ1 and φ2 be the given two surfaces.

rin
φ2 = x2 y − 2 + z.
▽φ1 = ~i
∂φ1 ~ ∂φ1 ~ ∂φ1
∂x
+j
∂y
+k
∂z g.n
x
= log z~i − 2y~j + ~k.

(▽φ1 )(1,1,1) = 0 × ~i − 2~j + ~k.


z et
= −2~j + ~k.
∂φ2 ~ ∂φ2 ~ ∂φ2
▽φ2 = ~i +j +k
∂x ∂y ∂z
= 2xy~i + x2 ~j + ~k

(▽φ2 )(1,1,1) = 2~i + ~j + ~k.

Downloaded From : www.EasyEngineering.net


Downloaded From : www.EasyEngineering.net

178 Engineering Mathematics - II

Let θ be the acute angle between the two surfaces.


▽φ1 .▽φ2

∴ cos θ =
|▽φ1 | |▽φ2 |

−2 + 1
= √ √
4 + 1 4 + 1 + 1
1 1
= √ √ = √
5 6 30
!
1
∴ θ = cos−1 √ .
30

ww Example 2.16. Show that the surfaces 5x2 − 2yz − 9x = 0 and 4x2 y + z3 − 4 = 0 are
orthogonal at (1, −1, 2).

w.E
Solution. Let the given surfaces be f and g.
∴ f = 5x2 − 2yz − 9x and g = 4x2 y + z3 − 4.

asy
∇ f = ~i
∂ f ~∂ f ~ ∂ f
∂x
+j
∂y
+k
∂z
= (10x − 9)~i + (−2z)~j − 2y~k.

En
(∇ f )(1,−1,2) = ~i − 4~j + 2~k.
∂g
gin
∂g
∇g = ~i + ~j + ~k
∂x ∂y
∂g
∂z
= 8xy~i + 4x2 ~j + 3z2~k.

ee
(∇g)(1,−1,2) = −8~i + 4~j + 12~k.

∇ f.∇g = (~i − 4~j + 2~k).(−8~i + 4~j + 12~k)


rin
= −8 − 16 + 24 = 0.

g.n
∴ The normals to the surfaces are orthogonal.
=⇒ The given two surfaces are orthogonal. et
Example 2.17. Find the angle between the normals to the surface xy = z2 at
the points (1,4,2) and (-3,-3,3). [May 2011, Jan 2004]
Solution. Given φ = xy − z2 .
Let ~n1 and ~n2 be the normals to the surface φ at (1,4,2) and (-3,-3,3)
respectively.

Downloaded From : www.EasyEngineering.net


Downloaded From : www.EasyEngineering.net

Vector Calculus 179

∂φ ~ ∂φ ~ ∂φ
▽φ = ~i +j +k = y~i + x~j − 2z~k.
∂x ∂y ∂z
~n1 = (▽φ)(1,4,2) = 4~i + ~j − 4~k.

~n2 = (▽φ)(−3,−3,3) = −3~i − 3~j − 6~k.


Let θ be the angle between the two normals.
~n1 .~n2
∴ cos θ = .
~n1 ~n2
−12 − 3 + 24 9 9
= √ √ = √ √ = √
16 + 1 + 16 9 + 9 + 36 33 54 33 × 54

ww = √
9
3 × 11 × 6 × 9
= √
9
81 × 22
= √
1
22

w.E
!
1
∴ θ = cos−1 √ .
22
Example 2.18. Find a and b if the surfaces ax2 − byz = (a + 2)x and 4x2 y + z3 = 4 cut
orthogonally at (1, −1, 2).
asy
Solution. Let the given surfaces be f and g.

En
∴ f = ax2 − byz − (a + 2)x and g = 4x2 y + z3 − 4.

gin
∇ f = (2ax − (a + 2))~i + ~j(−bz) − by~k.

ee
(∇ f )(1,−1,2) = (2a − a − 2)~i − 2b~j + b~k.

= (a − 2)~i − 2b~j + b~k.

∇g = ~i
∂g ~ ∂g ~ ∂g rin
g.n
+ j +k .
∂x ∂y ∂z
= 8xy~i + 4x2 ~j + 3z2~k.

(∇g)(1,−1,2) = −8~i + 4~j + 12~k.

Given that the surfaces are orthogonal.


et
∴ ∇ f · ∇g = 0

−8(a − 2) − 8b + 12b = 0

−8a + 16 + 4b = 0

2a − b = 4. (1)

Downloaded From : www.EasyEngineering.net


Downloaded From : www.EasyEngineering.net

180 Engineering Mathematics - II

(1, −1, 2) is a point on the surface f = 0.

∴ a + 2b = a + 2

2b = 2 ⇒ b = 1.

subtitute b = 1 in (1) we get


5
a= .
2

ww Example 2.19. Find the values of a and b so that the surfaces ax3 −by2 z = (a+3)x2
and 4x2 y − z3 = 11 may cut orthogonally at (2,-1,-3). [Dec 2013, June 2006]

w.E
Solution. Let φ1 and φ2 be the given surfaces.
∴ φ1 = ax3 − by2 z − (a + 3)x2 .

φ2 = 4x2 y − z3 − 11.
asy
∂φ1 ~ ∂φ1 ~ ∂φ1
▽φ1 = ~i
∂x
+j
∂y
+k
∂z
En
gin
= [3ax2 − 2(a + 3)x]~i − 2byz~j − by2~k
(▽φ1 )(2,−1,−3) = (12a − 4(a + 3))~i − 6b~j − b~k

▽φ2 = ~i
∂x
+j
∂y
ee
= (8a − 12)~i − 6b~j − b~k.
∂φ2 ~ ∂φ2 ~ ∂φ2
+k
∂z rin
▽φ2 = 8xy~i + 4x2 ~j − 3z2~k.
g.n
(▽φ2 )(2,−1,−3) = −16~i + 16~j − 27~k.
Given that the surfaces are orthogonal.
∴ ▽φ1 .▽φ2 = 0
et
−16(8a − 12) + 16(−6b) + 27b = 0

−128a + 192 − 96b + 27b = 0

−128a − 69b + 192 = 0


128a + 69b = 192. (1)
(2, −1, −3) lies on φ1 .

Downloaded From : www.EasyEngineering.net


Downloaded From : www.EasyEngineering.net

Vector Calculus 181

∴ 8a + 3b − 4(a + 3) = 0

8a + 3b − 4a − 12 = 0
4a + 3b = 12. (2)
(2) × 23 ⇒ 92a + 69b = 276. (3)
(1) − (3) ⇒ 36a = −84
−84 −7
a= = .
36 3
Substituting in (2) we get

ww
!
7
4 − + 3b = 12
3
28

w.E 3b = 12 +

3b =
3
3
36 + 28 64
=
3

asy b= .
64
9

En
Example 2.20. Find the directional derivative of the function φ = xy2 + yz3 at the
point (2, −1, 1) in the direction of the normal to the surface x log z − y2 − 4 = 0 at the
point (−1, 2, 1).
gin [Jan 2008]
Solution. Let φ = xy2 + yz3 .
ee
∇φ = ~iy2 + (2xy + z3 )~j + 3yz2~k.
rin
(∇φ)(2,−1,1) = ~i − 3~j − 3~k.

Let f = x log z − y2 − 4. g.n


x
∇ f = ~i log z + ~j(−2y) + ~k .

(∇ f )(−1,2,1) = −4~j − ~k = ~a.


z et
~a 0 + 12 + 3 15
Directional Derivative = ∇φ · = √ = √ .
|~a| 17 17
Example 2.21. Find the directional derivative of φ = x2 y2 z2 at the point (1,1,1)
along the normal to the surface x2 + 2xy − z2 at the point (1,1,1). [May 2002]
Solution. Given φ = x2 y2 z2 .

Downloaded From : www.EasyEngineering.net


Downloaded From : www.EasyEngineering.net

182 Engineering Mathematics - II

∂φ ~ ∂φ ~ ∂φ
▽φ = ~i +j +k
∂x ∂y ∂z
= 2xy2 z2~i + 2x2 yz2 ~j + 2x2 y2 z~k.

(▽φ)(1,1,1) = 2~i + 2~j + 2~k.


Let φ1 = x2 + 2xy − z2 .
∂φ1 ~ ∂φ1 ~ ∂φ1
▽φ1 = ~i +j +k
∂x ∂y ∂z
= (2x + 2y)~i + 2x~j + (−2z)~k.

ww (▽φ1 )(1,1,1) = 4~i + 2~j − 2~k.


Let ~n be the unit normal vector to the surface φ1 .
4~i + 2~j − 2~k

w.E
▽φ1
∴ ~n = = √
|▽φ1 | 16 + 4 + 4
4~i + 2~j − 2~k 4~i + 2~j − 2~k

asy
= √ = √
24 2 6
2~i + ~j − ~k

En
= √
6
Directional derivative of φ along ~n = ▽φ.~n

gin = (2~i + 2~j + 2~k).


(2~i + ~j − ~k)

ee =
4+2−2

4

6
.
6

rin
= √ .
6
g.n
Example 2.22. Find φ if ∇φ = (6xy + z3 )~i + (3x2 − z)~j + (3xz2 − y)~k.
Solution. We have ∇φ = ~i
∂φ ~ ∂φ ~ ∂φ
∂x
+j
∂y
+k .
∂z
et
But it is given that ∇φ = (6xy + z3 )~i + (3x2 − z)~j + (3xz2 − y)~k
Comparing the two values of ∇φ we obtain

∂φ
= 6xy + z3 . (1)
∂x
∂φ
= 3x2 − z. (2)
∂y

Downloaded From : www.EasyEngineering.net


Downloaded From : www.EasyEngineering.net

Vector Calculus 183

∂φ
= 3xz2 − y. (3)
∂z
Integrating (1), (2) and (3) partially w.r.to. x, y and z respectively we get,
x2
φ = 6y + xz3 + f1 (y, z)
2
i.e., φ = 3x2 y + xz3 + f1 (y, z). (4)

φ = 3x2 y − zy + f2 (x, z). (5)


z3
and φ = 3x − yz + f3 (x, y)

ww 3
i.e., φ = xz3 − yz + f3 (x, y). (6)

w.E From (4), (5) and (6) we obtain, φ = 3x2 y + xz3 − yz + c.

Example 2.23. If ∇φ = 2xyz3~i + x2 z3 ~j + 3x2 yz2~k, find φ if φ(1, −2, 2) = 4.


Solution. We have ∇φ = ~i
∂x asy
∂φ ~ ∂φ ~ ∂φ
+j
∂y
+k .
∂z

En
But it is given that ∇φ = 2xyz3~i + x2 z3 ~j + 3x2 yz2~k

∂φ
= 2xyz3 .
gin
Comparing the two values of ∇φ we obtain

ee
(1)
∂x
∂φ
∂y
∂φ
= x 2 z3 .
rin (2)

∂z
= 3x2 yz2 .
g.n (3)

Integrating (1), (2) and (3) partially w.r.to. x, y and z respectively we get,

φ = 2yz 3
Z
xdx + f1 (y, z)
et
x2
= 2yz3 + f1 (y, z)
2
= x2 yz3 + f1 (y, z). (4)
Z
2 3
Also, φ = x z dy + f2 (x, z)

φ = x2 yz3 + f2 (x, z). (5)

Downloaded From : www.EasyEngineering.net


Downloaded From : www.EasyEngineering.net

184 Engineering Mathematics - II

Z
Again, φ = 3x2 y z2 dz + f3 (x, y)

z3
φ = 3x2 y + f3 (x, y)
3
φ = x2 yz3 + f3 (x, y) (6)

From (4), (5) and (6) we obtain,

φ = x2 yz3 + c.

Given that φ(1, −2, 2) = 4 ⇒ 1(−2)8 + c = 4 ⇒ −16 + c = 4 ⇒ c = 20.

ww ∴ φ = x2 yz3 + 20.

w.E
Example 2.24. Find the equation of the tangent plane and normal to the surface
x2 + y2 + z2 = 25 at (4, 0, 3).
asy
Solution. Let the given point be (x0 , y0 , z0 )
∴ x0 = 4, y0 = 0, z0 = 3.
En
Now, φ = x2 + y2 + z2 − 25. gin
∂φ
∂x
∂φ
= 2x ⇒

= 2y ⇒
∂φ 
∂x (4,0,3)
∂φ 
= 8.

= 0.
ee rin
∂y (4,0,3)
∂y
∂φ
∂z
= 2z ⇒
∂φ 
∂z (4,0,3)
= 6. g.n
Equation of the tangent plane is et
∂φ ∂φ ∂φ
(x − x0 ) + (y − y0 ) + (z − z0 ) = 0.
∂x ∂y ∂z
8(x − 4) + 0 + (z − 3)6 = 0.

Dividing by 2 and expanding we get

4x − 16 + 3z − 9 = 0 ⇒ 4x + 3z = 25.

Downloaded From : www.EasyEngineering.net


Downloaded From : www.EasyEngineering.net

Vector Calculus 185

Equation of the normal to the surface at (4, 0, 3) is


x − x0 y − y0 z − z0
∂φ
= ∂φ
= ∂φ
∂x ∂y ∂z
x−4 y−0 z−3
= =
8 0 6
x−4 y z−3
= = .
8 0 6
Example 2.25. Find the equation of the tangent plane and normal to the surface
xz2 + x2 y − z + 1 = 0 at the point (1, −3, 2).

ww Solution. Let the given point be (x0 , y0 , z0 ).


∴ x0 = 1, y0 = −3, z0 = 2.

w.E Now, φ = xz2 + x2 y − z + 1.


∂φ

asy
 ∂φ 
= z2 + 2xy ⇒ = 4 + 2(1)(−3) = 4 − 6 = −2.
∂x ∂x (1,−3,2)
∂φ  ∂φ 
∂y
∂φ
= x2 ⇒

En
∂y (1,−3,2)
 ∂φ 
= 1.

∂z
= 2xz − 1 ⇒
gin
∂z (1,−3,2)
= 2(1)2 − 1 = 3.

ee
Equation of the tangent plane is
∂φ ∂φ ∂φ
(x − x0 )
∂x
+ (y − y0 )
∂y
+ (z − z0 )
∂z
= 0.
rin
− 2(x − 1) + 1(y + 3) + 3(z − 2) = 0.

− 2x + 2 + y + 3 + 3z − 6 = 0 g.n
− 2x + y + 3z − 1 = 0

2x − y − 3z + 1 = 0.
et
Equation of the normal is
x − x0 y − y0 z − z0
∂φ
= ∂φ
= ∂φ
∂x ∂y ∂z
x−1 y+3 z−2
= = .
−2 1 3

Downloaded From : www.EasyEngineering.net


Downloaded From : www.EasyEngineering.net

186 Engineering Mathematics - II

Example 2.26. If ~r = x~i + y~j + z~k and r = |~r| prove that


~r
(i) ∇r = .
r
(ii) ∇rn =! nrn−2~r.
1 ~r
(iii) ∇ = − 3.
r r
~r
(iv) ∇(log r) = 2 . [Jun 2006]
r
Proof. Given ~r = x~i + y~j + z~k
p
|~r| = x2 + y2 + z2 .

ww ∂r
= p
2x
∂x 2 x2 + y2 + z2
= p
x x
= .
x2 + y2 + z2 r

w.E Similarly,
∂r y ∂r z
= , = .
∂y r ∂z r

∂r ~ ∂r ~ ∂r
(i) ∇r = ~i + j +k asy
~
=i
∂x
x
r

~
+j
∂y
r
+k
∂z
y  ~ z 
r En
1 ~ ~ ~
= (xi + y j + zk) = .
r
~r
r gin
(ii) ∇rn = ~i
∂(rn ) ~ ∂(rn ) ~ ∂(rn )

= ~i.n.r
∂x
n−1 ∂r
+j
∂y
+ ~j.n.r
+k
n−1 ∂r
∂z ee
+ ~k.n.rn−1
∂r
rin
n−1 ~ x
∂x

r
y
= nr (i + ~j + ~k )
r
z
r
∂y ∂z

g.n
= nrn−2~r.

(iii) ∇
1
r
= (−1)r−3~r = −
~r
r3
. [put n=-1 in (ii)]
et
∂ ∂ ∂
(iv) ∇(log r) = ~i (log r) + ~j (log r) + ~k (log r)
∂x ∂y ∂z
1 ∂r 1 ∂r 1 ∂r
= ~i . + ~j . + ~k .
r ∂x r ∂y r ∂z
1 h x~i y~j z~k i ~r
= + + = 2.
r r r r r

Downloaded From : www.EasyEngineering.net


Downloaded From : www.EasyEngineering.net

Vector Calculus 187

Exercise 2 A

1. Find ∇φ at the point (1, 1, 1) if φ = x2 y + y2 x + z2 .

2. If φ = log(x2 + y2 + z2 ), find ∇φ. [Dec 2010]

f ′ (r)
3. Prove that ∇ f (r) = ~r where ~r = x~i + y~j + z~k.
r

4. Find gradφ at (1, 2, 1) if φ = log(x2 + y2 + z2 ).

ww 5. If φ = x + xy2 + yz2 , find gradφ.

w.E 6. Find the directional derivative of φ = x2 − 2y2 + 4z2 at the point (1, 1, −1) in the
direction 2~i − ~j − ~k.

asy
7. Find the directional derivative of the function φ = xy + yz + zx in the direction

En
of the vector 2~i + 3~j + 6~k at the point (3, 1, 2).

gin
8. Find the directional derivative of the function 2yz + z2 in the direction of the
vector ~i + 2~j + 2~k at the point (1, −1, 3).

ee rin
9. Find the directional derivative of x3 + y3 + z3 at the point (1, −1, 2) in the
direction of ~i + 2~j + ~k.

10. In what direction from (3, 1, −2) is the directional derivative of φ = x2 y2 z4 g.n
maximum? Find also the magnitude of this maximum.

11. If the directional derivative of φ(x, y, z) = a(x+y)+b(y+z)+c(z+ x) has maximum


et[May 2015]

x−1 y−2 z−1


value 12 at (1, 2, 1) in the direction parallel to the line = = ,
1 2 3
find the values of a, b and c.

12. Find the directional derivative of φ = 3x2 + 2y − 3z at (1, 1, 1) in the direction of


2~i + 2~j − ~k. [Jun 2010]

Downloaded From : www.EasyEngineering.net


Downloaded From : www.EasyEngineering.net

188 Engineering Mathematics - II

13. In what direction from the point (1, 1, −1) is the directional derivative of
φ = x2 − 2y2 + 4z2 maximum? What is the magnitude of this maximum
directional derivative?

14. Find a unit normal to the surface xy3 z2 = 4 at the point (−1, −1, 2).

15. Find a unit normal vector to the surface x2 + y2 − z2 = 1 at (1, 1, 1).

16. Find a unit normal to the surface xy2 z3 = 1 at (1, 1, 1).

ww 17. Find the angle between the normals to the surface xy = z2 at the points (1, 4, 2)

w.E and (−3, −3, 3).

18. Find the angle between the spheres x2 + y2 + z2 = 29 and x2 + y2 + z2 + 4x − 6y −

asy
8z − 47 = 0 at (4, −3, 2).

En
19. Find the angle between the surfaces 2yz + z2 = 3 and x log z = y2 − 1 at (1, 1, 1).

gin
20. Find the angle between the surfaces x2 + y2 + z2 = 9 and z = x2 + y2 − 3 at the
point (2, −1, 2).

ee
21. If ∇φ = 2xyz~i + x2 z~j + x2 y~k, find φ.
rin
     
22. If ∇φ = y2 − 2xyz3 ~i + 3 + 2xy − x2 z3 ~j + 6z3 − 3x2 yz2 ~k, find φ.
g.n
et
23. Find the equation of the tangent plane and the equation of the normal to the
surface x2 − 4y2 + 3z2 + 4 = 0 at the point (3, 2, 1).

24. Find the equation of the tangent plane and normal to the surface
2xz2 − 3xy − 4x = 7 at the point (1, −1, 2).

25. Find the directional derivative of φ = 5x2 y − 5y2 z + 25 z2 x at the point P(1, 1, 1)
x−1 y−3
in the direction of the line = = z.
2 −2

Downloaded From : www.EasyEngineering.net


Downloaded From : www.EasyEngineering.net

Vector Calculus 189

2.3.1 Divergence and curl of a vector point function

~ y, z) be a vector point function continuously differentiable in the region


Let F(x,
R. Then the divergence of F~ is defined as
∂F~ ~ ∂F~ ~ ∂F~
divF~ = ∇ · F~ = ~i · + j· +k· .
∂x ∂y ∂z
∂F1 ∂F2 ∂F3
If F~ = F1~i + F2 ~j + F3~k, then divF~ = + + .
∂x ∂y ∂z
Result

ww (1). If F~ is a constant vector, then ∇ · F~ = 0.


(2). F~ · ∇ , ∇ · F.
~

w.E
Solenoidal vector. If divF~ = 0 everywhere in a region R, then F~ is called a
solenoidal vector in R. [Dec 2013]

asy
Curl of a vector. If F~ is a vector point function continuously differentiable in a
region R, then curlF~ is defined by
curlF~ = ∇ × F~ = ~i × + j× En
∂F~ ~ ∂F~ ~ ∂F~
+k× .
∂x ∂y
gin ∂z
~i ~j ~k

ee

If F~ = F1~i + F2 ~j + F3~k, then ∇ × F~ = ∂x ∂ ∂
∂y
∂ .
∂z

rin
F F F
1 2 3

Note. If F~ is a constant vector, then curlF~ = ~0.

g.n
~ y, z) be a vector point function. If curlF~ = ~0 at all
Irrotational vector. Let F(x,
points in a region R, then F~ is said to be an irrotational vector in R.
Conservative vector field. A vector field F~ is said to be conservative, if there
et
exists a scalar point function φ such that F~ = ∇φ.
Note
(1) In a conservative vector field F~ = ∇φ.
∴ ∇ × F~ = ∇ × ∇φ = ~0.
⇒ F~ is irrotational.

Downloaded From : www.EasyEngineering.net


Downloaded From : www.EasyEngineering.net

190 Engineering Mathematics - II

(2) In the conservative vector field, the scalar function φ is called the scalar
~
potential of F.
✎ ☞
Worked Examples
✍ ✌

Example 2.27. Find the divergence and curl of the vectorF~ = xyz~i+3x2 y~j+(xz2 −y2 z)~k
at the point (2, −1, 1). [Jun 1996]
Solution. Given, F~ = xyz~i + 3x2 y~j + (xz2 − y2 z)~k
=⇒ F1 = xyz, F2 = 3x2 y, F3 = xz2 − y2 z.

ww ∂F1
∂x
= yz,
∂F2
∂y
= 3x2 ,
∂F3
∂z
= 2xz − y2 .

w.E ∇ · F~ =
∂F1 ∂F2 ∂F3
∂x
+
∂y
+
∂z
= yz + 3x2 + 2xz − y2 .

~ (2,−1,1) = −1 + 12 + 4 − 1 = 14.
(∇ · F)

asy

~i

~j ~k ~i ~j ~k

En

curlF~ = ∂x ∂ ∂
∂y
∂ = ∂
∂z ∂x

∂y ∂z


F F F xyz 3x2 y xz2 − y2 z

gin
1 2 3
 ∂(xz2 − y2 z) ∂(3x2 y)   ∂(xz2 − y2 z) ∂(xyz) 
= ~i − − ~j −

ee
∂y ∂z ∂x ∂z
2
 ∂(3x y) ∂(xyz) 
+ ~k

rin
− .
∂x ∂y
= ~i(−2yz) − ~j(z2 − xy) + ~k(6xy − xz).
~ (2,−1,1) = 2~i − 3~j − 14~k.
(curlF) g.n
~ and curl F~ of the vector point function
Example 2.28. Find ∇.F,
F~ = xz3~i − 2x2 yz~j + 2yz4~k at the point (1, −1, 1).
et [Apr 2008]
Solution. Given F~ = xz3~i − 2x2 yz~j + 2yz4~k.
=⇒ F1 = xz3 , F2 = −2x2 yz, F3 = 2yz4 .
∂F1 ∂F2 ∂F3
= z3 , = −2x2 z, = 8yz3
∂x ∂y ∂z

∂F1 ∂F2 ∂F3


∇.F~ = + + = z3 − 2x2 z + 8yz3 .
∂x ∂y ∂z

Downloaded From : www.EasyEngineering.net


Downloaded From : www.EasyEngineering.net

Vector Calculus 191

~ (1,−1,1) = 1 − 2.1.1 + 8(−1).1 = 1 − 2 − 8 = −9.


(∇.F)

~i ~j ~k

∇ × F~ = ∂x ∂ ∂
∂y

∂z

F F F
1 2 3

~i ~j ~k

= ∂x ∂ ∂ ∂
∂y ∂z
xz3 −2x2 yz 2yz4

ww
! ! !
~ ∂ 4 ∂ 2 ~ ∂ 4 ∂ 3 ~ ∂ 2 ∂ 3
=i (2yz ) − (−2x yz) − j (2yz ) − (xz ) + k (−2x yz) − (xz )
∂y ∂z ∂x ∂z ∂x ∂y
= ~i(2z4 + 2x2 y) − ~j(0 − 3xz2 ) + ~k(−4xyz − 0)

w.E = (2z4 + 2x2 y)~i + 3xz2 ~j − 4xyz~k

(∇ × F)
asy
~ (1,−1,1) = 0~i + 3~j + 4~k = 3~j + 4~k.

Example 2.29. Prove that ∇ × ∇φ = ~0 where φ is a scalar point function.


Solution. We have ∇φ = ~i En
∂φ ~ ∂φ ~ ∂φ
+j +k .


~i
∂x

gin
~j ~k
∂y

∂z


∇ × ∇φ = ∂x


∂φ
∂x

∂y
∂φ
∂y
∂φee
∂ .
∂z

∂z

rin
∂2 φ ∂2 φ ∂2 φ ∂2 φ ∂2 φ

g.n
∂2 φ
! ! !
= ~i − − ~j − + ~k −
∂y∂z ∂z∂y ∂x∂z ∂z∂x ∂x∂y ∂y∂x

et
2
∂ φ 2
∂ φ
=0 [assuming = etc]
∂y∂z ∂z∂y
∇ × ∇φ = 0 always.

Example 2.30. Find divergence F~ and curl F~ where F~ = grad (x3 + y3 + z3 − 3xyz).
[May 2001]
Solution. Given, F~ = grad(φ), where φ = x3 + y3 + z3 − 3xyz.

∂φ ∂φ ∂φ
= 3x2 − 3yz, = 3y2 − 3xz, = 3z2 − 3xy.
∂x ∂y ∂z

Downloaded From : www.EasyEngineering.net


Downloaded From : www.EasyEngineering.net

192 Engineering Mathematics - II

Now, F~ = gradφ
∂φ ~ ∂φ ~ ∂φ
i.e., F~ = ~i +j +k .
∂x ∂y ∂z
F~ = (3x2 − 3yz)~i + (3y2 − 3xz)~j + (3z2 − 3xy)~k

F1 = 3x2 − 3yz, F2 = 3y2 − 3xz, F3 = 3z2 − 3xy


∂F1 ∂F2 ∂F3
div F~ = + + = 6x + 6y + 6z = 6(x + y + z).
∂x ∂y ∂z
curl F = ∇ × F~ = ∇ × ∇φ = ~0. [ By the previous example.]

ww Example 2.31. Prove that div~r = 3, curl~r = ~0, where ~r is the position vector of a

w.E
point (x, y, z) in space.
Solution. Given, ~r = x~i + y~j + z~k.
[May 2005]

div~r =
asy
∂x ∂y ∂z
+
∂x ∂y ∂z
+ = 1 + 1 + 1 = 3.

En

~i ~j ~k

curl~r = ∂x ∂ ∂ ∂
∂y

gin
∂z
x y z

ee
 ∂z ∂y   ∂z ∂x   ∂y ∂x 
= ~i − − ~j − + ~k −
∂y ∂z ∂x ∂z ∂x ∂y
= ~0.
rin
g.n
Example 2.32. Prove that the vector F~ = (x + 3y)~i + (y − 3z)~j + (x − 2z)~k is solenoidal.
Solution. We have, F1 = x + 3y, F2 = y − 3z, F3 = x − 2z.
∇ · F~ =

∇ · F~ =
∂F1 ∂F2 ∂F3


∂x
+
∂y
+

∂z

(x + 3y) + (y − 3z) + (x − 2z)
et
∂x ∂y ∂z
=1+1−2
= 0.
∴ F~ is solenoidal.

Example 2.33. Prove that F~ = (2x2 y + yz)~i + (xy2 − xz2 )~j − (6xyz + 2x2 y2 )~k is solenoidal.

Downloaded From : www.EasyEngineering.net


Downloaded From : www.EasyEngineering.net

Vector Calculus 193

Solution. We have, F1 = 2x2 y + yz, F2 = xy2 − xz2 , F3 = −6xyz − 2x2 y2 .


∂F1 ∂F2 ∂F3
= 4xy, = 2xy, = −6xy.
∂x ∂y ∂z

∂F1 ∂F2 ∂F3


∇ · F~ = + +
∂x ∂y ∂z
= 4xy + 2xy − 6xy

= 0.

ww ∴ F~ is solenoidal.

w.E
Example 2.34. Find the value of a if the vector F~ = (2x2 y + yz)~i + x(y2 − z2 )~j
+(axyz − 2x2 y2 )~k is solenoidal.
Solution. Given, F~ is solenoidal.

asy
∴ ∇ · F~ = 0.


∂ ∂
En ∂
(2x2 y + yz) + (xy2 − xz2 ) + (axyz − 2x2 y2 ) = 0.
∂x
4xy + 2xy + axy = 0
∂y

gin ∂z

6xy + axy = 0
ee
xy(6 + a) = 0 ⇒ a = −6.
rin
solenoidal. g.n
Example 2.35. Find a such that (3x − 2y + z)~i + (4x + ay − z)~j + (x − y + 2z)~k is
[May 2015, Jun 2012, Jun 2004]
Solution. Let F~ = (3x − 2y + z)~i + (4x + ay − z)~j + (x − y + 2z)~k.
Given that F~ is solenoidal.
et
∴ ∇.F~ = 0.

∂ ∂ ∂
⇒ (3x − 2y + z) + (4x + ay − z) + (x − y + 2z) = 0.
∂x ∂y ∂z
i.e., 3 + a + 2 = 0
a+5=0 ⇒ a = −5.

Downloaded From : www.EasyEngineering.net


Downloaded From : www.EasyEngineering.net

194 Engineering Mathematics - II

~ = (x + 3y)~i + (y − 2z)~j + (x + 2λz)~k is solenoidal, find the value of


Example 2.36. If V
λ.
[Dec 2013, May 2011]
~ = (x + 3y)~i + (y − 2z)~j + (x + 2λz)~k.
Solution. Given V
~ is solenoidal.
Given V

~ =0
∴ ∇·V
∂ ∂ ∂
i.e., (x + 3y) + (y − 2z) + (x + 2λz) = 0
∂x ∂y ∂z

ww 1 + 1 + 2λ = 0

w.E 2λ = −2

λ = −1.

asy
Example 2.37. Show that the vector F~ = (6xy + z3 )~i + (3x2 − z)~j + (3xz2 − y)~k is
irrotational.
Solution. En

gin
~i ~j ~k

ee

∇ × F~ = ∂x ∂ ∂
∂y

∂z

rin
6xy + z3 3x2 − z 3xz2 − y

= ~i(−1 + 1) − ~j(3z2 − 3z2 ) + ~k(6x − 6x) = ~0

∴ F~ is irrotational. g.n
Example 2.38. Prove that F~ = yz~i + zx~j + xy~k is irrotational. [Dec 2012, Dec 2011]
Solution.
et

~i ~j ~k

∇ × F~ = ∂x ∂ ∂
∂y

∂z

yz zx xy
! ! !
~ ∂ ∂ ~ ∂ ∂ ~ ∂ ∂
=i (xy) − (zx) − j (xy) − (yz) + k (zx) − (yz)
∂y ∂z ∂x ∂z ∂x ∂y

Downloaded From : www.EasyEngineering.net


Downloaded From : www.EasyEngineering.net

Vector Calculus 195

= ~i(x − x) − ~j(y − y) + ~k(z − z)

= ~0.

∴ F~ is irrotational.

Example 2.39. Find the constants a, b, c so that


F~ = (x + 2y + az)~i + (bx − 3y − z)~j + (4x + cy + 2z)~k is irrotational.
Solution. Given that F~ is irrotational.
⇒ ▽ × F~ = ~0.

ww

i.e.


~i

~j

~k




= 0

w.E ∂x ∂y ∂z
x + 2y + az bx − 3y − z 4x + cy + 2z
~i(c + 1) − ~j(4 − a) + ~k(b − 2) = 0.

asy
Equating the coefficients of ~i, ~j, ~k both sides, we get
a = 4, b = 2, c = −1

En
Example 2.40. Show that F~ = (y2 + 2xz2 )~i + (2xy − z)~j + (2x2 z − y + 2z)~k is irrotational
and hence find its scalar potential.
gin [Dec 2014, Jun 2012]
Solution. Given that F~ =


~i
(y2

~j
+

ee2xz2 )~i +

~k

(2xy − z)~j + (2x2 z − y + 2z)~k.

rin
g.n


∇ × F~ = ∂x ∂ ∂
∂y

∂z

et
y2 + 2xz2 2xy − z 2x2 z − y + 2z
! !
∂ ∂ ∂ ∂
= ~i (2x z − y + 2z) − (2xy − z) − ~j
2 2 2 2
(2x z − y + 2z) − (y + 2xz )
∂y ∂z ∂x ∂z
!
∂ ∂
+ ~k (2xy − z) − (y2 + 2xz2 )
∂x ∂y
= ~i(−1 + 1) − ~j(4xz − 4xz) + ~k(2y − 2y)

= ~i.0 − ~j.0 + ~k.0

= ~0.

Downloaded From : www.EasyEngineering.net


Downloaded From : www.EasyEngineering.net

196 Engineering Mathematics - II

∴ F~ is irrotational.
Since ∇ × F~ = 0, then F~ = ∇φ where φ is the scalar potential.
∂φ ~ ∂φ ~ ∂φ
∴ F~ = ~i +j +k .
∂x ∂y ∂z
Comparing the components of F~ we obtain
∂φ
= y2 + 2xz2 (1)
∂x
∂φ
= 2xy − z (2)
∂y

ww ∂φ
∂z
= 2x2 z − y + 2z. (3)

w.E
Integrating partially (1), (2), (3) w.r.t. x, y and z respectively we get
Z
φ = (y2 + 2xz2 )dx + f1 (y, z)

= y2 x + 2z2
x2
2 asy
+ f1 (y, z)
2 2 2
= xy + x z + f1 (y, z).
En (4)

From (2) we get


Z gin
φ = (2xy − z)dy + f2 (x, z)

= 2x
y2
2
− zy + f2 (x, z)
ee rin
= xy2 − yz + f2 (x, z)
g.n (5)

From (3) we obtain


Z
φ = (2x2 z − y + 2z)dz + f3 (x, y) et
z2 z2
= 2x2 − yz + 2 + f3 (x, y)
2 2
= x2 z2 − yz + z2 + f3 (x, y). (6)

From (4), (5) and (6) we get

φ = xy2 + x2 z2 − yz + z2 + c.

Downloaded From : www.EasyEngineering.net


Downloaded From : www.EasyEngineering.net

Vector Calculus 197

Example 2.41. Prove that F~ = (y2 cos x + z3 )~i + (2y sin x − 4)~j + 3xz2~k is irrotational
and find its scalar potential. [Dec.2005]
Solution. Given F~ = (y2 cos x + z3 )~i + (2y sin x − 4)~j + 3xz2~k.

~i ~j ~k

▽ × F~ = ∂ ∂ ∂

∂x ∂y ∂z
y2 cos x + z3 2y sin x − 4 3xz2

= ~i(0 − 0) − ~j(3z2 − 3z2 ) + ~k(2y cos x − 2y cos x)

= ~0.

ww ∴ F~ is irrotational.
Since ▽ × F~ = ~0, F~ = ▽φ, where φ is a scalar potential.

w.E ∴ F~ = ~i
∂φ ~ ∂φ ~ ∂φ
∂x
+j
∂y
+k .
∂z
Comparing the components of F, ~ we get
∂φ
∂x
= y2 cos x + z3
asy (1)

En
∂φ
= 2y sin x − 4 (2)
∂y
∂φ
∂z
= 3xz2 .
gin
Integrating (1), (2), and (3) partially w.r.t. x, y and z respectively we get,
(3)

φ=
R  
φ = y2 cos x + z3 dx + f1 (y, z)
= yZ2 sin x + xz3 + f1 (y, z).ee
(2y sin x − 4)dy + f2 (x, z). rin (4)

y2
= 2 sin x − 4y + f2 (x, z) g.n
φ=
2
2
= yZ sinx − 4y + f2 (x, z).
3xz2 dz + f3 (x, y).
et (5)

z3
= 3x + f3 (x, y)
3
= xz3 + f3 (x, y). (6)
From (4), (5) and (6) we get
φ = y2 sin x + xz3 − 4y + c, where c is a constant.

Downloaded From : www.EasyEngineering.net


Downloaded From : www.EasyEngineering.net

198 Engineering Mathematics - II

Example 2.42. Show that the vector field F~ = (x2 + xy2 )~i + (y2 + x2 y)~j is irrotational.
Find its scalar potential. [Dec 2013]
Solution. Given F~ = (x2 + xy2 )~i + (y2 + x2 y)~j

~i ~j ~k

∇ × F~ = ∂x ∂ ∂
∂y

∂z

x2 + xy2 y2 + x2 y 0

= ~i(0 − 0) − ~j(0 − 0) + ~k(2xy − 2xy)

ww = ~0.

∴ F~ is irrotational.

w.E
Since ∇ × F~ = ~0, F~ = ∇φ, where φ is a scalar potential.

∴ F~ = ~i
∂φ ~ ∂φ ~ ∂φ
+j +k .
∂x ∂y
asy∂z
~ we get
Comparing the components of F,
∂φ
∂x
= x2 + xy2
En (1)
∂φ
∂y
= y2 + x2 y
gin (2)
∂φ
∂z
= 0.
ee rin
Integrating (1), (2) and (3) partially w.r.t. x, y and z respectively we get

g.n
Z
φ = (x2 + xy2 )dx + f1 (y, z)

φ=

φ=
x3 x2 y2
Z3
+
2
+ f1 (y, z)

(y2 + x2 y) + f2 (x, z)
et (4)

y3 x2 y2
φ= + + f2 (x, z) (5)
3 2
φ = f3 (x, y) (6)

From (4), (5) and (6) we get


x3 y3 x2 y2
φ= + + + c, where c is a constant.
3 3 2

Downloaded From : www.EasyEngineering.net


Downloaded From : www.EasyEngineering.net

Vector Calculus 199

Example 2.43. Show that F~ = (2xy − z2 )~i + (x2 + 2yz)~j + (y2 − 2zx)~k is irrotational and
hence find its scalar potential. [Dec 2012]
Solution. Given F~ = (2xy − z2 )~i + (x2 + 2yz)~j + (y2 − 2zx)~k.

~i ~j ~k

∇ × F~ = ∂x ∂ ∂
∂y

∂z


2xy − z2 x2 + 2yz y2 − 2zx

= ~i(2y − 2y) − ~j(−2z + 2z) + ~k(2x − 2x)

ww = ~0.

∴ F~ is irrotational.

w.E
Since ∇ × F~ = ~0, F~ = ∇φ, where φ is a scalar potential.

asy
∂φ ~ ∂φ ~ ∂φ
∴ F~ = ~i +j +k .
∂x ∂y ∂z

En
Comparing the components of F~ we get
∂φ
∂x
∂φ
= 2xy − z2 .

= x2 + 2yz.
gin (1)

(2)
∂y
∂φ
∂z
= y2 − 2zx. ee rin (3)

Integrating (1), (2) and (3) partially w.r.t. x, y and z respectively we get
Z
g.n
φ = (2xy − z2 )dx + f1 (y, z)

= 2y ·
x2
2
− xz2 + f1 (y, z)
et
= x2 y − xz2 + f1 (y, z). (4)
Z
φ = (x2 + 2yz)dy + f2 (x, z)

y2
= x2 y + 2z · + f2 (x, z)
2
= x2 y + y2 z + f2 (x, z). (5)

Downloaded From : www.EasyEngineering.net


Downloaded From : www.EasyEngineering.net

200 Engineering Mathematics - II

Z
φ= (y2 − 2zx)dz + f3 (x, y)

z2
= zy2 − 2x · + f3 (x, y)
2
= zy2 − xz2 + f3 (x, y). (6)

From (4), (5) and (6) we get


φ = x2 y − xz2 + y2 z + c, where c is a constant.

ww Example 2.44. Prove that F~ = (6xy + z3 )~i + (3x2 − z)~j + (3xz2 − y)~k is an irrotational
vector and find the scalar potential such that F~ = ∇φ. [Jun 2010]

w.E
Solution. Given F~ = (6xy + z3 )~i + (3x2 − z)~j + (3xz2 − y)~k

asy ~k
~i ~j

∇ × F~ = ∂x ∂ ∂
∂y

∂z

En

6xy + z3 3x2 − z 3xz2 − y

gin
= ~i(−1 + 1) − ~j(3z2 − 3z2 ) + ~k(6x − 6x)

= ~0.

∴ F~ is irrotational. ee
Since ∇ × F~ = ~0, F~ = ∇φ, where φ is a scalar potential. rin
∴ F~ = ~i
∂φ ~ ∂φ ~ ∂φ g.n
∂x
+j
∂y
+k .
∂z

Comparing the components of F~ we get


et
∂φ
= 6xy + z3 . (1)
∂x
∂φ
= 3x2 − z. (2)
∂y
∂φ
= 3xz2 − y. (3)
∂z

Downloaded From : www.EasyEngineering.net


Downloaded From : www.EasyEngineering.net

Vector Calculus 201

Integrating (1), (2) and (3) partially w.r.t. x, y and z respectively we get
Z
φ = (6xy + z3 )dx + f1 (y, z)

x2
= 6y · + xz3 + f1 (y, z)
2
= 3x2 y + xz3 + f1 (y, z). (4)
Z
φ = (3x2 − z)dy + f2 (x, z)

= 3x2 y − yz + f2 (x, z). (5)

ww Z
φ = (3xz2 − y)dz + f3 (x, y)

w.E = 3x ·
z3
3
− yz + f3 (x, y)

= xz3 − yz + f3 (x, y). (6)

From (4), (5) and (6) we get asy


En
φ = 3x2 y + xz3 − yz + c, where c is constant.

gin
Example 2.45. If ~r = x~i + y~j + z~k and r = |~r| prove that rn~r is solenoidal if n = −3 and
irrotational for all values of n.
Solution. Given ~r = x~i + y~j + z~k.
ee rin
[Dec 2007]

2 2
r = x +y +z .2 2

∂r x ∂r y ∂r z
= , = , =
∂x r ∂y r ∂z r
g.n
Let F~ = rn~r = rn x~i + rn y~j + rn z~k.

F1 = rn x, F2 = rn y, F3 = rn z
∂F1 ∂F2 ∂F3
et
∇ · F~ = + + .
∂x ∂x ∂x
∂ n ∂ ∂
= (r x) + (rn y) + (rn z).
∂x ∂x ∂x
∂r ∂r ∂r
= rn + x × n × r(n−1) + rn + y × n × r(n−1) + rn + z × n × r(n−1)
∂x ∂y ∂z
n (n−1)
 x y z
= 3r + nr x× +y× +z×
r r r

Downloaded From : www.EasyEngineering.net


Downloaded From : www.EasyEngineering.net

202 Engineering Mathematics - II

= 3rn + nr(n−2) (x2 + y2 + z2 )

= 3rn + nr(n−2) × r2

= 3rn + nrn = (n + 3)rn .

If rn~r is solenoidal then ∇ · F~ = 0.

i.e., (n + 3)rn = 0.

⇒ n + 3 = 0.

ww ⇒ n = −3.

~i ~j

~k

w.E

Now, curlF~ = ∂x ∂ ∂
∂y

∂z

rn x rn y rn z

= ~i
asy
 ∂(rn z) ∂(rn y) 
∂y

∂z
− ~j
 ∂(rn z) ∂(rn x) 
∂x

∂z
+ ~k
 ∂(rn y) ∂(rn x) 
∂x

∂y
.

En
 y z
= ~i z × n × r(n−1) × − y × n × rn−1 − ~j(0) + ~k(0) = ~0
r r

gin
∴ rn~r is irrotational for all values of n.

~ is a
Example 2.46. If ~r is the position vector of a point (x, y, z) in space and A
constant vector, prove that A
Solution. We have ~r = x~i + y~j + z~k.
ee
~ × ~r is solenoidal.

rin
Let A = a1~i + a2 ~j + a3~k, a1 , a2 , a3 are constants.

~i ~j ~k g.n
A

~ × ~r = a1 a2 a3

x y z

et
= ~i(a2 z − a3 y) − ~j(a1 z − a3 x) + ~k(a1 y − a2 x).

~ × ~r) = ∂ ∂ ∂
∇.(A (a2 z − a3 y) + (−(a1 z − a3 x)) + (a1 y − a2 x)
∂x ∂y ∂z
= 0 + 0 + 0 = 0.

~ × ~r is solenoidal.
=⇒ A

Downloaded From : www.EasyEngineering.net


Downloaded From : www.EasyEngineering.net

Vector Calculus 203

Example 2.47. Determine f (r) so that f (r)~r is both solenoidal and irrotational.
[Jan 2008]
Solution. Let ~r = x~i + y~j + z~k

r2 = x2 + y2 + z2

f (r)~r = f (r)x~i + f (r)y~j + f (r)z~k.

Given, f (r)~r is solenoidal.

ww =⇒∇ · f (r)~r = 0

w.E
∂ ∂ ∂
( f (r)x) + ( f (r)y) + ( f (r)z) = 0
∂x ∂y ∂z
∂r ∂r ∂r
f (r) + x f ′ (r) + f (r) + y f ′ (r) + f (r) + z f ′ (r) = 0

asy
3 f (r) + f ′ (r)
∂x
 x2 y2 z2 
+ + =0
∂y ∂z

En
r r r
r2
3 f (r) + f ′ (r) = 0

gin r
3 f (r) + f ′ (r)r = 0

f ′ (r)r = −3 f (r)
f ′ (r)
f (r)
3
=− .
r
ee rin
Integrating w.r.to. r. we get,
g.n
log f (r) = −3 log r + log c

= log r−3 + log c


et
c
= log 3
r
c
f (r) = 3 where c is the constant of integration.
r

Given f (r)~r is irrotational.


∇ × f (r)~r = ~0.

Downloaded From : www.EasyEngineering.net


Downloaded From : www.EasyEngineering.net

204 Engineering Mathematics - II


~i ~j ~k

∂ ∂ ∂ =0
∂x ∂y ∂z
f (r)x f (r)y f (r)z

~i ∂ ( f (r)z) − ∂ ( f (r)y) − ~j ∂ ( f (r)z) − ∂ ( f (r)x) + ~k ∂ ( f (r)y) − ∂ ( f (r)x) = 0


     
∂y ∂z ∂x ∂z ∂x ∂y

~i z f ′ (r) ∂r − y f ′ (r) ∂r − ~j z f ′ (r) ∂r − x f ′ (r) ∂r + ~k y f ′ (r) ∂r − x f ′ (r) ∂r = 0


h i h i h i
∂y ∂z ∂x ∂z ∂x ∂y

~i f ′ (r) yz − yz − ~j f ′ (r) xz − xz + ~k f ′ (r) xy − xy = 0


n o n o n o

ww r r
~i · 0 − ~j · 0 + ~k · 0 = 0
r r r r

w.E i.e., 0 = 0
∴ f (r)~r is irrotational for all f (r) and it is solenoidal for f (r) =
c
r3
where c is an
arbitrary constant.

Example 2.48. If ~v = w asy


~ ×~r, prove that w
1
~ = curl ~v where w
~ is a constant vector and

En
~r is the position vector of the point (x, y, z).
2

Solution. Given ~r = x~i + y~j + z~k.


~ = w1~i + w2 ~j + w3~k.
Let w gin
Also given that, ~v = w

~i
ee
~ × ~r

~j ~k rin

= w1 w2 w3


g.n
et
x y z

= ~i(w2 z − w3 y) − ~j(w1 z − w3 x) + ~k(w1 y − w2 x).


~i ~j ~k

Now, curl~v = ∂ ∂ ∂
∂x ∂y ∂z
w z − w y −w z + w x w y − w x
2 3 1 3 1 2
 ∂(w1 y − w2 x) ∂(−w1 z + w3 x)   ∂(w1 y − w2 x) ∂(w2 z − w3 y) 
= ~i − − ~j −
∂y ∂z ∂x ∂z

Downloaded From : www.EasyEngineering.net


Downloaded From : www.EasyEngineering.net

Vector Calculus 205

 ∂(−w1 z + w3 x) ∂(w2 z − w3 y) 
+ ~k − .
∂x ∂y
= ~i(w1 + w1 ) − ~j(−w2 − w2 ) + ~k(w3 + w3 ).

= 2(~iw1 + ~jw2 + ~kw3 ) = 2~


w.
1
~ = curl~v.
w
2

2.3.2 Vector identities

Identity I. ∇ · (F~ + G)
~ = ∇ · F~ + ∇ · G
~ (or) div(F~ + G)
~ = divF~ + divG.
~

ww Proof.

w.E ~ = (~i ∂ + ~j ∂ + ~k ∂ ) · (F~ + G)


∇ · (F~ + G)
∂x ∂y
∂(F~ + G)~
∂z
∂(F~ + G)
~
~

∂(F~ + G)~

asy
= ~i ·
∂x
 ∂F~ ∂G
+ ~j ·

~
∂y
 ∂F~ ∂G
+ ~k ·

~
∂z
 ∂F~ ∂G ~
= ~i ·
En
∂x
+
∂x
+ ~j ·

∂F~ ~ ∂F~ ~ ∂F~ ~ ∂G


∂y
+
∂y
~
+ ~k ·
∂z
~
+
∂z

gin
∂G ∂~g
= ~i · + j· +k· +i· + ~j · + ~k ·
∂x ∂y ∂z ∂x ∂y ∂z

ee
= ∇ · F~ + ∇ · G.
~

= divF~ + divG.

rin
~

~ = ∇f · G
∇ · ( f G) ~ + f (∇ · G).
~
~ is a vector point function, then
Identity II. If f is a scalar point function and G
g.n
Proof.

~ = ~i ·
∇ · ( f G)
∂x
~
∂( f G)
+ ~j ·
∂y
~
∂( f G)
+ ~k ·
~
∂( f G)
∂z
et
n ∂G ~ ∂f o n ~ n ~
~ + ~j · f ∂G + ∂ f G ~ + ~k · f ∂G + ∂ f G
o o
= ~i · f + G ~
∂x ∂x ∂y ∂y ∂z ∂z
 ∂G ~ ∂G~ ∂G~   ∂ f ~∂ f ~ ∂ f ~ 
= f ~i · + ~j · + ~k · + ~i +j +k ·G
∂x ∂y ∂z ∂x ∂y ∂z
~ + ∇ f · G.
= f (∇ · G) ~

Downloaded From : www.EasyEngineering.net


Downloaded From : www.EasyEngineering.net

206 Engineering Mathematics - II

~ is a vector point function then


Identity III. If f is a scalar point function and G
~ = ∇f × G
∇ × ( f G) ~ + f (∇ × G).
~
Proof.

~ = ~i × ∂ ~ + ~j × ∂ ( f G) ~ + ~k × ∂ ( f G) ~
∇ × fG ( f G)
∂x ∂y ∂z
~  ~  ~ 
     
 ∂ f ∂G  ∂ f ∂ G  ∂ f ∂ G
= ~i ×  G ~+ f  + ~j ×  G
 ~+ f  + ~k ×  G
 ~+ f 
∂x ∂x ∂y ∂y ∂z ∂z
∂f ~ ~ ∂f ~ ~ ∂f ~ ~ ∂G~ ∂G~ ∂G~
= ~i × G+ j× G+k× G+i× f + ~j × f + ~k × f

ww = ~i ×
∂x

∂x
f
G~ + ~j ×
∂y

∂y
f
G~ + ~k ×
∂z

∂z
f
G

 ∂G
~ + f ~i ×
∂x

∂x
~
+ ~j ×
∂y
∂G
∂y
~
+ ~k ×
∂G
∂z
∂z
~ 



w.E = i ~ ∂ f ~∂ f ~ ∂ f
∂x
+j
∂y
+k
∂z
!
×G ~ + f ∇×G

~


= ∇f × G
asy
~ + f (∇ × G).
~

Identity IV. If F~ and G


∇(F~ · G) ~ + (G En
~ are vector point functions, then
~ = (F~ · ∇)G ~ · ∇)F~ + F~ × (∇ × G)
~ +G
~ × (∇ × F).
~
Proof. We have ∇ f = ~i

∂x
f
+ ~j

∂y
f
gin
+ ~k

∂z
f
=
X ∂f
~i .
∂x

ee
∇(F~ · G)
~ =
X ∂
~i (F~ · G)
∂x
X  ∂G
F~ ·
~ 

~ =

~i +
X n ∂F~
~i
X  ∂F~
∂x
·G

~ ~i
~o
~ + F~ · ∂G
∂x
rin
g.n

=  ·G (1)
∂x ∂x
We know that ~a × (~b × ~c) = (~a · ~c)~b − (~a · ~b)~c

(~a · ~b)~c = (~a · ~c)~b − ~a × (~b × ~c)


~
∂G
(F~ · )~i = (F~ · ~i)
~
∂G
− F~ × (
∂G~
× ~i)
et
∂x ∂x ∂x
~
∂G ∂G~
= (F~ · ~i) + F~ × (~i × )
∂x ∂x
~
∂G X ∂ ~
~ + F~ × ( ~i × ∂G )
X X
(F~ · )~i = (F~ · ~i )G
∂x ∂x ∂x
= (F~ · ∇)G ~ + F~ × (∇ × G).
~ (2)

Downloaded From : www.EasyEngineering.net


Downloaded From : www.EasyEngineering.net

Vector Calculus 207

Interchanging F~ and G,
~ we get

~ · ∂F )~i = (G · ∇)F~ + G
X
(G ~ × (∇ × F)
~ (3)
∂x

Substituting (2) and (3) in (1), we get


∇(F~ · G)
~ = (F~ · ∇)G
~ + (G
~ · ∇)F~ + F~ × (∇ × G)
~ +G
~ × (∇ × F).
~
Identity V. If F~ and G
~ are vector point functions, then
∇ · (F~ × G)
~ =G
~ · (∇ × F)
~ − F~ · (∇ × G)
~
i.e., div(F~ × G)
~ =G
~ · curlF~ − F~ · curlG.
~

ww Proof. We have

w.E ∇ · (F~ × G)
~ =
X
~i · ∂ (F~ × G)
∂x
~
~i · ( ∂F × G
~

~
~ + F~ × ∂G )

asy
X
=
∂x ∂x
~ ~
~i · ( ∂F × G) ~i · (F~ × ∂G )
X X
~ +
=
X
En ∂x
∂ ~
F X ∂G~
∂x

gin
= ~i · ( ~ −
× G) ~i · ( ~
× F)
∂x ∂x
X  ∂F~  X  ∂G ~
~i × ~− ~i × · F~

ee
= ·G
∂x ∂x

rin
= (∇ × F)~ ·G ~ − (∇ × G)
~ · F~

∇ · (F~ × G)
~ =G
~ · (∇ × F)
~ − F~ · (∇ × G).
~

Identity VI. If F~ and G


~ are vector point functions, then g.n
∇ × (F~ × G)
Proof. We have
~ = F(∇
~ · G)
~ − G(∇
~ · F)
~ + (G
~ · ∇)F~ − (F~ · ∇)G.
~
et
~i × ∂ (F~ × G)
X
∇ × (F~ × G)
~ = ~
∂x
n  ∂F~ ~  o
   
X
~ ~
 
~ ∂G
= i ×  × G + F × 
∂x ∂x
 ~ ~
~i × ∂F × G ~i × F~ × ∂G
X  X 
= ~ +
∂x ∂x

Downloaded From : www.EasyEngineering.net


Downloaded From : www.EasyEngineering.net

208 Engineering Mathematics - II

We have ~a × (~b × ~c) = (~a · ~c)~b − (~a · ~b)~c

~ ~ o X n ∂G ~ ~o
~ ∂F − ~i · ∂F G ~ ∂G
Xn
∇ × (F~ × G)
~ = (~i · G) ~ + ~i · F~ − (~i · F)
∂x ∂x ∂x ∂x
~ ~ ~ ~
~ · ~i) ∂F − ~i · ∂F G ~i · ∂G F~ − ∂G
X X X X
= (G ~+ (F~ · ~i)
∂x ∂x ∂x ∂x
~  ~
X ∂ ! X  X  !
~ ~ ~

~ ∂ F ~

~ ∂G 
~ ~
X
~ ∂ ~
= G· i F +  i ·  G +  i ·  F − F · i G
 
 

∂x ∂x ∂x ∂x
~ · ∇)F~ − (∇ · F)
= (G ~G ~ + (∇ · G)
~ F~ − (F~ · ∇)G
~

ww ∇ × (F~ × G)
~ = F(∇
~ · G)
~ − G(∇
~ · F)
~ + (G
~ · ∇)F~ − (F~ · ∇)G
~

curl(F~ × G)
~ = F(div
~ ~ − G(div
· G) ~ ~ + (G
· F) ~ · ∇)F~ − (F~ · ∇)G.
~

w.E
Laplacian Operator.
∂ 2f
If f is a
∂2 f ∂2 f
div(grad f ) = ∇.∇ f = ∇2 f = 2 + 2 + 2 .
scalar point function, then

Solution. We have grad f = ~i


∂x
asy

∂x
f
∂y
+ ~j

∂y
f
∂z
∂f
+ ~k .
∂z

En
div(grad f ) = ∇.∇ f

= gin ∂ ∂
~i + ~j + ~k
∂x ∂y

∂z
!
. ~i

∂x
f
+ ~j

∂y
f
+ ~k

∂z
f
!

∇2 f = ee
∂ ∂f
∂x ∂x
2
!

2
+

∂ f ∂ f ∂ f
∂ ∂f
∂y ∂y
2
!
+
∂ ∂f
∂z ∂z
!

rin
∇2 f =

∂ 2
+
∂x2 ∂y2
∂ 2
+ 2
∂z
∂2 g.n
∇2 =

∇2 is called the Laplacian operator.


+
∂x2 ∂y2 ∂z2
+ .

et
Identity VII. If F~ is a vector point function then, div curlF~ = 0.
Solution. Let F~ = F1~i + F2 ~j + F3~k

~i ~j ~k

curl F~ = ∇ × F~ = ∂x ∂ ∂
∂y

∂z

F F F
1 2 3

Downloaded From : www.EasyEngineering.net


Downloaded From : www.EasyEngineering.net

Vector Calculus 209

 ∂F3 ∂F2   ∂F3 ∂F1   ∂F2 ∂F1 


= ~i − − ~j − + ~k − .
∂y ∂z ∂x ∂z ∂x ∂y
div curl F~ = ∇ · (∇ × F)
~
∂  ∂F3 ∂F2  ∂  ∂F3 ∂F1  ∂  ∂F2 ∂F1 
= − − − + −
∂x ∂y ∂z ∂y ∂x ∂z ∂z ∂x ∂y
2 2 2 2
∂ F3 ∂ F2 ∂ F3 ∂ F1 ∂ F2 ∂ F1 2 2
= − − + + − = 0.
∂x∂y ∂x∂z ∂y∂x ∂y∂z ∂z∂x ∂z∂y

Identity VIII. If F~ is a vector point function, then


~ = ∇ × (∇ × F)
curl(curl F) ~ = ∇(∇ · F)
~ − ∇2 F.
~

ww Solution. Let F~ = F1~i + F2 ~j + F3~k

w.E

~i ~j ~k

curl F~ = ∂x ∂ ∂
∂y

∂z

asy

F F F
1 2 3
 ∂F3 ∂F2   ∂F3 ∂F1   ∂F2 ∂F1 
= ~i − − ~j − + ~k − .

∂y
~i En
∂z
~j
∂x ∂z
~k

∂x ∂y

gin


~ =
curl(curlF) ∂ ∂ ∂
∂x ∂y ∂z

ee
∂F3 − ∂F2 ∂F1 − ∂F3 ∂F2 − ∂F1
∂y ∂z ∂z ∂x ∂x ∂y
!
∂ ∂F2 ∂F1 ∂ ∂F1 ∂F3 

rin
  
=i~ − − −
∂y ∂x ∂y ∂z ∂z ∂x

g.n
∂  ∂F2 ∂F1  ∂  ∂F3 ∂F2 !
− ~j − − −
∂x ∂x ∂y ∂z ∂y ∂z

et
!
~ ∂ ∂F1 ∂F3
  ∂ ∂F3 ∂F2 

+k − − −
∂x ∂z ∂x ∂y ∂y ∂z
2 2 2 2
!
∂ F2 ∂ F1 ∂ F1 ∂ F3
= ~i − − +
∂y∂x ∂y2 ∂z2 ∂z∂x
∂ F 1 ∂ F 2 ∂ F 3 ∂2 F 2
2 2 2
!
+ ~j − + −
∂x∂y ∂x2 ∂z∂y ∂z2
2 2 2 2F
!
∂ F 1 ∂ F 3 ∂ F 3 ∂ 2
+ ~k − − +
∂x∂z ∂x2 ∂y2 ∂y∂z
2 2 2 ∂2 F 1 ∂2 F 1 ∂2 F 1
!!
~ ∂ F1 ∂ F2 ∂ F3
=i + + − + +
∂x2 ∂x∂y ∂x∂z ∂x2 ∂y2 ∂z2

Downloaded From : www.EasyEngineering.net


Downloaded From : www.EasyEngineering.net

210 Engineering Mathematics - II

∂2 F 2 ∂2 F 1 ∂2 F 3 ∂2 F 2 ∂2 F 2 ∂2 F 2
!!
+j ~ + + − + +
∂y2 ∂y∂x ∂y∂z ∂x2 ∂y2 ∂z2
∂2 F 3 ∂2 F 1 ∂2 F 2 ∂2 F 3 ∂2 F 3 ∂2 F 3
!!
+ ~k + + − + +
∂z2 ∂z∂x ∂z∂y ∂x2 ∂y2 ∂z2
! !
~ ∂  ∂F1 ∂F2 ∂F3  2 ~ ∂  ∂F1 ∂F2 ∂F3  2
=i + + − ∇ F1 + j + + − ∇ F2
∂x ∂x ∂y ∂z ∂y ∂x ∂y ∂z
!
~ ∂  ∂F1 ∂F2 ∂F3  2
+k + + − ∇ F3
∂z ∂x ∂y ∂z
! ! !
∂ ∂ ~ − ∇ F2 + ~k ∂
= ~i (∇.F)~ − ∇ F1 + ~j
2
(∇.F) 2 ~ − ∇ F3
(∇.F) 2
∂x ∂y ∂z

ww ~ ∂
= i + j +k
∂x
~ ∂ ~∂
∂y ∂z
!
~ − ∇2 (F1~i + F2 ~j + F3~k)
(∇.F)

w.E ~ = ∇(∇ · F)
∇ × (∇ × F) ~ − ∇2 F.

1
~
!
Identity IX. Prove that ∇ n = − n+2 ~r.
n

asy r
Solution. Let ~r = x~i + y~j + z~k.
r

r2 = x2 + y2 + z2 .
1
!
En∂ 1
! !
~j ∂ 1 + ~k ∂ 1
!
We have, ∇ n = ~i
r
gin
∂x rn
+
∂y rn
∂r
∂z rn
∂r
= ~i(−n)r(−n−1) + ~j(−n)r(−n−1) + ~k(−n)r(−n−1)
∂r

ee
= n+1 i + j + k
r
−n
r
∂x
−n ~ x ~ y ~ z 
r r
n~r
∂y ∂z

rin
= n+2 (x~i + y~j + z~k) = − n+2 .

2 1
r
!
r
g.n
Identity X. Prove that ∇
  r
= 0.
   (−1) 
Solution. We have ∇2 1r = ∇.∇ 1r = ∇.∇(r−1 ) = ∇. r3 ~r [n = −1 in the previous
identity.]
et
[May 2015]

! "   #
~r 1 1
= −∇. 3 = − ∇ 3 .~r + 3 ∇.~r
r r r
" #
−3 1
= − 5 ~r.~r + 3 3 [∵ ∇.~r = 3]
r r
2
" #
−3r 3
=− + 3
r5 r

Downloaded From : www.EasyEngineering.net


Downloaded From : www.EasyEngineering.net

Vector Calculus 211

" #
−3 3
= − 3 + 3 = 0.
r r
Identity XI. Prove that ∇2 (rn ) = n(n + 1)rn−2 . [May 2015, Dec 2010]
Solution. We have ∇rn = nrn−2~r
∇2 rn = ∇ · ∇rn = ∇ · (nrn−2~r)

= n[∇rn−2 · ~r + rn−2 ∇ · ~r]

= n[(n − 2)rn−4~r · ~r + rn−2 · 3]

= n[(n − 2)rn−2 + 3rn−2 ] = nrn−2 [n − 2 + 3] = n(n + 1)rn−2 .


✎ ☞

ww ✍
Worked Examples

Example 2.49. Prove that ∇2 u = 0 if u = x2 − y2 .


w.E
Solution. u = x2 − y2 .
∂u ∂u ∂u
∂x
∂2 u
= 2x,
∂y
∂2 u asy
= −2y,
∂z
∂2 u
= 0.

En
= 2, = −2, = 0.
∂x2 ∂y2 ∂z2
∂2 u ∂2 u ∂2 u
Now, ∇2 u = 2 + 2 + 2
∂x ∂y ∂z
= 2 − 2 + 0 = 0. gin
ee rin
Example 2.50. If φ is a scalar point function, prove that ∇φ is solenoidal and
irrotational if φ is a solution of the Laplace equation. [Jun 2009]
Solution. Given that φ satisfies Laplace equation.
i.e, ∇2 φ = 0 g.n
∂2 φ ∂2 φ ∂2 φ
=⇒ 2 + 2 + 2 = 0.
∂x ∂y ∂z
Now div(∇φ) = ∇.∇φ = ∇2 φ = 0.
et
∴ ∇φ is solenoidal.
curl(∇φ) = ∇ × ∇φ = 0 (always).
=⇒ ∇φ is irrotational.
~ and B
Example 2.51. If A ~ are irrotational, prove that A×
~ B~ is solenoidal. [Jun.2013]
~ and B
Solution. Given that A ~ are irrotational.

Downloaded From : www.EasyEngineering.net


Downloaded From : www.EasyEngineering.net

212 Engineering Mathematics - II

~ = 0 and ∇ × B
Therefore, ∇ × A ~ = 0.
~ × B)
Now we have, ∇.(A ~ = A.(∇
~ × B)
~ − B.(∇
~ ~
× A)

~ ~0 − B.~0 = 0.
= A.

~×B
=⇒ A ~ is solenoidal.

Example 2.52. Prove that ∇ × (∇rn ) = ~0.


Solution. We have ∇rn = nrn−2~r.

ww Now, ∇ × (∇rn ) = ∇ × (nrn−2~r) = n[∇ × rn−2~r]

w.E = n[∇(rn−2 ) × ~r + rn−2 ∇ × ~r]

= n[(n − 2)rn−4~r × ~r + 0] = n · ~0 = ~0.

asy
Example 2.53. If φ and ψ satisfies Laplace equation, prove that (φ∇ψ − ψ∇φ) is

En
solenoidal.
Solution. Given φ and ψ satisfies Laplace equation.

gin
Therefore, ∇2 φ = 0 and ∇2 ψ = 0.
We have to prove that φ∇ψ − ψ∇φ is solenoidal.

ee
i.e., to prove div(φ∇ψ − ψ∇φ) = 0.

Now, div(φ∇ψ − ψ∇φ) = ∇.(φ∇ψ − ψ∇φ) rin


= ∇.(φ∇ψ) − ∇.ψ(∇φ)
g.n
= ∇φ.∇ψ + φ∇.∇ψ − [∇ψ.∇φ + ψ∇.∇φ]

= ∇φ.∇ψ + φ∇2 ψ − ∇ψ.∇φ − ψ∇2 φ et


= φ.0 − ψ.0 = 0.

=⇒ φ∇ψ − ψ∇φ is solenoidal.


d2 f 2 d f
Example 2.54. Show that ∇2 f (r) = + . .
dr2 r dr
~r
Solution. We have ∇ f (r) = f ′ (r) .
r

Downloaded From : www.EasyEngineering.net


Downloaded From : www.EasyEngineering.net

Vector Calculus 213

Hence, ∇2 f (r) = ∇.∇ f (r)


!
′ ~r
= ∇. f (r)
r
′ f ′ (r)
!
f (r)
= ∇ .~r + (∇.~r)
r! r
f ′ (r) 3 f ′ (r)
= ∇ .~r +
r r
r∇ f ′ (r) − f ′ (r)∇r 3 f ′ (r)
! !
f
= .~r + [using ∇ ]
r2 r g
 ′′ ~r
 r f (r) r − f ′ (r) ~rr 

ww


=   .~r + 3 f (r)
r2 r
r f ′′ (r) − f ′ (r) 3 f ′ (r)

w.E =

=
r3
~
r .~
r +
r f ′′ (r) − f ′ (r) 2 3 f ′ (r)
r +
r

asy
r3 r
r f ′′ (r) − f ′ (r) 3 f ′ (r)
= +
r r
= ′′
f (r) −
r
En
f ′ (r) 3 f ′ (r)
+
r
=
2
d f (r) 2 d f
dr2
+
r dr
.
gin
Example 2.55.
Solution. We have ~r = x~i + y~j + z~k
p
ee
Prove that ▽2 rn~r = n(n + 3)rn−2~r.


rin
r = ~r = x2 + y2 + z2 .
∂r x ∂r y ∂r z
= , = , = .
∂x r ∂y r ∂z r g.n
We know that

 X ∂2 n 
▽2 rn~r = r ~r
et
∂x2
X ∂ ∂(rn~r) !
=
∂x ∂x
X ∂ !
∂~r ∂r
= rn + ~rnrn−1
∂x ∂x ∂x
X ∂ !
n ∂~r n−1 x
= r + nr ~r
∂x ∂x r

Downloaded From : www.EasyEngineering.net


Downloaded From : www.EasyEngineering.net

214 Engineering Mathematics - II

X ∂  
= rn~i + nrn−2 x~r
∂x
X" ( )#
n−1 ∂r~ n−2 ∂~r n−2 n−3 ∂r
= nr i + n r x + r ~r.1 + (n − 2)r x~r
∂x ∂x ∂x
X x x 
= nrn−1 ~i + nrn−2 x~i + nrn−2~r + n(n − 2)rn−3 x~r
r r
= nr [xi + y~j + z~k] + nr [x~i + y~j + z~k] + 3nr ~r
n−2 ~ n−2 n−2

+ n(n − 2)rn−4~r(x2 + y2 + z2 )

= nrn−2 .~r + nrn−2 .~r + 3nrn−2~r + n(n − 2)rn−4~r.r2

ww = 2nrn−2~r + 3nrn−2~r + n(n − 2)rn−2~r

= (2n + 3n + n2 − 2n)rn−2~r

w.E = (n2 + 3n)rn−2~r

= n(n + 3)rn−2~r.

asy
Example 2.56. If φ and ψ are scalar point functions, prove that φ∇φ is irrotational
and ∇φ × ∇ψ is solenoidal.
En
gin
Solution. We have curl(φ∇φ) = ∇ × (φ∇φ)

∴ φ∇φ is irrotational.
ee
= φ(∇ × ∇φ) + ∇φ × ∇φ = φ.0 + 0 = 0.

rin
Also, div(∇φ × ∇ψ) = ∇.(∇φ × ∇ψ)
g.n
= ∇ψ.(∇ × ∇φ) − ∇φ.∇ × (∇ψ)

= ∇ψ.0 − ∇φ.0 = 0.
et
=⇒ ∇φ × ∇ψ is solenoidal.

Exercise 2 B
!
~r 2
1. Prove that div = . [Jun 2003]
r r

Downloaded From : www.EasyEngineering.net


Downloaded From : www.EasyEngineering.net

Vector Calculus 215

~ is a constant vector, prove that div(A)


2. If A ~ = ~0.
~ = 0 and curlA

3. If ~a is a constant vector, prove that ∇.(~a × ~r) = 0.

4. If F~ = x2~i + y2 ~j + z2~k, find ∇.F~ and ∇ × F.


~

5. Find the divergence and curl of the vector point function xy2~i + 2x2 yz~j − 3yz2~k.

6. If F~ = 3xyz2~i + 2xy3 ~j − x2 yz~k and f = 3x2 − yz, find (i) F.∇


~ f and (ii) ∇.∇ f at
(1, −1, 1).

ww 7. Prove that the vector F~ = z~i + x~j + y~k is solenoidal.

w.E ~ = xyz2~u is solenoidal where ~u = (2x2 + 8xy2 z)~i + (3x3 − 3xy)~j


8. Show that V
−(4y2 z2 + 2x3 z)~k.

asy
9. Show that the vector F~ = 3y4 z2~i + 4x3 z2 ~j − 3x2 y2~k is solenoidal.

En
10. Find the value of a so that the vector F~ = (x + 3y)~i + (y − 2z)~j + (x + az)~k is
solenoidal.
gin
ee
11. Find a such that (3x − 2y + z)~i + (4x + ay − z)~j + (x − y + 2z)~k is solenoidal.

rin [Jun 2004]

g.n
12. Show that F~ = (y2 − z2 + 3yz − 2x)~i + (3xz + 2xy)~j + (3xy − 2xz + 2z)~k is irrotational

et
and solenoidal.

13. Show that the vector 2xy~i + (x2 + 2yz)~j + (y2 + 1)~k is irrotational.

14. Show that F~ = yz~i + zx~j + xy~k is irrotational.

15. If ~a is a constant vector and ~r is the position vector of any point, then prove
that ∇(~a.~r) = ~a and curl(~a × ~r) = 2~a.

16. Find a, b, c if (x + y + az)~i + (bx + 2y − z)~j + (−x + cy + 2z)~k is irrotational.

Downloaded From : www.EasyEngineering.net


Downloaded From : www.EasyEngineering.net

216 Engineering Mathematics - II

17. Find the value of the arbitrary constant a so that the vector F~ = (axy − z3 )~i +
(a − 2)x2 ~j + (1 − a)xz2~k is irrotational.

18. Find the constants a, b and c so that the vector F~ = (x + 2y + az)~i + (bx − 3y −
z)~j + (4x + cy + 2z)~k is irrotational.

19. Show that the vector (3x2 + 2y2 + 1)~i + (4xy − 3y2 z − 3)~j + (2 − y3 )~k is irrotational
and find its scalar potential.

ww 20. Show that F~ = (6xy + z3 )~i + (3x2 − z)~j + (3xz2 − y)~k is irrotational and find its
scalar potential. [May 2005]

w.E
21. Prove that F~ = (y2 cos x + z3 )~i + (2y sin x − 4)~j + 3xz2~k s irrotational and find its
scalar potential.

asy
22. If F~ = xy~i + yz~j + zx~k show that ∇2 F~ = 0.

En
23. If F~ = x2 y~i + y2 z~j + z2 x~k, find curl curlF.
~

gin
24. If F~ = (x + y + 1)~i + ~j − (x + y)~k, prove that F.curl
~ F~ = 0.

ee
25. If u and v are scalar point functions, prove that ∇(u∇v − v∇u) = u∇2 v − v∇2 u.

26. If F~ is solenoidal, prove that ∇ × ∇ × ∇ × F~ = ∇4 F.


~
rin
 1 ! 3
27. Prove that ∇. ~r∇ 3 = 4 .
r r g.n
2.4 Integration of vector functions et
Let f~(t) be a vector function of the scalar variable t.
If f~(t) = f1 (t)Z
Z
~i + f2 (t)~j + f3 (t)~k, then
Z Z
~ ~
f (t)dt = i f1 (t)dt + j~ ~
f2 (t)dt + k f3 (t)dt
Line Integral
~ r) over a curve C where ~r is the
A line integral of a vector point function F(~

Downloaded From : www.EasyEngineering.net


Downloaded From : www.EasyEngineering.net

Vector Calculus 217

Z
position vector of any point on C is defined by {F~ · d~r}.
C
F3~k and ~r = x~i + y~j + z~k, then d~r = dx~i + dy~j + dz~k.
If F~ =ZF1~i + F2 ~j + Z
Now, {F~ · d~r} = (F1 dx + F2 dy + F3 dz).
C C
Note
d~r
is a tangent vector, to the curve C, the line integral C F~ · d~r is
R
1. Since
dt
also called the tangential line integral of F~ over C.

ww
R
2. This line integral is a scalar . The other types of line integrals are C
φd~r
and C F~ × d~r and these integrals are vectors.
R

w.E
Conservative Vector field. If the line integral depends only on the end points
but not on the path C, then F~ is called a conservative vector field.

asy
Result. If F~ is conservative, then curlF~ = ~0. Hence, if F~ is conservative, then
F~ = ∇φ.

Physical interpretation of En R
~ r.
F.d~

gin
c

Work done by a force [ Physical interpretation of


RB
F~ · d~r ] [May 2011]

RB
A
ee A
~ y, z) is a force acting on a particle which is moving along arc AB, then
If F(x,
F~ · d~r gives the total work done by the force F~ in displacing the particle from A
rin
to B.



Worked Examples


g.n
Example 2.57. If F~ = x2~i + xy2 ~j, evaluate F~ · d~r from (0, 0) to (1, 1) along the path
y = x.
R
c et
[Jan 2006, Jun 2005]
Solution.
Given F~ = x2~i + xy2 ~j.

~r = x~i + y~j + z~k.

d~r = dx~i + dy~j + dz~k.

F~ · d~r = x2 dx + xy2 dy.

Downloaded From : www.EasyEngineering.net


Downloaded From : www.EasyEngineering.net

218 Engineering Mathematics - II

Since the path is given by y = x, we have dy = dx and x varies from 0 to 1.

∴ F~ · d~r = x2 dx + xx2 dx.

= x2 dx + x3 dx

= (x2 + x3 )dx.
Z Z1
F~ · d~r = (x2 + x3 )dx
c 0
!1
x3 x4
1 1 7

ww =
03
+
4
+ = .
3 4 12
=

Example 2.58. If F~ = 3xy~i−y2 ~j, evaluate C F~ ·d~r where C is the arc of the parabola
R

w.E
y = 2x2 from (0, 0) to (1, 2).
Solution. F~ = 3xy~i − y2 ~j.
[Jan 2001]

~r = x~i + y~j.
asy
F~ · d~r = 3xydx − y2 dy.
En
Given that the path is along the parabola y = 2x2 =⇒ dy = 4xdx.
Now, F~ · d~r = 3x · 2x2 dx − 4x4 · 4xdx
gin
= 6x3 dx − 16x5 dx
Along y = 2x2 , x varies from 0 to 1.
Z Z
ee 1
x 4 x 6 1
!
rin
F~ · d~r = 3 5

C 0
(6x − 16x )dx = 6 ·

6 16 9 − 16
=− .
7
4
− 16 ·
6 0
g.n
et
= − =
4 6 6 6
Example 2.59. If F~ = 5xy~i + 2y~j, evaluate C F~ · d~r where C is the part of the curve
R

y = x2 between x = 1 and x = 2. [Jun 2003]


Solution. We have F~ = 5xy~i + 2y~j.

~r = x~i + y~j.

d~r = dx~i + dy~j.

F~ · d~r = 5xydx + 2ydy.

Downloaded From : www.EasyEngineering.net


Downloaded From : www.EasyEngineering.net

Vector Calculus 219

= 5x · x2 dx + 2 · x2 · 2xdx [since y = x2 , dy = 2xdx]

= 5x3 dx + 4x3 dx = 9x3 dx

Along y = x2 , x varies from 1 to 2.


Z 2 !2
x4
Z
~ 3 9 9 135
F · d~r = 9x dx = 9 = (16 − 1) = × 15 = .
C 1 4 1 4 4 4

Example 2.60. If F~ = x2~i + xy~j, evaluate C F~ · d~r from (0, 0) to (1, 1) along the line
R

C given by y = x. [May 2010]

ww Solution. Given F~ = x2~i + xy~j


~r = x~i + y~j.

w.E d~r = dx~i + dy~j.

F~ · d~r = x2 dx + xydy.

asy
The curve C is given by y = x.
∴ dy = dx and x varies from 0 to 1.

En
F~ · d~r = x2 dx + xxdx

Z Z1gin
= 2x2 dx.

 x 3 1 1
!
2
F~ · d~r =

ee
2
Now, 2x dx = 2 =2 = .
C 3 0 3 3

rin
0

F~ ·d~r where F~ = 3x2~i+(2xz−y)~j+z~k and C is the straight


R
Example 2.61. Evaluate C
line from A(0, 0, 0) to B(2, 1, 3).
Solution. We have F~ = 3x2~i + (2xz − y)~j + z~k. g.n [May 2008]

~r = x~i + y~j + z~k.

d~r = dx~i + dy~j + dz~k.


et
F~ · d~r = 3x2 dx + (2xz − y)dy + zdz.
The equation of the line joining the points (0, 0, 0) and (2, 1, 3) is

x−0 y−0 z−0


= = = t (say).
2−0 1−0 3−0
x y z
i.e., = = = t.
2 1 3

Downloaded From : www.EasyEngineering.net


Downloaded From : www.EasyEngineering.net

220 Engineering Mathematics - II

Hence, x = 2t, y = t, z = 3t.


dx = 2dt, dy = dt, dz = 3dt.

F~ · d~r = 3.4t2 .2dt + (2.2t.3t − t)dt + 3t.3dt

= (24t2 + 12t2 − t + 9t)dt

= (36t2 + 8t)dt.

As x varies from 0 to 2, we have t varies from 0 to 1.

ww Z
~
Z1
F · d~r = (36t2 + 8t)dt
C

w.E
0
 t 3 1  t2 1 1
!
= 36 +8 = 36 + 4(1) = 12 + 4 = 16.
3 0 2 0 3
R

asy
Example 2.62. Evaluate (x2 + xy)dx + (x2 + y2 )dy where C is the square bounded
C

En
by the lines x = 0, x = 1, y = 0 and y = 1. [Dec 2011]
Solution. Let F1 = x2 + xy and F2 = x2 + y2 .

gin Y

ee
The given integral can be written as
R
(F1 dx + F2 dy). y=1

rin
C
> B
C
C is the rectangle OABC. C contains four
x=0

x=1
>

>

paths namely, the lines OA, AB, BC and


CO. 0
>
y=0 A g.n X

R
c
(F1 dx + F2 dy) =
R
OA
(F1 dx + F2 dy) +
R

RAB
(F1 dx + F2 dy) +
R
BC
(F1 dx + F2 dy) et
+ (F1 dx + F2 dy).
CO
Along OA, y = 0 ⇒ dy = 0 and x varies from 0 to 1.
F1 dx + F2 dy = x2 dx.
!1
R R1 x3 1
2
(F1 dx + F2 dy) = x dx = = .
OA 0 3 0 3
Along AB, x = 1 ⇒ dx = 0 and y varies from 0 to 1.

Downloaded From : www.EasyEngineering.net


Downloaded From : www.EasyEngineering.net

Vector Calculus 221

F1 dx + F2 dy = (y2 + 1)dy.
Z1 !1
y3
Z
1 4
2
+ y 10 = + 1 = .

(F1 dx + F2 dy) = (y + 1)dy =
3 0 3 3
AB 0
Along BC, y = 1 ⇒ dy = 0 and x varies from 1 to 0.
F1 dx + F2 dy = (x2 + x)dx
Z0 !0 !0
x3 x2
Z ! !
2 1 1
(F1 dx + F2 dy) = (x + x)dx = + = 0− + 0−
3 1 2 1 3 2

ww
BC 1
1 1 −2 − 3 5
=− − = =− .
3 2 6 6

w.E
Along CO, x = 0 ⇒ dx = 0 and y varies from 1 to 0.
F1 dx + F2 dy = y2 dy.
Z0 !0

asy y3
Z
2 1 1
(F1 dx + F2 dy) = y dy = =0− =− .
3 1 3 3
CO Z 1

∴ (F1 dx + F2 dy) =
En 1 4 5 1 4 5 8−5 3 1
+ − − = − =
3 3 6 3 3 6 6
= = .
6 2

gin
c

F~ · d~r where F~ = (x2 + y2 )~i − 2xy~j and C is the rectangle


R
Example 2.63. Evaluate C

ee
in the XY plane bounded by x = 0, x = a, y = b and y = 0.
Solution. We have F~ = (x2 + y2 )~i − 2xy~j.

rin
g.n
Y

~r = x~i + y~j.
y=b

et
B
d~r = dx~i + dy~j. C
>
x=0

x=a

F~ · d~r = (x2 + y2 )dx − 2xydy.


>

>

Z Z
F~ · d~r = [(x2 + y2 )dx − 2xydy] 0
>
y=0 A
X

C C

Now, C is the rectangle OABC.


Now C contains four paths namely, the lines OA, AB, BC and CO.
Z Z Z Z Z
F~ · d~r = F~ · d~r + F~ · d~r + F~ · d~r + F~ · d~r.
C OA AB BC CO

Downloaded From : www.EasyEngineering.net


Downloaded From : www.EasyEngineering.net

222 Engineering Mathematics - II

Along OA, y = 0 =⇒ dy = 0 and x varies from 0 to a.


Za !a
x3 a3
Z
F~ · d~r = 2
x dx = = .
OA 3 0 3
0

Along AB, x = a =⇒ dx = 0 and y varies from 0 to b.

Zb !b
y2
Z
F~ · d~r = (−2ay)dy = −2a = −ab2 .
AB 2 0
0

ww Along BC, y = b =⇒ dy = 0 and x varies from a to 0.

Z0 !0

w.E x3 a3 a3
Z
F~ · d~r = 2 2
(x + b )dx = + b2 (x)0a = 0 − + b2 (0 − a) = − − ab2 .
BC 3 a 3 3
a

asy
Along CO, x = 0 =⇒ dx = 0 and y varies from b to 0.

Z0

En
Z
F~ · d~r = (0)dy = 0.
CO

gin
b

a3 a3
Z
∴ F~ · d~r = − ab2 − − ab2 = −2ab2 .
C

ee 3 3

Example 2.64. Show that F~ = (e x z − 2xy)~i − (x2 − 1)~j + (e x + z)~k is a conservative field.
Hence evaluate C F~ · d~r where the end points of C are (0, 1, −1) and (2, 3, 0).
R
rin
Solution. If F~ is conservative, we must have ∇ × F~ = ~0.
g.n
et

~i ~j ~k

∴ ∇ × F~ = ∂
∂x

∂y

∂z

e x z − 2xy 1 − x2 e x + z
! !
∂ ∂ ∂ ∂
= ~i (e + z) − (1 − x ) − ~j
x 2 x x
(e + z) − (e z − 2xy)
∂y ∂z ∂x ∂z
!
∂ ∂
+ ~k (1 − x2 ) − (e x z − 2xy)
∂x ∂y
= (0)~i − (e x − e x )~j + (−2x + 2x)~k = ~0.

Downloaded From : www.EasyEngineering.net


Downloaded From : www.EasyEngineering.net

Vector Calculus 223

∴ F~ is conservative.
Hence F~ = ∇φ ~.
∂φ ∂φ ∂φ
i.e., ~i + ~j + ~k = (e x z − 2xy)~i − (x2 − 1)~j + (e x + z)~k.
∂x ∂y ∂z
Equating the coefficients of ~i, ~j and ~k we get
∂φ
= e x z − 2xy (1)
∂x
∂φ
= 1 − x2 (2)
∂y
∂φ

ww ∂z
= e x + z.

On integration we obtain
(3)

w.E Z
φ = (e x z − 2xy)dx + f1 (y, z)

= ze x − 2y
asyx2
2
+ f1 (y, z)

Z
En
φ = ze x − x2 y + f1 (y, z)

Also, φ = (1 − x2 )dy + f2 (x, z)


(4)

= y − x2 y + f2 (x, z)gin (5)


Z
Again, φ = (e x + z)dz + f3 (x, y)

= ex z +
z2
ee
+ f3 (x, y) rin (6)
2
2
From (4), (5) and (6) we get φ = e x z − x2 y + y + z2 + C, where C is a constant. g.n
Now,
Z

C
~
F · d~r =

=
Z

ZC
∇φ · d~r
!
~i ∂φ + ~j ∂φ + ~k ∂φ .(~idx + ~jdy + ~kdz)
et
C ∂x ∂y ∂z
!
∂φ ∂φ ∂φ
Z
= dx + dy + dz
C ∂x ∂y ∂z
Z
= dφ
C

= [φ](2,3,0)
(0,1,−1)

Downloaded From : www.EasyEngineering.net


Downloaded From : www.EasyEngineering.net

224 Engineering Mathematics - II

!(2,3,0)
x z2 2
= e z+ −x y+y
2 (0,1,−1)
1
= [0 + 0 − 12 + 3] − [−1 + − 0 + 1]
2
1 19
= −9 − = − .
2 2

Example 2.65. Show that F~ = (2xy+z3 )~i+x2 ~j+3xz2~k is a conservative field. Find the
scalar potential and the work done in moving an object in this field from (1, −2, 1)

ww
to (3, 1, 4).
Solution. To prove F~ is conservative, we have to prove that ∇ × F~ = 0.

w.E
~i

Now, ∇ × F~ = ∂x ∂ ∂
~j


~k

asy
∂y ∂z
2xy + z3
x2 3xz2
! !
∂ ∂ 2 ∂ ∂
= ~i
En ∂y
2
(3xz ) − (x ) − j
∂z
~
∂x
2 3
(3xz ) − (2xy + z )
∂z

gin
!
∂ 2 ∂
+ ~k (x ) − (2xy + z3 )
∂x ∂y

= ~0. ee
= (0)~i − (3z2 − 3z2 )~j + (2x − 2x)~k

rin
∴ F~ is conservative.
Hence F~ = ∇φ ~. g.n
i.e., ~i
∂φ
∂x
+ ~j
∂φ
∂y
+ ~k
∂φ
∂z
= (2xy + z3 )~i + x2 ~j + 3xz2~k.
Comparing the coefficients of ~i, ~j and ~k we get
et
∂φ
= 2xy + z3 . (1)
∂x
∂φ
= x2 . (2)
∂y
∂φ
= 3xz2 . (3)
∂z

Downloaded From : www.EasyEngineering.net


Downloaded From : www.EasyEngineering.net

Vector Calculus 225

On integration we obtain
Z
φ = (2xy + z3 )dx + f1 (y, z)

x2
= 2y + z3 x + f1 (y, z)
2
φ = x2 y + xz3 + f1 (y, z). (4)
Z
Also, φ = (x2 )dy + f2 (x, z)

= x2 y + f2 (x, z). (5)

ww Again, φ =
Z
3xz2 dz + f3 (x, y)

w.E = 3x
z3
3
+ f3 (x, y)

= xz3 + f3 (x, y). (6)

asy
From (4), (5) and (6) we get φ = x2 y + xz3 + C, where C is a constant.

En
Z Z
Now, work done = F~ · d~r = ∇φ · d~r
C C

gin = [φ](3,1,4)

(1,−2,1)

= xz3 + x2 y
(3,1,4)

ee (1,−2,1)

rin
= [3(43 ) + 9.1] − [1.1 + 1(−2)]

= 192 + 9 + 1 = 202.

g.n
Example 2.66. Find the work done when the force F~ = (x2 − y2 + x)~i − (2xy + y)~j
moves a particle from the origin to the point (1, 1) along y2 = x.
Solution.
Z
et[Jan 1996]

Work done = F~ · d~r


C
Z  
= (x2 − y2 + x)dx − (2xy + y)dy
C

Along C, y2 = x =⇒ dx = 2ydy.

Downloaded From : www.EasyEngineering.net


Downloaded From : www.EasyEngineering.net

226 Engineering Mathematics - II

y varies from 0 to 1.

Z1  
∴ Work done = (y4 − y2 + y2 )2ydy − (2y3 + y)dy
0
Z1
= (2y5 − 2y3 − y)dy
0
!1
y6 y4 y2
= 2 −2 −
6 4 2

ww
0
1 1 1
= − −
3 2 2

w.E
2
=− .
3

Example 2.67.

asy
Find the work done in moving a particle in the vector field
~ = 3x2~i + (2xz − y)~j + z~k along the straight line from (0, 0, 0) to (2, 1, 3).
A

En
~ = 3x2~i + (2xz − y)~j + z~k.
Solution. Given A
[Jun 2012, Dec 2004]

gin
The equation of the straight line joining (0, 0, 0) and (2, 1, 3) is
x−0 y−0 z−0
x y z
=
2−0 1−0 3−0
= = = t.
2 1 3
=

∴ x = 2t, y = t, z = 3t.
= t (say)

ee rin
dx = 2dt, dy = dt, dz = 3dt.
Now A ~ = 3x2~i + (2xz − y)~j + z~k. g.n
d~r = dx~i + dy~j + dz~k.
~ r = 3x2 dx + (2xz − y)dy + zdz
A.d~
et
= 3.4t2 .2dt + (2.2t.3t − t)dt + 3t.3dt
= 24t2 dt + (12t2 − t)dt + 9tdt

= (36t2 + 8t)dt.
To find the limits for t.
when x = 0, t = 0.

Downloaded From : www.EasyEngineering.net


Downloaded From : www.EasyEngineering.net

Vector Calculus 227

when x = 2, t = 1. Z
∴ Now, work done = ~ r
A.d~
c
Z1
= (36t2 + 8t)dt
0
!1 !1
t3 t2
= 36. + 8.
3 0 2 0
1 1
= 36. + 8.

ww
3 2
= 12 + 4

w.E
Exercise 2 C
= 16.

1. Evaluate
R
asy
F~ · d~r where F~ = (3x2 + 6y)~i − 14yz~j + 20xz2~k, from (0, 0, 0) to (1, 1, 1)

En
C
along the curve C given by x = t, y = t2 , z = t3 .

2. Evaluate the line integral


gin R
C
(y2 dx − x2 dy) around the triangle whose vertices

joining (0, 0, 0) and (1, 1, 1).


ee
are (1, 0), (0, 1), (−1, 0) in the positive sense.

3. If F~ = (3x2 + 6y)~i − 14yz~j + 20xz2~k, evaluate


R
C
rin
F~ · d~r where C is the straight line

4. Show that the vector field F~ = x2~i + y2 ~j + z2~k is conservative vector field. g.n
R
5. Evaluate (xdy − ydx) around the circle x2 + y2 = 1.
et[Jun 2001]

F~ · d~r where F~ = xy~i + (x2 + y2 )~j, and C is the arc of the parabola
R
6. Evaluate C
y = x2 − 4 from (2, 0) to (4, 12) in the xy plane.

7. If F~ = 3xy~i − y3 ~j, evaluate F~ · d~r where C is the curve y = 2x2 in the xy plane
R
C
from (0, 0) to (1, 2).

Downloaded From : www.EasyEngineering.net


Downloaded From : www.EasyEngineering.net

228 Engineering Mathematics - II

8. Given the vector field F~ = xz~i+yz~j+z2~k, evaluate F~ ·d~r from the point (0, 0, 0)
R
C
to (1, 1, 1) where C is the curve (i) x = t, y = t2 , z = t3 and (ii) the straight line
from (0, 0, 0) to (1, 1, 1).

9. Find the total work done in moving a particle in a force field F~ = (2x − y + z)~i +
(x + y − z)~j + (3x − 2y − 5z)~k along a circle C in the xy plane x2 + y2 = 9, z = 0.
[Jan 2010]

10. If F~ = (4xy − 3x2 z2 )~i + 2x2 ~j − 2x3 z~k, check whether the integral F~ · d~r is
R

ww independent of the path C.


C
[May 2000]

w.E
11. Show that the vector field F~ = (y+y2 +z2 )~i+(x+z+2xy)~j+(y+2xz)~k is conservative
and find its scalar potential.

asy
12. Find the work done in moving a particle once round the ellipse
x2 y2
+
16 9
En
= 1 if the vector field is F~ = (3x−4y+2z)~i+(4x+2y−3z2 )~j+(2xz−4y2 +z3 )~k.

2.5 Green’s theorem in a plane gin


ee rin
In the previous semester, we have learned the methodology of evaluating double
integral over a region R. Green’s theorem demonstrates an easy way of evaluating

g.n
line integrals. It provides a relation between a double integral over a region R in
the xy plane and the line integral over a closed curve C which encloses the given
region R. et
[Dec 2014, Dec 2013]
Statement. If P(x, y) and Q(x, y) are continuous functions with continuous
partial derivatives in a region RZin the xy plane" and on its boundary C which
∂Q ∂P
is a simple closed curve, then (Pdx + Qdy) = ( − )dxdy where C is
C R ∂x ∂y
described in the anticlockwise sense.
" Z
1
Corollary. Area of the region bounded by C is dxdy = (xdy − ydx).
R 2 C
[Dec 2011]

Downloaded From : www.EasyEngineering.net


Downloaded From : www.EasyEngineering.net

Vector Calculus 229

∂P ∂Q
Proof. Assume P = −y, Q = x. = −1, = 1.
∂y ∂x
By Green’s theorem we have
" 
∂Q ∂P 
Z
∴ (Pdx + Qdy) = − dxdy
∂x ∂y
ZC "R
i.e., (−ydx + xdy) = (1 + 1)dxdy
C
Z "R

(xdy − ydx) = 2 dxdy


Z C
" R
1
(xdy − ydx) = dxdy.

ww !
2 C
But R dxdy = Area of the region R bounded by C.
R

w.E ∴ Area =
1
Z

2 C

(xdy − ydx).

asy Worked Examples


✍ ✌
R

En
Example 2.68. Using Green’s theorem, evaluate C
[(2x − y)dx + (x + y)dy] where
C is the circle x2 + y2 = 4 in the xy plane.

gin
Solution. Comparing the given integral with Green’s theorem we have
P = 2x − y and Q = x + y.
∂P
∂y
= −1
∂Q
∂x
= 1.
By Green’s theorem,
ee
"  rin
∂Q ∂P 
g.n
Z
(Pdx + Qdy) = − dxdy
C ∂x ∂y

et
R
Z "
i.e., {(2x − y)dx + (x + y)dy} = (1 + 1)dxdy
C
R
"
=2 dxdy
R

= 2 × Area of the region

= 2 × Area of the circle with radius 2

= 2 × π × 22 = 8π.

Downloaded From : www.EasyEngineering.net


Downloaded From : www.EasyEngineering.net

230 Engineering Mathematics - II

R
Example 2.69. Using Green’s theorem, evaluate C
[(2x2 − y2 )dx + (x2 + y2 )dy] where
C is the boundary of the square enclosed by the lines x = 0, y = 0, x = 2 and y = 2.
Solution. Comparing the given integral with Green’s theorem we have
P = 2x2 − y2 and Q = x2 + y2 .
∂P ∂Q
= −2y = 2x.
∂y ∂x
By Green’s theorem,
" 
∂Q ∂P 
Z
(Pdx + Qdy) = − dxdy

ww i.e.,
Z
C

{(2x2 − y2 )dx + (x2 + y2 )dy} =


R
"
∂x ∂y

(2x + 2y)dxdy = 2
"
(x + y)dxdy

w.E C
R
Z2 Z2 Z2  2
x 2
R

asy =2
0
Z2 
0
(x + y)dxdy = 2
2
+ xy dy
0
0

En =2
 
2 + 2y dy = 2 2y + 2
y2 2
2 0
dy

gin 0

= 2[4 + 4] = 16.

ee
Example 2.70. Using Green’s theorem, evaluate
the closed curve of the region y2 = x and x2 = y.
R
C
[(x2 − y2 )dx + 2xydy] where C is

rin
Solution.
g.n
Comparing the given integral with
Green’s theorem we have
Y

2
y
=
x
et
(1, 1)

P = x2 − y2 ,Q = 2xy. y = x2
∂P ∂Q (0, 0)
X
= −2y = 2y.
∂y ∂x
∂Q ∂P
− = 2y + 2y = 4y.
∂x ∂y

Downloaded From : www.EasyEngineering.net


Downloaded From : www.EasyEngineering.net

Vector Calculus 231

"
∂Q ∂P
Z
By Green’s theorem (Pdx + Qdy) = − (
)dxdy.
C R ∂x ∂y
Z " "
i.e., {(x2 − y2 )dx + 2xydy} = 4ydxdy = 4ydydx
C R R
√ √
Z 1 Z x Z 1 " 2# x
y
=4 ydydx = 4 dx
x=0 y=x2 0 2 x2
1 5 1
Z " 2 #
x x
(x − x4 )dx = 2
=2 −
0 2 5 0

ww
" # " #
1 1 5−2 3
=2 − =2 = .
2 5 10 5
Z

w.E
Example 2.71. Evaluate by Green’s theorem
π
the rectangle with vertices (0, 0), (π, 0), π, , 0, .
2
C
e−x (sin ydx + cos ydy) where C being
π
2
[Jun 1995]
Solution.
asy
Comparing the given integral with
En C
Y

(0, π/2) B(π, π/2)

Green’s theorem we have


P = e−x sin yQ = e−x cos y. gin y = π/2

x = π/2
ee
x=0

∂P ∂Q
= e−x cos y = −e−x cos y.
∂y ∂x
By Green’s theorem
"  rin X

g.n
∂Q ∂P  o
Z y=0 A(π, 0)
(0, 0)
(Pdx + Qdy) = − dxdy
∂x ∂y
C
"R
=

= −2
"
R
(−e−x cos y − e−x cos y)dxdy

R
e−x cos ydxdy
et
Z π Z π
2
= −2 e−x cos ydxdy
y=0 x=0
Z π
2
= −2 cos y(−e−x )πx=0 dy
y=0
Z π
2
=2 cos y(e−π − 1)dy
0

Downloaded From : www.EasyEngineering.net


Downloaded From : www.EasyEngineering.net

232 Engineering Mathematics - II

π
= 2(e−π − 1)(sin y)02
π
= 2(e−π − 1)(sin − sin 0)
2
= 2(e−π − 1).
Z
Example 2.72. Evaluate by Green’s theorem {(sin x − y)dx − cos xdy} where C is
π  π  C
the triangle with vertices (0, 0), , 0 , , 1 . [Jun 2003]
2 2
Solution. Comparing the given integral with Green’s theorem we have

ww P = sin x − yQ = − cos x.
∂P
= −1
∂Q
= sin x.

w.E ∂y
By
∂x
Z Green’s theorem
(Pdx + Qdy) =
" we have
(
∂Q ∂P
− )dxdy.
C

i.e.,
Z
asy R ∂x ∂y

{(sin x − y)dx − cos xdy} =


"
(sin x + 1)dxdy (1)
C

En R

gin
Y
y−0 x−0
Equation of OB is = π B(π/2, 1)
1−0 2 −0

ee
y x
= π
/2

1 x = π/2
πy

rin
=
x

2
y = x.
π
x− π
Equation to AB is
x − π
y−0
1−0
= π 2π
2 − 2
π
(0, 0) o
g.n
A(π/2, 0)
X

et
y 2
= ⇒x= .
1 0 2

Now (1) becomes


π
Z Z 1Z 2
{(sin x − y)dx − cos xdy} = (sin x + 1)dxdy
C 0 x= πy
2
Z 1  π2
= − cos x + x πy dy
0 2
1n
π π πy πy o
Z
= − cos + − (− cos + ) dy
0 2 2 2 2

Downloaded From : www.EasyEngineering.net


Downloaded From : www.EasyEngineering.net

Vector Calculus 233

1
π πy πy
Z
= ( + cos − )dy
0 2 2 2
πy 
π 1  sin 2 1 π  y2 1
= (y)0 + π −
2 2
0 2 2 0
π 2 π π
= + (sin − sin 0) − (1 − 0)
2 π 2 4
π 2 π
= + ·1−
2 π 4
π 2
= + .
4 π

ww Example 2.73. Find the area of the circle of radius a using Green’s theorem.
Solution.

w.E
Z
1
We know that, the area enclosed by a simple closed curve C is (xdy − ydx).
2 C
Here, the curve is a circle of radius a given by x2 + y2 = a2 .

asy
Consider the parametric coordinates x = a cos θ, y = a sin θ, 0 ≤ θ ≤ 2π
dx = −a sin θdθ, dy = a cos θdθ

Area of the circle = En


1 2π
Z
(xdy − ydx)

=
2 0
Z
gin
1 2π
a cos θa cos θdθ − a sin θ(−a sin θ)dθ

=
2 0
1 2π 2
Z

2 0
ee
(a cos2 θ + a2 sin2 θ)dθ
rin
1 2 2π

g.n
Z
1
= a dθ = a2 (θ)2π
0
2 0 2
1
= a2 · 2π = πa2 .
2

Example 2.74. Find the area of the ellipse


x2 y2
+
a2 b2
et
= 1 by using Green’s theorem.
[May 2004]
Solution. By Green’s theorem,
R
Area of the ellipse = 12 (xdy − ydx), where C is the ellipse.
c
The parametric coordinates of the ellipse are
x = a cos θ, y = b sin θ, 0 ≤ θ ≤ 2π

Downloaded From : www.EasyEngineering.net


Downloaded From : www.EasyEngineering.net

234 Engineering Mathematics - II

dx = −a sin θdθ, dy = b cos θdθ.


Z2π
1
∴ Area of the ellipse = (a cos θb cos θdθ − b sin θ(−a sin θdθ))
2
0
Z2π
1
= (ab cos2 θ + ab sin2 θ)dθ
2
0
Z2π
1
= ab dθ

ww 2
1
= ab[θ]2π
0

w.E 2
1
0

= ab2π = πab.
2

asy
Example 2.75. Using Green’s theorem, find the area bounded between the
parabolas y2 = 4x and x2 = 4y.
Solution.
En
gin
The given curves are y2 = 4x and x2 = 4y.
Squaring the second equation we get
x4 = 16y2 = 16.4x = 64x
ee Y

C2
x2 = 4y

rin (4, 4)
>

x4 − 64x = 0
g.n
>

C1
X
x(x3 − 64) = 0 (0, 0)

x = 0, x3 = 64.
x = 4. et
y2 = 4x

When x = 0, y = 0,
When x = 4, y = 4.
Hence, the points of intersection are (0, 0) and (4, 4).
Let A be (4, 4).
Now C is the curve OAO.

Downloaded From : www.EasyEngineering.net


Downloaded From : www.EasyEngineering.net

Vector Calculus 235

Divide C into two curves C1 and C2 where C1 represents the portion between O and
A along x2 = 4y and C2 represents the portion between A and O along y2 = 4x.
By Green’s theorem,
Z
1
Area = (xdy − ydx)
2 C
"Z Z #
1
= (xdy − ydx) + (xdy − ydx)
2 C1 C2
1
= [I1 + I2 ] where
2

ww
Z
I1 = (xdy − ydx)
ZC1

w.E I2 =

Consider I1 =
Z C2
(xdy − ydx).

(xdy − ydx)

Along C1 , x2 = 4y.
C1

asy
i.e., y =
x2
4 En
1 x
dy = 2xdx = dx.
4 2
gin
x varies from 0 to 4.

∴ I1 =
Z4
x x2
x dx − dx
2 4
ee!

rin
0
Z4
x2 x2
!
g.n
=

=
0
Z4
2

x2
dx − dx

dx =
4

1 x3
!4 et
4 4 3 0
0
1 3 1 16
= [4 − 0] = (64) = .
12
Z 12 3
Consider I2 = (xdy − ydx)
C2
2
Along C2 , y = 4x

Downloaded From : www.EasyEngineering.net


Downloaded From : www.EasyEngineering.net

236 Engineering Mathematics - II

y2
i.e., x =
4
1 y
dx = 2ydy = dy.
4 2

y varies from 4 to 0.

Z0
y2
!
y
∴ I2 = dy − y dy
4 2

ww 4
Z0
y2 y2
! Z0
y2
!

w.E
= − dy = − dy
4 2 4
4 4
Z4 !4
y2 1 y3
!
1 16
=
0
4
asy
dy =
4 3 0 12
= × 64 = .
3

En
1
∴ Area = [I1 + I2 ]
2" #

gin
1 16 16 16
= + = .
2 3 3 3

ee
Example 2.76. Apply Green’s theorem in a plane to evaluate
where C is the boundary of the area between y = x2 and y = x.
Z

C
rin
((xy + y2 )dx + x2 dy)
[Nov 2002]
Solution.
g.n
On solving the two equations we get the
Y et
y = x2
y=x

points of intersection as (0, 0) and (1, 1). A(1, 1)

By
Z Green’s theorem " 
∂Q ∂P 
{(Pdx + Qdy)} = − dxdy,
C R ∂x ∂y
X
Now, P = xy + y2 and Q = x2 . (0, 0)

∂P ∂Q
= x + 2y, = 2x.
∂y ∂x

Downloaded From : www.EasyEngineering.net


Downloaded From : www.EasyEngineering.net

Vector Calculus 237

" "
2 2
Now, (xy + y )dx + x dy = (2x − x − 2y)dxdy
R "R
= (x − 2y)dxdy
R
Z1 Zx
= (x − 2y)dydx
0 y=x2

Z1 !x
y2

ww
= xy − 2 dx
2 y=x2
0
Z1

w.E =
0
[(x2 − x2 ) − (x3 − x4 )]dx

asy
Z1 !1
4 3x5 x4
= (x − x )dx = −
5 4 0
0

En =
1 1 1
− =− .
5 4 20
Example
Z
gin
2.77. Verify Green’s theorem in a plane for the integral

Solution. By Green’s theorem,


circle x2 + y2 = 4.
Z

C
ee
((x − 2y)dx + xdy) taken around the circle x2 + y2 = 4.

{(Pdx + Qdy)} =
" 
∂Q ∂P 
R ∂x

∂y
[May 2000]

dxdy, C is the
rin
Now, P = x − 2y
∂P ∂Q
Q = x.
g.n
∂y
= −2

Now,
" 

R ∂x
∂x


=1

∂Q ∂P 
∂y
dxdy =
"

"
R
(1 + 2)dxdy
et
=3 dxdy
R

= 3 · Area of the circle = 3 · π · 22 = 12π.


R
Let us evaluate C
{(Pdx + Qdy)}.
The parametric equations are

Downloaded From : www.EasyEngineering.net


Downloaded From : www.EasyEngineering.net

238 Engineering Mathematics - II

x = 2 cos θ y = 2 sin θ
dx = −2 sin θdθ dy = 2 cos θdθ.
Since C is the entire circle, θ varies from 0 to 2π.
Z Z 2π
Now, {(Pdx + Qdy)} = (2 cos θ − 2 · 2 sin θ)(−2 sin θdθ) + (2 cos θ2 cos θdθ)
C 0
Z 2π
= (−4 sin θcosθ + 8sin2 θ + 4 cos2 θ)dθ
0
Z 2π
= (−4 sin θcosθ + 8sin2 θ + 4(1 − sin2 θ)dθ

ww =
Z
0

(−2 sin 2θ + 4 + 4 sin2 θ)dθ

w.E
0
Z 2π
1 − cos 2θ
= (−2 sin 2θ + 4 + 4 · )dθ
0 2

asy
Z 2π
= (−2 sin 2θ + 4 + 2 − 2 cos 2θ)dθ
0
!2π !2π
=
En 6(θ)2π
0 −2
− cos 2θ
2 0
− 2
sin 2θ
2 0

gin
= 6 · 2π + [cos 4π − cos 0 − (sin 4π − sin 0)]

= 12π + (1 − 1) − (0 − 0) = 12π = LHS .

Hence, Green’s theorem is verified. ee rin


Example
Z
2 2
2.78. Verify Green’s theorem in the plane for
[(3x − 8y )dx + (4y − 6xy)dy] where C is the boundary of the region bounded by g.n
C
x = 0, y = 0, x + y = 1.

Z By Green’s theorem,
Solution. " 
∂Q ∂P 
et
[Jun 2012, May 2011, Dec 2010, Dec 2007]

(Pdx + Qdy) = − dxdy.


C R ∂x ∂y
P = 3x2 − 8y2 Q = 4y − 6xy.
∂P ∂Q
= −16y = −6y.
∂y ∂x
"  " "
∂Q ∂P 
Now, − dxdy = (−6y + 16y)dxdy = 10ydxdy
R ∂x ∂y R R

Downloaded From : www.EasyEngineering.net


Downloaded From : www.EasyEngineering.net

Vector Calculus 239

" Z 1 Z 1−x
= 10 ydydx = 10 ydydx
R x=0 y=0
Z 1  y2 1−x Z 1
10
= 10 (1 − x)2 dx dx =
2 0
x=0 0 2
h (1 − x)3 i1 5 5
=5 = − [0 − 1] = .
−3 0 3 3
R R R R
Now, C (Pdx + Qdy) = OA (Pdx + Qdy) + AB (Pdx + Qdy) + BO (Pdx + Qdy).

ww
Y

Along OA, y = 0 ⇒ dy = 0 and x varies


B(0, 1)
from 0 to 1.

w.ER R1  3 1
∴ OA (Pdx + Qdy) = 0 3x2 dx = 3 x3 = 1.
x+y=1

x=0
0
Along AB, y = 1 − x, dy = −dx, x varies
from 1 to 0
asy (0, 0) o y=0 A(1, 0)
X


Z
(Pdx + Qdy) =
Z 0

En
(3x2 − 8(1 − x)2 )dx + (4(1 − x) − 6x(1 − x))(−dx)

gin
AB 1
Z 0
= {3x2 − 8 − 8x2 + 16x − (4 − 4x − 6x + 6x2 )}dx

ee
1
Z 0
= (−5x2 + 16x − 8 − 4 + 10x − 6x2 )dx
1
Z 0
(−11x2 + 26x − 12)dx rin
g.n
=
1
 x 3 0  x2 0  0
= −11 + 26 − 12 x
11
3 1 2 1 1

= − (0 − 1) + 13(0 − 1) − 12(0 − 1) =
11
3
8
11
3
− 13 + 12 et
= −1= .
3 3

Along BO, x = 0 ⇒ dx = 0 , y varies from 1 to 0.


Z Z 0 h y2 i0
(Pdx + Qdy) = 4ydy = 4 = 2[0 − 1] = −2.
BO
Z 1 2 1
8 8 5
∴ (Pdx + Qdy) = 1 + − 2 = − 1 = = RHS .
C 3 3 3

Downloaded From : www.EasyEngineering.net


Downloaded From : www.EasyEngineering.net

240 Engineering Mathematics - II

~ = (x2 + y2 )~i − 2xy~j taken around the


Example 2.79. Verify Green’s theorem for V
rectangle bounded by the lines x = ±a,Zy = 0, y = b. [Nov 2012]
Solution. Consider the line integral ~ · d~r where C is the given rectangle.
V
C
Z Z
∴ ~ · d~r =
V [(x2 + y2 )dx − 2xydy]
C C
Z
= (Pdx + Qdy) where P = x2 + y2 , Q = −2xy.
C
"

ww
!
∂Q ∂P
Z
By Green’s theorem (Pdx + Qdy) = − dxdy.
C ∂x ∂y
R

w.E ∂P
∂y
= 2y,
∂Q
∂x
= −2y.

asy
Y

" ! "
∂Q ∂P
Now,
R
∂x

∂y
dxdy =
R
" En
(−2y − 2y)dxdy D
y=b
C

gin
x = −a

x=a
=− 4ydxdy
R

=− ee
Za Zb

x=−a y=0
4ydydx
A y=0

rin
B
X

= −4
Za "
y2
2
#b
dx
g.n
et
0
x=−a
a
b2
Z
= −4 dx
2
−a

= −2b2 [x]a−a = −2b2 [a + a] = −4ab2 . (1)


Z
Now let us evaluate (Pdx + Qdy).
C
Z Z Z Z Z
(Pdx + Qdy) = (Pdx + Qdy) + (Pdx + Qdy) + (Pdx + Qdy) + (Pdx + Qdy).
C
AB BC CD DA

Downloaded From : www.EasyEngineering.net


Downloaded From : www.EasyEngineering.net

Vector Calculus 241

Along AB, y = 0, dy = 0 and x varies from −a to a.


Za " 3 #a
2a3
Z
2 x 1
(Pdx + Qdy) = x dx = = (a3 + a3 ) = .
3 −a 3 3
AB −a

Along BC, x = a, dx = 0 and y varies from 0 to b.


Zb #b
y2
Z "
(Pdx + Qdy) = −2aydy = −2a = −ab2 .
2 0
BC 0

Along CD, y = b, dy = 0 and x varies from a to −a.

ww ∴
Z Z−a
2 2
(Pdx + Qdy) = (x + b )dx =
" 3
x
3
+b x 2
#−a

w.E
a
CD a
a3 a3
!
= − − ab2 − + ab2
3 3

asy a3
= − − ab2 −
3
a3
3
− ab2

En =−
2a3
3
− 2ab2 .

gin
Along DA, x = −a, dx = 0 and y varies from b to 0.
Z0

ee
#0
y2
Z "
∴ (Pdx + Qdy) = 2aydy = 2a = a(0 − b2 ) = −ab2 .
2
DA
Z
b
2a3
− ab2 −
2a3
b

− 2ab2 − ab2 = −4ab2 . rin


g.n
∴ (Pdx + Qdy) = (2)
C 3 3
From (1) and (2) we have
Z

C
(Pdx + Qdy) =
"

R
∂Q ∂P
∂x

∂y
!
dxdy. et
∴ Green’s theorem is verified.
R
Example 2.80. Verify Green’s theorem in the plane for C
(3x2 − 8y2 )dx + (4y − 6xy)dy
where C is the boundary of the region bounded by the curves x = y2 , y = x2 .
[Jun 2010]
Solution. By Green’s theorem,

Downloaded From : www.EasyEngineering.net


Downloaded From : www.EasyEngineering.net

242 Engineering Mathematics - II

" ! Y

∂Q ∂P
Z
(Pdx + Qdy) = − dxdy.
C ∂x ∂y 2 (1, 1)

>
y
R x=
Now, P = 3x2 − 8y . Q = 4y − 6xy.
2

>
y = x2
∂P ∂Q
" = −16y. ! = −6y. " " (0, 0) O X

∂y ∂Q ∂P ∂x
− dxdy = (−6y + 16y)dxdy = 10ydxdy
∂x ∂y
R R R

Z1 Z y Z1 √
y
= 10ydxdy = 10 y[x]y2 dy

ww y=0 x=y2

Z1

y=0

Z1 

w.E

= 10 y( y − y2 )dy = 10 y3/2 − y3 dy
0 0
 y5/2 y4 1
 

asy
!
2 1
= 10  5 −  = 10 −
2
4 5 4
0

En
!
8−5 3
= 10 = . (1)
20 2

gin
Z
Now let us evaluate (Pdx + Qdy).
C

ee
Z Z Z
We have (Pdx + Qdy) = (Pdx + Qdy) + (Pdx + Qdy).
C
OA
Along OA, y = x2 , dy = 2xdx and x varies from 0 to 1.
AO

rin

Z
(Pdx + Qdy) =
Z1 n
(3x2 − 8x4 )dx + (4x2 − 6x · x2 )2xdx
o
g.n
OA

=
0
Z1

0
(3x2 − 8x4 + 8x3 − 12x4 )dx.
et
#1
x3 x5 x4 x5
"
= 3· − 8 + 8 − 12
3 5 4 5 0
8 12
=1− +2−
5 5
20
=3− = 3 − 4 = −1.
5

Downloaded From : www.EasyEngineering.net


Downloaded From : www.EasyEngineering.net

Vector Calculus 243

Along AO, x = y2 , dx = 2ydy and y varies from 1 to 0.


Z Z0 n o
∴ (Pdx + Qdy) = (3y4 − 8y2 )2ydy + (4y − 6y2 · y)dy
AO 1
Z0
= (6y5 − 16y3 + 4y − 6y3 )dy
1
Z0
= (6y5 − 22y3 + 4y)dy

ww "
= 6·
y6
6
1

y4
− 22 + 4
4
y2
#0

2 1

w.E "
11
= y6 − y4 + 2y2
2
#0

! 1

asy
!
11 11 5
=0− 1− +2 =− 3− = .
2 2 2

En
Z
5 3
∴ (Pdx + Qdy) = −1 + = . (2)
C 2 2

gin
From (1) and (2)
Z

C
ee
(Pdx + Qdy) =
"

R
∂Q ∂P
∂x

∂y
!
dxdy.
rin
∴ Green’s theorem is verified.
g.n
Example 2.81. If C is a simple closed curve in the xy plane not enclosing the
R
C
y~i − x~j
origin, show that F~ · d~r = 0 where F~ = 2
x + y2
. et
y~i − x~j  ~ 
Solution. F~ · d~r = 2 · dx i + dy~j
x + y2
ydx − xdy y x
= 2 2
= 2 2
dx − 2 dy
x +y x +y x + y2
Z Z Z
y x
F~ · d~r = 2 2
dx − 2 dy = Pdx + Qdy
x +y x + y2
C C C

Downloaded From : www.EasyEngineering.net


Downloaded From : www.EasyEngineering.net

244 Engineering Mathematics - II

y x
where P = Q=−
x2 + y2 x2 + y2
   
∂P x2 + y2 · 1 − y · 2y ∂Q x2 + y2 · 1 − x · 2x
= =−
∂y x2 + y2 2 ∂x x2 + y2 2
 

x2 + y2 − 2y2 x2 + y2 − 2x2
= =
x2 + y2 2 x2 + y2 2
 

x2 − y2 x2 − y2
=  . =  .
x2 + y2 2 x2 + y2 2

ww By Green’s theorem,
Z
(Pdx + Qdy) =
" 
∂Q ∂P 
∂x

∂y
dxdy

w.E
i.e.,
Z
2
y
2
C

dx − 2
x
x + y2
dy =
R
"  2
 x − y2
  − 2
x2 + y2 2

x2 − y2 
  dxdy = 0.
x + y2 2
C x +y

asy
R

Example 2.82. Apply Green’s theorem to evaluate


Z
[(2x2 − y2 )dx + (x2 + y2 )dy]

En C
where C is the boundary of the area enclosed by the x−axis and the upper half of
the circle x2 + y2 = a2 .
gin
Solution. Comparing
integral with Green’s theorem we have
the
ee given
R
Y

r
rin
P=
∂P
∂y
2x2
= −2y,
y2
− and Q =
∂Q
∂x
= 2x.
x2 + y2 .
O
θ
g.n
a X

By Green’s theorem,
Z
(Pdx + Qdy) =
" 
∂Q ∂P 
− dxdy
et
C ∂x ∂y
R
Z "
i.e., {(2x2 − y2 )dx + (x2 + y2 )dy} = (2x + 2y)dxdy
C
R
"
=2 (x + y)dxdy
R

Downloaded From : www.EasyEngineering.net


Downloaded From : www.EasyEngineering.net

Vector Calculus 245

By changing into polar coordinates we have x = r cos θ, y = r sin θ, dxdy = rdrdθ.


Along R, r varies from 0 to a and θ varies from 0 to π.
Z Za Zπ
2 2 2 2
{(2x − y )dx + (x + y )dy} = 2 (r cos θ + r sin θ)rdrdθ
C
0 0
Za Zπ
2
=2 r dr (cos θ + sin θ)dθ
0 0
Za  π
r2 dr sin θ − cos θ

ww
=2
0
0
Za

w.E =2
0
Za
r2 dr (sin π − sin 0 − cos π + cos 0)

!a

asy r3 4 3  4a3
2
=2 r dr · 2 = 4 = a −0 = .
3 0 3 3
0

Example
Z 2.83. Using
En
Green’s theorem evaluate the integral

gin
  2 2
(2x − y)dx + (x + 3y)dy where C is the boundary of the ellipse x + 4y = 4.
C
Solution. Comparing the given integral with Green’s theorem we have
P = 2x − y and Q = x + 3y.
∂P
∂y
= −1,
∂Q
∂x
By Green’s theorem,
= 1. ee rin
Z
(Pdx + Qdy) =
" 
∂Q ∂P 
− dxdy g.n
i.e.,
Z

C
C

(2x − y)dx + (x + 3y)dy =


R
"
∂x ∂y

(1 + 1)dxdy
R
et
"
=2 dxdy
R
x2 y2
" #
= 2 × Area of the ellipse ∵ + = 1 ⇒ a = 2, b = 1
4 1
= 2 × πab = 2 × π × 2 × 1 = 4π.

Downloaded From : www.EasyEngineering.net


Downloaded From : www.EasyEngineering.net

246 Engineering Mathematics - II

Exercise 2 D
R
1. Verify Green’s theorem in the plane for C
[x2 (1 + y)dx + (x3 + y3 )dy] where C is
the square bounded by x = ±a, y = ±a. [Dec 2006]
R
2. Verify Green’s theorem in the plane for C
[(x2 − y2 )dx + 2xydy] where C is the
boundary of the rectangle in the xy plane bounded by the lines x = 0, x = a,
y = 0, y = b. [Jan 2004]

ww
R
3. Verify Green’s theorem in the plane for C
[(x − 2y)dx + xdy] where C is the
unit circle x2 + y2 = 1. [Jun 2007]

w.E 4. Verify Green’s theorem in the plane for


R
C
is the square with vertices (0, 0), (2, 0), (2, 2), (0, 2).
[(x2 − xy3 )dx + (y2 − 2xy)dy] where C
[Apr 2002]

asy
5. Verify Green’s theorem for
R
C
[(x2 − 2xy)dx + (x2 y + 3)dy] where C is the

En
boundary of the region bounded by y2 = 6x and x = 2. [Jan 2005]

gin
R
6. Apply Green’s theorem to evaluate C
[(2xy − x2 )dx + (x2 + y2 )dy] where C is the
closed curve of the region bounded by y = x2 and y2 = x.

7. Apply Green’s theorem to evaluate ee R


C
the square formed by the lines x = ±1 and y = ±1.
[(x2 + 1 + y)dx + (x3 + y3 )dy] where C is

rin
8. Using Green’s theorem find the area between the curves y2 = 4ax and x2 = 4ay.
g.n
2.6 Surface Integral et
Consider a domain D in the cartesian coordinate system. Let a surface S be
bounded by a simple closed curve C. Because of the space curve C, the surface
S can be visualized as two sided, one arbitrarily chosen side can be regarded as
positive and the other as negative. If S is closed, then the outer side of S can
be taken as positive and the inner side can be considered as the negative side.

Downloaded From : www.EasyEngineering.net


Downloaded From : www.EasyEngineering.net

Vector Calculus 247

At any point P on the outer side of the surface, the unit normal ~n drawn at P is
considered as the outward drawn normal and is along the positive direction. The
inward drawn normal will be along the negative direction.
An integral which is evaluated over a surface is called a surface integral.
Definition. Let S be a surface of finite area which is smooth or piecewise smooth
~ y, z)
[Eg: Sphere is a smooth surface, cube is a piecewise smooth surface]. Let F(x,
be a vector point function defined at each point of S. Let P be any point on the
surface and ~n be the outward unit normal at P. Then the surface integral of F~ over

ww !
S is defined as S F~ · ~nds.
Note. If we associate d~s with the differential of the surface area ds such that

w.E
|d~s| = ds and direction of ds is ~n then d~s = ~n · ds
!
F~ · ~nds =
!
F~ · d~s.

asy
S S
!
Note. S
F~ · d~s is called the normal flux of F~ through the surface S.

En
gin
2.6.1 Evaluation of surface integral

ee Z

P
k
~n

rin
Y
g.n
et
O
R

dx dy

Let R be the orthonormal projection of S on the xy plane. Then, the element surface
ds is projected to an element area dxdy in the xy plane. Now dxdy = dscosθ where θ
is the angle between the planes of ds and the xy plane.
Let ~n be the unit normal to ds and ~k be the unit normal to the xy plane. Angle

Downloaded From : www.EasyEngineering.net


Downloaded From : www.EasyEngineering.net

248 Engineering Mathematics - II

between ~n and ~k is θ.

~n · ~k
∴ cosθ = = ~n · ~k
|~n||~k|
∴ dxdy = ds|(~n · ~k)| (Taking the acute angle)
dxdy
∴ ds =
|~n · ~k|
" " n
dxdy o
∴ F~ · ~nds = F~ · ~n
S R |~n · ~k|

ww Similarly,
" the "
{F~ · ~nds} =
projection on the yz plane and the zx planes give
F~ · ~n
dydz

w.ES "

and F~ · ~nds =
R" |~n · ~i|
F~ · ~n
dxdz
.
|~n · ~j| "

asy " " "


S R
dxdy dxdz dydz
Corollary. Surface area = ds = = = .
S ~
n · k|
R |~ R |~ ~
n · j| n · ~i|
R |~

En ✎ ☞

" gin ✍
Worked Examples

Example 2.84. Evaluate


ee
S
{F~ · ~nds} if F~ = 4y~i + 18z~j − x~k and S is the surface of
the portion of the plane 3x + 2y + 6z = 6 contained in the first octant.
Solution. rin [Jan 2005]

Z
g.n
F~ = 4y~i + 18z~j − x~k.
The surface is φ = 3x + 2y + 6z − 6.
Let R be the projection of S in the xy
C

S
~n et
B
Y
O
plane.
R is the ∆AOB"
∴" A
X
dxdy
∴ F~ · ~nds = F~ · ~n
S R |~n · ~k|

where ~n is the unit normal to S and ~k is the unit normal to the xy plane. Normal

Downloaded From : www.EasyEngineering.net


Downloaded From : www.EasyEngineering.net

Vector Calculus 249

to the surface is ∇φ.

∂φ ~ ∂φ ~ ∂φ
∴ ∇φ = ~i +j +k = 3~i + 2~j + 6~k.
∂x ∂y ∂z
∇φ 3~i + 2~j + 6~k 1
~n = = √ = (3~i + 2~j + 6~k).
|∇φ| 9 + 4 + 36 7
1 6
F~ · ~n = (12y + 36z − 6x) = (2y + 6z − x)
7 7
6
~n · ~k = .
7

ww ∴
"
~
F · ~nds =
"
6 (2y + 6z − x)
R 7
6
dxdy =
"
(2y + 6z − x)dxdy.

w.E S

We have, 3x + 2y + 6z = 6
7 R

asy
6z = 6 − 3x − 2y
2y + 6z − x = 6 − 3x − x = 6 − 4x
! !
∴ R F~ · ~nds = R (6 − 4x)dxdy.
En
The plane 3x + 2y + 6z = 6 meets the xy plane in line AB.

gin Y

Equation of AB is 3x + 2y = 6 [put z = 0].


ee
The point A is (2, 0) and B is (0, 3). Divide
the region OAB into strips parallel to the
B(0, 3)

rin
3x + 2y = 6

x−axis. PQ is one such strip. Along AB, x


6 − 2y
P Q

g.n X
varies from 0 to and finally as this

et
(0, 0) O A(2, 0)
3
strip traverses the surface OAB, y varies
from 0 to 3.
" Z 3 Z 6−2y
3
∴ F~ · ~nds = (6 − 4x)dxdy
S y=0 x=0
6−2y
3
x2 3
Z
= (6x − 4 ) dy
y=0 2 0
Z 3 n
6 − 2y 6 − 2y 2 o
= 6( ) − 2( ) dy
y=0 3 3

Downloaded From : www.EasyEngineering.net


Downloaded From : www.EasyEngineering.net

250 Engineering Mathematics - II

Z 3 n 2 o
= 2(6 − 2y) − (36 + 4y2 − 24y) dy
y=0 9
3
72 + 8y2 − 48y
Z
= (12 − 4y − )dy
y=0 9
1 3
Z
= (108 − 36y − 72 − 8y2 + 48y)dy
9 0
1 3
Z
= (36 + 12y − 8y2 )dy
9 0
1   3  y2 3  y3 3  1 8

ww = 36 y + 12
9
1
0 2 0
−8
1
3 0
= (36 · 3 + 6 · 9 − · 27)
9
1
= (108 + 54 − 72) = (162 − 72) = · 90 = 10.
3

w.E 9

Example 2.85. Evaluate


"
9 9

{F~ · ~n}ds where F~ = yz~i + zx~j + xy~k and S is the part of

asy
S
the surface of the sphere x2 + y2 + z2 = 1 which lies in the first octant. [Nov 2008]
Z
Solution.

Given F~ = yz~i + zx~j + xy~k. En


Let φ = x2 + y2 + z2 − 1 gin Y

ee
O
∂φ ~ ∂φ ~ ∂φ
∇φ = ~i +j +k = 2x~i + 2y~j + 2z~k.
∂x ∂y ∂z

~n =
∇φ
= p
2x~i + 2y~j + 2z~k X
rin
|∇φ| 4x2 + 4y2 + 4z2
2(x~i + y~j + z~k) g.n
= p
2
2 x +y +z 2 2
= x~i + y~j + z~k[∵ x2 + y2 + z2 = 1]

~ n = xyz + xyz + xyz = 3xyz.


F.~
Let R be the projection of S in the xy plane. Then R is bounded by the lines x =
et
0, y = 0 and the 2 + y2 = 1, z = 0. [Circle in the first quadrant of the xy plane]
" circle x "
Now ~ nds =
F.~ ~ n dxdy
F.~
|~n.~k|
S R
|~n.~k| = z

Downloaded From : www.EasyEngineering.net


Downloaded From : www.EasyEngineering.net

Vector Calculus 251

(0, 1)
" " "
~ nds = dxdy
F.~ 3xyz =3 xydxdy
z

x=0
S R R

Z1 Z1−x2 Z1 ! √1−x2 (1, 0)
X
y2 O y=0
=3 xydxdy = 3 x dx
2 0
0 y=0 0
Z1 Z1

ww
!1
3 x2 x4
!
3 2 3 3 3 1 1 3
= x(1 − x )dx = (x − x )dx = − = − = .
2 2 2 2 4 0 2 2 4 8
0 0

w.E
Example 2.86. Evaluate
"

S
{F~ · ~n}ds where F~ = 18z~i − 12~j + 3y~k and S is that part
of the plane 2x + 3y + 6z = 12 which lies in the first octant. [Dec 2011]
Solution.
asy
Given F~ = 18z~i − 12~j + 3y~k.
En Z

Let φ = 2x + 3y + 6z − 12

∇φ = ~i
∂φ ~ ∂φ ~ ∂φ
+j +k gin C(0, 0, 2)

∂x ∂y
= 2~i + 3~j + 6~k.
∇φ
∂z

2~i + 3~j + 6~k


ee
2~i + 3~j + 6~k A(6, 0, 0)
O

rin
B(0, 4, 0)
Y

g.n
~n = = √ = X
|∇φ| 4 + 9 + 36 7
 12−2x−3y   

et
36z − 36 + 18y 36 6 − 36 + 18y  Substitute z from the 
~n=
F.~ = .  
7 7 equation of the plane


1 1
= [72 − 12x − 18y − 36 + 18y] = [36 − 12x].
7 7

Let R be the projection of S in the xy plane and hence R is the triangle OAB. AB is
the line given by 2x + 3y = 12. Divide R into strips parallel to the x−axis. PQ is one
12−3y
such strip. Along this strip x varies from 0 to 2 and as this strip traverses the
entire R, y varies from 0 to 4.

Downloaded From : www.EasyEngineering.net


Downloaded From : www.EasyEngineering.net

252 Engineering Mathematics - II

" " " B(0, 4)

Now ~ nds =
F.~ ~ n dxdy =
F.~
1 dxdy
(36 − 12x) 6
|~n.~k| 7 7 P Q
S R R
12−3y
Z4 Z 2
(0, 0) O A(6, 0)
X
12
= (3 − x)dxdy
6
0 x=0
Z4 ! 12−3y Z4

ww x 2 2  12 − 3y  1  12 − 3y 2 !
=2 3x − dy = 2 3 − dy
2 x=0 2 2 2
0 0

w.E =2
Z4

0
9 1
8
2
!
18 − y − (144 + 9y − 72y) dy = 2
2
Z4
9 9 2
!
18 − y − 18 − y + 9y) dy
2 8
0

=2
Z4
9
2
9 2
!
y − y dy = 2
8 asy
9  y2 4 9  y3 4

2 2 0 8 3 0
!
9
4
9
!
= 2 16 − 64 = 2[36 − 24] = 24.
24
0

En "
{F~ · ~n}ds where F~ = 4xz~i − y2 ~j + yz~k and S is the
Example 2.87. Evaluate

gin S
surface of the cube bounded by the planes x = 0, x = 1, y = 0, y = 1, z = 0, z = 1.

Solution. F~ = 4xz~i − y2 ~j + yz~k. ee rin


[Dec 2008]

The given surface is the cube


Z

G
g.n D
OABCDEFG. S is piecewise smooth
which consists of six smooth surfaces
namely ABEF, OCDG, BCDE, OAFG, OABC
F

O
E
et C
Y

B
and DEFG. A
X

" " " "


∴ {F~ · ~n}ds = {F~ · ~n}ds + {F~ · ~n}ds + {F~ · ~n}ds
S ABEF OCDG OABC
" " "
+ {F~ · ~n}ds + {F~ · ~n}ds + {F~ · ~n}ds.
DEFG BCDE OAFG

Downloaded From : www.EasyEngineering.net


Downloaded From : www.EasyEngineering.net

Vector Calculus 253

The following table gives the necessary quantities for the computation of the
surface integral on each surface.

Face Equation Outward normal F~ · ~n ds


ABEF x=1 ~i 4z dydz
OCDG x=0 −~i −4xz = 0 dydz
BCDE y=1 ~j −y2 = −1 dxdz
OAFG y=0 −~j −y2 = 0 dxdz

ww OABC
DEFG
z=0
z=1
−~k
~k
−yz = 0
yz = y
dxdy
dxdy

w.E
"

" ABEF
~
F · ~nds =
Z 1Z 1

0
Z 1Z 10
4zdydz = 4
Z 1

0
Z 1
1
z(y)0 dz=4
Z 1

0
Z 1
zdz = 4
z2
!1

2 0
= 2.

"BCDE
F~ · ~nds =
asy
Z0 1 Z0 1
(−1)dxdz = − (x)10 dz=−
Z 1  02 
y 1 1
0
dz = −(z)10 = −1.
1
F~ · ~nds =

En
1
ydxdy = dx= (x)0 = .
DEFG 0 0 x=0 2 0 2 2
All"the other integrals will be zero.

S
F~ · ~nds = 2 − 1 + = .
1 3
2 2 gin
2.6.2 Volume Integral
ee
If V is the volume bounded by a surface S then
$
rin
φ(x, y, z)dv and
$
~ are
Fdv

called volume integrals.


V

g.n V

If we divide V into rectangular blocks by drawing planes parallel to the coordinate


planes,

$ then dv
φdv =
$= dxdydz
φ(x, y, z)dxdydz.
et
V
If F~ = F1~i + F2 ~j + F3~k then
$ $ $ $
~
Fdv = i~ ~
F1 dxdydz + j ~
F2 dxdydz + k F3 dxdydz.
V V V V

Downloaded From : www.EasyEngineering.net


Downloaded From : www.EasyEngineering.net

254 Engineering Mathematics - II

✎ ☞
Worked Examples
✍ ✌
$
Example 2.88. Evaluate ~ if F~ = x2~i + y2 ~j + z2~k and V is the volume of
∇ · Fdv
V
the region enclosed by the cube x = 0, x = 1, y = 0, y = 1, z = 0, z = 1. [Apr 2004]
Solution. F~ = x2~i + y2 ~j + z2~k
∂ 2 ∂ ∂
∇ · F~ = (x ) + (y2 ) + (z2 ) = 2x + 2y + 2z = 2(x + y + z).
∂x ∂y ∂z
$ Z 1 Z 1 Z 1
~ =
∇ · Fdv 2(x + y + z)dxdydz
z=0 y=0 x=0

ww V

=2
Z

2
1Z
x2 1 1h
(
i
)0 + y(x)10 + z(x)10 dydz

w.E
0 0
Z 1Z 1 Z 1
1 1 y2
=2 ( + y + z)dydz = 2 [ (y)10 + ( )10 + z(y)10 ]dz
0 0 2 0 2 2

asy
Z 1 Z 1
1 1  z2 1  1 3
=2 ( + + z)dz = 2 (1 + z)dz = 2 z + =2 1+ = 2 · = 3.
0 2 2 0 2 0 2 2
$
2

En ~
Example 2.89. If F~ = (2x − 3z)~i − 2xy~j − 4xk, evaluate ~
∇ × FdV where V is the

gin
V
region bounded by x = 0, y = 0, z = 0 and 2x + 2y + z = 4.
Solution.

∇ × F~ =


~i

∂x
ee ~j

∂y

∂z

~k

rin
g.n

(2x2 − 3z) −2xy −4x
" # " #
∂ ∂ ∂ ∂

et
=i~ (−4x) − (−2xy) − j ~ 2
(−4x) − (2x − 3z)
∂y ∂z ∂x ∂z
" #
∂ ∂
+ ~k (−2xy) − (2x2 − 3z)
∂x ∂y
= ~i.0 − ~j(−4 + 3) + ~k(−2y + 0) = ~j − 2y~k.
$ " Z 2 Z 2−x Z 4−2x−2y
~
∇ × FdV = ~ ~
( j − 2yk)dxdydz = (~j − 2y~k)dzdydx
V x=0 y=0 z=0
V
Z 2 Z 2−x
4−2x−2y
= (~j − 2y~k)(z)0 dydx
x=0 y=0

Downloaded From : www.EasyEngineering.net


Downloaded From : www.EasyEngineering.net

Vector Calculus 255

Z 2 Z 2−x
= (4 − 2x − 2y)~j − (8y − 4xy − 4y2 )~kdydx
x=0 y=0
Z 2
4
= ((4y − 2xy − y2 )~j − (4y2 − 2xy2 − y3 )~k)2−x
y=0 dx
x=0 3
Z2
4
= (4(2 − x) − 2x(2 − x) − (2 − x)2 )~j − (4(2 − x)2 − 2x(2 − x)2 − (2 − x)3 )~kdx
3
0
Z2
4
= (8 − 4x − 4x + 2x2 − (2 − x)2 )~j − (4(2 − x)2 − 2(4x + x3 − 4x2 ) − (2 − x)3 )~kdx
3

ww
0
2 2 3 2 3 2  (2 − x)3 2
 !   !2
x2
! ! !
 2 x x (2 − x)  ~ ~
= 8(x)0 − 8
 +2 −  j − k 4

  −8
2 0 3 0 −3 0 −3 0 2 0

w.E −2
x4
4 0
!2
+8
x3
!2

3 0 3

4 (2 − x)4 
−4
!2

0



asy
" #
2 1
= 8(2 − 0) − 4(4 − 0) + (8 − 0) + (0 − 8) ~j
3 3

En
" #
~ −4 1 8 1
−k (0 − 8) − 4(4 − 0) − (16 − 0) + (8 − 0) + (0 − 16)
3 2 3 3

gin
" # " #
16 8 ~ ~ 32 64 16 8 8 8h i
= 16 − 16 + − j−k − 16 − 8 + − = ~j − ~k = ~j − ~k .
3 3 3 3 3 3 3 3

2.7 Gauss Divergence theorem ee rin


g.n
[Dec 2012, Jun 2012, Jun 2010]

et
Gauss divergence theorem express a surface integral interms of a volume
integral.

Statement. Let V be the volume bounded by a closed surface S . If a vector


function F~ is continuous and has continuous partial derivatives inside and on
S , then the surface integral of F~ over S is equal to the volume integral of the
divergence of F~ taken throughout V.
! #
~
i.e., S F~ · ~nds = V ∇ · Fdv.

Downloaded From : www.EasyEngineering.net


Downloaded From : www.EasyEngineering.net

256 Engineering Mathematics - II

Gauss Divergence theorem in cartesian coordinates


If F~ = F1~i + F2 ~j + F3~k, then
F~ · ~n = F1 (~i · ~n) + F2 (~j · ~n) + F3 (~k · ~n)
F~ · ~nds = F1 (~i · ~n)ds + F2 (~j · ~n)ds + F3 (~k · ~n)ds = F1 dydz + F2 dxdz + F3 dxdy
∂F1 ∂F2 ∂F3
∇ · F~ = + +
∂x ∂y ∂z
"
∴ Gauss divergence theorem becomes
$ 
∂F1 ∂F2 ∂F3 
(F1 dydz + F2 dzdx + F3 dxdy) = + + dxdydz.
S V ∂x ∂y ∂z

ww "


Worked Examples

w.E
Example 2.90. Evaluate
sphere of radius a.
S
(xdydz + ydzdx + zdxdy) where S is the surface of the

Solution.
" We know that by the
(F1 dydz + F2 dzdx + F3 dxdy) = asy
$cartesian form of Gauss
∂F1 ∂F2 ∂F3
∂x
+
∂y
+
∂z
!
dV.
divergence theorem

S
Now F1 = x, F2 = y, F3 = z
∂F1 ∂F2 ∂F3 En V

∂x
= 1,
∂y
"
=1
∂z
=1
$ gin $
∴ (xdydz + ydzdx + zdxdy) =
S ee
(1 + 1 + 1)dV = 3dV = 3V
V V

= 3 × Volume of the sphere. rin


4
= 3 × πa3 = 4πa3 .
3 g.n
Example 2.91. If S is any closed surface enclosing volume V and F~ = ax~i+by~j+cz~k,
prove that
!
F~ · ~nds = (a + b + c)V.
S
[Jun 2004]
et
Solution. By Gauss Divergence theorem
" $
F~ · ~nds = ~
∇ · Fdv
S V
∂ ∂ ∂
∇ · F~ = (ax) + (by) + (cz)
∂x ∂y ∂z
= a + b + c.

Downloaded From : www.EasyEngineering.net


Downloaded From : www.EasyEngineering.net

Vector Calculus 257

" $
∴ F~ · ~nds = (a + b + c)dv
S V

= (a + b + c)V.
!
Example 2.92. Prove that ~ nds = 0 for any closed surface if F~ = curlF.
F.~ ~
S
or
!
Show that ~ nds = 0 where S is any closed surface.
curlF.~
S
Solution. By Gauss divergence theorem we have
" $ $
~ nds = ~ ~

ww F.~
S
divFdV = div(curlF)dV
V
$ V

~ = 0]

w.E =

= 0.
V
0dV[∵ div(curlF)

asy
Example 2.93. Verify Gauss divergence theorem for F~ = 4xz~i − y2 ~j + yz~k over the

En
cube bounded by x = 0, x = 1, y = 0, y = 1, z = 0, z = 1.
[May 2015, Dec
"2013, Dec 2012,
$ May 2011, Dec 2010]

" gin
Solution. By Gauss divergence theorem,
S
F~ · ~nds =
V
~
∇ · Fdv

From Example 2.87 we have

Now,
$
~ =
∇ · Fdv
S
Z 1Z 1Z 1 ee
F~ · ~nds = .
3
2
∂F1 ∂F2 ∂F3 
+ + dxdydz rin
V

=
0 0
Z 1Z 1Z 1
0 ∂x ∂y

(4z − 2y + y)dxdydz
∂z

g.n
=
0
Z

0
Z 1Z 1
0

0
0 0
1Z 1Z 1
(4z − y)dxdydz =
Z 1 
0 0
Z

2
1Z 1"

!1
z2
2

#1
− yz dxdy
4
0
et
y  
2(y)10 −

= (2 − y)dxdy = dx
0 0 0 2 0
Z 1 !
1 3 3
= 2 − dx = (x)10 = .
0 2 2 2
∴ LHS = RHS.
Hence, Gauss Divergence theorem is verified.

Downloaded From : www.EasyEngineering.net


Downloaded From : www.EasyEngineering.net

258 Engineering Mathematics - II

Example 2.94. Verify Gauss divergence theorem for F~ = (x2 −yz)~i+(y2 −zx)~j+(z2 −xy)~k
taken over the rectangular paralleopiped bounded by 0 ≤ x ≤ a, 0 ≤ y ≤ b, 0 ≤ z ≤ c.
[May 1994]
! #
Solution. By Gauss divergence theorem, S
F~ · ~nds = V
~
∇ · Fdv.
∂ 2 ∂ ∂
∇ · F~ = (x − yz) + (y2 − zx) + (z2 − xy)
∂x ∂y ∂z
= 2x + 2y + 2z = 2(x + y + z).
$

ww
Z aZ bZ c
RHS = ~
∇ · FdV = 2(x + y + z)dzdydx
V 0 0 0
Z aZ b
z2 c

w.E =2

=2
0
Z a
0
xz + yz +

c2
(cx + cy + )dydx
2 0
dydx

asy =2
0
Z a
2
y2
2
c2
[cx(y)b0 + c( )b0 + (y)b0 ]dx
2

En=2
0
Z a
(bcx +
2
b c bc
+
2
)dx

gin 0
x2
= 2[bc( )a0 +
2 2
b2 c a bc2 a
(x)0 + (x)0 ]

ee
2 2 2
a2 bc b2 ac c2 ab
= 2[ + + ] = abc(a + b + c). (1)

"
2 2 2
Z rin
Now we shall evaluate
S
F~ · ~nds. G
g.n D

The rectangular parallelopiped has six


faces,
S 1 → ABEF, S 2 → OCDG, S 3 → BCDE,
F

O
E

B
etC
Y

A
S 4 → OAFG, S 5 → OABC, S 6 → DEFG. X

" " " " "


∴ F~ · ~nds = F~ · ~nds + F~ · ~nds + F~ · ~nds + F~ · ~nds
S S 1" S 2" S3 S4

+ F~ · ~nds + F~ · ~nds.
S5 S6

Downloaded From : www.EasyEngineering.net


Downloaded From : www.EasyEngineering.net

Vector Calculus 259

The following table gives the necessary quantities for the computation of the
surface integral over each face.
Face Equation Outward normal F~ · ~n ds
S1 x=a ~i x2 − yz dydz
a2 − yz
S2 x=0 −~i −(x2 − yz) dydz
yz
S3 y=b ~j (y2 − zx) dxdz

ww S4 y=0 −~j
b2 − zx
−(y2 − zx) dxdz

w.E S5 z=0 −~k −(z2


zx
− xy) dxdy

S6
asy
z=c ~k
xy
(z2 − xy) dxdy

En c2 − xy

"
gin
ee
bZ c b# !c #
z2
Z  Z
F~ · ~nds = a2 − yz dzdy = a2 (z)c0 − y dy
S1 0
Z b
0

yc2
2
!
2 b
0

c2 y2
!b
= a2 bc −
2
b2 c2
0

. rin
g.n
= a c− dy = a c (y)0 −
0 2 2 2 0 4
" Z cZ b Z c 2 b Z c 2 2 z2 c b2 c2
! !
y b b

"
S2

S3
F~ · ~nds =

F~ · ~nds =
0 0
Z aZ c

0 0
yzdydz =

2

0

b − zx dzdx =
z
2 0
Z a#

0
dz =

2 c
0

b (z)0 − x
z dz =
2
z

!a
2 c
! #

2 0
2 2 0

dx
=
4
.

et
Z an
c2 o c2 x2 a2 c2
!
2 2 a
= b c − x dx = b c (x)0 − = ab2 c − .
0 2 2 2 0 4
" Z aZ c Z a !c !a
z2 c2 x2 a2 c2
F~ · ~nds = zxdzdx = x dx = = .
S4 0 0 0 2 0 2 2 0 4
" Z aZ b Z a !b
~ y2 a2 b2
F · ~nds = xydydx = x dx = .
S5 0 0 0 2 0 4

Downloaded From : www.EasyEngineering.net


Downloaded From : www.EasyEngineering.net

260 Engineering Mathematics - II

" aZ b

a !b 
y2 
Z  Z
F~ · ~nds = 2  2 b
c − xy dydx = c (y)0 − x dx
S6 0 0 0 2 0
a !a
b2 x b2 x2 a2 b2
Z !
= bc2 − dx = bc2 (x)a0 − = abc2 − .
0 2 2 2 0 4
"
b2 c2 b2 c2 a2 c2 a2 c2 a2 b2 a2 b2
{F~ · ~n}ds = a2 bc − + + ab2 c − + + + abc2 −
S 4 4 4 4 4 4
= abc (a + b + c) . (2)
" $

ww From (1) and (2)


S
{F~ · ~n}ds =

Hence, Gauss divergence theorem is verified.


V
~
∇.FdV.

w.E
Example 2.95. Verify divergence theorem for F~ = x2~i+z~j+yz~k over the cube formed
by the planes x = ±1, y = ±1, z = ±1.

asy
Solution. By Gauss divergence theorem we have
!
S
[Jun 2013, May 2006]
#
~
F~ · ~nds = V divFdv.

F~ = x2~i + z~j + yz~k


En
∂ 2
∂x

divF~ =

(x ) + (z) + (yz) = 2x + y
∂y ∂z gin
$

V
~
divFdV =
Z1 Z1 Z1
(2x + y)dzdydx
−1 −1 −1
Z1 Z1
ee Z1 Z1 rin
= (2x + y)(z)1−1 dydx =2 (2x + y)dydx
g.n
et
−1 −1 −1 −1
Z1  y2 1 ! Z1  x2 1
=2 2x(y)1−1 + dx = 2 4xdx = 8 = 8 × 0 = 0. (1)
2 −1 2 −1
−1 −1
! G F

We shall evaluate S
F~ · ~nds. Z

The given cube has 6 faces H E


Y
D
S 1 = BCEH, S 2 = ADFG, S 3 = CDEF, A
X

S 4 = ABGH, S 5 = EFGH, S 6 = ABCD.


B C

Downloaded From : www.EasyEngineering.net


Downloaded From : www.EasyEngineering.net

Vector Calculus 261

" 6 "
X
Now F~ · ~nds = F~ · ~nds
S i=1 S
i
The following table gives the necessary quantities for the computation of the
surface integral over each face.
Face Equation Outward normal F~ · ~n ds
S1 x=1 ~i x2 = 1 dydz
S2 x = −1 −~i −x2 = −1 dydz
S3 y=1 ~j z dxdz

ww S4
S5
y = −1
z=1
−~j
~k
−z
yz = y
dxdz
dxdy

w.E S6 z = −1

"
−~k −yz = y dxdy

asy
F~ · ~nds =
Z1 Z1
dydz =
Z1
(y)1−1 dz =2
Z1
dz = 2(z)1−1 = 4.

En
S1 −1 −1 −1 −1

"

S2gin F~ · ~nds = −4

"

S3
ee
F~ · ~nds =
Z1 Z1

−1 −1
zdxdz = 0

rin
"
F~ · ~nds =
Z1 Z1
(−z)dxdz = 0 g.n
"
S4

F~ · ~nds =
−1 −1
Z1 Z1
ydxdy = 0
et
S5 −1 −1
" Z1 Z1
F~ · ~nds = (−y)dxdy = 0.
S −1 −1
"6

∴ F~ · ~nds = 4 − 4 + 0 + 0 = 0. (2)
S

Downloaded From : www.EasyEngineering.net


Downloaded From : www.EasyEngineering.net

262 Engineering Mathematics - II

From (1) and (2) we get " $


F~ · ~nds = ~
divFdV.
S V

Hence, Gauss Divergence theorem is verified.

Example 2.96. Verify Gauss divergence theorem for the vector function f~ = (x3 −
yz)~i − 2x2 y~j + 2~k over the cube bounded by x = 0, y = 0, z = 0 and x = a, y = a, z = a.
[Dec 2011, Jun 2010]

ww Solution. By Gauss divergence theorem


"
~
$
∇ · f~dv.

w.E f · ~nds =
S

∇ · f~ =
∂ 3
V
∂ ∂
(x − yz) + (−2x2 y) + (2)

asy ∂x ∂y
= 3x2 − 2x2 + 0 = x2 .
∂z

$ En
gin
Za Za Za Za Za
Now, ∇ · f~dv = 2
x dzdydx = x2 [z]a0 dydx

ee
V x=0 y=0 z=0 x=0 y=0
Za Za Za

=
x=0 y=0
ax2 dydx = a
x=0
x2 [y]a0 dx
rin
=a
Za
ax2 dx = a2
" 3 a
x
3
#
=
a5
3
. g.n (1)

Now we shall evaluate


"
f~ · ~nds.
0
0

G D
et
s
The cube has six faces namely S 1 : F E
C
ABEF, S 2 : OCDG, S 3 : BCDE, S 4 : O
Y

OAFG, S 5 : OABC, S 6 : DEFG. A B

Downloaded From : www.EasyEngineering.net


Downloaded From : www.EasyEngineering.net

Vector Calculus 263

" " " " "


∴ f~ · ~nds = f~ · ~nds + f~ · ~nds + f~ · ~nds + f~ · ~nds
S S1 S2 " S3 " S4

+ f~ · ~nds + f~ · ~nds.
S5 S6
The following table gives the necessary quantities for the computation of the
surface integral over each face.
Face Equation Outward normal(~n) F~ · ~n ds
S1 x=a ~i x3 − yz = a3 − yz dydz
−~i

ww S2
S3
x=0
y=a ~j
−(x3 − y2 )
−2x2 y = −2ax2
= yz dydz
dxdz

w.E S4
S5
y=0
z=0
−~j
−~k
2x2 y = 0
−2
dxdz
dxdy

asy
S6 z=a ~k 2 dxdy

" Za Za Za " #a
f~ · ~nds = 3

En
(a − yz)dzdy = 3
a z−y
z2
2 0
dy

gin
S1 y=0 z=0 y=0
Za #a
a2 a2 y2 a4
! "
4 4
= a − y dy = a y − · = a5 − .

"
f~ · ~nds =
0
Za Za
2

yzdzdy =
ee2 2 0
Za " 2 #a
y
z
2 0
dy
4

rin
S2

1
y=0 z=0
Za
2
" #a
a2 y2
y=0

a2 a2 a4 g.n
et
= y · a dy = = · = .
2 2 2 0 2 2 4
0
" Za Za Za
f~ · ~nds = 2
−2ax dzdx = −2a x2 · [z]a0 dx
S3 x=0 z=0 x=0
Za #a
x3 2a5
"
2 2
= −2a x · adx = −2a =− .
3 0 3
0
"
f~ · ~nds = 0.
S4

Downloaded From : www.EasyEngineering.net


Downloaded From : www.EasyEngineering.net

264 Engineering Mathematics - II

" Za Za Za Za
f~ · ~nds = −2dxdy = −2 dx dy = −2 · [x]a0 [y]a0 = −2 · a · a = −2a2 .
S5 x=0 y=0 x=0 y=0
" Za Za
f~ · ~nds = 2dxdy = 2a2 .
S6 x=0 y=0
"
a4 a4 2a5 2a5 a5
∴ f~ · ~nds = a5 − + − + 0 − 2a2 + 2a2 = a5 − = . (2)
4 4 3 3 3
S

ww
From (1) and (2) we have
" $
f~ · ~nds = ∇ · f~dv

w.E
∴ Gauss divergence theorem is verified.
s V

asy
Example 2.97. Evaluate F~ · d~s, where F~ = 4x~i − 2y2 ~j + z2~k and S is the surface
R
S
bounding the region x2 + y2 = 4, z = 0 and z = 3.

En Z

Solution.
By
Z Gauss divergence theorem, gin z=3

ee
Z
{F~ · d~s} = ~
∇ · FdV
S V
Now, F~ = 4x~i − 2y2 ~j + z2~k O

rin
x 2 + y2 = 4
Y

∇· F~ = ∂ ∂ 2 ∂ 2
∂x (4x)+ ∂y (−2y )+ ∂z (z ) = 4−4y+2z.
X
g.n
Z

V
∇ · F~ dV =
$

V
(4 − 4y + 2z) dxdydz


et
$ Z2 Z4−x2 Z3
= (4 − 4y + 2z) dzdydx = (4 − 4y + 2z) dzdydx

V x=−2 y=− 4−x2 z=0

Z2 Z4−x2
= (4z − 4yz + z2 )3z=0 dydx

x=−2 y=− 4−x2

Downloaded From : www.EasyEngineering.net


Downloaded From : www.EasyEngineering.net

Vector Calculus 265

√ √
Z2 Z4−x2 Z2 Z4−x2
= (12 − 12y + 9) dydx = (21 − 12y) dydx
√ √
x=−2 y=− 4−x2 x=−2 y=− 4−x2

Z2   √4−x2
2
= 21y − 6y √ dx
y=− 4−x2
x=−2
Z2  p p   
= 21 4 − x2 + 4 − x2 − 6 (4 − x2 ) − (4 − x2 ) dx

ww
x=−2
Z2  p  Z2  p 
= 42 4 − x 2 dx = 42 × 2 4−x 2 dx

w.E = 84
x=−2

xp
2
2
4 −1  x 
4 − x + sin
2
!2

2 x=0

x=0
 π
= 84 2 sin−1 1 − 0 = 164 × = 84π.
2

asy
Example 2.98. Using divergence theorem, evaluate
Z 
yz~i + zx~j + xy~k

d · S~ ,

En
where S is the surface of the sphere x2 + y2 + z2 = a2 in the first octant.
S

gin
Solution. Let V be the volume bounded by the surface of the sphere. This region
is bounded by 4 surfaces namely

S 1 : The circular quadrant OBC in the yz


ee rin
plane.
Z

g.n
S 2 : The circular quadrant OAC in the xz
plane.
S 3 : The circular quadrant OAB in the xy S2
C

S1
S4

→ −~j
etB
Y
plane. O
S3

S : The surface ABC of the sphere in the


A
first octant. X

Here F~ = yz~i + zx~j + xy~k.

Downloaded From : www.EasyEngineering.net


Downloaded From : www.EasyEngineering.net

266 Engineering Mathematics - II

∴ ∇ · F~ = ∂
∂x (yz) + ∂
∂y (zx) + ∂
∂z (xy) = 0.
By Gauss divergence theorem,
~ dV = F~ · dS~
R R
∇ · F
RV RS
∇ · F dV = F~ · dS~ + F~ · dS~ + F~ · dS~ + F~ · dS~
~
R R R
V S1 S2 S3 S
~ ~ ~ ~ ~ ~ ~ ~
R R R R
0 = F · dS + F · dS + F · dS + F · dS (1)
S1 S2 S3 S
On S 1 , x = 0,

Z "

ww ∴
S1
F~ · dS~ =
S1
"
(yz~i) (dydz(− j))

w.E =−
S1
yz dydz

√2 2 √
=−
asy Za Za −y
yz dzdy = −
Za
y
z 2
! a2 −y2

2 z=0
dy

1
Za 
En
y=0 z=0

 1
Za  
y=0

gin
2 2
=− y(a − y ) dy = − a2 y − y3 dy
2 2
y=0 y=0
!a

ee
a2 y2 y4 a4 a41 a4 a4
!
1 1
=− − =− − =− × =− .
2 2 4 2 2 4 2 4 8

rin
y=0

In the same way we get


R a4 a4
F~ · dS~ = − and F~ · dS~ = − .
8
R
8 g.n
S2 S3

∴ (1)
a4 a4 a4
⇒0=− − − +
Z
F~ · dS~
et
8 8 8
S
3a4
Z
0=− + F~ · dS~
8
S
3a4
Z
∴ F~ · dS~ = .
8
S

Downloaded From : www.EasyEngineering.net


Downloaded From : www.EasyEngineering.net

Vector Calculus 267

2.8 Stoke’s theorem

Stoke’s theorem is the relation between line and surface integrals.


[Dec 2014, Dec 2010]
Statement. If S is an open surface bounded by a simple closed curve C and
if F~ is continuous having continuous partial derivatives in S and on C, then
!
F~ · d~r = CurlF~ · ~nds where C is traversed in the positive direction.
R
C S

ww Another Form. The line integral of the tangential component of a vector


function F~ around a simple closed curve C is equal to the surface integral of the

w.E
normal component of curlF~ over any surface S having C as its boundary.
Note. Green’s theorem in the plane is a particular case of Stoke’s theorem. If S
is the region R in the xy plane bounded by the simple closed curve C then ~n = ~k

asy
is the outward unit normal.
!

En F~ · d~r = CurlF~ · ~kdR.


R
∴ Stoke’s theorem in the plane is C R
Cartesian form of Stoke’s theorem

gin
If F~ = F1~i + F2 ~j + F3~k, then curlF~ = ~i
 ∂F3 ∂F2 
∂y

∂z
− ~j
 ∂F3 ∂F1 
∂x

∂z
+ ~k
 ∂F2 ∂F1 
∂x

∂y
.
F~ · d~r = F1 dx + F2 dy + F3 dz.

Z The cartesian form of Stoke’s



(F1 dx + F2 dy + F3 dz) =
ee
"  theorem is
∂F3 ∂F2
− dydz +
" 
∂F1 ∂F3 
− dzdx rin
C S "

+
∂y ∂z
 ∂F2 ∂F1 
S ∂x

∂y
S

dxdy.
∂z ∂x

g.n
Note: If F~ = P~i + Q~j,~r = x~i + y~j, d~r = dx~i + dy~j.

~i ~j ~k
 ∂Q ∂P 
et
~k ⇒ curlF~ · ~k = ∂Q − ∂P .

curlF~ = ∂x ∂ ∂
∂y
∂ =
∂z

∂x ∂y ∂x ∂y


P Q 0

" n
∂Q ∂P o
Z
∴ Stoke’s theorem in the plane is (Pdx + Qdy) = − dxdy
C R ∂x ∂y
which is the Green’s theorem.

Downloaded From : www.EasyEngineering.net


Downloaded From : www.EasyEngineering.net

268 Engineering Mathematics - II

✎ ☞
Worked Examples
✍ ✌
Z
Example 2.99. Evaluate (e x dx + 2ydy − dz) where C is the curve x2 + y2 = 4, z = 2
C
using Stokes theorem. [Dec 2005]
~ r = e x dx + 2ydy − dz.
Solution. We have F.d~

∴ F~ = e x~i + 2y~j − ~k.


" Z
By Stoke’s theorem, ~ nds =
curlF.~ ~ r.
F.d~

ww
S C

~i ~j ~k

w.E

Now curlF~ = ∂x ∂ ∂
∂y

∂z

e x 2y −1

asy = ~i(0 − 0) − ~j(0 − 0) + ~k(0 − 0)

= ~0


Z
En
~ r = 0.
F.d~
C
Zgin
Example 2.100. Prove that
Solution. Let ~r = x~i + y~j + z~k
Hence d~r = dx~i + dy~j + dz~k
C
ee
{~r · d~r} = 0 where C is the simple closed curve.

rin
Now ~r · d~r = xdx + ydy + zdz
g.n
et

~i ~j ~k

curl~r = ∂x ∂ ∂ ∂ =~ 0
∂y ∂z
x y z
Z "
By Stoke’s theorem {~r · d~r} = {curl~r · ~n}ds = 0.
C R

Example 2.101. Using Stoke’s theorem prove that curl(gradφ) = ~0.

Z " [Jun 2014, Dec 2008]


Solution. By Stoke’s theorem {F~ · d~r} = {curlF~ · ~n}ds, where S is an open
C S

Downloaded From : www.EasyEngineering.net


Downloaded From : www.EasyEngineering.net

Vector Calculus 269

surface bounded by a simple closed curve C.


Let F~ = ∇φ.
∴ By Stoke’s theorem
"
∂φ ∂φ ∂φ 
Z Z 
(∇ × ∇φ).~nds = ∇φ.d~r = dx + dy + dz
∂x ∂y ∂z
S C C
Z
= dφ = [φ]C
C

ww
= 0[∵ C is simple closed, both ends are same]
"
(∇ × ∇φ).~nds = 0.

w.E S

This is true for any surface S , we obtain ∇ × ∇φ = ~0.


=⇒ curl(gradφ) = ~0.
asy Z
Example 2.102. Evaluate
square in the xy plane with verticesEn C
{(xydx + xy2 dy)} by stoke’s theorem where C is the
(1, 0), (−1,
" 0), (0, 1), (0, −1). [Jun 2002]

gin
Z
Solution. By Stoke’s theorem {F~ · d~r} = {curlF~ · ~n}ds.
Z C Z S

C
∴ F~ = xy~i + xy2 ~j

ee
Given, {(xydx + xy2 dy)} = {F~ · d~r}.
C


rin
~i

curlF~ = ∂x ∂
~j

∂y
~k

∂ = ~i(0 − 0) − ~j(0 − 0) + ~
∂z
k(y2 − x) g.n
et

xy xy2 0

curlF~ = (y2 − x)~k.

Since C is a square in the xy plane, we have ~n = ~k and ds = dxdy


curlF~ · ~n = y2 − x. "
"
{curlF~ · ~n}ds = {(y2 − x)}dxdy, where R is the region inside the square.
S R

Downloaded From : www.EasyEngineering.net


Downloaded From : www.EasyEngineering.net

270 Engineering Mathematics - II

Y
x y
Equation to AB is + = 1 ⇒ x + y = 1. B (0, 1)
1 1
x y
Equation to BC is + =1
−1 1
⇒ −x + y = 1 ⇒ x − y = −1. C
X
x y (−1, 0)
O
A(1, 0)
Equation to CD is + =1
−1 −1
⇒ −x − y = 1 ⇒ x + y = −1.
x y D
Equation to DA is + = 1 ⇒ x − y = 1. (0, −1)
1 −1

ww "

R
curlF~ · ~nds =
"

ABD
+
"

BCD

w.E
Z 1 Z y=1−x Z 0 Z x+1
2
= (y − x)dydx + (y2 − x)dxdy
x=0 y=x−1 −1 y=−1−x
Z 1 h 3  Z 0 h 3  x+1
y (1−x)  1−x i y   x+1 i
=

1
0
Z 1 asy
3 x−1
−x y dx +
x−1 −1
Z 1
3 −1−x
−x y
−1−x
dx

0
=

1 0
Z
3
En
[(1 − x)3 − (x − 1)3 ]dx −
0
x(1 − x − x + 1)dx
Z 0

gin
3 3
+ [(x + 1) + (1 + x) ]dx − x(1 + x + x + 1)dx
3 −1 −1
#1 " #1 #0 #0
1 (1 − x)4 (x − 1)4 2x3 2 (x + 1)4 2x3
" " "

ee
2 2
= − − x − + − x +
3 −4 4 0 3 0 3 4 −1 3 −1

=
2
− +
1 2
12 3 12 3 12 3
+ =
1 4 1
= .
rin
Example 2.103. Evaluate
Z
(x + y)dx + (2x − z)dy + (y + z)dz where C is the
g.n
et
C
boundary of the triangle with vertices (2, 0, 0), (0, 3, 0), (0, 0, 6) using Stokes
theorem. Z "
Solution. By Stoke’s theorem {F~ · d~r} = {curlF~ · ~n}ds.
C S
Now, F~ = ~ ~ ~
(x + y)i + (2x − z) j + (y + z)k
~i ~j ~k

curlF~ = ∂x ∂ ∂ ∂ = ~i(1 + 1) − ~j(0 − 0) + ~
k(2 − 1) = 2~i + ~k.
∂y ∂z
x + y 2x − z y + z

Downloaded From : www.EasyEngineering.net


Downloaded From : www.EasyEngineering.net

Vector Calculus 271

Now, S is the surface of the triangle ABC


C(0, 0, 6)
bounded by the curve C.
Equation of the plane ABC is
x y z
+ + = 1. B(0, 3, 0)
Y
2 3 6
x y z O
∴φ= + + =1
2 3 6
~ ∂φ ~ ∂φ ~ ∂φ 1~ 1 ~ 1 ~
∇φ = i +j +k = i+ j+ k A(2, 0, 0)

ww |∇φ| =
∂x
r
1 1
∂y
+ +
4 9 36
1
∂z r2
=
3
9+4+1
36
6 √
=
14
6
X

w.E
~n =
∇φ
|∇φ|
= 2
1~
i + 13 ~j + 61 ~k 1 (3~i + 2~j + ~k) 3~i + 2~j + ~k

14
=
6

14
= √
14

asy
6 6
F~ = (x + y)~i + (2x − z)~j + (y + z)~k.
~ n = √6 + √1 = √7 .
curlF.~

Now,
" 14 14 "14
{curlF~ · ~n}ds = En 7
√ ds = √
7
"
{
dxdy
}
Where
"
S
R is the orthogonal
7
S

gin14
" projection "
dxdy
14 n · ~k|
R |~
of S on the xy plane
1
S
{curlF~ · ~n}ds = √

Example 2.104. Evaluate


14
ee
√ =7
R 1/ 14
"
R
dxdy= 7 × Area of ∆OAB= 7 × × 2 × 3 = 21.
2

rin
{curlF~ · ~nds} where F~ = (y − z)~i + yz~j − xz~k and S is the

g.n
S
open surface bounded by the planes x = 0, x = 1, y = 0, y = 1, z = 0, z = 1 above the xy
plane.
Solution.

" Z Y
et
By Stoke’s theorem {curlF~ · ~n}ds = {F~ · d~r}, y=1 B
S C C
where S is the open surface bounded by
x=0

x=1

C and C is the square in the xy plane,


z = 0 bounded by x = 0, y = 0, x = 1, y = 1. 0 y=0 A
X

Downloaded From : www.EasyEngineering.net


Downloaded From : www.EasyEngineering.net

272 Engineering Mathematics - II

" Z Z Z Z
Now, curlF~ · ~nds = F~ · d~r + F~ · d~r + F~ · d~r + F~ · d~r.
S
OA AB BC CO
F~ · d~r = (y − z)dx + yzdy − xzdz.
Since z = 0, dz = 0.
∴ F~ · d~r = ydx.
OnZ OA: y = 0, dy = 0, x varies from 0 to 1 and F~ · d~r = 0.
∴ F~ · d~r = 0.
OA
OnZ AB: x = 1, dx = 0, y varies from 0 to 1 and F~ · d~r = 0.

ww ∴ F~ · d~r = 0.
AB
On BC: y = 1, dy = 0, x varies from 1 to 0 and F~ · d~r = dx.

w.E

Z
F~ · d~r =
BC
Z 0
dx = [x]0 = 0 − 1 = −1.
1
1

OnZ CO: x = 0, dx = 0, y varies from 1 to 0 and F~ · d~r = 0.



CO
F~ · d~r = 0.
asy
En
Z
Now, F~ · d~r = −1
c
"

gin∴
s
~ nds = −1.
curlF.~

ee
Example 2.105. Verify Stoke’s theorem for F~ = (y − z + 2)~i + (yz + 4)~j − xz~k where S
is the surface of the cube x = 0, x = 2, y = 0, y = 2, z = 0, z = 2 above the xy plane.
rin [Jun 2005]
Solution.ZBy Stoke’s " theorem,
g.n
{F~ · d~r} =
C
{CurlF~ · ~n}ds.
S

~i

~j

~k

et
curlF~ = ∂x ∂ ∂
∂y

∂z

y − z + 2 yz + 4 −xz

= ~i(0 − y) − ~j(−z + 1) + ~k(0 − 1)

= −y~i + (z − 1)~j − ~k.

Downloaded From : www.EasyEngineering.net


Downloaded From : www.EasyEngineering.net

Vector Calculus 273

! Z

We shall evaluate ~ nds where S


curlF.~
S D
G
is the open surface consisting of 5 faces
except the face OABC.
F E

The following table gives the necessary C


Y
O
quantities for the computation of the
A B
surface integral.
X

ww Face Equation Normal~n curlF~ · ~n ds

w.E S 1 = ABEF
S 2 = OCDG
x=2
x=0
~i
−~i
−y
y
dydz
dydz
S 3 = BCDE
S 4 = OAFG asy y=2
y=0
~j
−~ j
z−1
1−z
dxdz
dxdz
S 5 = DEFG
En z=2 ~k −1 dxdy

"
curlF~ · ~nds =
Z
gin 2Z 2
−ydydz = −
Z 2  2 2
y
dz = −2(z)20 = −4.
"

"
S1

S2
curlF~ · ~nds =
Z
0

0
ee 0
2Z 2

0
ydydz =
Z

0
0
2  2 2
y
2
2

0
0

dz = 2(z)20 = 4.
rin
g.n
Z 2Z 2 Z 2 2 Z 2
z 2
curlF~ · ~nds = (z − 1)dzdx = − z dx = 0dx = 0.
S3 0 0 0 2 0 0
"

et
Z 2Z 2
curlF~ · ~nds = (1 − z)dxdz = 0.
S4 0 0
" Z 2Z 2 Z 2 2
curlF~ · ~nds = (−1)dydx = − y dx = −2(x)20 = −4.
0
"S 5 0 0 0

curlF~ · ~nds = −4. (1)


S

We shall now compute the line integral over the simple closed curve C where C is
the boundary of the surface consisting of the edges OA,AB,BC & CO in the z = 0
plane.

Downloaded From : www.EasyEngineering.net


Downloaded From : www.EasyEngineering.net

274 Engineering Mathematics - II

F~ · d~r = F~ · d~r + F~ · d~r + F~ · d~r + F~ · d~r


R R R R R
∴ C OA AB BC CO
F~ · d~r = (y − z + 2)dx + (yz + 4)dy − xzdz = (y + 2)dx + 4dy [∵ z = 0]
On OA: y = 0, dy = 0, x varies from 0 to 2
Z Z 2
∴ ~
{F · d~r} = 2dx = 2(x)20 = 4.
OA 0
On AB: x = 2, dx = 0, y varies from 0 to 2.
Z Z 2
∴ ~
{F · d~r} = 4dy = 4(y)20 = 8.
AB 0
On BC: y = 2, dy = 0, x varies from 2 to 0.

ww
Z Z 0
∴ ~
{F · d~r} = 4dx = −8.
BC 2
On CO: x = 0, dx = 0, y varies from 2 to 0.

w.E
Z Z 0
∴ ~
{F · d~r} = 4dy = 4(y)02 = −8.
CO 2
Now, C {F~ · d~r} = 4 + 8 − 8 − 8 = −4.
R
(2)
From (1) and (2) we obtain
"

asy Z
{curlF~ · ~n}ds = {F~ · d~r}
S C
∴ Stoke’s theorem is verified.

En
gin
Example 2.106. Verify Stoke’s theorem for F~ = (x2 − y2 )~i + 2xy~j in the rectangular
region in the xy plane bounded by the lines x = 0, x = a, y = 0, y = b. [Dec 2005]
Solution. We have F ~
By stoke’s theorem, {F~ · d~r} =

~i
C
~j
2
Z = (x − y )i"


~k
ee
2~
+ 2xy~j

S
{curlF~ · ~n}ds
rin

Now, CurlF~ = ∂x ∂ ∂
∂y ∂z

∂ = ~i(0 − 0) − ~j(0 − 0) + ~
k(2y + 2y) = 4y~k.
g.n
et

x2 − y2 2xy 0

Since the surface is a rectangle in the xy


(0, b) C y = b B(a, b)
plane, the normal is ~n = ~k.
x=0

x=a

curlF~ · ~n = 4y~k · ~k = 4y.


x
(0, 0) O y = 0 A(a, 0)

Downloaded From : www.EasyEngineering.net


Downloaded From : www.EasyEngineering.net

Vector Calculus 275

" "
curlF~ · ~nds = 4ydxdy
S R
Z a Z b Z a  y2 b Z a
= 4ydydx = 4 dx = 2 b2 dx = 2b2 [x]a0 = 2ab2 . (1)
x=0 y=0 0 2 0 0

Z
We shall now evaluate {F~ · d~r}.
Z Z C Z Z Z
~
{F · d~r} = ~
{F · d~r} + ~
{F · d~r} + {F~ · d~r} + {F~ · d~r}

ww
C OA AB BC CO
Now, F~ · d~r = (x − y2 )dx + 2xydy
2

Along OA: y = 0, dy = 0, x varies from 0 to a.

w.E ∴
Z

OA
~
{F · d~r} =
Z a

0
x2 dx =
 x3 a a3
3 0
= .
3
Along AB: x = a, dx = 0, y varies from 0 to b.


Z

AB
{F~ · d~r} =
Z b

0
2aydy = 2a asy
 y2 b
2 0
= ab2 .

Z
~
Z 0
En
Along BC: y = b, dy = 0, x varies from a to 0.
 x3 0  0 a3 a3

BC
{F · d~r} =
a
x2 − b2 dx =

gin
3 a
Along CO: x = 0, dx = 0, y varies from b to 0.
− b2 x = − + ab2 = ab2 − .
a 3 3

ee
Z Z 0
{F~ · d~r} = 0dx = 0.

rin
CO b

a3 a3
Z

C
{F~ · d~r} =
3
+ ab2 + ab2 −

"
3
+ 0 = 2ab2 .

g.n (2)

et
Z
From (1) and (2) we obtain {F~ · d~r} = {curlF~ · ~n}ds.
C S
Hence, Stoke’s theorem is verified.

Example 2.107. Verify Stoke’s theorem for F~ = y2 z~i+z2 x~j+ x2 y~k where S is the open
surface of the cube formed by the planes x = −a, x = a, y = −a, y = a, z = −a, z = a in
which z = −a is cut open.
Solution. By Stoke’s theorem,
!
{F~ · d~r} = S {curlF~ · ~n}ds
R
C

Downloaded From : www.EasyEngineering.net


Downloaded From : www.EasyEngineering.net

276 Engineering Mathematics - II

Now, F~ = y2 z~i + z2 x~j + x2 y~k


~i

~j ~k

CurlF~ = ∂x ∂ = ~i(x2 − 2zx) − ~j(2xy − y2 ) + ~
k(z2 − 2yz) = F~1
∂ ∂
∂y ∂z
y2 z z2 x x2 y

"
Now we shall complete {curlF~ · ~n}ds Z
S
where S is the open surface consisting H

ww of
"5 faces of the cube
~
{curlF · ~n}ds =
"
" except ABCD.
~
{F1 · ~n}ds +
"
"
{F~1 · ~n}ds
G
F
E

w.E
S s1 s2
D
{F~1 · ~n}ds + {F~1 · ~n}ds
> Y
+ A

>
" s3

>
s4
>
{F~1 · ~n}ds

asy
B C
+
s5 X

En
The following table gives the necessary quantities for the computation of the
surface integral.

Face gin
Equation normal ~n curlF~ · ~n ds
S 1 :BCFG
S 2 :ADEH
S 3 :EFGH
ee
x=a
x = −a −~i
~i

~k
a2 − 2az
−(a2 + 2az)
a2 − 2ax
dydz
dydz
rin
g.n
z=a dxdy
S 4 :ABGH y = −a −~j −(a2 + 2ax) dzdx

Now ,
"
S 5 :CDEF

{F~1 · ~n}ds =
y=a

Z a Z a
2
~j

{a − 2az}dydz =
a2 − 2ay

Z a
dzdx
et
[a2 (y)a−a − 2az(y)a−a ] dz
s1 −a −a −a
Z a Z a
2
= {a · 2a − 2a · 2az}dz = (2a3 − 4a2 z)dz
−a −a
 z2 a
= 2a3 (z)a−a − 4a2 · = 2a3 · 2a − 2a2 · 0 = 4a4 .
" 2 −a

Similarly CurlF~ · ~nds = −4a4 .


S2

Downloaded From : www.EasyEngineering.net


Downloaded From : www.EasyEngineering.net

Vector Calculus 277

"
CurlF~ · ~nds = 4a4 .
S3
"
CurlF~ · ~nds = −4a4 .
"S 4
CurlF~ · ~nds = 4a4 .
S5
"
CurlF~ · ~nds = 4a4 . (1)
S

A (−a, −a) D(−a, a)

ww
Z
We shall now compute F~ · d~r over the
C
simple closed curve C, consisting of the

w.E
edges AB, BC, CD, DA. B (a, −a) C(a, a)

Here z = −a, dz = 0.
asy
∴ F~ · d~r = −ay2 dx + a2 xdy.
OnZ AB: y = −a, Z dy
a
En
= 0, x varies from −a to a.
∴ F~ · d~r =
AB
OnZ BC: x = a, Z
−a
gin
−a3 dx = −a3 (x)a−a = −2a4 .
dx = 0, y varies from −a to a.

ee
a
∴ F~ · d~r = a3 dy = 2a4 .
BC −a
OnZ CD: y = a, Z
F~ · d~r =
dy = 0, x varies from a to −a.
−a
−a3 dx = −a3 (x)−a 4
a = 2a . rin
g.n

CD a
OnZ DA: x = −a, Z dx = 0, y varies from a to −a.
−a
∴ F~ · d~r =
DA
−a3 dy = 2a4 .
a

Hence,
Z

c
F~ · d~r = 4a4 .
et (2)
Z "
From (1) and (2) we obtain {F~ · d~r} = {curlF~ · ~n}ds
C S
Hence, Stoke’s theorem is verified.

Example 2.108. Verify Stoke’s theorem for F~ = (x2 + y2 )~i − 2xy~j taken around the
rectangle bounded by the lines x = ±a, y = 0, y = b. [Dec 2013, May 2001]

Downloaded From : www.EasyEngineering.net


Downloaded From : www.EasyEngineering.net

278 Engineering Mathematics - II

Z "
Solution. By Stoke’s theorem we have {F~ · d~r} = {curlF~ · ~n}ds.
C S

F~ = (x2 + y2 )~i − 2xy~j



~i ~j ~k

curlF~ = ∂x ∂ ∂ ∂ = ~i(0 − 0) − ~j(0 − 0) + ~
k(−2y − 2y) = −4y~k
∂y ∂z
x2 + y2 −2xy 0

ww The surface S is given as a rectangle in


the xy plane. C(−a, b)
Y

y=b B(a, b)

w.E
Hence, ~n = ~k.
" "
x = −a

x=a
asy
−4y~k.~kdxdy
~
{curlF · ~n}ds =
S S
Zb Za #b
y2
"

En
X
= −4 ydxdy = −4 [x]a−a D(−a, 0) 0 y=0 A(a, 0)
2 0
0 −a

gin
= −2[b2 ][a + a] = −4ab2 . (1)

ee
Z
Now, we shall evaluate the line integral {F~ · d~r}
F~ · d~r = [(x2Z+ y2 )~i − 2xy
Z
~ ~
Z j].(dxi + dy
~
Z j) = (x +Z
2
C
y2 )dx − 2xydy.
rin
C
F~ · d~r =
AB
F~ · d~r +
BC
F~ · d~r +
CD
F~ · d~r +
DA
F~ · d~r.
g.n
On AB: x = a, dx = 0, y varies from 0 to b and F~ · d~r = −2aydy.


Z

AB
F~ · d~r =
Z b

0
−2aydy = −2a
y
2
2 b
!

0
= −ab2 .

On BC: y = b, dy = 0, x varies from a to −a and F~ · d~r = (x2 + b2 )dx.


et
Z −a 3 −a 2a3
Z !
x 1
∴ F~ · d~r = 2 2
(x + b )dx = + b2 (x)−a
a = (−a3
− a3
) + b2
(−a − a) = − − 2ab2 .
BC a 3 a 3 3
On CD: x = −a, dx = 0, y varies from b to 0 and F~ · d~r = 2aydy.
Z 0 !0
y2
Z
∴ ~
F · d~r = 2aydy = 2a = a(0 − b2 ) = −ab2 .
CD b 2 b
On DA: y = 0, dy = 0, x varies from −a to a and F~ · d~r = x2 dx.

Downloaded From : www.EasyEngineering.net


Downloaded From : www.EasyEngineering.net

Vector Calculus 279

a !a
x3 2a3
Z Z
1
∴ F~ · d~r = x dx = 2
= (a3 + a3 ) = .
DA −a 3 −a 3 3

2a3 2a3
Z
Hence, F~ · d~r = −ab2 − − 2ab2 − ab2 + = −4ab2 . (2)
c 3 3
Z "
From (1) and (2) we obtain {F~ · d~r} = {curlF~ · ~n}ds
C S
Hence, Stoke’s theorem is verified.

Example 2.109. Verify Stoke’s theorem for the vector field F~ = (2x − y)~i − yz2 ~j − y2 z~k

ww over the upper half surface of x2 + y2 + z2 = 1 bounded by its projection on the xy


plane.

w.E
Solution. The projection of the upper
Z

half of the given sphere on the xy plane


asy S

is the circle x2 + y2 = 1 [z = 0].


Z
~
F · d~r =
Z 
En
(2x − y)dx − yz2 dy − y2 zdz
 A O
Y

C C
Z
gin X

=
C
(2x − y)dx.

ee
Since C is the unit circle, change x and y interms of the parametric coordinates. rin
x = cos θ, y = sin θ, dy = − sin θdθ.
g.n
et
Since C is the full circle, θ varies from 0 to 2π.
Z Z2π
F~ · d~r = (2 cos θ − sin θ)(− sin θ dθ)
C 0
Z2π
= − (2 sin θ cos θ − sin2 θ) dθ
0
Z2π !
1 − cos 2θ
=− sin 2θ − dθ
2
0

Downloaded From : www.EasyEngineering.net


Anna University
Downloaded Solved Question Papers
From : www.EasyEngineering.net 113

∴ Cayley Hamilton theorem is verified.


Step 3. To find A−1
By Cayley Hamilton theorem we have A3 − 6A2 + 8A − 3I = 0.
Multiply by A−1 we get

A2 − 6A + 8I − 3A−1 = 0
3A−1 = A2 − 6A + 8I
     
 7 −6 9   2 −1 2  1 0 0
     
3A−1 = −5 6 −6 − 6 −1 2
 
−1 + 8 0

1 0
     

ww 
5 −5 7
 
1 −1 2
 
0 0 1

w.E  7 −6 9  −12





= −5 6 −6 +  6

 


 

 −12
6 −12 8
 
6  + 0
 
0 0
8 0


a 
 3 0 −3 syE
5 −5 7

−6

 3
6 −12

0 −3

0 0 8

3A−1

=  1 2 0  ⇒ A =  1


 ngi
−1 1
3 

2 0  .


−1 1 3
nee
−1 1 3

11. (b) Reduce the quadratic form 3x + 5y2 + 3z2 − 2xy − 2yz + 2zx into 2
rin
canonical form.
g.n
Solution. Refer question no. 11. (b) (i) of Nov/Dec. 2014 [2.1.2]
et
2.2 Unit II Vector Calculus

2.2.1 May/June 2016 [R 2013]


Part A

1. Evaluate ∇2 log r.
Solution. We have ~r = x~i + y~j + z~k
p
r = |~r| = x2 + y2 + z2 .
∴ r2 = x2 + y2 + z2 .

Downloaded From : www.EasyEngineering.net


———-
114
Downloaded Supplement to Engineering Mathematics - II
From : www.EasyEngineering.net

∂r ∂r x ∂r y ∂r z
2r = 2x ⇒ = . Similarly, = , = .
∂x ∂x r ∂y r ∂z r
∂ ∂ ∂
∇(log r) = ~i (log r) + ~j (log r) + ~k (log r)
∂x ∂y ∂z
1 ∂r 1 ∂r 1 ∂r 1 h x~i y~j z~k i ~r
= ~i . + ~j . + ~k . = + + = 2.
r ∂x r ∂y ! r ∂z ! r r r r r
2 ~r 1 1 
∇ log r = ∇ · ∇ log r = ∇ · 2 = ∇ 2 · ~r + 2 ∇ · ~r
r r r
−2−2 1 −4 2 3 −2 3 1
= −2r ~r · ~r + 2 · 3 = −2r r + 2 = 2 + 2 = 2 .
r r r r r
2. State Stoke’s theorem.
Solution. Refer page 98 of the main text book.
ww
Part B
w.E
11. (a) (i) If ∇φ = 2xyz3~i + x2 z3 ~j + 3x2 yz2~k find φ(x, y, z) given that

a
φ(1, −2, 2) = 4.
syE
Solution. Refer Example 1.23 on page 19 of the main text
book. ngi
11. (a)Z (ii) Using Green’s theorem in a plane evaluate nee
C rin
x2 (1 + y)dx + (x3 + y3 )dy where C is the square formed by
x = ±1 and y = ±1.
Solution. Comparing the given integral with Green’s
g.n
theorem we have
2 3 3
et
P = x (1 + y), Q= x +y .
∂P ∂Q
= x2 , = 3x2 .
∂y ∂x
By Green’s theorem,
I "  "
∂Q ∂P 
(Pdx + Qdy) = − dxdy = (3x2 − x2 )dxdy
C ∂x ∂y
R R
" Z 1 Z 1
= 2x2 dxdy = 2x2 dxdy
y=−1 x=−1
R
Z 1 " #1 Z 1
2x3 2
= dy = [1 + 1] dy
y=−1 3 x=−1 3 y=−1

Downloaded From : www.EasyEngineering.net


———-
Anna University
Downloaded Solved Question Papers
From : www.EasyEngineering.net 115

4  1 4 8
= y y=−1 = [1 + 1] = .
3 3 3

11. (b) (i) Find a and b so that the surfaces ax3 − by2 z = (a + 3)x2 and
4x2 y − z3 = 11 cut orthogonally at (2, −1, −3).
Solution. Refer Example 1.19 on page 16 of the main text
book.

11. (b) (ii) Prove that CurlCurlF~ = graddivF~ − ∇2 F.


~
Solution. Refer Identity VIII on page 15 of the main text book.

ww
2.2.2 Dec.2015/Jan. 2016 [R 2013]

Part A w.E
a −~r
1. Prove that Grad(1/r) =
r3
.
syE
ngi
Solution. Refer Example 1.26 subdivision (iii) on page 22 of
the main text book.

2. Evaluate
Z
nee
(yz~i + xz~j + xy~k) · d~r where C is the boundary of a
C rin
surface S .
Solution. By
Z Stoke’s theorem we have g.n
~
F · d~r =
Z Z

c
curlF~ · ~nds.
S
et
Here, F~ = yz~i + xz~j + xy~k.

~i ~j ~k

curlF~ = ∂x ∂ ∂
∂y

∂z

yz xz xy

= ~i(x − x) − ~j(y − y) + ~k(z − z) = ~0.


Z Z Z
∴ F~ · d~r = 0 · ~nds = 0.
c S

Part B

Downloaded From : www.EasyEngineering.net


———-
116
Downloaded Supplement to Engineering Mathematics - II
From : www.EasyEngineering.net

11. (a) Verify Green’s theorem for F~ = (x2 + y2 )~i − 2xy~j taken round
the rectangle bounded by the lines x = ±a, y = 0 and y = b.
Solution. Refer Example 1.79 on page 76 of the main text
book.

~ = (y + z)~i + (z + x)~j + (x + y)~k.


11. (b) (i) A fluid motion is given by V
Is this motion irrotational and is this possible for an
incompressible fluid?
Solution. Given, V~ = (y + z)~i + (z + x)~j + (x + y)~k.

ww ~ =∇×V
~i

~ = ∂
~j

~k

w.E curlV ∂x

y+z
∂y
z+x
∂z
x + y

a syE = ~i(1 − 1) − ~j(1 − 1) + ~k(1 − 1) = ~0.

∴ The motion is irrotational.


ngi
11. (b) (ii) Verify Gauss divergence
nee theorem for
~ 2 ~ 2 ~ 2 ~
rin
F = (x − yz)i + (y − xz) j + (z − xy)k. And S is the surface of the

g.n
rectangular parallelepiped bounded by x = 0, x = a, y = 0, y = b,
z = 0 and z = c.
Solution. Refer Example 1.91 on page 92 of the main text
book.
et
2.2.3 April/May 2015 [R 2013]

Part A

1. In what direction form (3, 1, −2) is the directional derivative of


φ = x2 y2 z4 maximum? Find also the magnitude of this
maximum.
Solution. Given φ = x2 y2 z4 .

Downloaded From : www.EasyEngineering.net


———-
Anna University
Downloaded Solved Question Papers
From : www.EasyEngineering.net 117

∂φ ~ ∂φ ~ ∂φ
▽φ = ~i +j +k
∂x ∂y ∂z
= (2xy2 z4 )~i + (2x2 yz4 )~j + (4x2 y2 z3 )~k.
(▽φ)(3,1,−2) = (2 × 3 × 1 × (−2)4 )~i + (2 × 9 × 1 × (−2)4 )~j
+ (4 × 9 × 1 × (−2)3 )~k
= 96~i + 288~j − 288~k.
Directional derivative is maximum in the direction of
▽φ = 96~i + 288~j − 288~k.
Maximum valuepof the directional derivative

ww |▽φ| = 962 + 2882 + (−288)2


p
= 962 + (96 × 3)2 + (96 × 3)2
w.E p √
= (96)2 (1 + 9 + 9) = 96 19.

a syE
2. Find α such that F~ = (3x − 2y + z)~i + (4x + αy − z)~j + (x − y + 2z)~k
is solenoidal.
ngi
Solution. Let F~ = (3x − 2y + z)~i + (4x + αy − z)~j + (x − y + 2z)~k.

∂ ∂ nee
Given that F~ is solenoidal.i.e.,∇.F~ = 0.


∂x
i.e., 3 + α + 2 = 0
∂y
rin
(3x − 2y + z) + (4x + αy − z) + (x − y + 2z) = 0.
∂z
⇒ α + 5 = 0 ⇒ α = −5.
g.n
Part B

11. (a) (i) Show that ∇2 (rn ) != n(n + 1)rn−2 where r2 = x2 +y2 +z2 . Hence
et
1
find the value of ∇2 .
r
Solution. Let ~r = x~i + y~j + z~k.

r2 = x2 + y2 + z2 .
∂r ∂r x ∂r y ∂r z
2r = 2x ⇒ = . similarly = , = .
∂x ∂x r ∂y r ∂z r
∂ ∂ ∂
We have, ∇ (rn ) = ~i (rn ) + ~j (rn ) + ~k (rn )
∂x ∂y ∂z
∂r ∂r ∂r
= ~inr n−1
+ ~jnr n−1
+ ~knrn−1
∂x ∂y ∂z

Downloaded From : www.EasyEngineering.net


———-
118
Downloaded Supplement to Engineering Mathematics - II
From : www.EasyEngineering.net
 x y z
= nrn−1 ~i + ~j + ~k
r r r
= nr (xi + y~j + z~k) = nrn−2~r.
n−2 ~

 
We have ∇2 (rn ) = ∇.∇ (rn ) = ∇. nrn−2~r
h   i
= n ∇ rn−2~r + rn−2 ∇.~r
h i
= n (n − 2)rn−4~r · ~r + rn−2 3
h i
= n (n − 2)rn−4 r2 + 3rn−2
h i
= n (n − 2)rn−2 + 3rn−2
= nrn−2 [n − 2 + 3] = n(n + 1)rn−2 .
ww
w.E Put n!= −1 in the above result we get
∇2
1
r
= (−1)(−1 + 1)rn−2 = 0.

a syE Z
11. (a) (ii) Using Green’s theorem, evaluate (y − sin x)dx + cos x dy

ngi π
where C is the triangle formed by y = 0, x =,y=
C
2x
.
nee
2 π
Solution. Comparing the given integral with Green’s theorem
we have
rin
P = y − sin x
∂P
Q = cos x.
∂Q g.n
∂y
By
=1
I Green’s theorem
(Pdx + Qdy) =
∂x
= − sin x.
" we have
(
∂Q ∂P
− )dxdy.
et
C R ∂x ∂y
Z "
i.e., {(y − sin x)dx + cos xdy} = (− sin x − 1)dxdy
C R

π 2x
Where R is the triangle formed by y = 0, x = , y = (or)
2 π
πy
x= .
2 Z 1Z π
2
=− (sin x + 1)dxdy
0 x= πy
I 2

∴ {(y − sin x)dx + cos xdy}


C

Downloaded From : www.EasyEngineering.net


———-
Anna University
Downloaded Solved Question Papers
From : www.EasyEngineering.net 119

Z 1   π2 Y
=− − cos x + x πy dy B(π/2, 1)
0 2
Z n
1
π π πy πy o

/2

x = π/2
πy
=− − cos + − (− cos + ) dy

=
x
0 2 2 2 2
X
(0, 0) o A(π/2, 0)

Z 1
π πy πy
=− ( + cos − )dy
0 2 2 2
" πy #
π 1  sin 2 1 π  y2 1
= − (y)0 + π −
2 2
0 2 2 0
" #
ww =−
π 2 π π
+ (sin − sin 0) − (1 − 0)
2 π 2 4

=−
"
π 2
w.E
+ ·1−
2 π
π
4
#
=−
"
π 2
+
4 π
#
.

a syE
11. (b) Verify Gauss divergence theorem for F~ = (4xz)~i − (y2 )~j + (yz)~k

ngi
taken over the cube bounded by the planes x = 0, x = 1, y = 0,
y = 1, z = 0, z = 1.
Solution.
" By$
Gauss divergence theorem, nee
F~ · ~nds = ~
∇ · Fdv
rin
S V
Given, F~ = 4xz~i − y2 ~j + yz~k.
g.n
The given surface is the cube Z

G
et
OABCDEFG. S is piecewise D

smooth which consists of F E


C
Y
O
six smooth surfaces namely
A B
ABEF, OCDG, BCDE, OAFG, OABC X

and DEFG.
" " " "
∴ {F~ · ~n}ds = {F~ · ~n}ds + {F~ · ~n}ds + {F~ · ~n}ds
S ABEF OCDG OABC
" " "
+ {F~ · ~n}ds + {F~ · ~n}ds + {F~ · ~n}ds.
DEFG BCDE OAFG

Downloaded From : www.EasyEngineering.net


———-
120
Downloaded Supplement to Engineering Mathematics - II
From : www.EasyEngineering.net

The following table gives the necessary quantities for the


computation of the surface integral on each surface.
Face Equation Outward normal F~ · ~n ds
ABEF x=1 ~i 4z dydz
OCDG x=0 −~i −4xz = 0 dydz
BCDE y=1 ~j −y = −1 2
dxdz
OAFG y=0 −~j −y = 0 2
dxdz
OABC z=0 −~k −yz = 0 dxdy
DEFG z=1 ~k yz = y dxdy

ww "
~
F · ~nds =
Z1 Z1
4zdydz = 4
Z 1
1
Z1
z(y)0 dz=4 zdz = 4
z2
!1
= 2.
ABEF
" w.E 0
Z1 Z1
0
0 2 0
Z1
0
Z1

BCDE
"
a
F~ · ~nds =
0
Z1 Z1
0
syE
(−1)dxdz = −

Z1
0
!
2 1
(x)10 dz=−
0
dz = −(z)10 = −1.

F~ · ~nds = ydxdy =
ngi y
2 0
1 1
dx= (x)10 = .
2 2
DEFG
All"
0 0
the other integrals will be zero.
1 3
x=0

nee

S
F~ · ~nds = 2 − 1 + = .
2 2
rin (1)

$ g.n
Now,
V
~
∇ · Fdv
Z 1Z 1Z 1
∂F1 ∂F2 ∂F3
!
et
= + + dzdydx
0 0 0 ∂x ∂y ∂z
Z 1 Z 1 Z 1
= (4z − 2y + y)dzdydx
0 0 0
Z 1 Z 1 Z 1 #1 Z 1 Z 1 "
z2
= (4z − y)dzdydx = 4 − yz dydx
0 0 0 0 0 2 0
Z 1Z 1 Z 1  !
2 1 
 y 
= (2 − y)dydx = 2(y)10 − dx
0 0 0 2 0
Z 1 !
1 3 3
= 2 − dx = (x)10 = . (2)
0 2 2 2

Downloaded From : www.EasyEngineering.net


———-
Anna University
Downloaded Solved Question Papers
From : www.EasyEngineering.net 121

" $
From (1) and (2) we have F~ · ~nds = ~
∇ · Fdv.
S V
Hence, Gauss Divergence theorem is verified.

2.2.4 November/December 2014 [R 2013]


Part A

1. Find curlF~ if F~ = xy~i + yz~j + zx~k.


Solution. Given, F~ = xy~i + yz~j + zx~k.

~i ~j ~k

ww
curlF~ = ∂x

∂ ∂
∂y
xy yz zx

∂z

w.E
= ~i
∂ ∂ 
(zx) − (yz) − ~j
∂ ∂ 
(zx) − (xy) + ~k
∂ ∂
(yz) − (xy)


a ∂y
syE
∂z ∂x ∂z ∂x
= ~i(0 − y) − ~j(z − 0) + ~k(0 − x) = −y~i − z~j − x~k.
∂y

2. State Stoke’s theorem. ngi


nee
Solution. If S is an open surface bounded by a simple closed

H !
rin
curve C and if F~ is continuous having continuous partial
derivatives in S and on C, then F~ · d~r = CurlF~ · ~nds where C
is traversed in the positive direction.
C S
g.n
Part B et
11. (a) (i) Find the angle between the normals to the surface x2 = yz
at the points (1, 1, 1) and (2, 4, 1).
Solution. Given φ = x2 − yz.
Let ~n1 and ~n2 be the normals to the surface φ at (1,1,1) and
(2,4,1) respectively.
∂φ ~ ∂φ ~ ∂φ ~
▽φ = ~i +j +k = i(2x) + ~j(−z) + ~k(−y).
∂x ∂y ∂z
~n1 = (▽φ)(1,1,1) = 2~i − ~j − ~k.
~n2 = (▽φ)(2,4,1) = 4~i − ~j − 4~k.

Downloaded From : www.EasyEngineering.net


———-
122
Downloaded Supplement to Engineering Mathematics - II
From : www.EasyEngineering.net

Let θ be the angle between the two normals.

~n1 .~n2
∴ cos θ = .
~n1 ~n2
8+1+4
= √ √
4 + 1 + 1 16 + 1 + 16
13 13 13
= √ √ = √ = √
6 33 6 × 33 198
!
13
∴ θ = cos−1 √ .
198

ww
11. (a)Z (ii) Using
2 2
Green’s theorem evaluate
[(3x − 8y )dx + (4y − 6xy)dy] where C is the boundary of the
C
w.E
triangle formed by the lines x = 0, y = 0, x + y = 1 in the

Solution.
a
xy−plane.
I By Green’s theorem,
" syE
∂Q ∂P
!

C
(Pdx + Qdy) =
R ∂x

∂y
dxdy.
ngi (1)
P = 3x2 − 8y2
∂P
= −16y
Q = 4y − 6xy.
∂Q
= −6y. nee
∂y
Applying in (1) we get
∂x
rin
Z  
3x2 − 8y2 dx + (4y − 6xy) dy g.n
=
C
"
(−6y + 16y)dxdy
et
"R
= 10ydxdy
R

" Z 1 Z 1−x Z 1 !1−x


y2
= 10 ydydx = 10 ydydx = 10 dx
R x=0 y=0 x=0 2 0
Z 1 " #1
10 (1 − x)3 5 5
= (1 − x)2 dx = 5 = − [0 − 1] = .
2 0 −3 0 3 3

11. (b) (i) Verify divergence theorem for F~ = 4xz~i − y2 ~j + yz~k taken
over the cube bounded by the planes x = 0, x = 1, y = 0, y = 1,

Downloaded From : www.EasyEngineering.net


———-
Anna University
Downloaded Solved Question Papers
From : www.EasyEngineering.net 123

z = 0, z = 1.
Solution. Refer question 11. (b) of April/May 2015.

11. (b) (ii) Prove that F~ = (x2 − y2 + x)~i − (2xy + y)~j is irrational and
hence find its scalar potential.
Solution. Given F~ = (x2 − y2 + x)~i − (2xy + y)~j.

~i ~j ~k

∇ × F~ = ∂x
∂ ∂
∂y

∂z

x2 − y2 + x −2xy − y 0

ww = ~i(0 − 0) − ~j(0 − 0) + ~k(−2y + 2y) = ~0.


∴ F~ is irrotational.
w.E
a syE
Since ∇ × F~ = ~0, F~ = ∇φ, where φ is a scalar potential.
∂φ ~ ∂φ ~ ∂φ
∴ F~ = ~i
∂x
+j
∂y
+k
∂z ngi
= (x2 − y2 + x)~i − (2xy + y)~j.

Comparing the components of ~i,~j & ~k we get nee


∂φ
= x2 − y2 + x rin (1)
∂x
∂φ
= −2xy − y g.n (2)
∂y
∂φ
∂z
= 0.
et
(3)

Integrating (1), (2) and (3) partially w.r.t. x, y and z respectively


we get
Z
φ= (x2 − y2 + x)dx + f1 (y, z)

x3 x2
φ= − xy2 + + f1 (y, z) (4)
Z3 2
φ= (−2xy − y)dy + f2 (x, z)

−2xy2 y2 y2
φ= − + f2 (x, z) = −xy2 − + f2 (x, z) (5)
2 2 2

Downloaded From : www.EasyEngineering.net


———-
124
Downloaded Supplement to Engineering Mathematics - II
From : www.EasyEngineering.net

φ = f3 (x, y) (6)

From (4), (5) and (6) we get


x3 x2 y2
φ= − xy2 + − + c, where c is a complex constant.
3 2 2

2.2.5 May/June 2014 [R 2013]


Part A

1. Find the unit normal vector to the surface x2 +y2 = z at (1, −2, 5).
Solution. φ = x2 + y2 − z
ww ∇φ = ~i
∂φ ~ ∂φ ~ ∂φ ~
+j +k = i(2x) + ~j(2y) + ~k(−1).

w.E ∂x ∂y ∂z
(∇φ)(1,−2,5) = 2~i − 4~j − ~k.

a

|∇φ| = 4 + 16 + 1 = 21.
syE
∇φ

1
= √ (2~i − 4~j − ~k).
Unit normal =
|∇φ| 21
ngi
2. Prove that curl(grad φ) = ~0.
∂φ ~ ∂φ ~ ∂φ
nee
Solution. We have ∇φ = ~i
∂x
+j
∂y
+k .
∂z
rin

~i


~j ~k
g.n
∇ × ∇φ = ∂x ∂y ∂∂z .

∂ ∂

∂φ ∂φ ∂φ
∂x ∂y

∂z
et
! ! !
~ ∂2 φ ∂ φ 2
~ ∂2 φ ∂2 φ ~ ∂2 φ ∂2 φ
=i − −j − +k −
∂y∂z ∂z∂y ∂x∂z ∂z∂x ∂x∂y ∂y∂x
∂2 φ ∂2 φ
=0 [assuming = etc]
∂y∂z ∂z∂y
∇ × ∇φ = 0 always.

Part B

11. (a) Verify Gauss divergence theorem for F~ = x2~i + y2 ~j + z2~k taken
over the cube bounded by the planes x = 0, y = 0, z = 0, x = 1,

Downloaded From : www.EasyEngineering.net


———-
Anna University
Downloaded Solved Question Papers
From : www.EasyEngineering.net 125

y = 1 and z = 1.
Solution. By Gauss divergence theorem
" $
~
F · ~nds = ~
∇ · Fdv.
S V
∂F1 ∂F2 ∂F3
∇ · F~ = + +
∂x ∂y ∂z
∂ 2 ∂ 2 ∂
= (x ) + (y ) + (z2 ) = 2x + 2y + 2z.
∂x ∂y ∂z
$ Z1 Z1 Z1
Now, ~ =
∇ · Fdv (2x + 2y + 2z)dxdydz

ww V z=0 y=0 x=0

Z1 Z1  !1 

w.E =
 x2
2
2 0
1

+ 2(y)(x)0 + 2z(x)0  dydz
1

a
0 0

syE =
Z1 Z1
(1 + 2y + 2z)dydz
0
Z1
0
ngi !1
= y+2·
y2
2 nee
+ 2zy
y=0
dz
0
Z1 rin
Z1
= (1 + 1 + 2z)dz = (2 + 2z)dz
g.n
=2
0
Z1
(1 + z)dz =
0

2(1 + z)2
!1
= 4 − 1 = 3.
et
2 0
0
Now,
" we shall evaluate
F~ · ~nds.
S
The cube has six faces namely
S 1 : ABEF, S 2 : OCDG,
S 3 : BCDE, S 4 : OAFG,
S 5 : OABC, S 6 : DEFG.

Downloaded From : www.EasyEngineering.net


———-
126
Downloaded Supplement to Engineering Mathematics - II
From : www.EasyEngineering.net
" " "
∴ F~ · ~nds = F~ · ~nds + F~ · ~nds
S S S
" "1 "2 "
+ F~ · ~nds + F~ · ~nds + F~ · ~nds + F~ · ~nds.
S3 S4 S5 S6
The following table gives the necessary quantities for the
computation of the surface integral over each face.
Face Equation Outward normal ~n F~ · ~n ds
S1 x=1 ~i x2 = 1 dydz
S2 x=0 −~i 2
−x = 0 dydz
~j
ww
2
S3 y=1 y =1 dxdz
S4 y=0 −~j −y2 = 0 dxdz
S5
w.E z=0 −~k
~k
2
−z = 0
2
dxdy
S6
a z=1

syE
z =1 dxdy

"
F~ · ~nds =
Z1 Z1
ngi Z1 Z1

S1
"
y=0 z=0
dydz =
y=0 nee
(y)10 dz =
0
dz = (z)10 = 1.

F~ · ~nds = 0. rin
S2
"
g.n
S3
F~ · ~nds =
Z1 Z1

z=0 x=0
dxdz =
Z1

0
(x)10 dz =
Z1

0
dz = (z)10 = 1. et
"
F~ · ~nds = 0.
S4
"
F~ · ~nds = 0.
S5
" Z1 Z1 Z1 Z1
F~ · ~nds = dxdy = (y)10 dx = dx = (x)10 = 1.
S6 x=0 y=0 0 0
"
Now, F~ · ~nds = 1 + 0 + 1 + 0 + 0 + 1 = 3
S

Downloaded From : www.EasyEngineering.net


———-
Anna University
Downloaded Solved Question Papers
From : www.EasyEngineering.net 127

" $
From (1) and (2) we have F~ · ~nds = ~
∇ · Fdv
S V
∴ Gauss divergence theorem is verified.

11. (b) (i) Find the value of n such that the vector rn~r is both
solenoidal and irrotational.
Solution. Given ~r = x~i + y~j + z~k.
∂r x ∂r y ∂r z
r2 = x2 + y2 + z2 . ⇒ = , = , =
∂x r ∂y r ∂z r
Let F~ = rn~r = rn x~i + rn y~j + rn z~k.

ww F1 = rn x, F2 = rn y, F3 = rn z

w.E
∇ · F~ =
∂F1 ∂F2 ∂F3
∂x
+
∂y
+
∂z
.

a
=
∂ n
∂x
n

∂y
syE
(n−1) ∂r

(r x) + (rn y) + (rn z).
∂z
∂r ∂r
+ rn + y × n × r(n−1) + rn + z × n × r(n−1)
=r +x×n×r

∂x
x y ngiz 
∂y ∂z
= 3rn + nr(n−1) x × + y × + z ×
r r r
nee
= 3rn + nr(n−2) (x2 + y2 + z2 )
rin
= 3rn + nr(n−2) × r2 = 3rn + nrn = (n + 3)rn .
g.n
If rn~r is solenoidal then ∇ · F~ = 0.

n
i.e., (n + 3)r = 0. ⇒ n + 3 = 0. ⇒ n = −3.
et

~i ~j ~k

curlF~ = ∂x ∂ ∂
∂y

∂z

r n x r y r n z
n
! ! !
~ ∂(rn z) ∂(rn y) ~ ∂(rn z) ∂(rn x) ~ ∂(rn y) ∂(rn x)
=i − −j − +k − .
∂y ∂z ∂x ∂z ∂x ∂y
 y z 
= ~i z × n × r(n−1) × − y × n × rn−1 − ~j(0) + ~k(0) = ~0
r r
∴ rn~r is irrotational for all values of n.

Downloaded From : www.EasyEngineering.net


———-
128
Downloaded Supplement to Engineering Mathematics - II
From : www.EasyEngineering.net

11. (b) (ii) Verify Stokes theorem for F~ = (x2 − y2 )~i + 2xy~j in the
rectangular region of z = 0 plane bounded by the lines x = 0,
y = 0, x = a and y = b.
Solution. We have F ~ 2 2~
+ 2xy~j
I = (x − y )i"
By stoke’s theorem, {F~ · d~r} = {curlF~ · ~n}ds
C S
Now,
~i ~j ~k

CurlF~ = ∂
∂x

∂y
∂ = ~i(0 − 0) − ~j(0 − 0) + ~
∂z
k(2y + 2y) = 4y~k.

x2 − y2 2xy 0

ww Since the surface is a

w.E
rectangle in the xy plane, the
normal is ~n = ~k.

a
curlF~ · ~n = 4y~k · ~k = 4y.

" "
syE
curlF~ · ~nds = 4ydxdy ngi
S

=
Z
R
a Z b
4ydydx nee
!b Z
x=0
Z aa
y=0
rin
=
0
4
y2
2 0
dx = 2
0
b2 dx = 2b2 (x)a0 = 2ab2 .
g.n (1)
Z
We shall now evaluate {F~ · d~r}.
C
et
Z Z Z Z Z
{F~ · d~r} = {F~ · d~r} + {F~ · d~r} + {F~ · d~r} + {F~ · d~r}
C OA AB BC CO
Now, F~ · d~r = (x2 − y2 )dx + 2xydy
Along OA: y = 0, dy = 0, x varies from 0 to a.
Z Z a !a
~ 2 x3 a3
∴ {F · d~r} = x dx = = .
OA 0 3 0 3
Along AB: x = a, dx = 0, y varies from 0 to b.
Z Z b !b
~ y2
∴ {F · d~r} = 2aydy = 2a = ab2 .
AB 0 2 0
Along BC: y = b, dy = 0, x varies from a to 0.

Downloaded From : www.EasyEngineering.net


———-
Anna University
Downloaded Solved Question Papers
From : www.EasyEngineering.net 129

Z Z 0 !0
x3
∴ {F~ · d~r} = x − b dx =2 2
− b2 (x)0a .
BC a 3 a
a3 a3
= − + ab2 = ab2 − .
3 3
Along CO: x = 0, dx = 0, y varies from b to 0.
Z Z 0
~
{F · d~r} = 0dx = 0.
CO b
Z
a3 a3
∴ {F~ · d~r} =
+ ab2 + ab2 − + 0 = 2ab2 . (2)
C 3 3
I "
From (1) and (2) we obtain {F~ · d~r} = {curlF~ · ~n}ds.

ww Hence, Stoke’s theorem is verified.


C S

w.E
2.2.6 November/December 2013[R 2008]

Part A a syE
1. Define Solenoidal vector function. If V ngi
~ = (x + 3y)~i + (y − 2z)~j + (x +
2λz)~k is solenoidal, find the value of λ.
nee
Solution. Solenoidal vector. If divF~ = 0 everywhere in a
rin
region R, then F~ is called a solenoidal vector in R.
~ = (x + 3y)~i + (y − 2z)~j + (x + 2λz)~k.
Given V g.n
~ is solenoidal.
Given V

~ =0
et
∴ ∇·V
∂ ∂ ∂
i.e., (x + 3y) + (y − 2z) + (x + 2λz) = 0
∂x ∂y ∂z
1 + 1 + 2λ = 0
2λ = −2
λ = −1.

2. State Green’s theorem.


Statement: If P(x, y) and Q(x, y) are continuous functions with

Downloaded From : www.EasyEngineering.net


———-
130
Downloaded Supplement to Engineering Mathematics - II
From : www.EasyEngineering.net

continuous partial derivatives in a region R in the xy plane


and on its boundary
I " C which! is a simple closed curve, then
∂Q ∂P
(Pdx + Qdy) = − dxdy where C is described in the
∂x ∂y
C R
anticlockwise sense.

Part B

11. (a) (i) Show that the vector field F~ = (x2 + xy2 )~i + (y2 + x2 y)~j is
irrotational. Find its scalar potential.

ww Solution. Given F~ = (x2 + xy2 )~i + (y2 + x2 y)~j.

w.E

∇ × F =


~i ~j

~k

a ~

syE


∂x
x2 + xy2

∂y
y2 + x2 y
∂z
0

ngi
= ~i(0 − 0) − ~j(0 − 0) + ~k(2xy − 2xy)
= ~0.
nee
∴ F~ is irrotational. rin
Since ∇ × F~ = ~0, F~ = ∇φ, where φ is a scalar potential. g.n
∴ F~ = ~i
∂φ ~ ∂φ ~ ∂φ
∂x
+j
∂y
+k .
∂z
et
~ we get
Comparing the components of F,

∂φ
= x2 + xy2 (1)
∂x
∂φ
= y2 + x2 y (2)
∂y
∂φ
= 0. (3)
∂z

Integrating (1), (2) and (3) partially w.r.t. x, y and z respectively

Downloaded From : www.EasyEngineering.net


———-
Anna University
Downloaded Solved Question Papers
From : www.EasyEngineering.net 131

we get
Z
x3 x2 y2
φ = (x2 + xy2 )dx + f1 (y, z) ⇒ φ= + + f1 (y, z). (4)
3 2
Z
y3 x2 y2
φ = (y2 + x2 y)dy + f2 (x, z) ⇒ φ= + + f2 (x, z). (5)
3 2
φ = f3 (x, y). (6)

From (4), (5) and (6) we get


x3 y3 x2 y2
φ= + + + c, where c is a constant.
3 3 2
 
11. (a) (ii) Verify Stoke’s theorem for F~ = x2 + y2 ~i − 2xy~j taken
ww around the rectangle formed by the lines

w.E
x = −a, x = a, y = 0, y = b.
Solution. By Stoke’s theorem we have
I

C
a
{F~ · d~r} =
"

syE
{curlF~ · ~n}ds.
S

ngi
Here, F~ = (x2 + y2 )~i − 2xy~j

~i

~j

~k


nee
~
curl F = ∂x

∂ ∂
∂y

∂z
x2 + y2 −2xy 0

rin
= ~i(0 − 0) − ~j(0 − 0) + ~k(−2y − 2y) = −4y~k
g.n
The surface S is given as a
et
rectangle in the xy plane.
Hence, ~n = ~k.

" " Zb Za
{curlF~ · ~n}ds = −4y~k.~kdxdy = −4 ydxdy
S S
0 −a
" #b
y2
= −4 [x]a−a = −2[b2 ][a + a] = −4ab2 . (1)
2 0

Downloaded From : www.EasyEngineering.net


———-
132
Downloaded Supplement to Engineering Mathematics - II
From : www.EasyEngineering.net
I
Now, we shall evaluate the line integral {F~ · d~r}
C
F~ · d~r = [(x2Z+ y2 )~i − 2xy
I
~ ~
Z j].(dxi + dy
~ 2 2
Z j) = (x +Zy )dx − 2xydy.
F~ · d~r = F~ · d~r + F~ · d~r + F~ · d~r + F~ · d~r.
C
AB BC CD DA
On AB: x = a, dx = 0, y varies from 0 to b and F~ · d~r = −2aydy.
Z Z b !b
~ y2
∴ F · d~r = −2aydy = −2a = −ab2 .
AB 0 2 0
On BC: y = b, dy = 0, x varies from a to −a and F~ · d~r = (x2 + b2 )dx.
Z Z −a !−a
~ 2 2 x3
∴ F · d~r = (x + b )dx = + b2 (x)−a
a .
3 a
ww BC a
1
= (−a3 − a3 ) + b2 (−a − a)

w.E =−
3
2a3
− 2ab2 .

Z
a Z 0 syE y2
!0
3
On CD: x = −a, dx = 0, y varies from b to 0 and F~ · d~r = 2aydy.


CD
~
F · d~r =
b
2aydy = 2a
ngi2 b
= a(0 − b2 ) = −ab2 .

Z
~
F · d~r =
Z a
2
x dx =
x3 1 nee
On DA: y = 0, dy = 0, x varies from −a to a and F~ · d~r = x2 dx.
!a
= (a3 + a3 ) =
2a3
.
rin

DA −a 3 −a 3 3

Hence,
Z
F~ · d~r = −ab2 −
2a3
3
− 2ab2 − ab2 +
2a3
3 g.n
= −4ab2 . (2)
c
I
From (1) and (2) we obtain {F~ · d~r} =
C
"

S
{curlF~ · ~n}ds
et
Hence, Stoke’s theorem is verified.

11. (b) (i) Find the values of a and b so that the surfaces ax3 − by2 z =
(a + 3)x2 and 4x2 y − z3 = 11 may cut orthogonally at (2,-1,-3).
Solution. Let φ1 and φ2 be the given surfaces.

∴ φ1 = ax3 − by2 z − (a + 3)x2 , and φ2 = 4x2 y − z3 − 11.


∂φ1 ~ ∂φ1 ~ ∂φ1
▽φ1 = ~i +j +k
∂x ∂y ∂z
= [3ax2 − 2(a + 3)x]~i − 2byz~j − by2~k

Downloaded From : www.EasyEngineering.net


———-
Anna University
Downloaded Solved Question Papers
From : www.EasyEngineering.net 133

(▽φ1 )(2,−1,−3) = (12a − 4(a + 3))~i − 6b~j − b~k


= (8a − 12)~i − 6b~j − b~k.
∂φ2 ~ ∂φ2 ~ ∂φ2
▽φ2 = ~i +j +k
∂x ∂y ∂z
▽φ2 = 8xy~i + 4x2 ~j − 3z2~k.
(▽φ2 )(2,−1,−3) = −16~i + 16~j − 27~k.

Given that the surfaces are orthogonal.

ww ∴ ▽φ1 .▽φ2 = 0

w.E
−16(8a − 12) + 16(−6b) + 27b = 0
−128a + 192 − 96b + 27b = 0

a syE
−128a − 69b + 192 = 0
128a + 69b = 192. (1)
ngi
(2, −1, −3)lies on φ1 .
∴ 8a + 3b − 4(a + 3) = 0 nee
8a + 3b − 4a − 12 = 0 rin
4a + 3b = 12 g.n (2)
(2) × 23 ⇒ 92a + 69b = 276.

(1) − (3) ⇒ 36a = −84 ⇒ a =


−84 −7
= .
et
(3)

36 3

Substituting in (2) we get


!
7 28 36 + 28 64 64
4 − + 3b = 12 ⇒ 3b = 12 + = = ⇒b= .
3 3 3 3 9

11. (b) (ii) Verify Gauss divergence theorem for F~ = 4xz~i − y2 ~j + yz~k
over the cube bounded by x = 0, x = 1, y = 0, y = 1, z = 0, z = 1.
Solution. Refer question 11. (b) of April/May 2015.

Downloaded From : www.EasyEngineering.net


———-
134
Downloaded Supplement to Engineering Mathematics - II
From : www.EasyEngineering.net

2.2.7 May/June 2013 [R 2008]

1. Find the directional derivative of φ = xyz at (1, 1, 1) in the


direction of ~i + ~j + ~k.
Solution. φ = xyz
∂φ ~ ∂φ ~ ∂φ ~
∇φ = ~i +j +k = i(yz) + ~j(xz) + ~k(xy)
∂x ∂y ∂z
(∇φ)(1,1,1) = ~i + ~j + ~k.
Let ~a = ~i + ~j + ~k.
~a
Directional derivative in the direction of ~a = ∇φ.
|~a|

ww ~i + ~j + ~k
(~i + ~j + ~k) √
1+1+1
=
1+1+1 √

3
= 3.

w.E
~ and B
~ are irrotational, prove that A~×B ~ is solenoidal.
2. If A
a
Solution. Given that A
syE
~ and B~ are irrotational.
~ = 0 and ∇ × B
~ = 0.
Therefore, ∇ × A
Now we have, ngi
~ × B)
∇.(A ~ = A.(∇
~ × B)~ − B.(∇
~ ~ = A.
× A) nee
~ ~0 − B.~0 = 0.
=⇒ A ~×B ~ is solenoidal.
rin
Part B g.n
11.(a) Using Stoke’s theorem, evaluate
R
C
F~ · d~r where F~ = y2~i + x2 ~j − et
(x + z)~k where C is the boundary of the triangle with vertices
(0, 0, 0), (1, 0, 0) & (1, 1, 0).
H !
Solution. By Stoke’s theorem we have F~ · d~r = curlF~ · b
nds.
C S

Here, F~ = y2~i + x2 ~j − (x + z)~k



~i ~j ~k

∂ ∂ ∂
crul F~ = ∇ × F~ =
∂x ∂y ∂z
y2 x2 −(x + z)

Downloaded From : www.EasyEngineering.net


———-
Anna University
Downloaded Solved Question Papers
From : www.EasyEngineering.net 135

= ~i(0 − 0) − ~j(−1 − 0) + ~k(2x − 2y) = ~j + 2(x − y)~k

Now S is the surface of the triangle OAB and since it lies in the
n = ~k.
xy plane, we have b
h i
n = ~j + 2(x − y)~k · ~k = 2(x − y).
Now, curlF~ · b
" "
∴ ~
curl F · b
nds = 2 (x − y)dxdy
S S
Z1 Z1
=2 (x − y)dxdy

ww y=0 x=y

Z1 !1
w.E =2
x2
2
− yx
x=y
dy

a syE=2
0
Z1
1
−y−
y2 2
!
+ y dy
0
Z1 ngi
2 2

!
=2
1
2
−y+
y2
2
dy nee
0
"
y y2 y3
#1 rin "
1 1 1
#
2 1
=2 −
2 2
+
6 0
=2 − +
2 2 6
= = .
6 3 g.n
11. (b) Verify divergence theorem for F~ = x2~i + z~j + yz~k over the cube
et
formed by the planes x = ±1, y = ±1, z = ±1.
Solution.
! #
By Gauss divergence theorem we have S
F~ · ~nds = V
~
divFdv.

F~ = x2~i + z~j + yz~k


∂ 2 ∂ ∂
divF~ =
(x ) + (z) + (yz) = 2x + y
∂x ∂y ∂z
$ Z1 Z1 Z1 Z1 Z1
~
divFdV = (2x + y)dzdydx = (2x + y)(z)1−1 dydx
V −1 −1 −1 −1 −1

Downloaded From : www.EasyEngineering.net


———-
136
Downloaded Supplement to Engineering Mathematics - II
From : www.EasyEngineering.net

Z1 Z1 Z1  !1 
 1 y2 
=2 (2x + y)dydx = 2 2x(y)−1 +  dx
2 −1 
−1 −1 −1
Z1 !
2 1
x
=2 4xdx = 8 =8×0=0 (1)
2 −1
−1
!
We shall evaluate F~ · ~nds. G F
S
z
The given cube has 6 faces
H y E
S 1 = BCFG, S 2 = ADEH,
x
S 3 = CDEF, S 4 = ABGH, A
D
ww S 5 = EFGH, S 6 = ABCD.
B C

w.E" 6 "
X
Now
a S
F~ · ~nds =
syE
i=1 S
i
F~ · ~nds

The following table gives the necessary quantities for the

ngi
computation of the surface integral over each face.
Face Equation Outward normal
S1 x=1
F~ · ~n
~i
ds
nee x2 = 1 dydz
S2 x = −1 −~i rin−x = −1 2
dydz
S3 y=1 ~j z dxdzg.n
S4
S5
y = −1
z=1
−~j
~k
−z
yz = y
dxdz
dxdy
et
S6 z = −1 −~k −yz = −y dxdy

" Z1 Z1 Z1
F~ · ~nds = dydz = (y)1−1 dz
S1 −1 −1 −1
Z1
=2 dz = 2(z)1−1
−1
"
= 4. F~ · ~nds = −4
S2

Downloaded From : www.EasyEngineering.net


———-
Anna University
Downloaded Solved Question Papers
From : www.EasyEngineering.net 137

" Z1 Z1
F~ · ~nds = zdxdz = 0
S3 −1 −1
" Z1 Z1
F~ · ~nds = (−z)dxdz = 0
S4 −1 −1
" Z1 Z1
F~ · ~nds = ydxdy = 0
S5 −1 −1
" Z1 Z1
F~ · ~nds = (−y)dxdy = 0.

ww S6
"
−1 −1

w.E ∴
S
F~ · ~nds = 4 − 4 + 0 + 0 = 0. (2)

a
From (1) and (2) we get
"
~
syE $
~
F · ~nds =
S ngi
divFdV.
V

Hence, Gauss Divergence theorem is verified. nee


rin
2.2.8 November/December 2012 [R 2008]
g.n
Part A

1. Prove that F~ = yz~i + zx~j + xy~k is irrotational.


et

~i ~j ~k

Solution. ∇ × F~ = ∂x ∂ ∂
∂y

∂z

yz zx xy
= ~i(x − x) − ~j(y − y) + ~k(z − z) = ~0.
∴ F~ is irrotational.

2. State Gauss Divergence theorem.


Solution. Let V be the volume bounded by a closed surface S .
If a vector function F~ is continuous and has continuous partial

Downloaded From : www.EasyEngineering.net


———-
138
Downloaded Supplement to Engineering Mathematics - II
From : www.EasyEngineering.net

derivatives inside and on S , then the surface integral of F~ over


S is equal to the volume integral of the divergence of F~ taken
throughout V.
! #
i.e., S
F~ · ~nds = V
~
∇ · Fdv.

Part B

11. (a) (i) Show that F~ = (2xy−z2 )~i+(x2 +2yz)~j+(y2 −2zx)~k is irrotational
and hence find its scalar potential.
ww Solution. Given F~ = (2xy − z2 )~i + (x2 + 2yz)~j + (y2 − 2zx)~k.

w.E
~i ~j ~k


~
a
∇ × F =




∂x
2xy − z2
syE ∂
∂y
x2 + 2yz

∂z
y2 − 2zx



ngi
= ~i(2y − 2y) − ~j(−2z + 2z) + ~k(2x − 2x) = ~0.

nee
∴ F~ is irrotational.
Since ∇ × F~ = ~0, F~ = ∇φ, where φ is a scalar potential.
rin
g.n
∴ F~ = ~i
∂φ ~ ∂φ ~ ∂φ
∂x
+j
∂y
+k .
∂z
et
Comparing the components of F~ we get

∂φ
= 2xy − z2 . (1)
∂x
∂φ
= x2 + 2yz. (2)
∂y
∂φ
= y2 − 2zx. (3)
∂z

Integrating (1), (2) and (3) partially w.r.t. x, y and z respectively

Downloaded From : www.EasyEngineering.net


———-
Anna University
Downloaded Solved Question Papers
From : www.EasyEngineering.net 139

we get
Z
φ= (2xy − z2 )dx + f1 (y, z)

x2
= 2y · − xz2 + f1 (y, z)
2
= x2 y − xz2 + f1 (y, z). (4)
Z
φ = (x2 + 2yz)dy + f2 (x, z)

y2
= x2 y + 2z · + f2 (x, z) = x2 y + y2 z + f2 (x, z). (5)
Z 2

ww φ = (y2 − 2zx)dz + f3 (x, y)

w.E = zy2 − 2x ·
z2
2
+ f3 (x, y)

a = zy2 − xz2 + f3 (x, y).


syE
From (4), (5) and (6) we get
(6)

ngi
φ = x2 y − xz2 + y2 z + c, where c is a constant.

nee
~ = (x2 + y2 )~i − 2xy~j taken
11. (a) (ii) Verify Green’s theorem for V
around the rectangle bounded by the lines
rin
I x = ±a, y = 0, y = b.
Solution. Consider the line integral ~ · d~r where C is the
V
C g.n
given rectangle.


I
V
I
~ · d~r = [(x2 + y2 )dx − 2xydy]
et
C
IC
= (Pdx + Qdy) where P = x2 + y2 , Q = −2xy.
C
I " !
∂Q ∂P
By Green’s theorem (Pdx + Qdy) = − dxdy.
C ∂x ∂y
R
∂P ∂Q
= 2y, = −2y.
∂y ∂x

Downloaded From : www.EasyEngineering.net


———-
140
Downloaded Supplement to Engineering Mathematics - II
From : www.EasyEngineering.net

" !
∂Q ∂P
Now, − dxdy
∂x ∂y
R
"
= (−2y − 2y)dxdy
R
"
=− 4ydxdy
R

Za Zb Za " #b
y2
=− 4ydydx = −4 dx
ww x=−a y=0 x=−a
2 0

w.E = −4
Za
b2
2
dx = −2b2 [x]a−a = −2b2 [a + a] = −4ab2 . (1)

a
Now let us evaluate
−a

syEZ
(Pdx + Qdy).
C
ngi
Z
(Pdx+Qdy) =
Z Z
(Pdx+Qdy)+ (Pdx+Qdy) nee
C
AB BC
rin
Z Z g.n
+
CD
(Pdx + Qdy) +
DA
(Pdx + Qdy).
et
Along AB, y = 0, dy = 0 and x varies from −a to a.

Z Za " #a
x3 1 3 2a3
(Pdx + Qdy) = x2 dx = = (a + a3 ) = .
3 −a 3 3
AB −a

Along BC, x = a, dx = 0 and y varies from 0 to b.

Z Zb " #b
y2
(Pdx + Qdy) = −2aydy = −2a = −ab2 .
2 0
BC 0

Downloaded From : www.EasyEngineering.net


———-
Anna University
Downloaded Solved Question Papers
From : www.EasyEngineering.net 141

Along CD, y = b, dy = 0 and x varies from a to −a.


Z Z−a " 3 #−a
2 2 x 2
∴ (Pdx + Qdy) = (x + b )dx = +b x
3 a
CD a
!
a3 a3
= − − ab2 − + ab2
3 3
a3 a3 2a3
= − − ab2 − − ab2 = − − 2ab2 .
3 3 3
Along DA, x = −a, dx = 0 and y varies from b to 0.
Z Z0 " #0

ww ∴ (Pdx + Qdy) = 2aydy = 2a


y2
2 b
= a(0 − b2 ) = −ab2 .

∴ w.E
DA
Z
(Pdx + Qdy) =
b
2a3
3
− ab2 −
2a3
3
− 2ab2 − ab2 = −4ab2 . (2)
C

a
From (1) and (2) we have
Z
syE" !
(Pdx + Qdy) =
C ngi ∂Q ∂P
∂x

∂y
dxdy.
R

∴ Green’s theorem is verified. nee


rin
11. (b) Verify Gauss divergence theorem for F~ = 4xz~i − y2 ~j + yz~k over
the cube bounded by x = 0, x = 1, y = 0, y = 1, z = 0, z = 1. g.n
Solution. Refer question 11. (b) of April/May 2015. et
2.2.9 May/June 2012 [R 2008]
Part A

1. Find λ such that F~ = (3x − 2y + z)~i + (4x + λy − z)~j + (x − y + 2z)~k is


solenoidal.
Solution. Refer question 2. of April/May 2015.

2. State Gauss Divergence theorem.


Statement. Refer question 2. of Nov./Dec. 2012.

Downloaded From : www.EasyEngineering.net


———-
142
Downloaded Supplement to Engineering Mathematics - II
From : www.EasyEngineering.net

Part B

11. (a) (i) Show that F~ = (y2 + 2xz2 )~i + (2xy − z)~j + (2x2 z − y + 2z)~k is
irrotational and hence find its scalar potential.
Solution.
Given that F~ = (y2 + 2xz2 )~i + (2xy − z)~j + (2x2 z − y + 2z)~k.

~i ~j ~k

∇ × F =
~ ∂
∂x

∂y

∂z


y2 + 2xz2 2xy − z2x2 z − y + 2z

ww = ~i
∂ ∂
(2x2 z − y + 2z) − (2xy − z)
!

w.E ∂y

− ~j

∂z

(2x2 z − y + 2z) − (y2 + 2xz2 )
!

a syE
+ ~k
∂x

∂x

∂z

(2xy − z) − (y2 + 2xz2 )


∂y
!

ngi
= ~i(−1 + 1) − ~j(4xz − 4xz) + ~k(2y − 2y)
= ~i.0 − ~j.0 + ~k.0 = ~0. nee
∴ F~ is irrotational. rin
Since ∇ × F~ = ~0, then F~ = ∇φ where φ is the scalar potential. g.n
∴ F~ = ~i
∂φ ~ ∂φ ~ ∂φ
∂x
+j
∂y
+k .
∂z
et
Comparing the components of F~ we obtain

∂φ
= y2 + 2xz2 (1)
∂x
∂φ
= 2xy − z (2)
∂y
∂φ
= 2x2 z − y + 2z. (3)
∂z

Integrating partially (1), (2), (3) w.r.t. x, y and z respectively we

Downloaded From : www.EasyEngineering.net


———-
Anna University
Downloaded Solved Question Papers
From : www.EasyEngineering.net 143

get
Z
φ= (y2 + 2xz2 )dx + f1 (y, z)

x2
= y2 x + 2z2 + f1 (y, z) = xy2 + x2 z2 + f1 (y, z). (4)
2
From (2) we get
Z
φ = (2xy − z)dy + f2 (x, z)

y2
= 2x − zy + f2 (x, z)
2

ww = xy2 − yz + f2 (x, z) (5)

w.E
From (3) we obtain
Z

a
φ = (2x2 z − y + 2z)dz + f3 (x, y)

= 2x2
z2 syEz2
− yz + 2 + f3 (x, y)
2 2
ngi
= x2 z2 − yz + z2 + f3 (x, y). (6)
nee
From (4), (5) and (6) we get
rin
∴ φ = xy2 + x2 z2 − yz + z2 + c, where c is a constant.
g.n
11. (a)Z (ii) Verify

C
2 2
Green’s theorem in the plane for
[(3x − 8y )dx + (4y − 6xy)dy] where C is the boundary of the et
region bounded by x = 0, y = 0, x + y = 1.
I By Green’s theorem,
Solution. " !
∂Q ∂P
(Pdx + Qdy) = − dxdy.
C R ∂x ∂y
P = 3x2 − 8y2 Q = 4y − 6xy.
∂P ∂Q
= −16y = −6y.
∂y ∂x
" ! " "
∂Q ∂P
Now, − dxdy = (−6y + 16y)dxdy = 10ydxdy
R ∂x ∂y R R
" Z 1 Z 1−x
= 10 ydydx = 10 ydydx
R x=0 y=0

Downloaded From : www.EasyEngineering.net


———-
144
Downloaded Supplement to Engineering Mathematics - II
From : www.EasyEngineering.net

Z 1 !1−x Z
y2 10 1
= 10 dx = (1 − x)2 dx
x=0 2 0 2 0
" #1
(1 − x)3 5 5
=5 = − [0 − 1] = . (1)
−3 0 3 3
I
Now, (Pdx + Qdy)
C Z Z Z
= (Pdx + Qdy) + (Pdx + Qdy) + (Pdx + Qdy).
OA AB BO

Along OA, y = 0 ⇒ dy = 0 and

ww x varies from 0 to 1.
Z Z 1
3x2 dx

w.E
(Pdx + Qdy) =
OA 0

x3
!1

a =3

syE3 0
= 1.

Along AB, y = 1 − x, dy = −dx, x varies from 1 to 0


Z ngi

AB
(Pdx + Qdy)
Z 0 nee
=
1
(3x2 − 8(1 − x)2 )dx + (4(1 − x) − 6x(1 − x))(−dx)
rin
=
Z 0
{3x2 − 8 − 8x2 + 16x − (4 − 4x − 6x + 6x2 )}dx g.n
=
1
Z 0

1
(−5x2 + 16x − 8 − 4 + 10x − 6x2 )dx
et
Z 0 !0 !0
x3 x2
= (−11x2 + 26x − 12)dx = −11 + 26 − 12 (x)01
1 3 1 2 1
11 11 11 8
= − (0 − 1) + 13(0 − 1) − 12(0 − 1) = − 13 + 12 = −1= .
3 3 3 3

Along BO, x = 0 ⇒ dx = 0 , y varies from 1 to 0.


Z Z 0 " 2 #0
y
(Pdx + Qdy) = 4ydy = 4 = 2[0 − 1] = −2.
I
BO 1 2 1
8 8 5
∴ (Pdx + Qdy) = 1 + − 2 = − 1 = . (2)
C 3 3 3
From (1) and (2) The Green’s theorem is verified.

Downloaded From : www.EasyEngineering.net


———-
Anna University
Downloaded Solved Question Papers
From : www.EasyEngineering.net 145

R
11. (b) (i) Using Stoke’s theorem, evaluate F~ · d~r where
C
F~ = y2~i + x2 ~j − (x + z)~k where C is the boundary of the triangle
with vertices (0, 0, 0), (1, 0, 0) & (1, 1, 0).
Solution. Refer problem 11. (a) of May/June 2013 (R2008).

11. (b) (ii) Find the work done in moving a particle in the vector field
~ = 3x2~i + (2xz − y)~j + z~k along the straight line from (0, 0, 0) to
A
(2, 1, 3).

ww ~ = 3x2~i + (2xz − y)~j + z~k.


Solution. Given A
The equation of the straight line joining (0, 0, 0) and (2, 1, 3) is

w.E
x−0 y−0
=
2−0 1−0 3−0
x y z
=
z−0
= t (say)

a
= = = t.
2 1 3
syE
∴ x = 2t, y = t, z = 3t.

Now A ngi
dx = 2dt, dy = dt, dz = 3dt.
~ = 3x2~i + (2xz − y)~j + z~k.

d~r = dx~i + dy~j + dz~k. nee


~ r = 3x2 dx + (2xz − y)dy + zdz
A.d~ rin
= 3.4t2 .2dt + (2.2t.3t − t)dt + 3t.3dt g.n
2 2
= 24t dt + (12t − t)dt + 9tdt = (36t + 8t)dt.
To find the limits for t.
2
et
when x = 0, t = 0.
when x = 2, t = 1.

Z Z1
∴ Now, work done = ~ r=
A.d~ (36t2 + 8t)dt
c 0
!
3 1
!1
t t2
= 36. + 8.
3 0 2 0
1 1
= 36. + 8. = 12 + 4 = 16.
3 2

Downloaded From : www.EasyEngineering.net


———-
Downloaded From : www.EasyEngineering.net

ww
w.E
UNIT 3 asy
En
gin
ee
By EasyEngineering.net rin
g.n
et

Downloaded From : www.EasyEngineering.net


Downloaded From : www.EasyEngineering.net

3 Analytic Functions

3.1 Introduction

ww Any number of the form x + iy where i =



−1 and x, y ∈ R is called complex

w.E
number. x is called the real part and y is called the imaginary part of the complex
number.

asy
If z = x + iy is any complex number, then Re(z) = x, Im(z) = y.
Results

En
1. If z1 = x1 + iy1 and z2 = x2 + iy2 are two complex numbers, then z1 = z2 if and
only if x1 = x2 and y1 = y2 .
gin
2. z1 + z2 = x1 + x2 + i(y1 + y2 ).

3. z1 − z2 = x1 − x2 + i(y1 − y2 ). ee rin
4. z1 z2 = x1 x2 − y1 y2 + i(x1 y2 + x2 y1 ).
z1 x1 x2 + y1 y2 x2 y1 − x1 y2 g.n
et
5. = 2 2
+i 2 .
z2 x2 + y2 x2 + y22
6. Comparison of complex numbers such as z1 < z2 or z1 > z2 is not possible.

7. Always C denotes the set of all complex numbers.

Conjugate of a Complex number. If z = x + iy is any complex number then,


x − iy is defined to be the complex conjugate of z. It is denoted by z̄. Hence,
z̄ = x − iy.

Downloaded From : www.EasyEngineering.net


Downloaded From : www.EasyEngineering.net

288 Engineering Mathematics - II

Results

1. z = z̄ if and only if z is real.

2. z̄¯ = z.
z + z̄
3. = Re(z).
2
z − z̄
4. = Im(z).
2i

ww 5. z1 + z2 = z¯1 + z¯2 .

6. z1 − z2 = z¯1 − z¯2 .

w.E 7. z1 z2 = z¯1 .z¯2 .

asy
!
z1 z¯1
8. = .
z2 z¯2

En
Geometrical representation of a complex number

gin
If z = x + iy is a complex number, then z can be represented as a point P with
coordinates (x, y) in the Argand diagram. With this notation, we can represent
addition, subtraction,
ee
multiplication,
If z = x + iy, z1 = x1 + iy1 and z2 = x2 + iy2 then
division and conjugate as follows.

rin
1. z1 + z2 = (x1 + x2 , y1 + y2 ).

2. z1 − z2 = (x1 − x2 , y1 − y2 ). g.n
3. z1 z2 = (x1 x2 − y1 y2 , x1 y2 + x2 y1 ).
z1
!  
 x1 x2 + y1 y2 x2 y1 − x1 y2 
et
4. = 2 , .
z2 x2 + y22 x22 + y22
5. z̄ = (x, −y).

Modulus and amplitude form of a complex number


Let z = x + iy be a complex number.

Downloaded From : www.EasyEngineering.net


Downloaded From : www.EasyEngineering.net

Analytic Functions 289

In the argand plane let P(x, y) represents


z. Join OP. Let OP = r. Let OP makes Y

an angle θ with the positive direction P(x, y)

of the real axis. Then, r is called the


modulus and θ is called the amplitude y

r
(or argument) of z. The modulus of z
is denoted by |z| and amplitude of z is θ

ww denoted by arg(z).
O x N
X

w.E
i.e., |z| = r, arg(z) = θ.
Let PN be drawn perpendicular to the real axis.

asy
Then, ON = x, NP = y and OP = r.
From the right angled triangle OPN,

En
cos θ =
ON x
= =⇒ x = r cos θ. (1)

gin
OP r
NP y
sin θ = = =⇒ y = r sin θ. (2)

ee
OP r
Now, z = x + iy = r cos θ + i(r sin θ).
z = r(cos θ + i sin θ).
rin
z.
This expression represents the modulus amplitude form of the complex number

g.n
Squaring and adding (1) and (2) we obtain
q
x2 + y2 = r2 =⇒ r = x2 + y2
et
q
=⇒ |z| = x2 + y2 . (3)
(2) y y
=⇒ tan θ = =⇒ θ = tan−1 . (4)
(1) x x
(3) and (4) gives a formula to find the modulus and argument of a given complex
number z.

Downloaded From : www.EasyEngineering.net


Downloaded From : www.EasyEngineering.net

290 Engineering Mathematics - II

For finding the modulus of z, we can always use (3). But (4) is true only when
x and y are positive. (i.e.,) if z lies in the first quadrant. If z lies in other quadrants
we cannot use (4) to find arg(z). One can adopt the following procedure to find the
correct value of arg(z).

Step (i). Without considering the sign of the real and imaginary parts, find
y
θ = tan−1 .
x
Step (ii). Depending on the quadrant in which z lies, evaluate arg(z) in the

ww following way.

w.E
asy
En
gin
ee
Principal value. The value of the argument that lies between π and −π is called
rin
the principal value of the argument.
The following results can be easily proved with respect to the modulus and
g.n
argument of complex numbers.
Let z1 and z2 be two complex numbers. Then et
1. |z1 z2 | = |z1 |.|z2 | and arg(z1 z2 ) = arg(z1 ) + arg(z2 ).
!
z1 |z1 | z1
2. =
if z2 , 0 and arg = arg(z1 ) − arg(z2 ).
z2 |z2 | z2
3. |z̄| = |z|.

4. zz̄ = |z|2 .

Downloaded From : www.EasyEngineering.net


Downloaded From : www.EasyEngineering.net

Analytic Functions 291

5. |z| ≥ |Re(z)|.

6. |z| ≥ |Im(z)|.

Triangle inequality. The following two relations are true and called triangle
inequalities

1. |z1 + z2 | ≤ |z1 | + |z2 |.

2. |z1 − z2 | ≥ |z1 | − |z2 |.

ww Result. Using the Euler’s formula eiθ = cos θ + i sin θ, the modulus - amplitude
form of z can be expressed as z = reiθ .

w.E
Extended complex number system
Let C be the set of all complex numbers. The extended complex number

asy
system is C together with ∞ which satisfies the following properties.
(i) If z ∈ C, then z + ∞ = ∞, z − ∞ = −∞ and
z
= 0.

En

z
(ii) If z is a nonzero complex number, then z.∞ = ∞, = ∞.
0
(iii) ∞ + ∞ = ∞, ∞.∞ = ∞.
(iv)

z
= ∞ if z ∈ C. gin
ee
The extended complex number system C ∪ {∞} when represented in the plane

rin
geometrically is called the extended complex plane and ∞ is known as the point
at infinity.

g.n
Function of a complex variable. Let A be a set of complex numbers. Let f be a
function from A to C be a rule that assigns to each z ∈ A, a unique complex
et
number w ∈ C. w is the value of f at z denoted by w = f (z). Then, w is called a
function of the complex variable z. A is called the domain of f and C is called the
codomain of f . The collection of all values of f is called the range of f . Since
z = x + iy is a complex variable, the complex function w = f (z) can be considered as
having real and imaginary parts u and v respectively such that u and v are
functions of x and y. Hence, we have w = f (z) = u(x, y) + iv(x, y).

Downloaded From : www.EasyEngineering.net


Downloaded From : www.EasyEngineering.net

292 Engineering Mathematics - II

Examples

1. f (z) = z2 is a complex function. ie., f (z) = (x + iy)2 = x2 − y2 + 2ixy.


=⇒ u(x, y) = x2 − y2 and v(x, y) = 2xy.

2. ez is a complex function.
ez = e x+iy = e x .eiy = e x (cos y + i sin y) =⇒ u(x, y) = e x cos y, v(x, y) = e x sin y.

ww 3. sin z is a complex function.


sin z = sin(x + iy) = sin x cos(iy) + cos x sin(iy) = sin x cosh y + i cos x sinh y.

w.E u(x, y) = sin x cosh y, v = cos x sinh y.

Neighbourhood of a point

asy
Let z0 be a point in the complex plane. The neighbourhood of z0 is the set of

En
all points in the complex plane inside the circle with centre z0 .
i.e. the interior of the circle |z − z0 | = r.
The interior is an open circular disc.
gin
ee
A δ neighbourhood of z0 is the open circular disc |z − z0 | < δ.
A set is open if it contains none of its boundary points. eg. |z| < 1.

rin
Limit of a function. Let f be a function defined in some neighbourhood of z0
g.n
except possibly at z0 . We say a complex number w0 is the limit of f (z) as z tends to

et
z0 if for any ǫ > 0, there exists a δ > 0 such that | f (z) − f (z0 )| < ǫ for |z − z0 | < δ.
Symbolically it can be written as lim f (z) = w0 = f (z0 ).
z→z0

Result. State the basic difference between the limit of a real variable and that
of a complex variable. [Jun 2012]
Solution. In the case of the real variable, x approaches x0 only along the line,
whereas in complex variable, z approaches z0 in any direction.

Downloaded From : www.EasyEngineering.net


Downloaded From : www.EasyEngineering.net

Analytic Functions 293

Note

1. When the limit exists it is unique in whatever direction z approaches z0 .

2. If f (z) = u(x, y) + iv(x, y) is defined in a neighbourhoood of z0 =


x0 + iy0 , then lim f (z) = u0 + iv0 if and only if lim u(x, y) = u0 and
z→z0 (x,y)→(x0 ,y0 )
lim v(x, y) = v0 .
(x,y)→(x0 ,y0 )

1
3. lim f (z) = w if lim f ( ) = w.
z

ww
z→∞ z→0
1
lim f (z) = ∞ if lim = 0.
z→z0 z→z0 f (z)
1

w.E lim f (z) = ∞ if lim 1 = 0.


z→∞ z→0 f ( )
z

asy
Continuity of a function. Let f be a function defined in a neighbourhood of z0
(including z0 ). f is continuous at the point z0 if lim f (z) = f (z0 ).
z→z0

En
A function f is continuous in a region R of the complex plane if f is
continuous at each point of R.

gin
Derivative of f (z). Let f be a function defined in a neighbourhood of z0 . The
f (z) − f (z0 )
derivative of f at z0 is f ′ (z0 ) = lim

Note
z→z0

ee
z − z0
if the limit exists.
If f ′ (z0 ) exists, we say that the function is differentiable at z0 .

rin
(1) A function f is differentiable at z0 , then it is continuous at z0 . But the
converse is not true. g.n
i.e. A function f is continuous at z0 then it need not be differentiable.

Example. Let z = x + iy.


et
Consider f (z) = z̄ = x − iy, u = x, v = −y.
Consider the origin (0, 0).
f (0) = 0.
lim u(x, y) = lim x = 0.
(x,y)→(0,0) (x,y)→(0,0)
lim v(x, y) = lim (−y) = 0.
(x,y)→(0,0) (x,y)→(0,0)

Downloaded From : www.EasyEngineering.net


Downloaded From : www.EasyEngineering.net

294 Engineering Mathematics - II

lim f (z) = lim u(x, y) − i lim v(x, y) = 0 − i0 = 0 = f (0).


z→0 (x,y)→0 (x,y)→0
∴ f (z) is continuous at the point z = 0.
f (z) − f (z0 ) z̄ − 0 z̄
Now lim = lim = lim .
z→0 z−0 z→0 z − 0 z→0 z
Let us choose the path z → 0 along y = mx.

z̄ x − im x
lim = lim
z→0 z x + im x
x→0, y→0
1 − im
= lim
x→0 y→0 1 + im

ww =
1 − im
1 + im
,

w.E which is depending on m.


∴ The limit is not unique.

asy
∴ f (z) = z̄ is not differentiable at z = 0.
∴ f (z) continuous ; f (z) is differentiable.

En
Differentiation Formulae. If f (z) and g(z) are differentiable at z and c is a
constant then

gin
ee
d
1. (c f (z)) = c f ′ (z).
dz

2.
d
( f (z) ± g(z)) = f ′ (z) ± g′ (z). rin
dz

d g.n
3.

4.
dz
( f (z).g(z)) = f (z)g′ (z) + g(z) f ′ (z).

d  f (z)  g(z) f ′ (z) − f (z)g′ (z)


= .
et
dz g(z) (g(z))2

d
5. ( f (g(z))) = f ′ (g(z))g′ (z).
dz

d n
6. (z ) = nzn−1 .
dz

Downloaded From : www.EasyEngineering.net


Downloaded From : www.EasyEngineering.net

Analytic Functions 295

3.2 Analytic Function

A complex function f (z) is said to be analytic at a point z0 if f (z) is differentiable


at z0 and at every point of some neighbourhood of z0 .
A function is analytic in a domain D if it is analytic at each point of D.
Note
An analytic function is also known as a regular function or holomorphic
function.

ww 3.3 Necessary and sufficient conditions for f (z) to be analytic

w.E
Theorem

asy
The necessary and sufficient conditions for the function f (z) = u + iv to be analytic
in a domain D are
(i)
∂u ∂u ∂v ∂v
, , ,
∂x ∂y ∂x ∂y
En
are continuous functions of x and y in the domain D.

(ii)
∂u ∂v
=
∂x ∂y
and
∂u
∂y
=− .
i.e. u x = vy and uy = −v x .
∂v
∂x
gin
C-R equations.
ee
The conditions u x = vy and uy = −v x are called Cauchy - Riemann equations (or)

rin
Proof. Necessary condition

g.n
Let w = f (z) = u + iv be analytic. Then, by the definition of analytic function
dw
dz
= f ′ (z) exists and it is unique at every point.
Let ∆x and ∆y be the small incremental changes in x and y respectively. Then
the corresponding changes in u, v and z will be ∆u, ∆v and ∆z respectively.
et
f (z + ∆z) − f (z)
∴ f ′ (z) = lim
∆z→0 ∆z
u + ∆u + i(v + ∆v) − (u + iv)
= lim
∆z→0 ∆z
u + ∆u + iv + i∆v − u − iv
= lim
∆z→0 ∆z

Downloaded From : www.EasyEngineering.net


Downloaded From : www.EasyEngineering.net

296 Engineering Mathematics - II

∆u + i∆v
= lim
∆z→0 ∆z
!
∆u ∆v
= lim +i .
∆z→0 ∆z ∆z

Consider the path, a line parallel to the x-axis along which ∆z → 0. Then
y = constant and hence ∆y = 0.

∴ ∆z = ∆x + i∆y = ∆x.
!
′ ∆u ∆v
∴ f (z) = lim +i

ww ∆x→0 ∆x

= lim
∆u
∆x→0 ∆x
∆x
+ i lim
∆v
∆x→0 ∆x

w.E =
∂u
∂x
∂v
+i .
∂x
(1)

asy
Now, consider the path, a line parallel to the y- axis along which ∆z → 0. Then,
x = constant and hence ∆x = 0.

En ∴ ∆z = ∆x + i∆y = i∆y.

gin ′
Now, f (z) = lim
∆u
∆y→0 i∆y
+i
∆v
i∆y
!

ee 1
= lim
∂u ∂v
= −i + .
∆u
+ lim
i ∆y→0 ∆y ∆y→0 ∆y
∆v

rin (2)

From (1) and (2) we obtain


∂y ∂y
g.n
∂u ∂v
=
∂x ∂y
and
∂v
∂x
=−
∂u
∂y
et
∂u ∂v ∂u ∂v
i.e., = and =−
∂x ∂y ∂y ∂x
Sufficient conditions
Let us assume that f (z) = u + iv which satisfies
∂u ∂u ∂v ∂v
1. , , and are continuous.
∂x ∂y ∂x ∂y

Downloaded From : www.EasyEngineering.net


Downloaded From : www.EasyEngineering.net

Analytic Functions 297

∂u ∂v ∂u ∂v
2. = and =− .
∂x ∂y ∂y ∂x
We have to prove that f (z) is analytic.
We have, f (z + ∆z) = u + ∆u + i(v + ∆v).
By Taylor’s theorem we have
! !
∂u ∂u ∂v ∂v
f (z + ∆z) = u + iv + ∆x + ∆y + i ∆x + ∆y + · · ·
∂x ∂y ∂x ∂y
Since ∆x and ∆y are very small, omitting higher powers of ∆x and ∆y, we obtain
! !
∂u ∂u ∂v ∂v

ww
f (z + ∆z) = u + iv + ∆x + ∆y + i ∆x + ∆y
∂x ∂y ∂x ∂y
∂u ∂u ∂v ∂v
= f (z) + ∆x + ∆y + i ∆x + i ∆y

w.E
∂x ∂y ∂x ∂y
! !
∂u ∂v ∂u ∂v
= f (z) + +i ∆x + +i ∆y
∂x ∂x ∂y ∂y

asy
! !
∂u ∂v ∂u ∂v
f (z + ∆z) − f (z) = +i ∆x + +i ∆y.
∂x ∂x ∂y ∂y

En
Using C-R equations, we replace
∂u
∂y
by −
∂v
∂x
and
∂v
∂y
by
∂u
∂x
we obtain

gin
! !
∂u ∂v ∂v ∂u
f (z + ∆z) − f (z) = +i ∆x + − + i ∆y
∂x ∂x ∂x ∂x

ee
! !
∂u ∂v ∂u 1 ∂v
= +i ∆x + i − ∆y
∂x ∂x ∂x i ∂x

=
∂u
+i
∂v
!
∆x + i
∂u
+i
∂v
!
rin
∆y

=
∂x
∂u
+i
∂x
∂v
!
∂x

(∆x + i∆y)
∂x

g.n
=

f (z + ∆z) − f (z) ∂u
∂x
∂u
∂x
+i
∂x
∂v
∂x
∂v
!
∆z et
= +i .
∆z ∂x ∂x
Taking limit as ∆z → 0 we get
f (z + ∆z) − f (z) ∂u ∂v
lim = +i
∆z→0 ∆z ∂x ∂x
∂u ∂v
f ′ (z) = +i .
∂x ∂x

Downloaded From : www.EasyEngineering.net


Downloaded From : www.EasyEngineering.net

298 Engineering Mathematics - II

∴ f ′ (x) exists and it is unique.


Hence, f (z) is analytic.
C-R equations in polar form
We know that x = r cos θ, y = r sin θ

z = x + iy = r(cos θ + i sin θ) = reiθ

f (z) = u + iv = f (reiθ ) (1)

Differentiating (1) partially w.r.t. r we get

ww ∂u
∂r
∂v
+ i = f ′ (reiθ )eiθ .
∂r
(2)

w.E
Differentiating (1) partially w.r.t. θ we get

asy
∂u ∂v
+ i = f ′ (reiθ )rieiθ
∂θ ∂θ
1 ∂u 1 ∂v

En +
ir ∂θ r ∂θ
= f ′ (reiθ )eiθ

gin
−i ∂u 1 ∂v ∂u ∂v
+ = +i .
r ∂θ r ∂θ ∂r ∂r

ee
Equating the real and imaginary parts we get
∂u 1 ∂v ∂v −1 ∂u
∂r
=
r ∂θ
&
∂r
=
r ∂θ
rin
∂u 1 ∂v
∂r
=
r ∂θ
&
∂u
∂θ
∂v
= −r ,
∂r
g.n
which are the C-R equations in polar form.
Entire function et
A complex function f is said to be an entire function if it is analytic in the
entire complex plane.
Example
A polynomial function f (z) = a0 + a1 z + a2 z2 + · · · + an zn , an , 0 is an entire
function, since it is differentiable everywhere in the plane.

Downloaded From : www.EasyEngineering.net


Downloaded From : www.EasyEngineering.net

Analytic Functions 299

✎ ☞
Worked Examples
✍ ✌

Example 3.1. Show that the function f (z) = z̄ is nowhere differentiable.


[Dec 2012, Dec 2010, Jan 2001]
Solution. Let f (z) = u + iv = z̄ = x − iy.
∴ u = x, v = −y

u x = 1, vx = 0

uy = 0, vy = −1.

ww Since u x , vy , C.R equations are not satisfied.


∴ f (z) is not analytic anywhere.

w.E =⇒ f (z) is nowhere differentiable.

Example 3.2. Prove that f (z) = sin z is analytic. [Jun 2004]


Solution. f (z) = sin z.
asy
u + iv = sin(x + iy)

En
= sin x cos iy + cos x sin iy

gin
= sin x cosh y + i cos x sinh y

ee
∴ u = sin x cosh y, v = cos x sinh y

rin
u x = cos x cosh y, v x = − sin x sinh y

uy = sin x sinh y, vy = cos x cosh y.

g.n
Since u x = vy and uy = −v x , C-R equations are satisfied for all x and y.
∴ f (z) is analytic.

Example 3.3. Test whether the function f (z) = z2 is analytic or not.


Solution.
et[May 2007]

f (z) = u + iv = z2 = (x + iy)2 = x2 − y2 + 2ixy.


∴ u = x2 − y2 , v = 2xy.
u x = 2x, v x = 2y.
uy = −2y, vy = 2x.
The above equations imply u x = vy and uy = −v x .

Downloaded From : www.EasyEngineering.net


Downloaded From : www.EasyEngineering.net

300 Engineering Mathematics - II

⇒ C-R equations are satisfied for all x and y and the partial derivatives
being polynomials in x and y are continuous everywhere in the complex
plane.
∴ f (z) is analytic in the entire plane. f (z) is an entire function.

Example 3.4. Test whether the function f (z) = 2xy + i(x2 − y2 ) is analytic or not.
[Apr 2002]
Solution. f (z) = u + iv = 2xy + i(x2 − y2 ).
v = x2 − y2 .

ww
u = 2xy
u x = 2y v x = 2x.

w.E uy = 2x
u x , vy
vy = −2y.
uy , −v x .
∴ C-R equations are not satisfied.

asy
Therefore, f (z) is not analytic at any point.

En
Example 3.5. Test whether the function f (z) = |z|2 is analytic or not.

Solution. f (z) = u + iv = |z|2


gin
= x2 + y2 .
[May 2015, Jan 2006]

∴ u = x2 + y2
u x = 2x
uy = 2y
v = 0.
v x = 0.
vy = 0.
ee rin
At (0, 0) u x = vy = 0 and uy = −v x = 0.
g.n
⇒ f (z) is differentiable only at the origin.
But for all (x, y) , (0, 0), C-R equations are not satisfied.
Therefore, f (z) is not analytic.
et
Example 3.6. Check for analyticity of log z.
Solution. Let f (z) = log z = log reiθ = log r + log eiθ = log r + iθ.

u = log r, v = θ.
1
ur = , vr = 0.
r

Downloaded From : www.EasyEngineering.net


Downloaded From : www.EasyEngineering.net

Analytic Functions 301

uθ = 0, vθ = 1.
∂u 1 ∂v ∂u ∂v
= , = −r .
∂r r ∂θ ∂θ ∂r
∴ C-R equations are satisfied and partial derivatives are continuous for r , 0.
∴ f (z) is analytic except for r = 0 or z = 0.
∴ log z is analytic for all z , 0.

Example 3.7. Show that f (z) = zn is analytic.


Solution. Changing into polar coordinates we get,

ww
 n
f (z) = u + iv = reiθ = rn einθ

= rn (cos nθ + i sin nθ)

w.E ∴ u = rn cos nθ
∂u
v = rn sin nθ.
∂v
∂r
∂u asy
= nrn−1 cos nθ

= −nrn sin nθ
∂r
∂v
= nrn−1 sin nθ

= nrn cos nθ.


∂θ

En ∂θ
From the above equations we observe that,

gin
(i) the first partial derivatives of u and v with respect to r and θ exist,
(ii) all the partial derivatives are continuous and
(iii) r


∂u
∂r
∂u 1 ∂v
∂r
=
r ∂θ
.
∂v
= nrn cos nθ = .
∂θ ee rin
∂v
−r = −nrn sin nθ =
∂r
∂u
∂θ
.
g.n
Hence, C-R equations are satisfied.
∴ f (z) = zn is analytic.

Example 3.8. Prove that f (z) = ez is analytic and find its derivative.
et[Dec 2004]
Solution. f (z) = u + iv = ez = e x+iy

= e x .eiy

= e x (cos y + i sin y)

= e x cos y + ie x sin y.

Downloaded From : www.EasyEngineering.net


Downloaded From : www.EasyEngineering.net

302 Engineering Mathematics - II

∴ u = e x cos y, v = e x sin y.

u x = e x cos y, v x = e x sin y.

uy = −e x sin y, vy = e x cos y.

Hence, u x = vy and uy = −v x .
∴ C-R equations are satisfied.
∴ f (z) is analytic.
∂u ∂v

ww Also f ′ (z) =
∂x
+i
∂x
= e x cos y + ie x sin y

w.E = e x (cos y + i sin y)

= e x eiy

asy
= ez .

Example 3.9. Show that f (z) =


1
z−1 En is analytic at z = 1 + i.

gin
Solution. We have to show that u x , uy , v x , vy are continuous in some neighbourhood
of z = (1, 1) and the C-R equations are satisfied in this neighbourhood.

f (z) =
1
=
x − 1 − iy
1
ee =
z − 1 x + iy − 1 x − 1 + iy
x−1
1

y rin
u + iv =

∴u=
=
(x − 1)2 + y2 (x − 1)2 + y2
x−1
,v =
−y

.
i
(x − 1)2 + y2
.

g.n
(x − 1)2 + y2 (x − 1)2 + y2
Since u(x, y) and v(x, y) are rational functions of the real variables x and
y and are defined at (1, 1) and in a neighbourhood of (1, 1) u x , uy , v x , vy
et
exist and are continuous.
(x − 1)2 + y2 − (x − 1)2(x − 1) y2 − (x − 1)2
ux = =
((x − 1)2 + y2 )2 ((x − 1)2 + y2 )2

{(x − 1)2 + y2 }0 − (x − 1)2y −2y(x − 1)


uy = 2 2 2
=
((x − 1) + y ) ((x − 1)2 + y2 )2

Downloaded From : www.EasyEngineering.net


Downloaded From : www.EasyEngineering.net

Analytic Functions 303

2y(x − 1)
v x = −y(−1)((x − 1)2 + y2 )−2 2(x − 1) =
((x − 1)2 + y2 )2
((x − 1)2 + y2 )(−1) + y2y y2 − (x − 1)2
vy = =
((x − 1)2 + y2 )2 ((x − 1)2 + y2 )2
∴ u x = vy and uy = −v x .
∴ C-R equations are satisfied.
∴ f (z) is analytic.

Example 3.10. Verify whether w = x2 − y2 − 2xy + i(x2 − y2 + 2xy) is an analytic

ww function of z = x + iy.
Solution. w = u + iv = x2 − y2 − 2xy + i(x2 − y2 + 2xy).

w.E u = x2 − y2 − 2xy,
u x = 2x − 2y,
v = x2 − y2 + 2xy.
v x = 2y + 2x.
uy = −2y − 2x,
u x = vy ,
asy vy = −2y + 2x.
uy = −(2y + 2x) = −v x .

En
∴ C-R equations are satisfied.
∴ w is analytic.

gin
Example 3.11. Find the analytic region of f (z) = (x − y)2 + 2i(x + y).

u = (x − y)2 ,
ee
Solution. f (z) = u + iv = (x − y)2 + 2i(x + y)

v = 2(x + y).
rin
u x = 2(x − y), v x = 2.
g.n
et
uy = −2(x − y), vy = 2.

Since u and v are polynomials, their partial derivatives are continuous


everywhere. For analyticity it should satisfy C-R equations.
i.e., u x = vy and uy = −v x .
=⇒ 2(x − y) = 2 and −2(x − y) = −2.
i.e., x − y = 1 and x − y = 1.
∴ For points on x − y = 1, C-R equations are satisfied.
∴ f (z) is analytic for points on x − y = 1.

Downloaded From : www.EasyEngineering.net


Downloaded From : www.EasyEngineering.net

304 Engineering Mathematics - II

Example 3.12. If u + iv is analytic, show that v − iu and −v + iu are also analytic.


[Apr 2005]
Solution. Given f (z) = u + iv is analytic in a domain D.
∴ u and v have continuous partial derivatives and satisfy C-R equations.

i.e., u x = vy (1)

and uy = −v x . (2)

ww (i) Let h + ig = v − iu.


To prove h x = gy , hy = −g x .

w.E Now, h = v
∴ hx = vx
g = −u
g x = −u x

asy
and hy = vy gy = −uy .
Now, h x = v x = −uy = gy

En
and hy = vy = u x = −g x .
∴ h + ig = v − iu satisfies C-R equations and first order partial derivatives
are continuous.
gin
∴ v − iu is analytic in D.
(ii) Let p + iq = −v + iu
=⇒ p = −v, q = u.
ee rin
To prove p x = qy ,
Now, p x = −v x ,
py = −q x .
qx = ux . g.n
and py = −vy , qy = uy .
Now, p x = −v x = uy = qy .
et
py = −vy = −u x = −q x .
∴ p + iq = −v + iu satisfy C-R equations.
∴ −v + iu is analytic in D.

Example 3.13. If f (z) and f (z) are analytic functions, prove that f (z) is a constant.
Solution. Let f (z) = u + iv. Then, f (z) = u − iv.

Downloaded From : www.EasyEngineering.net


Downloaded From : www.EasyEngineering.net

Analytic Functions 305

Given, f (z) is analytic.


∴ It satisfies C-R equations.

∴ u x = vy and uy = −v x . (1)

Given that f (z) is also analytic

∴ u x = −vy and uy = v x . (2)

From (1) and (2) we obtain 2u x = 0 and 2uy = 0

ww =⇒ u x = 0 and uy = 0.
∴ u is a constant ⇒ u = c1 .

w.E Also v x = 0 and vy = 0.


∴ v is a constant ⇒ v = c2 .

asy
∴ f (z) = u + iv = c1 + ic2 = a constant.

En
Example 3.14. Prove that an analytic function with constant modulus is constant.
[Dec 2007]

gin
Solution. Let f (z) = u + iv be the analytic function.
Given, | f (z)| = constant.

⇒ u2 + v2 = c
ee
i.e., |u + iv| = c, c is a constant.

rin (1)

Differentiating w.r. to x and y we get g.n


2uu x + 2vv x = 0

2uuy + 2vvy = 0
et
i.e., uu x + vv x = 0 (2)

and uuy + vvy = 0 (3)

But f (z) is analytic.


∴ It satisfies C-R equations.

Downloaded From : www.EasyEngineering.net


Downloaded From : www.EasyEngineering.net

306 Engineering Mathematics - II

i.e., u x = vy and uy = −v x .
Applying C-R equations in (3) we obtain

−uv x + vu x = 0

i.e., vu x − uv x = 0. (4)

We can solve for u x and v x using Cramer’s rule for the equations (2) and (4).

u v
= −u2 − v2 = −(u2 + v2 ) = −c , 0.

ww
∆ =
v −u

w.E
Since (2) and (4) are homogeneous equations and since ∆ , 0, we get only
trivial solutions.
i.e., u x = 0 and v x = 0.
=⇒ uy = 0 and vy = 0.
asy
En
u x = 0, uy = 0 ⇒ u = c1 a constant.
v x = 0, vy = 0 ⇒ v = c2 a constant.

gin
∴ f (z) = u + iv = c1 + ic2 is a constant.
If ∆ = 0, then u2 + v2 = 0 ⇒ u = 0 and v = 0.

ee
⇒ f (z) = 0 a constant. Therefore f (z) is always a constant.

rin
Example 3.15. Show that an analytic function with constant imaginary part is
constant.
g.n [Dec 2011]

⇒ v x = 0, vy = 0.
Since f (z) is analytic, it satisfy C-R equations.
et
Solution. Let f (z) = u + iv be the given analytic function with v = c, a constant.

⇒ u x = vy and uy = −v x
⇒ u x = 0 and uy = 0.
⇒ u = k a constant.
∴ f (z) = u + iv = k + ic a constant.
⇒ f (z) = constant.

Downloaded From : www.EasyEngineering.net


Downloaded From : www.EasyEngineering.net

Analytic Functions 307

∂w ∂w ∂w
Example 3.16. If w = f (z) is analytic, prove that = = −i , where z = x + iy
∂z ∂x ∂y
∂2 w
and prove that = 0. [May 2011]
∂z∂z̄
Solution. Given, w = f (z) is analytic.
Let w = u + iv, then u, v satisfy C-R equations

i.e., u x = vy and uy = −v x .
dw
Now = f ′ (z) = u x + iv x
dz

ww =
∂u
∂x
∂w
+i
∂v
=
∂x ∂x

(u + iv)

w.E Also
dw
=
∂x
.

= f ′ (z) = u x + iv x

asy
dz
= vy − iuy = −i(uy + ivy )
 ∂u ∂v 
= −i
∂y
+i
∂y
En .

gin
∂ ∂w
= −i (u + iv) = −i
∂y ∂y
dw ∂w ∂w

dz
=
∂x
= −i .
∂y
ee
Since z = x + iy, z̄ = x − iy, z + z̄ = 2x, x =

and z − z̄ = 2iy, y =
z − z̄
.
z + z̄
2
.

rin
Now,
∂x 1 ∂y −1
= ,
∂z 2 ∂z̄
=
2i

2i
. g.n
u(x, y) and v(x, y) can be considered as functions of z and z̄
∂w ∂u ∂x ∂u ∂y  ∂v ∂x ∂v ∂y 
∂z̄
= +
∂x ∂z̄ ∂y ∂z̄
+i +
∂x ∂z̄ ∂y ∂z̄
et
!  !
∂u 1 ∂u −1 ∂v 1 ∂v (−1) 
= + +i +
∂x 2 ∂y 2i ∂x 2 ∂y 2i
" #
1  ∂u ∂v  i  ∂v ∂u  ∂u ∂v ∂v ∂u
= − + + ∵ = , =−
2 ∂x ∂y 2 ∂x ∂y ∂x ∂y ∂x ∂y
∂w 1 i 2
∂ w
= 0 + 0 = 0. =⇒ = 0.
∂z̄ 2 2 ∂z∂z̄

Downloaded From : www.EasyEngineering.net


Downloaded From : www.EasyEngineering.net

308 Engineering Mathematics - II

p
Example 3.17. Show that the function defined by f (z) = |xy| is not analytic at
the origin although C-R equations are satisfied. [Apr 2005]
p
Solution. Given f (z) = |xy|
p
u + iv =|xy|
p
u = |xy| v=0
∂u u(x, 0) − u(0, 0)
= lim
∂x x→0 x
0−0

ww ∂u
= lim
x→0

= lim
x
= 0.
u(0, y) − u(0, 0)
= lim
0−0
= 0.

w.E ∂v
Similarly,
∂y x→0

= 0,
∂v
= 0.
y x→0 x

asy
∂x ∂y
⇒ C-R equations are satisfied at (0, 0).
Now

f ′ (0) = lim Enf (z) − f (0)


p
|xy| − 0
.
z→0

gin z

ee
= lim
z→0 x + iy

If z → 0 along the line y = mx then x → 0, y → 0.


√ √ rin
g.n
p
′ |mx2 | |m|x |m|
Now, f (0) = lim = lim = .
x→0 x + imx x→0 (1 + im)x (1 + im)

Since the limit depends on m, for different paths we have different limits.
∴ The limit is not unique.
et
∴ f ′ (0) does not exist.
∴ f (z) is not analytic at z = 0 eventhough C-R equations are satisfied at the origin.

Example 3.18. Prove that every analytic function w = u + iv can be expressed as a


function of z alone, not as a function of z̄. [Jun 2012, Jun 2010]
Solution. Let z = x + iy.

Downloaded From : www.EasyEngineering.net


Downloaded From : www.EasyEngineering.net

Analytic Functions 309

z + z̄ z − z̄
Then, z̄ = x − iy, which implies x = and y = .
2 2i
Now, w = u + iv.

∂w ∂u ∂v
= +i
∂z̄ ∂z̄ ∂z̄
" #
∂u ∂x ∂u ∂y ∂v ∂x ∂v ∂y
= + +i +
∂x ∂z̄ ∂y ∂z̄ ∂x ∂z̄ ∂y ∂z̄
" #
∂u 1 ∂u −1 ∂v 1 ∂v −1
= + +i +
∂x 2 ∂y 2i ∂x 2 ∂y 2i
" # " #
1 ∂u ∂u i ∂v ∂v

ww =

=
2 ∂x
"
1 ∂u
+i
∂y
∂u
+i +i −
+
2 ∂x
∂v ∂v
+i
#
∂y

w.E =
2 ∂x
"
1 ∂u ∂v
2 ∂x ∂y

∂y

+i
∂x ∂y
∂u ∂v
+
∂y ∂x
!#

1
asy
= [0 + i.0][∴ u x = vy , uy = −v x ]
2
∂w
∂z̄
= 0.
En
⇒ w is independent of z̄.
gin
Hence, w can be written only as a function of z and not as a function of z̄.

Example 3.19. If u = x2 − y2 , v = −
function.
ee y
x2 + y2
rin
then prove that u + iv is not an analytic
[May 2005]
Solution. u = x2 − y2 , v=− 2
y
x + y2
.
2xy g.n
u x = 2x,

uy = −2y, vy = −
v x = −y(−1)(x2 + y2 )−2 .2x =
x2 + y2 − y2y
(x2 + y2 )2
= −
(x2 + y2 )2
x2 − y2
(x2 + y2 )2
.
.
et
We observe that u x , vy and uy , −v x .
∴ C-R equations are not satisfied.
Hence, u + iv is not analytic.

Example 3.20. Find the constants a, b if f (z) = x + 2ay + i(3x + by) is analytic.
[Dec 2013]

Downloaded From : www.EasyEngineering.net


Downloaded From : www.EasyEngineering.net

310 Engineering Mathematics - II

Solution. f (z) = x + 2ay + i(3x + by).

u = x + 2ay, v = 3x + by.

ux = 1 vx = 3

uy = 2a vy = b.

Since f (z) is analytic, C-R equations are satisfied.


∴ u x = vy and uy = −v x .
Taking u x = vy we get b = 1.

ww 3
Taking uy = −v x , we get 2a = −3 ⇒ a = − .
2
Example 3.21. Find the values of a and b such that the function

w.E
f (z) = x2 + ay2 − 2xy + i(bx2 − y2 + 2xy) is analytic. Also find f ′ (z).
Solution. f (z) = x2 + ay2 − 2xy + i(bx2 − y2 + 2xy).

asy
∴ u = x2 + ay2 − 2xy, v = bx2 − y2 + 2xy.

u x = 2x − 2y,
En v x = 2bx + 2y.

uy = 2ay − 2x,
gin
vy = −2y + 2x.

Since f (z) is analytic, C-R equations are satisfied.


∴ u x = vy and uy = −v x .
ee rin
Taking u x = vy we get 2x − 2y = 2x − 2y which is true for all x and y.
Taking uy = −v x we obtain, 2ay − 2x = −2bx − 2y.
Comparing the coefficients we get 2a = −2 and 2b = 2. g.n
=⇒ a = −1 and b = 1.

Now, f ′ (z) =
∂u
∂x
+i
∂v
∂x
et
= 2x − 2y + i(2x + 2y)

= 2x + 2ix + 2y(i − 1)
1
= 2[x(1 + i) + iy(1 − )]
i
= 2[x(1 + i) + iy(1 + i)] = 2(1 + i)(x + iy) = 2(1 + i)z.

Downloaded From : www.EasyEngineering.net


Downloaded From : www.EasyEngineering.net

Analytic Functions 311

Example 3.22. Find the constants a, b, c if f (z) = x + ay + i(bx + cy) is analytic.


[Jun 2010]
Solution. f (z) = u + iv = x + ay + i(bx + cy).
∴ u = x + ay v = bx + cy.
ux = 1 vx = b
uy = a vy = c.
Since f (z) is analytic, u x = vy and uy = −v x .
⇒ 1 = c and a = −b. If b = k then a = −k.

ww ∴ For a = −k, b = k and c = 1, the given function is analytic.

w.E
Exercise 3 A

asy
1. Show that f (z) = xy + iy is continuous every where but not differentiable any
where.

En
2. Prove that f (z) = sinh z is analytic. [Dec 2003]

gin
3. Prove that f (z) = z3 is analytic. [May 2001]

4. Prove that z3 + z is analytic. ee rin [Jan 2006]

5. Test the analyticity of f (z) = 2xy + i(x2 + y2 ).


1
6. Test the analyticity of the function f (z) = . g.n [Dec 2007]

[Jan 2004]
z
7. Prove that cos z and cosh z are analytic functions.
x − iy
et
8. Verify whether is an analytic function. [Jan 2005]
x2 + y2
z
9. Show that the function is analytic except at z = 1.
z−1
10. Verify whether w = (x2 − y2 − 2xy) + i(x2 − y2 + 2xy) is an analytic function of
z = x + iy.

Downloaded From : www.EasyEngineering.net


Downloaded From : www.EasyEngineering.net

312 Engineering Mathematics - II

 3
x (1+i)−y3 (1−i)


 x2 +y2
, z,0
11. If f (z) =  prove that f (z) is continuous and the C-R


 0,
 z=0
equations are satisfied at z = 0, yet f ′ (0) does not exist.
1 px
12. Determine p such that the function f (z) = log(x2 + y2 ) + i tan−1 is an
2 y
analytic function.

13. Show that f (z) = e−y (cos x + i sin x) is differentiable everywhere in the finite
plane and f ′ (z) = f (z).

ww 14. Show that ey (cos x + i sin x) is nowhere differentiable.

w.E
15. Find a such that the function f (z) = r2 cos 2θ + ir2 sin aθ is analytic.

asy
16. Find the constants a, b, c if f (z) = x + ay + i(bx + cy) is analytic.

En
3.4 Harmonic functions and properties of analytic functions

gin
Definition. A real function φ of two variables x and y is said to be harmonic in

∂2 φ ∂2 φ
Laplace equation 2 + 2 = 0.
∂x ∂y
Properties of Analytic functions
ee
a domain D if it has continuous second order partial derivatives and satisfy the

rin
g.n
Property (1). If f (z) = u + iv is analytic in a domain D, then u and v are harmonic
in D.
Proof. Given f (z) is analytic.
⇒ u and v have first partial derivatives and satisfy C-R equations in D.
et
u x = vy and uy = −v x

∂u ∂v
i.e., = (1)
∂x ∂y
∂u ∂v
and =− (2)
∂y ∂x

Downloaded From : www.EasyEngineering.net


Downloaded From : www.EasyEngineering.net

Analytic Functions 313

Differentiating (1) w.r.t x we get


∂2 u ∂2 v
= (3)
∂x2 ∂x∂y
Differentiating (2) w.r.t y we get
∂2 u ∂2 v
= − (4)
∂y2 ∂y∂x
Since u x , uy , v x , vy are continuous, we have
∂2 v ∂2 v
=

ww ∂2 u ∂2 u
∴ (3) + (4) ⇒ + = 0.
∂x∂y ∂y∂x

w.E ∂x2 ∂y2


∴ u is harmonic.
Differentiating (1) w.r.t y and (2) w.r.t x we get

asy ∂2 u ∂2 v
= 2 and
∂y∂x ∂y
∂2 u
∂x∂y
∂2 v
=− 2
∂x

En ⇒
∂2 v
= −
∂2 v

gin ⇒
∂y2
∂2 v ∂2 v
+
∂x2

= 0.

⇒ v is harmonic.
ee ∂x2 ∂y2

rin
Property (2). If f (z) = u+iv is an analytic function, then the level curves u(x, y) = c1
and v(x, y) = c2 form an orthogonal system of curves.
Proof. Given: f (z) = u + iv is analytic. g.n
∴ u and v has continuous partial derivatives and satisfy C-R equations.

i.e., u x = vy and uy = −v x .
et (1)

Let u(x, y) = c′1 and v(x, y) = c′2 be two members of the given families intersecting at
P(x0 , y0 ).
Since u(x, y) = c′1 , we have du = 0.
∂u ∂u
⇒ dx + dy = 0.
∂x ∂y

Downloaded From : www.EasyEngineering.net


Downloaded From : www.EasyEngineering.net

314 Engineering Mathematics - II

∂u ∂u
dy = − dx
∂y ∂x
dy ux
=⇒ =− .
dx uy
dy ux
At P, = − = m1 , the slope of the tangent at (x0 , y0 ) to the curve u(x, y) = c′1 .
dx uy
Similarly, the slope of the tangent at (x0 , y0 ) to the curve v(x, y) = c′2 is
dy vx
= − = m2 .
dx vy
ux  vx 
Now, m1 m2 = −

ww

uy vy
ux  vx 
=
uy vy

w.E =
ux  vx 
vy uy
= 1(−1) = −1

∴ The curves cut orthogonally.


asy
⇒ The two system of curves are orthogonal.
Result
En
By Property (1), if f (z) = u + iv is analytic, then u and v are harmonic. But for

gin
any two harmonic functions u and v, u + iv need not be analytic.

u xx = 0, uyy = 0, v xx = 0, vyy = 0
u xx + uyy = 0 and v xx + vyy = 0.
ee
For example, consider u = x, v = −y then u x = 1, uy = 0, v x = 0, vy = −1,

rin
∴ u and v are harmonic functions.
But u x , vy . g.n
⇒ C-R equations are not satisfied.
⇒ u + iv is not analytic.
et
Definition. If two harmonic functions u and v satisfy the C-R equations in a
domain D, then they are the real and imaginary parts of an analytic function f in
D. Then v is said to be the conjugate harmonic function or harmonic conjugate
function in D.

Downloaded From : www.EasyEngineering.net


Downloaded From : www.EasyEngineering.net

Analytic Functions 315

Polar form of the Laplace equation.

Proof. We know that if f (z) = u + iv is analytic, then u and v satisfy C-R equations

∂u 1 ∂v
= (1)
∂r r ∂θ
∂v 1 ∂u
=− . (2)
∂r r ∂θ

Differentiating partially (1) w.r.t. r we get

∂2 u 1 ∂v 1 ∂2 v

ww ∂r2
= − + ·
r2 ∂θ r ∂r∂θ
∂u ∂v
(3)

w.E
(2) can also be written as
∂θ
= −r .
∂r
Differentiating this partially w.r.t. θ we get

∂2 u
∂θ2
= −r
asy
∂2 v
∂θ∂r
= −r
∂2 v
∂r∂θ
(4)

∂2 u 1 ∂u 1 ∂2 u
Now, 2 + En
1 ∂v 1 ∂2 v 1 1 ∂v
!
1 ∂2 v
!

gin
+ =− 2 + + + 2 −r
∂r r ∂r r2 ∂θ2 r ∂θ r ∂r∂θ r r ∂θ r ∂r∂θ
2
1 ∂v 1 ∂ v 1 ∂v 1 ∂ v 2
=− 2 + + 2 − = 0.

ee
r ∂θ r ∂r∂θ r ∂θ r ∂r∂θ

This is the Laplace equation in Polar Coordinates.


rin 

g.n
Result. We know that if f (z) = u + iv is analytic then u and v satisfy Laplace
equation.
∂2 u ∂2 u
i.e., +
∂x2 ∂y2
= 0 and
∂2 v ∂2 v
+
∂x2 ∂y2
= 0.
et
In polar form, the harmonic function u(r, θ) satisfy the Laplace equation
∂2 u 1 ∂u 1 ∂2 u
+ + = 0.
∂r2 r ∂r r2 ∂θ2
In the same way, the harmonic function v satisfy the Laplace equation
∂2 v 1 ∂v 1 ∂2 v
+ + = 0.
∂r2 r ∂r r2 ∂θ2

Downloaded From : www.EasyEngineering.net


Downloaded From : www.EasyEngineering.net

316 Engineering Mathematics - II

✎ ☞
Worked Examples
✍ ✌

Example 3.23. Show that u = 2x − x3 + 3xy2 is harmonic. [May 2011, Jun 2000]
Solution. u = 2x − x3 + 3xy2 .
u x = 2 − 3x2 + 3y2 .
u xx = −6x.
uy = 6xy.
uyy = 6x.

ww u xx + uyy = −6x + 6x = 0.
∴ u is harmonic.

w.E
Example 3.24. Show that x3 − 3xy2 + 3x2 − 3y2 + 1 is harmonic.
[Jun 2010, Jun 1996]

asy
Solution. Let u = x3 − 3xy2 + 3x2 − 3y2 + 1.
u x = 3x2 − 3y2 + 6x.
u xx = 6x + 6.
En
uy = −6xy − 6y.
uyy = −6x − 6. gin
∴ u is harmonic.
ee
u xx + uyy = 6x + 6 − 6x − 6 = 0.

rin
Example 3.25. Prove that v = x2 − y2 + 2xy − 3x − 2y is harmonic.
Solution. Let v = x2 − y2 + 2xy − 3x − 2y. g.n [Jun 2004]

v x = 2x + 2y − 3.
v xx = 2.
et
vy = −2y + 2x − 2.
vyy = −2.
v xx + vyy = 2 − 2 = 0.
∴ v is harmonic.

Example 3.26. Prove that 3x2 y − y3 is harmonic. [Jun 2007]

Downloaded From : www.EasyEngineering.net


Downloaded From : www.EasyEngineering.net

Analytic Functions 317

Solution. Let u = 3x2 y − y3 .


u x = 6xy.
u xx = 6y.
uy = 3x2 − 3y2 .
uyy = −6y.
u xx + uyy = 6y − 6y = 0.
∴ u is harmonic.

ww Example 3.27. Show that u = 2x(1 − y) is harmonic.


Solution. Let u = 2x(1 − y).
[Jun 2008]

w.E u x = 2(1 − y).


u xx = 0.
uy = −2x.
uyy = 0. asy
u xx + uyy = 0.
En
∴ u is harmonic.
gin
Solution. Let u = e x (cos y − sin y).
u x = e x (cos y − sin y).
ee
Example 3.28. Show that u = e x (cos y − sin y) is harmonic.

rin
[May 2003]

u xx = e x (cos y − sin y).


g.n
uy = e x (− sin y − cos y).
uyy = e x (− cos y + sin y) = −e x (cos y − sin y).
u xx + uyy = 0.
et
∴ u is harmonic.

Example 3.29. If u(x, y) and


! v(x, y) are
! harmonic functions in a region R, prove
∂u ∂v ∂u ∂v
that the function − +i + is an analytic function of z = x + iy.
∂y ∂x ∂x ∂y
[Dec 2011]

Downloaded From : www.EasyEngineering.net


Downloaded From : www.EasyEngineering.net

318 Engineering Mathematics - II

Solution. Given, u and v are harmonic.

∴ u xx + uyy = 0 and v xx + vyy = 0.

!
∂u ∂v ∂u ∂v
Let h + ig = − +i +
∂y ∂x ∂x ∂y
= uy − v x + i(u x + vy ).

∴ h = uy − v x and g = u x + vy .

ww Now, h x = u xy − v xx .

hy = uyy − vyx .

w.E g x = u xx + v xy .

gy = uyx + vyy .

asy
h x − gy = u xy − v xx − uyx − vyy

En
= −v xx − vyy [∵ u xy = uyx ]

= −(v xx + vyy ) = 0.

⇒ h x = gy .
gin
ee
Also, hy + g x = uyy − vyx + u xx + v xy

= u xx + uyy [∵ v xy = vyx ]
rin
= 0.

⇒ hy = −g x . g.n
∴ h and g satisfy C-R equations.
⇒ h + ig is analytic. !
et
∂u ∂v ∂u ∂v
⇒ − +i + is analytic.
∂y ∂x ∂x ∂y

Exercise 3 B
1
1. Prove that u = log(x2 + y2 ) is harmonic. [Dec 2006]
2

Downloaded From : www.EasyEngineering.net


Downloaded From : www.EasyEngineering.net

Analytic Functions 319

2. Prove that u = e x cos y is harmonic.

3. Prove that v(x, y) = e−x (2xy cos y + (y2 − x2 ) sin y) is a harmonic function.

4. Show that the function y + e x cos y is harmonic. [May 2001]


ay
5. Prove that for all real values of the function u = is harmonic.
x2 + y2

3.5 Construction of an analytic function

ww Milne-Thomson method. This method is useful to find the analytic function


when the real or imaginary part alone is given.

w.E Let f (z) = u(x, y) + iv(x, y) (1)

asy
We have z = x + iy. Then, z̄ = x − iy.

En z + z̄
Now, x =
2
and y =
z − z̄
2i
.

gin
Now we have, f (z) = u
 z + z̄ z − z̄ 
2
,
2i
+ iv
 z + z̄ z − z̄ 
2
,
2i
.

Putting z = z̄ we get
ee
f (z) = u(z, 0) + iv(z, 0).

rin
(2) is same as (1) if we replace x by z and y by 0. This is valid for any function of
(2)

the form f (x + iy).


g.n
(i) Suppose, the real part u(x, y) is given.
Let u(x, y) be the given real part of an analytic function f (z).
We have to find f (z) and v(x, y).
et
From u(x, y), find u x and uy .
Since f (z) is analytic, we have

f ′ (z) = u x + iv x = u x − iuy

= u x (x, y) − iuy (x, y).

Downloaded From : www.EasyEngineering.net


Downloaded From : www.EasyEngineering.net

320 Engineering Mathematics - II

By Milne-Thomson method

f ′ (z) = u x (z, 0) − iuy (z, 0).


Z
Now, f (z) = (u x (z, 0) − iuy (z, 0))dz + c,

where c is an arbitrary complex constant.


Then, separating the real and imaginary parts, we can find v(x, y).
(ii) Suppose, the imaginary part v(x, y) is given. Find v x and vy .

ww We have, f ′ (z) = u x + iv x = vy + iv x

w.E
By Milne-Thomson method
i.e, f ′ (z) = vy (x, y) + iv x (x, y).

asy f ′ (z) = vy (z, 0) + iv x (z, 0).

En
Now, f (z) =
Z

gin
(vy (z, 0) + iv x (z, 0))dz + c.

By equating the real parts we can find u(x, y).


Complex form of the Laplace equation
ee
Let u(x, y) be a harmonic function.
z + z̄ z − z̄ rin
[May 2015]

g.n
Since z = x + iy, z̄ = x − iy, we have, x = ,y = .
2 2i
⇒ u is a function of z and z̄.

∂u ∂u ∂x ∂u ∂y
= +
∂z ∂x ∂z ∂y ∂z
=
∂u 1 ∂u 1
+
et
∂x 2 ∂y 2i
1 ∂u i ∂u
= −
2 ∂x 2 ∂y
∂u 1  ∂u ∂u 
= −i .
∂z 2 ∂x ∂y
∂2 u ∂ 1 
= (u x − iuy )
∂z̄∂z ∂z̄ 2

Downloaded From : www.EasyEngineering.net


Downloaded From : www.EasyEngineering.net

Analytic Functions 321

1 ∂ ∂x ∂ ∂y 
= (u x − iuy ) + (u x − iuy )
2 ∂x ∂z̄ ∂y ∂z̄
1  1  −1 
= (u xx − iu xy ) + (uyx − iuyy )
2 2 2i
1 1
= (u xx − iu xy + iuyx + uyy ) = (u xx + uyy )
4 4
∂2 u
u xx + uyy = 4 . (1)
∂z̄∂z
∂2 u ∂2 u ∂2 u
i.e., 2 + 2 = 4 .
∂x ∂y ∂z̄∂z

ww Since u is harmonic,
∂2 u ∂2 u
+
∂x2 ∂y2
= 0.

w.E ∴
∂2 u
∂z̄∂z
= 0.

asy
This is the complex form of the Laplace equation.
From (1), we get the Laplacian operator

En ∂2
+
∂2
= 4
∂2
.

gin ∂x2 ∂y2



∂z̄∂z

Worked Examples

ee✍ ✌
Example 3.30. Find a function w such that w = u + iv is analytic if u = e x sin y.

rin [Jun 2000]


Solution. We have u = e x sin y.
u x = e x sin y. g.n
uy = e x cos y.
By Milne Thomson Method
et
f ′ (z) = u x (z, 0) − iuy (z, 0)

= ez sin 0 − iez cos 0

i.e., f ′ (z) = −iez .

Integrating both sides w.r.t. z we get, f (z) = −iez + C.

Downloaded From : www.EasyEngineering.net


Downloaded From : www.EasyEngineering.net

322 Engineering Mathematics - II

Example 3.31. Construct an analytic function f (z) for which the real part is
e x cos y. [Dec 2001]
Solution. Let f (z) = u + iv be the required function.
∴ u = e x cos y
u x = e x cos y
uy = −e x sin y.
By Milne Thomson method
f ′ (z) = u x (z, 0) − iuy (z, 0) = ez cos 0 − iez sin 0

ww f ′ (z) = ez .
Integrating both sides w.r.t. z we get f (z) = ez + c.

w.E
Example 3.32. If φ = 3x2 y − y3 , find ψ such that w = φ + iψ is an analytic function.
[May 1996]
Solution. φ = 3x2 y − y3 .
asy
φ x = 6xy.
φy = 3x2 − 3y2 . En
By Milne’s Thomson method
gin
w′ (z) = φ x (z, 0) − iφy (z, 0)

= 0 − i3z2

= −i3z2 .
ee rin
Integrating both sides w. r. t. z we get
g.n
et
Z Z
w (z)dz = −3i z2 dz + c, where c is a complex constant.

z3
w(z) = −3i +c
3
φ + iψ = −iz3 + c

= −i(x + iy)3 + c1 + ic2 , where c1 and c2 are real

= −i(x3 + 3ix2 y − 3xy2 − iy3 ) + c1 + ic2 .

Equating the imaginary parts we get, ψ = −x3 + 3xy2 + c1 .

Downloaded From : www.EasyEngineering.net


Downloaded From : www.EasyEngineering.net

Analytic Functions 323

Example 3.33. Find the analytic function whose real part is e2x sin 2y. [Dec 1996]
Solution. Let f (z) = u + iv be the required analytic function.
Given that u = e2x sin 2y.
u x = 2e2x sin 2y, uy = 2e2x cos 2y.
By Milne’s Thomson method

f ′ (z) = u x (z, 0) − iuy (z, 0) = 2e2z sin 0 − i2e2z cos 0 = −2ie2z .

Integrating w.r.t. z we get

ww Z
f (z) = −2i e2z dz + c, where c is complex.

w.E = −2i
e2z
2
+ c = −ie2z + c.

asy
Example 3.34. Construct an analytic function f (z) = u+iv, given that v = y+e x cos y.
[May 1999]
Solution. v = y + e x cos y.
En
gin
v x = e x cos y, vy = 1 − e x sin y.
By Milne’s Thomson method,

ee rin
f ′ (z) = vy (z, 0) + iv x (z, 0) = 1 − ez sin 0 + iez cos 0 = 1 + iez .

Integrating w.r.t. z, we obtain,


R
f (z) = (1 + iez )dz + c where c is complex. g.n
= z + iez + c.

Example 3.35. Find the analytic function u + iv whose real part is


et
u = x3 − 3xy2 + 3x2 − 3y2 + 1. [Jun 2006]
Solution. Given: u = x3 − 3xy2 + 3x2 − 3y2 + 1.

u x = 3x2 − 3y2 + 6x, uy = −6xy − 6y.

u x (z, 0) = 3z2 + 6z, uy (z, 0) = 0.

Downloaded From : www.EasyEngineering.net


Downloaded From : www.EasyEngineering.net

324 Engineering Mathematics - II

By Milne Thomson method,

f ′ (z) = u x (z, 0) − iuy (z, 0)

f ′ (z) = 3z2 + 6z.

Integrating w.r.t. z, we get


Z Z
f (z)dz = (3z2 + 6z)dz

z3 z2
=3 + 6 + c.

ww
3 2
= z3 + 3z2 + c.

w.E
Example 3.36. If u =
f (z) = u + iv.
sin 2x
cos h2y + cos 2x
find the corresponding analytic function
[Dec 2006]
Solution. Given, u =
sin 2x

asy
cos h2y + cos 2x
.

ux =
En
(cos h2y + cos 2x)2 cos 2x − sin 2x(− sin 2x) × 2
(cos h2y + cos 2x)2
.

u x (z, 0) =
gin
(1 + cos 2z)2 cos 2z + 2 sin2 2z
(1 + cos 2z)2

=
2 cos 2z + 2
(1 + cos 2z)2
ee
2 cos 2z + 2 cos2 2z + 2 sin2 2z
(1 + cos 2z)2

rin
=
2(1 + cos 2z)
(1 + cos 2z) 2
=
2
2 cos2 z
= sec2 z.
g.n
uy = sin 2x(−1)(cos h2y + cos 2x)−2 2 sin h2y.

uy (z, 0) = 0.
et
By Milne Thomson method,
f ′ (z) = u x (z, 0) − iuy (z, 0) = sec2 z.
Integrating w.r.t. z, we get
R R
f ′ (z)dz = sec2 zdz + c
f (z) = tan z + c.

Downloaded From : www.EasyEngineering.net


Downloaded From : www.EasyEngineering.net

Analytic Functions 325

Example 3.37. Find the analytic function w = u + iv where


v = e−2y (y cos 2x + x sin 2x) and find u. [Dec 2011]
Solution. v = e−2y (y cos 2x + x sin 2x).

∂v
= e−2y [−2y sin 2x + 2x cos 2x + sin 2x].
∂x
v x (z, 0) = 2z cos 2z + sin 2z.

vy = e−2y [cos 2x] + (y cos 2x + x sin 2x)(−2e−2y ).

vy (z, 0) = cos 2z + z sin 2z(−2) = cos 2z − 2z sin 2z.

ww By Milne Thomson method

w.E f ′ (z) = vy (z, 0) + iv x (z, 0)

asy
= cos 2z − 2z sin 2z + i(2z cos 2z + sin 2z)

= cos 2z + i sin 2z + 2iz(cos 2z + i sin 2z) = ei2z + 2izei2z = ei2z (1 + 2iz).

Integrating w.r.t.z we get


En
f (z) =
Z
gin
ei2z (1 + 2iz)dz + c

=
Z

ei2z
+ 2i zd
Z
Z

ei2z
ee
e dz + 2i zei2z dz + c
i2z

!
+c rin
=
2i
ei2z
"
i2z
Z
2i

+ ze − e dz + c = i2z
#
ei2z
+ zei2z −
ei2z
g.n
+ c = zei2z + c.
2i
u + iv = (x + iy)e2i(x+iy) + c1 + ic2

= (x + iy)e2ix−2y + c1 + ic2
2i 2i

et
= (x + iy)e2ix · e−2y + c1 + ic2

= (x + iy)e−2y (cos 2x + i sin 2x) + c1 + ic2 .

Equating the real parts we get,


u = e−2y (x cos 2x − y sin 2x) + c1 .

Downloaded From : www.EasyEngineering.net


Downloaded From : www.EasyEngineering.net

326 Engineering Mathematics - II

Example 3.38. Show that the function u(x, y) = 3x2 y + 2x2 − y3 − 2y2 is harmonic.
Find the conjugate harmonic function v and express u + iv as an analytic function
of z.
[Dec 2014, May 2004]
Solution. To prove u to be harmonic, we have to prove that u xx + uyy = 0.
Given, u = 3x2 y + 2x2 − y3 − 2y2

u x = 6xy + 4x

ww uy = 3x2 − 3y2 − 4y

u x (z, 0) = 4z

w.E uy (z, 0) = 3z2 .

asy
By Milne Thompson method,

f ′ (z) = u x (z, 0) − iuy (z, 0)

En
= 4z − i3z2 .

gin
Integrating w.r.t. z we get
Z

=4
z2
2
z3
− 3i + c
3
ee
f (z) = (4z − i3z2 )dz + c

rin
= 2z2 − iz3 + c, c is a complex constant
i.e., f (x + iy) = 2(x + iy)2 − i(x + iy)3 + c g.n
u + iv = 2(x2 − y2 + 2ixy) − i(x3 + 3x2 iy − 3xy2 − iy3 ) + c

= 2(x2 − y2 ) + 4ixy − ix3 + 3x2 y + i3xy2 − y3 + c


et
= 2(x2 − y2 ) + 3x2 y − y3 + i(3xy2 + 4xy − x3 ) + c1 + ic2

where c1 , c2 are real.

Equating the imaginary parts we get,


v = 4xy − x3 + 3xy2 + c2 .

Downloaded From : www.EasyEngineering.net


Downloaded From : www.EasyEngineering.net

Analytic Functions 327

Example 3.39. Prove that the function u = e x (x cos y − y sin y) satisfies Laplace
equation and find the corresponding analytic function f (z) = u + iv.
[Jun 2013, Dec 2010, May 2005]
Solution. Let f (z) = u + iv be the analytic function.
Given u = e x (x cos y − y sin y) is the real part of f (z).

u x = e x cos y + e x (x cos y − y sin y)

= e x (cos y + x cos y − y sin y).

ww uy = e x (−x sin y − y cos y − sin y).

u xx = e x (cos y + x cos y − y sin y) + e x cos y

w.E = e x (2 cos y + x cos y − y sin y).

uyy = e x (−x cos y + y sin y − cos y − cos y)

asy
= e x (−x cos y + y sin y − 2 cos y).

En
u xx + uyy = e x (2 cos y + x cos y − y sin y) + e x (−x cos y + y sin y − 2 cos y) = 0.

gin
⇒ u satisfies Laplace equation.
Now, u x (z, 0) = ez (1 + z)
and uy (z, 0) = 0.
ee
By Milne-Thomson method, f ′ (z) = u x (z, 0) − iuy (z, 0) = ez (1 + z).
Integrating w.r.t z we get rin
f (z) =
Z
ez (1 + z)dz + c g.n
Z
= (1 + z)d(ez ) + c
Z
= (1 + z)e − ez dz + c
z
et
= (1 + z)ez − ez + c = ez + zez − ez + c

∴ f (z) = zez + c.

Example 3.40. Prove that the function v = e−x (x cos y + y sin y) is harmonic and
determine the corresponding analytic function f (z) = u + iv. [May 2004, May 2014]

Downloaded From : www.EasyEngineering.net


Downloaded From : www.EasyEngineering.net

328 Engineering Mathematics - II

Solution. We first prove that v xx + vyy = 0.

v x = e−x (cos y) + (x cos y + y sin y)(−e−x )

= e−x (cos y − x cos y − y sin y)

v xx = e−x (− cos y) − e−x (cos y − x cos y − y sin y)

= e−x (− cos y − cos y + x cos y + y sin y)

= e−x (−2 cos y + x cos y + y sin y)

vy = e−x (−x sin y + y cos y + sin y)

ww vyy = e−x (−x cos y − y sin y + cos y + cos y)

w.E = e−x (2 cos y − x cos y − y sin y).

v xx + vyy = 0. ∴ v is harmonic

asy
Now, v x (z, 0) = e−z (1 − z)

and vy (z, 0) = e−z 0 = 0.

By Milne Thomson method,


En
gin
f ′ (z) = vy (z, 0) + iv x (z, 0) = ie−z (1 − z).
Integrating we get,

ee
f (z) = i
Z

Z
e−z (1 − z)dz + c

(1 − z)d(−e)−z dz + c rin
g.n
=i
Z
=i (z − 1)d(e)−z + c
h Z
= i (z − 1)e − e−z dz + c

−z


= i (z − 1)e−z + e−z + c
i
et
= ie−z (z − 1 + 1) + c

f (z) = ize−z + c.

Example 3.41. Prove that u = e−2xy sin(x2 −y2 ) is harmonic. Find the corresponding
analytic function and the imaginary part. [Dec 2013, Jan 2016]

Downloaded From : www.EasyEngineering.net


Downloaded From : www.EasyEngineering.net

Analytic Functions 329

Solution. u = e−2xy sin(x2 − y2 )

u x = e−2xy cos(x2 − y2 ) · 2x + sin(x2 − y2 ) · e−2xy (−2y)


h i
= 2e−2xy x cos(x2 − y2 ) − y sin(x2 − y2 ) .
h i
u xx = 2e−2xy −x sin(x2 − y2 ) · 2x + cos(x2 − y2 ) − y cos(x2 − y2 ) · 2x +
h i
x cos(x2 − y2 ) − y sin(x2 − y2 ) 2e−2xy (−2y)
h
u xx = 2e−2xy −2x2 sin(x2 − y2 ) + cos(x2 − y2 ) − 2xy cos(x2 − y2 ) − 2xy cos(x2 − y2 )
i

ww +2y2 sin(x2 − y2 ) .

uy = e−2xy cos(x2 − y2 )(−2y) + sin(x2 − y2 )e−2xy (−2x)

w.E
h i
= −2e−2xy y cos(x2 − y2 ) + x sin(x2 − y2 ) .
h i
uyy = −2e−2xy −y sin(x2 − y2 )(−2y) + cos(x2 − y2 ) + x sin(x2 − y2 )(−2y) +
h

asy i
y cos(x2 − y2 ) + x sin(x2 − y2 ) (−2e−2xy )(−2x)

En
h
= −2e−2xy 2y2 sin(x2 − y2 ) + cos(x2 − y2 ) − 2xy sin(x2 − y2 ) − 2xy cos(x2 − y2 )
i
−2x2 sin(x2 − y2 )
h
gin
= 2e−2xy −2y2 sin(x2 − y2 ) − cos(x2 − y2 ) + 2xy sin(x2 − y2 ) + 2xy cos(x2 − y2 )

ee
i
+2x2 sin(x2 − y2 ) .

Now, u xx + uyy = 0.
rin
∴ u is harmonic.
Now u x (z, 0) = 2z cos z2 . g.n
uy (z, 0) = −2z sin z2 .
By Milne Thomson method
et
f ′ (z) = u x (z, 0) + iuy (z, 0)

= 2z cos z2 + 2iz sin z2

= 2z(cos z2 + i sin z2 )
2
= 2zeiz

Downloaded From : www.EasyEngineering.net


Downloaded From : www.EasyEngineering.net

330 Engineering Mathematics - II

Integrating w.r.t.z we get


Z
2
f (z) = 2zeiz dz + c t = z2
Z
= eit dt + c dt = 2zdz

eit 1 2
= + c = eiz + c
i i
2
f (z) = −ieiz + c
2
u + iv = −iei(x+iy) + c1 + ic2

ww = −iei(x

= −iei(x
2 −y2 +2ixy)

2 −y2 )−2xy
+ c1 + ic2

w.E
+ c1 + ic2
2 −y2 )
= −iei(x · e−2xy + c1 + ic2

asy
= −ie−2xy [cos(x2 − y2 ) + i sin(x2 − y2 )] + c1 + ic2

= −ie−2xy cos(x2 − y2 ) + e−2xy sin(x2 − y2 )

En
Equating the imaginary parts we get
v = −e−2xy cos(x2 − y2 ).
gin
1
Example 3.42. Show that u =
function. Find also its Conjugate.
1 2
Solution. u = log(x + y ).
2
2
2
ee
log(x2 + y2 ) is harmonic. Determine its analytic
[May 2011]

rin
1
ux = · 2
2 x +y
1
2
· 2x = 2
x
x + y2
. g.n
u xx =
x2 + y2 − x · 2x

1
uy = · 2
(x2 + y2 )2
1
=

· 2y = 2
y2 − x2
(x2 + y2 )2
y
.
.
et
2 x +y 2 x + y2
x2 + y2 − y · 2y x2 − y2
uyy = = .
(x2 + y2 )2 (x2 + y2 )2
y2 − x2 + x2 − y2
Now, u xx + uyy = = 0.
(x2 + y2 )2
∴ u is harmonic.

Downloaded From : www.EasyEngineering.net


Downloaded From : www.EasyEngineering.net

Analytic Functions 331

z 1
u x (z, 0) = 2
= .
z z
uy (z, 0) = 0.

By Milne-Thomson method
1
f ′ (z) = u x (z, 0) − iuy (z, 0) = .
z
Integrating w.r.t.z we get
Z
1
f (z) = dz + c = log z + c.
z

ww 1 y
u + iv = log(x + iy) + c = log(x2 + y2 ) + i tan−1 + c1 + ic2 .
2 x

w.E Equating the imaginary parts we get


v = tan−1
y
x
+ c2 .

not regular. asy


Example 3.43. Prove that u = x2 − y2 and v = −
y
x2 + y2
are harmonic, but u + iv is
[Dec 2010, Dec 2014, Jan 2016]
Solution. u = x2 − y2 .
En
u x = 2x.
gin
u xx = 2.

uy = −2y.

uyy = −2.
ee rin
u xx + uyy = 2 − 2 = 0.
g.n
∴ u is harmonic.

v=−
y
x2 + y2
= −y(x2 + y2 )−1 .
et
2xy
v x = −y(−1)(x2 + y2 )−2 · 2x = 2 .
(x + y2 )2
" 2
(x + y2 )2 − x2(x2 + y2 ) · 2x
#
v xx = 2y
(x2 + y2 )4
2y(x2 + y2 )[x2 + y2 − 4x2 ]
=
(x2 + y2 )4

Downloaded From : www.EasyEngineering.net


Downloaded From : www.EasyEngineering.net

332 Engineering Mathematics - II

2y(y2 − 3x2 )
v xx = .
(x2 + y2 )3
x2 + y2 − y · 2y
vy = −
(x2 + y2 )2
x − y2
2 y2 − x2
=− 2 = .
(x + y2 )2 (x2 + y2 )2
(x2 + y2 )2 · 2y − (y2 − x2 ) · 2(x2 + y2 ) · 2y
vyy = .
(x2 + y2 )4
2y(x2 + y2 )[x2 + y2 − 2(y2 − x2 )]
=
(x2 + y2 )4

ww =
2y(x + y − 2y2 + 2x2 )
2 2

(x2 + y2 )3

w.E vyy =
2y(3x2 − y2 )
(x2 + y2 )3
2y
.

v xx + vyy = 2

∴ v is harmonic.
asy
(x + y2 )3
[y2 − 3x2 + 3x2 − y2 ] = 0.

En
We have to prove that, if u + iv is regular then u + iv must satisfy C-R
equations.
gin
But here, u x = 2x , vy and uy = −2y , −v x .

∴ u + iv is not regular.
ee
∴ C-R equations are not satisfied.

rin
Example 3.44. Find the Conjugate harmonic of v(r, θ) =
Show that v is harmonic.
r2 cos 2θ
g.n
− r cos θ + 2.

Solution. v = r2 cos 2θ − r cos θ + 2.


∂v
= 2r cos 2θ − cos θ.
et
∂r
∂2 v
= 2 cos 2θ.
∂r2
∂v
= −2r2 sin 2θ + r sin θ.
∂θ
∂2 v
= −4r2 cos 2θ + r cos θ
∂θ2

Downloaded From : www.EasyEngineering.net


Downloaded From : www.EasyEngineering.net

Analytic Functions 333

∂2 v 1 ∂v 1 ∂2 v 1
Now, 2
+ + 2 2 = 2 cos 2θ + (2r cos 2θ − cos θ)
∂r r ∂r r ∂θ r
1
+ 2 (−4r2 cos 2θ + r cos θ)
r
1 1
= 2 cos 2θ + 2 cos 2θ − cos θ − 4 cos 2θ + cos θ
r r
= 0.

∴ v is harmonic.
Using C-R equations we have

ww r
∂u ∂v
=
∂r ∂θ
∂u
= −2r2 sin 2θ + r sin θ.

w.E ∂r
= −2r sin 2θ + sin θ.

Integrating w.r.t.r we get

asy
Z Z
u = −2 sin 2θ rdr + sin θ dr + f (θ)

r2

En
!
= −2 sin 2θ + r sin θ + f (θ)
2

gin
u = −r2 sin 2θ + r sin θ + f (θ). (1)

Also by C-R equations we have


1 ∂u ∂v
=
r ∂θ ∂r
∂u
ee
= 2r cos 2θ − cos θ.
rin
∂θ
= −2r2 cos 2θ + r cos θ.

Integrating w.r.t θ we get g.n


Z Z
u = −2r2 cos 2θdθ + r cos θdθ + g(r)

= −2r2 ·
sin 2θ
+ r sin θ + g(r)
et
2
u = −r2 sin 2θ + r sin θ + g(r). (2)

From (1) and (2) we get


u = −r2 sin 2θ + r sin θ + c where c is a constant, which is the conjugate
harmonic of v.

Downloaded From : www.EasyEngineering.net


Downloaded From : www.EasyEngineering.net

334 Engineering Mathematics - II

Example 3.45. Find the analytic function f (z) = u + iv where v = 3r2 sin 2θ − 2r sin θ.
Verify that u is a harmonic function. [Dec 2013]
Solution. v = 3r2 sin 2θ − 2r sin θ.

∂v
= 6r sin 2θ − 2 sin θ.
∂r
∂2 v
= 6 sin 2θ.
∂r2
∂v
= 6r2 cos 2θ − 2r cos θ.
∂θ

ww ∂2 v
∂θ2
= −12r2 sin 2θ + 2r sin θ.

w.E Now,
∂2 v 1 ∂v 1 ∂2 v
+ +
1
= 6 sin 2θ + [6r sin 2θ − 2 sin θ]

asy
∂r2 r ∂r r2 ∂θ2 r
1
+ 2 [−12r2 sin 2θ + 2r sin θ]
r

En 2 2
= 6 sin 2θ + 6 sin 2θ − sin θ − 12 sin 2θ + sin θ
r r

gin = 0.

ee
∴ v is harmonic.

Using C-R equations we have


rin
r
∂u ∂v
=
∂r ∂θ
= 6r2 cos 2θ − 2r cos θ.
g.n
∂u
∂r
= 6r cos 2θ − 2 cos θ.

Integrating w.r.t.r we get


et
Z Z
u = 6 cos 2θ rdr − 2 cos θ dr + f (θ)

r2
= 6 cos 2θ · − 2r cos θ + f (θ)
2
u = 3r2 cos 2θ − 2r cos θ + f (θ). (1)

Downloaded From : www.EasyEngineering.net


Downloaded From : www.EasyEngineering.net

Analytic Functions 335

Again by C-R equations we have

1 ∂u ∂v
− = = 6r sin 2θ − 2 sin θ.
r ∂θ ∂r
∂u
= −6r2 sin 2θ + 2r sin θ.
∂θ

Integrating w.r.t.θ we get


Z Z
u = −6r2 sin 2θdθ + 2r sin θdθ + g(r)
!

ww = −6r 2 − cos 2θ
2
+ 2r(− cos θ) + g(r)

u = 3r2 cos 2θ − 2r cos θ + g(r). (2)

w.E From (1) and (2) we get

asy
u = 3r2 cos 2θ − 2r cos θ + c, where c is a constant,
which is the harmonic conjugate of v.
To verify whether u is harmonic
En
We have
∂u
∂r gin
= 6r cos 2θ − 2 cos θ.
∂2 u
∂r2
∂u
∂θ
= 6 cos 2θ.
ee
= −6r2 sin 2θ + 2r sin θ.
rin
∂2 u
∂θ2
= −6r2 · (cos 2θ) × 2 + 2r cos θ.
g.n
∂2 u 1 ∂u 1 ∂2 u
= −12r2 cos 2θ + 2r cos θ.

1
et
Now, + + = 6 cos 2θ + (6r cos 2θ − 2 cos θ)
∂r2 r ∂r r2 ∂θ2 r
1
+ 2 (−12r2 cos 2θ + 2r cos θ)
r
2 2
= 6 cos 2θ + 6 cos 2θ − cos θ − 12 cos 2θ + cos θ = 0.
r r

∴ u is harmonic.

Downloaded From : www.EasyEngineering.net


Downloaded From : www.EasyEngineering.net

336 Engineering Mathematics - II

Example 3.46. Determine the analytic function f (z) = u + iv such that


u − v = e x (cos y − sin y). [April 1997]
Solution. Given f (z) = u + iv.
i f (z) = iu − v
Adding we get, (1 + i) f (z) = u − v + i(u + v).
Put U = u − v, V = u + v.
F(z) = (1 + i) f (z) = U + iV.
Since f (z) is analytic, F(z) is also analytic.

ww U = u − v = e x (cos y − sin y)
U x = e x (cos y − sin y)

w.E Uy = e x (− sin y − cos y)


U x (z, 0) = ez , Uy (z, 0) = −ez .

asy
By Milne-Thomson method,

En
F (z) = U x (z, 0) − iUy (z, 0) = ez + iez = (1 + i)ez

Z
gin
Integrating we get

F(z) = (1 + i)ez dz + c

ee
= (1 + i)ez + c

(1 + i) f (z) = (1 + i)ez + c rin


f (z) = ez + c′ where c′ =
c
1+i
.
g.n
Example 3.47. If u + v =
of z find f (z) in terms of z.
e2y
2 sin 2x
+ e−2y − 2 cos 2x
et
and f (z) = u + iv is an analytic function
[Dec 2012]
Solution. f (z) = u + iv
i f (z) = iu − v
Adding we get
(1 + i) f (z) = u − v + i(u + v)
Let U = u − v, V = u + v, (1 + i) f (z) = F(z).

Downloaded From : www.EasyEngineering.net


Downloaded From : www.EasyEngineering.net

Analytic Functions 337

∴ F(z) = U + iV, U = u − v, V = u + v.
Since f (z) is analytic, F(z) is also analytic.
2 sin 2x sin 2x
Now, u + v = V = =
2 cosh 2y − 2 cos 2x cosh 2y − cos 2x
(cosh 2y − cos 2x)2 cos 2x − sin 2x.2 sin 2x
Vx =
(cosh 2y − cos 2x)2
(1 − cos 2z)2 cos 2z − 2 sin2 2z
V x (z, 0) =
(1 − cos 2z)2
2 cos 2z − 2 −2(1 − cos 2z) −2
= = = = −cosec2 z

ww
2 2
(1 − cos 2z) (1 − cos 2z) 2 sin2 z
Vy = sin 2x(−1)(cosh 2y − cos 2x)−2 2 sinh 2y

w.E Vy (z, 0) = 0.

By Milne Thomson method,

asy
F ′ (z) = Vy (z, 0) + iV x (z, 0) = 0 + i(−cosec2 z) = −i cosec2 z.

En
Integrating we get,

gin
Z
F(z) = −i cosec2 zdz + c. = −i(− cot z) + c.

F(z) = i cot z + c.

(1 + i) f (z) = i cot z + c.
i
ee c rin
g.n
f (z) = cot z +
1+i 1+i
i(1 − i) c
= cot z + c1 where c1 =

f (z) =
1+i
2
2
cot z + c1 .

Example 3.48. Find the analytic function


1+i

f (z) =
et
u + iv given that
2u + 3v = e x (cos y − sin y). [Dec 2005]
Solution. 2u + 3v = e x (cos y − sin y)

Given f (z) = u + iv

3 f (z) = 3u + i3v

Downloaded From : www.EasyEngineering.net


Downloaded From : www.EasyEngineering.net

338 Engineering Mathematics - II

i2 f (z) = 2iu − 2v

Adding we get

(3 + 2i) f (z) = 3u − 2v + i(2u + 3v).

Let U = 3u − 2v, V = 2u + 3v.

and F(z) = (3 + 2i) f (z) = U + iV.

Since f (z) is analytic, F(z) is also analytic.

ww Now, V = e x (cos y − sin y)

w.E V x = e x (cos y − sin y)

Vy = e x (− sin y − cos y)
V x (z, 0) = ez

Vy (z, 0) = −ez

asy
By Milne Thomson method

En
F (z) = Vy (z, 0) + iV x (z, 0) = −ez + iez

F ′ (z) = (−1 + i)ez


gin
ee
R
Integrating we get, F(z) = (−1 + i) ez + c
(3 + 2i) f (z) = (−1 + i)ez + c

−1 + i z c rin
f (z) =

=
3 + 2i
e +
3 + 2i
(−1 + i)(3 − 2i) z
e + c1 Where c1 =
c g.n
=

f (z) =
13
−3 + 2i + 3i + 2 z
13
−1 + 5i z
e + c.
e + c1
3 + 2i
et
13

Example 3.49. Determine the analytic function f (z) = u + iv given that


3u + 2v = y2 − x2 + 16xy. [May 2007]
Solution. Given f (z) = u + iv
2 f (z) = 2u + i2v

Downloaded From : www.EasyEngineering.net


Downloaded From : www.EasyEngineering.net

Analytic Functions 339

3i f (z) = 3iu − 3v
(2 + 3i) f (z) = 2u − 3v + i(3u + 2v)
Let U = 2u − 3v, V = 3u + 2v and F(z) = U + iV.
where F(z) = (2 + 3i) f (z).
Since f (z) is analytic, F(z) is also analytic.

Now, V = y2 − x2 + 16xy

V x = −2x + 16y

ww Vy = 2y + 16x

w.E By Milne Thomson method,

F ′ (z) = Vy (z, 0) + iV x (z, 0) = 16z − i2z = (16 − 2i)z

Integrating we get,
asy
En
F(z) = (16 − 2i)
z2
2
+c

gin
(2 + 3i) f (z) = (8 − i)z2 + c
8−i 2 c
f (z) =

= ee
2 + 3i
z +

13
2 + 3i
(8 − i)(2 − 3i) 2
z + c1 , c1 =
c

rin
2 + 3i
= (1 − 2i)z2 + c1 .

g.n
Example 3.50. Find the analytic function
5u − 3v = e x (5 cos y − 3 sin y).
Solution. f (z) = u + iv.
f (z) =

et
u + iv given that

5 f (z) = 5u + 5iv.

3i f (z) = 3iu − 3v.

Adding 5 f (z) + 3i f (z) = 5u − 3v + i(5v + 3u)

(5 + 3i) f (z) = 5u − 3v + i(5v + 3u)

Downloaded From : www.EasyEngineering.net


Downloaded From : www.EasyEngineering.net

340 Engineering Mathematics - II

F(z) = U + iV, where

F(z) = (5 + 3i) f (z).

U = 5u − 3v and

V = 5v + 3u.

Since f (z) is analytic, F(z) is also analytic.

Given U = 5u − 3v = e x [5 cos y − 3 sin y].

U x = e x [5 cos y − 3 sin y].

ww U x (z, 0) = 5ez .

w.E Uy = e x [−5 sin y − 3 cos y].

Uy (z, 0) = −3ez .

asy
By Milne-Thomson method,

F ′ (z) = U x (z, 0) − iUy (z, 0).

En
= 5ez + 3iez = (5 + 3i)ez .

Integrating w.r.t. z we get


Z gin
F(z) = (5 + 3i)ez + c

= (5 + 3i)ez + c ee rin
(5 + 3i) f (z) = (5 + 3i)ez + c

f (z) = ez +
c
g.n
et
5 + 3i
c
f (z) = ez + c′ where c′ = .
5 + 3i

Example 3.51. Find the analytic function f (z) = u + iv given that


4u − 3v = e x [(4x − 3y) cos y − (3x + 4y) sin y].
Solution. f (z) = u + iv.

4 f (z) = 4u + i4v.

i3 f (z) = 3iu − 3v.

Downloaded From : www.EasyEngineering.net


Downloaded From : www.EasyEngineering.net

Analytic Functions 341

4 f (z) + 3i f (z) = 4u − 3v + i(4v + 3u)

(4 + 3i) f (z) = 4u − 3v + i(4v + 3u).

F(z) = U + iV where

F(z) = (4 + 3i) f (z).

U = 4u − 3v and V = 3u + 4v.

Since f (z) is analytic, F(z) is also analytic.

ww Now, U = 4u − 3v = e x [(4x − 3y) cos y − (3x + 4y) sin y].

U x = e x [4 cos y − 3 sin y] + [(4x − 3y) cos y − (3x + 4y) sin y]e x .

w.E U x (z, 0) = 4ez + ez (4z) = 4ez (1 + z).

Uy = e x [−4x sin y − 3(−y sin y + cos y) − 3x cos y + 4(y cos y + sin y)].

asy
Uy (z, 0) = ez [−3 − 3z] = −3ez (1 + z).

By Milne-Thomson method,
En
gin
F ′ (z) = U x (z, 0) − iUy (z, 0)

Integrating w.r.t.z, we get ee


= 4ez (1 + z) + i3ez (1 + z) = ez (1 + z)(4 + 3i).

rin
g.n
Z
F(z) = ez (1 + z)(4 + 3i)dz + c
Z
= (4 + 3i) (1 + z)d(ez ) + c
"
z
Z
z
#
= (4 + 3i) e (1 + z) − e dz + c
et
= (4 + 3i) ez (1 + z) − ez + c = (4 + 3i)ez (1 + z − 1) + c
 

(4 + 3i) f (z) = (4 + 3i)zez + c


c
f (z) = zez +
4 + 3i
c
f (z) = zez + c′ where c′ = .
4 + 3i

Downloaded From : www.EasyEngineering.net


Downloaded From : www.EasyEngineering.net

342 Engineering Mathematics - II

Example 3.52. If f (z) is an analytic function of z, prove that


 ∂2 ∂2 
+ | f (z)|2 = 4| f ′ (z)|2 . [Dec 2014, Jun 2013, May 2011, May 2009]
∂x2 ∂y2
Solution. Given f (z) is analytic.
Let f (z) = u + iv.
⇒ u and v have continuous partial derivatives and they satisfy
C-R equations.
∴ u x = vy and uy = −v x and f ′ (z) = u x + iv x , | f ′ (z)|2 = u2x + v2x .
Since u and v are harmonic functions, they satisfy Laplace equation.

ww The complex form of the Laplace operator is

w.E ∂2
+
∂2
∂x2 ∂y2
= 4
∂2
∂z̄∂z
.

Now,
 ∂2

asy
+
∂2 
∂x2 ∂y2
| f (z)|2
= 4
∂2
∂z̄∂z
| f (z)|2
= 4
∂2
∂z̄∂z
( f (z) f (z)).

En
Since f (z) is an analytic function, it is independent of z̄.
i.e., f (z) is a function of z only.

gin
Similarly its conjugate f (z) is an analytic function of z̄ only.

i.e., f (x + iy) = f¯(x − iy).

∂2 ∂2
!
ee
∴ We can denote f (z) by f¯(z̄).

∂ ∂ rin
Hence,
∂x 2
+ 2 | f (z)|2 = 4
∂y ∂z̄ ∂z

( f (z) f¯(z̄))

∂ g.n
= 4 ( f¯(z̄)) ( f (z))
∂z̄ ∂z
= 4 f ′ (z) f ′ (z) = 4| f ′ (z)|2 . et
Example 3.53. If w = f (z) is a regular function of z, prove that
 ∂2 ∂2 
+ log | f ′ (z)| = 0. [May 2015, Dec 2001]
∂x2 ∂y2
Solution. Since f (z) is analytic, if f (z) = u + iv, then u and v are harmonic
functions. Hence, they satisfy Laplace equation. The complex form of
the Laplace operator is

Downloaded From : www.EasyEngineering.net


Downloaded From : www.EasyEngineering.net

Analytic Functions 343

∂2 ∂2 ∂2
+ = 4 .
∂x2 ∂y2 ∂z̄∂z

 ∂2 ∂2  ∂2  
′ ′
Now, + log | f (z)| = 4 log | f (z)|
∂x2 ∂y2 ∂z̄∂z
∂2
=2 log | f ′ (z)|2
∂z̄∂z
∂2  
=2 log( f ′ (z) f ′ (z))
∂z̄∂z
∂2  
=2 log f ′ (z) + log f ′ (z)

ww =2
∂z̄∂z
∂2 
∂z̄∂z
log f ′ (z) + log f ′ (z)
¯


w.E =2
∂  1 ′′ 
∂z̄ f ′ (z)
f (z) = 2.0 = 0.

asy
Example 3.54. If f (z) is analytic, prove that
 ∂2
+
∂2 
∂x2 ∂y2
| f (z)| p = p2 | f (z)| p−2 | f ′ (z)|2 .

En
[May 2001]
Solution. Let f (z) = u + iv.

gin
Since f (z) is analytic, u and v are harmonic functions.
Hence, they satisfy Laplace equation.

∂2
+
∂2
∂x2 ∂y2
= 4
∂2
∂z̄∂z
ee
The complex form of the Laplace operator is

rin
Now,
 ∂2
+
∂2 
| f (z)| p
= 4
∂2
| f (z)| p

g.n
et
∂x 2 ∂y2 ∂z̄∂z
∂2 p
=4 | f (z)|2 2
∂z̄∂z
∂2  p
=4 f (z) f (z) 2
∂z̄∂z
∂2  p p
=4 f (z) 2 f¯(z̄) 2
∂z̄∂z
∂ ¯  2p ∂ p
=4 f (z̄) f (z) 2
∂z̄ ∂z
p ¯  2p −1 ¯′ ¯ p p
= 4 f (z̄) f (z) f (z) 2 −1 f ′ (z)
2 2

Downloaded From : www.EasyEngineering.net


Downloaded From : www.EasyEngineering.net

344 Engineering Mathematics - II

p
= p2 f (z) f¯(z̄) 2 −1 f¯′ (z̄) f ′ (z)
  p −1
= p2 | f (z)|2 2 f¯′ (z̄). f (z)
p−2
= p2 | f (z)2 | 2 | f ′ (z)|2

= p2 | f ′ (z)|2 | f (z)| p−2 .

Example 3.55. If f (z) is an analytic function of z, prove that


 ∂2 ∂2 
+ |Re f (z)|2 = 2| f ′ (z)|2 . [Jun 2003]

ww ∂x2 ∂y2
Solution. Let f (z) = u + iv.
Given f (z) is analytic.

w.E Since Re f (z) = u, we have to prove that


We have f ′ (z) = u x + iv x .
 ∂2
+
∂2  2
∂x2 ∂y2
(u ) = 2| f ′ (z)|2 .

| f ′ (z)|2 = u2x + v2x


asy
Now,
 ∂2
En
∂2  2 ∂2 u2 ∂2 u2
u =
∂ ∂

gin
2
+ 2 2
+ 2
= (2uu x ) + (2uuy )
∂x ∂y ∂x ∂y ∂x ∂y
= 2(uu xx + u2x ) + 2(uuyy + u2y )

ee
= 2u(u xx + uyy ) + 2(u2x + u2y ).

rin
Since f (z) is analytic, u is harmonic.
∴ u xx + uyy = 0. g.n
Hence,
 ∂2
+
∂x2 ∂y2
∂2  2
u = 2(u2x + u2y ) = 2(u2x + v2x ) = 2| f ′ (z)|2 .
et
Example 3.56. If f (z) is an analytic function, prove that
∂ 2  ∂ 2
| f (z)| + | f (z)| = | f ′ (z)|2 .
∂x ∂y
Solution. Given f (z) = u + iv.
Since f (z) is analytic, u and v are harmonic.
Also, f (z) satisfies C-R equations.

Downloaded From : www.EasyEngineering.net


Downloaded From : www.EasyEngineering.net

Analytic Functions 345

i.e., u x = vy , uy = −v x ⇒ f ′ (z) = u x + iv x , | f ′ (z)|2 = u2x + v2x .

p
| f (z)| = u2 + v2
∂ 1
| f (z)| = √ (2uu x + 2vv x )
∂x 2 u2 + v2
uu x + vv x
= √
u2 + v2
∂ 2 (uu x + vv x )2
| f (z)| =
∂x u2 + v2

ww Similarly,
∂ 2 (uuy + vvy )2
| f (z)| =

w.E Adding,
∂
∂x
| f (z)| +
∂y
2  ∂
∂y
| f (z)| =
u2 + v2
2 u2 u2x + v2 v2x + 2uvu x v x + u2 u2y + v2 v2y + 2uvuy vy
u2 + v2

asy =
u2 (u2x + u2y ) + v2 (v2x + v2y ) + 2uv(u x v x + uy vy )
u2 + v2

En =
u (u x + v x ) + v (uy + u2x ) + 2uv(−u x uy + u x uy )
2 2 2 2 2

gin
u2 + v2
u | f (z)| + v | f (z)|2
2 ′ 2 2 ′
=
u2 + v2
=
ee
| f (z)| (u2 + v2 )
′ 2

u2 + v2
= | f ′ (z)|2 .

rin
Example 3.57. If f (z) is analytic, prove that
1
Solution. ∇2 log | f (z)| = ∇2 log | f (z)|2
∇2 log | f (z)| = 0.
g.n [Jun 2012]

1
= ∇2 log( f (z) f (z))
2
et
1  ∂2 ∂2  
= 2
+ 2
log f (z) + log f¯(z)
¯
2 ∂x ∂y
1 ∂ 2  
= 4 log f (z) + log f¯(z)
¯
2 ∂z̄∂z
∂  1 ′ 
=2 f (z) = 0.
∂z̄ f (z)

Downloaded From : www.EasyEngineering.net


Downloaded From : www.EasyEngineering.net

346 Engineering Mathematics - II

Exercise 3 C

1. Construct the analytic function whose imaginary part is


v = x2 − y2 + 2xy − 3x − 2y. [Dec 2001]

2. If u = x3 − 3xy2 + y + 1, find f = u + iv is analytic. [Dec 2004]


y
3. Construct the analytic function whose real part is u = . [April 2004]
x2 + y2
4. Find the analytic function whose imaginary part is given by

ww v = e x (x sin y + y cos y). [Jan 2008]

w.E 5. Find the analytic function whose real part is given by u = e−x (x sin y − y cos y).
[May 2005]

asy
6. If u = log(x2 + y2 ), find v and f (z) such that f (z) = u + iv is analytic. [May 2005]

En
7. Find the analytic function w = u + iv given v = e−2xy sin(x2 − y2 ). [May 1997]

gin
8. If w = u + iv is an analytic function and v = x2 − y2 +
x2
x
+ y2
find u. [Dec 1997]

ee
9. Find the analytic function f (z) = u + iv if u − v = (x − y)(x2 + 4xy + y2 ).

10. If f (z) = u + iv and u + v = (x − y)(x2 + 4xy + y2 ) find f (z).


rin
11. Find the analytic function f (z) given that u + v = e x (cos y − sin y).
g.n
12. If f (z) = u + iv is an analytic function of z and u − v =
π
given that f ( ) = 0.
2
cos x + sin x − e−y
2(cos x − cos hy)
et
, find f (z)

13. Show that the following functions are harmonic

i. u = 2x − x3 + 3xy2 . [May 2000]


−y
ii. v = 2 . [Dec 2001]
x + y2
iii. u = 3x2 y − y3 . [Jun 2001]

Downloaded From : www.EasyEngineering.net


Downloaded From : www.EasyEngineering.net

Analytic Functions 347

14. Show that the function u = y3 −3x2 y is a harmonic function. Find its harmonic
conjugate and the corresponding analytic function.

15. Show that u = y + e x cos y is harmonic. Find its harmonic conjugate and also
find the function with u as its real part.

16. If u − v = x3 − y3 + 3xy(x − y), find f (z) = u + iv interms of z.

17. Find the analytic function f (z) = u + iv, given that u − 2v = e x (cos y − sin y).

ww 18. If 2u + v = e x (cos y − sin y) find f (z) = u + iv.

w.E
19. Find the analytic function f (z) = u + iv given that 2u + v = e2x ((2x + y) cos 2y +
(x − 2y) sin 2y).
asy
En
20. If f (z) = u + iv is an analytic function of z, find f (z) if 2u + v = e x (cos y − sin y).

21. Find
1
the analytic function
u + v = log(x2 + y2 ) + tan−1
y
. gin f (z) = u + iv given that

2 x

ee
22. Find the analytic function f (z) = u + iv given that u + v =
rin
sin 2x
cos h2y + cos 2x
.

23. Find the analytic function f (z) = u + iv, given that u + v = g.n
sin 2x
cos h2y − cos 2x
.

et
24. Find the analytic function f (z) = u + iv given that 3u + 7v = e x (3 cos y + 7 sin y).

25. Find the analytic function f (z) = u + iv, given that


2 −y2
3u + 5v = e x (3 cos 2xy + 5 sin 2xy).

26. Find the analytic function f (z) =! u + iv given


1
(i) u = a(1 + cos θ) (ii) v = r − sin θ, r , 0.
r

Downloaded From : www.EasyEngineering.net


Downloaded From : www.EasyEngineering.net

348 Engineering Mathematics - II

3.6 Bilinear Transformation (Mobius Transformation)

Definition. The Transformation

az + b
w= (1)
cz + d

where a, b, c, d are complex constants such that ad − bc , 0 is called a bilinear


transformation or Mobius transformation.

ww Results

w.E 1. If ad − bc = 0 then ad = bc ⇒

Now w =
a(z + ba ) a
d b
= .
c a
which is a constant.
=
c(z + d
c)
c

asy
∴ ad − bc , 0 is the necessary condition for (1) to be bilinear.

2. When z =
−d
c
, then w = ∞.
En
gin
Therefore, there is a one to one correspondence between the extended
a
z−plane and the extended w−plane. When w = , z = ∞.
c
3. Special cases.
(i) If c = 0, a = d , 0 then w =
ee
a
d
b
d
b
rin
z + = z + , which is a translation.
a

a
a
d
b g.n
(ii) If c = 0, b = 0, d , 0 then w = z which is rotation and magnification.

et
(iii) If c = 0, d , 0 then w = z + which is rotation, magnification and
d d
translation.

3.6.1 Fixed points or Invariant points

Result. Prove that a bilinear transformation has atmost two fixed points.
[Jun 2012]

Downloaded From : www.EasyEngineering.net


Downloaded From : www.EasyEngineering.net

Analytic Functions 349

The fixed points of the bilinear transformation are given by w = z


az + b
i.e., z =
cz + d
=⇒ cz2 + dz − az − b = 0

i.e., cz2 + (d − a)z − b = 0

If c , 0, it is a quadratic in z.
∴ It has two roots which are the fixed points of the transformation.
If c = 0, d , a then there is one fixed point.

ww In this case, it is a linear transformation.

w.E
∴ A bilinear transformation has atmost two fixed points in the extended plane.
Note

asy
1. If a bilinear transformation has three or more fixed points, then it must
be the identity mapping.

En
2. If a bilinear mapping has exactly two fixed points z1 and z2 , then for some

gin
nonzero k, they satisfy the equation.

w − z1 z − z1

ee w − z2
=k
z − z2

rin
3. If a bilinear transformation has only one fixed point z1 , then it can be
written as
1
w − z1
=k+
1
z − z1
for k , 0.
g.n
These two forms are called normal forms or canonical forms of a bilinear
transformation.

Cross ratio
et
(z1 − z2 )(z3 − z4 )
The cross ratio of four points z1 , z2 , z3 , z4 is defined as and it is
(z1 − z4 )(z3 − z2 )
denoted by (z1 , z2 , z3 , z4 ).
Note. The four letters can be arranged in 4! = 24 ways and hence we can write
24 cross ratios. But, only six of them are different. In each of them z1 is fixed
and the remaining 3 are arranged in 3! = 6 ways.

Downloaded From : www.EasyEngineering.net


Downloaded From : www.EasyEngineering.net

350 Engineering Mathematics - II

If λ is any cross ratio of 4 points then the 6 different cross ratios are
1 1 1 1 λ
λ, , 1 − λ, ,1 − , 1
=
λ 1−λ λ 1− λ
λ−1

Theorem. The bilinear transformation that maps the points z1 , z2 , z3 of the


z−plane on to the points w1 , w2 , w3 of the w plane is given by
(w − w1 )(w2 − w3 ) (z − z1 )(z2 − z3 )
= .
(w − w3 )(w2 − w1 ) (z − z3 )(z2 − z1 )
This equation defines a bilinear transformation that maps the distinct points

ww z1 , z2 , z3 onto the distinct points w1 , w2 , w3 of the finite w−plane.

w.ENote. If one of the given points is ∞, say z1 = ∞. Then


Now
1
z1
= 0.

RHS = asy
(z − z1 )(z2 − z3 )
(z − z3 )(z2 − z1 )
z1
En
z
z1

− 1 (z2 − z3 )
−1(z2 − z3 ) z2 − z3

gin
=  z2  = =
(z − z3 )z1 −1 (z − z3 )(−1) z − z3
z1

(w − w1 )(w2 − w3 ) z2 − z3
(w − w3 )(w2 − w1 )
=
z − z3
ee
∴ The modified form of the bilinear transformation is

rin
i.e., The equation is obtained formally by omitting the factors involving
z1 .
g.n
In the same way, we can rewrite the equation of the bilinear transformation if
any other prescribed point is ∞ in the z−plane or w−plane.

Properties of Bilinear transformation


et
1. A bilinear transformation always transforms circles into circles and lines into
lines.

2. Bilinear transformation preserves cross ratio of four points.


i.e., cross ratio of four points is invariant under bilinear transformation.

Downloaded From : www.EasyEngineering.net


Downloaded From : www.EasyEngineering.net

Analytic Functions 351

✎ ☞
Worked Examples
✍ ✌

Example 3.58. Find the bilinear map which maps the points ∞, i, 0 of the z plane
onto 0, i, ∞ of the w−plane. [May 2005]
Solution. Let z1 = ∞, z2 = i, z3 = 0 and w1 = 0, w2 = i, w3 = ∞.
The bilinear transformation is given by

w − w1 z2 − z3
=
w2 − w1 z − z3
w−0 i−0

ww i.e.,
i−0
=
w i
=
z−0

w.E i
w=
z
−1
z
.

asy
Example 3.59. Find the invariant points of the transformation w =
2z + 6
z+7
.

En
Solution. The fixed points are given by w = z
[May 2009]

i.e.,
2z + 6
=z gin
z+7
z(z + 7) = 2z + 6

z2 + 7z − 2z − 6 = 0
ee rin
z2 + 5z − 6 = 0
g.n
(z + 6)(z − 1) = 0

z = 1, −6. et
3−z
Example 3.60. Find the fixed points of the transformation w = .
1+z
Solution. The fixed points are given by w = z

3−z
i.e., =z
1+z
z(1 + z) = 3 − z

Downloaded From : www.EasyEngineering.net


Downloaded From : www.EasyEngineering.net

352 Engineering Mathematics - II

z + z2 − 3 + z = 0

z2 + 2z − 3 = 0

(z + 3)(z − 1) = 0 ⇒ z = 1, −3.

1+z
Example 3.61. Find the invariant points of the transformation w = .
1−z
[Dec 2011, Jun 2007]
Solution. The fixed points are given by w = z
1+z

ww i.e.,
1−z
=z

z(1 − z) = 1 + z

w.E z − z2 − 1 − z = 0

z2 + 1 = 0

asy
z2 = −1 ⇒ z = ±i.

En
Example 3.62. Find the bilinear transformation which maps the points −2, 0, 2
into the points w = 0, i, −i.

gin
Solution. Let z1 = −2, z2 = 0, z3 = 2 and w1 = 0, w2 = i, w3 = −i.
[May 2002]

ee
The bilinear transformation is given by
(w − w1 )(w2 − w3 ) (z − z1 )(z2 − z3 )
=
(w − w3 )(w2 − w1 ) (z − z3 )(z2 − z1 ) rin
(w − 0)(i + i) (z + 2)(0 − 2)
=
(w + i)(i − 0) (z − 2)(0 + 2) g.n
w2i
(w + i)i
=−
2(z + 2)
2(z − 2)
.

2w(z − 2) = −(z + 2)(w + i)


et
2w(z − 2) + w(z + 2) = −i(z + 2)

w(2z − 4 + z + 2) = −i(z + 2)

w(3z − 2) = −i(z + 2)
i(z + 2)
w=− .
3z − 2

Downloaded From : www.EasyEngineering.net


Downloaded From : www.EasyEngineering.net

Analytic Functions 353

Example 3.63. Find the Mobius transformation that maps z = 0, 1, ∞ into points
w = −5, −1, 3 respectively. What are the invariant points of the transformation.
[May 2004]
Solution. Let z1 = 0, z2 = 1, z3 = ∞, and w1 = −5, w2 = −1, w3 = 3.
The Mobius transformation is
(w − w1 )(w2 − w3 ) (z − z1 )
=
(w − w3 )(w2 − w1 ) (z2 − z1 )
(w + 5)(−4) z − 0
i.e., =

ww
(w − 3)4 1
⇒ −(w + 5) = z(w − 3)

w.E ⇒ −w − 5 = zw − 3z

⇒ −w − zw = 5 − 3z

asy
⇒ −w(1 + z) = −(3z − 5)
3z − 5

En
w= .
1+z

gin
The invariant points are given by w = z.
3z − 5
z=
1+z
z + z2 − 3z + 5 = 0

z2 − 2z + 5 = 0
ee rin
(z − 1)2 + 5 − 1 = 0
g.n
(z − 1)2 + 4 = 0

(z − 1)2 = −4

z − 1 = ±2i
et
z = 1 ± 2i.

Example 3.64. Find the bilinear transformation that maps the points z = ∞, i, 0
onto w = 0, i, ∞ respectively. [Dec 2012]
Solution. Given, z1 = ∞, z2 = i, z3 = 0.

Downloaded From : www.EasyEngineering.net


Downloaded From : www.EasyEngineering.net

354 Engineering Mathematics - II

w1 = 0, w2 = i, w3 = ∞.
The bilinear transformation is
(w − w1 ) z2 − z3
=
w2 − w1 z − z3
w−0 i−0
=
i−0 z−0
w i
=
i z
1
w=− .
z

ww Example 3.65. Find the bilinear transformation mapping the points z = 1, i, −1


into the points w = 2, i, −2. [Dec 2005]

w.E
Solution. Let z1 = 1, z2 = i, z3 = −1 and w1 = 2, w2 = i, w3 = −2.
The bilinear transformation is given by

asy
(w − w1 )(w2 − w3 ) (z − z1 )(z2 − z3 )
=
(w − w3 )(w2 − w1 ) (z − z3 )(z2 − z1 )
i.e.,
En
(w − 2)(i + 2) (z − 1)(i + 1)
=
(w + 2)(i − 2) (z + 1)(i − 1)
=
gin
w − 2 z − 1 (i + 1)(i − 2)
w + 2 z + 1 (i − 1)(i + 2)

ee
!
z − 1 (−1 − 2i + i − 2)
=
z + 1 (−1 + 2i − i − 2)

=
!
z − 1 (−3 − i)
z + 1 (−3 + i) rin
w−2
w+2
=
!
z − 1 (3 + i)
z + 1 (3 − i) g.n
w − 2 + w + 2 (z − 1)(3 + i) + (z + 1)(3 − i)
=
w − 2 − w − 2 (z − 1)(3 + i) − (z + 1)(3 − i)
2w 3z + iz − 3 − i + 3z − iz + 3 − i
=
et
−4 3z + iz − 3 − i − 3z + iz − 3 + i
6z − 2i
=
2iz − 6
2(3z − i)
=
2(iz − 3)
(6z − 2i)
w=− .
iz − 3

Downloaded From : www.EasyEngineering.net


Downloaded From : www.EasyEngineering.net

Analytic Functions 355

Example 3.66. Find the bilinear transformation which maps the points i, −1, 1 of
the z plane into the points 0, 1, ∞ of the w−plane respectively. [Jun 2008]
Solution. Let z1 = i, z2 = −1, z3 = 1 and w1 = 0, w2 = 1, w3 = ∞.
The required bilinear transformation is
(w − w1 ) (z − z1 )(z2 − z3 )
=
(w2 − w1 ) (z − z3 )(z2 − z1 )
(z − i)(−2) −2(z − i) 2(z − i)
i.e., w = = = .
(z − 1)(−1 − i) −(z − 1)(1 + i) (1 + i)(z − 1)
Example 3.67. Find the bilinear transformation which maps z1 = 0, z2 = 1, z3 = ∞

ww into the points w1 = i, w2 = 1, w3 = −i. [Jun 2013, Jun 2012, Jun 2010, Jun 2008]
Solution. Let z1 = 0, z2 = 1, z3 = ∞, and w1 = i, w2 = 1, w3 = −i.

w.E The mobius transformation is

(w − w1 )(w2 − w3 ) (z − z1 )

asy
=
(w − w3 )(w2 − w1 ) (z2 − z1 )
(w − i)(1 + i)
i.e., =z

En
(w + i)(1 − i)
w − i z(1 − i) z(1 − i)2
= =
w+i
=
2
1+i
= gin
z(1 − 1 − 2i) z(−2i)
2
2
= −iz
w − i + w + i −iz + 1
=
w − i − w − i −iz − 1
2w
=
1 − iz
ee rin
−2i −(1 + iz)
w=
i(1 − iz)
1 + iz
=
z+i
1 + iz
. g.n
Example 3.68. Find the bilinear map if 1 and i are fixed points and the origin
goes into −1.
et
Solution. Let z1 = 1, z2 = i, z3 = 0 and w1 = 1, w2 = i, w3 = −1.
The bilinear transformation is

(w − w1 )(w2 − w3 ) (z − z1 )(z2 − z3 )
=
(w − w3 )(w2 − w1 ) (z − z3 )(z2 − z1 )
(w − 1)(i + 1) (z − 1)(i − 0)
=
(w + 1)(i − 1) (z − 0)(i − 1)

Downloaded From : www.EasyEngineering.net


Downloaded From : www.EasyEngineering.net

356 Engineering Mathematics - II

(w − 1)(i + 1) (z − 1)i
=
w+1 z
w−1 (z − 1)i
=
w+1 z(1 + i)
w−1+w+1 (z − 1)i + z(1 + i)
=
w − 1 − (w + 1) (z − 1)i − z(1 + i)
2w z(1 + 2i) − i
=
−2 −(z + i)
z(1 + 2i) − i
w= .
z+i

ww Example 3.69. Show that w =


i−z
i+z
maps the real axis of the z−plane into the
circle |w| = 1 and the half plane y > 0 into the interior of the unit circle |w| = 1 in

w.E
the w plane.
Solution. Given w =
i−z
.

asy
i+z

|w| = 1 becomes

|
i−z
i+z
|=1
En

|i − z|
|i + z|
=1
gin
⇒ |i − z| = |i + z|
ee
i.e., |i − x − iy| = |i + x + iy|
rin
| − x + i(1 − y)| = |x + i(1 + y)|

| − x + i(1 − y)|2 = |x + i(1 + y)|2 g.n


x2 + (1 − y)2 = x2 + (1 + y)2

(1 − y)2 = (1 + y)2
et
1−y=1+y

i.e., 2y = 0

⇒ y = 0, which is the real axis.

∴ The real axis of the z−plane is mapped onto the circle |w| = 1.

Downloaded From : www.EasyEngineering.net


Downloaded From : www.EasyEngineering.net

Analytic Functions 357

Now, for the interior of the circle |w| = 1 we have

|w| < 1
|i − z|
⇒ <1
|i + z|
|i − z| < |i + z|

(1 − y)2 < (1 + y)2

⇒1−y<1+y

ww ⇒ −2y < 0

2y > 0

w.E i.e., y > 0.

Hence, the half plane y > 0 is mapped onto the interior of the circle |w| < 1.

asy
Example 3.70. Find the bilinear map which maps the points z = 0, −1, i onto the

En
points w = i, 0, ∞. Also find the image of the unit circle of the z−plane.

gin
[May 2015, Dec 2013]
Solution. Given, z1 = 0, z2 = −1, z3 = i.
w1 = i, w2 = 0, w3 = ∞.
ee
The bilinear transformation is
w − w1 (z − z1 )(z2 − z3 ) rin
g.n
=
w2 − w1 (z − z3 )(z2 − z1 )
w−i (z − 0)(−i − 1)
=
0−i
w−i
−i
w−i
=−
(z − i)(−1 − 0)
z(1 + i)
(z − i)(−1)
z(1 + i)
et
=
−i z−i
−iz(1 + i) −iz + z
w−i= =
z−i z−i
−iz + z iz + 1 − iz + z
w=i+ =
z−i z−i
1+z
w= .
z−i

Downloaded From : www.EasyEngineering.net


Downloaded From : www.EasyEngineering.net

358 Engineering Mathematics - II

w(z − i) = 1 + z

wz − wi = 1 + z

wz − z = 1 + wi

z(w − 1) = 1 + wi
1 + wi
z= .
w−1
Given |z| = 1

1 + wi = 1

ww w−1
|1 + wi|
=1

w.E |w − 1|
|1 + (u + iv)i| = |u + iv − 1|

asy
|1 + iu − v|2 = |(u − 1) + iv|2

(1 − v)2 + u2 = (u − 1)2 + v2

En
1 + v2 − 2v + u2 = u2 + 1 − 2u + v2

−2v = −2v

u = v, gin
which is a straight line in the w−plane. ee rin
Example 3.71. Prove that w =
z
1−z
g.n
maps the upper half of the z−plane to the
upper half of the w−plane and also find the image of the unit circle of the z−plane.

Solution. Given w =
z
1−z
.
et
[Dec 2013, Dec 2012, Jun 2010]

x + iy
u + iv =
1 − x − iy
x + iy (1 − x) + iy
= ×
(1 − x) − iy (1 − x) + iy
x(1 − x) − y2 + i[y(1 − x) + xy]
=
(1 − x)2 + y2

Downloaded From : www.EasyEngineering.net


Downloaded From : www.EasyEngineering.net

Analytic Functions 359

x(1 − x) − y2 y(1 − x) + xy
= 2 2
+i .
(1 − x) + y (1 − x)2 + y2
x(1 − x) − y2 y(1 − x) + xy
∴u= and v = .
(1 − x)2 + y2 (1 − x)2 + y2
Let v > 0.

y(1 − x) + xy
⇒ >0
(1 − x)2 + y2
y − xy + xy > 0

ww y > 0.

w.E This shows that the upper half of the z−plane is mapped onto the upper
half of the w−plane.

asy
To find the image of the unit circle.

z
we have w =

En
1−z

gin
w(1 − z) = z

w − wz = z

w = wz + z

z(1 + w) = w
w
ee rin
z=
1+w
.
g.n
et
Given |z| = 1.

w
⇒ =1
1 + w
|w| = |1 + w|

|w|2 = |1 + w|2

|u + iv|2 = |1 + u + iv|2

u2 + v2 = (1 + u)2 + v2 = 1 + u2 + 2u + v2

0 = 2u + 1

Downloaded From : www.EasyEngineering.net


Downloaded From : www.EasyEngineering.net

360 Engineering Mathematics - II

1
u=− .
2
1
The image of the unit circle |z| = 1 is the straight line u = − .
2
Example 3.72. Find the bilinear transformation that transforms 1, i, −1 of the
z−plane onto 0, 1, ∞ of the w−plane. Also show that the transformation maps
interior of the unit circle of the z−plane onto upper half of the w−plane.[Apr 2014]
Solution. Let z1 = 1, z2 = i, z3 = −1.
w1 = 0, w2 = 1, w3 = ∞.

ww The bilinear transformation is


w − w1 (z − z1 )(z2 − z3 )

w.E =
w2 − w1 (z − z3 )(z2 − z1 )
w − 0 (z − 1)(i + 1)
=
1 − 0 (z + 1)(i − 1)

asy
w=−
z−11+i 1+i
z+11−i 1+i
×

=−
En
z − 1 (1 + i)2
z+1 1+1

gin
!
z − 1 1 − 1 + 2i z−1
=− = −i
z+1 2 z+1
w(z + 1) = −iz + i

wz + w = −iz + i
ee rin
wz + iz = i − w

z(w + i) = i − w g.n
z=

Given |z| < 1


i−w
w+i
.
et

i − w
⇒ <1
w + i
|i − w| < |w + i|

|i − u − iv| < |u + iv + i|

| − u + i(1 − v)| < |u + i(1 + v)|

Downloaded From : www.EasyEngineering.net


Downloaded From : www.EasyEngineering.net

Analytic Functions 361

| − u + i(1 − v)|2 < |u + i(1 + v)|2

u2 + (1 − v)2 < u2 + (1 + v)2

(1 − v)2 < (1 + v)2

1 − v < 1 + v ⇒ −2v < 0

v > 0,

which is the upper half of the w−plane.

ww Hence, the interior of the unit circle of the z−plane is mapped onto the
upper half of the w−plane.

w.E
Exercise 3 D

asy
1. Find the bilinear transformation that maps z1 = −1, z2 = 0, z3 = 1 onto
w1 = −1, w2 = −i, w3 = 1.
En [May 2006]

points w = i, 0, −i. gin


2. Find the bilinear transformation which maps the points z = 1, i, −1 onto the
[Jun 2005]

ee rin
3. Find the bilinear transformation that maps z1 = 0, z2 = 1, z3 = ∞ onto
w1 = −1, w2 = −i, w3 = 1. [Apr 2006]

g.n
4. Find the bilinear transformation that maps the points −1, 0, 1 onto 1, −1, ∞.

et[Jan 2007]

5. Find the bilinear transformation which maps the points 1, i, −1 of the z−plane
onto 0, 1, ∞ of the w−plane. [May 2007]

6. Find the bilinear transformation which maps z = −1, i, 1 onto the points
w = 1, i, −1 respectively. [Dec 2004]

2i − 6z
7. Find the fixed points of the bilinear map w = .
iz − 3

Downloaded From : www.EasyEngineering.net


Downloaded From : www.EasyEngineering.net

362 Engineering Mathematics - II

8. Find the bilinear transformation that maps the points −1, 0, 1 in the z−plane
onto the points 0, i, 3i in the w−plane. [Apr 1999]

9. Obtain the bilinear transformation which maps z = 0 onto w = −i and has


−1, 1 as fixed points. [Apr 2000]

10. Find the bilinear transformation which maps z = −i, 0, i onto the points
w = ∞, −1, 0.

ww 3.7 Conformal mapping

w.E Y
C2
V

θ
C2′

asy P
θ C1
P′
C1′

O
En X
O
U

gin
Let C1 and C2 be two curves in the z−plane intersecting at P. Let C1′ and C2′ be

ee
the corresponding curves in the w plane intersecting at P′ under the

rin
transformation w = f (z). If the angle of intersection of the curves at P is the same
as the angle of intersection of the curves at P′ , both in magnitude and direction,
then the transformation is said to be conformal at P.
g.n
Result A mapping w = f (z) is said to be conformal in a domain D if it is
conformal at each point of the domain.
et
Theorem
A mapping w = f (z) is conformal at a point z0 if it is analytic at z0 and f ′ (z0 ) , 0.
Critical Points
Let f (z) be a non constant analytic function in a domain D. If f ′ (z0 ) = 0 for some z0
in D, then z0 is called a critical point of the transformation w = f (z).

Downloaded From : www.EasyEngineering.net


Downloaded From : www.EasyEngineering.net

Analytic Functions 363

Fixed point or invariant point


A fixed point or invariant point of a transformation w = f (z) is a point z0 such that
f (z0 ) = z0 .
Scale factor
Scalar factor of a conformal mapping w = f (z) at z0 is | f ′ (z0 )|.
If | f ′ (z0 )| > 1 then | f ′ (z0 )| represents an expansion.
If | f ′ (z0 )| < 1 then | f ′ (z0 )| represents an contraction.
Mapping of Elementary functions

ww 3.7.1 The transformation w = z + k

w.E
The mapping w = z + k where k is a complex constant represents a translation.
Let z = x + iy, w = u + iv, k = k1 + ik2

asy
u + iv = x + iy + k1 + ik2 = (x + k1 ) + i(y + k2 ).
The image of any point (x, y) in the z−plane is the point (x + k1 , y + k2 ) in the
w−plane.
En
Under translation any figure is shifted through a distance given by the vector k.

gin
But the size and shape remain the same.

ee
∴ Circles are transformed into circles, squares are transformed into squares etc.
In general, under the transformation, the image region is geometrically
congruent to the original region.
rin
If k = 0, then the mapping becomes w = z which is the identity mapping.

Worked Examples

g.n
✍ ✌
Example 3.73. Find the image of the circle |z| = 2 under the transformation
w = z + 3 + 2i.
et
[May 2011, May 2008]
Solution. w = z + 3 + 2i
u + iv = x + iy + 3 + 2i = x + 3 + i(y + 2).
Equating the real and imaginary parts, we get u = x + 3, v = y + 2.
The equation of the circle is |z| = 2.
p
i.e., x2 + y2 = 2

Downloaded From : www.EasyEngineering.net


Downloaded From : www.EasyEngineering.net

364 Engineering Mathematics - II

=⇒ x2 + y2 = 4
i.e., (u − 3)2 + (v − 2)2 = 4
which represents a circle with centre (3, 2) and radius 2 in the w−plane.

Example 3.74. What is the region of the w−plane of the rectangular region in the
z−plane bounded by the lines x = 0, x = 2, y = 0 and y = 1 in the region mapped
under the transformation w = z + (1 + 2i). [Dec 2006]
Solution. w = z + 1 + 2i

ww u + iv = x + iy + 1 + 2i = x + 1 + i(y + 2).
Equating the real and imaginary parts, we get u = x + 1, v = y + 2.

w.E
The lines x = 0, x = 2, y = 0 and y = 1 have images u = 1, u = 3, v = 2, v = 3
respectively in the w plane.

asy V
Y

En v=3

gin
u=1

u=3
y=1

ee
x=0

x=2

v=2

rin
U
0

g.n
X
0 y=0

Example 3.75. Find the map of the circle |z| = 3 by the transformation w = z + 1 + i.

Solution. w = z + 1 + i
et[May 2007]

u + iv = x + iy + 1 + i = x + 1 + i(y + 1).
Equating the real and imaginary parts, we get u = x + 1, v = y + 1.
The circle in the z−plane is |z| = 3.
q
i.e., x2 + y2 = 3

Downloaded From : www.EasyEngineering.net


Downloaded From : www.EasyEngineering.net

Analytic Functions 365

=⇒ x2 + y2 = 9

i.e., (u − 1)2 + (v − 1)2 = 9

which represents a circle with centre (1, 1) and radius 3 in the w−plane.

Example 3.76. Find the image of the circle |z| = 1 under the map w = z + (2 + 2i).
Solution. w = z + 2 + 2i
i.e., u + iv = x + iy + 2 + 2i = x + 2 + i(y + 2)
Equating the real and imaginary parts we obtain

ww u= x+2 v=y+2

w.E
x=u−2 y = v − 2.
The circle in the z−plane is
|z| = 1 ⇒ x2 + y2 = 1

asy
⇒ (u − 2)2 + (v − 2)2 = 1.

En
i.e., The unit circle |z| = 1 is mapped onto an equal circle with centre
(2, 2) and radius 1.

Y
gin V

O
ee X
O
(2, 2)

rin
U

g.n
3.7.2 The transformation w = az
et
Here a is a nonzero complex constant and z , 0.
Let z = reiθ , w = Reiφ , a = a1 eiα where |a| = a1 .
Now, the given transformation is w = az

Reiφ = a1 eiα reiθ = a1 rei(α+θ)

Downloaded From : www.EasyEngineering.net


Downloaded From : www.EasyEngineering.net

366 Engineering Mathematics - II

∴ The transformation equations are R1 = a1 r, φ = α + θ.


∴ The point (r, θ) in the z−plane is mapped onto (a1 r, α + θ) in the w plane.

i.e., The magnitude of the vector p(z)

representing z in the plane is magnified

r
or contracted by |a| = a1 in the w−plane
θ
and rotated through an angle α = arg a. O
X

ww ∴ This transformation
magnification and rotation.
consists
Hence,
of
V

w.E
p(az)
the image of the given region is
geometrically similar to that of the

r
a,
original.
asy O
α+θ
U

Note
En
gin
1. If a is real then α = 0. ⇒ w = az represents magnification or contraction
by |a|.
ee rin
2. The general transformation w = az + b where a , 0, a and b are complex
numbers is a composition of the transformation w = az and w = z + b.

g.n
∴ The general linear transformation represents expansion or contraction
by |a| and rotation followed by a transaction by vector b.

Worked Examples



et
Example 3.77. Find the image of the circle |z| = 2, under the transformation
w = 3z. [Jun 2008]
Solution. Given w = 3z
Taking modulus both sides we get
|w| = |3z| = 3|z| = 3 × 2 = 6.

Downloaded From : www.EasyEngineering.net


Downloaded From : www.EasyEngineering.net

Analytic Functions 367


=⇒ u2 + v2 = 6
or u2 + v2 = 36.
∴ The image of |z| = 2 under the transformation w = 3z is a circle with
centre (0, 0) and radius 6.

Example 3.78. Find the map of the circle |z| = 3 under the transformation w = 2z.
[Dec 2012, May 2011]
Solution. Given w = 2z

ww
|w| = |2z| = 2|z| = 2 × 3 = 6.
It is a circle in the w−plane with center at the origin and radius 6 units.

w.E
Example 3.79. Find the image of the circle |z| = λ under the transformation w = 5z.
[May 2001]
Solution. Given w = 5z
asy
Taking modulus both sides we get

En
|w| = |5z| = 5|z| = 5λ.
p
u2 + v2 = 5λ gin
u2 + v2 = 25λ2 .
ee
This is a circle in the w plane with centre (0, 0) and radius 5λ.
rin
Example 3.80. Find the image of the rectangular region bounded by x = 0,
g.n
et
y = 0, x = 1, y = 2 under the map w = (1 + i)z + 2.
Solution. Given w = (1 + i)z + 2

ie., u + iv = (1 + i)(x + iy) + 2 = x + iy + ix − y + 2

u + iv = x − y + 2 + i(x + y)

Equating the real and imaginary parts we get

u= x−y+2 v= x+y

Downloaded From : www.EasyEngineering.net


Downloaded From : www.EasyEngineering.net

368 Engineering Mathematics - II

Y B′ (1, 3)

C (0, 2) y=1 B(1, 2)

C ′ (0, 2)
x=0

x=1
A′ (3, 1)

u
O′ (0, 0) O′ (2, 0)
X
O (0, 0) y=0 A(1, 0)

ww Let us find the image of the points O, A, B, C.

w.E x = 0, y = 0 ⇒ u = 2, v = 0 ⇒ O(0, 0) has image O′ (2, 0)

asy
x = 1, y = 0 ⇒ u = 3, v = 1 ⇒ A(1, 0) is mapped onto A′ (3, 1)

En
x = 1, y = 2 ⇒ u = 1, v = 3 ⇒ B(1, 2) has the image B′ (1, 3)

x = 0, y = 2 ⇒ u = 0, v = 2 ⇒ C(0, 2) is mapped onto C ′ (0, 2).

gin
The image of the given rectangular region and its image are as
presented above.
ee
Example 3.81. Draw the image of the square whose vertices are at
rin
(0, 0), (1, 0), (1, 1), (0, 1) in the z− plane under the transformation w = (1 + i)z. What
has this transformation done to the original square. g.n [Dec 2004]
Solution. Given w = (1 + i)z

i.e., u + iv = (1 + i)(x + iy) = x + iy + ix − y = x − y + i(x + y).


et
Equating the real and imaginary parts we get u = x − y, v = x + y.
We shall find the images of the corners (x, y) → (u, v).

Downloaded From : www.EasyEngineering.net


Downloaded From : www.EasyEngineering.net

Analytic Functions 369

v
y B′ (1, 3)

C (0, 1) B(1, 1)

C′ A′
(−1, 1) (1, 1)

x
O (0, 0) A(1, 0) u
O′ (0, 0) O′ (2, 0)

ww Let the given points be O(0, 0), A(1, 0), B(1, 1) and C(0, 1).
Let O′ , A′ , B′ , C ′ be the images of O, A, B, C respectively.

w.E Now O′ is (0, 0), A′ is (1, 1), B′ is (0, 2) and C ′ is (−1, 1).

In the original square, the radius vector of each corner is extended 2 times and
π
asy
rotated through an angle of about the origin in the anticlockwise sense.
4

En√ √
OA′ = 2 = 2OA

gin √
OB′ = 2 = 2OB
√ √

ee
OC ′ = 2 = 2OC.

Example 3.82. Find the image of the region y > 1 under the transformation
rin
w = (1 − i)z.
Solution. Given w = (1 − i)z g.n [May 1999]

i.e., u + iv = (1 − i)(x + iy) = x + iy − ix + y = x + y + i(y − x)

Equating the real and imaginary parts we get


et
u= x+y (1)

v=y−x (2)
1
(1) + (2) =⇒ u + v = 2y ⇒ y = (u + v)
2

Downloaded From : www.EasyEngineering.net


Downloaded From : www.EasyEngineering.net

370 Engineering Mathematics - II

1
(1) − (2) =⇒ u − v = 2x ⇒ x = (u − v)
2
1
y > 1 ⇒ (u + v) > 1 ⇒ u + v > 2.
2
The regions corresponding to the original and image are presented below.

Y V

(0, 2)

ww O
X
O (2, 0)
U

w.E
Definition. The inverse of a point P with respect to a circle O and radius r is the
point P′ on OP such that OP.OP′ = r2 .

asy p

En |
p′

O gin
θ
−θ
X

ee rin
3.7.3 The transformation w =
1
z g.n
Given w =
1
1 z
⇒ |w| =
z
et
|w||z| = 1.
It represents inversion w.r.t the unit circle |z| = 1 followed by reflection
on the real axis.
Let z = reiθ , w = Reiφ
1
Now w =
z

Downloaded From : www.EasyEngineering.net


Downloaded From : www.EasyEngineering.net

Analytic Functions 371

1 1
=⇒ Reiφ = iθ
= e−iθ .
re r
Equating the modulii and amplitudes we get
1
R = , φ = −θ, i.e., arg w = − arg z
r
1
|w| =
|z|
1 1
The image of P(r, θ) is Q( , −θ) under the transformation w = .
r z
1
′ ′
If P is the inverse of P w.r.t |z| = 1 then P is ( , θ). The reflection of P′
r

ww 1
1
in the real axis is ( , −θ).
r
∴ w = consists of inversion w.r.t |z| = 1 followed by reflection in the real

w.E axis.
z

If |z| < 1, then |w| > 1.

asy
∴ Interior of the circle |z| = 1 is mapped onto the exterior of the circle

En
|w| = 1 and conversely.
The circle |z| = 1 is mapped onto |w| = 1.
1

gin
1
Now lim = ∞ and lim = 0, the centre of the circle is mapped onto ∞.
z→0 z z→∞ z
Hence, w = 1z defines a one to one correspondence between the extended

ee
z− plane and the extended w-plane.

rin
Result. When w = u + iv is the image of a nonzero point z = x + iy, under the
1
transformation w = then
z
z̄ z̄ x − iy g.n
w=

u=
x2
= 2 = 2
zz̄ |z|
x
+y2
x + y2

,v = 2
−y
x + y2
.
et
1
Also, z =
w
w̄ w̄ u − iv
⇒z= = =
ww̄ |w|2 u2 + v2
u −v
x= 2 2
,y = 2 .
u +v u + v2

Downloaded From : www.EasyEngineering.net


Downloaded From : www.EasyEngineering.net

372 Engineering Mathematics - II

✎ ☞
Worked Examples
✍ ✌
1
Example 3.83. Find the image of the line x = k, under the transformation w = .
z
[Jun 2013]
1
Solution. Given w = .
z
1 w̄ w̄ w̄
z= × = = 2
w w̄ ww̄ |w|
u − iv u v
x + iy = 2 = 2 −i 2

ww u +v 2 u +v 2 u + v2
u v
∴x= 2 ,y = − 2 .
u + v2 u + v2

w.E Given x = k.

i.e., k = 2
u
u + v2

asy
k(u2 + v2 ) − u = 0.

En
It represents a circle with centre
1
2k
!
1
, 0 and radius .
2k

gin 1
Example 3.84. Find the image of x = 2 under the transformation w = .

Solution. We have w =
1
z
ee z
[Dec 1998]

rin
g.n
1 w̄
=⇒ z = =
w ww̄

x + iy =

∴x= 2
u
u +v
u − iv

2
=

,y = − 2
v
u − iv
(u + iv)(u − iv) u2 + v2

u + v2
.
.
et
u
When x = 2, we have = 2.
u2 + v2
u
⇒ u2 + v2 − = 0,
2

which is a circle in the w−plane.

Downloaded From : www.EasyEngineering.net


Downloaded From : www.EasyEngineering.net

Analytic Functions 373

1
Example 3.85. Find the image of |z − 2i| = 2 under the transformation w = .
z
[Jun 2013, May 2005]
1
Solution. w =
z
u −v
⇒x= 2 2
,y = 2 .
u +v u + v2
The given circle is |z − 2i| = 2.

⇒ |z − 2i|2 = 4

|x + iy − 2i|2 = 4

ww x2 + (y − 2)2 = 4

x2 + (y − 2)2 = 22 .

w.E It represents circle center at (0, 2) radius 2.

2
u2
(u + v )2 2
+
asy
2
v2
(u + v )2 2
−4 2
−v
u + v2
=0

2
1
u +v 2
+ 2
4v
u + v2
En
=0

gin
1
4v + 1 = 0 ⇒ v = − ,
4

ee
which is a straight line in the w plane.
∴ The image of the circle |z − 2i| = 2 in the z−plane is a straight line in the w−plane.

rin
1
Example 3.86. Find the image of the strip 1 < x < 2 under the map w = .

1
z
g.n
[May 2001]
Solution. Given w =

⇒x= 2
z
u
u +v 2
,y = 2
−v
u + v2
et
u
1<x<2⇒1< 2 <2
u + v2
⇒ u2 + v2 < u < 2(u2 + v2 ).

Taking u2 + v2 < u ⇒ u2 + v2 − u < 0, which is the interior of the circle

u2 + v2 − u = 0. (1)

Downloaded From : www.EasyEngineering.net


Downloaded From : www.EasyEngineering.net

374 Engineering Mathematics - II

1
The centre is ( −1
2 , 0) and radius = 2 .
Taking u < 2(u2 + v2 ) we get

2u2 + 2v2 − u > 0

1
u2 + v2 − u > 0. (2)
2
1
which is the exterior of the circle u2 + v2 − u = 0.
2
1 1
The centre is ( , 0) and radius = = r.

ww 4 4
∴ The strip 1 < x < 2 is mapped onto the region between the circles
(1) and (2).

w.E V
Y

asy
(0, 2)

En (1/4, 0)
(1/2, 0)
U
x=1

x=2

O gin X O

ee
Example 3.87. Find the image of the infinite strip 0 < y <
1
2
rin
1
under w = .
z
[May 2005]
Solution. Given w =
1
z g.n
⇒x=

1
u2
u
+v2
,y = 2

−v
−v
u + v2
1
et
0<y< ⇒0< 2 2
<
2 u +v 2
1
⇒ 0 < −v < (u2 + v2 )
2
Taking 0 < −v ⇒ −v > 0 ⇒ v < 0 (1)
1
Taking − v < (u2 + v2 )
2

Downloaded From : www.EasyEngineering.net


Downloaded From : www.EasyEngineering.net

Analytic Functions 375

⇒ −2v < u2 + v2

u2 + v2 > −2v

u2 + v2 + 2v > 0 (2)

It is a circle with centre (0, −1), r = 1.


The shaded region below represents the original strip and its image in
the w−plane.

ww y = 1/2
V

w.E O y=0 X
(0,−1)

asy
Example 3.88. Find the image of the square whose vertices are z = 1 + i, 3 + i, 1 + 3i

En
1
and 3 + 3i under the transformation w = . [Dec 2001]
z
Solution. Let A, B, C, D represent the complex numbers. 1 + i, 3 + i, 1 + 3i and 3 + 3i
respectively in the z−plane.
gin
⇒x=
w=

u
1
z

,
eey=
−v
rin
u=
u2
x
+ v2
, v= 2
u2
−y
+ v2
g.n
Y
x2 + y2 x + y2

V et
(1, 3) (3, 3)
D C
y=3   U
x=1

x=3

1
  2,0
0, − 16
 
0, 21
A y=1 B
(1, 1) (3, 1)
X
O

Downloaded From : www.EasyEngineering.net


Downloaded From : www.EasyEngineering.net

376 Engineering Mathematics - II

Equation of AB is y = 1.
Equation of CD is y = 3.
Equation of AC is x = 1.
Equation of AD is x = 3.
−v
Image of y = 1 is 1 = 2 ⇒ u2 + v2 + v = 0.
u + v2
−1 1
It is a circle with centre (0, ), r = .
2 2
2 2 1 −1 1
Image of y = 3 is u + v + v = 0. It is a circle with centre (0, ), r = .
3 6 6

ww 2 2 1 1
Image of x = 1 is u + v − u = 0. It is a circle with centre ( , 0), r = .
2 2
2 2 1 1 1
Image of x = 3 is u + v − u = 0. It is a circle with centre ( , 0), r = .

w.E 3
The point A(1, 1) is mapped onto A′ ( ,

The point B(3, 1) is mapped onto B′ ( ,


1 −1
2 2
3 −1
).
6 6

asy
).
10 10
1 −1
C(3, 3) is mapped onto C ′ ( , ).
6 6

En
D(1, 3) is mapped onto D′ ( ,
1 −3
10 10
).

gin
∴ The image of the interior of the square ABCD is the region A′ B′C ′ D′
bounded by the 4 circles which are the images of the sides of the square.

ee
Example 3.89. Show that the transformation w =
into a circle in the w−plane or to a straight line.
1
1
z
maps a circle in the z−plane

rin
[Dec 2011, Jun 2009]
Solution. Given w =
z
Let w = u + iv, z = x + iy g.n
z=
1
=
w̄ u − iv
= 2
w ww̄ u + v2
u −v
et
x= 2 2
,y = 2 .
u +v u + v2
The general equation of the circle in the z−plane is

x2 + y2 + 2gx + 2 f y + c = 0 (1)
u2 v2 u −v
2 2 2
+ 2 2 2
+ 2g 2 2
+ 2f 2 +c=0
(u + v ) (u + v ) u +v u + v2

Downloaded From : www.EasyEngineering.net


Downloaded From : www.EasyEngineering.net

Analytic Functions 377

u2 + v2 2gu 2fv
2 2 2
+ 2 2
− 2 + c = 0.
(u + v ) u +v u + v2
1 2gu 2fv
+ 2 − 2 +c=0
u2 +v2 u +v 2 u + v2
1 + 2gu − 2 f v + c(u2 + v2 ) = 0

c(u2 + v2 ) + 2gu − 2 f v + 1 = 0. (2)

If (1) does not pass through the origin then c , 0.


(2) is a circle in the w plane.

ww 1
∴ w = transforms a circle in the z−plane not passing through the origin
z
into a circle not passing through the origin in the w plane.

w.E If (1) passes through the origin then c = 0.


(2)⇒ 2gu − 2 f v + 1 = 0 which is a straight line not passing through
the origin.
asy
∴ A circle not passing through the origin in the z-plane is mapped onto

En
a circle in the w-plane and a circle through the origin in the z-plane is

gin
mapped onto a straight line in the w-plane.

ee
Example 3.90. Find the image of the hyperbola x2 − y2 = 1 under the
1
transformation w = and prove that it is the lemniscate of Bernouillie r2 = cos 2θ.
z
rin
[Dec 2012, Jun 2012, Jun 2010]
Solution. Let z = x + iy and w =
1 1 1 1
Given w = or z = = iθ = e−iθ .
reiθ .
g.n
z
1
w re r

i.e., x + iy = (cos θ − i sin θ)


r
et
1 1
∴ x = cos θ, y = − sin θ.
r r
The given curve is x2 − y2 = 1
1 1
2
cos2 θ − 2 sin2 θ = 1
r r
1
2
(cos θ − sin2 θ) = 1
2
r

Downloaded From : www.EasyEngineering.net


Downloaded From : www.EasyEngineering.net

378 Engineering Mathematics - II

1
cos 2θ = 1
r2
⇒ r2 = cos 2θ.
1
Example 3.91. Under the transformation w = , find the image of the region (i)
z
x > c where c > 0 (ii) y > c where c < 0. Find also the fixed points of w. [Dec 2010]
1
Solution. w =
z
u −v
⇒x= 2 2
,y = 2
u +v u + v2

ww (i) x > c, c > 0 ⇒ 2


u
u +v 2
u
> c ⇒ > u2 + v2
c

w.E
u
u2 + v2 − < 0.
c
1 1
It is a circle with centre ( , 0) and radius .

asy 2c
The image is the interior of the circle.
2c

En
x=c

gin V

O
eeX

1
2c , 0
 U

rin
g.n
(ii) y > c, c < 0
−v
u2 + v2
>c et
−v
< u2 + v2
c
v
u2 + v2 + > 0.
c
v
The image is the exterior of the circle u2 + v2 + = 0 in the w−plane
c
−1 1
with centre (0, ) and radius .
2c 2|c|

Downloaded From : www.EasyEngineering.net


Downloaded From : www.EasyEngineering.net

Analytic Functions 379

Y V

y=0
X
 
−1
0,
O 2c

y=c U

The fixed points are given by w = z.


1
i.e., z = ⇒ z2 = 1 ⇒ z = ±1.

ww z

3.7.4 The transformation w = z2

w.E Let z = x + iy and w = u + iv.

asy
2
Given w=z

u + iv = (x + iy)2 = x2 − y2 + i2xy

∴ u = x2 − y2
En (1)

and v = 2xy
gin (2)

ee
Case(i). Image of lines parallel to the axes.
Consider the line y = b, parallel to the x-axis.
Now u = x2 − b2 , v = 2bx ⇒ x = .
v rin
v2
∴u= 2 −b 2
2b
g.n
v2
4b2
4b
= u + b2 ⇒ v2 = 4b2 (u + b2 )

Y
et (3)

y=b
• (a, b)

X
x=a

Downloaded From : www.EasyEngineering.net


Downloaded From : www.EasyEngineering.net

380 Engineering Mathematics - II

This is a parabola with vertex (−b2 , 0), axis u-axis.


Hence, the image of a line parallel to the x-axis in the z-plane is a
parabola in the w-plane.
Consider the line x = a, parallel to the y-axis.
v
Then, (1) and (2) are reduced to u = a2 − y2 , v = 2ay ⇒ ie., y = .
2a

v2
∴ u = a2 −
4a2

ww v2
4a2
= −(u − a2 ) v2 = −4a2 (u − a2 ) (4)

w.E
which represents a parabola with vertex
(a2 , 0), axis as u-axis and focus (0, 0). We
v

asy
shall prove that these two parabolas are
u

En
(−b2 , 0) (0, 0) (a2 , 0)
orthogonal. Let us find the slopes of
the tangents to the parabolas (3) and

gin
(4) corresponding to the point x = a and

ee
y = b.

ie., u = a2 − b2 and v = 2ab


Differentiating (3) w.r.to u, we get rin
2v
dv
= 4b2 g.n
dv 2b2
du
dv
!
=
du

v
2b2 b
et
= =
du (a2 −b2 ,2ab) 2ab a
b
i.e., Slope of the tangent at (a2 − b2 , 2ab) to the parabola (3) is m1 = .
a
Differentiating (4) w.r.to u, we get
dv
2v = −4a2
du

Downloaded From : www.EasyEngineering.net


Downloaded From : www.EasyEngineering.net

Analytic Functions 381

dv −2a2
=
du v
−2a2 −a
!
dv
= =
du (a2 −b2 ,2ab) 2ab b
−a
i.e., Slope of the tangent at (a2 − b2 , 2ab) to the parabola (4) is m2 = .
b

b −a
Now, m1 m2 = × = −1
a b
Hence, the tangents are perpendicular and therefore the parabolas are

ww orthogonal.
Hence, lines parallel to the axes in the z-plane are mapped on to orthogonal

w.E
parabolas in the w-plane.
Case(ii). Image of circles.

asy
We shall now study how circles in the z-plane are transformed by the mapping
w = z2 .

En
Changing into polar coordinates, let z = reiθ and w = Reiφ .
We have w = z2 ⇒
gin
Reiφ = r2 ei2θ .
Equating the modulii and arguments on both sides, we get

R = r2 , φ = 2θ.
ee
This shows that the modulus is squared and the amplitude is doubled. rin
Along the real axis in the z-plane θ takes values 0 and π.
g.n
When θ = 0
When θ = π


φ=0
φ = 2π
Hence, φ assumes values 0 and 2π, which is the positive side of the u-axis.
et
Hence, the real axis in the z-plane is mapped onto the positive real axis in the
π 3π
w-plane. Along the imaginary axis, θ takes values and while φ assumes
2 2
values π and 3π, which is only the negative side of the imaginary axis.
i.e., The imaginary axis of the z-plane is mapped on to the negative real axis in
π
the w-plane.In the first quadrant of the z-plane θ varies from 0 to while φ varies
2

Downloaded From : www.EasyEngineering.net


Downloaded From : www.EasyEngineering.net

382 Engineering Mathematics - II

from 0 to π.

i.e., the first quadrant of the


Y V
z-plane is mapped on to the
entire upper half of the w-
plane. Similarly, the upper
X U
O z−plane w−plane
half of the z-plane is mapped 0 ≤ θ ≤ π2 0≤Φ≤π
on to the whole of the w-plane.

ww Y V

w.E U

asy
X
z−plane w−plane
0≤θ≤π 0 ≤ Φ ≤ 2π

En
In the same way the lower half of the z-plane is mapped onto the entire w-plane.

gin
Hence, two distinct points z0 and −z0 = (z0 eiπ ) in the z-plane correspond to a single
point w = z20 in the w-plane.

domain R in the w-plane.


ee
In the same way two domains D1 and D2 in z-plane will be mapped onto the single

rin
y θ = π/3
θ = 2π/3
v
θ = π/3
g.n
et
θ = π/6
D1 R

x u
D2 1 2 1 4

Case (iii). The images of u(x, y) = c and v(x, y) = d where c and d are constants.
We shall study the images if the lines u(x, y) = c and v(x, y) = d under the mapping
w = z2 .

Downloaded From : www.EasyEngineering.net


Downloaded From : www.EasyEngineering.net

Analytic Functions 383

u=c ⇒ x2 − y2 = c (5)
v=d ⇒ 2xy = d (6)
(5) and (6) represent a family of rectangular hyperbolas.
y = −x x=0 u=4 u = 16

v
u=4

u = 16 D1 v = 32
R
u = 32 v = 16 32 •

v=2 24 •

y=0 16 •

ww D2
8 •


1

8

16

24
u

w.E
asy
For the family of R.H represented by (5), the asymptotes are the lines y = x and
y = −x. [Taking u = 0]

En
For the family of R.H represented by (6), the asymptotes are the axes given by y = 0
and x = 0. Infact, the families of rectangular hyperbolas (5) and (6) are orthogonal.
For, differentiating (5) w.r.to x, we get
gin
Differentiating (6) w.r.to x, we get
ee
2x − 2y
dy
dx
=0 ⇒
dy x
=
dx y

rin
"
dy
2 x +y =0
#

dy −y
= g.n
−y
dx
x
dx x

Hence, at any point (x, y), the slope of the tangent to (5) is m1 = and the slope of
y
et
the tangent to (6) is m2 = .
x

∴ m1 × m2 = −1.

∴ (5) and (6) are orthogonal.


Also we observe that, (x, y) and (−x, −y) satisfy both (5) and (6). Thus, the regions

Downloaded From : www.EasyEngineering.net


Downloaded From : www.EasyEngineering.net

384 Engineering Mathematics - II

D1 and D2 in the z-plane correspond to only one region R in the w-plane. This shows
that the transformation w = z2 is two-to-one.
Application. If we consider the axes to represent two walls, a single quadrant
could be used to represent the flow of a fluid at a corner wall. This transformation
can also represent the electrostatic field in the vicinity of a corner conductor.
✎ ☞
Worked Examples
✍ ✌
Example 3.92. Find the image of the circle |z| = a by the mapping w = z2 .
Solution. Let z = reiθ and w = Reiφ .

ww w = z2 ⇒ Reiφ = r2 ei2θ .

w.E ∴ R = r2 and φ = 2θ.


Now, |z| = a is a circle with centre (0, 0) and radius r = a. Along this circle, θ varies
form 0 to 2π. In the w-plane, we have R = a2 and φ varies from 0 to 4π.

asy
∴ |z| = a in the z-plane is mapped onto the circle |w| = a2 , whose centre is the origin
and radius = a2 in the w-plane.
En y v

gin |z| = a |w| = a2

ee
O
x
O
u

rin
Note: As θ varies from 0 to π, φ varies from 0 to 2π. Hence, the upper semicircle g.n
|z| = a is also mapped onto the full circle |w| = a2 .

Example 3.93. Find the image of the semi circular area r ≤ θ, 0 ≤ θ ≤ π by the
et
map w = z2 .
Solution. By the transformation w = z2 , if z = reiθ and w = Reiφ , we get, R = r2 and
φ = 2θ. r ≤ θ, 0 ≤ θ ≤ π is the interior of the upper semi circle |z| = a. When r ≤ a,
we have, R ≤ a2 and as 0 ≤ θ ≤ π,we obtain 0 ≤ φ ≤ 2π. This is
the interior of the whole circular area, |w| = a2 in the w-plane.

Downloaded From : www.EasyEngineering.net


Downloaded From : www.EasyEngineering.net

Analytic Functions 385

Example 3.94. Find the image of the hyperbola x2 − y2 = 10 under the


transformation w = z2 if w = u + iv. [May 1997].
Solution. For w = z2 we have
u = x2 − y2 (1)
and v = 2xy (2)
Given x2 − y2 = 10
⇒ u = 10

ww i.e., the image of the hyperbola x2 − y2 = 10 in the z-plane is mapped onto the line
u = 10 which is parallel to the v-axis which is at a distance 10 from it.

w.E
Example 3.95. Find the image of the region of the z-plane bounded by the line

asy
y = 0, x = 1 and y = x under the mapping w = z2 .
Solution. Under the mapping w = z2 , we have
u = x2 − y2 .
v = 2xy. En (1)
(2)

gin Y

The region bounded by y = 0, x = 1 and


y = x is the right angled triangle OAB ee rin
B
x=1
x

where A is (0, 1), B is (1, 1) and O is (0, 0).


=
y

g.n X

et
O y=0 A(0, 1)

(i) Consider y = 0.
The transformations (1) and (2) become
u = x2 and v = 0. v = 0 is the u-axis
and since 0 ≤ x ≤ 1, we have 0 ≤ u ≤ 1.
< >• u
Therefore y = 0 is mapped on the line O′ A′ (1, 0)

segment O′ A′ on the u-axis.

Downloaded From : www.EasyEngineering.net


Downloaded From : www.EasyEngineering.net

386 Engineering Mathematics - II

(ii) Consider x = 1.
v
(1) and (2) become u = 1 − y2 and v = 2y i.e., y = .
2
v2 v2
∴u=1− ⇒ =1−u
4 4
2
v = 4(1 − u) = −4(u − 1).

v
B′ (0, 2)
This is a parabola with vertex (1, 0) and

ww open to the left.


On x = 1, y varies form 0 to 1. ∴ u varies u

w.E
A′ (1, 0)

from 1 to 0 and v varies from 0 to 2.


∴ x = 1 is mapped onto the portion A′ B′

asy
of the parabola v2 = −4(u − 1).

En
(iii) Consider y = x where 0 ≤ x ≤ 1, 0 ≤ y ≤ 1.
(1) and (2) take the form u = 0 and v = 2x2 .

gin
As 0 ≤ x ≤ 1, 0 ≤ v ≤ 2.
ee
u = 0 is the v-axis and along this Varies
from 0 to 2.
v

rin
• B′ (0, 2)
>

g.n
>

∴ The image of the line y = x is the O′ (0, 0)


u

portion of the v-axis form O′ (0, 0) to


B′ (0, 2).

v
et
B′ (0, 2)
∴ The triangular region OAB in the z-
plane is mapped onto the region, O′ A′ B′
u
enclosed by u = 0, v = 0 and v2 = −4(u−1), O′ A′ (1, 0)

in the w-plane and this region is lying in


the first quadrant of the w-plane.

Downloaded From : www.EasyEngineering.net


Downloaded From : www.EasyEngineering.net

Analytic Functions 387

Example 3.96. Plot the image under the mapping w = z2 of the triangular region
bounded by y = 1, x = 1 and x + y = 1.
Solution. The region bounded by the line x = 1, y = 1 and x + y = 1 is the triangle
ABC where A(1, 0), B(0, 1) and C(1, 1).
From the transformation w = z2 , we have
u = x2 − y2 (1)
v = 2xy. (2)
To find the image of A(1, 0).

ww When x = 1, y = 0, u = 1, v = 0.
The image of A is A′ (1, 0).

w.E To find the image of B(0, 1).


When x = 0, y = 1, u = −1, v = 0.

asy
The image of B is B′ (−1, 0).
To find the image of C(1, 1)

En
When x = 1, y = 1, u = 0, v = 2.

gin
∴ The image of C is C ′ (0, 2).
(i) Consider y = 1.
(1) and (2) take the form
u = x2 − 1 and v = 2x ⇒ x =
v2
ee v
2
rin
∴u=
v2
4
−1
g.n
4
=u+1
⇒ v2 = 4(u + 1) et
which is a parabola with vertex B′ (−1, 0) and passing through C ′ (0, 2).
(ii) Consider x = 1
(1) and (2) take the form
v
u = 1 − y2 and v = 2y ⇒ y =
2
v2 v2
∴u=1− ⇒ =1−u
4 4

Downloaded From : www.EasyEngineering.net


Downloaded From : www.EasyEngineering.net

388 Engineering Mathematics - II

∴ v2 = 4(1 − u) = −4(u − 1).


It is a parabola whose vertex is A′ (1, 0) and open to the left. It passes
through C ′ (0, 2) and C1 (0, −2).
(iii) Consider x + y = 1.
(1) and (2) take the form
u = x2 − (1 − x)2 = x2 − 1 − x2 + 2x = 2x − 1
u+1
⇒ 2x = u + 1 or x =
2

ww v = 2x(1 − x)
u+1
!

w.E
= (u + 1) 1 −
2
!
2−u−1
= (u + 1)
2

asy =
(u + 1)(1 − u)
2

En
=
1 − u2
2
1 − u2 = 2v
gin
u2 = 1 − 2v

ee
1
= −2 v − .
2
!

rin
1
It is a parabola with vertex D( , 0) and passing through the points A′ (1, 0) and g.n
2
B′ (−1, 0). The required region is the shaded portion.

y
V
et
C1 (0, 2)

B (0, 1) C(1, 1)
 
1
D 2 ,0

B′ (−1, 0) A′ (1, 0)
x=1

U
x+
y
=
1

x C1 (0, −2)
O (0, 0) A(1, 0)

Downloaded From : www.EasyEngineering.net


Downloaded From : www.EasyEngineering.net

Analytic Functions 389

Example 3.97. By the map w = z2 , show that the straight lines through the origin
of the z-plane are mapped onto the straight lines through the origin of the w-plane.
Solution. The transformation w = z2 is equivalent to
u = x2 − y2 (1)
v = 2xy (2)
Let y = mx be te straight line passing through the origin.
(1) and (2) become
u = x2 − m2 x2 and v = 2mx2

ww u = x2 (1 − m2 ) and x2 =
v 2
(1 − m ) (or) v =
2m
v
2m

w.E
u= u.
2m 1 − m2
2m
This represents a straight line in the w-plane with slope and
1 − m2

asy
passing through the origin if m , ±1.
If m = 1, u = 0 and v = 2x2 > 0.

En
If m = −1, u = 0 and v = −2x2 < 0
and if m = −1, the image is the negative V-axis.

gin
∴ For all m, the image of a line through the origin in the z-plane is a

3.7.5 The transformation w = ez


ee
straight line through the origin in the w-plane.

rin
Let z = x + iy and w = reiθ . g.n
∴ w = ez becomes
reiθ = e x+iy = e x eiy .
Equating the modulii and amplitudes, we obtain
et
r = ex (1)
θ=y (2)
Discussion

Case(i). Image of lines parallel to the axes

Downloaded From : www.EasyEngineering.net


Downloaded From : www.EasyEngineering.net

390 Engineering Mathematics - II

Consider the line x = a, which is parallel to the y-axis.


∴ (1) ⇒ r = ea , which is a circle with center as origin and radius ea .
∴ Any line parallel to the y-axis in the z-plane is mapped onto a circle in the
w-plane.
Similarly, the line y = b, parallel to the x-axis has the image θ = b, which is a
radius vector in the w-plane.

ww y v

ea
w.E
r=
x u
O O′

asy
En
gin
y v

eey=b

rin
b
=
θ

x u
O′ O′

g.n
Case(ii). Image of a rectangular region
et
Consider the rectangular region bounded by the line a1 ≤ x ≤ a2 and
b1 ≤ y ≤ b2 . These are mapped onto circles ea1 ≤ r ≤ ea2 and radius vectors
b1 ≤ θ ≤ b2 . If D is the rectangular region bounded by a1 ≤ x ≤ a2 and
b1 ≤ y ≤ b2 is mapped onto the region R in the w-plane defined by
ea1 ≤ r ≤ ea2 , b1 ≤ θ ≤ b2 .

Downloaded From : www.EasyEngineering.net


Downloaded From : www.EasyEngineering.net

Analytic Functions 391

Y
r
x = a1 x = a2

D
y = b2

b2
=
e a2

θ
r=
R

e a1
r=
θ = b1
y = b1 U

ww
w.E
Case(iii). Consider the image of the x-axis.
i.e., when y = 0, we have θ = 0; when y = π, θ = π.

asy
i.e., The line y = 0 and y = π are mapped into the positive U-axis and the
negative U-axis respectively. The region bounded by y = 0 and y = π are

En
mapped into the upper half of the W-plane.

gin
Y

ee
y=π
V

rin
g.n
et
X U
y=0 θ=π θ=0

In the same way, the region bounded by the lines y = 0 and y = −π is mapped
into the entire lower half of the W-plane. Hence, the fundamental strip −π <
y ≤ π is mapped into the entire W-plane.

Downloaded From : www.EasyEngineering.net


Downloaded From : www.EasyEngineering.net

392 Engineering Mathematics - II

V
Y

θ = −π θ=0
y=0
U
X

y=π

ww There are an infinite number of strips of width 2π namely (2n − 1)π ≤ y ≤


(2n + 1)π are mapped into the whole w-plane. These strips of width 2π are

w.E called complementary strips.

asy y = 3π

complimentary strip

En X
y=π

gin
y=0
O
y = −π

complimentary strip

ee
y = −3π

This shows the many to one correspondence and the periodic nature of the
rin
the exponential function. The fundamental period is 2iπ. More generally,
each of the fundamental strips bounded by α ≤ y ≤ α + 2π where α is any g.n
constant is mapped into the entire w-plane.
Hence,
et
ez = ez+i2nπ , n is an integer.

From the relations r = e x ; θ = y, we notice that the imaginary axis x = 0 is


mapped into the unit circle, the right half plane x = Re(z) > 0 is mapped into
the outside of the unit circle and x = Re(Z) < 0 is mapped inside the unit
circle.

Downloaded From : www.EasyEngineering.net


Downloaded From : www.EasyEngineering.net

Analytic Functions 393

Application
This transformation can be used to obtain the circulation of a liquid around a
cylindrical obstacle, the electrostatic field due to a changed circular cylinder
etc.
✎ ☞
Worked Examples
✍ ✌
2
Example 3.98. Find the critical points of the map w = 1 + . [Dec 2013]
z
dw
Solution. Critical points are given by = 0.

ww w=1+
2
dz

= 1 + 2 · z−1

w.E dw
dz
z
= 0 gives

asy
2(−1)z−2 = 0
2

En
− =0
z2
− 2 = 0, which is not possible.

∴ w has no critical points. gin


Solution. Critical points are given by
ee
Example 3.99. Find the critical points of the map w =
dw
= 0.
2i − 6z
iz − 3
.

rin
i.e.,
(iz − 3)(−6) − (2i − 6z)i
dz

=0 g.n
(iz − 3)2
=⇒ −6iz + 18 + 2 + 6iz = 0. et
=⇒ 20 = 0 which is not possible.
Hence, there is no critical point for this transformation.

Example 3.100. For the conformal mapping w = z2 , find the scale factor at z = i.
Solution. f (z) = w = z2 .
f ′ (z) = 2z scale factor at (z = i) = | f ′ (i)| = |2i| = 2.

Downloaded From : www.EasyEngineering.net


Downloaded From : www.EasyEngineering.net

394 Engineering Mathematics - II

Exercise 3 E

1. Find the image of the region bounded by x = 0, y = 0, x = 1 and y = 2 under


the map w = z + 2 − i.

2. Find the region in the w−plane into which the rectangular region in the
z−plane bounded by the lines x = 0, y = 0, x = 3, x = 1 is mapped by the map
w = z + (3 + 2i).

ww 3. Find the image of the circle |z| = 1 under the map w = z + (2 + 2i).

4. Find the image of 2x + y − 3 = 0 under the transformation w = z + 2i.

w.E [May 2005]

asy
5. Determine the region of the w−plane into which the region in the z−plane
bounded by the lines x = 0, y = 0, y = 1, x = 1 is mapped under the

En
transformation w = (1 + i)z. [Dec 2004]

gin
6. Find the image of the circle |z| = 9 under the transformation w = 5z.
1
7. Find the image of |z + 1| = 1 under the mapping w = . [Apr 2006]

8. Under the transformation w = ,


1
ee
1
z
z

rin
determine the region in w−plane of the
1
infinite strip R bounded by ≤ y ≤
4 2
.

1 g.n [Jan 2005]

9. Find the line y − x + 1 = 0 under the mapping w = .


z
1
10. Find the mapping of |z − 3| = 5 under the transformation w = .
z
et [Apr 2006]

11. Find and draw the image of the infinite horizontal strip 2 < y < 4 under
1
w= . [Jan 2004]
z
12. Find the critical point of w = ez .

13. Find the critical point of w = z2 + az + b.

Downloaded From : www.EasyEngineering.net


146
Downloaded Supplement to Engineering Mathematics - II
From : www.EasyEngineering.net

2.3 Unit III Analytic Function

2.3.1 May/June 2016 [R 2013]


Part A

7. Give an example of a function where u and v are harmonic but


u + iv is not analytic.
Solution. Let u = x, v = −y.
u x = 1, uy = 0, v x = 0, vy = −1.
u xx = 0, uyy = 0, v xx = 0, vyy = 0.
ww u xx + uyy = 0, v xx + vyy = 0.

w.E ⇒ u and v are harmonic. But u x , vy .


∴ C. R. equations are not satisfied.
a syE
⇒ f (z) = u + iv is not analytic.

ngi
8. Find the critical points of the map w2 = (z − α)(z − β).
Solution. Given w2 = (z − α)(z − β)
Differentiating w.r.to z we get nee
2w
dw
rin
= (z − α) × 1 + (z − β) × 1 = z − α + z − β = 2z − (α + β)
dz
dw 2z − (α + β)
= . g.n
dz 2w

Critical points are given by


dw
= 0.
et
dz
2z − (α + β)
=0
2w
2z − (α + β) = 0
α+β
2z = α + β ⇒z= .
2
Part B

14. (a) (i) If u + !iv is an analytic function in z = x + iy then prove that


∂2 ∂2
2
+ 2 |u|2 = 2| f ′ (z)|2
∂x ∂y

Downloaded From : www.EasyEngineering.net


———-
Anna University
Downloaded Solved Question Papers
From : www.EasyEngineering.net 147

Solution. Refer Example 4.55 on page 368 of the main text


book.
z
14. (a) (ii) Prove that w = where a , 0 is analytic whereas
z+a

w= is not analytic.
z̄ + a
z
Solution. w = .
z+a
x + iy x + iy (x + iy)(x + a − iy)
u + iv = = = .
x + iy + a (x + a) + iy (x + a)2 + y2
x(x + a) + y2 + i(−xy + y(x + a)) x2 + ax + y2 + i(ay)
= = .

ww =
(x + a)2 + y2
x2 + ax + y2
+i
ay
.
(x + a)2 + y2

w.E 2
(x + a) + y
x2 + ax + y2
2 (x + a)2 + y2
ay
u=
a
(x + a)2 + y2
,
syE
v=
(x + a)2 + y2
.

ux =
2

ngi
(x + 2ax + a + y2 )(2x + a) − (x2 + ax + y2 )(2x + 2a)
2

(x2 + 2ax + a2 + y2 )2

= nee
2x(x2 + 2ax + a2 + y2 − x2 − ax − y2 ) + a(x2 + 2ax + a2 + y2 − 2x2 − 2ax − 2y2 )

2ax2 + 2a2 x − ax2 − ay2 + a3 rin


(x2 + 2ax + a2 + y2 )2

=
(x2 + 2ax + a2 + y2 )2
g.n
=
ax2 + 2a2 x − ay2 + a3
(x2 + 2ax + a2 + y2 )2
a(x2 + 2ax − y2 + a2 )
et
= .
(x2 + 2ax + a2 + y2 )2

(x2 + 2ax + a2 + y2 )(2y) − (x2 + ax + y2 )(2y)


uy =
(x2 + 2ax + a2 + y2 )2
2y(x + 2ax + a2 + y2 − x2 − ax − y2 )
2
=
(x2 + 2ax + a2 + y2 )2
2y(ax + a2 )
= 2
(x + 2ax + a2 + y2 )2
2ay(x + a)
= 2 . (2)
(x + 2ax + a2 + y2 )2

Downloaded From : www.EasyEngineering.net


———-
148
Downloaded Supplement to Engineering Mathematics - II
From : www.EasyEngineering.net

−ay(2x + 2a)
vx =
(x2
+ 2ax + a2 + y2 )2
−2ay(x + a)
= 2 . (3)
(x + 2ax + a2 + y2 )2

(x2 + 2ax + a2 + y2 )a − ay(2y)


vy =
(x2 + 2ax + a2 + y2 )2
a(x + 2ax + a2 + y2 − 2y2 )
2
=
(x2 + 2ax + a2 + y2 )2
a(x2 + 2ax + a2 − y2 )
= 2 . (4)
(x + 2ax + a2 + y2 )2

ww From (1) and (4) we find u x = vy

w.E
From (2) and (3) we find uy = −v x
∴ C.R. equations are satisfied.
∴ f (z) =

Now w =
a z
z+a
z̄ syE
is analytic.

z̄ + a
ngi
u + iv ==
x − iy
=
x − iy
x − iy + a (x + a) − iy
=
nee
(x − iy)(x + a + iy)
(x + a)2 + y2
.

= =rin
x(x + a) + y2 + i(xy − y(x + a)) x2 + ax + y2 − i(ay)
.
(x + a)2 + y2
x2 + ax + y2 −ay g.n
(x + a)2 + y2

= 2
(x + a) + y

x2 + ax + y2
2
+i
(x + a)2 + y2
.

−ay
et
u= , v= .
(x + a)2 + y2 (x + a)2 + y2
As before we get

a(x2 + 2ax − y2 + a2 )
ux = . (5)
(x2 + 2ax + a2 + y2 )2
2ay(x + a)
uy = 2 . (6)
(x + 2ax + a2 + y2 )2
2ay(x + a)
vx = 2 . (7)
(x + 2ax + a2 + y2 )2
−a(x2 + 2ax + a2 − y2 )
vy = . (8)
(x2 + 2ax + a2 + y2 )2

Downloaded From : www.EasyEngineering.net


———-
Anna University
Downloaded Solved Question Papers
From : www.EasyEngineering.net 149

From (5), (6), (7) and (8) we note that C.R. equation are not
satisfied

∴w= is not analytic.
z̄ + a

y
14. (b) (i) Can v = tan−1 be the imaginary part of an analytic
x
function? If so construct an analytic function f (z) = u + iv,
taking v as the imaginary part and hence find u.
y
Solution. Given v = tan−1 .
x
ww vx =
1
y2
1 + x2
 −y
x 2

= 2
x
−y
+ y2
.

w.E v xx = −y(−1)(x2 + y2 )−2 .2x = 2


! (x
2xy
+ y2 )2
.

a vy =
1
syE
y2 x
1 + x2
1
= 2
x +
x
y2
.

ngi
vyy = x(−1)(x2 + y2 )−2 .2y = 2
−2xy
(x + y2 )2
.
v xx + vyy = 0.
∴ v is harmonic. nee
rin
Since f is analytic, v can be the imaginary part of f .
By Milne Thomson method.
g.n
f ′ (z) = vy (z, 0) + iv x (z, 0) = zz2 + i(0) = 1z .
R
∴ f (z) = 1z dz + c = log z + c = log(x + iy) + c
 
et
u + iv = 12 log(x2 + y2 ) + i tan−1 yx + c.
1
∴u= 2 log(x2 + y2 ), which is the real part of f (z).

14. (b) (ii) Find the bilinear transformation that transforms the
points z = 1, i, −1of the z−plane into the points w = 2, i, −2 of the
w−plane.
Solution. Refer Example 4.65 on page 378 of the main text
book.

Downloaded From : www.EasyEngineering.net


———-
150
Downloaded Supplement to Engineering Mathematics - II
From : www.EasyEngineering.net

2.3.2 Dec.2015/Jan. 2016 [R 2013]


Part A

7. Prove that the family of curves u = c, v = k cuts orthogonally


for an analytic function f (z) = u + iv.
Solution. Refer property 2 on page 337 of the main text book.
z3 + 7z
8. Find the invariant points of the function f (z) = .
7 − 6zi
Solution. The invariant points are givey by f (z) = z.
z3 + 7z
i.e., =z
ww 7 − 6zi
z + 7z = 7z − 6z2 i
3

w.E z3 + 6z2 i = 0
z2 (z + 6i) = 0 ⇒ z = 0, z = −6i.

Part B a syE
2 2
14. (a) (i) If u = x − y , v =
y
x2 + y2 ngi
, prove that u and v are harmonic

nee
functions but f (z) = u + iv is not an analytic function.
Solution. Refer Example 4.43 on page 355 of the main text
book. rin
 
g.n
14. (a) (ii) Show that the function u = e−2xy sin x2 − y2 is a real part
of an analytic function. Also find its conjugate harmonic
function v and express f (t) = u + iv as function of z.
et
 
Solution. u = e−2xy sin x2 − y2 .
   
u x = e−2xy cos x2 − y2 (2x) + sin x2 − y2 e−2xy (−2y)
   
= 2xe−2xy cos x2 − y2 − 2ye−2xy sin x2 − y2 .
   
u xx = 2xe−2xy (− sin x2 − y2 )(2x) + 2xe−2xy (−2y) cos x2 − y2
   
+ 2e−2xy cos x2 − y2 − 2ye−2xy (cos x2 − y2 (2x)
 
− 2ye−2xy (−2y) sin x2 − y2
   
= −4x2 e−2xy (sin x2 − y2 − 4xye−2xy cos x2 − y2

Downloaded From : www.EasyEngineering.net


———-
Anna University
Downloaded Solved Question Papers
From : www.EasyEngineering.net 151

   
+ 2e−2xy cos x2 − y2 − 4xye−2xy (cos x2 − y2
 
+ 4y2 e−2xy sin x2 − y2 (1)
   
uy = e−2xy cos x2 − y2 (−2y) + sin x2 − y2 e−2xy (−2x)
   
= −2ye−2xy cos x2 − y2 − 2xe−2xy sin x2 − y2 .
   
uyy = −2ye−2xy (− sin x2 − y2 )(−2y) − 2ye−2xy (−2x) cos x2 − y2
   
− 2e−2xy cos x2 − y2 − 2xe−2xy (cos x2 − y2 (−2y)
 
− 2xe−2xy (−2x) sin x2 − y2
   
= −4y2 e−2xy sin x2 − y2 + 4xye−2xy cos x2 − y2

ww   
− 2e−2xy cos x2 − y2 + 4xye−2xy (cos x2 − y2


w.E 
+ 4x2 e−2xy sin x2 − y2

(2)

a
(1) + (2) ⇒ u xx + uyy = 0.

⇒ u is harmonic.
syE
ngi
Since f is analytic, u can be the real part of f .
By Milne Thomson method.
nee
f ′ (z) = u x (z, 0) − iuy (z, 0) rin
= 2z cos z2 − i(−2z sin z2 ) g.n
2
= 2z cos z + i2z sin z
2 2
= 2z(cos z + i sin z ) = 2ze
2

iz2
et
Z
2
f (z) = eiz 2zdz + c. Let t = z2 dt = 2zdz.
Z
eit 2 2 2 2
= eit dt + c = = −ieiz = −iei(x+iy) = −iei(x −y +2ixy)
i
2 2
h    i
= −iei(x −y )−2xy = −ie−2xy cos x2 − y2 + i sin x2 − y2
   
u + iv = e−2xy sin x2 − y2 − ie−2xy cos x2 − y2 .
 
∴ v = −e−2xy cos x2 − y2 .
2
Also f (z) = −ieiz .

Downloaded From : www.EasyEngineering.net


———-
152
Downloaded Supplement to Engineering Mathematics - II
From : www.EasyEngineering.net

14. (b) (i) Is f (z) = zn analytic function everywhere?


Solution. Refer Example 4.7 on page 325 of the main text
book.

14. (b) (ii) Find the image of the line u = a and v = b in w−plane into

z−plane under the transformation z = w.

Solution. The given transformation is z = w.
i.e., w = z2
u + iv = (x + iy)2 = x2 − y2 + 2ixy.
∴ u = x2 − y2 , v = 2xy.
ww u=a ⇒ x2 − y2 = a, v=b 2xy = b ⇒ xy = b2 .

w.E
Both are rectangular parabolas in the z− plane.
Hence, the lines u = a and v = b in the w−plane are mapped

a syE
onto rectangular hyperbolas in the z−plane.

ngi
14. (b) (iii) Find the bilinear transformation which maps i, −i, 1 in
z−plane into 0, 1, ∞ of the w−plane respectively.
Solution. Given, z1 = i, z2 = −i, z3 = 1. nee
w1 = 0, w2 = 1, w3 = ∞.
rin
The bilinear transformation is
g.n
w − w1
=
(z − z1 )(z2 − z3 )
w2 − w1 (z − z3 )(z2 − z1 )
w − 0 (z − i)(−i − 1)
et
=
1−0 (z − 1)(−i − i)
(z − i)(−i − 1)
w=
(z − 1)(−2i)
(z − i)(−i − 1)i
w=
2(z − 1)
(z − i)(1 − i)
w= .
2(z − 1)

2.3.3 April/May 2015 [R 2013]


Part A

Downloaded From : www.EasyEngineering.net


———-
Anna University
Downloaded Solved Question Papers
From : www.EasyEngineering.net 153

7. Show that |z|2 is not analytic at any point.


Solution. f (z) = u + iv = |z|2 = x2 + y2 .
∴ u = x2 + y2 v = 0.
u x = 2x v x = 0.
uy = 2y vy = 0.
At (0, 0) u x = vy = 0 and uy = −v x = 0.
⇒ f (z) is differentiable only at the origin.
But for all (x, y) , (0, 0), C-R equations are not satisfied.
Therefore, f (z) is not analytic.

ww8. Find the invariant points of the transformation w =


z−1
z+1
.

w.E
Solution. The invariant points are given by w = z
z−1

a
i.e.,
z+1
=z

z(z + 1) = z − 1 syE
z2 + z − z + 1 = 0 ngi
2
z +1=0
nee
z2 = −1 ⇒ z = ±i.
rin
Part B g.n
14. (a) (i) Determine the analytic function w
2x
u = e (x cos 2y − y sin 2y).
= u + iv ifet
Solution. Let w = f (z) = u + iv be the analytic function.
Given u = e2x (x cos 2y − y sin 2y) is the real part of f (z).

u x = e2x cos 2y + 2e2x (x cos 2y − y sin 2y)


= e2x (cos 2y + 2x cos 2y − 2y sin 2y).
u x (z, 0) = e2z (1 + 2z).
 
uy = e2x x(−2 sin 2y) − [y(cos 2y) · 2 + sin 2y]
uy = e2x (−2x sin 2y − 2y cos 2y − sin 2y)

Downloaded From : www.EasyEngineering.net


———-
154
Downloaded Supplement to Engineering Mathematics - II
From : www.EasyEngineering.net

uy (z, 0) = e2z [0] = 0.


By Miline Thomson method
f ′ (z) = u x (z, 0) − iuy (z, 0) = e2z (1 + z).
Integrating w.r.t Z we get
Z
f (z) = e2z (1 + z)dz + c
Z !
ez
= (1 + z)d +c
2
Z
e2z e2z
= (1 + z) − dz + c

ww = (1 + z)
2
e2z e2z

2

dz + c
w.E =
e2z
" 2
(1 + z) −
4#
1
+c

a =
2
e2z 1
2 2
"
syE #
+ z + c.
2

ngi
nee
14. (a) (ii) Show that a harmonic function ′ u′ satisfies the formal
∂2 u
differential equation
∂z∂z̄
is harmonic, where f (z) is a regular function. rin
= 0 and hence prove that log | f ′ (z)|

Solution. Given u is harmonic. g.n


∴ u satisfies the Laplace equation
∂2 u ∂2 u
+ = 0.
et
(1)
∂x2 ∂y2
we have z = x + iy, z̄ = x − iy.
z+z̄ z−z̄
Hence, x = 2 , y= 2i .
⇒ u is a function of z and z̄.

∂u ∂u ∂x ∂u ∂y
Now = · + ·
∂z̄ ∂x ∂z̄ ∂y ∂z̄
! !
∂u 1 ∂u −1
= · + ·
∂x 2 ∂y 2i
!
1 ∂u 1 ∂u
= −
2 ∂x i ∂y

Downloaded From : www.EasyEngineering.net


———-
Anna University
Downloaded Solved Question Papers
From : www.EasyEngineering.net 155

! " #
1 ∂u ∂u 1
= +i since, = −i
2 ∂x ∂y i
 !
∂2 u ∂ 1
= u x + iuy
∂z∂z̄ ∂z 2
"  ∂y #
1 ∂   ∂x ∂ 
= u x + iuy · + u x + iuy ·
2 ∂x ∂z ∂y ∂z
"    #
1 1 1
= u xx + iu xy + uyx + iuyy
2 2 2i
" #
1   1
= u xx + iu xy + uyx + iuyy
4 i
h i !
1 1

ww ∂2 u
=
4
u xx + uyy + iu xy − iu xy since, = −i, uyx = u xy
i

4
w.E
∂z∂z̄
= u xx + uyy .(2)

a Since, u is harmonic, u xx + uyy = 0, which implies



∂2 u
∂z∂z̄
syE
= 0.

14. (b) (i) Find the image in the w−plane ngi of the infinite strip
1
4
1
≤ y ≤ under the transformation w =
2 nee 1
z
.

Solution. Given , w =
1
z rin
⇒x=
u
,y = 2
−v g.n
u2 + v2
1
Consider ≤ y
4
u + v2
et
1 −v
⇒ ≤ 2
4 u + v2
i.e., u2 + v2 ≤ −4v
i.e., u2 + v2 + 4v ≤ 0.

This is a circle with centre (0, −2) and radius 2.

1
Consider y ≤
2
−v 1
⇒ 2 ≤
u + v2 2

Downloaded From : www.EasyEngineering.net


———-
156
Downloaded Supplement to Engineering Mathematics - II
From : www.EasyEngineering.net

i.e., −2v ≤ u2 + v2
i.e., u2 + v2 + 2v ≥ 0.

This is a circle with centre (0, −1) and radius 1.


The original and the image of the given transformation are
given below.

ww (0, 2)
(1/2, 0)
U
w.E x= 1
4 x= 1
2
(1/4, 0)

a syE
O
X O

ngi
14. (b) (ii) Find the bilinear transformation that maps the points z =
0, −1, i into the points w = i, 0, ∞ respectively.
Solution. Given, z1 = 0, z2 = −1, z3 = i. nee
w1 = i, w2 = 0, w3 = ∞. rin
The bilinear transformation is g.n
w − w1
=
w2 − w1
w−i
(z − z1 )(z2 − z3 )
(z − z3 )(z2 − z1 )
(z − 0)(−i − 1)
et
=
0−i (z − i)(−1 − 0)
w−i z(1 + i)
=−
−i (z − i)(−1)
w−i z(1 + i)
=
−i z−i
−iz(1 + i) −iz + z
w−i= =
z−i z−i
−iz + z iz + 1 − iz + z
w=i+ =
z−i z−i
1+z
w= .
z−i

Downloaded From : www.EasyEngineering.net


———-
Anna University
Downloaded Solved Question Papers
From : www.EasyEngineering.net 157

which is a straight line in the w−plane.

2.3.4 November/December 2014 [R 2013]

Part A

7. Verify f (z) = z3 is analytic or not.


Solution. f (z) = u+iv = z3 = (x+iy)3 = x3 +3(x2 )(iy)+3(x)(iy)2 +(iy)3 .

ww u + iv = x3 + 3ix2 y − 3xy2 − iy3 .


∴ u = x3 − 3xy2 , v = 3x2 y − y3 .

w.E u x = 3x2 − 3y2 , v x = 6xy.


vy = 3x2 − 3y2 .
a uy = −6xy,

syE
we observe that u x = vy and uy = −v x .

ngi
C-R equations are satisfied.
∴ f (z) is an entire function.

nee
8. Find the critical points of the transformation w2 = (z − α)(z − β).
Solution. w2 = (z − α)(z − β) rin
Differentiating w.r.to z. g.n
2w
dw
dz
= (z − α) × 1 + (z − β) × 1 et
=z−α+z−β
= 2z − (α + β)
dw 2z − (α + β)
= .
dz 2w
dw
Critical points are given by = 0.
dz
2z − (α + β)
=0
2w
2z − (α + β) = 0

Downloaded From : www.EasyEngineering.net


———-
158
Downloaded Supplement to Engineering Mathematics - II
From : www.EasyEngineering.net

2z = α + β
α+β
z= .
2

Part B

14. (a) (i) If f (z)


! is an analytic function of z, prove that
∂2 ∂2
+ | f (z)|2 = 4| f ′ (z)|2 .
∂x2 ∂y2
Solution. Given f (z) is analytic.
Let f (z) = u + iv.

ww ⇒ u and v have continuous partial derivatives and they satisfy

w.E
C-R equations.
∴ u x = vy and uy = −v x and f ′ (z) = u x + iv x , | f ′ (z)|2 = u2x + v2x .

a syE
Since u and v are harmonic functions, they satisfy Laplace
equation.

ngi
The complex form of the Laplace operator is

∂ 2
+
∂2
∂x2 ∂y2
= 4
∂2
∂z̄∂z
. nee
! rin
Now,
∂2
+
∂2
∂x2 ∂y2
| f (z)|2
= 4
∂2
∂z̄∂z
| f (z)|2
= 4
∂2
∂z̄∂z
( f (z) f (z)). g.n
Since f (z) is an analytic function, it is independent of z̄. et
i.e., f (z) is a function of z only.
Similarly its conjugate f (z) is an analytic function of z̄ only.
∴ We can denote f (z) by f¯(z̄).
i.e., f (x + iy) = f¯(x − iy).
!
∂2 ∂2 ∂ ∂
Hence, + | f (z)|2 = 4 ( f (z) f¯(z̄))
∂x2 ∂y2 ∂z̄ ∂z
∂ ∂
= 4 ( f¯(z̄)) ( f (z))
∂z̄ ∂z
= 4 f ′ (z) f ′ (z) = 4| f ′ (z)|2 .

Downloaded From : www.EasyEngineering.net


———-
Anna University
Downloaded Solved Question Papers
From : www.EasyEngineering.net 159

14. (a) (ii) Find the bilinear transformation which maps the points
z = 1, i, −1 onto the points w = i, 0, −i.
Solution. Given, z1 = 0, z2 = i, z3 = −1.
w1 = i, w2 = 0, w3 = −i.
The bilinear transformation is

(w − w1 )(w2 − w3 ) (z − z1 )(z2 − z3 )
=
(w − w3 )(w2 − w1 ) (z − z3 )(z2 − z1 )
(w − i)(0 + i) (z − 1)(i + 1)
=
(w + i)(0 − i) (z + 1)(i − 1)
(w − i)(i) (z − 1)(i + 1)
ww (w + i)(−i)
=
(z + 1)(i − 1)

w.E (w − i)
(w + i)
(w − i) + (w + i)
=
(1 − z)(i + 1)
(z + 1)(i − 1)
(1 − z)(i + 1) + (z + 1)(i − 1)
a
(w − i) − (w + i)
w−i+w+i
=
syE
=
(1 − z)(i + 1) − (z + 1)(i − 1)
i − zi + 1 − z + zi + i − z − 1
w−i−w−i
2w ngi
i − zi + 1 − z − zi − i + z + 1
2i − 2z
−2i
w
=
−2zi + 2
i−z nee
−i
=
1 − zi
rin
w=
zi + 1
1 − zi
.
g.n
14. (b) (i) Prove that u = x2 − y2 and v = −
y et
are harmonic
x + y2
2
functions but not harmonic conjugates.
Solution. u = x2 − y2 .

u x = 2x.
u xx = 2.
uy = −2y.
uyy = −2.
u xx + uyy = 2 − 2 = 0.

Downloaded From : www.EasyEngineering.net


———-
160
Downloaded Supplement to Engineering Mathematics - II
From : www.EasyEngineering.net

∴ u is harmonic.

y
v=− = −y(x2 + y2 )−1 .
x2 + y2
2xy
v x = −y(−1)(x2 + y2 )−2 · 2x = 2 .
(x + y2 )2
" 2 #
(x + y2 )2 − x2(x2 + y2 ) · 2x
v xx = 2y
(x2 + y2 )4
2y(x2 + y2 )[x2 + y2 − 4x2 ]
=
(x2 + y2 )4
2y(y2 − 3x2 )
v xx = .

ww vy = −
(x2 + y2 )3
x2 + y2 − y · 2y

w.E
=− 2
(x2 + y2 )2
x2 − y2
= 2
y2 − x2
.

vyy =
a(x + y ) 2 2

syE
(x + y2 )2
(x2 + y2 )2 · 2y − (y2 − x2 ) · 2(x2 + y2 ) · 2y
(x2 + y2 )4
.

= ngi
2y(x2 + y2 )[x2 + y2 − 2(y2 − x2 )]
(x2 + y2 )4
2y(x2 + y2 − 2y2 + 2x2 ) nee
=
(x2 + y2 )3
rin
vyy =
2y(3x2 − y2 )
(x2 + y2 )3
.
g.n
v xx + vyy = 2
2y
(x + y2 )3
[y2 − 3x2 + 3x2 − y2 ] = 0. et
∴ v is harmonic.
We have to prove that, if u + iv is regular then u + iv must satisfy
C-R equations.
But here, u x = 2x , vy and uy = −2y , −v x .
∴ C-R equations are not satisfied.
∴ u and v are not harmonic conjugates.
sin 2x
14. (b) (ii) Given that u = , find the analytic function
cosh 2y − cos 2x
f (z) = u + iv. (8)

Downloaded From : www.EasyEngineering.net


———-
Anna University
Downloaded Solved Question Papers
From : www.EasyEngineering.net 161

2.3.5 May/June 2014 [R 2013]

Part A

7. Is the function f (z) = z̄ analytic?


Solution. Let f (z) = u + iv = z̄ = x − iy.
∴ u = x, v = −y
u x = 1, vx = 0
uy = 0, vy = −1.
Since u x , vy , C.R equations are not satisfied.

ww ∴ f (z) is not analytic anywhere.

w.E =⇒ f (z) is nowhere differentiable.

8. Find the invariant points of f (x) = z2 .


a syE
Solution. Invariant points are given by f (z) = z
i.e., z2 = z ⇒ z2 − z = 0 ⇒ z(z − 1) = 0 ⇒ z = 0, z = 1.
ngi
Part B
nee
rin
14. (a) (i) Prove that the real and imaginary parts of an analytic
function are harmonic functions.
g.n
Solution. Given f (z) is analytic.
⇒ u and v have first partial derivatives and satisfy C-R et
equations in D.
u x = vy and uy = −v x
∂u ∂v
i.e., = (1)
∂x ∂y
∂u ∂v
and =− (2)
∂y ∂x
Differentiating (1) w.r.t x we get

∂2 u ∂2 v
= (3)
∂x2 ∂x∂y

Downloaded From : www.EasyEngineering.net


———-
162
Downloaded Supplement to Engineering Mathematics - II
From : www.EasyEngineering.net

Differentiating (2) w.r.t y we get

∂2 u ∂2 v
= − (4)
∂y2 ∂y∂x
Since u x , uy , v x , vy are continuous, we have

∂2 v ∂2 v
=
∂x∂y ∂y∂x

∂2 u ∂2 u
∴ (3) + (4) ⇒ + = 0.
∂x2 ∂y2
∴ u is harmonic.

ww Differentiating (1) w.r.t y and (2) w.r.t x we get

w.E ∂2 u ∂2 v
= 2 and
∂y∂x ∂y
∂2 u
∂x∂y
∂2 v
=− 2
∂x

a ∴
syE
∂2 v
∂y2
= −
∂2 v
∂x2

∂2 v ∂2 v
+
∂x2 ∂y2
= 0.

⇒ v is harmonic.
ngi
nee
14. (a) (ii) Find the bilinear transformation that maps 1, i and −1 of
the z−plane onto 0, 1 and ∞ of the w−plane.
Solution. Let z1 = 1, z2 = i, z3 = −1. rin
w1 = 0, w2 = 1, w3 = ∞. g.n
The bilinear transformation is
w − w1 (z − z1 )(z2 − z3 )
et
=
w2 − w1 (z − z3 )(z2 − z1 )
w − 0 (z − 1)(i + 1)
=
1−0 (z + 1)(i − 1)
z−11+i 1+i
w=− ×
z+11−i 1+i
z − 1 (1 + i)2
=−
z+1 1+1
!
z − 1 1 − 1 + 2i
=−
z+1 2
z−1
= −i
z+1

Downloaded From : www.EasyEngineering.net


———-
Anna University
Downloaded Solved Question Papers
From : www.EasyEngineering.net 163

w(z + 1) = −iz + i
wz + w = −iz + i
wz + iz = i − w
i−w
z(w + i) = i − w ⇒z= .
w+i

14. (b) (i) Show that v = e−x (x cos y + y sin y) is a harmonic function.
Hence find the analytic function f (z) = u + iv.
Solution.We first prove that v xx + vyy = 0.

ww v x = e−x (cos y) + (x cos y + y sin y)(−e−x )

w.E = e−x (cos y − x cos y − y sin y)


v xx = e−x (− cos y) − e−x (cos y − x cos y − y sin y)

a syE
= e−x (− cos y − cos y + x cos y + y sin y)
= e−x (−2 cos y + x cos y + y sin y)
ngi
vy = e−x (−x sin y + y cos y + sin y)
nee
vyy = e−x (−x cos y − y sin y + cos y + cos y)
= e−x (2 cos y − x cos y − y sin y).rin
v xx + vyy = 0. ∴ v is harmonic g.n
−z
Now, v x (z, 0) = e (1 − z)
−z
and vy (z, 0) = e 0 = 0.
et
By Milne Thomson method,
f ′ (z) = vy (z, 0) + iv x (z, 0) = ie−z (1 − z).
Integrating we get,
Z Z
f (z) = i e−z (1 − z)dz + c = i (1 − z)d(−e)−z dz + c
Z " Z #
= i (z − 1)d(e)−z + c = i (z − 1)e−z − e−z dz + c

= i (z − 1)e−z + e−z + c = ie−z (z − 1 + 1) + c

Downloaded From : www.EasyEngineering.net


———-
164
Downloaded Supplement to Engineering Mathematics - II
From : www.EasyEngineering.net

f (z) = ize−z + c.

1
14. (b) (ii) Find the image of |z + 1| = 1 under the map w = .
z
1
Solution. w = .
z
u v
⇒x= ,y=− 2 .
+vu22 u + v2
The given figure is

|z + 1| = 1
i.e., |x + iy + 1| = 1

ww i.e., (x + 1)2 + y2 = 1

w.E
x2 + y2 + 2x + 1 = 1

a
x + y2 + 2x = 0
2

u2
+
v2
(u2 + v2 )2 (u2 + v2 )2
syE
+2 2
u
u + v2
=0

u2 + v2
+
2u
=0 ngi
(u2 + v2 )2 u2 + v2
1 + 2u nee
u2 + v2
= 0 ⇒ 2u + 1 = 0
rin
which is a straight line in the w−plane.
g.n
2.3.6 November/December 2013[R 2008] et
Part A

7. Find the constants a, b if f (z) = x + 2ay + i(3x + by) is analytic.


Solution. f (z) = x + 2ay + i(3x + by).
u = x + 2ay, v = 3x + by.
ux = 1 vx = 3
uy = 2a vy = b.
Since f (z) is analytic, C-R equations are satisfied.
∴ u x = vy and uy = −v x .

Downloaded From : www.EasyEngineering.net


———-
Anna University
Downloaded Solved Question Papers
From : www.EasyEngineering.net 165

Taking u x = vy we get b = 1.
3
Taking uy = −v x , we get 2a = −3 ⇒ a = − .
2
2
8. Find the critical points of the map w = 1 + .
z
dw
Solution. Critical points are given by = 0.
dz
2
w = 1 + = 1 + 2 · z−1
z
dw
= 0 gives
dz
2(−1)z−2 = 0
2

ww ⇒− 2 =0
z
⇒ −2 = 0 which is not possible.

w.E ∴ w has no critical points.

Part B
a syE
14. (a) (i) Prove that u = e−2xy sin(x2 − y2 ) is harmonic. Find the
ngi
corresponding analytic function and the imaginary part.
Solution. u = e−2xy sin(x2 − y2 )
nee
rin
u x = e−2xy cos(x2 − y2 ) · 2x + sin(x2 − y2 ) · e−2xy (−2y)
h i
= 2e−2xy x cos(x2 − y2 ) − y sin(x2 − y2 ) . g.n
h
u xx = 2e−2xy −x sin(x2 − y2 ) · 2x + cos(x2 − y2 ) − y cos(x2 − y2 ) · 2x
h i
+ x cos(x2 − y2 ) − y sin(x2 − y2 ) 2e−2xy (−2y)
i
et
h
u xx = 2e−2xy −2x2 sin(x2 − y2 ) + cos(x2 − y2 ) − 2xy cos(x2 − y2 )
i
−2xy cos(x2 − y2 ) + 2y2 sin(x2 − y2 ) .
uy = e−2xy cos(x2 − y2 )(−2y) + sin(x2 − y2 )e−2xy (−2x)
h i
= −2e−2xy y cos(x2 − y2 ) + x sin(x2 − y2 ) .
h i
uyy = −2e−2xy −y sin(x2 − y2 )(−2y) + cos(x2 − y2 ) + x sin(x2 − y2 )(−2y)
h i
+ y cos(x2 − y2 ) + x sin(x2 − y2 ) (−2e−2xy )(−2x)
h
= −2e−2xy 2y2 sin(x2 − y2 ) + cos(x2 − y2 ) − 2xy sin(x2 − y2 )

Downloaded From : www.EasyEngineering.net


———-
166
Downloaded Supplement to Engineering Mathematics - II
From : www.EasyEngineering.net

i
−2xy cos(x2 − y2 ) − 2x2 sin(x2 − y2 )
h
= 2e−2xy −2y2 sin(x2 − y2 ) − cos(x2 − y2 ) + 2xy sin(x2 − y2 )
i
+2xy cos(x2 − y2 ) + 2x2 sin(x2 − y2 ) .

Now, u xx + uyy = 0.
∴ u is harmonic.
Now u x (z, 0) = 2z cos z2 and uy (z, 0) = −2z sin z2 .
By Milne Thomson method

ww f ′ (z) = u x (z, 0) + iuy (z, 0)


= 2z cos z2 + 2iz sin z2
w.E = 2z(cos z2 + i sin z2 ) = 2zeiz
2

a syE
Integrating w.r.t.z we get
Z
f (z) =
2
2zeiz dz + c
ngi t = z2

=
Z
eit dt + c nee
dt = 2zdz

=
eit
+c rin
i
1 2 g.n
= eiz + c
i
2
f (z) = −ieiz + c
et
2
u + iv = −iei(x+iy) + c1 + ic2
2
−y2 +2ixy)
= −iei(x + c1 + ic2
2 2
= −iei(x −y )−2xy
+ c1 + ic2
2 2
= −iei(x −y )
· e−2xy + c1 + ic2
= −ie−2xy [cos(x2 − y2 ) + i sin(x2 − y2 )] + c1 + ic2
= −ie−2xy cos(x2 − y2 ) + e−2xy sin(x2 − y2 )

Equating the imaginary parts we get v = −e−2xy cos(x2 − y2 ).

Downloaded From : www.EasyEngineering.net


———-
Anna University
Downloaded Solved Question Papers
From : www.EasyEngineering.net 167

14. (a) (ii) Find the bilinear map which maps the points z = 0, −1, i
onto the points w = i, 0, ∞. Also find the image of the unit circle
of the z−plane.
Solution. Given, z1 = 0, z2 = −1, z3 = i.
w1 = i, w2 = 0, w3 = ∞.
The bilinear transformation is

w − w1 (z − z1 )(z2 − z3 )
=
w2 − w1 (z − z3 )(z2 − z1 )
w−i (z − 0)(−i − 1)
=

ww 0−i
w−i
=−
(z − i)(−1 − 0)
z(1 + i)

w.E −i
w−i
=
(z − i)(−1)
z(1 + i)

a −i
w−i=
z−i

syE
−iz(1 + i) −iz + z
z−i
=
z−i
w=i+
z−i
= ngi
−iz + z iz + 1 − iz + z
z−i
w=
1+z
z−i
. nee
w(z − i) = 1 + z rin
wz − wi = 1 + z g.n
wz − z = 1 + wi
z(w − 1) = 1 + wi
et
1 + wi
z= .
w−1
Given |z| = 1

1 + wi = 1
w−1
|1 + wi|
=1
|w − 1|
|1 + (u + iv)i| = |u + iv − 1|
|1 + iu − v|2 = |(u − 1) + iv|2

Downloaded From : www.EasyEngineering.net


———-
168
Downloaded Supplement to Engineering Mathematics - II
From : www.EasyEngineering.net

(1 − v)2 + u2 = (u − 1)2 + v2
1 + v2 − 2v + u2 = u2 + 1 − 2u + v2
−2v = −2u
u = v,

which is a straight line in the w−plane.


z
14. (b) (i) Prove that w = maps the upper half of the z−plane to
1−z
the upper half of the w−plane and also find the image of the
unit circle of the z−plane.
ww Solution.

w.E
Given w =
z
1−z

a
u + iv =
x + iy
syE
1 − x − iy
x + iy (1 − x) + iy
= ×
ngi
(1 − x) − iy (1 − x) + iy

=
(1 − x)2 + y2 nee
x(1 − x) − y2 + i[y(1 − x) + xy]

=
x(1 − x) − y2
+i
y(1 − x) + xy
rin
.
(1 − x)2 + y2
x(1 − x) − y2
(1 − x)2 + y2
g.n
∴u=

v=
(1 − x)2 + y2
y(1 − x) + xy
.
and
et
(1 − x)2 + y2
Let v > 0.

y(1 − x) + xy
⇒ >0
(1 − x)2 + y2
y − xy + xy > 0
y > 0.

This shows that the upper half of the z−plane is mapped onto
the upper half of the w−plane.

Downloaded From : www.EasyEngineering.net


———-
Anna University
Downloaded Solved Question Papers
From : www.EasyEngineering.net 169

To find the image of the unit circle.


z
we have w =
1−z
w(1 − z) = z
w − wz = z
w = wz + z
z(1 + w) = w
w
z= .
1+w

ww Given |z| = 1.



w

w.E =1
1 + w
|w| = |1 + w|
a syE
|w|2 = |1 + w|2
|u + iv|2 = |1 + u + iv|2
u2 + v2 = (1 + u)2 + v2
ngi
= 1 + u2 + 2u + v2 nee
1 rin
0 = 2u + 1 ⇒u=− .
2
g.n 1
The image of the unit circle |z| = 1 is the straight line u = − .
2 et
14. (b) (ii) Find the analytic function f (z) = u+iv where v = 3r2 sin 2θ−
2r sin θ. Verify that u is a harmonic function.
Solution. v = 3r2 sin 2θ − 2r sin θ.
∂v
= 6r sin 2θ − 2 sin θ.
∂r
∂2 v
= 6 sin 2θ.
∂r2
∂v
= 6r2 cos 2θ − 2r cos θ.
∂θ
∂2 v
= −12r2 sin 2θ + 2r sin θ.
∂θ2

Downloaded From : www.EasyEngineering.net


———-
170
Downloaded Supplement to Engineering Mathematics - II
From : www.EasyEngineering.net

∂2 v 1 ∂v 1 ∂2 v
Now, + +
∂r2 r ∂r r2 ∂θ2
1 1
= 6 sin 2θ + [6r sin 2θ − 2 sin θ] + 2 [−12r2 sin 2θ + 2r sin θ]
r r
2 2
= 6 sin 2θ + 6 sin 2θ − sin θ − 12 sin 2θ + sin θ = 0.
r r
∴ v is harmonic.

Using C-R equations we have

∂u ∂v
r = = 6r2 cos 2θ − 2r cos θ.
∂r ∂θ

ww ∂u
∂r
= 6r cos 2θ − 2 cos θ.

w.E
Integrating w.r.t.r we get
Z Z
a syE
u = 6 cos 2θ rdr − 2 cos θ dr + f (θ)

r2
ngi
= 6 cos 2θ · − 2r cos θ + f (θ)
2
2
u = 3r cos 2θ − 2r cos θ + f (θ).
nee (1)

Again by C-R equations we have


rin

1 ∂u ∂v
= = 6r sin 2θ − 2 sin θ. g.n
r ∂θ ∂r
∂u
∂θ
= −6r2 sin 2θ + 2r sin θ. et
Integrating w.r.t.θ we get
Z Z
2
u = −6r sin 2θdθ + 2r sin θdθ + g(r)
!
− cos 2θ
= −6r2 + 2r(− cos θ) + g(r)
2
u = 3r2 cos 2θ − 2r cos θ + g(r). (2)

From (1) and (2) we get, u = 3r2 cos 2θ − 2r cos θ + c, where c is a


constant, which is the harmonic conjugate of v.

Downloaded From : www.EasyEngineering.net


———-
Anna University
Downloaded Solved Question Papers
From : www.EasyEngineering.net 171

To verify whether u is harmonic

∂u
We have = 6r cos 2θ − 2 cos θ.
∂r
∂2 u
= 6 cos 2θ.
∂r2
∂u
= −6r2 sin 2θ + 2r sin θ.
∂θ
∂2 u
= −6r2 · (cos 2θ) × 2 + 2r cos θ.
∂θ2
= −12r2 cos 2θ + 2r cos θ.

ww Now,
∂2 u 1 ∂u 1 ∂2 u
+ +
w.E∂r2 r ∂r r2 ∂θ2
1 1
= 6 cos 2θ + (6r cos 2θ − 2 cos θ) + 2 (−12r2 cos 2θ + 2r cos θ)

a r

syE 2
r
2
= 6 cos 2θ + 6 cos 2θ − cos θ − 12 cos 2θ + cos θ = 0.
r r

∴ u is harmonic. ngi
nee
2.3.7 May/June 2013 [R 2008]
rin
Part A
g.n
7. Find the image of x = k under the transformation w =

Solution. We have w =
1
1
z
.
et
z
u −v
x= 2 and y =
u + v2 u2 + v2
u u
When x = k, k = 2 or u2 + v2 =
u + v2 k
u
u2 + v2 = .
k
!
1 1
It is a circle with centre , 0 and radius .
2k 2k
6z − 9
8. Find the fixed points of the mapping w = .
z

Downloaded From : www.EasyEngineering.net


———-
172
Downloaded Supplement to Engineering Mathematics - II
From : www.EasyEngineering.net

Solution. The fixed points are given by w = z.


6z − 9
∴z =
z
⇒z2 = 6z − 9
⇒z2 − 6z + 9 = 0
⇒(z − 3)2 = 0 ⇒ z = 3, 3.

Part B

14. (a) (i) Prove that the function u = e x (x cos y − y sin y) satisfies

ww Laplace equation and find the corresponding analytic function

w.E
f (z) = u + iv.
Solution. Let f (z) = u + iv be the analytic function.

a syE
Given u = e x (x cos y − y sin y) is the real part of f (z).

ngi
u x = e x cos y + e x (x cos y − y sin y) = e x (cos y + x cos y − y sin y).
uy = e x (−x sin y − y cos y − sin y).
nee
u xx = e x (cos y + x cos y − y sin y) + e x cos y = e x (2 cos y + x cos y − y sin y).

rin
uyy = e x (−x cos y + y sin y − cos y − cos y) = e x (−x cos y + y sin y − 2 cos y).
g.n
u xx + uyy = e x (2 cos y + x cos y − y sin y) + e x (−x cos y + y sin y − 2 cos y)
= 0. et
⇒ u satisfies Laplace equation.
Now, u x (z, 0) = ez (1 + z) and uy (z, 0) = 0.
By Milne-Thomson method, f ′ (z) = u x (z, 0) − iuy (z, 0) = ez (1 + z).
Integrating w.r.t z we get
Z
f (z) = ez (1 + z)dz + c
Z
= (1 + z)d(ez ) + c
Z
= (1 + z)e − ez dz + c
z

Downloaded From : www.EasyEngineering.net


———-
Anna University
Downloaded Solved Question Papers
From : www.EasyEngineering.net 173

= (1 + z)ez − ez + c = ez + zez − ez + c
∴ f (z) = zez + c.

14. (a) (ii) Find the bilinear transformation which maps z1 = 0,


z2 = 1, z3 = ∞ into the points w1 = i, w2 = 1, w3 = −i.
Solution. Let z1 = 0, z2 = 1, z3 = ∞, and w1 = i, w2 = 1, w3 = −i.
The mobius transformation is

(w − w1 )(w2 − w3 ) (z − z1 )
=
(w − w3 )(w2 − w1 ) (z2 − z1 )

ww i.e.,
(w − i)(1 + i)
(w + i)(1 − i)
=z

w+i
w.E
w − i z(1 − i) z(1 − i)2
=
1+i
=
2
= a
z(1 − 1 − 2i) z(−2i)
2
=
w − i + w + i −iz + 1
syE
2
= −iz

=
w − i − w − i −iz − 1 ngi
2w
=
1 − iz
−2i −(1 + iz) nee
w=
i(1 − iz)
1 + iz
=
z+i
1 + iz
.
rin
g.n
14. (b) (i) Find the image of |z − 2i| = 2 under the transformation
w= .
1
z
et
1
Solution. w =
z
u −v
⇒x= 2 ,y = 2 .
u + v2 u + v2
The given circle is |z − 2i| = 2.

⇒ |z − 2i|2 = 4
|x + iy − 2i|2 = 4
x2 + (y − 2)2 = 4
x2 + y2 − 4y = 0.

Downloaded From : www.EasyEngineering.net


———-
174
Downloaded Supplement to Engineering Mathematics - II
From : www.EasyEngineering.net

It is a circle passing through the origin.

u2 v2 −v
2 2 2
+ 2 2 2
−4 2 =0
(u + v ) (u + v ) u + v2
1 4v
+ =0
u2 + v2 u2 + v2
4v + 1 = 0,

which is a straight line in the w plane.


∴ The image of the circle |z − 2i| = 2 in the z−plane is a straight

ww line in the w−plane.

∂2 w.E
14. (b) (ii) If f (z)
+
∂2
! is an analytic function of z,
| f (z)|2 = 4| f ′ (z)|2 .
prove that

∂x2 ∂y2
a syE
Solution. Given f (z) is analytic.
Let f (z) = u + iv.
ngi
⇒ u and v have continuous partial derivatives and they satisfy
C-R equations. nee
rin
∴ u x = vy and uy = −v x and f ′ (z) = u x + iv x , | f ′ (z)|2 = u2x + v2x .
Since u and v are harmonic functions, they satisfy Laplace
equation. g.n
The complex form of the Laplace operator is et
∂2 ∂2 ∂2
2
+ 2 =4 .
∂x ∂y ∂z̄∂z

!
∂2 ∂2 2 ∂2 2 ∂2
Now, + | f (z)| = 4 | f (z)| = 4 ( f (z) f (z)).
∂x2 ∂y2 ∂z̄∂z ∂z̄∂z

Since f (z) is an analytic function, it is independent of z̄.


i.e., f (z) is a function of z only.
Similarly its conjugate f (z) is an analytic function of z̄ only.
∴ We can denote f (z) by f¯(z̄).

Downloaded From : www.EasyEngineering.net


———-
Anna University
Downloaded Solved Question Papers
From : www.EasyEngineering.net 175

i.e., f (x + iy) = f¯(x − iy).


!
∂2 ∂2 ∂ ∂
Hence, + | f (z)|2 = 4 ( f (z) f¯(z̄))
∂x2 ∂y2 ∂z̄ ∂z
∂ ∂
= 4 ( f¯(z̄)) ( f (z))
∂z̄ ∂z
= 4 f ′ (z) f ′ (z) = 4| f ′ (z)|2 .

2.3.8 November/December 2012 [R 2008]


Part A

ww7. Show that the function f (z) = z̄ is nowhere differentiable.

w.E
Solution. Let f (z) = u + iv = z̄ = x − iy.
∴ u = x, v = −y

a u x = 1,
uy = 0,
syE vx = 0
vy = −1.
ngi
Since u x , vy , C.R equations are not satisfied.
∴ f (z) is not analytic anywhere.
=⇒ f (z) is nowhere differentiable.
nee
rin
8. Find the map of the circle |z| = 3 under the transformation
w = 2z. g.n
Solution. Let z = x + iy and w = u + iv. et
w = 2z
u + iv = 2(x + iy) = 2x + i2y
u = 2x, v = 2y
Now |z| = 3
⇒x2 + y2 = 9
 u 2  v 2
+ =9
2 2
u2 v2
+ =9
4 4

Downloaded From : www.EasyEngineering.net


———-
176
Downloaded Supplement to Engineering Mathematics - II
From : www.EasyEngineering.net

u2 + v2 = 36.

It is a circle with centre (0, 0) and radius 6.

Part B

14. (a) (i) Find the bilinear map which maps the points ∞, i, 0 of the
z plane onto 0, i, ∞ of the w−plane.
Solution. Let z1 = ∞, z2 = i, z3 = 0 and w1 = 0, w2 = i, w3 = ∞.
The bilinear transformation is given by
w − w1 z2 − z3
ww w2 − w1
=
w−0 i−0
z − z3

w.E
i.e.,
i−0
=
w i
z−0

a i
w=
=
z
−1
. syE
z
ngi
u=
sin 2x
. nee
14. (a) (ii) Determine the analytic function whose real part is

cos h2y − cos 2x


Solution. Given, u =
sin 2x
. rin
cos h2y − cos 2x
g.n
ux =
(cos h2y − cos 2x)2 cos 2x − sin 2x(2 sin 2x)
(cos h2y − cos 2x)2
(1 − cos 2z)2 cos 2z − 2 sin2 2z
.
et
u x (z, 0) =
(1 − cos 2z)2
2 cos 2z − 2 cos2 2z − 2 sin2 2z
=
(1 − cos 2z)2
2 cos 2z − 2(cos2 2z + sin2 2z)
=
(1 − cos 2z)2
2 cos 2z − 2
=
(1 − cos 2z)2
−2(1 − cos 2z) −2
= = = − sec2 z.
(1 − cos 2z)2 2 cos2 z
uy = sin 2x(−1)(cos h2y − cos 2x)−2 2 sin h2y.

Downloaded From : www.EasyEngineering.net


———-
Anna University
Downloaded Solved Question Papers
From : www.EasyEngineering.net 177

uy (z, 0) = 0.

By Milne Thomson method,


f ′ (z) = u x (z, 0) − iuy (z, 0) = − sec2 z.
Integrating w.r.t. z, we get
R R
f ′ (z)dz = − sec2 zdz + c
f (z) = − tan z + c.

14. (b) (i) Find the image of the hyperbola x2 − y2 = 1 under the
1
transformation w = and prove that it is the lemniscate of
ww z
Bernouillie r2 = cos 2θ.

w.E
Solution. Let z = x + iy and w = reiθ .
1 1 1
Given w = or z = = iθ = e−iθ .
1

a 1
z

syE
w re

(cos θ − i sin θ)
i.e., x + iy =
r

1
r
1
∴ x = cos θ, y = − sin θ. ngi
r r
The given curve is x2 − y2 = 1 nee
1 1
cos2 θ − 2 sin2 θ = 1 rin
r2
1
r
g.n
r2
1
r2
(cos2 θ − sin2 θ) = 1

cos 2θ = 1
et
⇒ r2 = cos 2θ.

z
14. (b) (ii) Prove that w = maps the upper half of the z−plane
1−z
to the upper half of the w−plane and also find the image of the
unit circle of the z−plane.
z
Solution. Given w = .
1−z
x + iy
u + iv =
1 − x − iy

Downloaded From : www.EasyEngineering.net


———-
178
Downloaded Supplement to Engineering Mathematics - II
From : www.EasyEngineering.net

x + iy (1 − x) + iy
= ×
(1 − x) − iy (1 − x) + iy
x(1 − x) − y2 + i[y(1 − x) + xy]
=
(1 − x)2 + y2
x(1 − x) − y2 y(1 − x) + xy
= 2 2
+i .
(1 − x) + y (1 − x)2 + y2
x(1 − x) − y2
∴u= and
(1 − x)2 + y2
y(1 − x) + xy
v= .
(1 − x)2 + y2
Let v > 0.

ww y(1 − x) + xy

w.E
(1 − x)2 + y2
>0

y − xy + xy > 0
a syE
y > 0.

ngi
This shows that the upper half of the z−plane is mapped onto
the upper half of the w−plane.
To find the image of the unit circle. nee
z rin
we have w =
1−z
g.n
w(1 − z) = z
w − wz = z
et
w = wz + z
z(1 + w) = w
w
z= .
1+w
Given |z| = 1.

w
⇒ =1
1 + w
|w| = |1 + w|
|w|2 = |1 + w|2

Downloaded From : www.EasyEngineering.net


———-
Anna University
Downloaded Solved Question Papers
From : www.EasyEngineering.net 179

|u + iv|2 = |1 + u + iv|2
u2 + v2 = (1 + u)2 + v2
= 1 + u2 + 2u + v2
0 = 2u + 1
1
u=− .
2
1
The image of the unit circle |z| = 1 is the straight line u = − .
2

ww
2.3.9 May/June 2012 [R 2008]
Part A
w.E
7. State the basic difference between the limit of a function of a
a syE
real variable and that of the complex variable.
Solution. Let f be a function defined in some neighbourhood
ngi
of z0 except possibly at z0 . We say a complex number w0 is the

nee
limit of f (z) as z tends to z0 if for any ǫ > 0, there exists a δ > 0

rin
such that | f (z) − f (z0 )| < ǫ for |z − z0 | < δ. Symbolically it can be
written as lim f (z) = w0 = f (z0 ).
z→z0
g.n
8. Prove that a bilinear transformation has at most two fixed
points.
az + b
et
Solution. The bilinear transformation is w = .
cz + d
The fixed points are given by w = z.
az + b
i.e., z =
cz + d
z(cz + d) = az + b
cz2 + dz − az − b = 0
cz2 + (d − a)z − b = 0.
This is a quadratic in z. It has two roots which are the fixed
points of the given bilinear transformation.
Hence, a bilinear transformation has at most two fixed points.

Downloaded From : www.EasyEngineering.net


———-
180
Downloaded Supplement to Engineering Mathematics - II
From : www.EasyEngineering.net

Part B

14. (a) (i) Prove that every analytic function w = u + iv can be


expressed as a function of z alone, not as a function of z̄.
Solution. Let z = x + iy.
z + z̄ z − z̄
Then, z̄ = x − iy, which implies x = and y = .
2 2i
Now, w = u + iv.
∂w ∂u ∂v
= +i
∂z̄ ∂z̄ ∂z̄
" #
∂u ∂x ∂u ∂y ∂v ∂x ∂v ∂y
= + +i +

ww =
∂x ∂z̄ ∂y ∂z̄
∂u 1 ∂u −1
+ +i
"
∂x ∂z̄ ∂y ∂z̄
∂v 1 ∂v −1
+
#

w.E=
∂x 2 ∂y 2i
"
1 ∂u
+i
∂u
#
+
"
∂x 2 ∂y 2i
i ∂v
+i
∂v
#

a
=
2 ∂x
"
1 ∂u
∂y
syE
∂u
+i +i −
2 ∂x
∂v ∂v
#
∂y

2 ∂x
"
1 ∂u ∂v
∂y
ngi
∂x ∂y
∂u ∂v
!#
=
2 ∂x ∂y
1
− +i +
∂y ∂x
nee
= [0 + i.0][∴ u x = vy , uy = −v x ]
2
rin
∂w
∂z̄
= 0.
g.n
⇒ w is independent of z̄.
Hence, w can be written only as a function of z and not as a
et
function of z̄

14. (a) (ii) Find the bilinear transformation which maps z1 = 0, z2 =


1, z3 = ∞ into the points w1 = i, w2 = 1, w3 = −i.
Solution. Let z1 = 0, z2 = 1, z3 = ∞, and w1 = i, w2 = 1, w3 = −i.
The mobius transformation is
(w − w1 )(w2 − w3 ) (z − z1 )
=
(w − w3 )(w2 − w1 ) (z2 − z1 )
(w − i)(1 + i)
i.e., =z
(w + i)(1 − i)

Downloaded From : www.EasyEngineering.net


———-
Anna University
Downloaded Solved Question Papers
From : www.EasyEngineering.net 181

w − i z(1 − i) z(1 − i)2


= =
w+i 1+i 2
z(1 − 1 − 2i) z(−2i)
= = = −iz
2 2
w − i + w + i −iz + 1
=
w − i − w − i −iz − 1
2w 1 − iz
=
−2i −(1 + iz)
i(1 − iz) z+i
w= = .
1 + iz 1 + iz

14. (b) (i) If f (z) is analytic, prove that ∇2 log | f (z)| = 0.

ww Solution. ∇2 log | f (z)| =


1 2
2
∇ log | f (z)|2

w.E 1
= ∇2 log( f (z) f (z))
2 !
a syE =
1 ∂2
2 ∂x 2
∂2 
+ 2 log f (z) + log f¯(z)
∂y
¯


1 ∂2  
= 4
ngi
2 ∂z̄∂z
log f (z) + log f¯(z)
!
¯

=2
∂ 1 ′
∂z̄ f (z) nee
f (z) = 0.

rin
14. (b) (ii) Find the image of the hyperbola x2 − y2 = 1 under the
1
transformation w = and prove that it is the lemniscate of g.n
z
Bernouillie r2 = cos 2θ.
Solution. Let z = x + iy and w = reiθ .
et
1 1 1 1
Given w = or z = = iθ = e−iθ .
z w re r
1
(cos θ − i sin θ)
i.e., x + iy =
r
1 1
∴ x = cos θ, y = − sin θ.
r r
The given curve is x2 − y2 = 1
1 1
2
cos2 θ − 2 sin2 θ = 1
r r
1
(cos θ − sin2 θ) = 1
2
r2

Downloaded From : www.EasyEngineering.net


———-
Downloaded From : www.EasyEngineering.net

ww
w.E
UNIT 4 asy
En
gin
ee
By EasyEngineering.net rin
g.n
et

Downloaded From : www.EasyEngineering.net


Downloaded From : www.EasyEngineering.net

4 Complex Integration

4.1 Introduction
ww Contour

w.E A curve that is constructed by joining finitely many smooth curves end to end
is called a contour. A curve is given by the equation z(t) = x(t) + iy(t), a ≤ t ≤ b where

asy
x(t) and y(t) are continuous functions of t.
Contour Integral. If f (z) is a function of the complex variable z which is defined
on Z
En
a given curve C in the complex plane, then the complex line integral is written
as
C
f (z)dz.
gin
z1 = z(a) to z2 = z(b) then we write
Z
f (z)dz as ee
If the equation of C is z(t) = x(t) + iy(t), a ≤ t ≤ b and C is the contour from
Zz2
f (z)dz.
C z1
rin
If f (z) = u(x, y) + iv(x, y) then
Z Z Z Z
g.n
C
f (z)dz =

Properties of Contour Integrals


C
(u + iv)(dx + idy) =
C
(udx − vdy) + i
C
et
(vdx + udy).

Z Z
1. z0 f (z)dz = z0 f (z)dz, where z0 is a constant.
C C
Z Z Z
2. ( f (z) ± g(z))dz = f (z)dz ± g(z)dz.
C C C

Downloaded From : www.EasyEngineering.net


Downloaded From : www.EasyEngineering.net

398 Engineering Mathematics - II

Z Z
3. f (z)dz = − f (z)dz.
−C C
4. ZIf C is broken
Z up intoZtwo parts C1 and C2 , then
f (z)dz = f (z)dz + f (z)dz.
C C1 C2

Note. The value of the integral is independent of the path of integration, when
the integrand is analytic.

ww ✎
Worked Examples


w.E
Z
Example 4.1. Evaluate (x − y + ix2 )dz where C is the line joining the points from
C
z = 0 to z = 1 + i. [Apr 2009]
Solution. Let z = x + iy.
asy
dz = dx + idy.

En
The points z = 0 and z = 1 + i represent O
gin Y

ee
P(1, 1)
and P respectively in the complex plane.
Hence O is (0, 0) and P is (1, 1).
Equation to OP is y = x.
rin
Along OP, y = x, dy = dx and x varies from
0 to 1.
O (0, 0)
g.n X


Z
2
(x − y + ix )dz =
Z1
(x − x + ix2 )(dx + idy)
et
C 0
Z1 Z1
= ix2 (dx + idx) = i x2 (1 + i)dx
0 0
 x3 1 i(1 + i) −1 + i 1 1
= i(1 + i) = = = − + i.
3 0 3 3 3 3

Downloaded From : www.EasyEngineering.net


Downloaded From : www.EasyEngineering.net

Complex Integration 399

Z
Example 4.2. Evaluate z2 dz where C is the arc from A(1, 1) to B(2, 4) along
C
(i) y = x2 (ii) y = 3x − 2. [Jun 2010]
Solution. Let z = x + iy.
dz = dx + idy.
Y •
Z
B(2, 4)
2
(i) Let us evaluate z dz where C is
C •
A(1, 1)

ww y = x2 .
Along y = x2 , dy = 2xdx, x varies from 1 to 2. (0, 0)
X

w.E
Now, dz = dx+idy = dx+i2xdx = (1+2ix)dx.
Z
2
z dz =
Z
2
(x + iy) (dx + idy) =
Z
(x2 − y2 + 2ixy)(dx + idy)

asy

C C C
Z2
=
En
(x2 − x4 + 2ix3 )(1 + i2x)dx

gin
1
Z2  
= (x2 − x4 − 4x4 ) + i(2x3 − 2x5 + 2x3 ) dx

=
1
Z2  
(x2 − 5x4 ) + i(4x3 − 2x5 ) dx
ee rin
=
1

−5
 x5 2
 x3 2
+i 4
 x4 2
−2
 x6 2 ! 1 1
= (8 − 1) − (32 − 1) + i(16 − 1 − (64 − 1)) g.n
7
3 1 5 1

= − 31 + i(15 − 21) =
3
Z
4 1

7 − 93
3
6 1

− 6i = −
3
h 86
3
i
+ 6i .
3

et
(ii) Let us evaluate z2 dz along y = 3x − 2.
C
Along y = 3x − 2, dy = 3dx, dz = dx + idy = dx + i3dx = (1 + 3i)dx

Z Z2
2
Now z dz = (x2 − (3x − 2)2 + 2ix(3x − 2))(1 + 3i)dx
C 1

Downloaded From : www.EasyEngineering.net


Downloaded From : www.EasyEngineering.net

400 Engineering Mathematics - II

Z2
= (1 + 3i) x2 − 9x2 − 4 + 12x + i(6x2 − 4x)dx
1
Z2 h i
= (1 + 3i) − 8x2 − 4 + 12x + i(6x2 − 4x) dx
1
h  x3 2  x 2 2   x 3 2  x2 2 i
= (1 + 3i) − 8 + 12 − 4(x)21 + i 6 −4
3 1 2 1 3 1 2 1
−8
= (1 + 3i)[ 7 + 6(3) − 4 + i(2(7) − 2(3))]

ww = (1 + 3i)[
3
−56
3
+ 14 + 8i] =
(1 + 3i)
3
[−56 + 42 + 24i]

w.E =
(1 + 3i)

1
3
= [−86 − 18i] = −
 86
1
[−14 + 24i] = [−14 + 24i − 42i − 72]
3

+ 6i .

Z2+i
3
asy x
3

En
Example 4.3. Evaluate (z̄)2 dz along the line y = .
0
2
[Dec 2010]

Solution. Let z = x + iy.


dz = dx + idy. gin
z̄ = x − iy.
ee
(z̄)2 = (x − iy)2 = x2 − y2 − 2ixy.
We have to evaluate the integral along
Y

rin
g.n

x P(2, 1)
y= from (0, 0) to (2, 1). Along
2
2
x/

x 1

et
=

y = , dy = dx, x varies from 0 to 2.


y

2 2
(z̄)2 dz = (x2 − y2 − 2ixy)(dx + idy) X
O (0, 0)
 x2 x  dx 
= x2 − − 2ix dx + i
4 2 2
 3x2  i 
= − ix2 1 + dx
4 2
Z2+i Z2  2  2 
2 3x 2
 i  i   3  x3 2  x3 
∴ (z̄) dz = − ix 1 + dx = 1 +  −i 
4 2 2 4 3 0 30 
0 0

Downloaded From : www.EasyEngineering.net


Downloaded From : www.EasyEngineering.net

Complex Integration 401

" #
 i 8 8 4 10 5
= 1+ 2−i =2− i+i+ = − i.
2 3 3 3 3 3
Z
Example 4.4. Evaluate sin zdz along the line z = 0 to z = i. [Apr 2010]
C
Y

Solution.
Z Z i • A(0, 1)

x=0
sin z dz = sin z dz
C 0

ww = [− cos z]i0
O (0, 0)
X

w.E
= −[cos i − cos 0]

= 1 − cos i.

Example 4.5. Prove that (i)


asy Z
dz
z−a
= 2πi (ii)
Z
(z − a)n dz = 0, n any integer , −1,

En
C C
where C is the circle |z − a| = r. Y z

Solution. The equation of C is z − a =

gin
θ

reiθ , where θ varies from 0 to 2π as z a

describes once in the positive sense.


(i) z − a = reiθ .

dz = reiθ · idθ.
ee O

rin
X

Z
dz
z−a
=
Z2π
1
reiθ
· reiθ · idθ g.n
C

=
0
Z2π
idθ
et
0

= i[θ]2π
0

= i(2π − 0) = 2πi.
Z Z2π   Z2π
n iθ n iθ
(ii) (z − a) dz = re · re idθ = rn einθ · reiθ idθ
C 0 0

Downloaded From : www.EasyEngineering.net


Downloaded From : www.EasyEngineering.net

402 Engineering Mathematics - II

Z2π !2π
n+1 i(n+1)θ n+1 ei(n+1)θ
= ir e dθ = ir
i(n + 1) 0
0
rn+1 h i2(n+1)π i rn+1
e = − e0 = [1 − 1] = 0.
n+1 n+1
Z
Example 4.6. Evaluate (z2 + 3z + 2)dz where C is the arc of the cycloid
C
x = a(θ + sin θ), y = a(1 − cos θ) between the points (0, 0) and (πa, 2a).

ww Solution.
Z Let f (z) = z2 + 3z + 2 which is a polynomial in z and henceA(πa,
∴ (z2 + 3z + 2)dz is independent of the
Y
it 2a)
is analytic.

w.EC

>
path of integration.

cycloid. Draw AL perpendicular to the asy


Let O be (0, 0) and A be (πa, 2a) on the
(0, 0) O
>
L

(πa, 0)
X

x-axis.Z
Now, f (z)dz =
Z
f (z)dz +
Z
f (z)dz. En
C OL

gin
LA
Along OL, y = 0 ⇒ dy = 0, x varies from 0 to πa.


Z

OL
Zπa
f (z)dz = (x2 + 3x + 2)dx
0
x3
"
x2
ee #πa ∵ z = x + iy rin
=
3
+3·
2
+2·x

π3 a3 3 2 2
0 dz = dx + iy
g.n
=
3
+ π a + 2πa.
2

Along LA, x = πa, dx = 0, y varies from 0 to 2a.


y = 0.
et
z = x + iy, dz = dx + idy = idy

f (z) = z2 + 3z + 2

= (x + iy)2 + 3(x + iy) + 2

= (πa + iy)2 + 3(πa + iy) + 2.

Downloaded From : www.EasyEngineering.net


Downloaded From : www.EasyEngineering.net

Complex Integration 403

Z2a h #2a
(πa + iy)3 3(πa + iy)2
Z i "
f (z)dz = (πa + iy)2 + 3(πa + iy) + 2 idy = i + + 2y
3i 2i 0
LA 0
(πa + 2ai)3 3 π3 a3 3 2 2
= + (πa + 2ai)2 + 4ai − − π a
3 2 3 2
a3 3a 2 π3 a3 3π2 a2
= (π + 2i)3 + (π + 2i)2 + 4ai − − .
3 2 3 2
π3 a3 3π2 a2 a3 3a2 π3 a3 3π2 a2
Z
∴ f (z)dz = + + 2πa + (π + 2i)3 + (π + 2i)2 + 4ai − −
3 2 3 2 3 2
C

ww =
a3
3
(π + 2i)3 +
3a2
2
(π + 2i)2 + 2a(π + 2i).

w.E
Exercise 4 A

asy
Z
1. Evaluate (z + 1)dz where C is the boundary of the square whose vertices
C

Z1+i En
are at the points z = 0, z = 1, z = 1 + i and z = i. [Mar 2010]

gin
2. Evaluate (x2 − iy)dz along the paths (i) y = x (ii) y = x2 .
0

3. Evaluate
Z

C
ee
f (z)dz where f (z) = y − x − i3x2 from z = 0 to z = 1 + i along the path

(i) from (0, 0) to A(1, 0) and to B(1, 1). (ii) y = x. rin


4. Evaluate
Z
ez dz where C is the circle |z| = 1. g.n
5. Evaluate
C
Z
log zdz where C is the circle |z| = 1.
et
C
Z
1
6. Prove that dz = 2πi where C is the circle |z − a| = r.
z−a
C
Z
7. Prove that (z − a)n dz = 0[n , −1] where C is the circle |z − a| = r.
C

Downloaded From : www.EasyEngineering.net


Downloaded From : www.EasyEngineering.net

404 Engineering Mathematics - II

2+3i
Z
8. Evaluate (z2 + z)dz along the line joining the points (1, −1) and (2, 3).
1−i

Z2+i
9. Evaluate the integral (x2 − iy)dz (i) along the straight line y = x. (ii) along
0
the curve y = x2 .

−2+i

ww
Z
10. Evaluate (2 + z)2 dz.
−2

w.E
Simply Connected and multiply connected domains

asy
A domain D in the complex plane is called simply connected if every closed
curve in D encloses points of D only.

En
Example. Interior of a circle, square and ellipse are simply connected domains. A

gin
domain D which is not simply connected is called multiply connected.

ee rin
g.n
Simply connected domains et

Multiply connected domains.

Downloaded From : www.EasyEngineering.net


Downloaded From : www.EasyEngineering.net

Complex Integration 405

4.2 Cauchy’s Integral theorem or Cauchy’s fundamental


theorem

Statement. If f (z) is analyticIand f ′ (z) is continuous at each point within and on


a simple closed curve C, then f (z)dz = 0. [May 2015]
C
Proof. Let f (z) = u(x, y) + iv(x, y).
z = x + iy.

ww Now
Z dz =Z dx + idy.
f (z)dz = (u + iv)(dx + idy)

w.E
C C
Z Z
= (udx − vdy) + i (udy + vdx). (1)

Given, f ′ (z) is continuous. asy C C


∂u ∂u ∂v ∂v
, , ,
∂x ∂y ∂x ∂y En
are also continuous in the domain D enclosed by C.

Z gin
∴ Applying Green’s theorem for the integrals on the RHS of (1) we get
" 
∂v ∂u 
" 
∂u ∂v 

C
f (z)dz = −
D ee
+
∂x ∂y
dxdy + i

Since f (z) is analytic, u and v satisfy C-R equations.



∂x ∂y
dxdy.
D

rin
(2)

ie.,
∂u ∂v
=
∂x ∂y
and
∂u
∂y
=− .
∂v
∂x g.n
et
∂u ∂v ∂u ∂v
=⇒ − = 0 and + = 0.
∂x Z ∂y ∂y ∂x
∴ f (z)dz = 0.
C
Extension of Cauchy’s theorem. I If f (z) is analytic
I in the region D between two
simple closed curves C and C1 , then f (z)dz = f (z)dz.
C C1
Proof. Let
I AB be the cross cut between C and C1 . Now, by the path indicated by
arrows, f (z)dz = 0.
C

Downloaded From : www.EasyEngineering.net


Downloaded From : www.EasyEngineering.net

406 Engineering Mathematics - II

A B C1 C

The path described is as follows. Along AB → along C1 in the clockwise sense →

ww along BA → along C in the anticlockwise sense.

i.e.,
Z Z Z Z

w.E
f (z)dz + f (z)dz + f (z)dz + f (z)dz = 0.
AB C1 BA C
Z Z

Z asy
The integrals along AB and BA are equal and opposite, ∴
Z AB
f (z)dz +
BA
f (z)dz = 0.

Hence we have
C1
f (z)dz +

En
C
f (z)dz = 0.

Z
f (z)dz = gin
Reversing Zthe direction of the integral along C1 and transposing, we obtain
f (z)dz, each integration is being taken in the anticlockwise sense.
C C1
Hence the theorem.
ee
Corollary. If C1 , C2 , C3 , . . . are any number of closed curves within C, then
rin
g.n
I I I I
f (z)dz = f (z)dz + f (z)dz + f (z)dz + · · ·

et
C C1 C2 C3
>
>

C3
>

> >
>

C1 C2
>
>

>

Downloaded From : www.EasyEngineering.net


Downloaded From : www.EasyEngineering.net

Complex Integration 407

4.3 Singular points

If a function ′ f ′ , is not analytic at a point z0 , but is analytic at some point in


every neighbourhood of z0 , then z0 is a singular point of f.
1
Example. If f (z) = , z = 0 is a singular point.
z

4.4 Cauchy’s Integral Formula

ww If f (z) is an analytic function within and on a simple closed contour C taken in


the positive sense and if a is any interior point of C, then

w.E
I
1 f (z)
f (a) = dz.
2πi z−a
C

asy Y
[Dec 2014]

En
> C

C1

gin a•
>

eeO (0, 0)
X

rin
Proof. Consider the function
f (z)
z−a
. This is analytic at all points within C except
at z = a. Draw a small circle C1 with a as centre and radius r lying entirely within g.n
C.
Now,
f (z)
z−a
is analytic in the region enclosed by C and C1 .
et
∴ By Cauchy’s integral theorem,
I I
f (z) f (z)
dz = dz [z − a = reiθ , dz = rieiθ dθ]
z−a z−a
C C1
f (a + reiθ ) iθ
I I
= rie dθ = i f (a + reiθ )dθ.
reiθ
C1 C1

Downloaded From : www.EasyEngineering.net


Downloaded From : www.EasyEngineering.net

408 Engineering Mathematics - II

In the limiting form, as the circle C1 tends to the point a, r → 0. Hence, the integral
is reduced to
I Z2π
f (z)
dz = i f (a)dθ = i f (a)(θ)2π
0 = 2πi f (a)
z−a
C 0
I
1 f (z)
∴ f (a) = dz,
2πi z−a
C

which is the required Cauchy’s integral formula.

ww Corollary. Cauchy’s
Statement.
Z Integral formula for derivatives
f (z)
(z − a)n+1
dz = 2πi
f n (a)
n!
.

w.E C
Proof. By Cauchy’s integral formula we have
I

asy
1 f (z)
f (a) = dz.
2πi z−a
C

Differentiating w.r.t. a we get


En
gin
" #
∂ f (z)
I I
′ 1 1 f (z)
f (a) = dz = dz.
2πi ∂a z − a 2πi (z − a)2
C C

Similarly, f ′′ (a) =
2!
2πi
C I
I
f (z)
(z − a)3
dz. ee rin
g.n
n! f (z)
In general, f (n) (a) = dz.
2πi (z − a)n+1
C


I

C
f (z)
(z − a)n+1

dz = 2πi
f (n) (a)
n!


. et
Worked Examples
✍ ✌
Z
Example 4.7. Evaluate ez dz where C is |z| = 1. [Jun 2003]
C
Solution. Let f (z) = ez .
ez is analytic everywhere in the complex plane.

Downloaded From : www.EasyEngineering.net


Downloaded From : www.EasyEngineering.net

Complex Integration 409

Hence it is analytic in |z| = 1. Z


∴ By Cauchy’s Integral theorem f (z)dz = 0.
Z C
z
i.e., e dz = 0.
C

z2 − z + 1
Z
1
Example 4.8. Evaluate dz where C is the circle (i) |z| = 1 (ii) |z| = .
z−1 2
C
Solution. (i) Consider the circle |z| = 1.

ww The singular point is z = 1 which lies on C.


∴ By Cauchy’s integral formula

w.E
Z
f (z)
dz = 2πi f (1) where f (z) = z2 − z + 1
z−1
C

asy
= 2πi × 1 = 2πi.

(ii) C is |z| = .
1
2
En
gin
In this case, z = 1 lies outside C.
f (z)
∴ is analytic every where within C.
z−1


Z

C
f (z)
z−1
dz = 0 ee rin
g.n
Z 2
z −z+2
dz = 0.
z−1
C

Example 4.9. Evaluate

1
Z

C
1
z+4
z2+ 2z
1
2
1
dz, where C is |z − | = .
3 et[Dec 2013]

Solution. C is the circle |z − | = .


Z Z 2 3
z+4 z+4
dz = dz.
z2 + 2z z(z + 2)
C C

Downloaded From : www.EasyEngineering.net


Downloaded From : www.EasyEngineering.net

410 Engineering Mathematics - II

The singular points are z = 0 and z = −2,


C
which lie completely outside C.
z+4 1/3
∴ 2 is analytic everywhere inside
z + 2z • X
z = −2 z=0 1/2
C.Z
z+4
∴ 2
dz = 0.
z + 2z
C

3z2 + 7z + 1
Z
1

ww Example 4.10. Evaluate

1
C
z+1
dz where C is |z| = . [Jun 2013, Jun 2010]
2

w.E
Solution. C is |z| = .
2
3z2 + 7z + 1
For the integrand , z = −1 is a singular point which lies
z+1

asy
completely outside |z| = .
3z2 + 7z + 1
1
2

z+1
En
is analytic everywhere inside C.
Z
3z2 + 7z + 1

gin
∴ By Cauchy’s integral theorem dz = 0.
z+1
C

ee
Z
zdz 1
Example 4.11. Evaluate where C is the circle |z| = .
(z − 1)(z − 2) 2
C

rin
[Dec 2012, Dec 2011]

g.n
1
Solution. C is |z| = .
2
z
The function has singular points z = 1 and z = 2 which lie


z
(z − 1)(z − 2)
(z − 1)(z − 2)
completely outside |z| = .
1
2
is analytic everywhere inside C.
et
Z
zdz
∴ = 0, by Cauchy’s integral theorem.
(z − 1)(z − 2)
C

ez
Z
Example 4.12. Evaluate dz if C is |z| = 2. [Dec 2010]
z−1
C
Solution. C is the circle |z| = 2.

Downloaded From : www.EasyEngineering.net


Downloaded From : www.EasyEngineering.net

Complex Integration 411

It is a circle with centre (0, 0) and radius = 2 units.


The singular point is at z = 1 which is an interior point of C.
By Cauchy’s Integral formula,
Z
1 f (z)
f (a) = dz.
2πi z−a
C

Here f (z) = ez and a = 1.

ez
Z
1
∴ f (1) = dz

ww e=
2πi
1
C
Z
z−1
ez
dz

w.E Z
2πi
ez
C
z−1

dz = 2πie.
C
asy
z−1

En
Z
z
Example 4.13. Using Cauchy’s integral formula, evaluate dz where C is
z−2
C
3
the circle |z − 2| = .
2
gin
Solution. The singular point is z = 2.
[May 2001]

Let f (z) = z.
3
C is |z − 2| = .
2
ee rin
3
When z = 2, |z − 2| = |2 − 2| = 0 < .
∴ z = 2 lies inside C.
2
g.n
By Cauchy’s integral formula

f (2) =
1
Z
f (z)
dz
et
2πi z−2
C
Z
1 z
2= dz
2πi z−2
C
Z
z
dz = 4πi.
z−2
C

Downloaded From : www.EasyEngineering.net


Downloaded From : www.EasyEngineering.net

412 Engineering Mathematics - II

cos πz2
Z
Example 4.14. Evaluate dz where C is |z| = 32 . [Jun 2001]
(z − 1)(z − 2)
C Z
f (z)
Solution. By Cauchy’s Integral formula dz = 2πi f (a).
z−a
C
The circle is |z| = 32 .
z = 1 and z = 2 are singular points.
When z = 1, |z| = |1| < 23 .
∴ z = 1 lies inside the circle.

ww When z = 2, |z| = |2| > 23 .


∴ z = 2 lies outside the circle.
Z cos πz2

w.E cos πz2


Z Z
(z−2) f (z)
Now, dz = dz = dz
(z − 1)(z − 2) (z − 1) (z − 1)
C C C
cos πz2
where f (z) =
z−2
asy
By Cauchy’s Integral formula f (1) =
1
Z
f (z)
dz

cos π 1
Z
En cos πz2
2πi
C
z−1

Z
1
=
2πi
cos πz2
C
gin
(z − 1)(z − 2)
dz

C
(z − 1)(z − 2)
dz = 2πi.
ee rin
Example 4.15. Evaluate
I
z2 + 1
dz where C is the circle |z − 1| = 1. g.n [Dec 2001]
C
z2 − 1
Solution. The singular points are given by z2 − 1 = 0.
(z − 1)(z + 1) = 0.
et
z = 1, −1.
The contour is |z − 1| = 1.
When z = 1, |z − 1| = |1 − 1| = 0 < 1.
=⇒ z = 1 lies inside C.
When z = −1, |z − 1| = | − 1 − 1| = | − 2| > 1.

Downloaded From : www.EasyEngineering.net


Downloaded From : www.EasyEngineering.net

Complex Integration 413

=⇒ z = −1 lies outside C.
z2 +1
z2 + 1 z2 + 1
Z Z Z
z+1 f (z)
Now, dz = dz = dz where f (z) =
z2 − 1 z−1 z−1 z+1
C C C
Z
1 f (z)
By Cauchy’s integral formula, f (1) = dz
2πi z−1
Z C
1+1 1 f (z)
= dz
1 + 1 2πi z−1
Z 2 C

ww
z +1
2πi = dz.
z2 − 1
C

w.E
I
z+2
Example 4.16. Evaluate dz where C is the circle |z| = 2 in the z plane.
z
C

asy
Solution. The singular point is z = 0.
[Jun 2007]

C is the circle |z| = 2.


En
When z = 0, |z| = |0| = 0 < 2.

I 0 lies inside
=⇒ z =
z+2
I C.
gin
f (z)
Now,
C
z
dz =
C
z
ee
dz where f (z) = z + 2.

By Cauchy’s integral formula, f (0) =


1
2πi
I
f (z)
z
dz
rin
2=
1
2πi
I
f (z)
z
dz
C

g.n
4πi =
IC
z+2

C
z
dz.

Z
et
z+4
Example 4.17. Evaluate dz where C is the circle (i) |z + 1 + i| = 2 (ii)
z2 + 2z + 5
C
|z + 1 − i| = 2 (iii) |z +
Z 1| = 1. [Dec 2012, Dec 2011, Dec 2010, Jun 2008]
z+4
Solution. Let I = 2
dz.
z + 2z + 5
C
The singular points are given by z2 + 2z + 5 = 0.

Downloaded From : www.EasyEngineering.net


Downloaded From : www.EasyEngineering.net

414 Engineering Mathematics - II

(z + 1)2 + 5 − 1 = 0
(z + 1)2 + 4 = 0
(z + 1)2 = −4
z + 1 = ±2i
z = −1 ± 2i.
(i) The circle is |z + 1 + i| = 2.
When z = −1 − 2i, |z + 1 + i| = | − 1 − 2i + 1 + i| = | − i| = 1 < 2.
∴ −1 − 2i is a point inside the circle.

ww When z = −1 + 2i, |z + 1 + i| = | − 1 + 2i + 1 + i| = |3i| = 3 > 2.


∴ z = −1 + 2i lies outside the circle.

w.E
Z
z+4
Now,

dz =
Z
z+4
dz =
Z z+4
z+1−2i
dz =
Z
f (z)
dz.
C
2
z + 2z + 5
C

Where f (z) = asy


(z + 1 − 2i)(z + 1 + 2i)
z+4
.
z + 1 + 2i
C
z + 1 + 2i
C

En
z + 1 − 2i
By Cauchy’s integral formula

gin
Z
1 f (z)
f (−1 − 2i) = dz
2πi z + 1 + 2i

ee
C
Z
−1 − 2i + 4 1 f (z)
= dz
−1 − 2i + 1 − 2i 2πi
3 − 2i 1
C
Z
z + 1 − 2i
z+4 rin
−4i
=
2πi
Z C
2
z + 2z + 5
dz
g.n
Z
z+4
π
− (3 − 2i) =
2

dz =
C
π
2
z+4
z + 2z + 5

(2i − 3).
dz
et
z2 + 2z + 5 2
C

(ii) The circle is |z + 1 − i| = 2.


When z = −1 − 2i, |z + 1 − i| = | − 1 − 2i + 1 − i| = | − 3i| = 3 > 2.
∴ z = −1 − 2i lies outside the circle.
When z = −1 + 2i, |z + 1 − i| = | − 1 + 2i + 1 − i| = |i| = 1 < 2.

Downloaded From : www.EasyEngineering.net


Downloaded From : www.EasyEngineering.net

Complex Integration 415

∴ z = −1 + 2i lies inside the circle.


Z Z z+4 Z
z+4 z+1+2i f (z)
Now dz = dz = dz,
z2 + 2z + 5 z + 1 − 2i z + 1 − 2i
C C C

z+4
where, f (z) = , a = −1 + 2i.
z + 1 + 2i

−1 + 2i + 4 3 + 2i
f (a) = = .
−1 + 2i + 1 + 2i 4i
By Cauchy’s integral formula

ww Z
f (z)
z−a
dz = 2πi f (a) = 2πi
3 + 2i 3 + 2i
4i
=
2
π.

w.E
C

(iii) C is |z + 1| = 1.

asy
When z = −1 − 2i, |z + 1| = | − 1 − 2i + 1| = | − 2i| = 2 > 1.
∴ −1 − 2i lies outside C.

En
When z = −1 + 2i, |z + 1| = | − 1 + 2i + 1| = |2i| = 2 > 1.
∴ −1 + 2i lies outside C.
∴ f (z) = 2
z+4
z + 2z + 5 gin
is analytic inside C.

ee
Z
∴ By Cauchy’s theorem f (z)dz = 0.

Z
z
C

rin
Example 4.18. Evaluate

formula.
C
(z − 1)3
g.n
dz where C is |z| = 2 using Cauchy’s integral

[May 1998]
Solution. z = 1 is a singular point.
C is |z| = 2.
et
When z = 1, |z| = |1| < 2.
∴ z =Z1 is an interior Z point.
z f (z)
Now 3
dz = dz
(z − 1) (z − 1)3
C C
where f (z) = z, n + 1 = 3 ⇒ n = 2.
f ′ (z) = 1, f ′′ (z) = 0, f ′′ (1) = 0.

Downloaded From : www.EasyEngineering.net


Downloaded From : www.EasyEngineering.net

416 Engineering Mathematics - II

2πi f (n) (a)


Z
f (z)
By Cauchy’s integral formula, dz = .
(z − a)n+1 n!
C

2πi f ′′ (1)
Z
z
i.e., dz = = πi.0 = 0.
(z − 1)3 2
C

3z2 + 7z + 1
Z
Example 4.19. If f (a) = dz where C is the circle |z| = 2, find
z−a
C
f (3), f (1), f ′ (1 − i), f ′′ (1 − i). [Dec 2014, Dec 2011]

ww Solution. Given
Z
3z2 + 7z + 1 1
Z
(2πi)(3z2 + 7z + 1)

w.E f (a) =
C
z−a
dz =
2πi
C
z−a
dz.

asy
Z
1 f (z)
By Cauchy’s integral formula f (a) = dz.
2πi z−a
C

En
∴ f (z) = 2πi(3z2 + 7z + 1).

C is the circle |z| = 2. gin


(i) When z = 3, |z| = 2 ⇒ |3| > 2.
∴ z = 3 lies outside the circle.

f (z)
is analytic inside and on C.
ee rin

z−3
Z
f (z)
z−3
dz = 0. g.n
C
By Cauchy’s
f (3) =
Z integral theorem,
f (2)
z−3
dz = 0 ⇒ f (3) = 0.
et
C
(ii) When z = 1, |z| = |1| < 2.
∴ z = 1 is an interior point of C.
∴ f (1) = 2πi(3 + 7 + 1) = 22πi.
(iii) f (z) = 2πi(3z2 + 7z + 1).
f ′ (z) = 2πi(6z + 7).

Downloaded From : www.EasyEngineering.net


Downloaded From : www.EasyEngineering.net

Complex Integration 417


When z = 1 − i, |z| = |1 − i| = 2 < 2.
∴ 1 − i is an interior point.

f ′ (1 − i) = 2πi(6 − 6i + 7) = 2πi(13 − 6i) = 2π(6 − 13i)

f ′′ (z) = 2πi.6 = 12πi

f ′′ (1 − i) = 12πi.

sin πz2 + cos πz2


Z
Example 4.20. Evaluate dz where C is |z| = 3.
(z − 1)(z − 2)

ww C

Solution. z = 1 and z = 2 are singular points.


[Jun 2013, Dec 2011]

w.E The circle is |z| = 3.


When z = 1, |z| = |1| < 3.

asy
∴ z = 1 lies inside the circle.

En
When z = 2, |z| = |2| < 3.
∴ z = 2 also lies inside the circle.
Let
(z −
1
Z 1)(z − 2)
=
gin
(z − 1) − (z − 2)
(z − 1)(z −Z2)
sin πz2 + cos πz2
=
1

z−2 z−1
1

sin πz2 + cos πz2


.
Z
sin πz2 + cos πz2
Now I =

=
C
Z
(z − 1)(z − 2)
f (z)
dz −
Z
f (z)
ee
dz =

dz
C
z−2
dz −
C
z−1

rin
dz

g.n
z−2 z−1
C C
I = 2πi f (2) − 2πi f (1) (1)
where f (z) = sin πz2 + cos πz2 .
When a = 2. f (2) = sin 4π + cos 4π = 1.
When a = 1, f (1) = sin π + cos π = −1.
et
(1) =⇒ I = 2πi f (2) − 2πi f (1) = 2πi.1 − 2πi(−1) = 4πi.
Z
dz 3
Example 4.21. Evaluate 2
where C is the circle |z| = .
(z + 1) (z − 2) 2
C
Solution. z = −1 and z = 2 are singular points.
C is |z| = 32 .

Downloaded From : www.EasyEngineering.net


Downloaded From : www.EasyEngineering.net

418 Engineering Mathematics - II

3
When z = −1, |z| = | − 1| = 1 < .
2
∴ z = −1 is an interior point of C.
3
When z = 2, |z| = 2 > .
2
∴ z = 2 lies outside C.
Z Z
dz f (z)
2
= dz where
(z + 1) (z − 2) (z + 1)2
C C
1
f (z) = , a = −1, n + 1 = 2.
z−2

ww f ′ (z) = −
1
(z − 2)2
, n = 1.

w.E
1
f ′ (−1) = − .
9

By Cauchy’s integral formula


Z
asyf (z)
dz = 2πi f ′
(−1) = 2πi
 −1 
=−
2πi
.
C
En
(z + 1) 2 9 9

gin
Z
z+1
Example 4.22. Evaluate dz where C is the circle |z − 2 − i| = 2.
z2 (z− 2)

ee
Z C
z+1
Solution. Let I = dz.

rin
2
z (z − 2)
C
z = 0 and z = 2 are singular points.
C is the circle |z − 2 − i| = 2.
When z = 0, |z − 2 − i| = | − 2 − i| =
√ √
4 + 1 = 5 > 2. g.n
⇒ z = 0 lies outside the circle.
When z = 2, |z − 2 − i| = |2 − 2 − i| = | − i| = 1 < 2.
et
⇒ z = 2 lies inside the circle.
Z Z z+1 Z
z+1 z2 f (z)
Now, 2
dz = dz = dz
z (z − 2) z−2 z−2
C C C
z+1
where f (z) = 2 and a = 2.
z
3
f (a) = f (2) = .
4

Downloaded From : www.EasyEngineering.net


Downloaded From : www.EasyEngineering.net

Complex Integration 419

Z
f (z) 3 3πi
By Cauchy’s Integral formula dz = 2πi f (2) = 2πi = .
z−2 4 2
C
Z
z+1
Example 4.23. By Cauchy’s integral formula, evaluate dz where C
z4 − 4z3 + 4z2
C
is the circle |z − 2 − i| = 2.
Solution. z4 − 4z3 + 4z2 = z2 (z2 − 4z + 4) = z2 (z − 2)2 .
z = 0 and z = 2 are singular points.
C is the circle |z − 2 − i| = 2.
√ √

ww When z = 0, |z − 2 − i| = | − 2 − i| =
∴ z = 0 lies outside the circle.
4 + 1 = 5 > 2.

w.E When z = 2, |z − 2 − i| = |2 − 2 − i| = | − i| = 1 < 2.


∴ z = 2 is an interior point.

Now,
Z
z+1
asy
z − 4z3 + 4z2
4
dz =
Z
z+1
z (z − 2)2
2
dz =
Z z+1
z2
(z − 2)2
dz =
Z
f (z)
(z − 2)2
dz,
C

En C

z+1
C C

where, f (z) =

gin
By Cauchy’s integral formula
z2
.
Z
f (z)
dz = 2πi f ′ (2).

We have f (z) =
ee
z+1 1 1
z2
= + 2.
z z
C
(z − 2)2

rin
1
f ′ (z) = −
2
− 3. g.n
et
z 2 z
1 2 −2 − 2 −1
f ′ (2) = − − = = .
4 8 8 2
Z
f (z) ′
 −1 
∴ dz = 2πi f (2) = 2πi = −πi.
(z − 2)2 2
C
Z
z+3
Example 4.24. Evaluate dz where C is |z| = 3. [May 2005]
2z + 5
Z C Z
z+3 1 z+3
Solution. Let I = dz = dz.
2z + 5 2 z + 25
C C

Downloaded From : www.EasyEngineering.net


Downloaded From : www.EasyEngineering.net

420 Engineering Mathematics - II

−5
z= is a singular point.
2
5 −5 5
C is |z| = 3. When z = − , |z| = = < 3.
2 2 2
−5
z= is an interior point.
2Z Z
1 z+3 1 f (z)
I= 5
dz = dz where f (z) = z + 3.
2 z+ 2 2 z + 25
C C
By Cauchy’s integral formula
1  −5   5 
I = 2πi f = iπ − + 3
2 2 2
1 πi

ww = πi = .
2 2
Z
dz

w.E
Example 4.25. Evaluate where C is the circle whose centre is (2, 0) and
z−2
C
radius 4. Z
dz
Solution. Let I =
C
z−2
asy
z = 2 is a singular point.
The circle is |z − 2| = 4.
En
gin
If z = 2, then |z − 2| = |2 − 2| = 0 < 4.
∴ z = 2 lies inside the circle C.
Now f (z) = 1, a = 2.
f (a) = f (2) = 1.
ee rin
By Cauchy’s Integral formula
Z
g.n
f (z)dz = 2πi f (a) = 2πi.
C

Example 4.26. Evaluate


Z
z
2
1
et
dz, where C is |z − 2| = using Cauchy’s
(z − 1)(z − 2) 2
C
integral formula [Jun 2012]
1
Solution. C is |z − 2| = .
2

Downloaded From : www.EasyEngineering.net


Downloaded From : www.EasyEngineering.net

Complex Integration 421

C is a circle with (2, 0) as centre and


1
radius units. 1
c
2 z=1
• •
2
• X
The singular points are z = 1 and z = 2. z=0 z=
3
2
z=2

z = 1 lies outside C and z = 2 lie within


C.
Z Z z
z

ww
z−1
∴ dz = dz
(z − 1)(z − 2)2 (z − 2)2
C C
Z
f (z) z

w.E =
C
(z − 2)2

= 2πi × f ′ (2)
dz where f (z) =

f ′ (z) =
z−1
z −1−z
(z − 1) 2
=−
1
(z − 1)2

asy
= 2πi × (−1) = −2πi. f ′ (2) =
−1
1
= −1.

En
Z
z+1

gin
Example 4.27. Evaluate dz where C is |z + 1 + i| = 2 using Cauchy’s
(z2 + 2z + 4)2
C
integral formula. [Dec 2013, May 2011]

Solution. z2 + 2z + 4 = (z + 1)2 + 4 − 1 ee rin


= (z + 1)2 + 3
√ √
= (z + 1 + 3i)(z + 1 − 3i). g.n
The singular points are −1 −
√ √
3i and −1 + 3i. et
√ √
When z = −1 − 3i, |z + 1 + i| = | − 1 − 3i + 1 + i|

= |i(1 − 3)|

= | 3 − 1| < 2.

∴ z = −1 − 3i lies inside C.

Downloaded From : www.EasyEngineering.net


Downloaded From : www.EasyEngineering.net

422 Engineering Mathematics - II

√ √
When z = −1 + 3i, |z + 1 + i| = | − 1 + 3i + 1 + i|
√ √
= |i( 3 + 1)| = 3 + 1

> 2.

∴ z = −1 + 3i lies out side C.

z+1 z+1
∴ = √ √
(z2 + 2z + 4)2 [(z + 1 3i)(z + 1) − 3i]2
z+1

ww

(z+1− 3i)2
= √
(z + 1 + 3i)2
f (z) z+1

w.E ∴
Z
z+1
=

dz =

(z + 1 + 3i)
Z
f (z)

2
where f (z) =

dz

(z + 1 − 3i)2
.

asy
2
(z + 2z + 4) 2
(z + 1 + 3i)2

= 2πi × f ′ (−1 − 3i) [By Cauchy’s integral formula]

Now, f (z) =
En z+1

(z + 1 − 3i)2

f ′ (z) = gin √ √
(z + 1 − 3i)2 − (z + 1) · 2(z + 1 − 3i)

(z + 1 − 3i)4

f ′ (−1 −

3i) =

ee
√ √





(−1 − 3i + 1 − 3i)4
(−2 3i)2 − 2(− 3i)(−2 3i)

rin

(−1 − 3i − 3i)2 − (−1 − 3i + 1) · 2(−1 − 3i + 1 − 3i)

= √
(−2 3i)4
4 × 3 × (−1) − 4 × 3(−1) g.n
Z
z+1
=

=
−12 + 12
144
16 × 9
= 0. et
∴ dz = 2πi × 0 = 0.
(z2 + 2z + 4)2
C
Z
4 − 3z
Example 4.28. Using Cauchy’s integral formula evaluate dz,
z(z − 1)(z − 2)
C
3
where C is the circle |z| = . [Jun 2010]
2

Downloaded From : www.EasyEngineering.net


Downloaded From : www.EasyEngineering.net

Complex Integration 423

Solution. The singular points are z = 0, 1, 2.


3
C is |z| = .
2
z = 0 and z = 1 lie inside C and z = 2 lies outside C.
4−3z
4 − 3z z−2 f (z) 4 − 3z
∴ = = where f (z) = .
z(z − 1)(z − 2) z(z − 1) z(z − 1) z−2
" #
1 1 f (z) f (z)
= f (z) − = − .
z−1 z z−1 z
Z Z Z
4 − 3z f (z) f (z)
∴ dz = dz − dz

ww z(z − 1)(z − 2)
C
z−1
C C
z

= 2πi × f (1) − 2πi f (0) [By Cauchy’s integral formula].

w.E Now, f (z) =


4 − 3z
z−2
4
.

asy
f (0) =

f (1) =
−2
1
= −2.

= −1.


Z
4 − 3z
En
−1
dz = 2πi(−1) − 2πi(−2) = −2πi + 4πi = 2πi.
C
z(z − 1)(z − 2)
gin I
cos πz

rectangle with vertices 2 ± i, −2 ± i.


Solution. z2 − 1 = (z + 1)(z − 1).
ee
Example 4.29. Evaluate using Cauchy’s integral formula
C
z2 − 1

rin
dz around the

Y g.n
C is the rectangle with
(2, 1), (2, −1), (−2, 1) and (-2,-1).
The singular points are z = 1& − 1, both
vertices
B(−2, 1) et
A(2, 1)

X
lie inside C.
cos πz cos πz
Now, 2
=
z − 1 (z − 1)(z + 1) C(−2, −1) D(2, −1)
" #
1 1 1
= cos πz −
2 z−1 z+1
1  cos πz cos πz 
= − .
2 z−1 z+1

Downloaded From : www.EasyEngineering.net


Downloaded From : www.EasyEngineering.net

424 Engineering Mathematics - II

 
cos πz 1  cos πz cos πz 
Z Z Z 
∴ dz =  dz − dz 
z2 − 1 2 z−1 z + 1 
C C C
 
Z Z
1  f (z) f (z) 

=  dz − dz where f (z) = cos πz.
2 z−1 z + 1 
C C
1
= [2πi × f (1) − 2πi f (−1)] [by Cauchy’s integral formula]
2
1
= 2πi[ f (1) − f (−1)]

ww 2
= πi[cos π − cos(−π)]

w.E = πi[−1 + 1] = 0.

sin2 z
Z

asy
Example 4.30. Evaluate  dz where C is the cicle |z| = 1.
π 3

C z − 6
π
Solution. The singular point is z = , which lies inside C.
Z 2
sin z
Z
En
f (z)
6


C

z− 6
 dz =
π 3
C

z− 6
gin dz
π 3
where f (z) = sin2 z.

ee
 
f ′′ π6
= 2πi × [by Cauchy’s integral formula]

= πi × f ′′
2!
π
6

.
rin
Now f (z) = sin z.2

g.n
f ′ (z) = 2 sin z cos z = sin 2z.

f ′′ (z) = 2 cos 2z.


′′ π
  2π
! π 1
et
f = 2 cos = 2 cos = 2 × = 1.
6 6 3 2
sin2 z
Z
∴  dz = πi × 1 = πi.
π 3

C z − 6

ez
Z
Example 4.31. Evaluate dz where C is the circle |z| = 4.
(z2 + π2 )2
C

Downloaded From : www.EasyEngineering.net


Downloaded From : www.EasyEngineering.net

Complex Integration 425

Solution. The singular points are given by


z2 + π2 = 0
z2 = −π2
z = ±πi, both lie within the circle |z| = 4.

1 1 1
Now, = = .
(z2 + π2 )2 [(z + πi)(z − πi)]2 (z + πi)2 (z − πi)2
1 A B C D
Let 2 2
= + 2
+ +
(z + πi) (z − πi) z + πi (z + πi) z − πi (z − πi)2

ww =
A(z + πi)(z − πi)2 + B(z − πi)2 + C(z − πi)(z + πi)2 + D(z + πi)2
(z + πi)2 (z − πi)2
∴ 1 = A(z + πi)(z − πi)2 + B(z − πi)2 + C(z − πi)(z + πi)2 + D(z + πi)2 .

w.E When z = πi, D(2πi)2 = 1

D(−4π2 ) = 1

D=−
1
. asy
4π2

En
When z = −πi, B(−2πi)2 = 1

4π2 (−1)B = 1
gin
ee
1
B=− .
4π2

Equating the coefficients of z3 , we get


rin
A + C = 0.
When z = 0, g.n (1)

πi(−πi)2 A + (−πi)2 B + (−πi)(πi)2C + (πi)2 D = 1

− π3 iA − π2 B + π3 iC − π2 D = 1
et
! !
3 2 1 3 2 1
− π iA − π − 2 + π iC − π − 2 = 1
4π 4π
1 1
− π3 iA + + π3 iC + = 1
4 4
3 3 1
− π iA + π iC = (2)
2

Downloaded From : www.EasyEngineering.net


Downloaded From : www.EasyEngineering.net

426 Engineering Mathematics - II

From (1) C = −A.

1
∴ −π3 iA + π3 i(−A) =
2
3 1
−2π iA =
2
1
A=− .
4π3 i
1
C= 3 .
4π i
1 1 1 1 1 1 1 1 1
=− 3 · − 2 + 3 · − 2 .

ww
2
(z + πi) (z − πi) 2 4π i z + πi 4π (z + πi) 2 4π i z − πi 4π (z − πi)2
ez ez ez ez
Z Z Z Z
1 1 1
∴ dz = − dz − dz + dz
(z2 + π2 ) 4π3 i z + πi 4π2 (z + πi)2 4π3 i z − πi

w.E C C

− 2
1

ZC
ez
(z − πi)2
dz.
C

asy
1
4π i
1

C
1
= − 3 × 2πi f (−πi) − 2 × 2πi f ′ (−πi) + 3 × 2πi f (πi)
4π i

En 1
− 2 × 2πi × f ′ (πi), where f (z) = ez .

(1)

Now, f (z) = ez . gin


f ′ (z) = ez .

f (−πi) = e−πi .
ee rin
f (πi) = e .πi

g.n
Now from (1) we have
f ′ (πi) = eπi .

f ′ (−πi) = e−πi . et
ez
Z
1 1 1 1
2 2 2
dz = − 3 × 2πie−πi − 2 × 2πie−πi + 3 × 2πieπi − 2 × 2πieπi
(z + π ) 4π i 4π 4π i 4π
C
1 −πi i 1 i
=− 2
e − e−πi + 2 eπi − eπi
2π 2π 2π 2π
1  πi −πi
 i  πi −πi

= 2 e −e − e +e
2π 2π

Downloaded From : www.EasyEngineering.net


Downloaded From : www.EasyEngineering.net

Complex Integration 427

1 i
2
=2i sin π − · 2 cos π
2π 2π
i
= − (−1)
π
i
= .
π
etz
I
Example 4.32. Evaluate where C is the circle |z| = 3.
z2 + 1
C
Solution. The singular points are given by
z2 + 1 = 0

ww z2 = −1
z = ±i, both lie inside |z| = 3.

w.E 1
=
1
=
1
"

z2 + 1 (z + i)(z − i) 2i z − i z + i
1

1
#

etz
asy
1 etz etz
" #
= −
z2 + 1 2i z − i z + i

En
 
Z
e tz Z
1  etz Z
etz 
dz =  dz − dz

∴ 2
z +1 2i z−i z+i 

gin

C C C
1
= [2πi × f (i) − 2πi × f (−i)] where f (z) = etz .
2i
1
2i ee
= × 2πi[ f (i) − f (−i)]

= π[ f (i) − f (−i)]. rin (1)

Now f (z) = etz .


g.n
f (i) = eit

f (−i) = e−it .

f (i) − f (−i) = eit − e−it


et
= 2i sin t.

Now, (1) becomes


etz
Z
dz = π[2i sin t] = 2πi sin t.
z2 + 1
C

Downloaded From : www.EasyEngineering.net


Downloaded From : www.EasyEngineering.net

428 Engineering Mathematics - II

e2z
I
Example 4.33. Evaluate dz, where C is the circle |z| = 3. [Jan 2016]
(z + 1)4
C
Solution. The singular point is z = −1, which lies completely inside the circle C,
|z| = 3.
By Cauchy’s integral formula
e2z
Z Z
f (z)
4
dz = dz where f (z) = e2z .
(z + 1) (z + 1)4
C C
f ′′′ (−1)
= 2πi × (1)

ww Now, f (z) = e2z .


3!

w.E f ′ (z) = 2e2z .

f ′′ (z) = 4e2z .

asy
f ′′′ (z) = 8e2z .

f ′′′ (−1) = 8e−2

Now, (1) becomes En


Z
e2z
(z + 1)4
dz = 2πi ×
6 gin
8e−2 8 −2
= πie .
3

Exercise 4 B
C

ee rin
g.n
Z
dz
1. Evaluate where C is the circle |z| = 2. [Dec 2004]
z+4

et
C

cos πz
Z
2. Evaluate dz where C is |z| = 2. [May 2003]
z−1
C

3z2 + 7z + 1
Z
1
3. Evaluate dz where C is |z| = . [Dec 2003]
z+1 2
C
Z
1
4. Use Cauchy’s integral formula to evaluate dz where C is the circle
z2 −1
C
with centre 1 and radius = 1. [May 2000]

Downloaded From : www.EasyEngineering.net


Downloaded From : www.EasyEngineering.net

Complex Integration 429

Z
z
5. Evaluate dz where C is the circle |z| = 2. [May 1999]
(z − 1)3
C

e−z
Z
6. Evaluate dz where C is the circle |z| = 2.
z+1
C

sin πz2 + cos πz2


Z
7. Evaluate dz where C is |z| = 4. [May 2000]
(z − 2)(z − 1)
C

sin πz2 + cos πz2


Z
8. Use Cauchy’s integral formula, evaluate dz where C is

ww π
|z| = .
2
C
(z + 1)(z + 2)

[Dec 2011]

w.E 9. Evaluate
Z

(z2
1
+ 4)2
dz where C is |z − i| = 4. [May 2003]

asy
C
Z
z 1
10. Evaluate dz where C is |z − 2| = . [May 2004]

En
(z − 1)(z − 2) 2
C

gin
sin πz2 + cos πz2
Z
11. Evaluate dz where C is |z| = 3. [Dec 2003]
(z − 1)2 (z − 2)
C

12. Evaluate
Z

C
ez
(z + 2)(z + 1)2 ee
dz where C is |z| = 3.

rin
[May 2007]

g.n
Z
z 1
13. Evaluate 2
dz where C is |z − 2| = . [Dec 2009, Jun 2012]
(z − 1)(z − 2) 2

et
C
Z
1
14. Evaluate dz where C is the circle |z| = 1. [May 2009]
2z − 3
C
Z
1
15. Evaluate dz where C is the circle |z| = 1. [Dec 2009]
zez
C

z2 + 5
Z
1
16. Evaluate using Cauchy’s integral formula dz on the circle
2πi z−3
C
(i) |z| = 4 and (ii) |z| = 1. [May 2010]

Downloaded From : www.EasyEngineering.net


Downloaded From : www.EasyEngineering.net

430 Engineering Mathematics - II

Z
sin 3z
17. Evaluate using Cauchy’s integral formula dz if C is the circle |z| = 5.
z + π2
C
[Apr 2007]
e2z
Z
18. Evaluate dz where C is the circle |z| = 3. [Nov 2011]
(z − 1)(z − 2)
C

3z2 + z
Z
19. Evaluate dz where C is the circle |z − 1| = 1. [Apr 2008]
z2 − 1
C

sin6 z

ww
Z
20. Evaluate  dz if C is the circle |z| = 1. [Nov 2010]
z − π6 3
C

w.E
Z
z+1
21. Evaluate dz where C is the unit circle |z| = 1. [May 2010]
z3− 2z2
C

22. Evaluate
Z
tan 2z
(z − a)2 asy
dz, −2 < a < 2 where C is the boundary of the square

En
C
whose sides are x = ±2, y = ±2. [Apr 2010]

gin z2 + 7
Z
23. Evaluate using Cauchy’s integral formula dz on the circle (i) |z| = 3
z−2
C
and (ii) |z| = 1.

24. Evaluate
Z
2
1
z + 4z + 1
ee
dz where C is |z| = 1.
rin
[May 2003]

[Apr 2008]

g.n
C

4.5 Taylor’s and Laurent’s series

4.5.1 Power series et


A series of the form

X
an (z − a)n = a0 + a1 (z − a) + a2 (z − a)2 + · · ·
n=0

where z is a complex variable, a0 , a1 , a2 , . . . and a are complex constants is called a


power series in powers of z − a or a power series about the point a.

Downloaded From : www.EasyEngineering.net


Downloaded From : www.EasyEngineering.net

Complex Integration 431

a0 , a1 , a2 , . . . are called the coefficients of the series and a is called the centre of the
series.
Result 1. The power series converges at all points inside a circle |z − a| = R for
some positive number R and diverges outside the circle. This circle is called the
circle of convergence and its
radius R is called the radius of convergence.
an+1
Result 2. Let lim = ℓ. Then, the radius of convergence of the series
n→∞ an

X 1
an (z − a)n is R = .
n=0

ww Result 3. If ℓ = 0, then R = ∞ and so, the power series converges for all z in the
finite plane.

w.E
an+1
Result 4. If → ∞, then R = 0 and hence the series converges only at the
an
centre z = a.

asy
Result 5. If a = 0, then we get a particular power series in powers of z or power
series about the origin.

En

X
an zn = a0 + a1 z + a2 z2 + · · ·

Taylor’s Series gin


n=0

ee
If f (z) is analytic inside a circle C with centre at a then at each point z inside
the circle, f (z) has the power series representation
rin
f (z) = f (a) +
f ′ (a)
1!
(z − a) +
f ′′ (a)
2!
(z − a)2 + · · · +
f n (a)
n! g.n
(z − a)n + . . . ∞.

Note. If a = 0, we get the Taylor’s series of f (z) about the origin which is called
the Maclaurin’s series.
et
f ′ (0) f ′′ (0) 2 f (n) n
i.e., f (z) = f (0) + z+ z + ··· + z + ··· .
1! 2! n!

Maclaurin’s series for some elementary functions


z z2 z3
1. ez = 1 + + + + · · · if |z| < ∞.
1! 2! 3!

Downloaded From : www.EasyEngineering.net


Downloaded From : www.EasyEngineering.net

432 Engineering Mathematics - II

2. (1 − z)−1 = 1 + z + z2 + z3 + + · · · if |z| < 1.

3. (1 + z)−1 = 1 − z + z2 − z3 + + · · · if |z| < 1.

4. (1 − z)−2 = 1 + 2z + 3z2 + 4z3 + + · · · if |z| < 1.


z2 z3 z4
5. log(1 + z) = z − + − + + · · · if |z| < 1.
2 3 4
z3 z5
6. sin z = z − + + · · · if |z| < ∞.
3! 5!

ww 7. cos z = 1 −
z2 z4
+ + · · · if |z| < ∞.
2! 4!

w.E
Laurent’s series. If f (z) is analytic in
the ring shaped region R bounded by two
R
r1

asy C1

>
concentric circles C1 and C2 of radii r1 a r2

En
and r2 (r1 > r2 ) and with centre at a, then > C2

for all z in R, f (z) has the power series


representation.
gin
f (z) =

X

n=0
ee
an (z − a)n +

X

n=1
bn (z − a)−n , r2 < |z − a| < r1

rin (1)

g.n
I
1 f (w)
where an = dw, n = 1, 2, 3, . . ..
2πiI C1 (w − a)n+1
1 f (w)
bn =
2πi C2 (w − a)1−n
dw, n = 1, 2, 3, . . ..

The series (1) is called Laurent’s series about z = a. Laurent’s series is a


et
series with positive and negative integral powers of (z − a). The part of the series
with positive powers of z − a is called the analytic part or regular part and the
part with negative powers of z − a is called the principal part of the Laurent’s
series.

Downloaded From : www.EasyEngineering.net


Downloaded From : www.EasyEngineering.net

Complex Integration 433

✎ ☞
Worked Examples
✍ ✌
Example 4.34. Expand f (z) = sin z in a Taylor’s series about the origin.
Solution. f (z) = sin z. f (0) = sin 0 = 0.

f ′ (z) = cos z. f ′ (0) = 1.

f ′′ (z) = − sin z. f ′′ (0) = 0.

f ′′′ (z) = − cos z. f ′′′ (0) = −1.

f ′v (z) = sin z. f ′v (0) = 0.

ww ... ...

w.E By Taylor’s series we have


f ′ (0)
f (z) = f (0) + z+
f ′′ (0) 2 f ′′′ (0) 3
z + z + ···

asy
1! 2! 3!
1 1 1
sin z = 0 + z − z3 + z5 + · · ·
1! 3! 5!
z3 z5
sin z = z − + + · · ·
3! 5!
En
π ginπ 1
π
Example 4.35. Find the Taylor’s series for f (z) = sin z about z = .
4
[Jun 2009]

Solution. f (z) = sin z,


f (z) = cos z, f ′
f
π
4
4 4
πee
= sin = √

= cos = √
4
1
2
2

rin
′′ π π 1

g.n
 
′′
f (z) = − sin z, f = − sin = − √
4 4 2
′′′ π π 1
 
′′′
f (z) = − cos z,

′v
f (z) = − sin z,
... ...
f
f

′v
π
4
4
= − cos = − √
π
= sin = √ .
4
4
1
2
2
et
By Taylor’s series we have
     
 π  f ′ π4  ′′ π
π f 4 
′′′ π
π 2 f 4  π 3
f (z) = f + z− + z− + z− + ···
4 1! 4 2! 4 3! 4
1 1  π 1  π 2 1  π 3
sin z = √ + √ z − − √ z− − √ z− + ···
2 2 4 2 2 4 6 2 4

Downloaded From : www.EasyEngineering.net


Downloaded From : www.EasyEngineering.net

434 Engineering Mathematics - II

Example 4.36. Obtain the expansion of log(1 + z) when |z| < 1. [May 2006]
Solution. Since |z| < 1, z = 0 is a regular point of f (z).
f (z) = log(1 + z), f (0) = log 1 = 0
′ 1 ′
f (z) = , f (0) = 1
1+z
1
f ′′ (z) = − , f ′′ (0) = −1
(1 + z)2
2
f ′′′ (z) = , f ′′′ (0) = 2
(1 + z)3
6
f ′v (z) = − , f ′v (0) = −6.

ww
(1 + z)4
... ...
The Taylor’s series expansion about z = 0 is

w.E f (z) = f (0) +


f ′ (0)
1!
z+
f ′′ (0) 2 f ′′′ (0) 3
2!
z +
3!
z + ···

asy 1 1 2 6
log(1 + z) = 0 + z − z2 + z3 − z4 + · · ·
1 2! 3! 4!

En
log(1 + z) = z −
z2 z3 z4
+ − + ··· .

gin
2 3 4
1
Example 4.37. Expand at z = 1 in Taylor’s series.

ee
z−2
1
Solution. Let f (z) = .
z−2
z = 1 is a regular point.
rin
∴ We can find Taylor’s series about z = 1.
∴ The series is g.n
f (z) = f (1) +
f ′ (1)
1!
(z − 1) +

1
f ′′ (1)
2!
(z − 1)2 +
f ′′′ (1)
3! et
(z − 1)3 + · · ·

Now, f (z) = = (z − 2)−1 f (1) = −1


z−2
f ′ (z) = −1(z − 2)−2 f ′ (1) = −1

f ′′ (z) = 2(z − 2)−3 f ′′ (1) = −2

f ′′′ (z) = −6(z − 2)−4 f ′′′ (1) = −6

Downloaded From : www.EasyEngineering.net


Downloaded From : www.EasyEngineering.net

Complex Integration 435

1 2 6
f (z) = −1 − (z − 1) − (z − 1)2 − (z − 1)3 − . . .
1! 2! 3!
t= −[1 + z − 1 + (z − 1)2 + (z − 1)3 + . . . ].

1
Example 4.38. Obtain the Taylor’s series to represent the function
(z + 2)(z + 3)
in the region |z| < 2. [Apr 2006]
1
Solution. Let f (z) =
(z + 2)(z + 3)
z + 3 − (z + 2)
=
(z + 2)(z + 3)
z+3 z+2

ww =

f (z) = −
z+2 z+3
1
.

(z + 2)(z + 3) (z + 2)(z + 3)
1

w.E z
z 2
Since |z| < 2 we have < 1 and < < 1.
2 3 3
1 1 z z
Hence and can be expanded as powers of and .
z+2
asy
1
z+3
1
2 3

∴ f (z) = 

En
2 1 + 2z
− 
3 1 + 3z


1
= 1+
2"
1
2
gin
z −1 1 
− 1+
z z2 z3
3
z −1
3 #
1
"
z z2 z3
#
=
2 2 4
1 z z2 z3
= − + −
2 4 8 16
ee
1 − + − + ··· − 1 − + −
8

+ ··· − + −
3
1 z z2
3 9 27 81
3 9 27

+
z3
+ ···
+ ···

rin
1
f (z) = − z +
6 36
5 9 2
216
z −
65 3
1296
z + ··· .
g.n
et
Since the expansion contains only positive powers of z, this is the required Taylor’s
series of f (z).

z2 − 1
Example 4.39. Find the Taylor’s series to represent the function in
(z + 2)(z + 3)
|z| < 2. [Jun 2010, Dec 2015]
z2 − 1
Solution. Let f (z) = .
(z + 2)(z + 3)
The singular points are z = −2 and z = −3.
Both the singular points lie outside |z| < 2.

Downloaded From : www.EasyEngineering.net


Downloaded From : www.EasyEngineering.net

436 Engineering Mathematics - II

∴ f (z) is analytic in the open disc |z| < 2.


Hence f (z) can be expanded as a power series about z = 0.
Splitting f (z) into partial fractions we get

z2 − 1 B C
= A+ +
(z + 2)(z + 3) z+2 z+3

z2 − 1 = A(z + 2)(z + 3) + B(z + 3) + C(z + 2).

ww Put z = −2, we obtain B = 3.

w.E Put z = −3, we obtain C = −8


Equating the coeff. of z2 we get A = 1.


z2 − 1 asy =1+
3

8
(z + 2)(z + 3)
f (z) = 1 + 
3
En z+2 z+3
 −8 
1

gin

2 1 + 2z 3 1 + 3z
3 z −1 8  z −1
=1+ 1+ − 1+
2
3
2
z z2 z3
3
ee
= 1 + 1 − + − ··· − 1 − + −
1
2
5
2 4 8
17 2 115 3
3
 8
3
z z2 z3
3 9 27
···


rin
=− + z+
6 36 216
z −
1296
z + ··· .

g.n
Example 4.40. Expand f (z) =
(ii) |z| > 3.
z2 − 1
(z + 2)(z + 3) et
as a Laurent’s series if (i) 2 < |z| < 3
[May 2015, Jun 2013, Dec 2011, May 2011, May 2009]
z2 − 1 3 8
Solution. By the previous problem we have =1+ − .
(z + 2)(z + 3) z+2 z+3
Given 2 < |z| < 3. This region is annular about z = 0 and f (z) is analytic
in this region.
∴ f (z) has a Laurent’s series about z = 0.
z
|z| < 3 ⇒ < 1.
3

Downloaded From : www.EasyEngineering.net


Downloaded From : www.EasyEngineering.net

Complex Integration 437


z 2
|z| > 2 ⇒ > 1 ⇒ < 1
2 z
3 8
Now, f (z) = 1 + −
z+2 z+3
3 8
=1+  −  
z 1+ z2
3 1 + 3z
3 2 −1 8  z −1
=1+ 1+ − 1+
z z 3 3
3  2 4 8  8 z z3 z3 
= 1 + 1 − + 2 − 3 + ··· − 1 − + − + ···
z z z z 3 3 9 27

ww 8 z z2 z3
= 1 + 3(z−1 − 2z−2 + 4z−3 − 8z−4 + · · · ) − 1 − + −
3 3 9 27

+ ··· .

w.E (ii) |z| > 3. This


z
region
is annular
3
about z = 0 where f (z) is analytic.
1
|z| > 3 ⇒ > 1 ⇒ < 1 ⇒ <
1

asy
3 z z 3
2 2
< <1
|z| 3

f (z) = 1 +
3
En

8

gin
z+2 z+3
3 8
=1+  −  
z 1 + 2z z 1 + 3z

=1+

=1+
3
z
3
1+
z
2 4
ee
2 −1 8 
− 1+
z
8
3 −1
z
 8 3 9 27
1 − + 2 − 3 + ··· − 1 − + 2 − 3 + ···

rin
=1+
z z z
3 6 12 24
− +
z 2 z z
8 24 72
− 4 + ··· − + 2 − 3 + ···
z
g.n
2 z2 z3 z
= 1 − 5z−1 + 16z−2 − 60z−3 · · · .

1
z z z

et
Example 4.41. Expand in the region (i) 1 < |z| < 2 (ii) 0 < |z − 1| < 2.
z2
− 3z + 2
[May 2002]
1 1 z − 1 − (z − 2) 1 1
Solution. 2 = = = − .
z − 3z + 2 (z − 1)(z − 2) (z − 1)(z − 2) z−2 z−1

Downloaded From : www.EasyEngineering.net


Downloaded From : www.EasyEngineering.net

438 Engineering Mathematics - II

2
Since 1 < |z| < 2, then z lies in the
z
annular region about z = 0 where f (z)
1
is analytic.
Hence, we can expand f (z) as a
Laurent’s series about z = 0.

z 1
Since |z| < 2, < 1 and 1 < |z| ⇒ |z| > 1 ⇒ < 1.
2 |z|

ww Hence, f (z) can be written as,


1 1

w.E
f (z) =  −  
−2 1 − 2z z 1 − 1z
1 z −1 1  1 −1
=− 1− − 1−
1
2

asy
2
z z2 z3
z z
 1
= 1 + + + + ··· − 1 + + 2 + 3 + ···
2 2 4 8 z
1 1
z z
1
z


1  z z
En2 z 3   
= − 1 + + + + · · · − z−1 + z−2 + z−3 + z−4 + · · · .

gin
2 2 4 8
1 1
(ii) We have f (z) = − .
z−2 z−1
Given 0 < |z − 1| < 2.
ee
This region is annular about z = 1.
rin
g.n
∴ The expansion of f (z) in this region is a Laurent’s series about z = 1.
Put t = z − 1 ⇒ z = t + 1.
∴ The region is o < |t| < 2.

f (z) =
1

1
=
1

z−2 z−1 t+1−2 t+1−1
1 et
1 1 1
= − = −t−1 −
t−1 t 1−t
= −t−1 − (1 − t)−1 [ Since |t| < 2 ⇒ |t| < 1]

= −(z − 1)−1 − [1 + t + t2 + t3 + · · · ]
 
= − (z − 1)−1 − (1 + z − 1 + (z − 1)2 + (z − 1)3 ) + · · · .

Downloaded From : www.EasyEngineering.net


Downloaded From : www.EasyEngineering.net

Complex Integration 439

1
Example 4.42. Expand in the region |z| > 2.
z2
− 3z + 2
1 1
Solution. By the previous problem we have f (z) = − Given |z| > 2.
z−2 z−1
This region is annular about z = 0 where f (z) is analytic.
∴ The expansion is a Laurent’s
series about z = 0.
z 2
Since |z| > 2, > 1 ⇒ < 1.

2 z
1 1
Also < < 1.
|z| 2

ww Hence, f (z) = −
1

1
+
z−1 z−2
1

w.E
=−  +  
1
z 1− z z 1 − 2z
1 1 −1 1  2 −1
=− 1− + 1−
z

asy
1
z
1 1
z z
z

z
1
z
 1
z
2 4
= − 1 + + 2 + 3 + ··· + 1 + + 2 + 3 + ···
z z z
8
z


1 1
En 1 1 1 2 4
= − − 2 − 3 − 4 − ··· + + 2 + 3 + 4 ···
z z z z z z z
8
z

gin
= z−2 + 3z−3 + 7z−4 + . . . .

Example 4.43. Expand f (z) =


1 < |z| < 3 and |z| > 3.
1
ee 1
(z + 1)(z + 3)

A
in Laurent’s series valid for the regions

B rin [Jun 2012]


Solution. Let f (z) = = +
(z + 1)(z + 3) z + 1 z + 3
.
g.n
1 = A(z + 3) + B(z + 1).
1
When z = −1, 2A = 1, A = .
2
et
1
When z = −3, −2B = 1, B = − .
2
1 1 1 1 1
∴ = · − · .
(z + 1)(z + 3) 2 z + 1 2 z + 3

(i) Consider 1 < |z| < 3.


This is annular about z = 0 and f (z) is analytic in the region.

Downloaded From : www.EasyEngineering.net


Downloaded From : www.EasyEngineering.net

440 Engineering Mathematics - II

∴ f (z) has a Laurent’s series expansion about z = 0.

z
|z| < 3 ⇒ | | < 1.
3
1
|z| > 1 ⇒ < 1.
|z|
1 1 1 1
∴ f (z) = ·  − ·  
2 z 1+ 1 2 3 1+ z
z 3
!−1
1 1 1 z −1
= 1+ − 1+
2z z 6 3

ww z z2 z3 z4
" # " #
1 1 1 1 1 1
= 1 − + 2 − 3 + 4 ··· − 1 − + − + ···
2z z z z z 6 3 9 27 81

w.E
(ii) Consider the region |z| > 3.
This region is annular about z = 0, where f (z) is analytic.

asy
1 1 3
|z| > 3 ⇒ < ⇒ < 1.
|z| 3 |z|
1 1
Since < < 1, we have
|z| 3
En
f (z) =
1
· 
1
2 z 1+ 1
z gin
1
− · 
1
2 z 1+ 3
z


ee
!−1 !−1
1 1 1 3
= 1+ − 1+

rin
2z z 2z z
" # " #
1 1 1 1 1 1 3 9 27 81
= 1 − + 2 − 3 + 4 ··· − 1 − + 2 − 3 + 4 ··· .
2z z z z

1
z 2z z z z z

g.n
Example 4.44. Obtain the series for
(ii) |z| > 3.
Solution. Let f (z) =
1
z−3
.
z−3
valid in (i) |z| < 3

et
z = 3 is a singular point.
(i) Given |z| < 3
f (z) is analytic in the region |z| < 3.
∴ f (z) has a Taylor’s series
about z = 0.
z
Since |z| < 3, we have < 1.

3

Downloaded From : www.EasyEngineering.net


Downloaded From : www.EasyEngineering.net

Complex Integration 441

Hence, the Taylor’s series about z = 0 is


1 1
f (z) = =  
z − 3 −3 1 − z
3
−1  z −1
= 1−
3 3
1 z z2 z3 
=− 1+ + + ··· .
3 3 9 27
(ii) Given |z| > 3.
f (z) is analytic in the annular region 3 < |z| < ∞

ww ∴ f (z) has a Laurent’s series expansion.


1 1 3

w.E
As |z| > 3, < < 1 =⇒ < 1.
|z| 3 |z|
1 1
f (z) = = 

asy

z−3 z 1− 3
z
1 3 −1 1 3 9 27 
= 1− = 1 + + 2 + 3 + ···
z

En z z z z z
= z−1 + 3z−2 + 9z−3 + 27z−4 + · · · .

gin
Example 4.45. Find the Laurent’s series expansion of f (z) =
7z − 2
in

ee
z(z − 2)(z + 1)
1 < |z + 1| < 3. [Jun 2010, May 2005]
Solution. The singular points are z = 0, z = −1, z = 2, which lies outside the
annular region 1 < |z + 1| < 3. rin
∴ We can expand f (z) as a Laurent’s series about z = −1 or interms
g.n
of z + 1.
Let f (z) =
7z − 2
z(z − 2)(z + 1)
A
= +
B
+
C
z z−2 z+1
. et
7z − 2 = A(z − 2)(z + 1) + Bz(z + 1) + Cz(z − 2)

When z = 0 we obtain −2 = −2A ⇒ A = 1.


When z = −1 we have −9 = 3C ⇒ C = −3.
When z = 2 we obtain 6B = 12 ⇒ B = 2.
Let t = z + 1.

Downloaded From : www.EasyEngineering.net


Downloaded From : www.EasyEngineering.net

442 Engineering Mathematics - II

∴ 1 < |z + 1| < 3 ⇒ ∴ 1 < |t| < 3,


1
|t| > 1 ⇒ < 1.
t
t
|t| < 3 ⇒ < 1.
3

1 2 3 1 2 3
f (z) = + − = + −
z z−2 z+1 t−1 t−3 t
1 2 3 1 1 −1 2  t −1 3
=  +   − = 1− − 1− −
t 1 − 1t −3 1 − 3t t t t 3 3 t
1 1 1 1  2 t t2 t3  3

ww = 1 + + 2 + 3 + ··· − 1 + + +
t
2 1
t t
1
t
1
= − + 2 + 3 + 4 + ··· − 1 + + +
2
3 3 9 27
t t2 t3
+ ··· −

+ ···

t

w.E t t t t 3 3 9 27
= −2(z + 1)−1 + (z + 1)−2 + (z + 1)−3 + · · ·
z + 1 (z + 1)2 (z + 1)3
2

asy

− 1+ + + + ··· .
3 3 9 27

Example 4.46. Expand f (z) =


En z
(z − 1)(z − 3)
as Laurent’s series valid in the regions

gin
(i) 1 < |z| < 3 (ii) 0 < |z − 1| < 2 (iii) |z| > 3. [ Dec 2001, Apr 1998]
Solution. The singular points are z = 1 and z = 3.
z A B
Let =
z = A(z − 3) + B(z − 1).
+
(z − 1)(z − 3) z − 1 z − 3

Put z = 1 =⇒ 1 = −2A ⇒ A =
−1
.
ee rin
Put z = 3 =⇒ 3 = 2B ⇒ B = .
3
2
2

g.n
∴ f (z) =
− 21 3
+ 2 .
z−1 z−3
et
(i) Consider 1 < |z| < 3
This region is annular about z = 0 and f (z) is analytic in 1 < |z| < 3

|z| > 1 =⇒ 1 < 1. Also |z| < 3 =⇒ z < 1
z 3

1 1 3 1
Now, f (z) = −  +  
2z 1− 1 2(−3) 1 − z
z 3

Downloaded From : www.EasyEngineering.net


Downloaded From : www.EasyEngineering.net

Complex Integration 443

1 1 −1 1  z −1
=− 1− − 1−
2z z 2 3
1  1 1 1  1 z z2 z3 
= − 1 + + 2 + 3 + ··· − 1 + + + + ···
2z z z z 2 3 9 27
1   1  z z2 z3 
= − z−1 + z−2 + z−3 + z−4 + · · · − 1 + + + + ··· .
2 2 3 9 27
(ii) Consider 0 < |z − 1| < 2.
This region is annular about z = 1 which does not contain z = 1 and
z = 3.

ww ∴ f (z) is analytic in 0 < |z − 1| < 2.


t
Let t = z − 1. Now, 0 < |z − 1| < 2 =⇒ 0 < |t| < 2 ⇒ < 1
2

w.E ∴ f (z) =
−1 1 3
2 t 2t+1−3
+
1

=
asy
−1 1 3 1
2 t 2t−2
11 3 1
+

=−

En − 
2 t 4 1− t
2


=− − 1−
2t 4
t −1gin
1 3

3
2
t −1

t t2 t3

ee
 
=− − 1 + + + + ···
2 4 2 4 8
=−
(z − 1)−1 3 
2
− 1+
4
z − 1 (z − 1)2 (z − 1)3
2
+
4
+
8
rin 
+ ··· .

g.n

z
(iii) Consider |z| > 3 ⇒ > 1
3

et

1 < 1 < 1 ⇒ 3 < 1.
z 3 z
This region is annular about z = 0.
f (z) is analytic in |z| > 3.
1 1 3 1
f (z) = −  +  
2z 1− 1 2z 1− 3
z z
1 1 −1 3  3 −1
=− 1− + 1−
2z z 2z z
1  1 1 1  3 3 9 27 
= − 1 + + 2 + 3 + ··· + 1 + + 2 + 3 + ···
2z z z z 2z z z z

Downloaded From : www.EasyEngineering.net


Downloaded From : www.EasyEngineering.net

444 Engineering Mathematics - II

1 1 1 1 3 9 27 81
=− − 2 − 3 − 4 + ··· + + 2 + 3 + 4 + ···
2z 2z 2z 2z 2z 2z 2z 2z
1 4 13 40
= + 2 + 3 + 4 + ··· .
z z z z
1
Example 4.47. Find the Laurent’s series of f (z) = valid in the region
z(1 − z)
(i) |z + 1| < 1 (ii) |z + 1| > 2. [Dec 2011, Jun 2008]
1 1−z+z 1−z z 1 1
Solution. f (z) = = = + = + .
z(1 − z) z(1 − z) z(1 − z) z(1 − z) z 1 − z
The singular points are z = 0, z = 1.

ww
w.E
(i) Consider |z + 1| < 1
The singular points lie outside the open
disc with centre z = −1.

z = −1 z=0

z=1

asy
∴ f (z) is analytic in |z + 1| < 1.

En
gin
∴ We can expand f (z) as a Laurent’s series about z = −1.
Let t = z + 1.

ee

∴ The region becomes |t| < 1 =⇒ 2t < 1
2 < 1.

f (z) =
1
+
t−1 2−t
1
rin
=−
1
+ 
1−t 2 1− t
1
2
1
 = −(1 − t)−1 + 1 −
2
t −1
2
g.n
1
= −(1 + t + t2 + t3 + · · · ) +
2
t t2 t3
1 + + + + ···
2 4
1
= −1 − (z + 1) − (z + 1)2 − (z + 1)3 + +
z
8
+


1 (z + 1)2 (z + 1)3
+ + + ...
et
2 4 8 16
1 3 7 15
= − − (z + 1) − (z + 1)2 − (z + 1)3 + · · ·
2 4 8 16
1 3 7 15 
= − 1 + (z + 1) + (z + 1) + (z + 1)3 · · · .
2
2 2 4 8

Downloaded From : www.EasyEngineering.net


Downloaded From : www.EasyEngineering.net

Complex Integration 445

(ii) Consider the region |z + 1| > 2.


If |z + 1| > 2, f (z) is analytic in the
annular region about z = −1.
• •
∴ f (z) can be expanded as a Laurent’s z = −2 z = −1 z=0 z=1 z=2

series about z = −1.


Let t = z + 1.

ww region is |t| > 2.


∴ The
t
⇒ > 1.
2

w.E

2 1 1
⇒ < 1, < < 1.

t t 2
1 1
f (z) =

=  asy
t−1 2−t
1
+

+ 
1 1
 = 1−
1 −1 1 
− 1−
2 −1
t 1 − 1t
1 En
1 1
t 2t − 1
1
t
 1
t t
2 4
t
8

gin

= 1 + + 2 + 3 + ··· − 1 + + 2 + 3 + ···
t t t t t t t t
1 1 1 1 1 2 4 8
= + 2 + 3 + 4 + ··· − − 2 − 3 − 4 ···
t t
1 3
t
7
ee
t

t t t t

rin

= − 2 − 3 − 4 − · · · = − (z + 1)−2 + 3(z + 1)−3 + 7(z + 1)−4 + · · · .
t t t
z
Example 4.48. Find the Laurent series expansion of f (z) = 2
1 < |z| < 2. g.n
(z + 1)(z2 + 4)
in
[Dec 2005]
Solution.

The singular points are z = ±i, z = ±2i


(0, 2)
et
(0, 1)
which lies outside 1 < |z| < 2.
∴ f (z) is analytic in this annular region
z = 0.
(0, −1)
z Az + B Cz + D
Let = 2 + 2 .
(z2 + 1)(z2 + 4) z +1 z +4 (0, −2)
2 2
z = (Az+B)(z +4)+(Cz+D)(z +1).

Downloaded From : www.EasyEngineering.net


Downloaded From : www.EasyEngineering.net

446 Engineering Mathematics - II

Equating the coeff. of z3 =⇒ A + C = 0. (1)


Equating the coeff. of z2 =⇒ B + D = 0. (2)
Equating the coeff. of z =⇒ 4A + C = 1. (3)
Equating the constants =⇒ 4B + D = 0. (4)
From (2) and (4) we obtain B = D = 0
(1) ⇒ C = −A
(3) ⇒ 4A − A = 1 ⇒ 3A = 1 ⇒ A = 13 , C = −1
3
z 1 z 1 z
∴ 2 = −

ww (z + 1)(z2 + 4) 3 z2 + 1
Consider |z| > 1 ⇒
1
< 1 =⇒
|z|2
3 z2 + 4
1
|z2 |
<1

w.E

z z
|z| < 2 ⇒ < 1 ⇒ < 1
2 4

=
asy
1

z
3 z2 1 + 1
z2
−
1  z 
3(4) 1 + z2
4

=
1
3z
En
1+
1 −1
z2

z 
12
1 +
z2 −1
4

=
1
3z
1−
gin1
+
z2 z4
1
· · ·


z 
12
1 −
z2 z4
+
4 16
− · · ·

.

(ii) f (z) =
e2z
(z − 1)3
about z = 1.
ee
Example 4.49. Find the Laurent’s series of (i) f (z) = z2 e z about z = 0.
1

rin
g.n
1
Solution. f (z) = z2 e z
z = 0 is a singular point.
f (z) is analytic in |z| > 0.
 1 1 1
f (z) = z2 1 + + 2 + 3 + · · ·
z 2z 6z

et
e2z
(iii) f (z) = .
(z − 1)3
z = 1 is a singular point.
f (z) is analytic in |z − 1| > 0.
∴ f (z) has a Laurent’s series about z = 1.

Downloaded From : www.EasyEngineering.net


Downloaded From : www.EasyEngineering.net

Complex Integration 447

Put t = z − 1.

e2(t+1) e2t e2 e2  4t2 8t3 


f (z) = 3
= 3
= 3
1 + 2t + + + ···
t t t 2 6
e2  4t 3  1 2 2 4 2 
= 3 1 + 2t + 2t2 + + · · · = e2 3 + 2 + + + t + · · ·
t 3 t t t 3 3
 1 2 2 4 2 
= e2 + + + + (z − 1) + · · · .
(z − 1)3 (z − 1)2 (z − 1) 3 3

1
Example 4.50. Find the Taylor’s series expansion of f (z) = about z = −i.

ww (z + 1)2
Solution. To expand f (z) about z = −i, we have to express f (z) in powers of z + i.
Let z + i = t.

w.E Now, f (z) =


1
2
=
1
t 2

asy
(t − i + 1)
 
(1 − i)2 1 + 1−i
1  t −2
= 1 +

En
1+ 2 1−i
" i − 2i
3t2 4t3
#
i 2t
= 1− + − ···

=
2
i
"
1−
2
gin
1 − i (1 − i)2 (1 − i)3

(z + i) +
3 2
(z + i) −
4 3
(z + i) · · ·
#

2 1−i

ee (1 − i)2

Example 4.51. Find the Taylor’s series expansion of f (z) =


(1 − i)3

rin
2z3 + 1
z2 + z
about the point
z = i.
Solution. By actual division we get g.n
2z3 + 1
2
z +z
= 2z − 2 +
2z + 1
z(z + 1)
et
1
= 2z − 2 + (2z + 1)
z(z + 1)
" #
1 1
= (2z − 2) + (2z + 1) − . (1)
z z+1

1 1
Let us expand and about z = i.
z z+1

Downloaded From : www.EasyEngineering.net


Downloaded From : www.EasyEngineering.net

448 Engineering Mathematics - II

1 1
i.e., We have to expand and in powers of z − i.
z z+1
Let t = z − i

i.e., z = t + i
1 1 1  t −1
Now = =   = −i 1 +
z t+i i 1 + ti i
t t2 t3 t4
" #
= −i 1 − + − + ···
i i 2 i3 i4
z − i (z − i)2 (z − i)3 (z − i)4
" #

ww = −i 1 −
i
+
i2

i3
(z − i)2 (z − i)3 (z − i)4
+
i4
···

w.E Now,
1
= −i + (z − i) −

=
1
=
z + 1 t + i + 1 (1 + i) 1 + t
i
1

+
i2


i3
···

= asy
1 
1+i
1+
t −1
1+i
1+i

=
1
1+i En"
1−
t
+
t2
1 + i (1 + i)2 (1 + i)3

t3
···
#

=
1
1+i
"
gin
1−
z − i (z − i)2 (z − i)3
+
1 + i (1 + i)2 (1 + i)3
− ···
#

Substituting the expansion of and

2z3 + 1
1
z
"
ee z+1
1
in (1) we get

(z − i)2 (z − i)3 rin 1 z−i


2
z +z
= (2z − 2) + (2z + 1) −i + (z − i)

(z − i)

2
i
(z − i)
+
3
i 2
#
··· +

g.n
1+i
{1 −
1+i

Exercise 4 C
+
(1 + i) 2

(1 + i)3
···} .

et
1
1. Obtain the Laurent’s series for f (z) = for (i) 1 < |z| < 3 (ii) |z| < 1.
(z + 1)(z + 3)
[May 2005]
z2
2. Expand f (z) = in a Laurent’s series expansion if (i) |z| < 2
(z + 2)(z − 3)
(ii) 2 < |z| < 3. [Nov 2005]

Downloaded From : www.EasyEngineering.net


Downloaded From : www.EasyEngineering.net

Complex Integration 449

z−1
3. Expand f (z) = as a Laurent’s series valid in 2 < |z| < 3.
(z + 2)(z + 3)
[May 2006]
z
4. Expand f (z) = in a series of positive and negative powers of z − 1
(z − 1)(z − 3)
if 0 < |z − 1| < 2. [Nov 2010]
(z − 2)(z + 2)
5. Obtain a Laurent’s series expansion for f (z) = valid in
(z + 1)(z + 4)
(i) 1 < |z| < 4 (ii) |z| > 4.

ww
1
6. Expand f (z) = for 0 < |z| < 1 and 0 < |z − 1| < 1. [May 1999]
z(z − 1)
z−1

w.E 7. Expand f (z) =

8. Expand f (z) =
z2
valid in |z − 1| > 1.

z+3
if 1 < |z| < 2.
[Dec 1997]

[May 2003]
z(z2
asy
− z − 2)
1
9. Expand f (z) =

En
(z − 1)(z − 2)
(iv) 0 < |z − 1| < 1.
in the region (i) |z| < 1 (ii) 1 < |z| < 2 (iii) |z| > 2

10. Expand f (z) =


12
gin
z(2 − z)(z + 1)
if (i) 0 < |z| < 1 (ii) 1 < |z| < 2.

4.5.2 Singularities ee rin


g.n
Zero of a function. Let f (z) be analytic at z = z0 . If f (z0 ) = 0, then z0 is called a
zero of f (z). If there is a positive integer m such that f (m) (z0 ) , 0 and

m at z0 .
∴ f (z) = (z − z0 )m g(z), g(z0 ) , 0.
et
f ′ (z0 ) = 0 = f ′′ (z0 ) = f ′′′ (z0 ) = · · · = f (m−1) (z0 ), then f (z) is said to have a zero for order

p(z)
Note. f (z) = where p(z) and q(z) are analytic at z = a and p(a) , 0 and if a
q(z)
is a zero of order m for q(z) then a is a pole of order m for f (z).

Regular point. A point at which a complex function f (z) is analytic is called a

Downloaded From : www.EasyEngineering.net


Downloaded From : www.EasyEngineering.net

450 Engineering Mathematics - II

regular point or ordinary point of f (z).


Singular point. A point z = a is a singular point of f (z) if f (z) is not analytic
(or not even defined) but is analytic at some point in every deleted neighbourhood
of a. [Jun 2012, May 2011]
Example.
1
1. f (z) = has z = 2 as a singular point.
z−2
1
2. f (z) = 2 has z = 0 & z = 1 as singular points.
z (z − 1)
Isolated Singularity. A singular point z = a is called an isolated singularity of

ww f (z) if there exists a neighbourhood of a in which there is no other singularity.

w.E
A singular point which is not isolated is called a nonisolated singularity.
Example.
1
f (z) = . z = 0 and z = −2 are isolated singular points.
z(z + 2)
asy
Pole. An isolated singular point a of f (z) is said to be a pole of order m if the

En
principal part of the Laurent’s Series of f (z) about a contains m terms where m is
finite.

gin
i.e., there exists a positive integer m such that bm , 0 and bm+1 = 0 = bm+2 = . . .

ee
∴ The Laurents’s series becomes

X b1 b2 bm
f (z) = an (z − a)n + + + ··· + , r2 < |z − a| < r1 where bm , 0.
n=0
z − a (z − a) 2

If m = 1, then a is called a simple pole.


(z − a)m
rin
Example. Consider f (z) = 2
1
z (z − 1)
pole of order 2 and z = 1 is a simple pole. g.n
. The poles of f (z) are z = 0 and z = 1. z = 0 is a

Essential Singularity. An isolated singular point a is said to be an essential


singularity of f (z) if the principal part of the Laurent’s series of f (z) about a
et
contains infinitely many terms.
1
Example.Consider f (z) = e z
1 1 1 1 1
=1+ + 2
+ + · · · if 0 < |z| < ∞
z 2! z 3! z3
1 1
= 1 + z−1 + z−2 + z−3 + . . .
2! 3!

Downloaded From : www.EasyEngineering.net


Downloaded From : www.EasyEngineering.net

Complex Integration 451

Since f (z) has infinitely many terms in the principal part, z = 0 is an essential
Singularity.
Removable Singularity. An isolated singular point z = a of f (z) is called a
removable singularity of f (z) if in some neighbourhood of a, the Laurent’s series
expansion of f (z) has no principal part.
sin z
Example.Consider f (z) = , z,0
z
1 z3 z5  z2 z4
= z − + − · · · = 1 − + + · · · 0 < |z| < ∞.
z 3! 5! 3! 5!

ww It has no principal part.


∴ z = 0 is a removable singularity if
sin z
z
is not defined at z = 0.

w.E sin z

if z , 0




 z
Suppose f (z) =   Then the series converges to 1 at z = 0 and hence
 1 if z = 0.


f (z) is analytic at z = 0.
Results
asy
En
1. A singularity z = a is an essential singularity if there is no integer n such
that lim(z − a)n f (z) = A , 0.
z→a

gin
2. A singularity z = a is a pole of order m if lim(z − a)m f (z) = A , 0.
z→a

ee
3. A singularity z = a is a removable singularity of f (z) if lim f (z) = A, A finite.

Working rule for detecting the type of singularity


z→a

rin
1. If lim f (z) exists and finite then z = a is a removable singularity.
z→a
2. If lim f (z) = ∞ then z = a is a pole of f (z). g.n
et
z→a
3. If lim f (z) does not exist, then z = a is an essential singularity.
z→a ✎ ☞
Worked Examples
✍ ✌
1
z2
Example 4.52. Classify the singularity of ee . [Dec 2011]
1
e z2
Solution. Let f (z) = e .
z = 0 is a singular point.
1
z2
lim f (z) = lim ee which does not exist.
z→0 z→0
∴ z = 0 is an essential singularity.

Downloaded From : www.EasyEngineering.net


Downloaded From : www.EasyEngineering.net

452 Engineering Mathematics - II

ez
Example 4.53. Discuss the nature of the singularity of . [May 2001]
z2 + 4
ez
Solution. Let f (z) = .
z2 + 4
The singularities are given by z2 + 4 = 0.
i.e., z2 = −4
z = ±2i.
∴ z − 2i and z = −2i are the singular points.
ez
lim f (z) = lim = ∞.
z→2i z→2i (z + 2i)(z − 2i)

ww Also lim f (z) = lim


z→−2i
ez
z→−2i (z + 2i)(z − 2i)
Hence, z = 2i and z = −2i are simple poles.
= ∞.

w.E
Example 4.54. Classify the singularity of
sin z − z
sin z − z
z3
. [May 2000]

asy
Solution. Let f (z) = .
z3
The singularities are given by z3 = ! 0 =⇒ z = 0.
1 z3 z5

1 z3
z
z5 En
Now, f (z) = 3 z − + − · · · − z
3! !5!
= 3 − +
z
1
6 120
z2 gin
− ···

=− +
6 120
lim f (z) = lim − +
z→0
−1
z→0
− ···
1
6 120
z2
ee
− ···
!

rin
=
6
= finite.
Hence, z = 0 is a removable singularity. g.n
Example 4.55. Find the nature and location of the singularities of the function
f (z) =
z − sin z
z2
.
z − sin z
et
Solution. Let f (z) = .
z2
z = 0 is a singularity.
z3 z5
" ( )#
1
f (z) = 2 z − z − + · · ·
z 3! 5!
3 z5 z7 z3 z5
" #
1 z z
= 2 z − z + − + ··· = − + ···
z 3! 5! 7! 3! 5! 7!

Downloaded From : www.EasyEngineering.net


Downloaded From : www.EasyEngineering.net

Complex Integration 453

Since there is no negative powers of z in the expansion of f (z), z = 0 is a


removable singularity.

Example 4.56. Find! the nature and location of the singularity of the function
1
f (z) = (z + 1) sin .
z−2 !
1
Solution. f (z) = (z + 1) sin .
z−2
Let z − 2 = t.
!
1
∴ f (z) = (t + 2 + 1) sin

ww "
= (t + 3) −
1 1 1
t

+
t 3! t3 5! t5
1 1
···
#

w.E 1 1 3
= 1 − 2 + 4 ··· + − 3 + 5 ···
3!t 5!t t 2t
1 3
5!t

asy
3 1 1 1 3 1
= 1 + − 2 − 3 + t4 + ···
t 3!t 2t 5! 5! t5
3 1 1 1 1 1
=1+

En −
z − 2 3! (z − 2) 2

2(z − 2) 3
+
5! (z − 2)4
···

gin
Since there are infinite number of terms in the negative powers of z − 2,
z = 2 is an essential singularity.

Solution. Let f (z) =


1
cos z − sin z
.
ee
Example 4.57. Find the nature and location of the singularity of

rin
1
cos z − sin z
.

The singularities are given by


g.n
sin z − cos z = 0

sin z = cos z ⇒
sin z
cos z
=1
π
et
i.e., tan z = 1 ⇒z= .
4
π
∴z= is a singularity.
4
1 1
Now limπ f (z) = limπ = = ∞.
z→ 4 z→ 4 (cos z − sin z) cos π4 − sin π4
π
∴z= is a simple pole.
4

Downloaded From : www.EasyEngineering.net


Downloaded From : www.EasyEngineering.net

454 Engineering Mathematics - II

1
ez
Example 4.58. Discuss the singularities of . [May 2001]
(z − a)2
1
ez
Solution. Let f (z) = .
(z − a)2
The singular points are given by z = 0 and (z − a)2 = 0.
i.e., z = a, a.
1
ez
lim f (z) = lim = does not exist.
z→0 z→0 (z − a)2
∴ z = 0 is an essential singularity.
1
ez

ww Now lim f (z) = lim


z→a z→a (z − a)2
∴ z = a is a pole of order 2.
= ∞.

w.E
Example 4.59. Find the singularities of the function

Solution. Let f (z) =


1 − e2z
.
1 − e2z
z4
. [May 1999]

1
" 
z4
asy
= 4 1 − 1 + 2z +
4z2 8z3
+ + ···
#

En
z " 2! 3!#
1 4z3
= − 4 2z + 2z2 + + ···

gin
z 3
2
2 + 2z + 4z3 + · · ·
=−
z3

lim f (z) = ∞.
z→0
∴ z = 0 is a pole of order 3.
ee
The singular points are given by z3 = 0 =⇒ z = 0, 0, 0.

rin
Example 4.60. Find the nature of the singularities of the function g.n
cot πz
.

Solution. Let f (z) =


cot πz
(z − a)3
cos πz
et
(z − a)3
[May 2005]

= .
sin πz(z − a)3
The singularities of f (z) are given by sin πz = 0 and (z − a)3 = 0.
=⇒ πz = nπ, n ∈ Z and z = a, a, a.
=⇒ z = n, n ∈ Z and z = a.
Now, lim f (z) = ∞
z→n

Downloaded From : www.EasyEngineering.net


Downloaded From : www.EasyEngineering.net

Complex Integration 455

∴ z = 0, ±1, ±2, . . . are simple poles.


Also lim f (z) = ∞.
z→a
z = a is a pole of order 3.
1
Example 4.61. What type of singularity has the function f (z) = .[May 2008]
1 − ez
Solution. The poles are given by

1 − ez = 0

ez = 1 = e2nπi , n ∈ z.

ww ∴ z = 2nπi, n = 0, ±1, ±2, ±3, ...

w.E When n = 1, z = 2πi is a simple pole.


Similarly, we can prove that all the poles are simple.

asy
Example 4.62. Discuss the nature of the singularity of the function
1
ez
z2
.

En
1
ez
Solution. Let f (z) = 2
"z

gin
#
1 1 1 1 1
= 2 1+ + + + ···
z z 2!z2 3! z3
1 1
6
ee
= z−2 + z−3 + z−4 + z−5 · · ·
2

rin
Since there are infinite number of terms in the negative powers of z, z = 0
is an essential singularity, of f (z).

Example 4.63. Find the nature of the singularity of the function


g.ne2z
(z − 1)4
.
Solution. The singularity is at z = 1.
Let z − 1 = t
∴ z = 1 + t.
et
e2z e2(1+t) e2+2t e2 2t
= = 4 = 4 ·e
(z − 1)4 t4 t t
2 2 3 16t4 32t5
" #
e 4t 8t
= 4 1 + 2t + + + + ···
t 2! 3! 4! 5!
" #
2 −4 −3 −2 4 −1 2 4
= e t + 2t + 2t + t + + t + · · ·
3 3 15

Downloaded From : www.EasyEngineering.net


Downloaded From : www.EasyEngineering.net

456 Engineering Mathematics - II

" #
2 −4 −3 −2 4 −1 2 4
= e (z − 1) + 2(z − 1) + 2(z − 1) + (z − 1) + + (z − 1) · · ·
3 3 5

Since the expansion contains only 4 terms with negative powers of z − 1,


z = 1 is a pole of order 4.

Exercise 4 D

ww 1. Discuss the nature of singularities of the following functions.

w.E (i).
z−2
z 2
sin
1
z−1
.
(viii). sin
1
(ix). e z .
z−2
.

asy
1
(ii). sin . 1
z−1 (x). .
(z − 5)3 (z
− 4)2
1 − ez
(iii).
1 + ez
.
En (xi). 2
ez
z + π2
.

(iv). z
1
z(e − 1)
.
gin (xii).
z2
(1 + z2 )3
.

ee
z cos 2z
(v). z . (xiii). 2 .
e −1 (z + 1)2 (z2 + 16)
(vi).
sin z
z5
. (xiv).
sin z
z cos z
.
rin
(vii).
tan z
z
. (xv). 2
1
(z + a2 )2
.
g.n
4.6 Residue
et
Let z = a be an isolated singular point of f (z). The coefficient b1 of (z − a)−1 in
the Laurent’s series expansion of f (z) about a is called the residue of f (z) at z = a.
The residue of f (z) at z = a is denoted by b1 = [Res fZ(z)]z=a or R(a).
1
The residue at z = a is given by b1 = R(a) = f (z)dz.
2πi
C

Downloaded From : www.EasyEngineering.net


Downloaded From : www.EasyEngineering.net

Complex Integration 457

Methods of finding Residue


1. If z = a is a simple pole, then R(a) = lim(z − a) f (z).
z→a
1  dm−1 
2. If z = a is a pole of order m then R(a) = lim m−1
(z − a)m f (z) .
(m − 1)! z→a dz
g(z)
3. Let f (z) = , where g(z) and h(z) are analytic functions at z = a.
h(z)

If h(a) = 0, h (a) , 0 and g(a) , 0 and finite, then z = a is a simple pole of f (z) and
g(z)
R(a) = lim ′ .
z→a h (z)
Cauchy’s residue theorem

ww If f (z) is analytic inside and on a simple closed curve C, except at a finite

w.E
number of singular points z1 , z2 , . . . , zn lying inside C, then
Z
f (z)dz = 2πi(R(z1 ) + R(z2 ) + · · · + R(zn ))
C

asy
where the integral over C is taken in the anticlockwise sense.

En
Proof. Let C be the closed curve and z1 , z2 , . . . , zn be the isolated singular points of
f (z) which lie inside C.

gin
Let C1 , C2 , . . . , Cn be non overlapping
ee
neighbourhoods of z1 , z2 , . . . , zn respectively. >
z1

C1
rin
zn

g.n
Hence, f (z) is analytic in the >
Cn
z2
multiply connected region bounded by

et
>
C1 , C2 , . . . , Cn and C. C2

C
>

By Cauchy’s theorem we have


Z Z Z Z Z
f (z)dz = f (z)dz + f (z)dz + f (z)dz + · · · + f (z)dz
C1 C2 C3 Cn
C
Z Z Z Z Z
1 1 1 1 1
f (z)dz = f (z)dz + f (z)dz + f (z)dz + · · · + f (z)dz
2πi 2πi C1 2πi C2 2πi C3 2πi Cn
C

Downloaded From : www.EasyEngineering.net


Downloaded From : www.EasyEngineering.net

458 Engineering Mathematics - II

= R[z1 ] + R[z2 ] + · · · + R[zn ]


Z
∴ f (z)dz = 2πi (R[z1 ] + R[z2 ] + · · · + R[zn ]) .
C

Z
Result. According to Cauchy’s residue theorem, f (z)dz = 2πi [Sum of the
C
residues at the singular points].
✎ ☞
Worked Examples
✍ ✌

ww Example 4.64. Find the residue of f (z) =


4
z3 (z− 2)
at a simple pole.

w.E
Solution. f (z) =
4
z3 (z − 2)
.
[Dec 2012, Jun 2012]

asy
The poles are given by z = 0, z = 2.
z = 2 is a simple pole.

En
Now R(2) = lim(z − 2) f (z)
z→2

gin
= lim(z − 2)
z→2
4
z3 (z− 2) z→2 z
4
z
4 4 1
= lim 3 = 3 = = .
8 2

Example 4.65. Find the singular point of


ee z+2
(z + 1)2
and hence find its residue.

rin [May 2001]

g.n
z+2
Solution. Let f (z) =
(z + 1)2
z = −1 is a pole of order 2.

∴ R(−1) =
1
lim
d
(2 − 1)! z→−1 dz
d
[(z + 1)2 f (z)]
z+2
et
= lim (z + 1)2
z→−1 dz (z + 1)2
d
= lim (z + 2) = lim 1 = 1.
z→−1 dz z→−1

e2z
Example 4.66. Calculate the residue of f (z) = at its pole.
(z + 1)2
[Apr 2004, Dec 2011]

Downloaded From : www.EasyEngineering.net


Downloaded From : www.EasyEngineering.net

Complex Integration 459

e2z
Solution. f (z) = .
(z + 1)2
z = −1 is a pole of order 2.

1 d
∴ R(−1) = lim [(z + 1)2 f (z)]
(2 − 1)! z→−1 dz
e2z
" #
d 2 d 2z
= lim (z + 1) = lim e
z→−1 dz (z + 1)2 z→−1 dz
2
= lim 2e2z = 2e−2 = 2 .
z→−1 e

ww Example 4.67. Test the singularity of


z2
1
+1
and find the corresponding residues.

w.E
Solution. Let f (z) =
1
z2
+1
.
[Apr 1997]

asy
The poles are given by z2 + 1 = 0.
i.e., z2 = −1
z = ±i.
En
z = i and −i are simple poles.

gin
R(i) = lim(z − i) f (z)
z→i

= lim(z − i)
z→i
1
1
ee
(z + i)(z − i)
1 −i rin
= lim
z→i z + i
=
2i
R(−i) = lim (z + i) f (z)
= .
2
g.n
z→−i

= lim (z + i)
z→−i
1
1
(z + i)(z − i)
−1 i
et
= lim = = .
z→i z − i 2i 2

z+2
Example 4.68. Determine the residue at the simple pole of .
(z + 1)2 (z − 2)
[May 2000]
z+2
Solution. Let f (z) = .
(z + 1)2 (z − 2)

Downloaded From : www.EasyEngineering.net


Downloaded From : www.EasyEngineering.net

460 Engineering Mathematics - II

z = 2 is a simple pole of f (z).

R(2) = lim(z − 2) f (z)


z→2
z+2
= lim(z − 2)
z→2 (z + 1)2 (z − 2)
(z + 2) 4
= lim 2
= .
z→2 (z + 1) 9
z
Example 4.69. Find the residue of f (z) = at its pole.
(z − 1)2
z

ww Solution. f (z) =
(z − 1)2
.
z = 1 is a pole of order 2.

w.E ∴ R(1) =
1 d
lim (z − 1)2 f (z)
(2 − 1)! z→1 dz
d
"
z
#
= lim
z→1 dz
d asy
(z − 1) 2
(z − 1)2
= lim z = 1.
z→1 dz

En
Example 4.70. Find the residue of
1 − e2z gin 1 − e2z
z4
at z = 0. [Jun 2013]

Solution. Let f (z) =


z4
.
ee
z = 0 is a pole of order 3. [Expand e2z and simplify]

1 d2 rin
∴ R(0) =

1
lim 2 (z − 0)3 f (z)
(3 − 1)! z→0 dz
d2 3 1 − e2z
" #
g.n
=

=
1
lim
2 z→0 dz2

lim 2
2 z→0 dz
"
z

d2 1 − e2z
z
z4
#
et
4z2 8z3
 
1 d2  1 − 1 − 2z − 2! − 3! − · · · 
= lim  
2 z→0 dz2  z
" #
1 d 8z
= lim 2− − ···
2 z→0 dz 3
" #
1 −8
= lim + powers of z
2 z→0 3

Downloaded From : www.EasyEngineering.net


Downloaded From : www.EasyEngineering.net

Complex Integration 461

!
1 −8 4
= =− .
2 3 3

1 − e−z
Example 4.71. Find the residue of f (z) = at z = 0. [Dec 2013]
z3
1− e−z
Solution. f (z) =
z3
z2 z3 z4 z5
 
1− 1−z+ 2! − 3! + 4! − 5! ···
=
z3
z2 z3 z4 z5
" #
1
= 3 1 − 1 + z − + − + ···

ww z
1
= 3
"
z 2 z 3
z − + − + ···
2! 3! 4! 5!
z4 z5
#

w.E z 2! 3! 4! 5!
1 11 1 z z2
= 2 − + − + ···
z 2! z 3! 4! 5!

asy
z is the pole of order 2.
∴ The residues of f (z) at z = 0 is the coefficient of
1 1
=− .

En z2
z 2

Example 4.72. Find the residues of


gin
singularities using Laurent’s series expansion.
f (z) =
(z + 2)(z − 1)2
at its isolated
[Dec 2013]

Solution.
z2
(z + 2)(z − 1) 2
=
A
+
B
ee
+
C
z + 2 z − 1 (z − 1)2
z2 = A(z − 1)2 + B(z − 1)(z + 2) + c(z + 2). rin
1
When z = 1, 3C = 1 ⇒ C = .
3 g.n
When z = −2, 9A = 4 ⇒ A = .

Equating the coefficients of z2 we get


4
9 et
A+B=1
4 5
B=1−A=1− = .
9 9
z2 4 1 5 1 1 1
∴ 2
= + + .
(z + 2)(z − 1) 9 z + 2 9 z − 1 3 (z − 1)2

Downloaded From : www.EasyEngineering.net


Downloaded From : www.EasyEngineering.net

462 Engineering Mathematics - II

1
The residues at the singularity z = −2 is the coefficient of in the
z
4 1
expansion of .
9z+2
!−1
4 1 4 1 41 2
Now, · = ·   = 1+
9 z+2 9 z 1+ 2 9z z
z
" # " #
41 2 4 8 4 1 2 4
= 1 − + 2 − 3 ··· = − + ···
9z z z z 9 z z2 z3
1 4
∴ Residue at z = −2 = Coefficient of = .
z 9

ww The residue at the singularity z = 1 is the coefficient of


1
z
in the

w.E expansion of
5 1
+
1 1
9 z − 1 3 (z − 1)2
.

asy
5 1 1 1 5 1
Now, + 2
=  + 2
9 z − 1 3 (z − 1) 1
9z 1 − z

3z2 1 − 1z

En
!−1 !−2
5 1 1 1
= 1− + 2 1−
9z z 3z z

gin
" # " #
5 1 1 1 1 2 3
= 1 + + 2 + 3 + ··· + 2 1 + + 2 + ··· .
9z z z z 3z z z

Example 4.73. Find the residues of


ee 1 5
Residues at z = 1 = Coefficient of = .
z 9
z2
f (z) = rin
at its isolated
(z − 1)2 (z + 2)2
g.n
singularities using Laurent’s series expansions. Also state the valid region.

Solution. Let
z2
=
A
+
B
+
C
+
D
(z − 1)2 (z + 2)2 z − 1 (z − 1)2 z + 2 (z + 2)2
. et
[Dec 2012, Dec 2010]

∴ z2 = A(z − 1)(z + 2)2 + B(z + 2)2 + C(z + 2)(z − 1)2 + D(z − 1)2 .

When z = 1, 9B = 1
1
⇒B= .
9
4
When z = −2, 9D = 4 ⇒D= .
9

Downloaded From : www.EasyEngineering.net


Downloaded From : www.EasyEngineering.net

Complex Integration 463

Equating the coefficients of z3 we get


A + C = 0 ⇒ C = −A.

When z = 0, −4A + 4B + 2C + D = 0
4 4
− 4A + − 2A + = 0
9 9
8
6A =
9
8 4
A= = .
6 × 9 27

ww ∴C=− .
4
27

w.E
4 1 1 1 4 1 4 1
f (z) = · + · 2
− · + · .
27 z − 1 9 (z − 1) 27 z + 2 9 (z + 2)2
4 1 1 1 4 1 4 1
= · + · − · + · .

asy
27 z − 1 9 (z − 1) 2 27 (z − 1) + 3 9 ((z − 1) + 3)2
!−1 !−2
4 1 1 1 4 z−1 4 z−1
= · + · − 1+ + 1+ .
27
4
·
z − 1 9 (z − 1)2 81
1 1
+ ·
1
− En
4 X

(−1)
3
n z−1
81
!n
4 X

3

(−1) n z−1
!n
(n + 1). (1)

gin
= +
27 z − 1 9 (z − 1)2 81 n=0 3 81 n=0 3
z−1
Expansion is valid if |
3
Residue of f (z) at z = 1 = co-efficient of
z−1
ee
| < 1. i.e., |z − 1| < 3.
1
z−1
4
in (1) = .
27
| < 1 ⇒ 0 < |z − 1| < 3. rin
g.n
The validity of the region is |
3
The find the residue of f (z) at z = −2, we have to expand f (z) in series of powers of
z + 2 valid 0 < |z + 2| < r.

f (z) =
4
·
1
+ ·
1 1

4
·
1 4
+ ·
27 (z + 2) − 3 9 ((z + 2) − 3)2 27 z + 2 9 (z + 2)2
!−1 !−2
1
. et
4 z+2 1 z+2 4 1 4 1
=− 1− + 1− − · + · . (2)
81 3 81 3 27 z + 2 9 (z + 2)2
z+2
Expansion is valid if | | < 1. i.e., |z + 2| < 3.
3
1 4
Residue of f (z) at z = 2 = co-efficient of in (2) = − .
z+2 27
The validity of the region 0 < |z + 2| < 3.

Downloaded From : www.EasyEngineering.net


Downloaded From : www.EasyEngineering.net

464 Engineering Mathematics - II

sin z
Example 4.74. Find the sum of the residues of f (z) = at its poles inside the
z cos z
circle |z| = 2.
Solution. The poles are given by z cos z = 0
i.e., z = 0, cos z = 0
π 3π
i.e., z = 0, z = ± , ± , · · ·
2 2
Given c is |z| = 2.
When z = 0, |z| = |0| = 0 < 2.
∴ z = 0 lies inside C,

ww π
π
2
π
π
When z = ± , |z| = | | < 2.
2

w.E ∴ z = and − lie inside C.


2

2

When z = ± , |z| = | | > 2.
2 2

asy
∴ All other singular points lie out side C.
z = 0 is a simple pole.

En
sin z sin z
∴ R(0) = lim z f (z) = lim z · = lim = 0.
z→0 z→0 z cos z z→0 cos z
π
z= is a simple pole.
2
π 
gin
π

ee
∴R = limπ z − f (z)
2 z→ 2 2

rin
 π  sin z 0
= limπ z − · [ form]
z→ 2 2 z cos z 0

g.n
 
π
z − 2 · cos z + sin z
= limπ [by L’hospital rule]
z→ 2 −z sin z + cos z

et
 
π π π π
2 − 2 cos 2 + sin 2 1 2
= π π π = π =− .
− 2 · sin 2 + cos 2 −2 π

π
z=− is also a simple pole.
2
 π  π
R − = limπ z + f (z)
2 z→(− 2 ) 2
 
z + π2 sin z 0
= limπ [ form]
z→(− 2 ) z cos z 0

Downloaded From : www.EasyEngineering.net


Downloaded From : www.EasyEngineering.net

Complex Integration 465

 
z + π2 cos z + sin z
= limπ
z→(− 2 ) −z sin z + cos z
     
π π
− 2 + 2 cos − π2 + sin − π2
=      
− − π2 sin − π2 + cos − π2
 
0 − sin π2 sin π2 1 2
=   =− π = π = .
π
− sin π −2 2 π
2 2

π  π
∴ Sum of the residues = R(0) + R +R −

ww
2 2
2 2
=0+ − = 0.
π π

w.E
Example 4.75. If f (z) = −
f (z) at z = 1.
1
z−1
h i
− 2 1 + (z − 1) + (z − 1)2 + · · · find the residue of
[Dec 2012, Dec 2010, Jun 2010]
Solution.
1 = Coefficient of
In
1  asy
the
(z − 1) −1

Laurent’s
= −1.
series expansion, residue at

z−1

En
Example 4.76. If C is the circle |z| = 2, evaluate
Z
tan zdz. [May 2009]

gin
sin z
C

ee
Solution. Let f (z) = tan z = .
cos z
The poles are given by cos z = 0
=⇒ z = (2n + 1) π2 , n ∈ Z.
π rin
g.n
−π
Since C is |z| = 2, z = and lie inside |z| = 2 which are simple poles.
2 2
π g(z)
Now R
2
= limπ ′
z→ 2 h (z)

= limπ
sin z
z→ 2 − sin z
= −1
et
 −π  sin z
R = limπ = −1.
2 z→ 2 − sin z

By Cauchy’s residue theorem,


Z
f (z)dz = 2πi[ sum of the residues ] = 2πi[−1 − 1] = −4πi.
C

Downloaded From : www.EasyEngineering.net


Downloaded From : www.EasyEngineering.net

466 Engineering Mathematics - II

Z
1
Example 4.77. Evaluate dz where C is the circle |z − i| = 2. [May 2008]
(z2 + 4)2
C
1 1
Solution. Let f (z) = 2 2
= .
(z + 4) (z + 2i) (z − 2i)2
2
z = 2i and z = −2i are poles of order 2.
C is the circle |z − 2i| = 2.
When z = 2i, |z − i| = |2i − i| = |i| = 1 < 2.
∴ z = 2i lies inside C.
When z = −2i, |z − i| = | − 2i − i| = | − 3i| = 3 > 2.

ww ∴ z = −2i lies outside C.


∴ By Cauchy’s residue theorem
R
f (z)dz = 2πiR(2i)

w.E R(2i) =
1
lim
d n
(z − 2i)2
C

1 o
1! z→2i dz
d
asy
= lim (z + 2i)−2
z→2i dz
(z + 2i)2 (z − 2i)2

En
= lim (−2)(z + 2i)−3
z→2i

= (−2)
1
=
−2
=
gin
1
(4i)3 64(−i) 32i
π
ee
Z
1
∴ f (z)dz = 2πi = .
32i 16
C

rin
g.n
Z
2z + 3
Example 4.78. Evaluate using Cauchy’s residue theorem dz where
z(z − 1)(z − 2)
C
C is |z| = 3.
Solution. Let f (z) =
2z + 3
z(z − 1)(z − 2)
.
The poles are given by z = 0, z = 1 and z = 2.
et[May 2004]

C is |z| = 3.
When z = 0, |z| = |0| = 0 < 3.
∴ z = 0 lies inside C.
When z = 1, |z| = 1 < 3.
∴ z = 1 lies inside C.

Downloaded From : www.EasyEngineering.net


Downloaded From : www.EasyEngineering.net

Complex Integration 467

When z = 2, |z| = |2| = 2 < 3.


∴ z = 2 lies inside C. Z
By Cauchy’s residue theorem, f (z)dz = 2πi[R(0) + R(1) + R(2)]
C
z = 0 is a simple pole.

∴ R(0) = lim(z − 0) f (z)


z→0
2z + 3
= lim z
z→0 z(z − 1)(z − 2)

ww = lim z
2z + 3
z→0 (z − 1)(z − 2)
3

w.E
=
(−1)(−2)
3
R(0) = .
2

asy
z = 1 is a simple pole.

En
∴ R(1) = lim(z − 1) f (z)
z→1

= lim(z − 1)
z→1
5 gin
2z + 3
z(z − 1)(z − 2)
= lim
z→1
2z + 3
z(z − 2)
=
1(−1)
= −5.

z = 2 is a simple pole.
ee rin
∴ R(2) = lim(z − 2) f (z)
z→2
2z + 3 g.n
= lim(z − 2)
z→2

= lim
2z + 3
z(z − 1)(z − 2)

z→2 z(z − 1)
et
7 7
= = .
Z 2.1 2 !
3 7
f (z)dz = 2πi − 5 +
2 2
C
!
3 − 10 + 7
= 2πi = 0.
2

Downloaded From : www.EasyEngineering.net


Downloaded From : www.EasyEngineering.net

468 Engineering Mathematics - II

z2
Example 4.79. Determine the poles of f (z) = and the residue at each
(z − 1)2 (z + 2)
z2
Z
pole. Hence evaluate dz where C is |z| = 3. [May 2000]
(z − 1)2 (z + 2)
C
z2
Solution. Let f (z) = .
(z − 1)2 (z + 2)
The poles are given by z = 1, z = −2.
z = −2 is a simple pole, and z = 1 is a pole of order 2.
C is |z| = 3.

ww When z = −2, |z| = | − 2| = 2 < 3.


∴ z = −2 lies inside C. When z = 1, |z| = |1| = 1 < 3.

w.E ∴ z = 1 also lies inside C.


∴ By Cauchy’s residue theorem
Z

f (z)dz = 2πi R(1) + R(−2) .

asy
Since z = 1 is a pole of order 2,
C

R(1) =
1 d
En
lim (z − 1)2 f (z) = lim
d (z − 1)2 z2
1! z→1 dz

= lim
d z2
gin
!
= lim
z→1 dz (z − 1)2 (z + 2)
(z + 2)2z − z2 6 − 1 5
(z + 2)2
= = .

ee
z→1 dz z + 2 z→1 9 9
z = −2 is a simple pole.

R(−2) = lim (z + 2) f (z) = lim (z + 2)


z2 4
= . rin
g.n
z→−2 z→−2 2
(z − 1) (z + 2) 9
Z 5 4
Now, f (z)dz = 2πi + = 2πi.

et
9 9
C

z3
Example 4.80. Find the residue of f (z) = at its poles and hence
H (z − 1)4 (z − 2)(z − 3)
evaluate f (z)dz where C is the circle |z| = 2 · 5.
C
Solution. The poles are given by (z − 1)4 (z − 2)(z − 3) = 0
i.e., z = 1, z = 2, z = 3.
z = 1 and z = 2 lie inside C whereas, z = 3 lies outside C.

Downloaded From : www.EasyEngineering.net


Downloaded From : www.EasyEngineering.net

Complex Integration 469

∴ z = 1 is a pole of order 4.
1 d3
R(1) = lim 3 (z − 1)4 · f (z)
3! z→1 dz
d3 z3
( )
1 4
= lim 3 (z − 1)
6 z→1 dz (z − 1)4 (z − 2)(z − 3)
d3 z3
( )
1
= lim 3
6 z→1 dz (z − 2)(z − 3)
z3 C D
Let = Az + B + +
(z − 2)(z − 3) z−2 z−3

ww z3 = (Az + B)(z − 2)(z − 3) + C(z − 3) + D(z − 2).

When z = 2, −C = 8 ⇒ C = −8.

w.E When z = 3, D = 27.


Equating the coefficients of z3 we get A = 1.

asy
When z = 0, 6B − 3C − 2D = 0

En
6B = 3C + 2D = −24 + 54

6B = 30 ⇒ B = 5.

z3
gin
ee
8 27
∴ =z+5− +
(z − 2)(z − 3) z−2 z−3
3

rin
( )
1 d 8 27
∴ R(1) = lim 3 z + 5 − +
6 z→1 dz z−2 z−3
1 d3 n
= lim 3 z + 5 − 8(z − 2)−1 + 27(z − 3)−1
6 z→1 dz
o
g.n
1 d2 n
= lim 2 1 − 8(−1)(z − 2)−2 + 27(−1)(z − 3)−2
6 z→1 dz
1
= lim
d n
8(−2)(z − 2)−3 − 27(−2)(z − 3)−3
o
o
et
6 z→1 dz
1 n o
= lim −16(−3)(z − 2)−4 + 54(−3)(z − 3)−4
6 z→1
" # " #
1 48 162 1 162
= − = 48 −
6 (1 − 2)4 (1 − 3)4 6 16
27 128 − 27 101
=8− = = .
16 16 16

Downloaded From : www.EasyEngineering.net


Downloaded From : www.EasyEngineering.net

470 Engineering Mathematics - II

z = 2 is a simple pole.

∴ R(2) = lim(z − 2) f (z)


z→2
z3
= lim(z − 2)
z→2 (z − 1)4 (z − 2)(z − 3)
z3 8
= lim = = −8.
z→2 (z − 1)4 (z − 3) 1 × (−1)
I
∴ f (z)dz = 2πi × [sum of the residues]
C

ww = 2πi
101
16
" #
− 8 = 2πi
"
101 − 128
16
# "
= 2πi × −
27
16
#
=−
27πi
8
.

w.E
Example 4.81. Evaluate
Z
z−1
(z + 1)2 (z − 2)
C
dz where C is |z − i| = 2.

Solution. Let f (z) =


z−1
asy
(z + 1)2 (z − 2)
.
[June 2014, June 2012]

En
The poles are given by z = −1 and z = 2.
z = −1 is a pole of order 2 and z = 2 is a simple pole.
C is |z − i| = 2.
gin √ √

⇒ z = −1 lies inside C.
When z = 2, |z − i| = |2 − i| =
ee
When z = −1, |z − i| = | − 1 − i| =


1 + 1 = 2 < 2.


4 + 1 = 5 > 2. rin
∴ z = 2 lies outside C.
∴ By Cauchy’s residue theorem
Z
g.n
et
f (z)dz = 2πiR(−1).
C
Since z = −1 is a pole of order 2,

1 d d  z−1 
(z + 1)2 f (z) = lim (z + 1)2

R(−1) = lim
(2 − 1)! z→−1 dz z→−1 dz (z + 1)2 (z − 2)
d z−1
  z − 2 − (z − 1) −3 + 2 −1
= lim = lim = = .
z→−1 dz z − 2 z→−1 (z − 2)2 9 9
Z
z−1  −1  2πi
Hence, 2
dz = 2πiR(−1) = 2πi =− .
(z + 1) (z − 2) 9 9
C

Downloaded From : www.EasyEngineering.net


Downloaded From : www.EasyEngineering.net

Complex Integration 471

I
z−3
Example 4.82. Evaluate dz where C is the circle (i) |z| = 1,
z2 + 2z + 5
C
(ii) |z + 1 − i| = 2 (iii) |z + 1 + i| = 2.
z−3
Solution. Let f (z) = 2 .
z + 2z + 5
The singularities are given by

z2 + 2z + 5 = 0

(z + 1)2 + 5 − 1 = 0

(z + 1)2 + 4 = 0

ww (z + 1)2 = −4

w.E z + 1 = ±2i

z = −1 ± 2i.

asy
The singular points are z = −1 + 2i and z = −1 − 2i.
(i) Consider C as |z| = 1.

En
When z = −1 + 2i, |z| = | − 1 + 2i| =
√ √
1 + 4 = 5 > 1.
∴ −1 + 2i lie outside C.
gin √ √

ee
When z = −1 − 2i, |z| = | − 1 − 2i| = 1 + 4 = 5 > 1.
∴ −1 − 2i lies outside C.
∴ Zf (z) is analytic inside C.
rin

C
f (z)dz = 0.
g.n
(ii) C is |z + 1 − i| = 2.
When z = −1 + 2i, |z| = | − 1 + 2i + 1 − i| = |i| = 1 < 2.
∴ −1 + 2i lies inside C.
et
When z = −1 − 2i, |z| = | − 1 − 2i + 1 − i| = | − 3i| = 3 > 2.
∴ −1 − 2i lie outside C.

R(−1 + 2i) = lim (z + 1 − 2i) f (z)


z→(−1+2i)
z−3
= lim (z + 1 − 2i)
z→(−1+2i) (z + 1 + 2i)(z + 1 − 2i)

Downloaded From : www.EasyEngineering.net


Downloaded From : www.EasyEngineering.net

472 Engineering Mathematics - II

z−3
= lim
z→(−1+2i) (z + 1 + 2i)
−1 + 2i − 3 −4 + 2i
= =
−1 + 2i + 1 + 2i 4i
1
= (−4 + 2i)(−i)
4
1 1 + 2i 1
= (4i + 2) = = + i.
Z 4 2 2 !
1
∴ f (z)dz = 2πiR(−1 + 2i) = 2πi + i = π(i − 2).
2
C

ww (iii) C is |z + 1 + i| = 2
When z = −1 + 2i, |z + 1 + i| = | − 1 + 2i + 1 + i| = |3i| = 3 > 2.

w.E ∴ z = −1 + 2i lies outside C.


When z = −1 − 2i, |z + 1 + i| = | − 1 − 2i + 1 + i| = | − i| = 1 < 2.

asy
∴ z = (−1 − 2i) lies inside C.

R(−1 − 2i) =
En lim (z + 1 + 2i) f (z)
z→−1−2i

=
ginlim (z + 1 + 2i)
z→−1−2i
z−3
z−3
(z + 1 + 2i)(z + 1 − 2i)
−1 − 2i − 3
= lim

=
ee
z→−1−2i z + 1 − 2i
−4 − 2i 4 + 2i
−4i
=
4i
.
=
−1 − 2i + 1 − 2i

rin
g.n
I
∴ f (z)dz = 2πiR(−1 − 2i)
C

et
!
4 + 2i
= 2πi = π(2 + i).
4i

ez dz
Z
Example 4.83. Evaluate where C is the circle |z| = 4 using Cauchy’s
(z2 + π2 )2
C
residue theorem. [May 2006]
ez ez
Solution. Let f (z) = 2 = .
(z + π2 )2 (z + πi)2 (z − πi)2
The poles are z = πi and z = −πi both are of order 2.
C is |z| = 4.

Downloaded From : www.EasyEngineering.net


Downloaded From : www.EasyEngineering.net

Complex Integration 473

When z = πi, |z| = |πi| = π < 4.


When z = −πi, |z| = | − πi| = π < 4.
∴ Both the poles lie inside C. Z
∴ By Cauchy’s residue theorem f (z)dz = 2πi[R(πi) + R(−πi)]
C
z = πi is a pole of order 2.

ez ez
!
1 d d
R(πi) = lim (z − πi)2 = lim
1! z→πi dz (z + πi)2 (z − πi)2 z→πi dz (z + πi)2

ww = lim
z→πi
(z + πi)2 ez − ez 2(z + πi)
(z + πi)4
(2πi) e − eπi 4πi eπi (−4π2 − 4πi)
2 πi

w.E =

=
(2πi)4
−4πeπi (π + i)
=−
=
16π4
(cos π + i sin π)(π + i)

asy
16π 4 4π3
π+i
R(πi) = .
4π3

Changing i to −i En
Z
R(−πi) =
π−i
4π3
gin
π + i π − i

C
f (z)dz = 2πi

2πi
3
2π = .
=
π
4π3
i
+
4π3
ee rin
g.n

ez
I
Example 4.84. Evaluate dz where C is the unit circle |z| = 1.

Solution. Let f (z) =


ez
cos πz

cos πz
.
C
et
The singular points are given by
cos πz = 0
π 3π 5π
i.e., πz = ± , ± , ± , · · ·
2 2 2
1 3 5
i.e., z = ± , ± , ± , · · ·
2 2 2
1 1
Among all the singular points, z = and z = − lie inside |z| = 1.
2 2

Downloaded From : www.EasyEngineering.net


Downloaded From : www.EasyEngineering.net

474 Engineering Mathematics - II

1
z= is a simple pole.
2
! !
1 1
R = lim z − f (z)
2 z→ 12 2
ez
!
1 0
= lim z − [ form]
z→ 12 2 cos πz 0
 
z − 21 ez + ez
= lim [By L’Hospital rule]
z→ 21 −π sin πz
1 √
e2 e

ww 1
!
=
−π sin π2
=−

1
!
π
.

w.E
R − = lim z + f (z)
2 z→− 12 2
ez
!
1 0
= lim z + [ form]

asy 2 cos πz 0
1
z→− 2
 
z + 12 ez + ez
= lim

En
z→− 12

e− 2
1
−π sin πz
1
=
gin  = √ .
π
−π sin − 2 π e

ee
I
∴ f (z)dz = 2πi × Sum of the residues.
C

= 2πi
" √
− e
+ √
1
#
rin
"
1
π

π e
#
= 2i √ − e = 2i e− 2 − e 2
 1 1

g.n
 e 2 − e− 21 
 1
= −4i  
e

2

 = −4i sin h
 1
2
!
. et
sin πz2 + cos πz2
Z
Example 4.85. Evaluate where C is |z| = 4.
(z − 1)2 (z − 2)
C
[Dec 2011, May 2009]
Solution. z = 1 is a pole of order 2 and z = 2 is a simple pole.
sin πz2 + cos πz2
Let f (z) =
(z − 1)2 (z − 2)

Downloaded From : www.EasyEngineering.net


Downloaded From : www.EasyEngineering.net

Complex Integration 475

C is |z| = 4.
When z = 1, |z| = |1| = 1 < 4.
⇒ z = 1 lies inside C.
When z = 2, |z| = |2| = 2 < 4.
⇒ z = 2 also lies inside C. Z
∴ By Cauchy’s residue theorem, f (z)dz = 2πi[R(1) + R(2)].
C
z = 1 is a pole of order 2.

ww
1 d 
(z − 1)2 f (z)

∴ R(1) = lim
1! z→1 dz
d n sin πz2 + cos πz2 o

w.E
= lim
z→1 dz z−2
(z − 2){cos πz2 (π2z) − sin πz2 (π2z)} − (sin πz2 + cos πz2 )
= lim
(z − 2)2

asy
z→1
−1(−2)π + 1
= = 2π + 1.
1

En
z = 2 is a simple pole.

z→2
gin
∴ R(2) = lim(z − 2) f (z)

sin πz2 + cos πz2


Z

C
= lim
z→2
h
(z − 1)2
i
ee= 1.

f (z)dz = 2πi R(1) + R(2) = 2πi[2π + 1 + 1] = 2πi[2π + 2] = 4πi(π + 1).


rin
Example 4.86. Evaluate
Z
1
z2 e z dz where C is the unit circle.
g.n [May 2001]

2 1
Solution. f (z) = z e = z 1 +
z

1 1 −1
= z2 + z +
2


+ z + · · ·.
C
1 1 1
+ +
1 1
z 2! z2 3! z3
+ · · ·

et
2 6
Since the principal part contains infinite number of terms, z = 0 is an
essential singularity.
1
∴ R(0) = Coeff. of z−1 = .
6
1 πi
Z
By Cauchy’s residue theorem f (z)dz = 2πiR(0) = 2πi = .
6 3
C

Downloaded From : www.EasyEngineering.net


Downloaded From : www.EasyEngineering.net

476 Engineering Mathematics - II

3z2 + z − 1
Z
Example 4.87. Use Cauchy’s residue theorem to evaluate dz
(z2 − 1)(z − 3)
C
where C is the circle |z| = 2. [May 2007]
3z2 + z − 1
Solution. Let f (z) = 2 .
(z − 1)(z − 3)
The poles are given by (z2 − 1)(z − 3) = 0
(z − 1)(z + 1)(z − 3) = 0
i.e., z = 1, −1, 3.
All the poles are simple poles.

ww C is |z| = 2.
When z = −1, |z| = | − 1| = 1 < 2.

w.E ∴ z = −1 lies inside C.


When z = 1, |z| = |1| = 1 < 2.

asy
∴ z = 1 lies inside C.
When z = 3, |z| = |3| = 3 > 2.
∴ z = 3 lies outside C.
En Z

z = −1 is a simple pole. gin


By Cauchy’s residue theorem,
C
f (z)dz = 2πi[R(−1) + R(1)]

∴ R(−1) = lim (z + 1) f (z)


z→−1
ee rin
= lim (z + 1)
z→−1
3z2 + z − 1
(z + 1)(z − 1)(z − 3) g.n
= lim
3z2 + z − 1
z→−1 (z − 1)(z − 3)

z = 1 is a simple pole.
1
= .
8 et
∴ R(1) = lim(z − 1) f (z)
z→1
3z2 + z − 1 3z2 + z − 1 3
= lim(z − 1) = lim =− .
z→1 (z + 1)(z − 1)(z − 3) z→1 (z + 1)(z − 3) 4
Z h1 3i h −5 i 5πi
∴ f (z)dz = 2πi − = 2πi =− .
8 4 8 4
C

Downloaded From : www.EasyEngineering.net


Downloaded From : www.EasyEngineering.net

Complex Integration 477

I
Example 4.88. Evaluate tan zdz where C is the circle |z| = 2.
C
sin z
Solution. Let f (z) = tan z = .
cos z
The poles are given by cos z = 0.
π 3π 5π
∴ z = ± ,± ,± ,···
2 2 2
π π
Among the poles z = and z = − lie inside C and all others lie outside C.
2 2
π  π
Now, R = limπ z − f (z)
2 2

ww
z→ 2
!
 π  sin z 0
= limπ z − form
z→ 2 2 cos z 0

w.E = limπ
z→ 2
 
z − π2 cos z + sin z
− sin z
[By L’Hospital rule]

asy
=
sin π2
− sin π2
= −1.

En
 π !
 π  sin z 0
R − = limπ z + form
2 z→− 2 2 cos z 0

gin
   
z + π2 cos z + sin z sin − π2
= limπ =   = −1.
z→− 2 − cos z − cos − π2


I

C
ee
f (z)dz = 2πi × Sum of the residues = 2πi[−1 − 1] = −4πi.

rin
Exercise 4 E g.n
1. Find the residue of f (z) =
1
z2 ez
at is poles. et
[Nov 2010]

z2
2. Find the residue of f (z) = at each of the poles. [May 2010]
(z − 1)2 (z − 2)
1
3. Find the residue of f (z) = about each singularity. [Apr 2009]
(z2 + 1)2
z+1
4. Find the residue of f (z) = at each of its poles. [May 2010]
z2 (z
− 2)

Downloaded From : www.EasyEngineering.net


Downloaded From : www.EasyEngineering.net

478 Engineering Mathematics - II

tan z
5. Find the residue of f (z) = at each of its poles inside |z| = 2. [Dec 2010]
z
Z
2z − 1
6. Evaluate dz where C is the circle |z| = 2, using Cauchy’s
z(z + 1)(z − 3)
C
residue theorem. [May 2011]

z2 − 2z
Z
7. Evaluate dz where C is the circle |z| = 3. [Dec 2011]
(z + 1)2 (z2 + 4)
C
Z
2 sec z
dz where C is the ellipse 4x2 + 9y2 = 9.

ww
8. Evaluate [Dec 2010]
(1 − z2 )
C

w.E z2 + 4
Z
9. Evaluate dz where C is |z − 1| = 3. [Apr 2005]
(z − 1)(z2 − 1)
C

10. Evaluate
Z

asy
4z2 − 4z + 1
(z − 2)(z2 + 4)
dz where C is the circle |z| = 1.

En
C


Z
12z − 7

gin
11. Evaluate dz where C is |z − i| = 3.
(z2 − 1)(2z + 3)
C

tan 2z
ee
Z
12. Evaluate dz where C is the boundary of the square whose sides
(z − 1 − i)2
C
are the lines x = ±2 and y = ±2.
rin
13. Evaluate
Z
2
z−3
z + 2z + 5
dz where C is |z + 1 − i| = 2. g.n
14. Evaluate
C
Z

C
(z2
1
2
+ 1)(z − 4)
3
dz where C is |z| = .
2
et
e2z
Z
15. Evaluate dz where C is |z| = 2.
(z + 1)3
C

e−z
Z
16. Evaluate dz where C is |z| = 2.
(z − 1)4
C

Downloaded From : www.EasyEngineering.net


Downloaded From : www.EasyEngineering.net

Complex Integration 479

Z
1
17. Evaluate dz where C is |z| = 2.
z3 (z + 4)
C
Z
z+1 3
18. Evaluate 2
dz where C is |z| = .
(z − 1) (z + 2) 2
C

3z2 + 5z − 1
Z
19. Evaluate dz where C is |z| = 2.
(z − 1)2
C
Z
z−1
20. Evaluate dz where C is the circle |z − i| = 2.

ww Z
C
(z + 1)2 (z − 2)

w.E
21. Evaluate dz where C is the circle |z − i| = 2 using residue theorem.
(z2 + 4)2
C
Z
z−2
22. Evaluate
C
z2 − z
asy
dz where C is the circle x2 + y2 = 4 using residue theorem.

En
Z
4 − 3z 3
23. Evaluate dz where C is the circle |z| = . [Jun 2010]
z(z − 1)(z − 2) 2

gin
C
Z
z−2
24. Evaluate dz where C is |z| = 2.

25. Evaluate
Z
C
z(z − 1)

ez
(z + 1)2
ee
dz around the circle C : |z − 1| = 3.
rin
C

g.n
4.7 Evaluation of real integrals using residue theorem

Type 1. Real definite integrals of the form


Z

0
2π et
F(sin θ, cos θ)dθ where F(sin θ, cos θ)
is a real rational function of cos θ and sin θ and is finite on the interval of
integration.
Working rule.
1
Put z = eiθ ⇒ = e−iθ
z
1
z = cos θ + i sin θ, = cos θ − i sin θ
z

Downloaded From : www.EasyEngineering.net


Downloaded From : www.EasyEngineering.net

480 Engineering Mathematics - II

1 1 1 1
cos θ = z + , sin θ = z−
2 z 2i z
dz iθ dz
= ie = iz ⇒ dθ = .
dθ iz
As θ varies from 0 to 2π, z moves on the unit circle |z| = 1.
∴ The contour C is the unit circle |z| = 1.
Z 2π Z
dz
F(sin θ, cos θ)dθ = f (z) where C is |z| = 1.
0 iz
C

The integral can be evaluated using Cauchy’s residue theorem.

ww ✎
Worked Examples


w.E
Z 2π
1
Example 4.89. Evaluate dθ by contour integration.
0 2 + cos θ
[May 2011, Jun 2010, Dec 2010, May 2001]

asy
dz
Solution. Let z = eiθ . Then dθ = .
iz
1 1
Also, cos θ = z + .
2 z
En
As θ varies from 0 to 2π, z moves along the unit circle |z| = 1.
∴ C is |z| = 1.
Z 2π
1 gin
Z
1 dz
Z
2 dz
Now,
0 2 + cos θ
dθ =

=
C
2
ee
Z
1


z
2
1

2 + 2 z + z iz
=

dz 2
=
C
Z
1
4 + z + z iz

2
1
rin dz

g.n
i 4z + z + 1 z i (z + 2) + 1 − 4
C C
Z
2 1
= √ dz

=
i
2
i
C
Z

C
(z + 2)2 − ( 3)2


1
(z + 2 + 3)(z + 2 − 3)
√ dz =
2
i
C
Z
et
f (z)dz

1
where f (z) = √ √ .
(z + 2 + 3)(z +√2 − 3) √
The simple poles are −(2 + 3) and −(2 − 3). C is |z| = 1.
√ √ √
When z = −(2 + 3), |z| = | − (2 + 3)| = 2 + 3 > 1.

∴ z = −(2 + 3) lies outside C.

Downloaded From : www.EasyEngineering.net


Downloaded From : www.EasyEngineering.net

Complex Integration 481

√ √
When z = −(2 − 3), |z| = | 3 − 2| < 1.

∴ z = −(2 − 3) lies inside C. Z

By Cauchy’s residue theorem, f (z)dz = 2πiR(−(2 − 3)).
C

√ √ 1
Now, R(−(2 − 3)) = lim √ (z + 2 − 3) √ √
z→−(2− 3) (z + 2 + 3)(z + 2 − 3)
1 1
= √ √ = √
(−2 + 3 + 2 + 3) 2 3
Z 2π √
1 2 1 2π

ww ∴
0 2 + cos θ
dθ = 2π × R(−(2 − 3)) = 4π √ = √ .

Z 2π
i 2 3 3

w.E
Example 4.90. Evaluate
0
1
13 + 5 sin θ
dθ by using contour integration.
[Apr 2010]

asy
dz
Solution. Let z = eiθ , then dθ = .
iz
1 1
Also, sin θ = z− .
2i
En
z
As θ varies from 0 to 2π, z moves, on the unit circle |z| = 1.


Z 2π

0
1
gin
13 + 5 sin θ
dθ =
Z
1

13 + 2i5 z − 1z iz

dz

ee =
C
Z

C
2i

26i + 5 z − 1z iz

dz

rin
g.n
Z
1
=2 dz
26iz + 5z2 − 5
C

et
Z
1
=2   dz
2 26i
5 z + 5 z−1
C
Z
2 1
= 2 dz
5 
13
z + 5 i − 1 + 25 169
C
Z
2 1
= dz
5 13 2
 
144
C z + 5 i + 25
Z
2 1
=   2 dz
5 13 2

C z + 5 i + 12 5

Downloaded From : www.EasyEngineering.net


Downloaded From : www.EasyEngineering.net

482 Engineering Mathematics - II

Z
2 1
= 2 dz
5 i12 2
  
13
C z+ 5i − 5
Z
2 1
=    dz
5 z+ 13i
+ 12i
z+ 13i
− 12i
C 5 5 5 5
Z
2 1
=   dz
5 (z + 5i) z + 5i
C

1
Here f (z) = .

ww

(z + 5i) z + 5i
−i
The poles are z = −5i and z = , which are simple poles.
5

w.E Now, C is |z| = 1.


When z = −5i, |z| = | − 5i| = 5 > 1.
∴ z = −5i lies outside
−i
5
asy
C.
−i
When z = , |z| = = < 1.
5
1
5
i
∴ z = lies inside C.
5 En
gin
Z  −i 
By Cauchy’s residue theorem, f (z)dz = 2πiR .
5
C

Now, R
 −i 
5 z→ −i
5

= lim z +
i

ee1

5 (z + 5i) z + i
5
 =
1
=
−i + 5i 24i
5
.

rin

Z

0

1
13 + 5 sin θ
2
dθ = 2πiR
5
 −i  4πi 5
5
=
π
= .
5 24i 6 g.n
Example 4.91. Evaluate
cos 2θ
Z2π
dθ using contour integration.
et [Dec 2013]
5 + 4 cos θ
0 !
iθ dz 1 1
Solution. Let z = e , then dθ = and cos θ = z+ .
iz 2 z
As θ varies from 0 to 2π, z moves along |z| = 1.
∴ C is |z| = 1.
Now z2 = ei2θ = cos 2θ + i sin 2θ.

Downloaded From : www.EasyEngineering.net


Downloaded From : www.EasyEngineering.net

Complex Integration 483

∴ cos 2θ = R.P.ei2θ .

Z2π
z2
Z
cos 2θ dz
∴ dθ = R.P  
5 + 4 cos 2θ 5+4× 1
z+ 1 iz
0 C 2 z
!Z
1 z
= R.P  dz
z2 +1

i 5+2
C z

z2
Z
= R.P(−i) dz
2z2 + 5z + 2
C

ww = R.P
 −i  Z
2
C
z2
z2 + 52 z + 1
dz

w.E = R.P
 −i  Z
2
C
 2
z2
z + 45 + 1 − 25
16
dz

asy = R.P
 −i  Z
2 
z+ 5
z2
2
− 9
dz

En
C 4 16
 −i  Z z2
= R.P 2  2 dz

gin
2 
z + 45 − 34
C
 −i  Z z2
= R.P  dz

ee
 
2 z+ 5
− 3
z+ 5
+ 3
C 4 4 4 4

= R.P
 −i  Z
2

z2

z + 21 (z + 2)
dz
rin
= R.P
 −i
C
Z
f (z)dz where f (z) =  g.n
z2

et

2 z + 21 (z + 2)
C

1
The singular points are z = − and z = −2.
2
Now C is |z| = 1.
1 1 1
When z = − , |z| = | − | = < 1.
2 2 2
∴ z = − 12 lies inside C.
When z = −2, |z| = | − 2| = 2 > 1.
∴ z = −2 lies outside C.

Downloaded From : www.EasyEngineering.net


Downloaded From : www.EasyEngineering.net

484 Engineering Mathematics - II

1
z = − is a simple pole.
2
z2
! ! !
1 1 1
R − = lim z + f (z) = lim z +  
2 z→− 12 2 z→− 12 2 z + 1 (z + 2)
2
1 1
z2 4 4 1 2 1
= lim = = = × = .
z→− 12 z + 2 − 21 + 2 3
2
4 3 6

By Cauchy’s residue theorem


!
1 πi
Z
1
f (z)dz = 2πi × R − = 2πi × = .

ww
2 6 3
C
Z2π  i   πi  π π
cos 2θ

w.E ∴
0
5 + 4 cos θ
Z 2π
cos 3θ
dθ = R.P − ×
2 3
= R.P
6
= .
6

Example 4.92. Evaluate


0
asy
5 − 4 cos θ
dθ using contour integration.
[Jun 2013, May 2005]

En
dz 1 1
 
Solution. Let z = eiθ , then dθ = and cos θ = z + .
iz 2 z

gin
As θ varies from 0 to 2π, z moves along |z| = 1.
∴ C is |z| = 1.

∴ cos 3θ = R.P of z3 .
Z
cos 3θ
Z
ee
Now z3 = ei3θ = cos 3θ + i sin 3θ.

z3 dz
Z
z2rin dz

C
5 − 4 cos θ
dθ = R.P
C

5 − 4 2 z + z iz
1 1
 = R.P
C
g.n
2
5 − 2z − z i
z3 z3

et
Z  −1  Z
1
= R.P dz = R.P dz
i 5z − 2z2 − 2 i 2z2 − 5z + 2
C C
 −1  Z z 3
= R.P   dz
i 2 z2 − 52 z + 1
C
 i Z z3
= R.P dz
2 5 2
 
25
C z − 4 + 1 − 16
 i Z z 3
= R.P  2 dz
2 5 2
 
3
C z − 4 − 4

Downloaded From : www.EasyEngineering.net


Downloaded From : www.EasyEngineering.net

Complex Integration 485

 i Z z3
= R.P    dz
2 z− 5
+ 3
z− 5
− 3
C 4 4 4 4
 i Z z3
= R.P   dz
2 z − 21 (z − 2)
C
 i Z z3
= R.P f (z)dz where f (z) =   .
2 z − 12 (z − 2)
C

1
Singular points are and 2, which are simple poles.
2

ww Now, C is |z| = 1.
1 1 1
When z = , |z| = < 1 =⇒ z = lies inside C.

w.E
2 2 2
When z = 2, |z| = 2 > 1 =⇒ z = 2Zlies outside C.
1
By Cauchy’s residue theorem, f (z)dz = 2πiR .
2

asy
1 1
C

En

Now R = lim z − f (z)
2 z→ 21 2

gin
1
1  1 z3 8 12 1
R = lim z −   = −3
=− =− .
2 z→ 21 2 z − 1 (z − 2) 2
83 12
2


Z

0

cos 3θ
5 − 4 cos θ
i
dθ = R.P 2πiR
2 2ee
1
= −π
 −1 
12
π
= .
12

rin
g.n
Z2π
cos 2θ 2πa2 2
Example 4.93. Show that dθ = (a < 1).
1 − 2a cos θ + a2 1 − a2

et
0
dz iθ
Solution. Let z = e , dθ = .
iz
1
2 cos θ = z + .
z !
1 2 1
cos 2θ = z + 2 .
2 z
As θ varies from 0 to 2π, z varies along the unit circle C, |z| = 1.

Z2π Z 
z2 + z12
1

cos 2θ 2 dz
∴ dθ =
1 − 2a cos θ + a2
 
1 − a z + 1z + a2 iz
0 C

Downloaded From : www.EasyEngineering.net


Downloaded From : www.EasyEngineering.net

486 Engineering Mathematics - II

z4 + 1
Z
1 dz
=
2i a(z2 +1)

+ a2 z

z2 1 − z
C
z(z4 + 1)
Z
1 dz
= 2 2 2
2i z (z − az − a + a z) z
C
z4 + 1
Z
1
= dz
2i z2 ({(z − a) − az(z − a)})
C
z4 + 1
Z
1
= dz
2i z2 (z − a)(1 − az)

ww
C
z4 + 1
Z
1
= f (z)dz where f (z) = .
2i z2 (z − a)(1 − az)

w.E
C

1
The singular points are z = 0, z = a, z = .
a
C is |z| = 1.
asy
When z = 0, |z| = |0| = 0 < 1.
∴ z = 0 lie inside C.
En
When z = a, |z| = |a| = a < 1 [∵ a2 < 1 ⇒ a < 1].
∴ z = a lies inside C
gin
ee
1 1 1 1
When z = , |z| = | | = > 1 [∵ a < 1 ⇒ > 1].
a a a a

rin
1
∴ z = lies outside C.
a
By Cauchy’s residue theorem,
Z
f (z)dz = 2πi × Sum of the residues.
g.n
C
Now, z = 0 is a pole of order 2.

∴ R(0) =
1
lim
d 
1! z→0 dz
(z − 0)2 f (z)

et
z4 + 1
!
d 2
= lim z 2
z→0 dz z (z − a)(1 − az)
z4 + 1
" #
d
= lim
z→0 dz (z − a)(1 − az)

(z − a)(1 − az)4z3 − (z4 + 1){(z − a)(−a) + (1 − az) · 1}


= lim
z→0 [(z − a)(1 − az)]2

Downloaded From : www.EasyEngineering.net


Downloaded From : www.EasyEngineering.net

Complex Integration 487

0 − 1{a2 + 1} a2 + 1
= = −
a2 a2
z = a is a simple pole.

R(a) = lim(z − a) f (z)


z→a
z4 + 1
= lim(z − a)
z→a z2 (z − a)(1 − az)
z4 + 1
= lim 2
z→a z (1 − az)

ww Z
= 2
a4 + 1
a (1 − a2 )

w.E ∴
C
f (z)dz = 2πi[R(0) + R(a)]

"
a4 + 1 a2 + 1
#

asy
= 2πi 2 −
a (1 − a2 ) a2
" 4
a + 1 − (1 − a2 )(1 + a2 )
#
= 2πi

En " 4
a2 (1 − a2 )
a + 1 − (1 − a4 )
#

gin= 2πi
"
a2 (1 − a2 )
2a4
#

ee
= 2πi 2
a (1 − a2 )

= 2πi
2a2
=
4πia2
1 − a2 1 − a2
rin

Z2π
cos 2θ
1 − 2a cos θ + a2
dθ =
1 4πia2
· =
2πa2
2i 1 − a2 1 − a2
.
g.n
0

Example 4.94. Evaluate


0
Z 2π
1
1 − 2p sin θ + p2
dz
dθ, |p| < 1. et
[May 2009]

Solution. Let z = eiθ , then dθ = .


iz
1 1
Also, sin θ = z− .
2i z
As θ varies from 0 to 2π, z moves along the unit circle |z| = 1.
Z 2π Z
1 1 dz
Now, 2
dθ =
1 − 2p sin θ + p
 
0 1 − 2p 2i1 z − 1z + p2 iz
C

Downloaded From : www.EasyEngineering.net


Downloaded From : www.EasyEngineering.net

488 Engineering Mathematics - II

Z
1 dz
= −i p z2 −1
 
1− i z + p2 z
C
Z
iz dz
= −i
iz − pz2 + p + p2 iz z
ZC
1
=− dz
pz2 − iz − p2 iz − p
C
Z
1
=− dz
pz2 − i(1 + p2 )z − p
C

ww
Z
1 1
=−   dz
p z2 − i 1+p2
z−1
C p

w.E
Z
1 1
=−
p   p2 +1 2  p2 +1 2 dz
C z − i 2p − 1 + 2p

asy
Z
1 1
=−  p2 +1 2 dz
p 
z − i 2p + (1+p2 )2 −4p2
C 4p2

En
Z
1 1
=−
p   p2 +1 2  1−p2 2 dz

gin
C z − i 2p + 2p
Z
1 1
=−  p2 +1 2  i(1−p2 ) 2 dz

ee
p 
z − i 2p − 2p
C
Z
1 1
=−
p
C
  p2 +1   1−p2 
z − i 2p + i 2p
 2   2  dz
z − i p2p+1 − i 1−p
2p
rin
g.n
Z
1 1
=−      dz
p z+ i
1 − p2 − 1 − p2 z− i
1 + p2 + 1 − p2

et
C 2p 2p
Z
1 1
=−    dz
p z+ i 2 i
C 2p (−2p ) z− 2p 2
Z Z
1 1 1
=−   dz = − f (z)dz
p i
(z − ip) z − p p
C C
1
where f (z) =  
(z − ip) z − pi

i
The simple poles are ip and .
p

Downloaded From : www.EasyEngineering.net


Downloaded From : www.EasyEngineering.net

Complex Integration 489

C is |z| = 1.
When z = ip, |z| = |ip| = |p| < 1.
∴ z = ip lies inside
C.
i i 1
When z = , |z| = = > 1.
p p |p|
i
∴ z = p lies outside C.
Z  
By Cauchy’s residue theorem, f (z)dz = 2πiR ip .
C
1
Now, R(ip) = lim (z − ip)

ww
 
z→ip (z − ip) z − pi
1 1 p
= =  2 =

w.E ∴
ip −
Z 2π
i
p

1
i p p−1
2
i(p − 1)

1
dθ = − 2πi × R(ip) = − 2πi 2
1 p
=

.

asy
1 − 2p sin θ + p 2 p p i(p − 1) 1 − p2
0
Z 2π
sin2 θ
Example 4.95. Evaluate dθ, a > b > 0. [Dec 2012, May 1998]

Solution. Let, I =
Z 2π 0
sin2 θ
En
a + b cos θ
dθ =
Z 2π
1 − cos 2θ
dθ.
0 a + b cos θ
Put z = eiθ then dθ = .
dz
iz gin 0 2(a + b cos θ)

Now, z2 = ei2θ = cos 2θ + i sin 2θ.


1 − z2 = 1 − cos 2θ − i sin 2θ.
∴ 1 − cos 2θ = R.P (1 − z2 ).
ee rin
As θ varies from 0 to 2π, C varies along the unit circle |z| = 1.
1 − z2 1 − z2 g.n
et
Z Z
dz 1 dz
Hence, I = R.P   iz = R.P 2 1  iz
2a + b z + z

b 1
C 2 a+ 2 z+ z C

z(1 − z2 ) 1 − z2
Z Z
1 dz 1
= R.P = R.P dz
2i (2az + bz2 + b) z 2i 2az + bz2 + b
C C
1
Z
1−z 2 1
Z
1 − z2
= R.P dz = R.P dz
b z2 + 2a z + ab 2 + 1 − ba2
2
2i 2bi

z+1

C b C
1 − z2
Z
1
= R.P dz
z + ba 2 − a b−b
2 2
2bi 
2
C

Downloaded From : www.EasyEngineering.net


Downloaded From : www.EasyEngineering.net

490 Engineering Mathematics - II

1 − z2
Z
1
= R.P √ dz
2bi z+ a 2
− a2 −b2 2
C b b
1 − z2
Z
1
= R.P √ √ dz
2bi z+ a
+ a2 −b2 
z+ a
− a2 −b2 
C b b b b
1 − z2
Z
1
= R.P dz
2bi (z − z1 )(z − z2 )
C
1 − z2
Z
1
= R.P f (z)dz where f (z) = where
2bi (z − z1 )(z − z2 )

ww
C
√ √
a a2 − b2 a a2 − b2
z1 = − − and z2 = − + .
b b b b

w.E z1 and z2 are simple poles.


a
When z = z1 , |z| = |z1 | = − −
b

b

a2 − b2 a
= +
b

a2 − b2 a
b
> > 1.
b
a
b asy a 2
As a > b =⇒ > 1 =⇒ 2 > 1, a2 > b2 , a2 − b2 > 0.
b
∴ z1 lies outside C.
En
Also, z1 z2 = 1 (products of the roots)
Now, z2 =
1
z1
=⇒ |z2 | =
gin
1
|z1 |
< 1.
∴ z2 lies inside C.
By Cauchy’s residue theorem, ee Z
f (z)dz = 2πiR(z2 ).
C
rin
Now, R(z2 ) = lim (z − z2 ) f (z)
z→z2
√ g.n
2
 a
R − +

Now, 1 − z2 = 1 −
b
a2 − b2 
b

 −a + a2 − b2 2
= lim (z − z2 )
z→z2
1 − z2
=
1 − z22
(z − z1 )(z − z2 ) z2 − z1
.
et
b √
b − (a + a2 − b2 − 2a a2 − b2 )
2 2
=
b2 √
b − a − a + b2 + 2a a2 − b2
2 2 2
=
b√2
2b2 − 2a2 + 2a a2 − b2
=
b2

Downloaded From : www.EasyEngineering.net


Downloaded From : www.EasyEngineering.net

Complex Integration 491

√ √ √
2a a2 − b2 − 2(a2 − b2 ) 2 a2 − b2 (a − a2 − b2 )
= =
√ b2 b2
2 a2 − b2
z2 − z1 =
√ b √ √
2 a2 − b2 (a − a2 − b2 ) a − a2 − b2
∴ R(z2 ) = √ ×b=
b2 2 a2 − b2 b
 √ 
1  a − a2 − b2  π p
∴ I = R.P · 2πi   = (a − a2 − b2 )
2bi b b2

ww
π
1 + 2 cos θ
Z
Example 4.96. Evaluate dθ. [Dec 2011]
0 5 + 4 cos θ !
iθ dz 1 1

w.E
Solution. Let Z = e . Then dθ = and cos θ = z+ .
iz 2 z
As θ varies from 0 to 2π, z moves along |z| = 1.
By the properties of definite integrals,

Z π asy
1 + 2 cos θ 1
Z 2π
1 + 2 cos θ

0 5 + 4 cos θ
En
dθ =
2 0 5 + 4 cos θ

1 + 2 21 (z + 1z ) dz

gin
Z
1
=
2 5 + 4 12 (z + 1z ) iz
C

ee
=
1
2i

1
Z

C
Z
1+z+
5 + 2(z
1
z dz
+ 1z ) z
(z2 + z + 1)z dz rin
=
2i
C
·
z(2z2 + 5z + 2) z
g.n
z2 + z + 1

et
Z
1
= dz
2i z(2z2 + 5z + 2)
C
z2 + z + 1
Z
1
= dz
2i 2z(z2 + 25 z + 1)
C
z2 + z + 1
Z
1
=   dz
4i z (z + 45 )2 + 1 − 25
C 16

z2 + z + 1
Z
1
=   dz
4i z (z + 45 )2 − ( 43 )2
C

Downloaded From : www.EasyEngineering.net


Downloaded From : www.EasyEngineering.net

492 Engineering Mathematics - II

z2 + z + 1
Z
1
=    dz
4i z z+ 5 3 5 3
C 4 + 4 z+ 4 − 4

z2 + z + 1
Z
1
=   dz
4i z(z + 2) z + 21
C
z2 + z + 1
Z
1
= f (z)dz, where f (z) =  .
4i z(z + 2) z + 12
C

1
The simple poles are z = 0, z = −2 and z = − .
2

ww C is |z| = 1.
When z = 0, |z| = 0 < 1.

w.E ∴ z = 0 lies inside C.


When z = −2, |z| = | − 2| = 2 > 1.
∴ z = −2 lies outside
When z =
−1
2
−1
asy
C.
1
, |z| = = < 1.
2 2
1
∴ z = − lies inside C.
2 En
gin
Z
1
By Cauchy’s residue theorem, f (z)dz = 2πi[R(0) + R(− )].
2
C

z→0
ee
Now, R(0) = lim(z − 0) f (z) = lim z
z→0
z2 + z + 1

z(z + 2) z + 12
1
 = = 1.
1
rin
g.n
−1 1
R( ) = lim (z + ) f (z)
2 1
z→− 2 2

z→− 12

= lim
1
= lim (z + )

+z+1z2
z2 + z + 1

=

2 z(z + 2) z + 1


1

 42
2
1
+1
 =
3 −4
!
= −1.
et
1 z(z + 2) −1 −1 4 3
2 +2
z→− 2
2
Z
∴ f (z)dz = 2πi[−1 + 1] = 0.
C
π
1 + 2 cos θ
Z
1
dθ = 0 = 0.
0 5 + 4 cos θ 4i

Downloaded From : www.EasyEngineering.net


Downloaded From : www.EasyEngineering.net

Complex Integration 493

Evaluation of improper integrals


Z of rational functions ∞
We consider integrals of the type f (x)dx. We define
−∞
Z ∞ Z 0 Z R2
f (x)dx = lim f (x)dx + lim f (x)dx (1)
−∞ R1 →∞ −R1 R2 →∞ 0

when both limits exists on the RHS.


The Cauchy principal value is defined by
Z ∞ Z R

ww if the limit exists. Z


PV f (x)dx = lim
−∞ R→∞ −R
f (x)dx (2)

w.E
When f (x) is even
Cauchy’s lemma

−∞
f (x)dx = 2
Z

0

f (x)dx or
Z

0

f (x)dx =
1
2
Z ∞

−∞
f (x)dx.

asy
If f (z) is a continuous function such that |z f (z)| → 0 as |z| → ∞ on the upper
R

En
semicircle S : |z| = R, then S f (z)dz → 0 as R → ∞.
Z ∞
P(x)
Type II. We consider integrals of the form dx where P(x) and Q(x) are

gin −∞ Q(x)
polynomials in x such that the degree of Q(x) is atleast 2 more than the degree of
P(x) and Q(x) does not vanish for any real x.

To evaluate this, consider


Z eeP(z)
dz rin S

C
Q(z)

g.n
>

>

where C is the simple closed curve


consisting of the real axis from −R to R
and the upper semicircle, S : |z| = R for
large R taken in the anticlockwise sense.
(−R, 0)
et (R, 0)

Z Z R Z
P(z)
Now f (z) = . Then f (z)dz = f (x)dx + f (z)dz (1)
Q(z) −R S
C Z
By Cauchy’s residue theorem f (z)dz = 2πi[ sum of the residues ].
R C
By Cauchy’s lemma S
f (z)dz → 0 as R → ∞

Downloaded From : www.EasyEngineering.net


Downloaded From : www.EasyEngineering.net

494 Engineering Mathematics - II

RR
∴ From (1) lim −R f (x)dx = 2πi[ sum of the residues ].
R∞ R→∞
i.e., −∞ f (x)dx = 2πi[ sum of the residues ].
Z ∞
p(x)
=⇒ dx = 2πi[ sum of the residues ].
−∞ Q(x) ✎ ☞
Worked Examples
✍ ✌

x2 dx
Z
Example 4.97. Evaluate 2 2
using contour integration.
−∞ (x + 1)(x + 4)
Z ∞ [Dec 2010, May 2007]
x2 dx

ww Solution. Let I = 2

The integrand is of the form


2
−∞ (x + 1)(x + 4)
P(x)
.

where degree of Q(x) is 2 more than that of P(x)

w.E Q(x)
and Q(x) does not vanish for any real x. Consider the integral
Z
z2 dz
(z2 + 1)(z2 + 4)

asy
C
where C is the simple closed curve consisting of the real axis from −R to R and the
upper semi circle S : |z| = R taken in the anticlockwise sense.
Let f (z) = 2
z2 dz
(z + 1)(z2 + 4)
.
En
Now Z Z
gin
f (z)dz =
Z R
f (x)dx + f (z)dz. (1)

Evaluation of
R
f (z)dz.
C

ee −R S

• 2i
rin S

g.n
C
>

>

Poles are given by (z2 + 1)(z2 + 4) = 0. • i

i.e., (z + i)(z − i)(z + 2i)(z − 2i) = 0

et
−R R
• −i
i.e., z = i, −i, 2i and −2i. • −2i

z = i and z = 2i lie inside C.

Z
By Cauchy’s residue theorem, f (z)dz = 2πi[R(i) + R(2i)].
C

z2 −1 1
Now, R(i) = lim(z − i) f (z) = lim(z − i) 2
= =−
z→i z→i (z + i)(z − i)(z + 4) 2i(3) 6i

Downloaded From : www.EasyEngineering.net


Downloaded From : www.EasyEngineering.net

Complex Integration 495

z2 4 1
R(2i) = lim (z − 2i) 2
= = .
z→2i (z + 2i)(z − 2i)(z + 1) 3(4i) 3i
1 π
Z  −1 1  2πi  1 1  2 − 1
∴ f (z)dz = 2πi + = + = 2π = 2π = .
6i 3i i 3 6 6 6 3
C

R
π
Z Z
(1) ⇒ f (x)dx + f (z)dz = .
−R S 3

z3 z3 1
Now, lim z f (z) = lim = lim = = 0.
z→∞ (z2 + 1)(z2 + 4) z→∞ z4 1 + 4 1 + 1 ∞
z→∞

z2 z2

ww ∴ By Cauchy’s lemma

w.E
Z
f (z)dz = 0 as R → ∞
S
Z R Z ∞
π π
f (x)dx = .

asy
lim f (x)dx = ⇒
R→∞ −R 3 −∞ 3
Z ∞
1
Example 4.98. Evaluate
Z ∞
1 En 2
−∞ (x + 1)
3
dx. [Dec 2009]

gin
Solution. Let I = 2 + 1)3
dx.
−∞ (x
P(x)
The integrand is of the form where the degree of Q(x) is at least 2

Consider the integral


ee 1
Q(x)
more than that of P(x)Zand Q(x) does not vanish for any real x.

(z + 1)3
2
dz where C is the simple closed curve
rin
g.n
C
consisting of the real axis from −R to R and the upper semi circle
S : |Z| = R taken in the anti clockwise sense.
Let f (z) = 2
1
(z + 1)3
Z
.

ZR Z
et
Now, f (z)dz = f (x)dx + f (z)dz (1)
S
C −R
R
Let us evaluate f (z)dz.
C
The poles are given by (z2 + 1)3 = 0.
(z + i)3 (z − i)3 = 0

Downloaded From : www.EasyEngineering.net


Downloaded From : www.EasyEngineering.net

496 Engineering Mathematics - II

i and −i are poles of order 3. i will


Z lie inside C and −i lie outside C.
By Cauchy’s residue theorem f (z)dz = 2πiR(i).
C

1 d2
lim 2 (z − i)3 f (z)

>
Now, R(i) =
2! z→i dz • i
2
" #
1 d 3 1 >
= lim (z − i) (−R, 0) (R, 0)
2! z→i dz2 (z + i)3 (z − i)3

ww
• −i

1 d2 h i
= lim 2 (z + i)−3
2! z→i dz

w.E 1 d h i
= lim (−3)(z + i)−4
2! z→i dz
1 1 6 6 3
= lim(−3)(−4)(z + i)−5 = 12(2i)−5 = 5 5 = =
2! z→i
Z
asy
2
3 3π
2 i 32i 16i

En
∴ f (z)dz = 2πi × = .
16i 8
C

Now (1) =⇒
ZR

gin
f (x)dx +
Z
f (z)dz =

8
. (2)

z→∞
−R

z
Also lim z f (z) = lim
+ 1) 3
= lim 
z→∞ (z2
z→∞ 6
ee
S

z
z 1 + z12
3 = 0.
rin
g.n
Z
∴ By Cauchy’s lemma f (z)dz = 0 as R → ∞.
S
ZR

et
Z

∴ (2) =⇒ lim f (x)dx + lim f (z)dz =
R→∞ R→∞ 8
−R S
Z∞

f (x)dx = .
8
−∞


x2 dx
Z
Example 4.99. Evaluate the integral using contour integration.
0 x4 + 1
[Jun 2013, May 2009]
x2 dx
Solution. Let f (x) = .
x4 + 1

Downloaded From : www.EasyEngineering.net


Downloaded From : www.EasyEngineering.net

Complex Integration 497


x2 dx 1 ∞ x2 dx
Z Z
Since f (x) is even 4+1
= .
0 x Z 2 −∞ x4 + 1

x2 dx
We shall evaluate f (z) = 4
.
Z ∞ 2 −∞ x + 1
x dx
Let I = 4
.
−∞ x + 1
P(x)
The integrand is of the form where degree of Q(x) is 2 more than
Q(x)
that of P(x) and Q(x) does
Z ∞ not2 vanish for any real x.
x dx
Consider the integral where C is the simple closed curve

ww
4
−∞ x + 1
consisting of the real axis from −R to R and the upper semi circle
S : |z| = R taken
Z ∞ in the anticlockwise sense.

w.E Let f (z) =


z2 dz
4
−∞ z + 1
.
The poles are given by z4 + 1 = 0.
i.e., z4 = −1
1 asy
π
z4 = (−1) 4 = ei(2n+1) 4 , n = 0, 1, 2, 3.

En
π 3π 5π 7π
∴ z = ei 4 , ei 4 , ei 4 , ei 4 .
gin S

Let the poles be z1 , z2 , z3 and z4 .


π

i 3π
π
4

π
4

1
z1 = ei 4 = cos + i sin = √ + i √ .
2
−1
1
2
1
ee (−R, 0)
• z2

• z3

rin
>
z1 •

z4 •
>

(R, 0)

g.n
z2 = e 4 = cos + i sin = √ +i√ .
4 4 2 2

i 7π
−1

z4 = e 4 = cos

1
z3 = ei 4 = √ − i √ .
2 2
+ i sin
7π 1
= √ −i√ .
1
et
4 4 2 2
Only z1 and z2 lie inside C. Z  π 3π

By Cauchy’s residue theorem, f (z)dz = 2πi R(ei 4 ) + R(ei 4 ) .
C

π
Now, R(ei ) = R(z1 ) = lim (z − z1 ) f (z)
4 z→z1

Downloaded From : www.EasyEngineering.net


Downloaded From : www.EasyEngineering.net

498 Engineering Mathematics - II

z2
= lim (z − z1 )
z→z1 (z − z1 )(z − z2 )(z − z3 )(z − z4 )
z21
=
(z1 − z2 )(z1 − z3 )(z1 − z4 )
( √1 + i √1 )2
2 2
= √ √ √ √
2( 2 + i 2)( 2i)
(1 + i)2 1+i
= √ √ √ = √
2 2 2(1 + i) 2i i4 2

R(ei ) = R(z2 ) = lim (z − z2 ) f (z)

ww 4

= lim (z − z2 )
z→z2
z→z2
z2
(z − z1 )(z − z2 )(z − z3 )(z − z4 )

w.E =
z22
(z2 − z1 )(z2 − z3 )(z2 − z4 )

= √
( −1

2
− 2(− 2 + i 2)( 2i)asy
√ + i √1 )2
2
√ √

=
(−1 + i)2

2(−2)i 2(−1 + i) En=− √
1 (−1 + i)
4 2 i
= √
1 1 − i
4 2 i


Z
f (z)dz = 2πi √
"

gin
1 1+i
4 2 i
+ √
1 1 − i
4 2 i
 #

=
C
2πi
√ [1 + i + 1 − i]
4 2i
π
= √ 2= √ .
π
ee rin
2 2
ZR
2
Z
π g.n
∴⇒ (1) f (x)dx +
−R
f (z)dz = √
S
2
et (2)

z3
Now, lim z f (z) = lim → 0.
z→∞ z→∞ z4 +
Z1
∴ By Cauchy’s lemma, f (z)dz → ∞ as R → ∞.
S
ZR
π
Z
∴ (2) =⇒ lim f (x)dx + lim f (z)dz = √
R→∞ R→∞ 2
−R S

Downloaded From : www.EasyEngineering.net


Downloaded From : www.EasyEngineering.net

Complex Integration 499

Z∞
π
f (x)dx = √
2
−∞
Z∞
π
∴ f (x)dx = √
2 2
0
Z∞
x2 π
i.e., 4
dx = √ .
x +1 2 2
0

ww

x2 dx
Z
Example 4.100. Evaluate a > 0, b > 0.[June 2013, May 2009]
0 (x2 + a2 )(x2 + b2 )
x2

w.E
Solution. Let f (x) = 2
(x + a2 )(x2 + b2 )
R∞
Since f (x) is even, f (x)dx = 21
.
R∞
f (x)dx.

asy
0 −∞
Z ∞
x2
Let I = 2 2 2 2
.
−∞ (x + a )(x + b )

P(x) En S
The integrand is of the form
Q(x)
gin
where
degree of Q(x) is 2 more than that of P(x)
• ai

• bi

>
and Q(x) does not vanish for any real x.
ee (−R, 0)
• −bi

rin
• −ai
>
(R, 0)

Consider the integral


Z
z2
(z2 + a2 )(z2 + b2 )
dz where C is the simple closed curveg.n
Z Z
C
consisting of the real axis from −R to R and the upper semi circle S : |z| = R taken
in the anticlockwise sense.
R Z
et
Now, f (z)dz = f (x)dx + f (z)dz (1)
−R S
C
R
We shall evaluate f (z)dz.
C

z2 z2
We have, f (z) = =
(z2 + a2 )(z2 + b2 ) (z + ai)(z − ai)(z + bi)(z − bi)

Downloaded From : www.EasyEngineering.net


Downloaded From : www.EasyEngineering.net

500 Engineering Mathematics - II

±ai and ±bi are the simple poles.


But ai and bi only lie inside C.
∴ By Cauchy’s residue theorem
Z
f (z)dz = 2πi[R(ai) + R(bi)].
C
z2
R(ai) = lim (z − ai)
z→ai (z + ai)(z − ai)(z2 + b2 )
−a2 a
= = .

ww 2ai(b2 − a2 ) 2i(a2 − b2 )
R(bi) =
b
2i(b − a2 )
2

w.E
Z  a b 
Now, f (z)dz = 2πi +
2i(a2 − b2 ) 2i(b2 − a2 )
C
2πi  a−b

asy π

= = .
2i (a − b)(a + b) a+b

En
Z R
π
Z
∴ (1) =⇒ f (x)dx + f (z)dz = . (2)
−R S a+b

z→∞ gin
Also, lim z f (z) = lim
(z2 + a2 )(z2 + b2 )
z→∞
z 3
→ 0 as z → ∞

ee
Z
∴ By Cauchy’s lemma, f (z)dz → 0 as R → ∞.

ZR Z
S

π rin
Now (2) =⇒ lim
R→∞
−R
f (x)dx + lim
R→∞
S
f (z)dz =
a+b
.

g.n
R
π
et
Z
lim f (x)dx =
R→∞ a+b
Z −R

π
f (x)dx =
a+b
Z−∞

π
f (x)dx = .
0 2(a + b)
Z∞
x2 − x + 2 5π
Example 4.101. Show that 4 2
dx = .
x + 10x + 9 12
−∞
[Dec 2013, May 2011, Jun 2010]

Downloaded From : www.EasyEngineering.net


Downloaded From : www.EasyEngineering.net

Complex Integration 501

Z∞
x2 − x + 2
Solution. Let I = dx.
x4 + 10x2 + 9
−∞
P(x)
The integrand is of the form where, the degree of Q(x) is atleast 2
Q(x)
more than that of P(x) and
Z Q(x) does not vanish for any real x.
z2 − z + 2
Consider the integral dz, where C is the simple closed
z4 + 10z2 + 9
C

curve consisting of the real axis from −R to R and the upper semi circle

ww S : |z| = R, taken in the anticlockwise sense.


Let f (z) = 4
z2 − z + 2
z + 10z 2
Z +9
.

w.E Now
Z

C
f (z)dz =
R

−R
Z
f (x)dx +
s
f (z)dz. (1)

asy
Z
Let us evaluate f (z)dz.
Y
C
The poles are given by

z4 + 10z2 + 9 = 0 En • 3i

(z2 + 1)(z2 + 9) = 0
gin • i

>

>
X

ee
−R R
2 2 • −i
z + 1 = 0, z +9=0
• −3i

z2 = −1, z2 = −9
rin
z = ±i, z = ±3i.

g.n
et
z = i and 3i lies inside C.
By
Z Cauchy’s residue theorem,
f (z)dz = 2πi[R(i) + R(3i)]
C

R(i) = lim(z − i) f (z)


z→i
z2 − z + 2
= lim(z − i)
z→i (z + i)(z − i)(z2 + 9)
−1 − i + 2 1 − i
= = .
2i(−1 + 9) 16i

Downloaded From : www.EasyEngineering.net


Downloaded From : www.EasyEngineering.net

502 Engineering Mathematics - II

R(3i) = lim (z − 3i) f (z)


z→3i

✘ z2 − z + 2
= lim ✘ −✘
(z ✘ 3i) ✘ + 3i)
z→3i (z2 + 1)✘ −✘
(z ✘ 3i)(z
−9 − 3i + 2 −7 − 3i 7 + 3i 7 + 3i
= = =− = .
(−9 + 1)6i −48i −48i 48i
Z " # " #
1 − i 7 + 3i 3 − 3i + 7 + 3i 10 5π
∴ f (z)dz = 2πi + = 2πi = 2π × = .
16i 48i 48i 48 12
C
Z
By Cauchy’s lemma f (z)dz = 0 as R → ∞.

ww
S

ZR Z

∴ (1) ⇒ = f (x)dx + f (z)dz

w.E 12
−R

Taking lim as R → ∞ we get


s


= lim
12 R→∞
ZR
asy
f (x)dx + lim
R→∞
Z
f (z)dz


Z∞
−R

En s

12
=
−∞
f (x)dx + 0
gin
ee
Z∞
x2 − x + 2 5π
∴ dx = .

rin
4 2
x + 10x + 9 12
−∞
Z∞
Example 4.102. Evaluate using contour integration
−∞
x2
(x2 + 1)2
g.n
dx. [Dec 2011]

Solution. Let I =
Z∞

−∞
x2
(x2 + 1)2
P(x)
dx.
et
The integrand is of the form where the degree of Q(x) is atleast 2
Q(x)
more than that of P(x)Zand Q(x) does not vanish for any real x.
z2
Consider the integral dz where C is the simple closed curve
(z2 + 1)2
C
consisting of the real axis from −R to R and the upper semi circle
S : |z| = R taken in the anticlockwise direction.

Downloaded From : www.EasyEngineering.net


Downloaded From : www.EasyEngineering.net

Complex Integration 503

S
z2
Let f (z) = 2

>
(z + 1)2 • i

z2 > X
= −R R
[(z + i)(z − i)]2 • −i

z2
= .
(z + i)2 (z − i)2
Z ZR Z

ww Now
C
f (z)dz =

Let us evaluate
−R
Z
f (x)dx +
s
f (z)dz (1)

w.E
f (z)dz.
C
The poles are given by (z + i)2 (z − i)2 = 0

asy
i.e., z = i and − i.
i and −i are poles of order 2. Only
Z i lies inside C.

En
∴ By Cauchy’s residue theorem f (z)dz = 2πiR(i).

gin
C
1 d
Now, R(i) = lim {(z − i)2 f (z)}
1! z→i dz

ee
z2 z2
( ) ( )
d 2 d
= lim (z − i) = lim
z→i dz (z + i)2 (z − i)2 z→i dz (z + i)2

= lim
(z + i)2 · 2z − z2 2(z + i) (−4)2i − (−4)2(2i) −8i + 16i
4
= = =
8i i
= .
rin
g.n
z→i (z + i) 16 16 16 2
Z i
∴ f (z)dz = 2πi × R(i) = 2πi = −π.
2
C

By Cauchy’s lemma,
Z

S
f (z)dz = 0 as R → ∞.
et
ZR Z
∴ (1) ⇒ −π = f (x)dx + f (z)dz
s
−R
ZR Z
− π = lim f (x)dx + lim f (z)dz
R→∞ R→∞ s
−R

Downloaded From : www.EasyEngineering.net


Downloaded From : www.EasyEngineering.net

504 Engineering Mathematics - II

Z∞
x2
−π= dx + 0
(x2 + 1)2
−∞
Z∞
x2
dx = −π.
(x2 + 1)2
−∞

Jordan’s lemma. If f (z) is a continuous function such that | f (z)| → 0 uniformly as


R
|z| → ∞, then S f (z)eimz dz → 0 as R → ∞, where S is the upper semi circle |z| = R.
Type III

ww Evaluation of integrals of the form


−∞
P(x)
Q(x)
Z ∞
sin mxdx and
Z ∞

−∞
P(x)
Q(x)
cos mxdx, m > 0,

w.E
where P(x) and Q(x) are polynomials in x such that the degree of Q(x) is greater
than the degree of P(x) and Q(x) does not vanish for any real x.

asy
To evaluate this integral we consider
P(z)
S

En
R

>
f (z)eimz dz where f (z) = and C is
C Q(z)
a simple closed curve consisting of the >

real axis from −R → R and the upper


gin (−R, 0) (R, 0)

semi circle S : |z| = R taken in the


anticlockwise sense. Now
Z
imz
f (z)e dz =
Z R
imx
Z
f (x)e dx+ f (z)eimz dz.
ee rin
C
−R S

g.n
et
R
We evaluate f (z)eimz dz using Cauchy’s residue theorem.
C Z
By Jordan’s lemma f (z)eimz dz → 0 as R → ∞.
S
Z R Z
Hence, (1) =⇒ lim f (x)eimx dx = f (z)eimz dz.
R→∞ −R
C
Z ∞ Z
f (x)eimx dx = f (z)eimz dz.
−∞
C

Downloaded From : www.EasyEngineering.net


Downloaded From : www.EasyEngineering.net

Complex Integration 505

Equating the real and imaginary parts we get the values of


Z∞ Z∞
f (x) cos mxdx and f (x) sin mxdx respectively.
−∞ −∞ ✎ ☞
Worked Examples
✍ ✌
Z∞
cos ax
Example 4.103. Evaluate dx, a > 0. [Nov 2010]
x2 + 1
0
Solution.

ww Let us consider the integral


Z
eiaz
dz
Y

w.E
S
z2 + 1
C

>
where C is the simple closed curve • i

consisting of the real axis from −R to R >

asy
X
(−R, 0) (R, 0)
• −i
and the upper semicircle S : |z| = R taken
in the anticlockwise sense.
Let f (z) = 2
1
. En
Now,
Z
z +1

iaz
ZR
gin
Z
e f (x)dx + eiaz f (z)dz
iax
(1)

ee
e f (z)dz =
C −R S

rin
Z
iaz
Let us evaluate e f (z)dz.

g.n
C
The poles are given by z2 + 1 = 0.
i.e., z2 = −1
=⇒ z = ±i are the simple poles.
Only z = i lies inside C. Z
et
∴ By Cauchy’s residue theorem, eiaz f (z)dz = 2πiR(i)
C

eiaz e−a
Now R(i) = lim(z − i)eiaz f (z) = lim(z − i) =
z→i z→i (z + i)(z − i) 2i
Z
e−a
∴ eiaz f (z)dz = 2πi = πe−a .
2i
C

Downloaded From : www.EasyEngineering.net


Downloaded From : www.EasyEngineering.net

506 Engineering Mathematics - II


1
Also | f (z)| = → 0 as |z| → ∞.
z2 + 1 Z
∴ By Jordan’s Lemma, eiz f (z)dz → 0 as R → ∞.
S
ZR Z
iax
Now, (1) =⇒ πe −a
= e f (x)dx + eiaz f (z)dz.
−R S
Taking limit as R → ∞ on both sides we get
ZR Z
iax
−a
πe = lim e f (x)dx + lim eiaz f (z)dz.
R→∞ R→∞

ww
−R S
Z∞
eiax
i.e., πe−a = dx
x2 + 1

w.E Z∞
−∞

cos ax + i sin ax
x2 + 1
dx = πe−a
−∞
Z∞
cos ax
dx + i asy
Z∞
sin ax
dx = πe−a
−∞
x2 + 1
−∞

En
x2 + 1
Equating the real parts on both sides, we get
Z∞
cos ax
2
x +1
dx = πe−a
gin
ee
−∞
Z∞
cos ax  cos ax 

rin
−a
2 dx = πe [∵ is even]
x2 + 1 x2 + 1
0

g.n
Z∞
cos ax πe−a
∴ dx = .
x2 + 1 2

et
0

Z ∞
x sin mx
Example 4.104. Evaluate 2 + a2
dx, a > 0, m > 0. [May 2005]
0 xZ
z sin mz
Solution. Consider the integral dz, where C is the simple closed curve
z2 + a2
C
consisting of the real axis from −R to R and the upper semi circle
S : |z| = R, taken in the anticlockwise.
z
Let f (z) = 2 dz.
z + a2

Downloaded From : www.EasyEngineering.net


Downloaded From : www.EasyEngineering.net

Complex Integration 507

ZR
zeimz xeimx zeimz
Z Z
Now, dz = dx + dz.
z2 + a2 x2 + a2 S z2 + a2
C −R
Z ZR Z
imz imx
i.e., e f (z)dz = e f (x)dx + eimz f (z)dz. (1)
S
C −R

Y
Z
imz
Let us evaluate e f (z)dz. S
C

ww

>
The poles are given by z2 + a2 = 0. • ai

i.e., z2 = −a2 (−R, 0)


>
(R, 0)
X

w.E
• −ai
z = ±ai which are simple.
But z = ai is the only pole which lies
inside C.
asy
En
Z
∴ By Cauchy’s residue theorem, eimz f (z)dz = 2πi × R(ai)

gin
C

Now R(ai) = lim (z − ai)eimz f (z)

ee z→ai

= lim (z − ai)eimz
z→ai
ai e−ma
z
(z + ai)(z − ai)
rin
= eimai


Z
eimz f (z)dz = 2πi
e−ma
2ai
=
2
= πie−ma . g.n

Also | f (z)| =
C

1

→ 0 as |z| → ∞.
2

et
z2 + a2 Z
∴ By Jordan’s Lemma, eimz f (z)dz → 0 as R → ∞.
S
ZR Z
imx
Now, (1) =⇒ πie −ma
= e f (x)dx + eimz f (z)dz.
−R S
Taking limit as R → ∞ on both sides we get

Downloaded From : www.EasyEngineering.net


Downloaded From : www.EasyEngineering.net

508 Engineering Mathematics - II

ZR Z
imx
πie −ma
= lim e f (x)dx = lim eimz f (z)dz.
R→∞ R→∞
−R S
Z∞ Z∞ Z∞
x(cos mx + i sin mx) x cos mx x sin mx
= dx = dx + i dx
x2 + a2 x2 + a2 x2 + a2
−∞ −∞ −∞
Equating the imaginary parts on both sides, we get
Z∞
x sin mx
dx = πe−ma
x2 + a2
−∞

ww
Z∞ !
x sin mx −ma x sin mx
2 dx = πe [∵ 2 is even]
x2 + a2 x + a2
0

w.E ∴
Z∞

0
x sin mx
x2 + a2
dx =
πe−ma
2
.

Example 4.105. Evaluate asy Z∞


cos x
dx using contour integration.

En 0
(x2 + a2 )(x2 + b2 )
[Dec 2011]

gin eiz
Z
Solution. Let us consider the integral dz where C is the simple
(x2 + a2 )(x2 + b2 )
C

ee
closed curve consisting of the real axis from −R to R and the upper
semicircle S : |z| = R taken in the anticlockwise sense.

rin
Let f (z) =
(x2 +
1
a2 )(x2 + b2 )
. • bi g.n S

et
>

Z ZR Z • ai
iz ix iz
Now, e f (z)dz = e f (x)dx + e f (z)dz (1) >
(−R, 0) (R, 0)
• −ai
C Z −R S
Let us evaluate eiz f (z)dz. • −bi

C
The poles are given by (x2 + a2 )(x2 + b2 ) = 0.
i.e., (z + ai)(z − ai)(z + bi)(z − bi) = 0
⇒ z = ai, −ai, bi, −bi are the simple poles.
But only z = ai and z = bi lie inside C.

Downloaded From : www.EasyEngineering.net


Downloaded From : www.EasyEngineering.net

Complex Integration 509

Z
∴ By Cauchy’s residue theorem, eiaz f (z)dz = 2πi[R(ai) + R(bi)]
C

Now R(ai) = lim (z − ai)eiz f (z)


z→ai
eiz
= lim (z − ai)
z→ai (z + ai)(z − ai)(z + bi)(z − bi)
ei×ai e−a e−a
= = = −
2ai(ai − bi)(ai + bi) −2ai(a2 − b2 ) 2ai(a2 − b2 )
iz

ww R(bi) = lim (z − bi)


z→bi
e
e
(z + ai)(z − ai)(z + bi)(z − bi)
i×bi

w.E =

=
2bi(bi − ai)(bi + ai)
e−b
=−
e−b
=
e−b

asy
2bii(b − a)i(b + a) 2bi(b2 − a2 ) 2bi(a2 − b2 )
e−a e−b
Z " #
iz
∴ e f (z)dz = 2πi − +
2ai(a2 − b2 ) 2bi(a2 − b2 )
C
2πi
En " −a
e e−b
#

gin
= − +
2i(a2 − b2 ) a b
" −b −a
#
π e e
= 2 −

π
2
(a − b ) b
ee
" −b
e
a

e−a
# ZR Z
rin
Now, (1) =⇒ 2
(a − b2 ) b

a
=
−R
e ix
f (x)dx +
S
eiz f (z)dz
g.n (2)

et

1
Also | f (z)| = 2
→ 0 as |z| → ∞.

(x + a2 )(x2 Z+ b2 )
∴ By Jordan’s Lemma, eiz f (z)dz → 0 as R → ∞.
S
ZR " −b
e−a
#
π
Z
e
Now, (2) =⇒ eix f (x)dx + eiz f (z)dz = − . (3)
(a2 − b2 ) b a
−R S
Taking limit as R → ∞ on both sides we get
Z∞ " −b
e−a
#
ix π e
e f (x)dx = 2 −
(a − b2 ) b a
−∞

Downloaded From : www.EasyEngineering.net


Downloaded From : www.EasyEngineering.net

510 Engineering Mathematics - II

Z∞ " −b
e−a
#
cos bx + i sin bx π e
dx = 2 −
(x2 + a2 )(x2 + b2 ) (a − b2 ) b a
−∞
Z∞ Z∞ " −b
e−a
#
cos bx sin bx π e
dx + i dx = 2 −
(x + a2 )(x2 + b2 )
2 (x2 + a2 )(x2 + b2 ) (a − b2 ) b a
−∞ −∞
Equating the real parts on both sides, we get
Z∞ " −b
e−a
#
cos bx π e
dx = 2 −
(x2 + a2 )(x2 + b2 ) (a − b2 ) b a
−∞
Z∞

ww
" −b
e−a
# !
cos bx π e cos bx
2 dx = 2 − [∵ is even]
(x2 + a2 )(x2 + b2 ) (a − b2 ) b a (x2 + a2 )(x2 + b2 )
0

w.E ∴
Z∞

0
cos bx
(x2 + a2 )(x2 + b2 )
dx =
π
" −b
e
2(a2 − b2 ) b

e−a
a
#
.

cos mxasy Z∞
Example 4.106. Evaluate
0
x2 + a2
En
dx using contour integration. [Jun 2012]

gin
eimz
Z
Solution. Let us consider the integral dz, where C is the simple closed
z2 + a2
C

ee
curve consisting of the real axis from −R to R and the upper semi circle
S : |z| = R taken in the anticlockwise sense.
Let f (z) = 2
1
rin
g.n
z + a2
Z ZR Z
imz
Now e f (z)dz = eimx f (x)dx + eimz f (z)dz (1)
C

Let us evaluate
Z
imz
e
−R

f (z)dz.
s

Y
et
S
C
>

The poles are given by z2 + a2 = 0 • ai

(z + ai)(z − ai) = 0 > X


−R R
• −ai
i.e., z = ai, −ai
z = ai lies inside C.

Downloaded From : www.EasyEngineering.net


Downloaded From : www.EasyEngineering.net

Complex Integration 511

Z
By Cauchy’s residue theorem, eimz f (z)dz = 2πiR(ai).
C
z = ai is a simple pole.

∴ R(ai) = lim (z − ai)eimz f (z)


z→ai

✘ eimz e−am
−✘
(z ✘
= lim ✘ ai) ✘ = .
z→ai −✘
(z ✘
(z + ai)✘ ai) 2ai
2πie−am π
Z
∴ eimz f (z)dz = = e−am .
2ai a
C

ww Also | f (z)| = |
z2
1
+ a2
| → 0 as |z| → ∞.
Z

w.E ∴ By Jordon’s lemma,

π
Now (1)⇒ e−am =
ZR
imx
e
s
eimz f (z)dz → 0 as R → ∞.

f (x)dx +
Z
eimz f (z)dz.
a
asy
Taking limit as R → ∞ we get
−R
s

π −am
e = lim En ZR
e imx
f (x)dx + lim
Z
eimz f (z)dz
a R→∞

gin
Z∞
−R

eimx
R→∞ s

ee
π −am
e = dx + 0
a x2 + a2
−∞
Z∞
cos mx + i sin mx rin
g.n
= dx.
x2 + a2
−∞

Equating the real parts we get

−∞
Z∞
cos mx
x2 + a2
dx =
πe−am
a
et
Z∞
cos mx πe−am
2 dx =
x2 + a2 a
0
Z∞
emx πe−am
dx = .
x2 + a2 2a
0

Downloaded From : www.EasyEngineering.net


Downloaded From : www.EasyEngineering.net

512 Engineering Mathematics - II

Exercise 4 F

Evaluate the following integrals using contour integration

Z2π Z∞
1 1
1. dθ. [Jun 2008] 11. dx. [Jun 2011]
5 + 4 cos θ (x2 + 1)2
0 0

Z2π Z∞
1 1
2. dθ. [Apr 2011] 12. dx.
a + b cos θ (x4 + 1)

ww 0 0

Z2π Z∞
1 x2 − x + 2
3. dθ. [Jun 2008] 13. dx.

w.E 0


5 − 4 cos θ
−∞

Z∞
x4 + 10x2 + 9

asy
1
4. dθ. [Jun 2010] 14. dx.
a + b cos θ (1 + x2 )2
0 0

5.
Z2π
1
dθ. En 15.
Z∞
x2
dx.
0


(a + b cos θ)2

gin 0

Z∞
(x2 + 1)2

6.
0

Z2π
a
a2 + sin2 θ
dθ, a > 0.
ee 16.
0

Z∞
x2
(x2 + 9)(x2 + 4)2
dx.

rin
[Dec 2009]

7.
0
1 + cos 2θ
5 + 4 cos θ
dθ. [May 2005] 17.
−∞
x2 − x + 2
x4 + 10x2 + 9
dx.
g.n [Dec 2010]

8.

0
1 + cos 2θ
5 + 4 cos θ
dθ. [May 2003] 18.
Z∞

−∞
(x2
1
+ 1)(x2 + 4)
dx. et
Z2π Z∞
cos 2θ x sin x
9. dθ, |a| < 1. 19. dx.
1 − 2a cos θ + a2 x2 + a2
0 0

Zπ Z∞
1 cos ax
10. 2
dθ, a > 0. 20. dx.a > 0.
a2 + sin θ x2 + 1
0 0

Downloaded From : www.EasyEngineering.net


182
Downloaded Supplement to Engineering Mathematics - II
From : www.EasyEngineering.net

1
cos 2θ = 1 ⇒ r2 = cos 2θ.
r2

2.4 Unit IV Complex Integration

2.4.1 May/June 2016 [R 2013]


Part A
1
9. Expand f (z) = as a Taylor series about the point z = 2.
z2
1

ww Solution. Let f (z) = 2 .


z
z = 2 is a regular point.

w.E ∴ We can find Taylor’s series about z = 2.

a ∴ The series is

f (z) = f (2) + syE


f ′ (2)
(z − 2) +
f ′′ (2)
(z − 2)2 +
f ′′′ (2)
(z − 2)3 + · · ·
1!
ngi 2! 3!

Now, f (z) =
1
z2 nee f (2) =
1
4
−2
f ′ (z) = 3 rin f ′ (2) =
−1
z
6
f ′′ (z) = 4 f ′′ (2) =
4
3 g.n
′′′
f (z) = 5
z
−24
z
f ′′′ (2) =
−3
4
8
et
1 1 3 3
f (z) = − − (z − 2) + (z − 2)2 − (z − 2)3 − . . .
4 4 × 1! 8 × 2! 4 × 3!
1 1 3 1
f (z) = − − (z − 2) + (z − 2)2 − (z − 2)3 − . . ..
4 4 16 8
10. Express the residue of f (z) = tan z at its singularities.
sin z
Solution. f (z) = tan z = .
cos z
The singularities are given by cos z = 0.
⇒ z = (2n + 1) π2 , n ∈ Z.
All the poles are simple poles.

Downloaded From : www.EasyEngineering.net


———-
Anna University
Downloaded Solved Question Papers
From : www.EasyEngineering.net 183

g(z)
We know that if f (z) = where g(z) and h(z) are analytic at
h(z)

z = a and if h(a) = 0, h (a) , 0 and g(a) , 0 and finite then z = a
g(z)
is a simple pole of f (z) and R(a) = lim ′ .
z→a h (z)
Using the above result,
! we have !
  sin z sin z
R (2n + 1) π2 = lim π = lim π = −1.
z→(2n+1) 2 cos z z→(2n+1) 2 − sin z

Part B

15. (a)Z (i) Evaluate using Cauchy’s integral formula


(z + 1)
dz where C is the circle |z| = 2.

ww C
(z − 3)(z − 1)
Solution. The singular points are z = 3, 1.

w.E
Given C is |z| = 2.

a
z = 1 lie inside C and z = 3 lies outside C.


z+1 syE
=
z−3
z+1
=
f (z)
where f (z) =
z+1
.
Z
z+1
Z
f (z) ngi
(z − 3)(z − 1) (z − 1) (z − 1) z−3

(z − 3)(z − 1)
C
dz =
z−1
C
dz
nee
= 2πi × f (1)
rin
g.n
[By Cauchy’s integral formula].

Now, f (z) =
z+1
z−3
2
.
et
f (1) = = −1.
Z −2
z+1
∴ dz = 2πi(−1) = −2πi.
(z − 3)(z − 1)
C

Z2π

15. (a) (ii) Evaluate using contour integration.
13 + 12 sin θ
0
dz
Solution. Let z = eiθ , then dθ = .
! iz
1 1
Also, sin θ = z− .
2i z

Downloaded From : www.EasyEngineering.net


———-
184
Downloaded Supplement to Engineering Mathematics - II
From : www.EasyEngineering.net

As θ varies from 0 to 2π, z moves, on the unit circle |z| = 1.

Z 2π Z
1 1 dz
∴ dθ =  
0 13 + 12 sin θ 13 + 1 iz
z− z 12
C 2i
Z
i dz
=  
1 iz
13i + 6 z − z
C
Z
1
= dz
13iz + 6z2 − 6
C
Z
1
=   dz
13i
6 z2 + 6 z −1
ww =
C
1
Z
 
1
dz
w.E 6
1
C
Z
z+ 13 2
12 i
1
−1+ 169
144

a syE
=

1
6
C
Z

z+ 13 2
12 i

+
1
25
144
dz

=
6
ngi C

z+ 13 2
12 i

+
 2 dz
5
12

=
1
6
Z
 nee 1
2  2 dz

rin
13 5i
C
z+ 12 i − 12
Z
1 1
=
6

z+ 13i
12 + 5i
12

z+ 13i
12 g.n
− 5i
12
 dz

et
C
Z
1 1
=    dz
6 z+ 3i
z+ 2i
C 2 3

1
Here f (z) =   .
3i
z + 2 z + 2i3
−3i −2i
The poles are z = and z = , which are simple poles.
2 3
Now, C is |z| = 1.
−3i −3i 3i
When z = , |z| = | |= > 1.
2 2 2
−3i
∴ z= i lies outside C.
2
−2i −2i 2
When z = , |z| = = < 1.
3 3 3

Downloaded From : www.EasyEngineering.net


———-
Anna University
Downloaded Solved Question Papers
From : www.EasyEngineering.net 185

−2i
∴ z= lies inside C.
3 Z !
−2i
By Cauchy’s residue theorem, f (z)dz = 2πiR .
3
! ! C
−2i 2i 1
Now, R = lim z +   
3 z→ −2i
3
3 z + 2 z + 2i3
3i

1 6 6
= 2i 3i = = .
3 + 2
4i + 9i 13i
Z 2π !
1 1 −2i πi 6 2π
∴ dθ = 2πiR = × = .
0 13 + 12 sin θ 6 3 3 13i 13

ww
15. (b) (i) Expand
f (z) = 2
z
as a Laurent’s
in the regions
series the function

w.E
(z − 3z + 2)
(1) |z| < 1

a
(2) 1 < |z| < 2
(3) |z| > 2 .
z
syE z
Solution.
z
=
z2 − 3z + 2 (z − 1)(z − 2)
A B ngi
.

Let = +
(z − 1)(z − 2) (z − 1) (z − 2)
∴ z = A(z − 2) + B(z − 1). When z = 1 A = −1
nee
When z = 2 A=2 rin
∴ z
(z−1)(z−2) =
2
z−2
− 1
z−1 .
z
2
1
(1) When |z| < 1, < < 1.
2 g.n
Hence, f (z) = −
1
+
z−1 z−2
2 et
1 2
=−  +  
− 1−z −2 1 − 2z
 −1 2  z −1
= 1−z + 1−
−2 2
   z z2 z3 
= 1 + z + z2 + z3 + · · · − 1 + + + + · · ·
2 4 8
(2) Consider 1 < |z| < 2.
1
When 1 < |z| we have |z| > 1 ⇒ < 1.
|z|
z
When |z| < 2, < 1.
2

Downloaded From : www.EasyEngineering.net


———-
186
Downloaded Supplement to Engineering Mathematics - II
From : www.EasyEngineering.net

Hence, f (z) can be written as,


2 1
f (z) =  −  
z
−2 1 − 2 z 1 − 1z
 z −1 1  1 −1
=− 1− − 1−
2 z z
 z z2 z3  1 1 1 1 
= − 1 + + + + ··· − 1 + + 2 + 3 + ···
2 4 8 z z z z
 z z2 z3   
= − 1 + + + + · · · − z−1 + z−2 + z−3 + z−4 + · · · .
2 4 8

z 2
(3) Consider |z| > 2, then > 1 ⇒ < 1.


2 z
ww Also
1
|z| 2
1
< < 1.

w.E
f (z) = −
1
z−1 z−2
+
2

a =− 
z 1− z
1
syE
1
+ 
2
z 1 − 2z


1
=− 1−
z z ngi
1 −1 2 
+ 1−
z
2 −1
z
1 1 1 1
nee  2
= − 1 + + 2 + 3 + ··· + 1 + + 2 + 3 + ···
z z z z z
2 4
z z
8
z


1 1 1 1
rin
= − − 2 − 3 − 4 − ··· + + 2 + 3 + 4 ···
2 4 8 16
z z z z
= z−1 + 3z−2 + 7z−3 + 15z−4 + . . . .
z z z
g.n
z

Z∞
x sin mx
et
15. (b) (ii) Evaluate dx where a > 0, m > 0.
x2 + a2
0
Solution. Refer Example 5.104 on page 530 of the main text
book.

2.4.2 Dec.2015/Jan. 2016 [R 2013]


Part A

9. Define and give an example of essential singular points.


Solution. Refer page 474 of the main text book.

Downloaded From : www.EasyEngineering.net


———-
Anna University
Downloaded Solved Question Papers
From : www.EasyEngineering.net 187

Z2π

10. Express as complex integration.
2 cos θ + sin θ
0
dz
Solution. Let z = eiθ . Then dθ = .
! iz !
1 1 1 1
cos θ = z + , sin θ = z− .
2 z 2i z
As θ varies from 0 to 2π, z moves along the unit circle |z| = 1.
Z2π Z
1 1 dz
∴ dθ =  × where c is |z| = 1.
2 cos θ + sin θ 1 1
z + z + 2i z − z1 iz
0 c

Part B
ww
15. (a) (i) Using Cauchy’s integral formula evaluate
I
e2z
dz
w.E
where C is |z| = 2.
C
(z + 1)4

a syE
Solution. Given, C is |z| = 2.
The singular points are given by (z + 1)4 = 0 ⇒ i.e., z = −1.

ngi
z = −1 is a singular point which lies inside C.
By Cauchy’s integral formula.
nee
rin
where f (z) = e2z
Z
f (z)
dz f ′ (z) = 2e2z g.n
c (z + 1)

= 2πi
4

f ′′′ (−1)
3!
f ′′ (z) = 4e2z
′′′ 2z
et
f (z) = 8e
2πi 8
= × 2 8
6 e ⇒ f ′′′ (−1) = 8e−2 = .
8πi e2
= 2.
3e

Z∞
dx
15. (a) (ii) Evaluate using contour integration.
x4 + a4
0
1
Solution. Let f (x) = 4 .
Z ∞x + a4 Z
1dx 1 ∞ dx
Since f (x) is even = .
0 x4 + a4 2 −∞ x4 + a4

Downloaded From : www.EasyEngineering.net


———-
188
Downloaded Supplement to Engineering Mathematics - II
From : www.EasyEngineering.net
Z ∞
dx
We shall evaluate f (z) = .
Z ∞ −∞ x4 + a4
dx
Let I = 4 4
.
−∞ x + a
P(x)
The integrand is of the form where degree of Q(x) is 2
Q(x)
more than that of P(x)Zand Q(x) does not vanish for any real x.

dx
Consider the integral 4 4
where C is the simple closed
−∞ x + a
curve consisting of the real axis from −R to R and the upper
semi circleZS : |z| = R taken in the anticlockwise sense.

dz
Let f (z) = 4 + a4
.
z
ww −∞
The poles are given by z4 + a4 = 0.
i.e., z4 = −a4
w.E 1 1 π
z = (−a4 ) 4 = a(−1) 4 = aei(2n+1) 4 , n = 0, 1, 2, 3.

a π 3π

syE
5π 7π
∴ z = aei 4 , aei 4 , aei 4 , aei 4 .
Let the poles be z1 , z2 , z3 and z4 .
 π π
π
z1 = aei 4 = a cos + i sin
4
a a
= √ +i√ .
4 ! 2 2 ngi
i 3π
z2 = ae 4 = a cos

4
+ i sin

4
−a
= √ +i√ .
2
a
2 nee
rin
5π −a a
z3 = aei 4 = √ − i √ . g.n
i 7π
2
z4 = ae 4 = a cos

4
2
+ i sin

4
!
a a
= √ −i√ .
2 2
et
Only z1 and z2 lie inside C.
By
Z Cauchy’s residue theorem,
h 3π i
i π4
f (z)dz = 2πi R(e ) + R(ei 4 ) .
C

π
Now, R(ei ) = R(z1 ) = lim (z − z1 ) f (z)
4 z→z1
1
= lim (z − z1 )
z→z1 (z − z1 )(z − z2 )(z − z3 )(z − z4 )
1
=
(z1 − z2 )(z1 − z3 )(z1 − z4 )

Downloaded From : www.EasyEngineering.net


———-
Anna University
Downloaded Solved Question Papers
From : www.EasyEngineering.net 189

1
= √ √ √ √
a3 2( 2 + i 2)( 2i)
1 1
= √ √ √ = √
a 2 2(1 + i) 2i 2(i − 1)a3 2
3


R(ei ) = R(z2 ) = lim (z − z2 ) f (z)
4 z→z2
1
= lim (z − z2 )
z→z2 (z − z1 )(z − z2 )(z − z3 )(z − z4 )
1
=
(z2 − z1 )(z2 − z3 )(z2 − z4 )
1
= √ √ √ √

ww =
−a3 2(− 2 + i 2)( 2i)

1
= √
1

w.E a3 (−2)i 2(−1 + i) 2a3 2(1 + i)


Z
f (z)dz = 2πi
"

1
+ √
1
#

a

2a3 2(i − 1) 2a3 2(i + 1)

=
C
2πi syE
"
1+i+1−i
#

2a3 2
−π

−2
ngi
= √ .
a3 2 nee
∴⇒ (1)
ZR
f (x)dx +
Z
rin
f (z)dz = √
−π
(2)
−R S
3
a 2
g.n
Now, lim z f (z) = lim
z→∞ z→∞ z4
z
+Z1
→ 0.
et
∴ By Cauchy’s lemma, f (z)dz → ∞ as R → ∞.
S

ZR Z
π
∴ (2) =⇒ lim f (x)dx + lim f (z)dz = √
R→∞ R→∞ a3 2
−R S
Z∞
π
f (x)dx = √
a3 2
−∞
Z∞
π
∴ f (x)dx = √
2a3 2
0

Downloaded From : www.EasyEngineering.net


———-
190
Downloaded Supplement to Engineering Mathematics - II
From : www.EasyEngineering.net

Z∞
1 π
i.e., dx = √ .
x4 + 1 3
2a 2
0

z2 − 4z + 2
15. (b) (i) Obtain the Laurent’s expansion of f (z) =
z3 − 2z2 − 5z + 6
in 3 < |z + 2| < 5.
z2 − 4z + 2
Solution. Given, f (z) =
z3 − 2z2 − 5z + 6
z2 − 4z + 2
= .
(z − 1)(z − 3)(z + 2)
2
z − 4z + 2 A B C
Let = + + .

ww 2
(z − 1)(z − 3)(z + 2) z − 1 z − 3 z + 2
z − 4z + 2 = A(z − 3)(z + 2) + B(z − 1)(z + 2) + C(z − 1)(z − 3).

w.E
When z = 1
When z = 3
9 − 12 + 2 = B(2)(5)
When z = −2
4 + 8 + 2 = C(−3)(−5)
a
1 − 4 + 2 = A(−2)(3) syE B=
−1
10
. C=
14
15
.

A=
1
6
.
ngi
z2 − 4z + 2 1 1 1 1 nee
14 1
.
f (z) = = − +
rin
(z − 1)(z − 3)(z + 2) 6 z − 1 10 z − 3 15 z + 2
C is 3 < |z + 2| < 5.
Let z + 2 = t. g.n
C is 3 < |t| < 5.
∴ f (z) =
1 1

1 1
+
14 1
6 t − 3 10 t − 5 15 t
.
et
3
When 3 < |t| ⇒ < 1.
|t|
|t|
When |t| < 5 ⇒ < 1.
5
1 1 1 1 14 1
∴ f (z) =  +  +
6t 1− 3 50 1 − t 15 t
t 5
!−1
1 3 1  t −1 14
= 1− + 1− +
6t t 50 5 15t
! !
1 3 9 1 t t2 14
= 1 + + 2 + ··· + 1+ + + ··· +
6t t t 50 5 25 15t

Downloaded From : www.EasyEngineering.net


———-
Anna University
Downloaded Solved Question Papers
From : www.EasyEngineering.net 191

!
1 1 3 1 t t2 14
= + 2 + 3 + ··· + 1+ + + ··· +
6t 2t 2t 50 5 25 15t
!
7 1 3 1 t t2
= + 2 + 3 + ··· + 1+ + + ···
30t 2t 2t 50 5 25
7 1 3
∴ f (z) = + + + ···
30(z + 2) 2(z + 2)2 2(z + 2)3
!
1 (z + 2) (z + 2)2
+ 1+ + + ··· .
50 5 25
Z
z3 dz
15. (b) (ii) Evaluate where C is |z| = 2.5; using
(z − 1)4 (z− 2)(z − 3)
C

ww residue theorem.
Solution. Refer Example 5.80 on page 492 of the main text
book. w.E
a
2.4.3 April/May 2015 [R 2013]syE
Part A ngi
9. State Cauchy’s integral theorem. nee
rin
Solution. If f (z) is an analytic function within and on a simple
closed contour C taken in the positive sense and if a is any
interior point of C, then g.n
f (a) =
1
2πi
I
f (z)
z−a
dz.
et
C

!
1
10. Identify the type of singularity of function sin .
1−z
3 5
θ θ
Solution. we know that sin θ = θ − + − ···
! 3! 5!
1 1 1 1 1 1
sin = − + − ···
1 − z ! 1 − z 6 (1 − z)3 120 (1 − z)5
−3 −5
1 (1 − z) (1 − z)
sin = (1 − z)−1 − + − ···
1−z 6 120 !
1
Since the Laurent’s series expansion of sin contains
1−z

Downloaded From : www.EasyEngineering.net


———-
192
Downloaded Supplement to Engineering Mathematics - II
From : www.EasyEngineering.net

infinite number of negative powers of 1 − z, z = 1 is an


essential singularity.

Part B
Z
z2
15. (a) (i) Evaluate dz where C is |z| = 3.
(z − 1)2 (z + 2)
C
Solution. The singular points are given by (z − 1)2 (z + 2) = 0.
⇒ z = 1 and z = −2.
C is the circle |z| = 3
The center is (0, 0) and radius 3.

ww z = 1 and z = −2 both lie inside the circle |z| = 3.


Consider z = 1
Z w.E z2
Z z2
(z+2)

C a
(z − 1)2 (z + 2)
dz =

syE C
Z
(z − 1)2

f (z)
dz

z2
=
C
(z − 1)2
ngi
dz Where f (z) =
(z + 2)

= 2πi
f ′ (1)
1! nee
by Cauchy’s integral formula.

= 2πi
5
rin
f ′ (z) =
(z + 2) · 2z − z2

10πi
9
f ′ (1) =
g.n
(z + 2)2
(3)(2) − 1 5
= .
=

Consider z = −2
9 32 9
et
Z Z z2
z2 (z−1)2
dz = dz
(z − 1)2 (z + 2) (z − 2)
C C
Z
f (z) z2
= dz Where f (z) =
(z − 2) (z − 1)2
C

= 2πi f (2) by Cauchy’s integral formula.


4 4
= 2πi f (−2) =
9 9
8πi
= .
9

Downloaded From : www.EasyEngineering.net


———-
Anna University
Downloaded Solved Question Papers
From : www.EasyEngineering.net 193

Z
z2 10πi 8πi
∴ dz = +
(z − 1)2 (z + 2) 9 9
C
18πi
= = 2πi.
9

z2 − 1
15. (a) (ii) Find the Laurent’s series expansion of f (z) =
(z + 2) (z + 3)
valid in the regions |z| < 2 and 2 < |z| < 3.
z2 − 1
Solution. Let f (z) = .
(z + 2)(z + 3)
The singular points are z = −2 and z = −3.

ww Both the singular points lie outside |z| < 2.


∴ f (z) is analytic in the open disc |z| < 2.

w.E
Hence f (z) can be expanded as a power series about z = 0.
Splitting f (z) into partial fractions we get
a syEz2 − 1
= A+
B
+
C

ngi
(z + 2)(z + 3) z+2 z+3

nee
z2 − 1 = A(z + 2)(z + 3) + B(z + 3) + C(z + 2).

Put z = −2, we obtain B = 3.


rin
Put z = −3, we obtain C = −8
Equating the coeff. of z2 we get A = 1. g.n

z2 − 1
(z + 2)(z + 3)
=1+
3

z+2 z+3
8

Given 2 < |z| < 3. This region is annular about z = 0 and f (z) is
et
analytic in this region.
∴ f (z) has a Laurent’s series about z = 0.
z
|z| < 3 ⇒ < 1.
3
z 2
|z| > 2 ⇒ > 1 ⇒ < 1

2 z
3 8
Now, f (z) = 1 + −
z+2 z+3
3 8
=1+  −  
2
z 1+ z 3 1 + 3z

Downloaded From : www.EasyEngineering.net


———-
194
Downloaded Supplement to Engineering Mathematics - II
From : www.EasyEngineering.net

!−1
3 2 8 z −1
=1+ 1+ − 1+
z z 3 3
! !
3 2 4 8 8 z z3 z3
=1+ 1 − + 2 − 3 + ··· − 1− + − + ···
z z z z 3 3 9 27
!
−1 −2 −3 −4 8 z z2 z3
= 1 + 3(z − 2z + 4z − 8z + · · · ) − 1− + − + ··· .
3 3 9 27

Z∞
dx
15. (b) Evaluate 2 , (a > 0) using contour integration.
x2 + a2
0
Z∞
x2

ww Solution. Let I =
−∞
(x2 + a2 )2
dx.

w.E
The integrand is of the form
P(x)
Q(x)
where the degree of Q(x) is
atleast 2 more than that of P(x) and Q(x), does not vanish for
a
any real x.
Consider the integral
syE Z
z2
dz where C is the simple
C ngi
(z2 + a2 )2

nee
closed curve consisting of the real axis from −R to R and the
upper semi circle S : |z| = R taken in the anticlockwise
direction.
rin
g.n
et
Y

z2
Let f (z) = S

(z2 + a2 )2
>

z2 • ia
=
[(z + ia)(z − ia)]2 > X

z2 −R
• −ia
R

= .
(z + ia)2 (z − ia)2
Z ZR Z
Now f (z)dz = f (x)dx + f (z)dz (1)
s
C −R
Z
Let us evaluate f (z)dz.
C
The poles are given by (z + ia)2 (z − ia)2 = 0

Downloaded From : www.EasyEngineering.net


———-
Anna University
Downloaded Solved Question Papers
From : www.EasyEngineering.net 195

i.e., z = ai and − ai.


ai and −ai are poles of order 2. Only
Z ai lies inside C.
∴ By Cauchy’s residue theorem f (z)dz = 2πiR(ai).
1 d n o C
Now, R(ai) = lim (z − ai)2 f (z)
1! z→ai dz
( ) ( )
d z2 d z2
= lim (z − ai)2 = lim
z→ai dz (z + ai)2 (z − ai)2 z→ai dz (z + ai)2

(z + ai)2 · 2z − z2 2(z + ai)


= lim
z→ai (z + ai)4
(ai + ai)2 · 2ai − (ai)2 2(ai + ai)
=
ww (ai + ai)4
(2ai)2 · 2ai + a2 × 2(2ai)

w.E =
(2ai)4
(−4a × 2ai) + 4ia3
2

a =

=
16a4
syE
(−8a i) + 4a3 i
3

16a4
=
−4a3 i −i
= ngi

Z 16a4 4a
f (z)dz = 2πi × R(ai) = 2πi
 −i 
=
π
.
nee
C
4a 2a
rin
By Cauchy’s lemma,
Z
g.n
f (z)dz = 0 as R → ∞.

π
ZR
S

Z
et
∴ (1) ⇒ = f (x)dx + f (z)dz
2a s
−R
ZR Z
π
= lim f (x)dx + lim f (z)dz
2a R→∞ R→∞ s
−R
Z∞
π x2
= dx + 0
2a (x2 + a2 )2
−∞
Z∞
x2 π
dx = .
(x2 + a2 )2 2a
−∞

Downloaded From : www.EasyEngineering.net


———-
196
Downloaded Supplement to Engineering Mathematics - II
From : www.EasyEngineering.net

2.4.4 November/December 2014 [R 2013]


Part A
Z
zdz
9. Evaluate where C is the circle |z| = 1.
z−2
CZ Z
z f (z)
Solution. dz = dz where f (z) = z.
C z − 2 C −2
z
C is |z| = 1.
The singular point is z = 2, which lies outside C.
z
Hence, is analytic inside C.
z−2 Z
z
ww ∴ By Cauchy’s integral theorem
C z−2
dz = 0.

w.E
10. Find the residue of f (z) =
z2
(z − 2)(z + 1)2
Solution. z = 2 is a simple pole.
at z = 2.

a syE R(2) = lim(z − 2) f (z)


z→2

ngi −✘
(z ✘
= lim ✘ 2)
✘(z ✘ ✘
z2
− 2)(z + 1)2
z→2

= lim
z2 nee = .
4
z→2 (z + 1) 2

rin 9
Part B
Z
z+1 g.n
15. (a) (i) Evaluate
C
z2 + 2z + 4
using Cauchy’s integral formula.
et
dz where C is the circle |z + 1 + i| = 2

Solution. The singular points are given by

z2 + 2z + 4 = 0
z2 + 2z + 1 + 3 = 0
(z + 1)2 + 3 = 0
(z + 1)2 = −3

z + 1 = ± 3i

z = −1 ± 3i

Downloaded From : www.EasyEngineering.net


———-
Anna University
Downloaded Solved Question Papers
From : www.EasyEngineering.net 197

C is the circle |z + 1 + i| = 2
√ √
z = −1 + i 3 lies inside C where as z = −1 − i 3 lies out side C.
Z Z
z+1 z+1
dz = √ √ dz
z2 + 2z + 4 (z + 1 + i 3)(z + 1 − i 3)
C C
Z z+1√
z+1−i 3
= √ dz
(z + 1 + i 3)
C
Z
f (z) z+1
= √ dz Where √
(z + 1 + i 3) z+1−i 3
C

ww = 2πi f (−1 − i 3) By Cauchy’s integral formula.



−1 − i 3 + 1

w.E = 2πi ×

−i 3
√ √
−1 − i 3 + 1 − i 3

a = 2πi ×

syE −2i 3
√ = πi.

1
ngi
15. (a) (ii) Find the Laurent’s series expansion of f (z) =
(z − 1)(z − 2)

Solution. f (z) =
1
= nee
valid in the regions |z| > 2 and 0 < |z − 1| < 1.
(z − 1) − (z − 2)
=
1

1
.

1 1
(z − 1)(z − 2)
Since |z| > 2, < < 1 Also 1 >
2
rin
(z − 1)(z − 2)

2
< 1.
z−2 z−1

z 2
Hence, f (z) can be written as,
|z| |z|
g.n
∴ f (z) = 
1
z 1− z2
− 
1
z 1 − 1z
 et
!−1 !−1
1 2 1 1
= 1− − 1−
z z z z
! !
1 2 4 8 1 1 1 1
= 1 + + 2 + 3 + ··· − 1 + + 2 + 3 + ···
z z z z z z z z
! !
1 2 4 8 1 1 1 1
= 1 + + 2 + 3 + ··· − 1 + + 2 + 3 + ···
z z z z z z z z
!
1 1 3 7
= + + + ··· .
z z z2 z3
1 1
(ii) We have f (z) = − .
z−2 z−1

Downloaded From : www.EasyEngineering.net


———-
198
Downloaded Supplement to Engineering Mathematics - II
From : www.EasyEngineering.net

Given 0 < |z − 1| < 1.


Let t = z − 1 ⇒ z = t + 1.
∴ The region is 0 < |t| < 1.

1 1 1 1
f (z) = − = −
z−2 z−1 t+1−2 t+1−1
1 1
= −
t−1 t
1
= −t−1 −
1−t
= −t−1 − (1 − t)−1

ww = −(t)−1 − [1 + t + t2 + t3 + · · · ]
" #

w.E "
1
= − + 1 + t + t2 + t3 + · · ·
t
#

a =−
1
(z − 1)
syE
+ 1 + (z − 1) + (z − 1)2 + (z − 1)3 ) + · · · .

Z2π
dθ ngi
15. (b) (i) Evaluate
0
13 + 5 cos θ nee
using contour integration.

dz
Solution. Let z = eiθ , then dθ = .
! iz rin
Also, cos θ =
1 1
z+ . g.n
2 z
As θ varies from 0 to 2π, z moves along the unit circle
|z| = 1.
et
Z 2π Z
1 1 dz
∴ dθ =  
0 13 + 5 cos θ 5
13 + z + z 1 iz
C 2
Z
1 2 dz
= 2 
i 26 + 5 z z +1 z
C
Z
2 z✄ dz
=
i 26z + 5z2 + 5 z✄
C
Z
2 1
= dz
i 5(z + 26
2
5 z + 1)
C

Downloaded From : www.EasyEngineering.net


———-
Anna University
Downloaded Solved Question Papers
From : www.EasyEngineering.net 199

Z
2 1
=  2 dz
5i z+ 13
+1− 169
C 5 25
Z
2 1
=   dz
5i 13 2 25−169
C z+ 5 + 25
Z
2 1
=   dz
5i 13 2 144
C z+ 5 − 25
Z
2 1
=
5i  2  2 dz
13 12
C z+ 5 − 5
Z
2 1
=    dz

ww 5i 13 12 13 12
z+ 5 + 5 z+ 5 − 5
C
Z
2 1

w.E =
5i
C
Z

(z + 5) z + 15
 dz

a syE
=
2
5i
C
f (z)dz where f (z) =
1

(z + 5) z + 15


The singular points


1
! are given by
ngi
(z + 5) z +
5
−1
=0
nee
z = −5, z =
5
−1 rin
Since C is |z| = 1, z = −5 lies outside C and z =
−1
5
, lies inside C.
g.n
z=
5
is a simple pole.

By Cauchy’s residue theorem,


Z
f (z)dz = 2πiR
−1
5
!
.
et
! ! C
−1 1 1 1 1 5
R = lim z +   = lim = −1 = .
5 −1
z→ 5 5 (z + 5) z + 1 z→ 5 (z + 5)
−1
5 +5
24
5
Z 2π !
1 2 −1 4πi 5 π
∴ dθ = 2πiR = = .
0 13 + 5 cos θ 5i 5 5i 24 6

Z∞
x2 dx
15. (b) (ii) Evaluate using contour integration.
(x2 + 9)(x2 + 4)
0
x2
Solution. Let f (x) = .
(x2 + 9)(x2 + 4)

Downloaded From : www.EasyEngineering.net


———-
200
Downloaded Supplement to Engineering Mathematics - II
From : www.EasyEngineering.net

R∞ 1
R∞
Since f (x) is even, f (x)dx = 2 f (x)dx.
0 −∞
Z ∞
x2
Let I = 2 2
.
−∞ (x + 9)(x + 4)

The integrand is of the form


P(x) S
where degree of Q(x) is • bi
Q(x)

>
2 more than that of P(x) and • ai

Q(x) does not vanish for any >


(−R, 0) (R, 0)
• −ai
real x.
• −bi

ww Consider the integral


Z

(z2
z2
+ 9)(z2 + 4)
dz where C is the simple

w.E C
closed curve consisting of the real axis from −R to R and the

Z
a
upper semi circle S : |z| = R taken in the anticlockwise sense.
Z R syE Z
Now, f (z)dz =
C
f (x)dx+ f (z)dz
−R
ngi S

We shall evaluate
R
f (z)dz. nee (1)

C
rin
We have, f (z) =
(z2
z2
2
=
z2
+ 9)(z + 4) (z + 3i)(z − 3i)(z + 2i)(z − 2i) g.n
±3i and ±2i are the simple poles. et
But 3i and 2i only lie inside C.
∴ By Cauchy’s residue theorem
Z
f (z)dz = 2πi[R(3i) + R(2i)].
C
z2
R(3i) = lim (z − 3i)
z→3i (z + 3i)(z − 3i)(z2 + 4)
−9 3 3
= = = .
2 × 3i(4 − 9) 2i(9 − 4) 10i
2 −2
R(bi) = =
2i(4 − 9) 10i

Downloaded From : www.EasyEngineering.net


———-
Anna University
Downloaded Solved Question Papers
From : www.EasyEngineering.net 201

Z !
3 2
Now, f (z)dz = 2πi −
10i 10i
C
1 π
= 2πi = .
10i 5
Z R Z
π
∴ (1) =⇒ f (x)dx + f (z)dz = . (2)
−R S 5
z3
Also, lim z f (z) = lim → 0 as z → ∞
z→∞ z→∞ (z2
+ 9)(z2 + 4)
Z
∴ By Cauchy’s lemma, f (z)dz → 0 as R → ∞.

ww ZR
S

Z
π
w.E Now (2) =⇒ lim
R→∞
−R
f (x)dx + lim
R→∞
S
f (z)dz =
5
.

a syE lim
R→∞
Z R
f (x)dx =
π
5
Z −R

ngi f (x)dx =
π
5
Z−∞

nee
f (x)dx =
π
.
0
rin
10

2.4.5 May/June 2014 [R 2013] g.n


Part A
Z
et
z
9. Evaluate dz where C is (a) |z| = 1 (b) |z| = 3.
C z−2
Z Z
z f (z)
Solution. dz = dz where f (z) = z.
C z−2 C z−2
(a) C is |z| = 1.
The singular point is z = 2, which lies outside C.
z
Hence, is analytic inside C.
z−2 Z
z
∴ By Cauchy’s integral theorem dz = 0.
C z−2
(b) C is |z| = 3.
The singular point z = 2 lies inside C.

Downloaded From : www.EasyEngineering.net


———-
202
Downloaded Supplement to Engineering Mathematics - II
From : www.EasyEngineering.net

By Cauchy’s integral formula


Z Z
z f (z)
dz = dz where f (z) = z ⇒ f (2) = 2.
C z−2 C −2
z
= 2πi × f (2) = 2πi × 2 = 4πi.

10. State Cauchy’s residue theorem.


Solution. If f (z) is analytic inside and on a simple closed curve
C, except at a finite number of singular points z1 , z2 , . . . , zn lying
inside C, then

ww Z
f (z)dz = 2πi(R(z1 ) + R(z2 ) + · · · + R(zn ))

w.E C

where the integral over C is taken in the anticlockwise sense.

Part B
a syE
15. (a) (i) Obtain the ngi
Laurent’s series expansion of
f (z) =
z2 − 1
(z + 2)(z + 3)
in 2 < |z| < 3. nee
Solution. Let f (z) =
z2 − 1
. rin
(z + 2)(z + 3)
The singular points are z = −2 and z = −3.
g.n
Both the singular points lie outside |z| < 2.
∴ f (z) is analytic in the open disc |z| < 2.
et
Hence f (z) can be expanded as a power series about z = 0.
Splitting f (z) into partial fractions we get

z2 − 1 B C
= A+ +
(z + 2)(z + 3) z+2 z+3

z2 − 1 = A(z + 2)(z + 3) + B(z + 3) + C(z + 2).

Put z = −2, we obtain B = 3.


Put z = −3, we obtain C = −8
Equating the coeff. of z2 we get A = 1.

Downloaded From : www.EasyEngineering.net


———-
Anna University
Downloaded Solved Question Papers
From : www.EasyEngineering.net 203

z2 − 1 3 8
∴ =1+ −
(z + 2)(z + 3) z+2 z+3
Given 2 < |z| < 3. This region is annular about z = 0 and f (z) is
analytic in this region.
∴ f (z) has a Laurent’s series about z = 0.
z
|z| < 3 ⇒ < 1.
3
z 2
|z| > 2 ⇒ > 1 ⇒ < 1


2 z
3 8
Now, f (z) = 1 + −
z+2 z+3

ww =1+ 
3
z 1+ z2
− 
8
3 1 + 3z


=1+ w.E
3
z
1+
2
z
!−1
8
− 1+
3
z −1
3

=1+
3
z
a 2 4 8
1 − + 2 − 3 + ··· −
z z z
!
syE 8
3
z z3 z3
1− + −
3 9 27
+ ···
!

−1 −2 −3 −4
= 1 + 3(z − 2z + 4z − 8z + · · · ) −
8
3
1− + −ngi
z z2 z3
3 9 27
!
+ ··· .

nee
15. (a) (ii) Evaluate
Z2π

. rin
0
13 + 5 sin θ
dz g.n
Solution. Let z = eiθ , then dθ = .

Also, sin θ =
1
z− .
1
! iz et
2i z
As θ varies from 0 to 2π, z moves, on the unit circle
|z| = 1.
Z 2π Z
1 1 dz
∴ dθ =  
0 13 + 5 sin θ 13 + 5 1 iz
z− z
C 2i
Z
2i dz
=  
1 iz
26i + 5 z − z
C
Z
1
=2 dz
26iz + 5z2 − 5
C

Downloaded From : www.EasyEngineering.net


———-
204
Downloaded Supplement to Engineering Mathematics - II
From : www.EasyEngineering.net
Z
1
=2   dz
26i
5 z2 + 5 z −1
C
Z
2 1
=  2 dz
5 z+ 13 169
C 5 i −1+ 25
Z
2 1
=   dz
5 13 2 144
C
z+ 5 i + 25
Z
2 1
=
5  2  2 dz
13 12
C z+ 5 i + 5
Z
2 1
=     dz
13 2 i12 2

ww 2
5
C
Z
z+ 5 i −
1
5

w.E =
5
C
Z

z+ 13i
5 + 12i
5

z+ 13i
5 − 12i
5
 dz

a syE =
2
5
C
1

(z + 5i) z + 5i
 dz

Here f (z) =
1
 ngi
.
(z + 5i) z + 5i
−i
The poles are z = −5i and z = , which are simple poles. nee
Now, C is |z| = 1.
5
rin
When z = −5i, |z| = | − 5i| = 5 > 1. g.n
∴ z = −5i lies outside
−i
5
C.
−i
When z = , |z| = = < 1.
5
1
5
et
i
∴ z = lies inside C.
5 Z  −i 
By Cauchy’s residue theorem, f (z)dz = 2πiR .
5
C
 −i   i 1 1 5
Now, R = lim−i z +  = = .
5 z→ 5 5 (z + 5i) z + i −i + 5i 24i
5

Z 2π  −i  4πi 5
1 2 π
∴ dθ = 2πiR = = .
0 13 + 5 sin θ 5 5 5 24i 6

15. (b) (i) Using Cauchy’s residue theorem evaluate

Downloaded From : www.EasyEngineering.net


———-
Anna University
Downloaded Solved Question Papers
From : www.EasyEngineering.net 205

Z
(z − 1)
2
dz, where C is |z − i| = 2.
C (z + 1) (z − 2)
z−1
Solution. Let f (z) = .
(z + 1)2 (z − 2)
The poles are given by z = −1 and z = 2.
z = −1 is a pole of order 2 and z = 2 is a simple pole.
C is |z − i| = 2.
√ √
When z = −1, |z − i| = | − 1 − i| = 1 + 1 = 2 < 2.
⇒ z = −1 lies inside C.
√ √
When z = 2, |z − i| = |2 − i| = 4 + 1 = 5 > 2.

ww ∴ z = 2 lies outside C.
∴ By Cauchy’s residue theorem
Z
f (z)dz = 2πiR(−1).

w.E Since z = −1 is a pole of order 2,


C

R(−1) = a 1
lim
d 
syE
(2 − 1)! z→−1 dz
!
2

(z + 1) f (z) = lim
d
z→−1 dz
(z + 1) 2 z−1
(z + 1)2 (z − 2)
!

= lim
d z−1
z→−1 dz z − 2
= lim
z→−1 ngi
z − 2 − (z − 1) −3 + 2 −1
(z − 2)2
=
9
=
9
.

Z
z−1 nee −1
!
2πi
Hence,
C
2
(z + 1) (z − 2)
rin
dz = 2πiR(−1) = 2πi
9
=−
9
.

Z∞ g.n
15. (b) (ii) Evaluate by using contour integration

1
0
dx
(1 + x2 )2
.
et
Solution. Let f (x) =
(1 + x2 )2
Z∞ Z∞
1
Since f (x) is even, f (x)dx = f (x)dx.
2
0 −∞
Z∞
1
Let I = dx.
(1 + x2 )2
−∞
P(x)
The integrand is of the form , where degree of Q(x) is
Q(x)
atleast 2 more than that of P(x) and
Z Q(x) does not vanish for
1
any real x. Consider the integral dz where C is the
C (1 + z2 )2

Downloaded From : www.EasyEngineering.net


———-
206
Downloaded Supplement to Engineering Mathematics - II
From : www.EasyEngineering.net

simple closed curve consisting of the real axis from −R to R


and the upper semi circle S : |z| = R taken in the anticlockwise
sense.
Z ZR Z
Now, f (z)dz = f (x)dx + f (z)dz. (1)
C −R S

Z
We shall evaluate f (z)dz.
C
1 1 1
We have f (z) = = = .
(z2 + 1)2 [(z + i)(z − i)]2 (z + i)2 (z − i)2

ww The poles are at i and −i which

w.E
are of order 2. Only i will lie

>
inside C.
By a syE
Z Cauchy’s residue theorem (−R, 0)
• i

>
(R, 0)

ngi
• −i
f (z)dz = 2πiR(i).
C

R(i) =
1 d
lim (z − i)2 f (z)
nee
1! z→i dz
d 1 rin
= lim (z − i)2
z→i dz (z + i)2 (z − i)2
g.n
2
d
= lim (z + i)−2 = lim −2(z + i)−3
z→i dz
2
z→i
1
et
=− =− = .
(2i)3 −8i 4i
Z
1 π
∴ f (z)dz = 2πi × = .
C 4i 2
z z 1
Also lim z f (z) = lim 2 = lim   = lim   = 0.
z→∞ z→∞ (z + 1)2 z→∞ 4 1 2 z→∞ 3 1 2
z 1+ z2
z 1+ z2

Z
∴ By Cauchy’s lemma, f (z)dz = 0.
S
R RR R
Now (1)⇒ C
f (z)dz = f (x)dx + f (z)dz.
−R S

Downloaded From : www.EasyEngineering.net


———-
Anna University
Downloaded Solved Question Papers
From : www.EasyEngineering.net 207

Taking limits as R → ∞ we get

Z Z∞ Z
f (z)dz = f (x)dx + lim f (z)dz
C z→∞ S
−∞
Z∞
π 1
= dx + 0
2 (1 + x2 )2
−∞
Z∞ Z∞
π 1 1 π
=2 dx ⇒ dx = .
2 (1 + x2 )2 (1 + x2 )2 4
0 0

ww
2.4.6 November/December 2013[R 2008]

Part A
w.E
9. Evaluate
a Z
z+4
z2 + 2z
syE 1 1
dz, where C is |z − | = .

Solution.
C
ngi
2 3

nee
1 1
C is the circle |z − | = .
2 3 rin Y

The singular points are


g.n C

z = 0 and z = −2, which lie


completely outside C.
z+4

z = −2 z=0
1/3

1/2
X
et
∴ 2 is analytic everywhere
z + 2z
inside C.

Z Z
z+4 z+4
2
dz = dz = 0.
z + 2z z(z + 2)
C C

1 − e2z
10. Find the residue of at z = 0.
z4
2z
1−e
Solution. Let f (z) = .
z4

Downloaded From : www.EasyEngineering.net


———-
208
Downloaded Supplement to Engineering Mathematics - II
From : www.EasyEngineering.net

z = 0 is a pole of order 3. [Expand e2z and simplify]

1 d2
∴ R(0) = lim 2 (z − 0)3 f (z)
(3 − 1)! z→0 dz
" #
1 d2 3 1 − e2z
= lim z
2 z→0 dz2 z4
2
" #
1 d 1 − e2z
= lim 2
2 z→0 dz z
 4z2 8z3

1 2 
d  1 − 1 − 2z − 2! − 3! − · · · 
= lim  
2 z→0 dz2  z
" #
1 d 8z

ww =

1
lim
2 z→0 dz
"
−8
2−
3
− ···
#
1 −8
!
4
w.E = lim
2 z→0 3
+ powers of z =
2 3
=− .
3

Part B
a syE z2
15. (a) (i) Find the residues of f (z) =
ngi
(z + 2)(z − 1)2
singularities using Laurent’s series expansion.
at its isolated

z2 A B neeC
Solution.
(z + 2)(z − 1) 2
= + +
rin
z + 2 z − 1 (z − 1)2

g.n
z2 = A(z − 1)2 + B(z − 1)(z + 2) + c(z + 2).

When z = 1, 3C = 1 ⇒ C =
1
3
When z = −2, 9A = 4 ⇒ A = .
.
4
et
9

Equating the coefficients of z2 we get

A+B=1
4 5
B=1−A=1− = .
9 9
z2 4 1 5 1 1 1
∴ = + + .
(z + 2)(z − 1)2 9 z + 2 9 z − 1 3 (z − 1)2
1
The residues at the singularity z = −2 is the coefficient of in
z

Downloaded From : www.EasyEngineering.net


———-
Anna University
Downloaded Solved Question Papers
From : www.EasyEngineering.net 209

4 1
the expansion of .
9z+2
!−1
4 1 4 1 41 2
Now, · = ·  = 1+
9 z+2 9 z 1+ 2 9z z
z
" # " #
41 2 4 8 4 1 2 4
= 1 − + 2 − 3 ··· = − 2 + 3 ···
9z z z z 9 z z z
1 4
∴ Residue at z = −2 = Coefficient of = .
z 9
1
The residue at the singularity z = 1 is the coefficient of in the
z
5 1 1 1
ww expansion of +
9 z − 1 3 (z − 1)2
.

w.E
Now,
5 1
+
1 1
= 
9 z − 1 3 (z − 1)2 9z 1 − 1
5
z
+ 
1
3z2 1 − 1z
2

= a5
9z
"
1−
1
z
!−1
syE1
+ 2 1−
3z
1
z
!−2

# " #
=
5
9z
1 1 1
ngi 1 2 3
1 + + 2 + 3 + ··· + 2 1 + + 2 + ··· .
z z z 3z z z
nee
1 5
Residues at z = 1 = Coefficient of = .
z 9
rin
15. (a) (ii) Evaluate
Z2π
cos 2θ
dθ using contour integration. g.n
0
5 + 4 cos θ


Solution. Let z = e , then dθ =
dz
and cos θ =
1 1
z+ .
! et
iz 2 z
As θ varies from 0 to 2π, z moves along |z| = 1.
∴ C is |z| = 1.
Now z2 = ei2θ = cos 2θ + i sin 2θ.
∴ cos 2θ = R.P.ei2θ .
Z2π Z
cos 2θ z2 dz
∴ dθ = R.P  
5 + 4 cos 2θ 1
5+4× z+ z 1 iz
0 C 2
!Z
1 z
= R.P  2  dz
i 5 + 2 z z+1
C

Downloaded From : www.EasyEngineering.net


———-
210
Downloaded Supplement to Engineering Mathematics - II
From : www.EasyEngineering.net

Z
z2
= R.P(−i) dz
2z2 + 5z + 2
C
 −i  Z z2
= R.P dz
2 z2 + 52 z + 1
C
 −i  Z z2
= R.P  2 dz
2 z + 54 + 1 − 25
C 16
 −i  Z z2
= R.P  2 dz
2 z + 54 − 9
C 16
 −i  Z z2

ww = R.P
2
 −i  Z
C
 2  2 dz
z + 54 − 43

w.E = R.P
2

z+ 5
4
z2

− 43 z + 5
4 + 3
4
 dz

a
C
 −i  Z
syE
= R.P
2

z2

z + 12 (z + 2)
dz

= R.P ngi
 −i  Z
C

f (z)dz where f (z) = 


z2

2
C
nee 1
z + (z + 2)
2

rin 1
The singular points are z = −
Now C is |z| = 1.
2
and z = −2.
g.n
1 1 1
When z = − , |z| = | − | = < 1.
2
∴ z = − 21 lies inside C.
2 2
et
When z = −2, |z| = | − 2| = 2 > 1.
∴ z = −2 lies outside C.
1
z = − is a simple pole.
2

! ! !
1 1 1 z2
R − = lim z + f (z) = lim z +  
2 z→− 12 2 z→− 21 2 z + 1 (z + 2)
2
1 1
z2 4 4 1 2 1
= lim = = = × = .
z→− 12 z + 2 − 12 + 2 3
2
4 3 6

Downloaded From : www.EasyEngineering.net


———-
Anna University
Downloaded Solved Question Papers
From : www.EasyEngineering.net 211

By Cauchy’s residue theorem


Z !
1 1 πi
f (z)dz = 2πi × R − = 2πi × = .
2 6 3
C
Z2π  i   πi  π π
cos 2θ
∴ dθ = R.P − × = R.P = .
5 + 4 cos θ 2 3 6 6
0

Z∞
x2 − x + 2 5π
15. (b) (i) Show that dx = .
x4 + 10x2 + 9 12
−∞
Z∞
ww Solution. Let I =
x2 − x + 2
x4 + 10x2 + 9
dx.

w.E
The integrand is of the form
−∞
P(x)
Q(x)
where, the degree of Q(x) is

a
atleast 2 more than that of P(x) and Q(x) does not vanish for
any real x. syE Z
z2 − z + 2
Consider the integral
ngi
z4 + 10z2 + 9
dz, where C is the simple

nee
C
closed curve consisting of the real axis from −R to R and the
upper semi circle
S : |z| = R, taken in the anticlockwise sense. rin
Let f (z) = 4
z2 − z + 2
. g.n
et
z + 10z 2
Z Z +9 Z R
Now f (z)dz = f (x)dx + f (z)dz. (1)
−R s
C Z
Let us evaluate f (z)dz.
C
The poles are given by Y

z4 + 10z2 + 9 = 0
• 3i
(z2 + 1)(z2 + 9) = 0
>

• i

2 2
z + 1 = 0, z +9=0 > X
−R R
• −i
z2 = −1, z2 = −9
• −3i

z = ±i, z = ±3i.

Downloaded From : www.EasyEngineering.net


———-
212
Downloaded Supplement to Engineering Mathematics - II
From : www.EasyEngineering.net

z = i and 3i lies inside C.


By
Z Cauchy’s residue theorem,
f (z)dz = 2πi[R(i) + R(3i)]
C

R(i) = lim(z − i) f (z)


z→i
z2 − z + 2
= lim(z − i)
z→i (z + i)(z − i)(z2 + 9)
−1 − i + 2 1 − i
= = .
2i(−1 + 9) 16i

ww R(3i) = lim (z − 3i) f (z)


z→3i


✘ z2 − z + 2
w.E = lim ✘
z→3i
(z ✘
− 3i)
−9 − 3i + 2 −7 − 3i
(z ✘
+ 1)✘ ✘ + 3i)

(z2
− 3i)(z
7 + 3i 7 + 3i


Z a
=
(−9 + 1)6i

f (z)dz = 2πi
"
=

syE
1 − i 7 + 3i
+
−48i
#
=−
−48i
=
48i
.

C
"
16i 48i
# ngi
= 2πi
3 − 3i + 7 + 3i
48i
= 2π ×
10 5π
=
48 12
.
nee
By Cauchy’s lemma
Z
f (z)dz = 0 as R → ∞. rin
S
g.n
∴ (1) ⇒

12
=
ZR

−R
f (x)dx +
Z

s
f (z)dz et
Taking lim as R → ∞ we get
ZR Z

= lim f (x)dx + lim f (z)dz
12 R→∞ R→∞
−R s
Z∞

= f (x)dx + 0
12
−∞
Z∞
x2 − x + 2 5π
∴ dx = .
x4 + 10x2 + 9 12
−∞

Downloaded From : www.EasyEngineering.net


———-
Anna University
Downloaded Solved Question Papers
From : www.EasyEngineering.net 213

Z
z+1
15. (b) (ii) Evaluate dz where C is |z + 1 + i| = 2 using
(z2 + 2z + 4)2
C
Cauchy’s integral formula.

Solution. z2 + 2z + 4 = (z + 1)2 + 4 − 1
√ √
= (z + 1)2 + 3 = (z + 1 + 3i)(z + 1 − 3i).
√ √
The singular points are −1 − 3i and −1 + 3i.
√ √
When z = −1 − 3i, |z + 1 + i| = | − 1 − 3i + 1 + i|
√ √
= |i(1 − 3)| = | 3 − 1| < 2.
ww ∴ z = −1 −

3i lies inside C.

w.E √ √
When z = −1 + 3i, |z + 1 + i| = | − 1 + 3i + 1 + i|
a √
syE √
= |i( 3 + 1)| = 3 + 1 > 2.

ngi
∴ z = −1 + 3i lies out side C.


(z2
z+1
+ 2z + 4) 2
= √
z+1
nee

z+1
√ rin
[(z + 1 3i)(z + 1) − 3i]2

=
(z+1− 3i)2

(z + 1 + 3i)2 g.n
Z
=
f (z)

(z + 1 + 3i)
Z
2
where f (z) =
z+1



(z + 1 − 3i)2
. et
z+1 f (z)
∴ 2 2
dz = √ dz = 2πi × f ′ (−1 − 3i)
(z + 2z + 4) (z + 1 + 3i)2
[By Cauchy’s integral formula]

z+1
Now, f (z) = √
(z + 1 − 3i)2
√ √
(z + 1 − 3i)2 − (z + 1) · 2(z + 1 − 3i)
f ′ (z) = √
(z + 1 − 3i)4
√ √ 2 √ √ √

√ (−1 − 3i − 3i) − (−1 − 3i + 1) · 2(−1 − 3i + 1 − 3i)
f (−1 − 3i) = √ √
(−1 − 3i + 1 − 3i)4

Downloaded From : www.EasyEngineering.net


———-
214
Downloaded Supplement to Engineering Mathematics - II
From : www.EasyEngineering.net

√ √ √
(−2 3i)2 − 2(− 3i)(−2 3i)
= √
(−2 3i)4
4 × 3 × (−1) − 4 × 3(−1) −12 + 12
= = = 0.
16 × 9 144
Z
z+1
∴ dz = 2πi × 0 = 0.
(z + 2z + 4)2
2
C

2.4.7 May/June 2013 [R 2008]


Part A
Z
3z2 + 7z + 1
ww9. Evaluate
C z+1
dz where C is |z| = .
Solution. The singular point is z = −1.
1
2

w.E 1
Since C is |z| = , z = −1 lies outside C.
2
∴ a
3z2 + 7z + 1
z+1
syE
is analytic inside C.
Z
3z2 + 7z + 1
∴ By Cauchy’s theorem,
C
ngi
z+1
dz = 0.

10. Find the residue of


1 − e2z
z4
at z = 0. nee
Solution. Let f (z) =
1−e
z4
2z
. rin
z = 0 is a pole of order 3. [Expand e2z and simplify]
g.n
∴ R(0) =
1
(3 − 1)! z→0 dz
"
d2
lim 2 (z − 0)3 f (z)
#
et
1 d2 1 − e2z
= lim 2 z3
2 z→0 dz z4
" #
1 d2 1 − e2z
= lim
2 z→0 dz2 z
 4z2 8z3

1 d2  1 − 1 − 2z − 2! − 3! − · · · 
= lim  
2 z→0 dz2  z
" #
1 d 8z
= lim 2− − ···
2 z→0 dz 3
" # !
1 −8 1 −8 4
= lim + powers of z = =− .
2 z→0 3 2 3 3

Downloaded From : www.EasyEngineering.net


———-
Anna University
Downloaded Solved Question Papers
From : www.EasyEngineering.net 215

Part B

z2 − 1
15. (a) (i) Expand the function f (z) = in Laurent’s series
z2 + 5z + 6
for |z| > 3.
z2 − 1 z2 − 1
Solution. Let f (z) = 2 = .
z + 5z + 6 (z + 2)(z
+ 3)
z 3 1 1
|z| > 3 ⇒ > 1 ⇒ < 1 ⇒ <
3 z z 3
2 2
< <1
|z| 3

ww f (z) = 1 +
3

8

w.E=1+
z+2 z+3

3
− 
8


a
2
z 1+ z z 1 + 3z

=1+
3
1+ syE
2
!−1

8
1+
3
!−1

=1+
z
3
z
2 4
z
ngi
8
1 − + 2 − 3 + ··· −
z
!
8 3 9 27
1 − + 2 − 3 + ···
!

z z z
3 6 12 24
z
nee
2 z z
8 24 72
z

=1+ − +
2 z2 z3 z z z
rin
− 4 + ··· − + 2 − 3 + ···
z
= 1 − 5z−1 + 16z−2 − 60z−3 · · · .
g.n
Z
et
sin πz2 + cos πz2
15. (a) (ii) Evaluate dz where C is |z| = 3.
(z − 1)(z − 2)
C
Solution. z = 1 and z = 2 are singular points.
The circle is |z| = 3.
When z = 1, |z| = |1| < 3.
∴ z = 1 lies inside the circle.
When z = 2, |z| = |2| < 3.
∴ z = 2 also lies inside the circle.
1 (z − 1) − (z − 2) 1 1
Let = = − .
(z − 1)(z − 2) (z − 1)(z − 2) z−2 z−1

Downloaded From : www.EasyEngineering.net


———-
216
Downloaded Supplement to Engineering Mathematics - II
From : www.EasyEngineering.net

Z
sin πz2 + cos πz2
Now I = dz
(z − 1)(z − 2)
C
Z Z
sin πz2 + cos πz2 sin πz2 + cos πz2
= dz − dz
z−2 z−1
C C
Z Z
f (z) f (z)
= dz − dz
z−2 z−1
C C
I = 2πi f (2) − 2πi f (1) (1)
where f (z) = sin πz2 + cos πz2 .
When a = 2. f (2) = sin 4π + cos 4π = 1.

ww When a = 1, f (1) = sin π + cos π = −1.


(1) =⇒ I = 2πi f (2) − 2πi f (1) = 2πi.1 − 2πi(−1) = 4πi.

w.E Z ∞

a
15. (b) (i) Evaluate 2
x2 dx

syE
2
0 (x + a )(x + b )

Solution. Let f (x) = 2


2

x2
2
a > 0, b > 0.

.
2
R∞ ngi
(x + a )(x2 + b2 )
R∞
Since f (x) is even, f (x)dx = 21
Z ∞ 0
x2
−∞
nee
f (x)dx.

Let I = 2 2 2
−∞ (x + a )(x + b )
2
.
rin
g.n
The integrand is of the form
P(x)
Q(x)
where degree of Q(x) is • ai
S et
>

2 more than that of P(x) and • bi

Q(x) does not vanish for any >


(−R, 0) (R, 0)
• −bi
real x.
• −ai

Z
z2
Consider the integral dz where C is the
(z2 + a2 )(z2 + b2 )
C
simple closed curve consisting of the real axis from −R to R
and the upper semi circle S : |z| = R taken in the anticlockwise

Downloaded From : www.EasyEngineering.net


———-
Anna University
Downloaded Solved Question Papers
From : www.EasyEngineering.net 217

sense.
Z Z R Z
Now, f (z)dz = f (x)dx+ f (z)dz
−R S
C
(1)
R
We shall evaluate f (z)dz.
C

z2 z2
We have, f (z) = =
(z2 + a2 )(z2 + b2 ) (z + ai)(z − ai)(z + bi)(z − bi)

±ai and ±bi are the simple poles.


ww But ai and bi only lie inside C.

Z
w.E ∴ By Cauchy’s residue theorem

C
a syE
f (z)dz = 2πi[R(ai) + R(bi)].

R(ai) = lim (z − ai)


z→ai
z2
ngi
(z + ai)(z − ai)(z2 + b2 )

=
−a
2
2

2
=
2ai(b − a ) 2i(a − b2 )2
a
. nee
b rin
R(bi) =
Z
2i(b2 − a2 )
a b
! g.n
Now,

2πi
C
f (z)dz = 2πi

a−b
!
π
+
2i(a2 − b2 ) 2i(b2 − a2 ) et
= = .
2i (a − b)(a + b) a+b

Z R Z
π
∴ (1) =⇒ f (x)dx + f (z)dz = . (2)
−R S a+b

z3
Also, lim z f (z) = lim → 0 as z → ∞
z→∞ z→∞ (z2 + a2 )(z2 + b2 )

Z
∴ By Cauchy’s lemma, f (z)dz → 0 as R → ∞.
S

Downloaded From : www.EasyEngineering.net


———-
218
Downloaded Supplement to Engineering Mathematics - II
From : www.EasyEngineering.net

ZR Z
π
Now (2) =⇒ lim f (x)dx + lim f (z)dz = .
R→∞ R→∞ a+b
−R S

Z R
π
lim f (x)dx =
R→∞ a+b
Z −R

π
f (x)dx =
a+b
Z−∞

π
f (x)dx = .
0 2(a + b)
Z 2π
cos 3θ
ww
15. (b) (ii) Evaluate
0 5 − 4 cos θ
dθ using contour integration.
dz 1 1
!

w.E

Solution. Let z = e , then dθ = and cos θ = z+ .
iz 2 z
As θ varies from 0 to 2π, z moves along |z| = 1.

a syE
∴ C is |z| = 1.
Now z3 = ei3θ = cos 3θ + i sin 3θ.
∴ cos 3θ = R.P of z3 .
ngi

Z
cos 3θ
5 − 4 cos θ
dθ = R.P
Z
1
z3
 
dz
1 iz
nee
= R.P
Z
z2
2 i
5 − 2z − z
dz

C C
Z
5 − 42 z + z
z3 rinC
!Z
z3
= R.P
1
i 2
5z − 2z − 2
dz = R.P
−1
i 2
g.n
2z − 5z + 2
dz

= R.P
C
−1
i
!Z

z3
2 z2 − 52 z + 1
 dz
C

et
C
 i Z z3
= R.P  2
dz
2 z − 54 + 1 − 16 25
C
 i Z z3
= R.P    2 dz
2 5 2 3
C z − 4 − 4
 i Z z3
= R.P    dz
2 z − 4 + 4 z − 54 − 43
5 3
C
 i Z z3
= R.P   dz
2 z − 12 (z − 2)
C

Downloaded From : www.EasyEngineering.net


———-
Anna University
Downloaded Solved Question Papers
From : www.EasyEngineering.net 219

 i Z z3
= R.P f (z)dz where f (z) =   .
2 z − 12 (z − 2)
C

1
Singular points are and 2, which are simple poles.
2
Now, C is |z| = 1.
1 1 1
When z = , |z| = < 1 =⇒ z = lies inside C.
2 2 2
When z = 2, |z| = 2 > 1 =⇒ z = 2Zlies outside C. !
1
By Cauchy’s residue theorem, f (z)dz = 2πiR .
2
C
! !

ww Now R
1
2
= lim z −
z→ 12
1
2
f (z)

w.E R
1
2
!
= lim z −
z→ 12
1
!

z3

2 z − 1 (z − 2)
= −3
1
8
=−
12
8 3
1
=− .
12


Z 2π
a cos 3θ
5 − 4 cos θ
syE
dθ = R.P
 i
2

2πiR
1
2
!
2

= −π
−1
12
!
=
2

π
12
.

ngi
0

2.4.8 November/December 2012 [R 2008]


nee
Part A
Z rin
9. Evaluate
z
C (z − 1)(z − 2)
dz where C is the circle |z| = .
1
2 g.n
Solution. The singular points are z = 1 and z = 2.
1
Since C is |z| = , z = 1 and z = 2 lie outside C.
z
2
et
∴ is analytic inside C.
(z − 1)(z − 2) Z
z
∴ By Cauchy’s theorem, dz = 0.
C (z − 1)(z − 2)
1 h i
10. If f (z) = − − 2 1 + (z − 1) + (z − 1)2 + · · · find the residue of
z−1
f (z) at z = 1.
Solution. In the Laurent’s series expansion,
1
residue at 1 = Coefficient of = −1.
z−1
Part B

Downloaded From : www.EasyEngineering.net


———-
220
Downloaded Supplement to Engineering Mathematics - II
From : www.EasyEngineering.net
Z
z+4
15. (a) (i) Evaluate dz where C is the circle (i) |z+1+i| = 2
z2 + 2z + 5
C
(ii) |z + 1 − i| = 2 (iii)Z |z + 1| = 1.
z+4
Solution. Let I = 2
dz.
z + 2z + 5
C
The singular points are given by z2 + 2z + 5 = 0.
(z + 1)2 + 5 − 1 = 0
(z + 1)2 + 4 = 0
(z + 1)2 = −4
z + 1 = ±2i

ww z = −1 ± 2i.

w.E (i) The circle is |z + 1 + i| = 2.


When z = −1 − 2i, |z + 1 + i| = | − 1 − 2i + 1 + i| = | − i| = 1 < 2.

a ∴ −1 − 2i is a point inside the circle.


syE
When z = −1 + 2i, |z + 1 + i| = | − 1 + 2i + 1 + i| = |3i| = 3 > 2.
∴ z = Z−1 + 2i lies outside
z+4 ngi
Z the circle.
z+4
Now,

Z
C
z2 + 2z + 5
dz =

Z
C nee
(z + 1 − 2i)(z + 1 + 2i)
dz

=
z+4
z+1−2i
dz =
f (z)
dz. rin
C
z + 1 + 2i
z+4
z + 1 + 2i
C
g.n
Where f (z) =
z + 1 − 2i
.
By Cauchy’s integral formula et
Z
1 f (z)
f (−1 − 2i) = dz
2πi z + 1 + 2i
C
Z
−1 − 2i + 4 1 f (z)
= dz
−1 − 2i + 1 − 2i 2πi z + 1 − 2i
C
Z
3 − 2i 1 z+4
= dz
−4i 2πi z2 + 2z + 5
C
Z
π z+4
− (3 − 2i) = 2
dz
2 z + 2z + 5
C

Downloaded From : www.EasyEngineering.net


———-
Anna University
Downloaded Solved Question Papers
From : www.EasyEngineering.net 221

Z
z+4 π
dz = (2i − 3).
z2 + 2z + 5 2
C

(ii) The circle is |z + 1 − i| = 2.


When z = −1−2i, |z+1−i| = |−1−2i+1−i| = |−3i| = 3 > 2.
∴ z = −1 − 2i lies outside the circle.
When z = −1 + 2i, |z + 1 − i| = | − 1 + 2i + 1 − i| = |i| = 1 < 2.
∴ z = −1 + 2i lies inside the circle.
Z Z z+4 Z
z+4 z+1+2i f (z)
Now dz = dz = dz,
z2 + 2z + 5 z + 1 − 2i z + 1 − 2i
C C C

ww z+4

w.E where, f (z) =


z + 1 + 2i
, a = −1 + 2i.

a syE
f (a) =
−1 + 2i + 4
−1 + 2i + 1 + 2i
=
3 + 2i
4i
.

ngi
By Cauchy’s integral formula
Z
f (z)
z−a
dz = 2πi f (a) = 2πi nee
3 + 2i 3 + 2i
4i
=
2
π.
C
rin
(iii) C is |z + 1| = 1.
g.n
When z = −1 − 2i, |z + 1| = | − 1 − 2i + 1| = | − 2i| = 2 > 1.
∴ −1 − 2i lies outside C. et
When z = −1 + 2i, |z + 1| = | − 1 + 2i + 1| = |2i| = 2 > 1.
∴ −1 + 2i lies outside C.
z+4
∴ f (z) = 2 is analytic inside C.
z + 2z + 5 Z
∴ By Cauchy’s theorem f (z)dz = 0.
C

z2
15. (a) (ii) Find the residues of f (z) = at its isolated
(z − + 2)2 1)2 (z
singularities using Laurent’s series expansions. Also state the
valid region.

Downloaded From : www.EasyEngineering.net


———-
222
Downloaded Supplement to Engineering Mathematics - II
From : www.EasyEngineering.net

z2 A B C D
Solution. Let = + + + .
(z − 1)2 (z + 2)2 z − 1 (z − 1)2 z + 2 (z + 2)2

∴ z2 = A(z − 1)(z + 2)2 + B(z + 2)2 + C(z + 2)(z − 1)2 + D(z − 1)2 .
1
When z = 1, 9B = 1 ⇒B= .
9
4
When z = −2, 9D = 4 ⇒D= .
9
Equating the coefficients of z3 we get
A + C = 0 ⇒ C = −A.

When z = 0, −4A + 4B + 2C + D = 0
ww − 4A +
4 4
− 2A + = 0

6A =
8
9
w.E9 9

A=
8
=a
6 × 9 27
4
.
syE
∴C=− .
4
27 ngi

z2
=
4
·
1 1
+ ·
1

4
·
1 nee
4
+ ·
1
.
(z − 1)2 (z + 2)2 27 z − 1 9 (z − 1)2 27 z + 2 9 (z + 2)2
4 1 1 1 4 1 rin
Consider · + · = 
27 z − 1 9 (z − 1)2 27z 1 − 1
+ 
9z 1 − 1
2
2
z g.n
z

=
4
27z
"
1−
1
z
!−1
1
+ 2 1−
9z
1
z
# "
!−2

#
et
4 1 1 1 1 2 3
= 1+ + 2 + 3 + ··· + 2 1 + + 2 + ···
27z z z z 9z z z
1 4
Residue at z = 1 = Coefficient of = .
z 27
4 1 4 1 4 4
Also, − = −  
9 (z + 2)2 27 z + 2 9z2 1 + 2 2 27z 1 + 2
z z
!−2 !−1
4 2 4 2
= 1+ − 1+
9z2 z 27z z
 !2  " #
4  2 2 
 4 2 4
= 2 1 − 2 · + 3 · · ·  − 1− + 2 .
9z z z 27z z z

Downloaded From : www.EasyEngineering.net


———-
Anna University
Downloaded Solved Question Papers
From : www.EasyEngineering.net 223

1 4
Residue at z = −2 = Coefficient of =−
z 27
The valid region is |z| < 2 · 5.

Z 2π
sin2 θ
15. (b) Evaluate dθ, a > b > 0.
0 a + b cos θ
Z 2π Z 2π
sin2 θ 1 − cos 2θ
Solution. Let, I = dθ = dθ.
0 a + b cos θ 0 2(a + b cos θ)
dz
Put z = eiθ then dθ = .
iz
Now, z2 = ei2θ = cos 2θ + i sin 2θ.
1 − z2 = 1 − cos 2θ − i sin 2θ.
ww ∴ 1 − cos 2θ = R.P (1 − z2 ).

w.E
|z| = 1.
As θ varies from 0 to 2π, C varies along the unit circle

a
Hence, I = R.P
Z
syE 1 − z2
 
 iz
dz
= R.P
1
2
Z
1 − z2

2a + b z + z iz1

dz

ngi
b 1
C 2 a+ 2 z+ z C
Z Z
z(1 − z2 ) 1 − z2
= R.P
1
2i (2az + bz2 + b) z nee
dz
= R.P
1
2i 2az + bz2 + b
dz

rin
C C
Z Z
1 1 − z2 1 1 − z2
= R.P   dz = R.P dz
2i
C
Z
b z2 + 2a b z+1 2bi
C

z +
g.nb

a 2
+ 1 − a2
b2

= R.P
1
2bi
Z
C

z +
1 − z2

b

a 2
− a2 −b2
b2
dz et
1 1 − z2
= R.P  2  √ 2 2 2 dz
2bi
C z + ab − a b−b
Z
1 1 − z2
= R.P  √  √  dz
2bi 2 2
z + ba + a b−b z + ba − a b−b
2 2
C
Z
1 1 − z2
= R.P dz
2bi (z − z1 )(z − z2 )
C
Z
1 1 − z2
= R.P f (z)dz where f (z) = where
2bi (z − z1 )(z − z2 )
C

Downloaded From : www.EasyEngineering.net


———-
224
Downloaded Supplement to Engineering Mathematics - II
From : www.EasyEngineering.net

√ √
a a2 − b2 a a2 − b2
z1 = − − and z2 = − + .
b b b b
z1 and z2 are simple poles.
√ √
a
a2 − b2 a a2 − b2 a
When z = z1 , |z| = |z1 | = − − = + > > 1.
b b b b b
a a2
As a > b =⇒ > 1 =⇒ 2 > 1, a2 > b2 , a2 − b2 > 0.
b b
∴ z1 lies outside C.
Also, z1 z2 = 1 (products of the roots)
1 1
Now, z2 = =⇒ |z2 | = < 1.
z1 |z1 |
∴ z2 lies inside C. Z
ww By Cauchy’s residue theorem, f (z)dz = 2πiR(z2 ).

w.E
Now, R(z2 ) = lim (z − z2 ) f (z)
C

a 

R − +
z→z2
 a
syE √ 
a2 − b2 
 = lim (z − z2 )
1 − z2
=
1 − z22
.
b b
ngi (z − z1 )(z − z2 ) z2 − z1
z→z2
 √ 2
 −a + a2 − b2 
2
Now, 1 − z2 = 1 −  
b nee


=
b2 − (a2 + a2 − b2 − 2a a2 − b2 )
b2 rin
=
b − a − a + b2 + 2a a2 − b2
2 2 2

g.n
=
2 2
2b − 2a + 2a a2 − b2
b2
√ et
b2

2a a − b − 2(a2 − b2 )
2 2
=
b2
√ √
2 a − b (a − a2 − b2 )
2 2
=
b2

2 a −b 2 2
z2 − z1 =
b
√ √
2 a − b2 (a − a2 − b2 )
2
∴ R(z2 ) = √ ×b
b2 2 a2 − b2

a − a2 − b2
=
b

Downloaded From : www.EasyEngineering.net


———-
Anna University
Downloaded Solved Question Papers
From : www.EasyEngineering.net 225

 √ 
1  a − a2 − b2 
∴ I = R.P · 2πi   
2bi b
π √
= 2 (a − a2 − b2 )
b

2.4.9 May/June 2012 [R 2008]


Part A

9. Define singular point.


If a function fails to be analytic at a point z0 , but is analytic

ww at some point in every neighborhood of z0 , then z0 is a singular


point of f.
w.E
10. Find the residue of f (z) =
4
at a simple pole.

a
Solution. f (z) =
syE
4
z3 (z − 2)
.
z3 (z− 2)

ngi
The poles are given by z = 0, z = 2.
z = 2 is a simple pole.
Now R(2) = lim(z − 2) f (z)
z→2 nee
= lim(z − 2) 3
4
rin
4 4 4 1
= lim 3 = 3 = = .

Part B
z→2 z (z − 2) z→2 z z 8 2
g.n
15. (a) (i) Evaluate
Z
z
(z − 1)(z − 2)2
dz, where C is |z − 2| =
1
2
using
et
C
Cauchy’s integral formula.
1
Solution. C is |z − 2| = .
2

C is a circle with (2, 0) as Y

1
centre and radius units.
2 c
The singular points are z = 1 z=1
1
2
• • • X
and z = 2. z = 1 lies outside C z=0 z=
3
2
z=2

and z = 2 lie within C.

Downloaded From : www.EasyEngineering.net


———-
226
Downloaded Supplement to Engineering Mathematics - II
From : www.EasyEngineering.net

Z Z z
z z−1
∴ dz = dz
(z − 1)(z − 2)2 (z − 2)2
C C
Z
f (z) z
= dz where f (z) =
(z − 2)2 z−1
C
z−1−z
= 2πi × f ′ (2) f ′ (z) =
(z − 1)2
1
= 2πi × (−1) =−
(z − 1)2
−1

ww = −2πi f ′ (2) =
1
= −1.

w.E
15. (a) (ii) Expand f (z) =
1
(z + 1)(z + 3)
in Laurent’s series valid for

a
the regions 1 < |z| < 3 and |z| > 3.
Solution. Let f (z) = syE 1
=
A
+
B
(z + 1)(z + 3) z + 1 z + 3
.

1 = A(z + 3) + B(z + 1). ngi


When z = −1, 2A = 1, A =
1
. nee
1
2
rin
When z = −3, −2B = 1, B = − .
1 1 1
2
1 1 g.n
∴ = · − ·
(z + 1)(z + 3) 2 z + 1 2 z + 3

(i) Consider 1 < |z| < 3.


.
et
This is annular about z = 0 and f (z) is analytic in the
region.
∴ f (z) has a Laurent’s series expansion about z = 0.

z
|z| < 3 ⇒ | | < 1.
3
1
|z| > 1 ⇒ < 1.
|z|
1 1 1 1
∴ f (z) = ·  − ·  
2 z 1+ 1 2 3 1+ z
z 3

Downloaded From : www.EasyEngineering.net


———-
Anna University
Downloaded Solved Question Papers
From : www.EasyEngineering.net 227

!−1
1 1 1 z −1
= 1+ − 1+
2z z 6 3
" # " #
1 1 1 1 1 1 z z2 z3 z4
= 1− + − + ··· − 1 − + − + ···
2z z z2 z3 z4 6 3 9 27 81

(ii) Consider the region |z| > 3.


This region is annular about z = 0, where f (z) is
analytic.
1 1 3
|z| > 3 ⇒ < ⇒ < 1.
|z| 3 |z|
1 1
Since < < 1, we have
|z| 3
ww f (z) =
1
· 
1 1
− · 
1

w.E
2 z 1+

1 1
1
z
!−1
2 z 1+ 3

1 3
z
!−1
=

1
a
2z
"
1+
z
1 1

syE
2z
1
1+

1
z
#
1
"
3 9 27 81
#
=
2z
1 − + 2 − 3 + 4 ··· −
z z z z
ngi 2z
1 − + 2 − 3 + 4 ··· .
z z z z

15. (b) (i) Evaluate


Z
z−1
(z + 1)2 (z − 2)
nee
dz where C is |z − i| = 2.
C
z−1 rin
Solution. Let f (z) =
(z + 1)2 (z − 2)
.
The poles are given by z = −1 and z = 2. g.n
z = −1 is a pole of order 2 and z = 2 is a simple pole.
C is |z − i| = 2.
et
√ √
When z = −1, |z − i| = | − 1 − i| = 1 + 1 = 2 < 2.
⇒ z = −1 lies inside C.
√ √
When z = 2, |z − i| = |2 − i| = 4 + 1 = 5 > 2.
∴ z = 2 lies outside C. Z
∴ By Cauchy’s residue theorem f (z)dz = 2πiR(−1).
C
Since z = −1 is a pole of order 2,
!
1 d   d z−1
R(−1) = lim (z + 1)2 f (z) = lim (z + 1)2
(2 − 1)! z→−1 dz z→−1 dz (z + 1)2 (z − 2)

Downloaded From : www.EasyEngineering.net


———-
228
Downloaded Supplement to Engineering Mathematics - II
From : www.EasyEngineering.net
!
d z−1 z − 2 − (z − 1) −3 + 2 −1
= lim = lim = = .
z→−1 dz z − 2 z→−1 (z − 2)2 9 9
Z !
z−1 −1 2πi
Hence, 2
dz = 2πiR(−1) = 2πi =− .
(z + 1) (z − 2) 9 9
C

Z∞
cos mx
15. (b) (ii) Evaluate dx using contour integration.
x2 + a2
0 Z
eimz
Solution. Let us consider the integral dz, where C is
z2 + a2
C
the simple closed curve consisting of the real axis from −R to R

ww and the upper semi circle

w.E
S : |z| = R taken in the anticlockwise sense.
Let f (z) = 2
1
z + a2

Now
Z
a
imz
e f (z)dz =
ZR
syE
Z
eimx f (x)dx + eimz f (z)dz
s
(1)
C

Let us evaluate
Z −R

ngi
eimz f (z)dz.
Y

The poles are given by


C
nee S

rin
>

z2 + a2 = 0 • ai

(z + ai)(z − ai) = 0
−R
>
g.n R
X

i.e., z = ai, −ai


z = ai lies inside C.
• −ai

et
Z
By Cauchy’s residue theorem, eimz f (z)dz = 2πiR(ai).
C
z = ai is a simple pole.

∴ R(ai) = lim (z − ai)eimz f (z)


z→ai


✘ eimz e−am
= lim ✘(z ✘
− ai) ✘
✘ = .
z→ai (z ✘
(z + ai)✘ − ai) 2ai
Z
2πie−am π
∴ eimz f (z)dz = = e−am .
2ai a
C

Downloaded From : www.EasyEngineering.net


———-
Anna University
Downloaded Solved Question Papers
From : www.EasyEngineering.net 229

1
Also | f (z)| = | | → 0 as |z| → ∞.
z2 + a2 Z
∴ By Jordon’s lemma, eimz f (z)dz → 0 as R → ∞.
s
ZR Z
π
Now (1)⇒ e−am = eimx f (x)dx + eimz f (z)dz.
a s
−R
Taking limit as R → ∞ we get

ZR Z
π −am
e = lim eimx f (x)dx + lim eimz f (z)dz
a R→∞ R→∞ s
−R
Z∞
ww π −am
a
e =
eimx
x2 + a2
dx + 0

w.E =
−∞
Z∞
cos mx + i sin mx
dx.
a syE −∞
x2 + a2

Equating the real parts we get

Z∞ ngi
cos mx
x2 + a2
dx =
πe−am
a nee
−∞
Z∞
πe−am rin
2
cos mx
2
x +a 2
dx =
a g.n
0
Z∞
emx
x2+a 2
dx =
πe−am
2a
.
et
0

2.5 Unit V Laplace Transforms.

2.5.1 May/June 2016 [R 2013]

Part A

5. State convolution theorem on laplace transforms.


Solution. Refer page 289 of the main text book.

Downloaded From : www.EasyEngineering.net


———-
Downloaded From : www.EasyEngineering.net

ww
w.E
UNIT 5 asy
En
gin
ee
By EasyEngineering.net rin
g.n
et

Downloaded From : www.EasyEngineering.net


Downloaded From : www.EasyEngineering.net

5 Laplace Transform

ww 5.1 Introduction

w.E The method of solving a differential equation involves many steps, namely
1) Computation of complementary function. 2) Computation of particular integral.

asy
3) Evaluation of the arbitrary constants using the initial conditions. But Laplace
transform is a powerful tool for solving differential equations without involving all

En
the above steps. By the application of Laplace transforms, the given differential

gin
equation can be converted into an algebraic equation and the process of finding
the solution becomes easy.

ee
Laplace transforms was first used by the French Mathematician Pierre Simon
De Laplace in the 18th century. Laplace transform has wide applications in the
Engineering fields like network analysis, control systems, circuit theory and many rin
other areas. Z ∞
g.n
Definition. Let f (t) be defined for all t ≥ 0, then
Laplace transform of f (t), if the integral exists.
It is denoted by L[ f (t)]. It is a function of s.
0
et
e−st f (t)dt is defined as the

Z ∞
Hence, L[ f (t)] = e−st f (t)dt = F(s).
0

Result. The parameter s may be real or complex.

Downloaded From : www.EasyEngineering.net


Downloaded From : www.EasyEngineering.net

516 Engineering Mathematics - II

5.2 Sufficient condition for existence of Laplace Transforms

[May2015, Dec 2014, May 2011]


Let f (t) be defined for all t ≥ 0 such that (i) f (t) is piecewise continuous in the
interval [0, ∞) and (ii) f (t) is of exponential order α > 0, then the Laplace transform
of f (t) exists for s > α.
Note

1. Piecewise continuous on [0, ∞) means that the function is continuous on

ww every finite subinterval 0 ≤ t ≤ α except possibly at a finite number of


points where they have jumps i.e., f (x+) and f (x−) exist but not equal.

w.E 2. f (t) is of exponential order α > 0 if | f (t)| ≤ Meαt for all t ≥ 0 and M is a

asy
constant. In other words, lim e−αt f (t) is finite.

t→∞

Example. tn is of exponential order as t → ∞.

En tn n!

gin
lim e−αt tn = lim αt = lim n αt = 0.

t→∞ t→∞ e t→∞ α e

ee
Result. The above conditions are sufficient but not necessary.
h 1 i
Example. L √ exists but it is not continuous at t = 0.
t
Linearity Property
rin
g.n
If f (t) and g(t) are two continuous functions of t and k is a constant, then
(i) L[ f (t) ± g(t)] = L[ f (t)] ± L[g(t)].
(ii) L[k f (t)] = kL[ f (t)] where k , 0.
Laplace Transform of Standard functions
Z ∞
et
h e−st i∞ 1 1
1. L[1] = e−st 1dt = = − [0 − 1] = .
0 −s 0 s s
Z ∞ Z ∞ h e−(s−a)t i∞
at −st at 1 1
2. L[e ] = e e dt = e−(s−a)t dt= =− [0 − 1] = .
0 0 −(s − a) 0 s−a s−a
1
3. L[e−at ] = .
s+a

Downloaded From : www.EasyEngineering.net


Downloaded From : www.EasyEngineering.net

Laplace Transform 517

∞ ∞ h e−st i∞ Z ∞ e−st
 e−st  1 h e−st i∞
Z Z
1
4. L[t] = e−st tdt = td = t − dt = = 2.
0 0 −s −s 0 0 −s s −s 0 s
Z ∞ Z ∞  −st  h −st i Z ∞ −st
n −st n n e ne ∞ e
5. L[t ] = e t dt = t d = t − ntn−1 dt
0 0 −s −s 0 0 −s
n ∞ −st n−1
Z
n n n − 1 n−2
= e t dt = L[tn−1 ] = L[t ]
s 0 s s s
nn−1n−2
= · · · L[t]
s s s
nn−1n−2 1 n!
= · · · 2 = n+1 .
s s s s s
h eat + e−at i 1 h

ww
i 1h 1 1 i
at −at
6. L[cosh at] = L = L[e ] + L[e ] = +
2 2 2 s−a s+a
1 h s + a + s − a i 1 h 2s i s
= = = 2 .

w.E 7. L[sinh at] = L


2 s2 − a2
h eat − e−at i 1 h
2
2 s2 − a2
= L[eat ] − L[e−at ]
2
i
s − a2

=
1h 1
2 s−a s+a

asy1 i

h eiat + e−iat i 1 h 1
= 2
a
s − a2
.
1 i
8. L[cos at] = L
2
En
=
1 h s + ai + s − ai i
+
2 s − ai s + ai
s
.

gin
= 2 2
= 2
2 s +a s + a2
h eiat − e−iat i 1 h 1 1 i
9. L[sin at] = L = −

ee
2i 2i s − ai s + ai
1 h s + ai − s + ai i 1 h 2ai i a
= = = 2 .
2i 2
s +a 2 2
2i s + a

2

Worked Examples
s + a2

rin
✍ ✌

Example 5.1. Write a function for which Laplace transform does not exist. g.n
Explain why Laplace transform does not exist?
Solution. Let f (t) =
2
et .
2
et [May 2007]

lim e−st f (t) = lim e−st et


t→∞ t→∞
2
= lim e−st+t
t→∞

= lim et(−s+t) = e∞ = ∞.
t→∞
2
∴ lim e−st et is not finite.
t→∞

Downloaded From : www.EasyEngineering.net


Downloaded From : www.EasyEngineering.net

518 Engineering Mathematics - II

2
i.e., et is not of exponential order.
∴ Laplace transform of f (t) does not exist.
# "
1 − cos t
Example 5.2. Is the linearity property applicable to L ? Reason out.
t
[Dec 2012]
1 cos t
Solution. Linearity property can be applied when and are continuous.
t t
1 cos t
Here, and are not even defined at t = 0. Hence, they are not continuous.
t t
1 − cos t
Therefore, linearity property is not applicable to .
t

ww Example 5.3. Find L[3e5t + 5 cos t].


Solution. L[3e5t + 5 cos t] = 3L[e5t ] + 5L[cost]
[Jan 2009]

w.E =3
1
s−5
+5 2
s
=
3
+ 2
5s
s +1 s−5 s +1
.

Solution. L[cos πt + e asy


Example 5.4. Find L[cos πt + e− 3 t + sin 8t].
− 23 t
2

+ sin 8t] = L[cos πt] + L[e − 23 t


] + L[sin 8t]
[Jun 2009]

En =
s
+
1
+
8
.

gin
Example 5.5. Find L[t2 + e−5t + 8 + sinh 5t].
s2 + π2 s + 2
3
s2 + 64

[Jun 2007]

=
2!
s 3
+
s
1
+ 5
1
+8 + 2
s
5
ee
Solution. L[t2 + e−5t + 8 + sinh 5t] = L[t2 ] + L[e−5t ] + 8L[1] + L[sinh 5t]
2
= 3+
s − 25 s s
1
+ 5
8
+ + 2
s
5
s − 25
.
rin
Example 5.6. Find L[(t + 1)2 ].
g.n [Jun 2002]
Solution. L[(t + 1)2 ] = L[t2 + 2t + 1]
2!
s 3
1 1
= L[t2 ] + 2L[t] + L[1] =
+2 2 +
s s
et
2 2 1 2 + 2s + s2
= 3+ 2+ = .
s s s s3
Example 5.7. Find L[cos2 2t]. [Dec 2005]
h 1 + cos 4t i 1 h i 1h1 s i
Solution. L[cos2 2t] = L = L[1] + L[cos 4t] = +
2 2 2 s s2 + 16
1 h s2 + 16 + s2 i 1 h 2s2 + 16 i s2 + 8
= = = .
2 s(s2 + 16) 2 s(s2 + 16) s(s2 + 16)

Downloaded From : www.EasyEngineering.net


Downloaded From : www.EasyEngineering.net

Laplace Transform 519

Example 5.8. Find L[sin 2t sin 3t]. [Dec 2004]


1 1h s s i
Solution. L[sin 2t sin 3t] = L[cos t − cos 5t] = −
2 2 s2 + 1 s2 + 25
h 2 2
s s + 25 − s − 1 i sh 24 i
= =
2 (s2 + 1)(s2 + 25) 2 (s2 + 1)(s2 + 25)
12s
= 2 .
(s + 1)(s2 + 25)

Example 5.9. Find L[sin3 2t]. [May 2004]

ww Solution. L[sin3 2t] = L


4
h1 i 1h
(3 sin 2t − sin 6t) = 3 2
6 h s2 + 36 − s2 − 4 i 3 h
2
− 2
4 s + 4 s + 36
6 i

w.E
32 i 48
= 2 2
= 2 2
= 2 .
4 (s + 4)(s + 36) 2 (s + 4)(s + 36) (s + 4)(s2 + 36)

Example 5.10. Find


1
Solution. L[t− 2 ] =
Z ∞L[t 2 ].
1 asy
e−st t− 2 dt
−1
[May 2005]

Let st = x ⇒ t =
x
0
1
⇒ dt = dx. En
s s

gin
When t = 0, x = 0, when t = ∞, x = ∞

−1
∴ L[t 2 ] =

1
0
Z
e−x
Z ∞

 x − 21 dt
s see
= √
1
Z ∞

s 0
1
e−x x− 2 dx

rin
g.n
1
= √ e−x x 2 −1 dx
s 0
! Z ∞
1 1
= √ Γ

1
s

= √ π=
s

2
r
π
s
.
[Γn =
0
e−x xn−1 dx]

et
 1
Example 5.11. Find L t 2 . [Dec 1996]
Z ∞
1 1
Solution. L[t 2 ] = e−st t 2 dt
0
x 1
Put st = x ⇒ t = ⇒ dt = dx.
s s

Downloaded From : www.EasyEngineering.net


Downloaded From : www.EasyEngineering.net

520 Engineering Mathematics - II

When t = 0, x = 0, when t = ∞, x = ∞
 1 Z ∞  x  12 dx Z ∞
1 1
L t2 = e−x = 3 e−x x 2 dx
0 s s s2 0
Z ∞
1 3 1 3
= 3
e−x x 2 −1 dx = 3 Γ
s2 2
! s
0 2
! √
1 3 3 1 1 1 1 √ π
= 3
−1 Γ −1 = 3 Γ = 3 π= 3.
s2 2 2 s2 2 2 2s 2 2s 2

Example 5.12. Find L[cos 4t." sin 2t]. # [Dec 2009]


sin 6t − sin 2t 1

ww Solution. L[cos 4t. sin 2t] = L


2
= [L[sin 6t] − L[sin 2t]]
2

w.E
" #
1 6 2 3 1
= 2
− 2 = 2 − 2 .
2 s + 36 s + 4 s + 36 s + 4

2 3
Solution. We have sin t cos t =asy
Example 5.13. Find L[sin2 t cos3 t]. !
1 − cos 2t cos 3t + 3 cos t
2 4
! [May 2008]

1 En
gin
= [cos 3t + 3 cos t − cos 2t cos 3t − 3 cos 2t cos t]
8" #
1 1 3
= cos 3t + 3 cos t − (cos 5t + cos t) − (cos 3t + cos t)

=
8
1
8
1 1
"

"
ee 2
1
2
1
#
1
2
2
3
2
rin
3
cos 3t + 3 cos t − cos 5t − cos t − cos 3t − cos t
2
#

=
8 2
" h
1
− cos 3t + cos t − cos 5t

1
2
1 i# g.n
et
2 3
L[sin t cos t] = L − cos 3t + cos t − cos 5t
8 2 2
1
= L[2 cos t − cos 3t − cos 5t]
16 " #
1 2s s s
= − − .
16 s2 + 1 s2 + 9 s2 + 25

Example 5.14. Find the Laplace transform of the function defined by



t 0 < t < 1




f (t) = 
 . [Dec 2007]
0 t > 1

Downloaded From : www.EasyEngineering.net


Downloaded From : www.EasyEngineering.net

Laplace Transform 521

Z∞
Solution. L[ f (t)] = e−st f (t)dt
0

Z1 Z∞
= e−st f (t)dt + e−st f (t)dt
0 1
Z1 Z∞
= e−st · tdt + e−st × 0dt
0 1
Z1 Z1

ww e−st
!
−st
= e tdt = td
−s
0 0

w.E " −st #1


= t
e
−s 0

Z1
e−st
−s
dt.

asy
0
#1
1 e−st
"
1
= − [e−s − 0] +
s s −s 0

=−
e−s

s
1
− 2 e−s − 1
s

En
=−
e−s e−s
s
− 2 + 2.
s
1
s gin
ee 
e




Example 5.15. Find the Laplace transform of f (t) defined by f (t) = 


0

t

rin
0<t<1

t > 1.

Solution. L[ f (t)] =
Z∞
e−st f (t)dt g.n
=
Z1
0

e−st
f (t)dt +
Z∞
e−st f (t)dt
et
0 1
Z1 Z1

= e−st et dt = e−(s−1)t dt
0 0
#1
e−(s−1)t i 1 − e−(s−1)
"
1 h −(s−1)
= =− e −1 = .
−(s − 1) 0 s−1 s−1

Downloaded From : www.EasyEngineering.net


Downloaded From : www.EasyEngineering.net

522 Engineering Mathematics - II


cos t 0<t<π



Example 5.16. Find the Laplace transform of f (t) = 


t > π.

0

Z∞

Solution. L[ f (t)] = e−st f (t)dt


0

Zπ Z∞
−st
= e f (t)dt + e−st f (t)dt
0 π

ww =
0
e−st cos tdt

w.E e−st
"
= (−s cos t + sin t)
s2 + 1 0
e−πs 1
=

=
2
s +1
e−πs asy
(−s cos π + sin π) − 2

(s) + 2
s
s +1
(−s cos 0 + sin 0)

En
2
s +1 s +1
s
= (1 + e−πs ).
s2 + 1

gin
Example 5.17. Find the Laplace transform of the function defined by

sin t 0 < t < π
ee




f (t) =  . [May 2006]

rin

t>π

0

Z∞
Solution. L[ f (t)] = e−st f (t)dt
0 g.n
=

0
e−st f (t)dt +
Z∞

π
e−st f (t)dt et

= e−st sin tdt
0

e−st
"
= 2 (−s sin t − cos t)
s +1 0
e−πs 1
= 2 (−s sin π − cos π) − 2 (−s sin 0 − cos 0)
s +1 s +1

Downloaded From : www.EasyEngineering.net


Downloaded From : www.EasyEngineering.net

Laplace Transform 523

e−πs 1 1 + e−πs
= + = .
s2 + 1 s2 + 1 s2 + 1

Exercise 5 A

Find the Laplace transform of the following functions.

10. (α + βt)2 . 19. (t2 + 1)2 .

ww
2
1. sin 2t.
c
11. a + bt + √ .
2. cos2 3t. t 20. cosh at + sin 2t + t3 .

w.E 3. sin3 2t.


12. sinh2 2t. 
t 0 < t < 2




21. f (t) = 

asy 13. cosh(5t + 2). 


4. cos3 3t. 0 t > 2.


[Jun 2012]
14. cosh 2t − cosh 3t.
5. sin 3t sin 2t.

En15. sinh ωt − sin ωt.



e

 −t 0<t<4

gin
6. sin 3t cos t.


22. f (t) = 


0
 t ≥ 4.
7. (sin t − cos t)2 . 16. e3t+5 .

8. cos4 t.

9. sin(ωt + α).
ee
17. t2 + cos 2t + sin2 2t.

18. e−bt − e−at . rin



0 0 < t < 2.




23. f (t) = 


3 t ≥ 2.

g.n

Shifting Property - First Shifting theorem


1. Statement. IfZL[ f (t)] = F(s) then L[e−at f (t)] = F(s + a).
Proof. L[ f (t)] =
0

e−st f (t)dt = F(s)
et
Z ∞ Z ∞
L[e−at f (t)] = e−st e−at f (t)dt = e−(st+at) f (t)dt
Z0 ∞ 0
−(s+a)t  
= e f (t)dt = F(s + a) = L[ f (t)] s→s+a
0
2. L[eat f (t)] = F(s − a) = L[ f (t)] s→s−a .

Downloaded From : www.EasyEngineering.net


Downloaded From : www.EasyEngineering.net

524 Engineering Mathematics - II

Change of scale property


1  s
Statement. If L[ f (t)] = F(s) then L[ f (at)] = F , a > 0.
Z ∞ a a
−st
Proof. L[ f (t)] = e f (t)dt = F(s)
0
Z ∞
L[ f (at)] = e−st f (at)dt
0
x 1
Let at = x ⇒t= ⇒ dt = dx
a a
When t = 0, x = 0, when t = ∞, x = ∞

ww
Z ∞
x dx
∴ L[ f (at)] = e−s a f (x)
0 a
1 ∞ − s x
Z
1  s 1 1

w.E = [F(s)] s→ as = L[ f (t)] s→ s .



= e a f (x)dx = F
a 0 a a a a a

1
Example 5.18. Find L[et t− 2 ]. [May 1996]

asy
r r
1 h 1i h πi π
Solution. L et t− 2 = L t− 2

= = .
s→s−1 s s→s−1 s−1

En
Example 5.19. Find the Laplace transform of t .
t
e
[Jun 2013]

gin
t " #
−t 1 1
Solution. L t = L[e t] = L[t] s→s+1 = 2 = .
e s s→s+1 (s + 1)2

ee
h i
Example 5.20. Find L e−3t sin t · cos t [Dec 2011]
" #
sin 2t 1 h

rin
h i i
Solution. L e−3t sin t cos t = L e−3t = L e−3t sin 2t
2 2

g.n
!
1 1 2
= L[sin 2t] s→s+3 =
2 2 s2 + 4 s→s+3

=
1
2
= 2
(s + 3) + 4 s + 6s + 13
Example 5.21. Find L[e−3t sin2 t].
1
.

# "
et[Jun 2008]
−3t 2 2 1 − cos 2t
Solution. L[e sin t] = L[sin t] s→s+3 = L
2 s→s+3
" #
1 1 1 s
= [L[1] − L[cos 2t]] s→s+3 = −
2 2 s s2 + 4 s→s+3
" #
1 1 s+3
= − .
2 s + 3 (s + 3)2 + 4

Downloaded From : www.EasyEngineering.net


Downloaded From : www.EasyEngineering.net

Laplace Transform 525

Example 5.22. Find L[cosh "t sin 2t]. [Dec 2009]


et + e−t
#
1h t i
Solution. L[cosh t sin 2t] = L sin 2t = L(e sin 2t + e−t sin 2t)
2 2

1
= [L[sin 2t] s→s−1 + L[sin 2t] s→s+1 ]
2" #
1 h 2 i h 2 i
= + 2
2 s2 + 4 s→s−1 s + 4 s→s+1
" #
1 2 2
= +
2 (s − 1)2 + 4 (s + 1)2 + 4
1 1

ww =

Example 5.23. Find L[e2t cos 5t].


+
(s − 1) + 4 (s + 1)2 + 4
2
.

[May 2008]

w.E 2t
Solution. L[e cos 5t] = L[cos 5t] s→s−2 = 2
s 
=
s−2
s + 25 s→s−2 (s − 2)2 + 25
.

asy
Example 5.24. Find the Laplace transforms of (i) cosh at sinh bt (ii) (1 + te−t )3 .
Solution.

En
h eat + e−at i 1 
sinh bt = L eat sinh bt + e−at sinh bt

(i) L[cosh at sinh bt] = L
2 2

gin
1 h at −at
i
= L[e sinh bt] + L[e sinh bt]
2
1h i

=
2
1 h b 
2 s2 + b2
1h b
ee
= L[sinh bt] s→s−a + L[sinh bt] s→s+a

s→s−a +
 b 
s2 + b2
b
s→s+a
i

rin
g.n
i
= 2 2
+ 2 2
.
2 (s − a) + b (s + a) + b
(ii) L (1 + te−t )3 = L 1 + 3te−t + 3t2 e−2t + t3 e−3t
   

= L[1] + 3L[te−t ] + 3L[t2 e−2t ] + L[t3 e−3t ]

=
1
s
+ 3L[t] s→s+1 + 3L[t2 ] s→s+2 + L[t3 ] s→s+3
et
1 h1i h2i h6i
= +3 2 +3 3 + 4
s s s→s+1 s s→s+2 s s→s+3
1 3 6 6
= + + + .
s (s + 1)2 (s + 2)3 (s + 3)4

Downloaded From : www.EasyEngineering.net


Downloaded From : www.EasyEngineering.net

526 Engineering Mathematics - II

Example 5.25. Find L[cosh at cos at]. [Dec 2008]


h eat + e−at i 1h i
Solution. L[cosh at cos at] = cos at = L[eat cos at] + L[e−at cos at]
2 2
1h i
= L[cos at] s→s−a + L[cos at] s→s+a
2
1 h s   s  i
= +
2 s2 + a2 s→s−a s2 + a2 s→s+a
1 h s−a   s+a i
= + .
2 (s − a)2 + a2 (s + a)2 + a2
Example 5.26. Find L[e−3t (2 cos 5t − 3 sin 5t)].

ww Solution. L e−3t (2 cos 5t − 3 sin 5t) = L 2e−3t cos 5t − 3e−3t sin 5t


   
[May 1999]

w.E = 2L e−3t cos 5t − 3L e−3t sin 5t


   

= 2L[cos 5t] s→s+3 − 3L[sin 5t] s→s+3

asy
h s i h 5 i
=2 2 −3 2
s + 25 s→s+3 s + 25 s→s+3
2(s + 3) 15

En= −
(s + 3) + 25 (s + 3)2 + 25
2
.

Exercise 5 B
gin
N
ee
Find the Laplace Transform of the following functions.

8. e−3t cos2 t.
rin [Dec 2007]

g.n
X
−bt
1. an e cos nt. [Jan 2006]
9. e2t sin3 t. [May 2003]
n=0

2. t2 e−2t .

3. et (cosh 2t + 12 sinh 2t).


10. e4t sin 2t sin t.

11. e−3t cos 4t cos t.


et[May 2006]

4. e−t (3 sinh 2t − 5 cosh 2t). 12. e2t cos 3t sin t.


3
5. e− 2 t sin 3t. [May 2008] 13. et cos3 t.

6. sinh 3t cos t. [Nov 2009] 14. et sin 4t cos 2t.


1
7. e−7t t− 2 . [Jan 2009] 15. (t + 1)3 e−t .

Downloaded From : www.EasyEngineering.net


Downloaded From : www.EasyEngineering.net

Laplace Transform 527

dn 
Theorem. If L[ f (t)] = F(s), then L[tn f (t)] = (−1)n n F(s) .

Z ∞ ds
−st
Proof. We have L[ f (t)] = e f (t)dt
0
Z ∞
F(s) = e−st f (t)dt.
0

Differentiating w.r.t s we get,


Z ∞
d   d  −st 
F(s) = e f (t) dt
ds ds
Z0 ∞
= e−st (−t) f (t)dt

ww −
d 
F(s) =

0
Z ∞
e−st t f (t)dt = L[t f (t)].

w.E
ds 0
d
Now L[t2 f (t)] = L[t t f (t)] = − L[t f (t)]
ds

asy
=−
d  d
ds

d
ds
n h i
 d2
F(s) = (−1)2 2 F(s)
ds

En
In general, L[tn f (t)] = (−1)n n F(s) .
ds

gin


Worked Examples

Example 5.27. Find L[t cosh 3t].


Solution. L[t cosh 3t] = −
d
ds
n ee o
L[cosh 3t] = −
d h s
ds s2 − 9
i
=−
h 2

(s2 − 9)2
rin
s + 9 − s × 2s
[May 1995]
i

=− 2
9−s
(s − 9)
2

2
= 2
2
s −9
(s − 9) 2
= 2
1
(s − 9)
.
g.n
Example 5.28. Find L[t2 sin at].

2
d 2 h
Solution. L[t2 sin at] = (−1)2 2 L[sin at] = 2 2
ds
i d2 h a i
ds s + a2
et [May 2001]

d d  d h s i
= a 2 (s2 + a2 )−1 = a (−1)(s2 + a2 )−2 2s = −2a

ds ds 2
ds (s + a )2 2
2 2 2 2 2
!
(s + a ) .1 − s.2.(s + a )2s
= −2a
(s2 + a2 )4
h i
(s2 + a2 ) s2 + a2 − 4s2 [−3s2 + a2 ] [3s2 − a2 ]
= −2a = −2a = 2a .
(s2 + a2 )4 (s2 + a2 )3 (s2 + a2 )3

Downloaded From : www.EasyEngineering.net


Downloaded From : www.EasyEngineering.net

528 Engineering Mathematics - II

Example 5.29. Find L[t cos 3t].


d n o d h s i
Solution. L[t cos 3t] = − L[cos 3t] = −
ds ds s2 + 9
h s2 + 9 − s2s i 9 − s2 s2 − 9
=− = − = .
(s2 + 9)2 (s2 + 9)2 (s2 + 9)2
Example 5.30. Find Laplace transform of t sin 2t. [Dec 2010]
d
Solution. L[t sin 2t] = − {L[sin 2t]}
ds
" #
d 2
=−
ds s2 + 4

ww =−
d h
ds
2 · (s2 + 4)−1
i

w.E = −2 · (−1) · (s2 + 4)−2 · 2s

=
4s
.

asy
(s2 + 4)2

Example 5.31. Find L[t sin at].

En
d n o d  a  d
Solution. L[t sin at] = − L[sin at] = − 2 2
= −a (s2 + a2 )−1
ds ds s + a ds
2as

gin
= −a(−1)(s2 + a2 )−2 .2s = 2
(s + a2 )2
.

Example 5.32. Find L[t cos at].


Solution. L[t cos at] = −
d h
ds ee
i
L[cos at] = −
h a2 − s2 i
d h s i
ds s2 + a2
s2 − a2
= −
s2 + a2 − s2s

rin
(s2 + a2 )2

=− 2
(s + a2 )2
=

Example 5.33. Find L[sin at − at cos at].


(s2 + a2 )2
.

g.n [May 2003]


Solution. L[sin at − at cos at] = L[sin at] − aL[t cos at]

=
a  d
− a − L[cos at]

et
s + a2
2 ds
a d  s 
= 2 +a
s + a2 ds s2 + a2
a (s2 + a2 ).1 − s2s
= 2 +a
s + a2 (s2 + a2 )2
a s + a2 − 2s2
2
= 2 +a
s + a2 (s2 + a2 )2

Downloaded From : www.EasyEngineering.net


Downloaded From : www.EasyEngineering.net

Laplace Transform 529

a(a2 − s2 )
" 2
s + a2 + a2 − s2 2a3
#
a
= 2 + = a = .
s + a2 (s2 + a2 )2 (s2 + a2 )2 (s2 + a2 )2

Example 5.34. Find L[te−t sin t].


d n −t o d h i
Solution. L[te−t sin t] = − L[e sin t = − L[sin t] s→s+1
ds ds
d h 1 i d h 1 i
=− = −
ds s2 + 1 s→s+1 ds (s + 1)2 + 1
d h 1 i
=− 2
= −(−1)(s2 + 2s + 2)−2 (2s + 2)
ds s + 2s + 2
2s + 2
= 2 .

ww (s + 2s + 2)2
Example 5.35. Find L[t sin 3t" cos 2t]. [May 2008]

w.E
#
[sin 5t + sin t]
Solution. L[t sin 3t cos 2t] = L t
2
1  d  
= − L[sin 5t] + L[sin t]

=−
2 ds
1d h 5
asy+
2 ds s2 + 25 s2 + 1
1 i

=−
1d h 2
2 ds En
5(s + 25)−1 + (s2 + 1)− 1
i

=−
1 h −5. 2s
2
2 (s + 25) 2
− 2
gin2s i
(s + 1)2

= 2
1
= − (−2) 2
2
5s
h 5s
(s + 25)
+
(s + 25)2 (s2 + 1)2
s
2
+ 2
ee s
(s + 1)
.
2
i

rin
Example 5.36. Find L[t2 e−3t sin 2t].
g.n
[Jun 2013, May 2000]

Solution. L[t2 e−3t sin 2t] =


d2 h −3t
ds2
= 2
2
d h
L[e
2
sin 2t]
i
i d2 h
=
ds2
L[sin 2t] s→s+3
i
et
2
ds (s + 3) + 4
d2 h 2 i
= 2 2
ds s + 6s + 13
d2
= 2 2 (s2 + 6s + 13)−1
ds
d h i
=2 (−1)(s2 + 6s + 13)−2 (2s + 6)
ds

Downloaded From : www.EasyEngineering.net


Downloaded From : www.EasyEngineering.net

530 Engineering Mathematics - II

d h 2s + 6 i
= −2
ds (s2 + 6s + 13)2
d h s+3 i
= −4
ds (s2 + 6s + 13)2
(s2 + 6s + 13)2 − (s + 3)2(s2 + 6s + 13)(2s + 6)
= −4
(s2 + 6s + 13)4
s + 6s + 13 − 4(s2 + 6s + 9)
2
= −4
(s2 + 6s + 13)3
s2 + 6s + 13 − 4s2 − 36 − 24s
= −4

ww = −4 2
(s2 + 6s + 13)3
−3s2 − 18s − 23
(s + 6s + 13)3
= 4
3s2 + 18s + 23
(s2 + 6s + 13)3
.

w.E
Exercise 5 C

asy
Find the Laplace transform of the following functions.

En 6. t2 cosh 2t. [Jan 2005]


1. t cos3 t.

2. te−4t sin 3t. gin 7. te2t sin 3t. [Dec 2006]

3. t2 e−t cos t.

4. tet cos t.
ee
[May 2005]
8. t cosh 5t.

9. t2 e−3t . rin
[Dec 2003]

[Dec 2009]

5. t2 sin 2t. [Dec 2004] 10. t2 et sin t. g.n [May 2007]

Theorem. If L[ f (t)] = F[s] and if lim


 
Proof. We have, F(s) = L f (t) =
Zt→0

f (t)
t
exists, then L

e−st f (t)dt
h f (t) i Z ∞
t
=
s
F(s)ds. et
0
Integrating both sides w.r.t s from s to ∞
Z ∞ Z ∞Z ∞ Z ∞ hZ ∞ i
F(s)ds = e−st f (t)dtds = f (t) e−st ds dt
s
Z s ∞ 0  −st  Z ∞0 s
Z ∞
e ∞ f (t) −∞ −st f (t) h f (t) i
= f (t) dt = − (e − e )dt = e−st dt = L .
0 −t s 0 t 0 t t

Downloaded From : www.EasyEngineering.net


Downloaded From : www.EasyEngineering.net

Laplace Transform 531

Extension
h f (t) i Z ∞Z ∞ Z ∞
L = ··· F(s)(ds)n .
tn s s s
✎ ☞
Worked Examples
✍ ✌
h 1 − et i
Example 5.37. Find L . [Dec 2008]

t Z ∞
h 1 − et i Z
1 1 
t
Solution. L = L[1 − e ]ds = − ds
t s s s s−1
h i∞  s ∞

ww = log s − log(s − 1) = log

= log
 1 ∞
1 s
s

= log 1 − log(
s−1 s
s
s−1
)

w.E
1− s
 s − 1
= log .
s

asy
Example 5.38. Find the Laplace transform of
1 − e−t
t
. [Dec 2013]

1 − e−t
" # Z∞
En Z∞
1 1
!

gin
−t
Solution. L = L[1 − e ]ds = − ds
t s s+1
s s
s ∞

ee
 ∞ 
= log s − log(s + 1) s = log
s+1 s
  1 ∞

rin
  
 = log 1 − log s
 
= log 
1 + 1s s

s+1

= log
s+1
s
!
.
g.n
Example 5.39. Find L
h sin 2t i

h sin 2t i Z ∞
t
.
Z ∞
2
et[Jan 2006]

Solution. L = L[sin 2t]ds = 2


ds
t s s s +4
1 s ∞  s
= 2 tan−1 = tan−1 ∞ − tan−1
2 2 s 2
π −1
 s
= − tan
2 2
−1 s
 
= cot .
2

Downloaded From : www.EasyEngineering.net


Downloaded From : www.EasyEngineering.net

532 Engineering Mathematics - II

h 1 − cos 2t i
Example 5.40. Find L . [Apr 2004]
Z t∞ Z ∞
h 1 − cos 2t i h i 1 s 
Solution. L = L 1 − cos 2t ds = − 2 ds
t s s s s +4
1 ∞ 2s
Z
= log s − ds
2 s s2 + 4
h 1 i∞
= log s − log(s2 + 4)
2 s
h s i∞
= log √
s2 + 4 s

s2 + 4

ww = log

h sin at i
s
.

w.E
Example 5.41. Find L

Solution. L
h sin at i
t
=
Z ∞
t
.

L[sin at]ds =
Z ∞

s + a2
2
a
ds
[Dec 2009]

asy
s s
1 s ∞  s
= a tan−1 = tan−1 ∞ − tan−1
a a s a

En
π −1 s
 
−1 s
 
= − tan = cot .
2 a a

Example 5.42. Find L


t gin
h cos at − cos bt i

h cos at − cos bt i Z ∞ h
. [Dec 2012, May 2011, Dec 2007]

ee
i
Solution. L = L cos at − cos bt ds
t
Zs ∞ 
=
s
s

s2 + a2 s2 + b2
s 
ds
rin
=
1h
2
log(s2 + a2 ) − log(s2 + b2 )
i∞
s
g.n
=
1
2
h
log 2

h e−at − e−bt i
2 2
s +a ∞ 1
s + b2 s
i
=
2
log
s + b2
2

s2 + a2
.
et
Example 5.43. Find L . [May 2006]
t
h e−at − e−bt i Z ∞ h i Z ∞
1 1 
−at −bt
Solution. L = L e − e ds = − ds
t s s s+a s+b
h i∞ h s + a i∞
= log(s + a) − log(s + b) = log
s s+b s
s+b
= log .
s+a

Downloaded From : www.EasyEngineering.net


Downloaded From : www.EasyEngineering.net

Laplace Transform 533

eat − e−bt
Example 5.44. Find the Laplace transform of . [Jun 2012]
t
# Z∞ h Z∞
eat − e−bt
" !
at −bt
i 1 1
Solution. L = L e −e ds = − ds
t s−a s+b
s s
 s − a ∞
= log(s − a) − log(s + b) ∞
 
s = log
s+b s
s+b
= log .
s−a

ww
h e−3t sin 2t i
Example 5.45. Find L . [May 2007]
t
h e−3t sin 2t i Z ∞ h i Z ∞ h i
−3t
Solution. L = L e sin 2t ds = L sin 2t s→s+3 ds

w.E t
=
Zs ∞

s (s + 3) + 2
2
2 2
ds
s

asy1
= 2 tan−1
2
s + 3 ∞
2 s
π

En s + 3

= − tan−1
2 2

gin
 s + 3
= cot−1 .
2

Exercise 5 D
ee
Find the Laplace transform of the following functions.
rin
cos t − cos 2t 6.
sin2 t
. g.n[Dec 2007]
1.

2.
1 − e−t
t
.
t
.

7.
t
sin 3t cos t
t
. et
[Nov 2009]

cos 4t sin 2t
et − e2t 8. . [Dec 2009]
3. . t
t
eat − cos bt e−3t − e−4t
4. . 9. . [Jan 2009]
t t
1 − cos t cos 2t − cos 3t
5. . [Jun 2009] 10. .
t t

Downloaded From : www.EasyEngineering.net


Downloaded From : www.EasyEngineering.net

534 Engineering Mathematics - II

Laplace transform of periodic functions


A function f (t) is said to be periodic if there exists a positive constant T such that
f (t + T ) = f (t) for all t.The smallest of such T is called the period of the function.
Example. sin(t + 2π) = sin t. Sine function is a periodic function with period 2π.
Theorem. If f (t) is a periodic
Z Tfunction with period T , then
1
L[ f (t)] = e−st f (t)dt.
1 − e−sT 0
Proof. By the definition of Laplace transforms, we have
Z∞ ZT Z2T Z3T

ww L[ f (t)] =
0
e −st
f (t)dt =
0
e −st
f (t)dt +
T
e−st
f (t)dt +
2T
e−st f (t)dt + · · · (1)

w.E Consider
Z2T
e−st f (t)dt.

Let t = u + T .
T

asy
En
dt = du.
When t = T, u = 0.

Z2T
When t = 2T, u = T .
ZT gin ZT ZT
Now
T
−st
e f (t)dt =
0
e−s(u+T )

ZT
ee
f (u + T )du =
0
−su −sT
e e f (u + T )du = e −sT

rin 0
e−su f (u + T )du

= e−sT
0
e−su f (u)du[∴ f (u) is periodic in the period T ]
g.n
=e −sT
ZT

0
e−st f (t)dt. et
Z3T
In a similar way in the integral e−st f (t)dt if we assume t = u + 2T we arrive at
2T

Z3T ZT
−st −s2T
e f (t)dt = e e−st f (t)dt.
2T 0

Downloaded From : www.EasyEngineering.net


Downloaded From : www.EasyEngineering.net

Laplace Transform 535

If we replace t by u + 3T in the next integral we obtain

Z4T ZT
−st −s3T
e f (t)dt = e e−st f (t)dt.
3T 0

In the same way the subsequent integrals can be replaced by equivalent


quantities.
Substituting everything in (1) we obtain

ZT ZT ZT

ww L[ f (t)] =
0
e−st f (t)dt + e−sT
0
e−st f (t)dt + e−s2T
0
e−st f (t)dt + · · ·

w.E 
= 1 + e−sT + e−s2T + e−s3T + · · ·
Z

0
T

e−st f (t)dt

= 1+e

asy
−sT
+ (e −sT 2
) + (e −sT 3
) + ···
Z
T

e−st f (t)dt

En ZT
0

gin
1
= e−st f (t)dt.
1 − e−sT
0

ee✎

Worked Examples

Example 5.46. Find the Laplace transform of the rectangular wave function given

rin
 1



by f (t) = 
0<t<b
 −1 b < t < 2b
 with f (t + 2b) = f (t). [May 2009]
g.n
et

Solution. Since f (t + 2b) = f (t), it is a periodic function with period 2b.

Z2b
 b
Z2b

Z
1 1

−st  e−st f (t)dt + e−st f (t)dt
∴ L[ f (t)] = e f (t)dt =
1 − e−2bs 1 − e−2bs  
0 0 b
 b
Z2b

Z
1

−st −st 
=  e dt − e dt

1 − e−2bs
0 b
" −st   −st  #
1 e b e 2b
= −
1 − e−2bs −s 0 −s b

Downloaded From : www.EasyEngineering.net


Downloaded From : www.EasyEngineering.net

536 Engineering Mathematics - II

1  1 1 
= −2bs
− (e−bs − 1) + (e−2bs − e−bs )
1−e s s
1 
−bs −2bs −bs

= − e + 1 + e − e
s(1 − e−2bs )
1 
−bs −2bs

= 1 − 2e + e
s(1 − e−2bs )
1  
= −bs 2
1 − 2e−bs + (e−bs )2
s(1 − (e ) )
bs bs
(1 − e−bs )2 1 − e−bs e 2 − e− 2
!
1 bs
= = =   = tan h .
s(1 − e−bs )(1 + e−bs ) s(1 + e−bs ) s e bs2 + e− bs2 s 2

ww Example 5.47. Find the Laplace transform  of the square-wave function (or

1

 when 0 < t <
a

w.E

2 [Jun 2013]

Meoander function) of period a defined as f (t) =  a
−1 when < t < a.



2
Solution. Since f (t) is a periodic function of period a, we have

L[ f (t)] =
1
asy Za
e−st f (t)dt

En
1 − e−as
0
 a 
Za

gin
Z2 
1
 e−st f (t)dt + e−st f (t)dt
 
=
1 − e−as  
0 a

ee
2
 a 
Z2 Za

rin

1  −st −st 
=  e dt + e (−1)dt
1 − e−as  
0 a

g.n
2
 !a !a 
1  e−st 2 e−st 
=  −
1 − e−as −s 0

et

−s a 
2
1 h  a
−2 s

−as − a2 s
i
= − e − 1 + e − e
s(1 − e−as )
1 h
− 2a s −as
i
= 1 − 2e + e
s(1 − e−as )
1 − e− 2 s ✁
 a
2
=
✘e✘ −✘ a a
  
s ✘
1− 2s 1 + e− 2 s
a
1 − e− 2 s
=  a

s 1 + e− 2 s

Downloaded From : www.EasyEngineering.net


Downloaded From : www.EasyEngineering.net

Laplace Transform 537

as as
1 e 4 − e− 4
= · as as
s e 4 + e− 4
1  as 
= tan h .
s 4

Example  5.48. Find the Laplace transform of a square wave function given by
 a
ǫ for 0 ≤ t ≤



2

f (t) = 
 a
−ǫ for ≤ t ≤ a



2
and f (t + a) = f (t). [Dec 2011]

ww Solution. Since f (t + a) = f (t), f (t) is a periodic function of period a.

1
Za

w.E ∴ L[ f (t)] =
1 − e−as
0
 a
e−st f (t)dt

Za

asy
Z2 
1  −st −st 
=  e f (t)dt + e f (t)dt
1 − e−as  
0 a

En
2
 a 
Z2 Za 
1 −st −st

gin
 
=  e ǫdt + e (−ǫ)dt
1 − e−as  
0 a
2

ee
 !a −st a 
! 
ǫ  e−st 2 e 
=  −
1 − e−as  −s 0

−s a 

=
ǫ h  sa 
2

− e− 2 − 1 + e−as − e− 2
as i

rin
g.n
s(1 − e ) −as
ǫ h
− as
i
= 1 − 2e 2 + e−as
s(1 − e−as )

= 
ǫ 1 − e− 2 ✁


s 1 + e− 2 ✘
as

as 2

1−


✘e✘−✘as
2
 et
− as
 
ǫ 1 − e 2
=
s 1 + e− as2
 

as as
ǫ e 4 − e− 4
= · as as
s e 4 + e− 4
ǫ  as 
= tan h .
s 4

Downloaded From : www.EasyEngineering.net


Downloaded From : www.EasyEngineering.net

538 Engineering Mathematics - II


ǫ 0≤t≤a



Example 5.49. Find the Laplace transform of f (t) = 



−ǫ
 a ≤ t ≤ 2a
and f (t + 2a) = f (t) for all t. [Dec 2010]
Solution. Since f (t + 2a) = f (t) for all t, f (t) is a periodic function with period 2a.

Z2a
1
∴ L[ f (t)] = e−st f (t)dt
1 − e−2as
0
 a
Z2a

Z
1


ww = e−st
f (t)dt + e−st
f (t)dt
 
1 − e−2as
 
0 a

w.E
 a
Z2a

Z
1

−st −st
=  e ǫdt + e (−ǫ)dt
 
1−e −2as
0 a

asy

−st
!a −st
!2a 
ǫ  e
 e 
= − 
1 − e−2as −s 0

 −s a 
ǫ

En
h i
−as  −2as −as
= − e − 1 + e − e
s(1 − e−2as )

gin
 
ǫ 1 − 2e−as + e−2as
=
s (1 − e−as ) (1 + e−as )

=
s ✘1−
ǫ 1 − e−as ✁2
✘e✘

ǫ 1 − e−as
✘
−as ee


(1 + e−as )
 as
ǫ e2 −e 2
− as

rin
= = 
s (1 + e−as ) s e as2 + e− as2
ǫ  as 


g.n
= tan h
s 2
.

Example 5.50. Find the Laplace transform of the triangular wave function
et
 t 0≤t≤a



defined by f (t) =  and f (t) is of period 2a. [May 2015,May
 2a − t a < t ≤ 2a


2011]
Solution. Since f (t) is of period 2a,we have
Z 2a
1
L[ f (t)] = e−st f (t)dt
1 − e−2as 0

Downloaded From : www.EasyEngineering.net


Downloaded From : www.EasyEngineering.net

Laplace Transform 539

"Z a Z 2a #
1 −st −st
= e f (t)dt + e f (t)dt
1 − e−2as 0 a
1 hZ a Z 2a i
−st −st
= e tdt + e (2a − t)dt
1 − e−2as 0 a
Z a −st Z 2a
1 h e e−st i
= td( ) + (2a − t)d( )
1 − e−2as 0 −s a −s
1 h e−st a Z a e−st  e−st 2a
Z 2a −st
e i
= −2as
t − dt + (2a − t) − (−dt)
1−e −s 0 0 −s −s a a −s
1 h e −as 1 e −st  a a 1 e −st  2a i
= a + + 0 + e−as −
1 − e−2as −s s −s 0 s s −s a

ww =
1−e
1
−2as
h
−a
e−as
s
e−as
1 −as
− 2 (e − 1) + e + 2 (e
s
a −as 1 −2as
s
e−as
s
− e−as )
i

w.E
1 h 1 −as 1 1 1 i
= −2as
−a − 2e + 2 + a + 2 e−2as − 2 e−as
1−e s s s s s s
1 h e−2as − e−as − e−as + 1 i 1 h 1 − 2e−2as + e−2as i
= =

=
1 − e−2as
1
asy
(1 − e−as )(1 + e−as )
.
s2
(1 − e−as )2
s2
=
1 − e−as
(1 − e−as )
s2 .(1 + e−as )
s2

=
as
e2 −e 2 − as

En 1
 = 2 tan h
 as 
.

gin
 as as
s2 e 2 + e− 2 s 2

t




by f (t) = 

0≤t≤π
2π − t π ≤ t ≤ 2π


ee
Example 5.51. Find the Laplace transform of the triangular wave function given

rin
and f (t + 2π) = f (t). [Jun 2012, Jun 2010]
Solution. Since f (t + 2π) = f (t) for all t, f (t) is a periodic function with period 2π. g.n
In the previous Example, we get the solution by replacing 2a by 2π
i.e., a = π.
1 π 
et
∴ L[ f (t)] = tan h s.
s2 2

Example 5.52. Find  the Laplace transform of the half-sine wave rectifier function
π
 sin ωt 0 < t < ω



defined by f (t) =  [Jun 2014, Dec 2012]
π
 0 < t < 2π .


ω ω
Solution. f (t) is defined in the interval (0, 2π
ω)

Downloaded From : www.EasyEngineering.net


Downloaded From : www.EasyEngineering.net

540 Engineering Mathematics - II

2π 2π
) = sin ω(t + ) = sin(ωt + 2π) = sin ωt = f (t)
f (t +
ω ω

∴ f (t) is periodic with period T = .
Z T ω Z 2π
1 −st 1 ω
We know that,L[ f (t)] = f (t)e dt = e−st sin ωtdt
1 − e−sT 0 s2π
1 − e− ω 0
1 h e−st i ωπ
= s2π 2 2
[−s sin ωt − ω cos ωt]
1 − e− ω s + w 0
−sπ
1 h eω −ω i
= 2 2
[−s.0 − ω(−1)] − 2
s2π
1 − e− ω s + w ω + s2

ww 1 n −sπ o
= s2π 
ωe ω + ω
1 − e− ω (s2 + ω2 )

ω(1 + e− ω )

w.E =
(s2 + ω2 )(1 − e
−sπ
ω )(1 + e
−sπ
ω )
=
ω
(s2 + ω2 )(1 − e
−sπ
ω )
.

t asy
Example 5.53. Find the Laplace transform of the saw-toothed wave function of
period T given by f (t) = , 0 < t < T .
T
En
Solution. Since f (t)is a periodic function of period T, we have

L[ f (t)] =
1 gin Z T
e−st f (t)dt

=
1 − e−sT 0

1−e
1
−sT
ee
Z T

0
e−st t
T
dt =
1
−sT
T (1 − e ) 0
Z T  −st 
td
e
−s
rin
g.n
−st
!T Z T −st i
1 h te e
= −sT
− dt
T (1 − e ) −s 0 0 −s

=
1

sT (1 − e−sT )
h T e−sT 1  e−st T i
T (1 − e−sT ) −s
1 h
− T e
+

−sT
s −s 0

1 −sT
s
(e − 1)
i et
1 h 1 1 −sT −sT
i
= − e − T e
sT (1 − e−sT ) s s
1 h 1 − e−sT i
= −sT
− T e−sT
sT (1 − e ) s
1 e−sT
= − .
Ts 2 s(1 − e−sT )

Downloaded From : www.EasyEngineering.net


Downloaded From : www.EasyEngineering.net

Laplace Transform 541

Exercise 5 E

1. Find the Laplace transform of the periodic function f (t) = t for 0 < t < 4 and
f (t + 4) = f (t). [Dec 2009]

 t


 0<t<1
2. Find the Laplace transform of the periodic function f (t) = 
 2 − t 1 < t < 2.


[Oct 2001]

 t

 0 < t < π2

ww

3. Find the Laplace transform of the function f (t) =  and
 π−t π <t <π


2
f (π + t) = f (t). [May 2009]

w.E 4. Find the Laplace transform of the function f (t) = 



 t 0≤t≤3



 0 3≤t≤6
 and

asy

f (t + 6) = f (t). [May 2006]

En

 k 0≤t≤a



5. Find the Laplace transform of the function f (t) =  and
 −k 3 ≤ t ≤ 2a


f (2a + t) = f (t).
gin [Dec 2008]

ee
6. Find the Laplace transform of the periodic saw - tooth wave function given
αt
by f (t) = , 0 < t < ω and f (t) = f (t + ω).
ω
rin [May 2007]

g.n

 cos t 0 < t < π



7. Find the Laplace transform of the periodic function f (t) = 
 0 π < t < 2π


and f (t + 2π) = f (t).

 0



8. Find the Laplace transform of f (t) = 
0 < t < ωπ
et
[May 2004]

and f (t+2π) = f (t).


 − sin ωt π < t < 2π


ω ω
[May 2003]

9. Find the Laplace transform of the periodic function defined by


t


 a
 0≤t≤a
f (t) =  and f (t + 2a) = f (t). [May 2007]
 2a−t a ≤ t ≤ 2a


a

Downloaded From : www.EasyEngineering.net


Downloaded From : www.EasyEngineering.net

542 Engineering Mathematics - II

5.3 Laplace transform of derivatives

Theorem 1. Let f (t) be a continuous function for all t ≥ 0 and f ′ (t) is piecewise
continuous on every finite interval 0 ≤ t ≤ a in [0, ∞). If f (t) and f ′ (t) are of
exponential order as Zt → ∞, then L[ f Z ′ (t)] = sL[ f (t)] − f (0).
∞ ∞
h i d
Solution. L f ′ (t) = e−st f ′ (t)dt = e−st

f (t) dt
dt
Z0 ∞ 
0
∞ Z ∞
= e−st d( f (t)) = e−st f (t) − f (t)e−st (−s)dt
0 0 0 .

ww
Z ∞
−st
= 0 − f (0) + s e f (t)dt = sL[ f (t)] − f (0).
0

w.E
h dy i
∴L = sL[y] − y(0).
dt

Theorem 2. Find L fZ′′ (t) .


h
′′
i
Solution. L f (t) =




−st ′′ asy
e f (t)dt =
Z ∞
e −st d



f (t) dt =
Z ∞ 
e−st d f ′ (t)



0
−st ′
= e f (t) −
∞ Z ∞
En 0
′ −st
dt
f (t)e (−s)dt
0


= 0 − f (0) + s
0 0
Z ∞
gin
e−st f ′ (t)dt


ee ′
0

= − f (0) + sL[ f (t)] = − f ′ (0) − s[sL[ f (t)] − f (0)]

= − f ′ (0) + s2 L[ f (t)] − s f (0)


rin
h
′′
i
2
∴ L f (t) = s L[ f (t)] − s f (0) − f (0). ′

g.n
Extension

L[ f ′′′ (0)] = s3 L[ f (t)] − s2 f (0) − s f ′ (0) − f ′′ (0).


et
In general L[ f (n) ] = sn L[ f (t)] − sn−1 f (0) − sn−2 f ′ (0) − · · · − s f (n−2) (0) − f (n−1) (0).

Note. The above results will be used later in solving differential equations
using Laplace transforms.

Downloaded From : www.EasyEngineering.net


Downloaded From : www.EasyEngineering.net

Laplace Transform 543

5.3.1 Laplace transform of an integral

If f (t) is piecewise continuous in every finite interval 0 ≤ t ≤ a in [0, ∞) and f (t)


is of exponential order α > 0 and if L[ f (t)] = F(s) then
"Z t #
F(s) L[ f (t)]
L f (t)dt = = for s > α.
0 s s
Zt
Proof. Let g(t) = f (u)du.
0

ww Then g′ (t) = f (t) and g(0) =


Z0
f (u)du = 0.

w.E
0
Now, L[ f (t)] = L[g′ (t)] = sL[g(t)] − g(0)
L[ f (t)]
= L[g(t)].
 st

asy

Z
 L[ f (t)]

i.e., L  f (u)du =

  s
 t
Z
0 
En
 L[ f (t)]

i.e., L  f (t)dt = .

gin

  s
0
t 

ee
R
 t t  L  f (t)dt
Z Z
L[ f (t)]

0
Result. L  f (t)dtdt = = .

rin
 
  s s2
0 0


Worked Examples


g.n
t

et
h Z i
Example 5.54. Find L e−t t cos tdt . [May 2005]
0
h Z t i hZ t i "
L[t cos t]
#
−t
Solution. L e t cos tdt = L t cos tdt =
0 0 s→s+1 s s→s+1
1  d n o
= − L(cos t)
s+1 ds s→s+1
1 d n s+1 o
=−
s + 1 ds s2 + 2s + 2
1 n (s2 + 2s + 2) − (s + 1)(2s + 2) o
=−
s+1 (s2 + 2s + 2)2

Downloaded From : www.EasyEngineering.net


Downloaded From : www.EasyEngineering.net

544 Engineering Mathematics - II

1 n (s2 + 2s + 2) − 2(s + 1)2 o


=−
s+1 (s2 + 2s + 2)2
1 n s + 2s + 2 − 2s2 − 4s − 2) o
2
=−
s+1 (s2 + 2s + 2)2
1 n −s2 − 2s o
=−
s + 1 (s2 + 2s + 2)2
s(s + 2)
= .
(s + 1)(s2 + 2s + 2)2
h Z t e−t sin t i
Example 5.55. Find L dt . [May 2008]

ww 0 t
h −t i
h Z t e−t sin t i L e tsin t 1 ∞
Z
Solution. L dt = = L[e−t sin t]ds

w.E 0 t
=
1 ∞
s s
Z
s s s

L[sin t] s→s+1 ds

asy
Z ∞
1 1
= ds
s s (s + 1)2 + 1
1 h −1

En
i∞
= tan (s + 1)
s s
1hπ

gin
i
= − tan−1 (s + 1)
s 2
1 −1

Example 5.56. Find L


h t
Z
= cot (s + 1).
s

0
i
te−t sin tdt .
ee rin [Jun 2005]

Solution. L
hZ t i L[te−t sin t] 1  d 
te−t sin tdt = = − L[e−t sin t] g.n
et
0 s s ds
1d n o
=− L[sin t] s→s+1
s ds
1d h 1 i
=−
s ds (s + 1)2 + 1
1d n 2 o
=− (s + 2s + 2)−1
s ds
1
= − (−1)(s2 + 2s + 2)−2 (2s + 2)
s
2(s + 1)
= .
s(s + 2s + 2)2
2

Downloaded From : www.EasyEngineering.net


Downloaded From : www.EasyEngineering.net

Laplace Transform 545

Z t
h
−t sin t i
Example 5.57. Find L e dt .
0 t
h Z t sin t i h Z t sin t i h 1 h sin t ii
Solution. L e−t dt = L dt = L
0 t 0 t s→s+1 s t s→s+1
Z ∞
1 h i
= L[sin t]ds
s+1 s s→s+1
Z ∞
1 h 1 i
= 2
ds
s + 1 s s + 1 s→s+1
1 h −1 ∞ i 1
= tan (s) s = cot−1 (s + 1).
s+1 s+1

ww
s→s+1

5.3.2 Evaluation of integrals

w.E Certain real integrals can be evaluated by comparing the definition of the
Laplace transform. The following examples illustrate this.

asy ✎

Worked Examples

Example 5.58. Evaluate


Z

En
e−t cos 2tdt.

gin
Z ∞0

Solution. We know that L[cos 2t] = e−st cos 2tdt.


0

Z ∞ integral we observe that


e−t cos 2tdt = L[cos 2t] s=1 =
ee
Comparing the definition of the Laplace transform with the
given
s 
=
1 1
= .
s2 + 4 s=1 1 + 4 5 rin
g.n
0
Z ∞
Example 5.59. Evaluate e−2t sin 3tdt.
0 Z

Solution. We have L[sin 3t] =



e−st sin 3tdt.
0
Comparing the definition of the Laplace transform with the given
et
integral,
Z ∞ we observe that " #
−2t 3 3 3
e sin 3tdt = L[sin 3t] s=2 = 2
= = .
0 s +9 s=2 4 + 9 13
R∞
Example 5.60. Find te−2t cos tdt using Laplace transform [ Jun 2012, Dec 2011]
0
Solution. Comparing the definition of the Laplace transform and the given

Downloaded From : www.EasyEngineering.net


Downloaded From : www.EasyEngineering.net

546 Engineering Mathematics - II

integral, we observe that


Z∞
d
te−2t cos tdt = L[t cos t] s=2 = −
[L(cos t)] s=2
ds
0
" 2
1 − s2
# " #
d  s  s + 1 − s · 2s
=− =− =− 2
ds s2 + 1 s=2 (s2 + 1)2 s=2 (s + 1)2 s=2
" 2 #
s −1 4−1 3
= = = .
(s2 + 1)2 s=2 (4 + 1)2 25
Z ∞

ww Example 5.61. Evaluate e−2t t sin 3tdt.


0
Solution. Comparing the definition of the Laplace transform with the given
[May 2004]

w.E integral
Z ∞

0
we observe that
h d
e−2t t sin 3tdt = L[t sin 3t] s=2 = − L[sin 3t]
ds
i
s=2

=−
h 6s i asy
d  3 
2
ds s + 9 s=2
h
= − 3(−1)(s2 + 9)−2 2s
12 12
i
s=2

=
En =
(s2 + 9)2 s=2 132 169
= .

gin
Z ∞
Example 5.62. Evaluate e−3t t sin tdt.
0
Solution. Comparing the definition of the Laplace transform with the given
integral
Z ∞

0
−3t
we observe that
h d ee
e t sin tdt = L[t sin t] s=3 = − L(sin t)
ds
i
s=3
=−
d h 1 i
ds s2 + 1 s=3 rin
h
= − (−1)(s2 + 1)−2 2s
i
s=3
=
h 2s i
2
(s + 1) 2 s=3
=
6
= .
100 50
3
g.n
Example 5.63. Evaluate
Z ∞

0
cos at − cos bt
t
dt.
Solution. Comparing the definition of the Laplace transform with the given
[May 2015]
et
integral we observe that
Z ∞ hZ ∞
cos at − cos bt h cos at − cos bt i i
dt = L = L[cos at − cos bt]ds
0 t t s=0 s s=0
hZ ∞ s s  i h 1 h 2 2
s + a ∞ii
= − ds = log
s s2 + a2 s2 + b2 s=0 2 s2 + b2 s s=0
1h s2 + b2 i 1 b2 b
= log 2 2
= log 2
= log .
2 s +a s=0 2 a a

Downloaded From : www.EasyEngineering.net


Downloaded From : www.EasyEngineering.net

Laplace Transform 547


e−3t − e−6t
Z
Example 5.64. Evaluate dt.
0 t
Solution. Comparing the given integral and the definition of the Laplace
transform
Z ∞ −3t we get
e − e−6t e−3t − e−6t hZ ∞ i
dt = L[ ] s=0 = L(e−3t − e−6t )ds
0 t t s s=0
hZ ∞ 1 1  i
= − ds
s s + 3 s + 6 s=0
h s+3 ∞  i
= log
s + 6 s s=0

ww s + 6i
h
= log = log 2.
s + 3 s=0
Z ∞
sin2 t

w.E
Example 5.65. Evaluate

Z we
Solution.

notice that
2
−t sin t
0
e−t

h sin2 t i
t
dt.

1 h 1 − cos 2t i h1 Z ∞ i
0
e
t
dt = L

= asy
t s=1 2
h1 Z ∞ 1
( − 2
= L
s
)ds
i
t
=
h
s=1
1 h
=
2 s
L[1 − cos 2t]ds
p
log s − log s2 + 4
i ∞ i
s=1

h1h
En
2 s s s +4
s i∞ i
s=1

1h
2

s2 + 4 i
s s=1

1
2
log √
√ gin
= log 5 = log 5.
s2 + 4 s s=1 2
1
= log
s s=1

Example 5.66. Evaluate


Z We
Solution. have
2
Z ∞

0
e−t ee 4
1 − cos t
t
dt.
rin
0

e−t 1 − cos t
t
dt = L
h 1 − cos t i
t s=1
=
hZ ∞
s
L[1 − cos t]ds
i
s=1
g.n
et
hZ ∞ i Z ∞h i
= L[1] − L[cos t]ds = L[1] − L[cos t]ds
s s=1 s s=1
Z ∞ h1 s i h p i
= − 2 ds = log s − log s2 + 1
s s s + 1 s=1 s=1

h s ∞ i h s2 + 1 i
= log √ = log
s2 + 1 s s=1 s s=1
√ 1
= log 2 = log 2.
2
Z ∞ √
sin 3t
Example 5.67. Evaluate e−t dt.
0 t

Downloaded From : www.EasyEngineering.net


Downloaded From : www.EasyEngineering.net

548 Engineering Mathematics - II


√ √
Z hZ ∞ h √ i i
3t−t sin
h h sin 3t ii
Solution. We have e dt = L = L sin 3t ds
√ 0 t t s=1 s s=1
Z ∞ h√ 1
h 3 i  s ∞ i
= 2
ds = 3 √ tan−1 √
s s + 3 s=1 3 3 s s=1
hπ s i π 1 π π
= − tan−1 √ = − tan−1 √ = −
2 3 s=1 2 3 2 6
3π − π 2π π
= = = .
6 6 3

ww Initial value theorem.


L[ f (t)] = F(s), then lim f (t) = lim sF(s).
t→0
If the Laplace transform of f (t) and f ′ (t) exist and

s→∞

w.E
Proof. By Laplace transform of derivative of f (t) we have
L[ f ′ (t)] = sL[ f (t)] − f (0).
Z∞

0 asy
e−st f ′ (t)dt = sF(s) − f (0).

Taking limit as s → ∞ we obtain.


Z∞
En
e−st f ′ (t)dt = lim sF(s) − f (0).
lim
s→∞

Z∞
0
s→∞

gin
0
s→∞

0 = lim sF(s) − f (0).


s→∞
s→∞
ee
lim e−st f (t)dt = lim sF(s) − f (0).

rin
f (0) = lim sF(s).
s→∞
lim f (t) = lim sF(s). g.n
et
t→0 s→∞
Final value theorem. If the Laplace transform of f (t) and f ′ (t) exist and
F(s) = L[ f (t)], then lim f (t) = lim sF(s). [Dec 2014]
t→∞ s→0
Proof. We have L[ f ′ (t)]
= sL[ f (t)] − f (0).
Z∞
ie., e−st f ′ (t)dt = sF(s) − f (0).
0
Taking limit as s → 0 we obtain.
Z∞
lim e−st f ′ (t)dt = lim sF(s) − f (0).
s→0 s→0
0

Downloaded From : www.EasyEngineering.net


Downloaded From : www.EasyEngineering.net

Laplace Transform 549

Z∞
lim e−st f ′ (t)dt = lim sF(s) − f (0).
s→0 s→0
0
Z∞
f ′ (t)dt = lim sF(s) − f (0).
s→0
0
[ f (t)]∞
0 = lim sF(s) − f (0).
s→0
f (∞) − f (0) = lim sF(s) − f (0)
s→0
f (∞) = lim sF(s)
s→0
∴ lim f (t) = lim sF(s).

ww t→∞ s→0

✎ ☞

w.E Worked Examples


✍ ✌

Example 5.68. Verify the initial value theorem for the function 2 + 3 cos t.
Solution. f (t) = 2 + 3 cos t
asy
1 s En
L[ f (t)] = L[2 + 3 cos t] = 2L[1] + 3L[cos t]

=2 +3 2
2
s
= + 2
s +1
3s
= F(s) gin
t→0
s s +1

2 3s
!
ee
lim f (t) = lim(2 + 3 cos t) = 2 + 3 cos 0 = 2 + 3 = 5.
t→0
3s2
!
rin
g.n
∴ lim sF(s) = lim s + 2 = lim 2 + 2
s→∞ s→∞ s s +1 s→∞ s +1
 
 s2 
= 2 + 3 lim  
s→∞ s2 1 + 1

 1 
= 2 + 3 lim 
s→∞ 1 + 1 

s2
 

 = 2 + 3 = 5.
et
s2
∴ lim f (t) = lim sF(s).
t→0 s→∞

Example 5.69. Verify the initial and final value theorems for the function
f (t) = ae−bt . [Jun 2013, May 1997]
Solution. Let f (t) = ae−bt .

Downloaded From : www.EasyEngineering.net


Downloaded From : www.EasyEngineering.net

550 Engineering Mathematics - II

a
L[ f (t)] = = F(s)
s+b
lim f (t) = lim ae−bt = a
t→0 t→0
as as
lim sF(s) = lim = lim =a
s→∞ s→∞ s + b s→∞ s(1 + bs )

∴ lim f (t) = lim sF(s)


t→0 s→∞

lim f (t) = lim ae−bt = 0


s→∞ t→0

lim f (t) = lim ae−st = 0

ww t→∞ t→∞
lim sF(s) = lim 2
s→0
as
s→0 s + a2
=0

w.E ∴ lim f (t) = lim sF(s).


t→∞ s→0

Example 5.70. Verify the initial value theorem for the function
f (t) = 1 + e−t (sin t + cos t).
asy
−t
Solution. Let f (t) = 1 + e (sin t + cos t).
[Jun 2012, Dec 2010, Jun 2010]

En
L[ f (t)] = L[1 + e−t (sin t + cos t)]

gin
= L[1] + L[e−t (sin t + cos t)]
1
=

=
s
1
s
+L 2
"
1
+ee
+ L[sin t + cos t] s→s+1
s
#

s + 1 s2 + 1 s→s+1
rin
g.n
" #
1 s+1
= + 2
s s + 1 s→s+1
F(s) =
1
s
+
s+2
(s + 1)2 + 1
lim f (t) = lim[1 + e−t (sin t + cos t)] = 1 + 1 = 2.
t→0 t→0
" #
et
s(s + 2)
lim sF(s) = lim 1 +
s→∞ s→∞ (s + 1)2 + 1
s2 (1 + 2s ) 1
= 1 + lim = 1 + = 2.
s→∞ s2 (1 + 1 )2 + 1 1
s s 2

∴ lim f (t) = lim sF(s).


t→0 s→∞

Downloaded From : www.EasyEngineering.net


Downloaded From : www.EasyEngineering.net

Laplace Transform 551

Example 5.71. Verify the final value theorem for the function 1 − e−3t .
[May 2006]
−3t
Solution. Let f (t) = 1 − e .
1 1
L[ f (t)] = L[1 − e−3t ] = L[1] − L[e−3t ] = − .
s s+3
lim f (t) = lim [1 − e−3t ] = 1 − 0 = 1.
" t→∞t→∞ #
1 1  s  1
lim sF(s) = lim s − = lim 1 + = 1 − lim 3
= 1 − 0 = 1.
s→0 s→0 s s+3 s→0 s+3 s→0 1+ s

ww
∴ lim f (t) = lim sF(s).
t→∞ s→0

Example 5.72. Verify the initial and final value theorems for the function

w.E
f (t) = e−t sin t.
Solution. f (t) = e−t sin t

t→0 asy
lim f (t) = lim e−t sin t = 0
t→0
(1)

t→∞
En
lim f (t) = lim e−t sin t = 0
t→∞
(2)

L[ f (t)] =
gin
1
(s + 1)2 + 1
s
= F(s)
s
lim sF(s) = lim 2
s→∞

lim sF(s) = lim 2


s→0
ee
s→∞ s + 2as + 2

s
s→0 s + 2as + 2
= lim

=0
s→∞ s2 1 + 2as +
s
2
s2
=0

rin
(3)

(4)

(1) = (3) =⇒ Initial value theorem is verified.


g.n
(2) = (4) =⇒ Final value theorem is verified.

Exercise 5 F
et
I. Evaluate the following integrals using Laplace transforms
Z ∞ Z ∞ Z ∞
−t −2t cos 6t − cos 4t
1. te sin tdt. 2. e t sin 3tdt. 3. dt.
0 0 0 t

Downloaded From : www.EasyEngineering.net


Downloaded From : www.EasyEngineering.net

552 Engineering Mathematics - II

∞ √
e−t − e−3t ∞
e−t sin
Z Z
3t
Z ∞ 5. dt. 6. dt.
4. t3 e−t sin tdt. 0 t 0 t
0

II. Prove the initial and final value theorems for the following functions.

4. t2 e−3t .
1. 3e−t . [Dec 2011]

ww 2. (2t − 3)2 .
5. e−t (t + 2)2 .

6. If F(s) =
3s2 + 5s + 2

w.E 3. 1 − e−at .
s3 + 4s2 + 2s
find f (0) and f (∞).

Unit Step Function. asy


The unit step function u is defined as

En

 0 if t < 0



u(t) = 
 1 if t ≥ 0.

gin

u(t) has jump discontinuity at t = 0.
 0
 if t < a

ee


More generally we define u(t − a) = 
 1 if t ≥ a.


Laplace transform of unit step function
rin [Jun 2010]

L[u(t − a)] =
Z

Z0 a

e−st u(t − a)dt =
0
Z a
e−st u(t − a)dt +
Z

a

g.n
e−st u(t − a)dt

et
Z ∞  e−st ∞
= e−st .0dt + e−st dt = 0 +
0 a −s a
−1 e −as
= [0 − e−as ] = if s > 0.
s s

Second Shifting property


If L[ f (t)] = F(s), then −as −as
 L[ f (t − a)u(t − a)] = e F(s) = e L[ f (t)].
 0 if t < a



Proof. We have u(t − a) = 
 1 if t ≥ a.

Downloaded From : www.EasyEngineering.net


Downloaded From : www.EasyEngineering.net

Laplace Transform 553


 0


 if t < a
Now f (t − a)u(t − a) = 
 f (t − a) if t ≥ a.

Z∞
∴ L[ f (t − a)u(t − a)] = e−st f (t − a)u(t − a)dt
0
Za Z∞
= e−st f (t − a)u(t − a)dt + e−st f (t − a)u(t − a)dt
0 a
Za Z∞

ww =
0
e−st 0dt +
a
e−st f (t − a)dt

w.E Let t − a = x

dt = dx
When t = a, x = 0.

When t = ∞, x = ∞.

∴ L[ f (t − a)u(t − a)] =
asy Z∞
e−s(a+x)
f (x)dx =
Z∞
−as−sx
e f (x)dx =
Z∞
e−as e−sx f (x)dx

En
0

= e−as
Z∞
0

e−sx f (x)dx = e−as


Z∞
0

e−st f (t)dt = e−as F(s).

gin 0 0

ee
Note. The second shifting property can be stated as follows.

rin
If L[ f (t)] = F(s), then L[ f (t − a)u(t − a)] = e−as L[ f (t)] where u(t − a) is the unit step
function.

The unit impulse  function. For any positive ǫ, the impulse function δǫ is g.n
1
 ǫ (u(t) − u(t − ǫ)), 0 ≤ t < ǫ
et



defined as δǫ (t) = 
 0, Otherwise


Dirac delta function. lim δǫ (t) is called the Dirac delta function, denoted by δ(t).
ǫ→0

 0, t , 0



∴ δ(t) = 
 ∞, t = 0



 0, t , a



In general δ(t − a) = 
 ∞, t = a.

Downloaded From : www.EasyEngineering.net


Downloaded From : www.EasyEngineering.net

554 Engineering Mathematics - II

Results. 1. Find L δǫ (t − a) .
 

Solution
By the definition, δǫ (t − a) = 1ǫ (u(t − a) − u(t − a − ǫ)) if a ≤ t ≤ a + ǫ [0 ≤ t − a < ǫ].
 1h i 1h i
Now, L δǫ (t − a) = L[u(t − a) − u(t − a − ǫ)] = L[u(t − a)] − L[u(t − a) − ǫ]

ǫ ǫ
1h i 1 h e−as e−(a+ǫ)s i
= L[u(t − a)] − L[u(t − (a + ǫ)] = −
ǫ ǫ s s
1 h e−as e−as .e−ǫ s i
= −
ǫ s s
 1 −as

ww
−ǫ s
L δǫ (t − a) = e (1 − e )

ǫs
Taking limit as ǫ → 0 we obtain
1 − e−ǫ s
L lim δǫ (t − a) = e−as lim

w.E
 
ǫ→0 ǫ s→0 ǫs
1 − ex
L δ(t − a) = e−as .1
   
lim =1
x→0 x
= e−as .
2. When a = 0, L[δ(t)] = 1. asy
En ∞
Z
3. One important property of Dirac Delta funtion is f (t)δ(t − a)dt = f (a).
0

gin


Worked Examples

Example 5.73. If f (t) = 






 3, When t > 2


ee
 0, When 0 < t < 2
find L[ f (t)]. [May 2007]
rin
Solution. L[ f (t)] =
Z∞
−st
e f (t)dt =
Z2
−st
Z∞
−st
e f (t)dt + e f (t)dt =
Z∞
e−st 3dt
g.n
=3
e−st
−s
0


!∞

2
3
0

= − [0 − e−2s ] =
s
3e−2s
s
.
2 2

et
t
 e, 0<t<1



Example 5.74. If f (t) =  find L[ f (t)]. [Dec 2009]
 0, t > 1

Z∞ Z1 Z∞ Z1
Solution. L[ f (t)] = −st
e f (t)dt = −st
e f (t)dt + −st
e f (t)dt = e−st et dt
0 0 1 0

Downloaded From : www.EasyEngineering.net


Downloaded From : www.EasyEngineering.net

Laplace Transform 555

Z1 Z1 !1
et(1−s)
= e −st+t
dt = et(1−s) dt =
1−s 0
0 0
1 e1−s − 1
= [e1−s − 1] = .
1−s 1−s

 sin 2t, 0 < t < π



Example 5.75. Find L[ f (t)] if f (t) =  [Apr 2010]
 0, t > π.


Z∞ Zπ Z∞
−st
Solution. L[ f (t)] = e f (t)dt = −st
e f (t)dt + e−st f (t)dt.

ww 0 0 π

w.E =

0
e−st
sin 2tdt = e −st
 −s sin 2t − 2 cos 2t !π
s2 + 4 0

=
s2
+4
2
asy
e−sπ
(−2) +
s2
2
+4
= 2
s +4
En
(1 − e−πs ).

 


gin
 cos t − 3 , t > 3



Example 5.76. If f (t) = 

find L[ f (t)]. [Apr 2010]
 0,

ee
t < 2π


3
  
2π 2π
 cos t − 3 , t > 3

rin
 2π  


Solution. Let g t − = f (t) =  .
3

 0,
 t < 2π
3

" 
Now L[ f (t)] = L g t −
2π 
#
−2πs −2πs −2π
= e 3 L[g(t)] = e 3 L[cos t] = e 3 s 2
s
. g.n
3

Example 5.77. Find L[(t − 1)2 u(t − 1)].


2
s +1

2e−s
et
Solution. L[(t − 1)2 u(t − 1)] = e−s L[t2 ] = e−s = .
s3 s3

Example 5.78. Find L[e−4t u(t − 1)].


Solution. L[e−4t u(t − 1)] = L[e−4(t−1+1) u(t − 1)] = L[e−4(t−1)−4) u(t − 1)]
1 e−(4+s)
= e−4 e−s L[e−4t ] = e−4 e−s = .
s+4 s+4

Downloaded From : www.EasyEngineering.net


Downloaded From : www.EasyEngineering.net

556 Engineering Mathematics - II

π
Example 5.79. Find L[sin tu(t − )].
2
π π π π h π π i
Solution. L[sin tu(t − )] = L[sin t − + u(t − )] = L cos(t − )u(t − )
2 2 2 2 2 2
−πs
−πs −πs s se 2
= e 2 L[cos t] = e 2 2 = .
s + 1 s2 + 1

Exercise 5 G

Find the Laplace transform of the following functions.

ww 

 1, 0 < t < 1



6. f (t) = 

w.E  t + 1, 0 ≤ t ≤ 2



1. f (t) =   0, t > 1.


 3, t > 2.



0 < t < 12

asy  t,
 


 sin t, 0 < t < π 7. f (t) = 



2. f (t) =   t − 1, 1 < t < 1.


2
 0, t > π.



 cos t, 0 < t < π


En 
 t, 1 < t < 2



8. f (t) = 

gin

3. f (t) =   0, otherwise.


 sin t, t > π.

ee
 
 t − 1, 1 < t < 2

  0,


 0<t<1
9. f (t) = 

4. f (t) = 

rin
 1 + 5t, t ≥ 1.

 3 − t, 2 < t < 3.

 

g.n
 
2 t
 (t − 1) , t > 1  T, 0 < t < T


 


5. f (t) =  10. f (t) = 
 0, 0 < t < 1.  1, t > T.

 

5.4 Inverse Laplace Transforms


et
Definition. If the Laplace transform of a function f (t) is F(s), then f (t) is called
the inverse Laplace transform of F(s).
i.e., If L[ f (t)] = F(s) then L−1 [F(s)] = f (t).

Downloaded From : www.EasyEngineering.net


Downloaded From : www.EasyEngineering.net

Laplace Transform 557

Standard Results
1 h1i
1. L[1] = ⇒ L−1 = 1.
s s
1 h 1 i
2. L[eat ] = ⇒ L−1 = eat .
s−a s−a
1 h 1 i
3. L[e−at ] = ⇒ L−1 = e−at .
s+a s+a
1 h1i
4. L[t] = 2 ⇒ L−1 2 = t.
s s

ww 5.
n! h n! i
L[tn ] = n+1 ⇒ L−1 n+1 = tn .
s s

w.E 6. L[cosh at] = 2


s
s −a
a
2
⇒ L−1 2
h s i
s − a2
h a i
= cosh at.

asy
−1
7. L[sinh at] = 2 ⇒ L = sinh at.
s − a2 s2 − a2
s h s i
8. ⇒ L−1 2

En
L[cos at] = 2 2
= cos at.
s +a s + a2
a h a i

gin
−1
9. L[sin at] = 2 ⇒ L = sin at.
s + a2 s2 + a2
2as h 2as i

ee
−1
10. L[t sin at] = 2 ⇒ L = t sin at.
(s + a2 )2 (s2 + a2 )2

11. L[t cos at] =


s2 − a2
(s2 + a2 )2
⇒ L
h 2
−1 s − a
2 i

(s2 + a2 )2
= t cos at.
rin
Basic theorems
g.n
1. L−1 [aF(s) + bG(s)] = aL−1 [F(s)] + bL−1 [G(s)] where a and b are constants.
Shifting theorems
2. (i) If L[ f (t)] = F(s), then
et
L[e−at f (t)] = F(s + a)

=⇒ L−1 [F(s + a)] = e−at f (t) = e−at L−1 [F(s)]

(ii) L[eat f (t)] = F(s − a)

=⇒ L−1 [F(s − a)] = eat f (t) = eat L−1 [F(s)].

Downloaded From : www.EasyEngineering.net


Downloaded From : www.EasyEngineering.net

558 Engineering Mathematics - II

✎ ☞
Worked Examples
✍ ✌

h1 1 1 s i
Example 5.80. Find L−1 2 + + 2 + 2 . [Jan 2008]
s s+4 s +4 s −9
h1 1 1 s i
Solution. L−1 2 + + +
s s + 4 s2 + 4 s2 − 9
h1i h 1 i h 1 i h s i
= L−1 2 + L−1 + L−1 2 + L−1 2
s s+4 s +4 s −9
1 h 2 i
= t + e−4t + L−1 2 + cosh 3t
2 s +4
1

ww = t + e−4t + sin 2t + cosh 3t.


2

h s2 − 3s + 4 i

w.E
Example 5.81. Find L−1

Solution. L−1
h s2 − 3s + 4 i
s3
= L
s3
−1
h1 3

.

+
s s2 s3
4i
= L −1 1
h i
− 3L −1 1
h i
s2
+ 4L −1 1
h i
s3

asy
s
4 h 2 i
= 1 − 3t + L−1 3 = 1 − 3t + 2t2 .
2 s

Example 5.82. Find L−1 2 En


h 3s + 5 i
.
h 3s + 5 i
Solution. L−1 2
s +9
s +9
= 3L−1 2
gin
h s i
s +9
h 1 i
+ 5L−1 2
s +9
5
= 3 cos 3t + sin 3t
3
=
3
h 3s + 2 i
ee
9 cos 3t + 5 sin 3t
.

rin
Example 5.83. Find L−1 2
h 3s + 2 i
Solution. L−1 2
s −4
h s i
= 3L−1 2
.
h 2 i
+ L−1 2 = 3 cosh 2t + sinh 2t. g.n
h
s −4

s i
h
Example 5.84. Find L−1 2 2
s
s −4

a s + b2
h
i
.
s
s −4

i 1 −1 h s i 1
et
b 
Solution. L−1 2 2 2
= L−1 = L = cos t.
a s +b 2 2
a (s + b2
) a2 s2 + b2 a2 a
a2 a2

h 1 i
Example 5.85. Find L−1 .
(s − 3)5
h 1 i h i e3t e3t t4
3t −1 1 4
Solution. L−1 = e L = t = .
(s − 3)5 s5 4! 24

Downloaded From : www.EasyEngineering.net


Downloaded From : www.EasyEngineering.net

Laplace Transform 559

s i h
Example 5.86. Find L−1 .
(s + 6)3 " h i
h s i h i#
−1 s + 6 − 6 −6t −1 s − 6 −6t −1 1 −1 1
h i h i
Solution. L−1 = L = e L = e L − 6L
(s + 6)3 (s + 6)3 s3 s2 s3
h 6 i
= e−6t t − t2 = e−6t [t − 3t2 ].
2
h s+3 i
Example 5.87. Find L−1 2 .
s − 4s + 13
h s+3 i h s+3 i h s+3 i
Solution. L−1 2 = L−1 = L −1
s − 4s + 13 (s − 2)2 + 13 − 4 (s − 2)2 + 9

ww
h s − 2 + 2 + 3i
= L−1
(s − 2)2 + 9
n h s i h 5 io
= e2t L−1 2 + L−1 2

w.E h
s +9
5
= e2t cos 3t + sin 3t .
3
i
s +9

Example 5.88. Find L−1 2


h
asy
s
s − 4s + 5
i
.
h
Solution. L−1 2
s
s − 4s + 5
i
= L−1
En
h
2
s
(s − 2) + 5 − 4
i
= L−1
h s−2+2 i
(s − 2)2 + 1
= e2t L−1 2
gin
h s+2 i

h  s 
s +1
 1 i
= e2t L−1 2
2t
h ee
s +1
= e cos t + 2 sin t .
+ 2L−1 2
i
s +1

rin
Example 5.89. Find L−1
"
1
s2 + 4s + 2
#
.
g.n [Dec 2010]

Solution. L−1
"
1
s2 + 4s + 2
#
= L −1
"
1
(s + 2)2 + 2 − 4
 
#
= L −1
"
1
(s + 2)2 − 2
#
et
 1 
= e−2t L−1  √ 2 
s2 − 2
 √ 
e−2t −1  2 
= √ L  √ 2 
2 s2 − 2
e−2t √ 
= √ sin 2t .
2

Downloaded From : www.EasyEngineering.net


Downloaded From : www.EasyEngineering.net

560 Engineering Mathematics - II

h 2s − 3 i
Example 5.90. Find L−1 2 .
s + 4s + 13
h 2s − 3 i h 2s − 3 i h 2(s + 2) − 7 i
Solution. L−1 2 = L−1 2
= L−1
s + 4s + 13 (s + 2) + 13 − 4 (s + 2)2 + 9
h 2s − 7 i h  s   1 i
= e−2t L−1 2 = e−2t 2L−1 2 − 7L−1 2
s +9 s +9 s +9
h 7 i e−2t h i
= e−2t 2 cos 3t − sin 3t = 6 cos 3t − 7 sin 3t .
3 3
h s i
Example 5.91. Find L−1 . [May 2007]
(s + 2)2 + 1
h s i
−1 s + 2 − 2
h i
−2t −1 s − 2
h i
Solution. L−1 = L = e L

ww (s + 2)2 + 1 (s + 2)2 + 1
" h
−2t −1
=e L
s
s2 + 1
i
− 2L −1
h
s2 + 1
1 i
s2 + 1
#

w.E = e−2t [cos t − 2 sin t].


h s i
Example 5.92. Find L−1

Solution. L−1
h s i
= L asy
(s + 3)2
.
−1 s + 3 − 3
h i
= e−3t −1 s − 3
L
h i
[Dec 2009]

(s + 3)2
" h
En
(s + 3)2
−3t −1 1
=e L −
3i
#
=e L
"
s2
−3t −1 1
h i
−L
h i#
−1 3

gin
s s2
= e−3t [1 − 3t].
s s2

Example 5.93. Find L−1

Solution. L−1
h 1
+
h 1
ee
+

+
5

s+3
+
(s − 4)5 (s − 2)2 + 25 (s + 3)2 + 36
5 i
s+3 i
.

rin
[Dec 2007]

(s − 4) 5

= L−1
2
(s − 2) + 25 (s + 3) + 36
h 1 i
+ L −1
h
2

5 i
+ L −1
h
g.n s+3 i

=e 4t t
(s − 4)5
h1i
= e4t L−1 5 + e2t L−1 2
s
2t
(s − 2)2 + 25
h 5 i
s + 25
−3t
+ e sin 5t + e cos 6t
h s i
+ e−3t L−1 2
s + 36 et
(s + 3)2 + 36

4!
t
= e4t + e2t sin 5t + e−3t cos 6t.
24
h 3 i
Example 5.94. Find L−1 . [Dec 2009]
(s − 3)2 + 25
h 3 i
3t −1
h 3 i 3
3t −1
h 5 i 3
Solution. L−1 = e L = e L = e3t sin 5t.
(s − 3)2 + 25 s2 + 25 5 s2 + 25 5

Downloaded From : www.EasyEngineering.net


Downloaded From : www.EasyEngineering.net

Laplace Transform 561

Exercise 5 H

Find the inverse Laplace transform of the following functions.

s
8. .
1 1 s (s − 6)2 + a2
1. + + 2 .
s−3 s s −4
1 2
2 s 9. + .
2. + 2 . (s − 3) 5 (s + 1)2 + 4
2s − 3 s − 1

ww 3. 2
s
+
s + 49 s + 7
1
√ . 10.
s+6
(s + 6)2 + 9
.

w.E 2s 1 s−a
4. + 3. 11. .
2
s − 25 s (s − a)2 − b2
s 1 1 s+7
5. 2 + √ + 2
s − 225 s − 7 s + 16
1
.

asy 12.
(s + 7)2 + 49
5s + 3
.

6.
(s + 1)2 + 1
.

En 13.
s2 + 2s + 5
.

gin
s−3 3s − 2
7. . 14. .
(s − 3)2 + 4 s2 − 4s + 20


ee
5.4.1 Inverse Laplace transform by the method of partial fractions

Worked Examples

rin

Example 5.95. Find the inverse Laplace transform of
1

(s + 1)(s + 2) g.n [Dec 2012]

Solution. Let
1
=
A
+
B
(s + 1)(s + 2) s + 1 s + 2
∴ 1 = A(s + 2) + B(s + 1).
=
A(s + 2) + B(s + 1)
(s + 1)(s + 2) et
when s = −1, A = 1.
when s = −2, −B = 1 ⇒ B = −1.

1 1 1
∴ = −
(s + 1)(s + 2) s + 1 s + 2
" # " #
−1 1 −1 1 1
L =L −
(s + 1)(s + 2) s+1 s+2

Downloaded From : www.EasyEngineering.net


Downloaded From : www.EasyEngineering.net

562 Engineering Mathematics - II

" # " #
−1 1 −1 1
=L −L = e−t − e−2t .
s+1 s+2
1 h i
Example 5.96. Find L−1 . [May 2008]
(s + 1)(s + 3)
1 A B
Solution. Let = +
(s + 1)(s + 3) s + 1 s + 3

1 = A(s + 3) + B(s + 1)

Put s = −1, 1 = 2A ⇒ A = 12 .

ww Put s = −3, 1 = −2B ⇒ B =

1
−1
2 .
1 1

w.E ∴ = 2 − 2
(s + 1)(s + 3) s + 1 s + 3

asy h 1 1 i
1 h i
L−1 = L−1 2 − 2
(s + 1)(s + 3) s+1 s+3
1 −1 h 1 i 1 −1 h 1 i
= L
En
2
1 −t
s+1
− L
2 s+3

gin
−3t
= (e − e ).
2
1

ee
h i
Example 5.97. Find L−1 .
(s + a)(s + b)

rin
1 A B
Solution. Let = +
(s + a)(s + b) s + a s + b
1 = A(s + b) + B(s + a)
1 g.n
Put s = −a ⇒ A =

Put s = −b ⇒ B =
b−a
1
a−b
et
1 1
h 1 i h i
L −1
= L−1 b−a + a−b
(s + a)(s + b) s+a s+b
1 −at 1 −bt
= e + e
b−a a−b
1
= (e−bt − e−at ).
a−b

Downloaded From : www.EasyEngineering.net


Downloaded From : www.EasyEngineering.net

Laplace Transform 563

h1 i
Example 5.98. Find L−1 3
. [Nov 2005]
s(s + 3)
1 A B C D
Solution. Let 3
= + + 2
+ .
s(s + 3) s s + 3 (s + 3) (s + 3)3

1 = A(s + 3)3 + Bs(s + 3)2 + Cs(s + 3) + Ds.

1
When s = 0, 1 = 27A ⇒ A = .
27
−1
When s = −3, 1 = −3D ⇒ B = .
3

ww Equating the coeff. of s3 we get


−1

w.E A + B = 0 ⇒ B = −A =
27
.
Equating the coeff. of s2 we get

asy
9A + 6B + C = 0
1  −1 
9× +6 +C = 0
27 27

En
9  6 
− +C = 0
27 27
3
27
+C = 0

C=− .
1
gin

9

1
s(s + 3)3
1
ee
1
= 27 − 27 −
s
9
1
+
1
3
s + 3 (s + 3)2 (s + 3)3
rin
L−1
h 1
s(s + 3) 3
i
=
1 −1 h 1 i 1 h 1 i 1 h 1 i 1 h 1 i
27
L
s

27 s + 3

9 (s + 3)2

g.n
3 (s + 3)3

=
1
27
1
27

1 1

e−3t e−3t t t2 e−3t


27

9

6
.
1
× 1 − e−3t − e−3t t − e−3t
27 9 3
t2
2!
et
h s i
Example 5.99. Find L−1 .
(s − 4)(s + 4)
s A B
Solution. Let = +
(s − 4)(s + 4) s − 4 s + 4
s = A(s + 4) + B(s − 4)
1
Put s = 4 ⇒ A = 2

Downloaded From : www.EasyEngineering.net


Downloaded From : www.EasyEngineering.net

564 Engineering Mathematics - II

1
Put s = −4 ⇒ B = 2

h s i h 1 1 i
L−1 = L−1 2 + 2
(s − 4)(s + 4) s−4 s+4
1 4t 1 −4t
= e + e = cosh 4t.
2 2
h 4s + 5 i
Example 5.100. Find L−1 .
(s − 1)2 (s + 2)
4s + 5 A B C
Solution. Let 2
= + 2
+
(s − 1) (s + 2) s − 1 (s − 1) s+2
1 1

ww On computation, we get A = , B = 3, C = .

4s + 5
1
3
3
3
1

w.E
3
∴ = + + 3 .
(s − 1)2 (s + 2) s − 1 (s − 1)2 s + 2
h 4s + 5 i 1 t 1 1
∴ L−1 = e + 3et t + e−2t = (et + 9tet + e−2t ).

asy
2
(s − 1) (s + 2) 3 3 3
2s + 1
h i
Example 5.101. Find L−1 .

Solution. Let
2s + 1
2
(s + 2) (s − 1) 2
=
En
(s + 2)2 (s − 1)2
A
+
s + 2 (s + 2)
B
2
+
C
+
D
s − 1 (s − 1)2

= gin
A(s + 2)(s − 1)2 + B(s − 1)2 + C(s − 1)(s + 2)2 + D(s + 2)2

ee (s + 2)2 (s − 1)2

rin
∴ 2s + 1 = A(s + 2)(s − 1)2 + B(s − 1)2 + C(s − 1)(s + 2)2 + D(s + 2)2 .
When s = −2 When s = 1
1
9B = −3 ⇒ B = − .
3
9D = 3 ⇒ D = .
Equating the coefficients of s3 we get
1
3
g.n
A + C = 0.
Equating the constants we get
et (1)

2A + B − 4C + 4D = 1
1 4
2A − − 4C + = 1
3 3
2A − 4C + 1 = 1

2A − 4C = 0

Downloaded From : www.EasyEngineering.net


Downloaded From : www.EasyEngineering.net

Laplace Transform 565

A − 2C = 0. (2)

(1) − (2) ⇒ 3C = 0 ⇒ C = 0.

(1) ⇒ A = 0.
2s + 1 − 13 1
3
∴ = +
(s + 2)2 (s − 1)2 (s + 2)2 (s − 1)2
h 2s + 1 i 1 −1 h 1 i 1 −1 h 1 i
Now, L−1 = − L + L
(s + 2)2 (s − 1)2 3 (s + 2)2 3 (s − 1)2
1 h1i 1 h1i
= − e−2t L−1 2 + et L−1 2

ww
3 s 3 s
1 −2t 1 t t t
= − e t + e t = e − e−2t .

3 3 3

w.E
Example 5.102. Find L−1

Solution. Let
5s + 3
h 5s + 3
(s − 1)(s2 + 2s + 5)
=
A
+ 2
i
.
Bs + C
2

asy
(s − 1)(s + 2s + 5) s − 1 s + 2s + 5
5s + 3 = A(s2 + 2s + 5) + (Bs + C)(s − 1)

En
Put s = 1 ⇒ 8A = 8 ⇒ A = 1.
Equating the coeff. s2 ⇒ A + B = 0 ⇒ B = −1.

gin
s = 0 ⇒ 5A − C = 3 ⇒ C = 5 − 3 = 2.

ee
" # " #
5s + 3 1 −s + 2
L−1 = L −1
+
(s − 1)(s2 + 2s + 5) s − 1 s2 + 2s + 5

rin
" #
t −1 s−2
=e −L
(s + 1)2 + 5 − 1

=e −L t −1
"
s+1−3
(s + 1)2 + 4
#

g.n
t
=e −e L

t −t
"
"
−t −1 s − 3
s2 + 4

= e − e cos 2t − sin 2t
3
#

#
et
2
e−t
= et − [2 cos 2t − 3 sin 2t].
2
h s+9 i
Example 5.103. Find L−1 .
(s + 2)(s2 + 3)
s+9 A Bs + C
Solution. Let 2
= + 2
(s + 2)(s + 3) s + 2 s +3

Downloaded From : www.EasyEngineering.net


Downloaded From : www.EasyEngineering.net

566 Engineering Mathematics - II

A(s2 + 3) + (Bs + c)(s + 2)


= .
(s + 2)(s2 + 3)
∴ s + 9 = A(s2 + 3) + (Bs + c)(s + 2).
When s = −2, 7A = 7 ⇒ A = 1.
Equating the coefficient of s2 we get
A + B = 0 ⇒ B = −A = −1.
Equating the constants we get

3A + 2c = 9

ww 3 + 2c = 9

2c = 6 ⇒ c = 3.

w.E ∴
s+9
2
(s + 2)(s + 3)
=
s +
1
2
−s + 3
+ 2
s +3
=
s
1
+ 2
− 2
s−3
s +3
Now, L−1
h s + 9
(s + 2)(s2 + 3)
i
=
asy
L −1
h 1 i
s+2
− L −1
h s i
s2 + 3
− 3L −1
h 1 i
s2 + 3

En √ 3
= e−2t − cos 3t − √ sin 3t.
3

Example 5.104. Find L−1


h
gin
s2 + 16
(s2 + 1)(s2 + 4)
i
.

Solution. Let 2
h
L−1 2
s2 + 16

s2 + 16
2
i
= 2
A
ee + 2
B
(s + 1)(s + 4) s + 1 s + 4 s + 1 s + 4

= 5 sin t − 2 sin 2t.


= 2
5
− 2
4

rin
2
(s + 1)(s + 4)

Example 5.105. Find L−1 2


h s i
. g.n
Solution. Let
1
(s + a2 )(s2 + b2 )

A
=+ 2
B
et
(s2
+ a2 )(s2
+ b2 )
+a 2 s2
s + b2
1 1 1 1
= 2 2 2 2
− 2
b −a s +a b − a s + b2
2 2
s 1 h s s i
∴ 2 = −
(s + a2 )(s2 + b2 ) b2 − a2 s2 + a2 s2 + b2
h s i 1
Hence, L−1 2 2 2 2
= 2 [cos at − cos bt].
(s + a )(s + b ) b − a2

Downloaded From : www.EasyEngineering.net


Downloaded From : www.EasyEngineering.net

Laplace Transform 567

h 2s2 − 6s + 5 i
Example 5.106. Find L−1 3 .
s − 6s2 + 11s − 6
2
h 2s − 6s + 5 i h 2s2 − 6s + 5 i
Solution. Let L−1 3 2
= L −1
s − 6s + 11s − 6 (s − 1)(s − 2)(s − 3)

2s2 − 6s + 5 A B C
= + +
(s − 1)(s − 2)(s − 3) s − 1 s − 2 s − 3

2s2 − 6s + 5 = A(s − 2)(s − 3) + B(s − 1)(s − 3) + C(s − 1)(s − 2)


1
s = 1 ⇒ 2A = 1 ⇒ A = .
2

ww s = 2 ⇒ −B = 1 ⇒ B = −1.
5
s = 3 ⇒ 2C = 5 ⇒ C = .
2

w.E ∴ L−13
2s2 − 6s + 5 i
h
2
s − 6s + 11s − 6
h
= L−1 2 −
1
1
+
5 1 i
s−1 s−2 2s−3

asy 1 t 5
= e − e2t + e3t .
2 2

Example 5.107. Find L−1


" 2

En
3s + 16s + 26
s(s2 + 4s + 13)
#
. [Dec 2013]

Solution. Let
3s2 + 16s + 26 A
2
s(s + 4s + 13)
= + 2
gin
Bs + c
s s + 4s + 13

When s = 0, 13A = 26 ⇒ A = 2.
Equating the coefficients of s2
ee
3s2 + 16s + 26 = A(s2 + 4s + 13) + (Bs + c)s.

rin
A+B=3
g.n
2+B=3

B = 1.
Equating the coefficients of s
et
4A + c = 16

8 + c = 16

c = 8.

3s2 + 16s + 26 2 s+8


∴ 2
= + 2 .
s(s + 4s + 13) s s + 4s + 13

Downloaded From : www.EasyEngineering.net


Downloaded From : www.EasyEngineering.net

568 Engineering Mathematics - II

3s2 + 16s + 26
" # " #
−1 −1 2 s+8
L =L +
s(s2 + 4s + 13) s s2 + 4s + 13
" # " #
−1 1 −1 s+8
= 2L +L
s s2 + 4s + 13
" #
−1 s+2+6
=2·1+L
(s + 2)2 + 13 − 4
" #
−1 s+2+6
=2+L
(s + 2)2 + 9
" #
−2t −1 s + 6
=2+e L
s2 + 9

ww =2+e −2t
"
L −1
 s 
s2 + 9
+ 2L −1 3
s2 + 9
!#

w.E = 2 + e−2t [cos 3t + 2 sin 3t].

asy s2
" #
Example 5.108. Find L−1 . [Dec 2011]
s4 + 4a4
s2 s2
Solution. We have =

En
s4 + 4a4 (s2 − 2as + 2a2 )(s2 + 2as + 2a2 )
s2 As + B Cs + D
Let 2
2
2 2

gin
2
= 2
(s − 2as + 2a )(s + 2as + 2a ) s − 2as + 2a
2 2 2
2
+ 2
∴ s = (As + B)(s + 2as + 2a ) + (Cs + D)(s − 2as + 2a ).2
s + 2as + 2a2

A+C = 0
Equating the coefficient of s2 we get
ee
Equating the coefficients of s3 we get

rin (1)

2aA + B − 2aC + D = 1.
g.n (2)
Equating the coefficient of s we get
2a2 A + 2aB + 2a2C − 2aD = 0
Equating the constants we get
et (3)

2a2 B + 2a2 D = 0
2a2 (B + D) = 0
B + D = 0. (4)
From (1) and (4) we get C = −A and D = −B.
(2) ⇒ 2aA + B + 2aA − B = 1

Downloaded From : www.EasyEngineering.net


Downloaded From : www.EasyEngineering.net

Laplace Transform 569

4aA = 1
1
A= .
4a
1
⇒C=− .
4a
(3) ⇒ 2a2 A + 2aB − 2a2 A + 2aB = 0
4aB = 0

B = 0.

⇒ D = 0.

ww ∴ 4
s2
4
=
1
2
s
2

1 s
4a s + 2as + 2a2
2

w.E
s + 4a 4a s − 2as + 2a
s2
" #
1 −1  s  1  s 
L−1 4 = L − L −1
s + 4a4 4a s2 − 2as + 2a2 4a s2 + 2as + 2a2

asy
" # " #
1 −1 s − a + a 1 −1 s + a − a
= L − L
4a (s − a)2 + a2 4a (s + a)2 + a2

En
1 at −1 s + a
  1 −at −1 s − a 

= e L − e L
4a s2 + a2 4a s2 + a2
1 at −1 s a 

gin
   
−1
= e L + L
4a s2 + a2 s2 + a2
1 −at −1  s  −1
 a 

ee
− e L −L
4a s2 + a2 s2 + a2
1 at 1 −at
=
4a
e [cos at + sin at] − e [cos at − sin at]
1 h at
4a
rin
g.n
i
= e cos at + eat sin at − e−at cos at + e−at sin at
4a
1 h    i
= cos at eat − e−at + sin at eat + e−at

=
4a
1
4a
1
[cos at · 2 sin hat + sin at · 2 cos hat]

[cos at sin hat + sin at cos hat] .


et
2a
 s 
Example 5.109. Find L−1 4 [Dec 2008]
s + 4a4
s s
Solution. Let 4 =
s + 4a4 (s2 − 2as + 2a2 )(s2 + 2as + 2a2 )
s As + B Cs + D
Let 2 = 2 + 2
(s − 2as + 2a )(s + 2as + 2a ) (s − 2as + 2a ) (s + 2as + 2a2 )
2 2 2 2

Downloaded From : www.EasyEngineering.net


Downloaded From : www.EasyEngineering.net

570 Engineering Mathematics - II

∴ s = (As + B)(s2 + 2as + 2a2 ) + (Cs + D)(s2 − 2as + 2a2 ).


Equating the coefficients of s3 , we get
A + C = 0. (1)
Equating the coefficients of s2 , we get
2aA + B − 2aC + D = 0. (2)
Equating the coefficients of s, we get
2a2 A + 2aB + 2a2C − 2aD = 1. (3)
Equating the constants we get

ww 2a2 B + 2a2 D = 0
2a2 (B + D) = 0

w.E B + D = 0.
From (1) and (4) we get
(4)

C = −A and D = −B.
asy
En
∴ (2) ⇒ 2aA + 2aA = 0

gin
4aA = 0

⇒ A = 0.

ee
∴ C = 0.

(3) ⇒ 2a2 (A + C) + 2a(B − D) = 1


rin
2a2 × 0 + 2a(B + B) = 1
1 g.n
4aB = 1 ⇒ B =

⇒D=−
1
4a
.
4a
.

et
s 1 1 1 1
∴ = · 2 −
s4 + 4a4 4a s − 2as + 2a 2 4a s + 2as + 2a2
2
1 1 1 1
= −
4a (s − a)2 + a2 4a (s + a)2 + a2
" ! !#
−1
 s  1 −1 1 −1 1
L = L −L
s4 + 4a4 4a (s − a)2 + a2 (s + a)2 + a2

Downloaded From : www.EasyEngineering.net


Downloaded From : www.EasyEngineering.net

Laplace Transform 571

" ! !#
1 at −1 1 −at −1 1
= e L −e L
4a s2 + a2 s2 + a2
" #
1 at sin at sin at
= e − e−at
4a a a
1
= sin at[eat − e−at ]
4a2
1
= sin at · 2 sin hat
4a2
sin at sin hat
= .
2a2

ww
w.E
Exercise 5 I

asy
Find the Laplace inverse of the following.
En
gin
1.
s+2
s(s + 4)(s + 9)
.
ee 7.

8.
5s2 − 15s − 11
(s + 1)(s − 2)3
1
.
. [Apr 2009]

rin
[Nov 2009]

g.n
1 s4 −1
2. .
s(s + 1)(s + 2) s
9. .
3.
1−s
(s + 1)(s2 + 4s + 13)

4s2 − 3s + 5
. [Apr 2009]
10.
(s +

s2 (s2
1)2 (s2
1
+ 81)
.
+ 1)

et
[Jun 2008]

4. . [Dec 2007] 3s2 + 2s − 1


(s + 1)(s2 − 3s + 2) 11. .
(s2 + 4)(s2 − 2s + 1)
1
5. . 2s − 1
(s2 + a2 )(s2 + b2 ) 12. .
s2 (s − 1)2
s2 1
6. . [Nov 2008] 13. .
(s2 + a2 )(s2 + b2 ) s3 (s2 + 1)

Downloaded From : www.EasyEngineering.net


Downloaded From : www.EasyEngineering.net

572 Engineering Mathematics - II

Results
h i
1. If L−1 F(s) = f (t) and f (0) = 0 then
h i d   d h −1  i
L−1 sF(s) = f ′ (t) = f (t) = L F(s) .
h dti dt
In general L−1 sn F(s) = f (n) (t) if f (0) = 0 = f ′ (0) = · · · = f (n−1) (0).
h i dn
i.e., L−1 sn F(s) = n [L−1 [F(s)]].
dt
h i h F(s) i Z t Z t
2. If L−1 F(s) = f (t) then L−1 L−1 F(s) dt
 
= f (t)dt =
s 0 0

ww Similarly, L −1 F(s)
h
s2
i Z tZ t
=
0 0
−1  
L F(s) dtdt.

w.E 3. We know that L[t f (t)] = −


h
d 
ds
f (s) = −F ′ (s).
i


h i
L−1 F ′ (s) = −t f (t) = −tL−1 F(s) .

asy ✎
Worked Examples

En ✍ ✌

gin
s
h i
Example 5.110. Find L−1 . [May 2008]
(s + 2)2 + 4
s 1 i d 1 i!

ee
h i h h
Solution. L−1 = L −1
s = L −1
(s + 2)2 + 4 (s + 2)2 + 4 dt (s + 2)2 + 4

d −2t −1 h 1 i
!
rin
=
dt
d e
e L
−2t
−1
s2 + 4
h 2 i! g.n
=

=
dt 2
d e−2t
dt 2
L

sin 2t
s2 + 4
!
et
1 
= e−2t 2 cos 2t − 2e−2t sin 2t
2
= e−2t (cos 2t − sin 2t).

h s2 i
Example 5.111. Find L−1 2 2 2
.
(s + a )

Downloaded From : www.EasyEngineering.net


Downloaded From : www.EasyEngineering.net

Laplace Transform 573

s2
h i h s i d n h s io
Solution. L−1 = L −1
s = L −1
(s2 + a2 )2 (s2 + a2 )2 dt (s2 + a2 )2
d t sin at
n o 1 n o
= = t cos at.a + sin at
dt 2a 2a
1n o
= sin at + at cos at .
2a
h s+2 i
Example 5.112. Find L−1 2 . [May 2006]
(s + 4s + 5)2
h s+2 i h s+2 i h s i h t sin t i
Solution. L−1 2 2
= L−1 2 2
= e−2t L−1 2 2
= e−2t .
(s + 4s + 5) (s + 2) + 1) (s + 1) 2

ww Example 5.113. Find L−1


s2 i
(s − 1)4
h
. [Dec 2008]

w.E
h s2 i h s i d h s i!
−1 −1 −1
Solution. L =L s = L
(s − 1)4 (s − 1)4 dt (s − 1)4

asy
d i! d i!
−1 s + 1 − 1 t −1 s + 1
h h
= L = eL
dt (s − 1)4 dt s4
! h i!

En
d t −1 h 1 1i d t −1 h 1 i −1 1
= eL + = eL +L
dt s3 s4 dt s3 s4
d t −1 h t2 t3 i

gin
!
1d t 2 3
= eL + = (e (3t + t ))
dt 2 6 6 dt
 et 
1

ee

= et (6t + 3t2 ) + et (3t2 + t3 ) = 6t + 6t2 + t3 .
6 6

Example 5.114. Find L−1 2


h s
(s + a2 )2
i
. [Dec 2013, May 1991]
rin
Solution. From the standard results L−1 2
h s
(s + a )2 2
i t sin at
=
2a
.
g.n
Example 5.115. Find L−1

Solution. L−1
h 1 i
=
h

L
2
1
s(s + a )
−1 F(s)
h
2
i
i
.

where F(s) = 2
1
et
[Jun 2007]

2 2
s(s + a ) s s + a2

Zt Zt Zt
−1 −1
h 1 i sin at
= L [F(s)]dt = L dt = dt
s + a2
2 a
0 0 0
1  cos at t 1 1 − cos at
= − = − 2 (cos at − 1) = .
a a 0 a a2

Downloaded From : www.EasyEngineering.net


Downloaded From : www.EasyEngineering.net

574 Engineering Mathematics - II

h 1 i
Example 5.116. Find L−1 3
. [May 2007]
s(s + 2)
h 1 i
−1 F(s)
h i 1
Solution. L−1 3
= L where F(s) =
s(s + 2) s (s + 2)3
Z t Z t h 1 i Z t
−2t −1 1
h i h i
−1 −1
= L F(s) dt = L dt = e L dt
0 0 (s + 2)3 0 s3
Z t
1 t 2  e−2t  1 h t2 e−2t t
2 Z Z t  −2t 
−2t t e i
= e dt = t d = − 2tdt
0 2 2 0 −2 2 −2 0 0 −2
2 −2t Z t −2t 2 −2t −2t Z t  e−2t  i
1ht e  e i 1 h t e  te t
= + td = + − dt
2 −2 −2 2 −2 −2 0 −2

ww
0 0
1 h t2 e−2t te−2t 1  e−2t t i 1 h t2 e−2t te−2t 1  −2t i
= − + = − − e −1
2 −2 2 2 −2 0 2 −2 2 4

w.E 1 t e
h 2 −2t te −2t 1 −2t 1 i 1 h  i
= − − e + = 1 − e−2t 2t2 + 2t + 1 .
2 −2 2 4 4 8
1

asy
h i
Example 5.117. Find L−1 2 2 2
. [Dec 2013]
(s + a )
h 1 i
−1 1
h s i
−1 F(s)
h i s
Solution. L−1 2 = L = L where F(s) = 2
(s + a2 )2

= En
Z t
L
s (s2 + a2 )2
−1
h s i
dt =
Z t
s
t sin at
dt =
1
Z t 
(s + a2 )2

td
− cos at 

=
0

gin
1 −t cos at
h
(s2 + a2 )2
t

Z t 0
− cos at
2a

dt
i
2a 0 a

=
2a

2a
1 h
ee a
a
+
0
1 h −t cos at 1  sin at t i
a a 0
0

i
a
=
1 h −t cos at
2a
rin
a
1
+ 2 sin at
a
i

= 3 sin at − at cos at .
"
2a
1
#
g.n
et
−1
Example 5.118. Find L . [Dec 2013]
" (s#2 + 9s +" 13)2 #
1 1
Solution. L−1 2 = L−1
(s + 9s + 13)2 [(s + 3)2 + 13 − 9]2
" #
−3t −1 1 1
=e L 2 2
, which is of the form 2 with a = 2.
(s + 4) (s + a2 )2
" #
−3t 1
=e (sin 2t − 2t cos 2t)
2 × 23
e−3t
= [sin 2t − 2t cos 2t].
16

Downloaded From : www.EasyEngineering.net


Downloaded From : www.EasyEngineering.net

Laplace Transform 575

" # " #
−1 s −1 1
Example 5.119. Find L and find L and hence find
" # (s + a2 )2
2 (s + a2 )2
2
1
L−1 2 . [Dec 2013]
(s + 9s + 13)2
Solution. Solve examples 3.114, 3.117&3.118.
h 1 i
Example 5.120. Find L−1 2 . [Nov 2009]
s (s + a)
h 1 i h F(s) i 1
Solution. L−1 2 = L−1 2 where F(s) = , L−1 [F(s)] = e−at
s (s + a) s s+a

ww =
Zt Zt
L−1 [F(s)]dtdt

w.E
0 0
Zt Zt Zt !t
−at e−at
= e dtdt = dt
−a 0

asy
=
−1
0 0
Zt
−at
((e − 1))dt =
0

−1  e−at t
"
t
− (t)0
#

En
a
"
0
−1 e−at 1
#
a −a 0

=
gin
a −a a
1
+ −t
1 1
a
ee
= 2 e−at − 2 + t
1  −at
= 2 e − 1 + at .
a
a a


rin
s

g.n
h i
Example 5.121. Find L−1 2 2 2
. [Dec 2007]
(s − a )

et
Z
s
′ s F(s) = 2 − a2 )2
ds
Solution. Let F (s) = 2 (s
(s − a2 )2
Z
1 2s
s = ds
2 Z (s − a2 )2
2
h i h i
−1 −1 ′
L = L F (s)
(s2 − a2 )2 1 d(s2 − a2 )
= ds
= −tL−1 [F(s)] 2 (s2 − a2 )2
1 (s2 − a2 )−1
−1
h 1 i =
= −tL − 2 −1
2(s2 − a2 ) −1 1
t sinh at = .
= 2 s2 − a2
2 a
t
= sinh at.
2a

Downloaded From : www.EasyEngineering.net


Downloaded From : www.EasyEngineering.net

576 Engineering Mathematics - II

" #
2(s + 1)
−1
Example 5.122. Find L . [Dec 2009]
(s2 + 2s + 2)2
h 2(s + 1) i h 2(s + 1) i h 2(s + 1) i
Solution. L−1 2 2
= L−1 2 2
= L−1
(s + 2s + 2) ((s + 1) + 2 − 1) ((s + 1)2 + 1)2
h 2s i h i
= e−t L−1 2 2
= e−t L−1 F ′ (s) .
(s + 1)
2s
Where F ′ (s) = 2 .
(s + 1)2
d(s2 + 1)
Z Z
2s 1
F(s) = 2 2
ds = 2 2
=− 2 .
(s + 1) (s + 1) s +1

ww
h 2(s + 1) i n h io h −1 i
L−1 2 = e−t
−tL −1
F(s) = −te −t −1
L = te−t sin t.
(s + 2s + 2)2 s2 + 1
" #

w.E −1 s+3
Example 5.123. Find L . [Jun 2005]
(s2 + 6s + 13)2
h s+3 i h s+3 i h s+3 i
Solution. L−1 2 = L −1
= L −1
(s + 6s + 13)2

asy ((s + 3)2 + 13 − 9)2


= e−3t L−1 2
h
(s + 4)
s
2
i h
= e−3t L−1 F ′ (s) .
((s + 3)2 + 4)2
i


where F (s) = 2
s
(s + 4)2
.
En
∴ F(s) =
Z
s
(s2 + 4)2
ds =
1
2
Z
2s
gin
(s2 + 4)2
ds =
1
2
Z
d(s2 + 4) 1 
(s2 + 4)2
=
2

1 
s2 + 4
=− 2
1 1
2s +4
.

∴ L−1
h s+3
(s2 + 6s + 13)2
i
= eee
−3t

−3t
n
−tL −1
h
F(s)

te sin 2t te sin 2t
io

−3t
= −te−3t −1 −1
L
h 1 i

rin
2 s2 + 4

=
2 2
=
4
.
g.n
Exercise 5 J

Find the Inverse Laplace transform of the following functions.


et
s s−3
1. . [Nov 2008] 3. . [Decc 2009]
(s + 2)2 s2 + 4s + 13

s2 s
2. . 4. . [May 2009]
(s − 2)3 (s − a)5

Downloaded From : www.EasyEngineering.net


Downloaded From : www.EasyEngineering.net

Laplace Transform 577

1
1 8. .
5. . (s2 + a2 )2
(s − 2)2 + 132
s
1 9. .
6. . (s2 + a2 )2
s(s + 3)
1 1
7. . 10. .
s(s2 − 2s + 5) s2 (s2 + a2 )2

5.4.2 Inverse Laplace Transform of Logarithmic Functions


✎ ☞

ww ✍
Worked Examples

w.E
h 1 + si
Example 5.124. Find L−1 log 2 . [Dec 2009]
s
s+1
Solution. Let F(s) = log 2 = log(s + 1) − log s2 = log(s + 1) − 2 log s
s

F (S ) =
1
s+1 s

2
asy
we know that
En
L[t f (t)] = −F ′ (s)

gin
h 1 2i 2 1
=− − = −
s "+ 1 s # s s +"1 # " #
−1 2 1 −1 1 1

ee
−1
t f (t) = L − = 2L −L
s s+1 s s+1
∴ t f (t) = 2 − e−t
2 − e−t rin
f (t) =
t
.
g.n
s(s + 1) i

et
h
Example 5.125. Find L−1 log 2 . [May 2009]
s +1
s(s + 1)
Solution. Let F(s) = log 2 = log s + log(s + 1) − log(s2 + 1)
s +1
1 1 2s
F ′ (S ) = + − 2 .
s s+1 s +1
we have
2s 1 1
L[t f (t)] = −F ′ (s) = 2 − −
" s + 1 s + 1# s " # " #
−1 2s 1 1 −1
 s 
−1 1 −1 1
t f (t) = L − − = 2L −L −L
s2 + 1 s + 1 s s2 + 1 s+1 s

Downloaded From : www.EasyEngineering.net


Downloaded From : www.EasyEngineering.net

578 Engineering Mathematics - II

t f (t) = 2 cos t − e−t − 1


2 cos t − e−t − 1
f (t) = .
t
h s2 + a2 i
Example 5.126. Find L−1 log 2 . [Jun 2002]
s − b2
s2 + a2
Solution. Let F(s) = log 2 2
= log(s2 + a2 ) − log(s2 − b2 )
s −b
′ 2s 2s
F (S ) = 2 − .
s + a2 s2 − b2

ww
2s 2s
F ′ (s) = − 2
s2
+ a2 s − b2
s  s 
L−1 [F ′ (s)] = 2L−1 2 − 2L −1

w.E s + a2
−t f (t) = 2 cos at − 2 cosh bt
s2 − b2

asy
2
f (t) = (cosh bt − cos at).
t

En
h s + 1i
Example 5.127. Find L−1 log . [Dec 2013]
s−1
s+1
Solution. Let F(s) = log

F ′ (S ) =
1

s−1
1
gin
= log(s + 1) − log(s − 1)

s+1 s−1
We know that

L[t f (t)] = −F ′ (s) = −


1
ee
+
1
rin
g.n
s+1 s−1
h 1 1 i h 1 i h 1 i
∴ t f (t) = L−1 − = L−1 − L−1 = et − e−t
s−1 s+1 s−1 s+1
∴ f (t) =
et − e−t
t

Example 5.128. Find L−1 s log


h
.

s+1 i
+2 .
et
[Apr 1995]
s−1
s+1
Solution. Let F(s) = s log + 2 = s[log(s + 1) − log(s − 1)] + 2
s−1
h 1 1 i
F ′ (s) = s − + log(s + 1) − log(s − 1)
s+1 s−1
h s − 1 − s − 1i s+1
=s 2
+ log
s −1 s−1

Downloaded From : www.EasyEngineering.net


Downloaded From : www.EasyEngineering.net

Laplace Transform 579

−2s s+1
= 2
+ log
s −1 s−1
h s i h s + 1i
t f (t) = 2L−1 2 − L−1 log
s −1 s−1
et − e−t 2
= 2 cos ht − = 2 cos ht − cos ht.
t t
2 cos ht 2 cos ht 2 cos ht
f (t) = − = (t − 1).
t t2 t2

5.4.3 Inverse Laplace Transform of inverse Trignometric Functions

ww
h ai
Example 5.129. Find L−1 tan−1 .
a s
Solution. Let F(s) = tan−1

w.E
s
1  −a  s2  −a  a
F ′ (s) = 2 2
= 2 2 2
=− 2 .
a s s + a s s + a2
1+ 2
s
asy
We have L[t f (t)] = −F ′ (s) = 2
a2

En
t f (t) = L−1 2
s + a2
h a i
= sin at

gin s + a2
f (t) =
sin at
.

ee
t
h i
Example 5.130. Find L−1 cot−1 (s) . [May 2011, Jun 2010]
Solution. Let F(s) = cot−1 (s)
1 rin
F ′ (s) = −
1 + s2
we have L[t f (t)] = −F ′ (s) = 2
1 g.n
∴ t[ f (t)] = L

∴ f (t) =
−1

sin t
.
"
1
s2 + 1
#
= sin t.
s +1
et
t
h a  s i
Example 5.131. Find L−1 tan−1 + cot−1 .
s b
a  s
Solution. Let F(s) = tan−1 + cot−1
s b
1  a 1 1 −a b
F ′ (s) = − 2
− = 2 2
− 2 .
1+ 2 a2 s 1+ 2 b
s2
s +a s + b2
s b

Downloaded From : www.EasyEngineering.net


Downloaded From : www.EasyEngineering.net

580 Engineering Mathematics - II

a b
We know that L[t f (t)] = −F ′ (s) = + 2
s2 +a2 s + b2
t f (t) = sin at + sin bt
sin at + sin bt
f (t) = .
t
h  2 i
Example 5.132. Find L−1 cot−1 . [May 2008]
s+1
 2 
Solution. Let F(s) = cot−1 .
s+1
1  −2  2
F ′ (s) = − 4 2
= .
(s + 1) (s + 1)2 + 4

ww
1+ 2
(s+1)
−2
L[t f (t)] = −F ′ (s) = .
(s + 1)2 + 4

w.E h
t f (t) = L−1 −
2
2
(s + 1) + 4
sin 2t
i
= −e−t L−1 2
h 2 i
s +4
= −e−t sin 2t.

f (t) = −e−t
asy
h
Example 5.133. Find L−1 s log √
t
.

s i
+ cot−1 s .

Solution. Let F(s) = s log √ En


s
s2 + 1
+ cot−1 s

h
2
s +1
gin
1 i


h 1 1 2s i
F (s) = s − 2
s 2s +1
s2
ee
= s log s − log(s2 + 1) + cot−1 (s)
2
1
+ log s − log(s2 + 1) −
2
1
1 + s2
rin
g.n
1 1
=1− 2 − 2 + log s − log(s2 + 1)
s +1 s +1 2
2
s +1 1


=1− 2
s +1
1
+ log s − (log s2 + 1).

F (s) = log s − log(s + 1)


2
1
2
2
et
L[t f (t)] = −F ′ (s) = log(s2 + 1) − log s
2
−1 1
h i
t f (t) = L log(s2 + 1) − log s . (1)
2
1
Let G(s) = log(s2 + 1) − log s
2
′ 1 2s 1 s 1
G (s) = − = −
2 s2 + 1 s s2 + 1 s

Downloaded From : www.EasyEngineering.net


Downloaded From : www.EasyEngineering.net

Laplace Transform 581

1 s
L[tg(t)] = −G′ (s) = − 2
s s +1
tg(t) = 1 − cos t
1 − cos t
g(t) =
t
1 − cos t
(1) ⇒ t f (t) =
t
1 − cos t
f (t) = .
t2
" !#
−1 2

ww
−1
Example 5.134. Find L tan 2
.
! s
2
Solution. Let F(s) = tan−1 2

w.E
s
1
F ′ (s) = 4
· 2(−2)(s−3 )
1 + s4

= 4
s4
asy
s +4 s
−4
3
!
=− 4
4s
s +4
.

En
gin
We know that L[ f (t)] = −F ′ (s) = 4
"
4s
4s
s #+ 4

ee
−1
t f (t) = L
s4 + 4
4

rin
 s 
∴ f (t) = L−1 4
t s +4
Now, 4
s
= 2
s
s + 4 (s + 2s + 2)(s2 − 2s + 2)
As + B Cs + D g.n
= 2 + 2
(s + 2s + 2) (s − 2s + 2)
et
s = (As + B)(s2 − 2s + 2) + (Cs + D)(s2 + 2s + 2).

Equating the coefficients of s3 , we get


A + C = 0. (1)
Equating the coefficients of s2 , we get
−2A + B + 2C + D = 0. (2)
Equating the coefficients of s, we get

Downloaded From : www.EasyEngineering.net


Downloaded From : www.EasyEngineering.net

582 Engineering Mathematics - II

2A − 2B + 2C + 2D = 1. (3)
Equating the constants we get
2B + 2D = 0
B + D = 0. (4)
From (1) and (4) we get
C = −A, D = −B.

(2) ⇒ −2A − 2A = 0

ww −4A = 0

A = 0.

w.E ⇒ C = 0.

−2B − 2B = 1

asy
−4B = 1

En
B=−
1
4
1
D= .
4
gin
∴ 2

L−1 2
s
s +4
 s 
=

=
1
ee
·
1
− ·
1
4 s2"− 2s + 2 4# s2 + 2s" + 2
1 −1
L
1
− L
1 −1
1

1
rin
#

s +4

=
4
"
1 −1
L
s2 − 2s + 2
1
!
4

−L −1
s2 + 2s + 2
1 g.n
!#

=
4

4
"
1 t −1

1 t
h
eL
(s − 1)2 + 1
1
s2 + 1
!
−e L
i
−t −1
(s + 1)2 + 1
1
s2 + 1
!#
et
= e sin t − e−t sin t
4
1
= sin t(et − e−t )
4
sin t
= · 2 sin ht
4
sin t sin ht
=
2

Downloaded From : www.EasyEngineering.net


Downloaded From : www.EasyEngineering.net

Laplace Transform 583

4 sin t sin ht
∴ f (t) = ·
t 2
2 sin t sin ht
= .
t

Exercise 5 K

Find the inverse Laplace transform of the following functions.

ww ω2 
6. log
 (s + 3)(s + 7) 
s−1
. [Apr 2005]

w.E

1. log 1 + . [May 2009]
s2  s2 + 4 
 s−a  7. log . [Dec 2004]
2. log 2 . [Apr 2007] s2 + 1

3. log
s + a2
 s + 2
s+4
. asy 8. tan−1
 s
a
. [Apr 2007]

−1
4. cot (s + 1).
En 9. log
 s2 + 9 
s2 + 16
.

5. log
 s2 (s + 3) 
s−5
.
gin
[May 2006] 10. cot−1
 s
k
.

ee
5.4.4 Inverse Laplace transform using contour integration
rin
g.n
Let L[ f (t)] = F(s). If F(s) → 0 as s → ∞ then f (t) = sum of the residues of e st F(s)
at the poles of F(s).

"
2s + 3


Worked Examples
#

✌ et
Example 5.135. Find L−1 using the method of residues.
(s − 2)(s + 1)2
2s + 3
Solution. Let F(s) = .
(s − 2)(s + 1)2
Clearly F(s) → 0 as s → ∞.
∴ f (t) = sum of the residues of e st F(s) at the poles of F(s).
The poles of F(s) are at s = 2 and at s = −1.

Downloaded From : www.EasyEngineering.net


Downloaded From : www.EasyEngineering.net

584 Engineering Mathematics - II

s = 2 is a simple pole and s = −1 is a pole of order 2.

2s + 3 7e2t
R(2) = lim(s − 2)e st F(s) = lim(s − 2)e st = .
s→2 s→2 (s − 2)(s + 1)2 9
1 d d 2s + 3
R(−1) = lim (s + 1)2 e st F(s) = lim (s + 1)2 e st
(2 − 1)! s→−1 ds s→−1 ds (s − 2)(s + 1)2
" st
(s − 2){e st 2 + (2s + 3)te st } − e st (2s + 3)
#
d e (2s + 3)
= lim = lim
s→−1 ds s−2 s→−1 (s − 2)2
−t −t
(−3)(2e + te ) − e −t −t
−6e − 3te − e−t −t
= =
9 9
−7e−t − 3te−t −e−t

ww =

f (t) =
9
=
9
(7 + 3t).

sum of the residues = e2t −


7 e−t
(7 + 3t).

w.E 9

Example 5.136. Evaluate the inverse Laplace transform of


9
1
by the
method of residues.
Solution. Let F(s) =
1 asy (s + 1)(s − 2)2

(s + 1)(s − 2)2
By the method of residues,
En
gin
f (t) =sum of the residues of e st F(s) at the poles of F(s).
Poles of F(s) are −1and 2

ee
−1 is a simple pole and 2 is a pole of order 2.

∴ R(−1) = lim (s + 1)e st


1
=
e−t
. rin
s→−1

R(2) = lim
d
(
(s + 1)(s − 2)2

(s − 2)2 e st
1
9
)
g.n
(s + 1)(s − 2)2

et
s→2 ds

(s + 1)te st − e st 3te2t − e2t


= lim =
s→2 (s + 1)2 9
e 2t
= (3t − 1)
9
e−t e2t
∴ f (t) = sum of the residues = + (3t − 1).
9 9
2s − 2
Example 5.137. Find the inverse Laplace transform of using
(s + 1)(s2 + 2s + 5)
contour integral.

Downloaded From : www.EasyEngineering.net


Downloaded From : www.EasyEngineering.net

Laplace Transform 585

2s − 2
Solution. Let F(s) =
(s + 1)(s2 + 2s + 5)
Poles are given by s + 1 = 0 ⇒ s = −1 and s2 + 2s + 5 = 0 ⇒ (s + 1)2 + 4 = 0
⇒ (s + 1 + 2i)(s + 1 − 2i) = 0 ⇒ s = −(1 + 2i) and s = −(1 − 2i).
All are simple poles.
h e st (2s − 2) i
R(−1) = lim (s + 1)
s→−1 (s + 1)(s2 + 2s + 5)
−t
e (−4) −4 −t
= = e = −e−t
1−2+5 4

ww (s + 1 + 2i)e st (2s − 2)
h i
R(−(1 + 2i)) = lim
s→−(1+2i) (s + 1)(s + 1 + 2i)(s + 1 − 2i)
e−(1+2i)t (−2(1 + 2i) − 2)

w.E =

=
(−1 − 2i + 1)(−1 − 2i + 1 − 2i)
(−4 − 4i)e−t e−2it e−t e−2it (1 + i)
= .

asy −8
changing i to -i
2

R(−(1 − 2i)) =
2
En
e−t e2it (1 − i)

By the method of residues


gin
h
e−t e−2it
f (t) = −e−t +

= e−t − 1 +
2
(1 + i) +
ee
e−t e2it
2
(1 − i)
e−2it + ie−2it e2it − ie2it i
+ rin
h
= e−t − 1 +
e −2it
2
+e
2
−2it
−i
2
e − e2it i
2it

2 g.n
−t
h
= e − 1 + 2 cos 2t − i2i sin 2t
h
= e−t − 1 + cos 2t + 2 sin 2t
h i
i
i
et
= e−t cos 2t + sin 2t − 1 .

1h i
Example 5.138. Find L−1 2
by the method of residues.
(s − 1)(s + 1)
1 1
Solution. Let F(s) = = .
(s − 1)(s2 + 1) (s − 1)(s + i)(s − i)
Clearly F(s) → 0 as s → ∞.

Downloaded From : www.EasyEngineering.net


Downloaded From : www.EasyEngineering.net

586 Engineering Mathematics - II

∴ f (t) = sum of the residues of e st F(s) at the poles of F(s).


The poles of F(s) are 1, i, −i and all are simple poles.
1 et
R(1) = lim(s − 1)e st F(s) = lim(s − 1)e st = .
s→1 s→1 (s − 1)(s2 + 1) 2
1 eit −ieit
R(i) = lim(s − i)e st = =
s→i (s − 1)(s + i)(s − i) (i − 1)2i (i − 1)2
i ieit
= eit = (1 + i).
(1 − i)2 4
−i
R(−i) = e−it (1 − i).
4

ww f (t) = sum of the residues = R(1) + R(i) + R(−i)


et i i
+ eit (1 + i) − e−it (1 − i)

w.E =

=
2
et
2
4
i
4
+ eit + ieit − e−it + ie−it
4


=
et
2
i
asy
+ eit − e−it + i(eit + e−it )
4


=
et
2
i

En 1
+ 2i sin t − 2 cos t
4 4
=
et
2
1
− sin t −
2
gin
cos t 1 t
2
= (e − sin t − cos t).
2

ee
1
Example 5.139. Find the inverse Laplace transform of by the method of
(s2 + 1)2
residues.
Solution. Let F(s) =
1
rin
(s2
+ 1)2
Poles of F(s) are s = i and −i
g.n
i and −i are poles of order 2.

R(i) = lim
d h
s→i ds
(s − i)2 e st 2
2 st
1
(s + 1)2
st
i
= lim
d h
s→i ds
it
(s − i)
it
2 st
e
1 et
(s + i)2 (s − i)2
i

(s + i) te − e 2(s + i) −4te − 4ie 1h i


= lim 4
= = − teit + ieit
s→i (s + i) 16 4
Replacing i by -i, we get
1h i
R(−i) = − te−it − ie−it .
4
By the method of residues,

Downloaded From : www.EasyEngineering.net


Downloaded From : www.EasyEngineering.net

Laplace Transform 587

f (t) = sum of the residues


1h i
= − teit + te−it + i(eit − e−it )
4
1
= − [2t cos t − 2 sin t]
4
1
= − (t cos t − sin t) .
2
1
Example 5.140. Evaluate the inverse Laplace transform of by the
s2 (s2 − a2 )
method of residues.

ww Solution. Let F(s) =


1
s2 (s2
2
= 2
1
− a ) s (s − a)(s + a)
Poles of F(s) are s = 0, a, −a.

w.E s = 0 is a pole of order 2.


∴ R(0) = lim
d h 2 st
s→0 ds
i
s e F[s] = lim
d h 2 st
s→0 ds
s e 2 2
1
s (s − a2 )
i

= lim
s→0 asy
(s2 − a2 )te st − e st 2s −a2 t −t
(s2 − a2 )2
= 4 = 2.
a a

En
s = a is a simple pole.

R(a) = lim(s − a)e st


s→a
gin 1
s2 (s − a)(s + a)
=
eat
a2 2a
=
eat
2a3
s = −a is a simple pole.
ee rin
R(−a) = lim (s + a)e st
s→−a
at
2
1
s (s − a)(s + a)
e−at g.n
et
e
= 2 =− 3.
a (−2a) 2a
By the method of residues

f (t) = sum of the residues of e st F(s)

f (t) = R(0) + R(a) + R(−a)


t eat e−at
=− + −
a2 2a3 2a3
t 1
= − 2 + 3 sinh at.
a a

Downloaded From : www.EasyEngineering.net


Downloaded From : www.EasyEngineering.net

588 Engineering Mathematics - II

5.5 Inverse Laplace transform using Second Shifting theorem

Result. If L[ f (t)] = F(s) then, L[ f (t − a)u(t − a)] = e−as L[ f (t)] = e−as F(s).

∴ L−1 e−as F(s) = f (t − a)u(t − a)


 

= f (t − a), when t > a.

= [ f (t)]t→t−a = [L−1 {F(s)}]t→t−a .

e−as
" #

ww Example 5.141. Find L

Solution. F(s) =
1
−1
s
. [Dec 2011]

w.E s
L−1 [F(s)] = 1.
Now by Second Shifting theorem,

h
L−1 [e−as F(s)] = L−1 F(s)asy
i
u(t − a) = [1]t→t−a u(t − a)

En
t→t−a

= 1 · u(t − a) = u(t − a).

Example 5.142. Find L −1


"
gin
e−3s
s3
#
.
1 −1
Solution. F(s) = 3 · L [F(s)] = L
s
" #
−1 1
s3
=
t2
2!
.
ee rin
g.n
" # " #
−1 −3s 1 −1 1
L e · 3 =L , t>3
s s3 t→t−3

et
" 2#
t
= , t>3
2! t→t−3
(t − 3)2
, = t > 3.
2
" −πs #
−1 e
Example 5.143. Find L .
s2 + 1
1
Solution. F(s) = 2 .
s +1 " #
−1 −1 1
L [F(s)] = L = sin t.
s2 + 1

Downloaded From : www.EasyEngineering.net


Downloaded From : www.EasyEngineering.net

Laplace Transform 589

By Second Shifting theorem,


" # " #
−1 −πs 1 −1 1
L e =L , t>π
s2 + 1 s2 + 1 t→t−π
= [sin t]t→t−π , t>π

= sin(t − π), t > π.

e−πs
" #
−1
Example 5.144. Find L .
s2
1
Solution. F(s) = .

ww −1
s2
L [F(s)] = L
1
−1
s2
# "
= t.

w.E By Second Shifting theorem,

L
" −πs #
−1 e
=L
" #
−1 1
, t>π
s2

asy
s2 t→t−π
= [t]t→t−π , t>π

= t − π,
En t > π.

Example 5.145. Find L −1


"

gin
se−2s
s2 − 1
#
.

Solution. F(s) = 2
s
s −1 
L−1 [F(s)] = L−1
.
s 
s2 − 1
= cos ht.
ee rin
By Second Shifting theorem,

L
" −2s #
−1 se
=L −1
 s 
, t>2 g.n
s2 − 1 s2 − 1 t→t−2
= [cos ht]t→t−2 ,

= cos h(t − 2),


t>2

t > 2.
et
e−2s
" #
−1
Example 5.146. Find L .
s−3
1
Solution. F(s) = .
s−3 " #
1
−1
L [F(s)] = L −1
= e3t .
s−3

Downloaded From : www.EasyEngineering.net


Downloaded From : www.EasyEngineering.net

590 Engineering Mathematics - II

By Second Shifting theorem,


" −2s # " #
e 1
L−1 = L−1 , t>2
s−3 s − 3 t→t−2
h i
= e3t , t>2
t→t−2
3(t−2)
=e , t > 2.
e−s
" #
−1
Example 5.147. Find L .
(s − 1)(s − 2)
1 1 1
Solution. F(s) = = −
(s − 1)(s
" − 2) s − 2# s − 1

ww −1
L [F(s)] = L −1 1
s−2 s−1
By Second Shifting theorem,

1
= e2t − et .

w.E L −1
"
e−s
(s − 1)(s − 2)
#
=L −1
"
1
#

(s − 1)(s − 2) t→t−1
, t>1

asy 
= e2t − et


2(t−1)
, t>1
t→t−1
(t−1)
=e
" −s
En
−e

se 2 + πe−s
#
, t > 1.

Example 5.148. Find L−1


s2 + π2
gin
Solution. By Second Shifting theorem we have
.

L−1
" −s
se 2 + πe−s
s2 + π2
#
= L


ee
−1 − 2s
e
 s 
= L−1 2
·
s
s2 + π2
+

+
e−s

L
π 
s2 + π2
−1
 π 
rin
s + π2 t→t− 21 s2 + π2 t→t−1
= [cos πt]t→t− 1 + [sin πt]t→t−1 g.n
= cos π t −
1
2
!2
+ sin π(t − 1), t > 1.
et
5.6 Inverse Laplace transform by the method of convolution

Convolution. Let f (t) and g(t) be two functions defined for all t ≥ 0. The
convolution of f (t) and g(t) is defined as
Z t
( f ∗ g)(t) = f (t) ∗ g(t) = f (u)g(t − u)du
0

Downloaded From : www.EasyEngineering.net


Downloaded From : www.EasyEngineering.net

Laplace Transform 591

(or)
Z t
( f ∗ g)(t) = g(u) f (t − u)du.
0

Convolution theorem. If L[ f (t)] = F(s) and L[g(t)] = G(s) then L[ f (t) ∗ g(t)] =
F(s).G(s). Also L−1 [F(s)G(s)] = f (t) ∗ g(t) = L−1 [F(s)] ∗ L−1 [G(s)].
Zt
Proof. By definition ( f ∗ g)(t) = f (u)g(t − u)du
0

ww L[ f (t) ∗ g(t)] =
Z∞
e−st ( f (t) ∗ g(t))dt =
Z∞
 t
Z



e−st  f (u)g(t − u)du dt

w.E
 
0 0 0
Z∞ Zt
= e−st f (u)g(t − u)dudt. (1)

asy0 0

En
In the above double integration, the region of integration is given by u = 0, u = t,
t = 0 and t = ∞.

gin u
We shall evaluate the double integral
ee
t

rin
=

by changing the order of integration.


u
t=0

Divide the region in to strips parallel to


the t-axis. PQ is one such strip. Along
P

g.n Q

this strip t varies from u to ∞ and as


this strip traverses the entire region, u
varies from 0 to ∞.
O u=0
et t

Z∞ Z∞
∴ L[ f (t) ∗ g(t)] = e−st f (u)g(t − u)dtdu.
u=0 t=u

Let v = t − u. When t = u, v = 0.

dv = dt When t = ∞, v = ∞.

Downloaded From : www.EasyEngineering.net


Downloaded From : www.EasyEngineering.net

592 Engineering Mathematics - II

Z∞ Z∞ Z∞ Z∞
−s(u+v)
= e f (u)g(v)dvdu = e−su e−sv f (u)g(v)dvdu
u=0 v=0 u=0 v=0
Z∞ Z∞
= e−su f (u)du e−sv g(v)dv = L[ f (u)]L[g(v)]
u=0 v=0

= L[ f (t)]L[g(t)] = F(s)G(s).

∴ L−1 [ f (t)g(t)] = f (t) ∗ g(t).


✎ ☞

ww Worked Examples

Example 5.149. Using convolution theorem,evaluate L−1



h 1 i

w.E
.
(s + 1)(s + 2)
[May 2007]
1 1 1
Solution. = .

Let F(s) = asy


(s + 1)(s + 2) s + 1 s + 2
1
and G(s) =
1
.

En
s+1 s+2
h 1 i h i h i h i
L−1 = L−1 F(s)G(s) = L−1 F(s) ∗ L−1 G(s)
(s + 1)(s + 2)
= L−1
gin
h 1 i
s+1
∗ L−1
h 1 i
s+2
= e−t ∗ e−2t

ee
Z t Z t
= e−u e−2(t−u) du = e−u e−2t+2u du

=
Z0 t
e−2t+u du =
Z t 0
e−2t eu du
rin
g.n
0 0

= e−2t .(eu )t0 = e−2t (et − 1) = e−t − e−2t .

Example 5.150. Using Convolution theorem find L

Solution. L −1
"
1
#
=L −1
"
1
·
1
#
−1
"
1
(s + a)(s + b)
#
et [May 2011]

(s + a)(s + b) (s + a) (s + b)
" # " #
−1 1 −1 1
=L ∗L
(s + a) (s + b)
Zt
= e−at ∗ e−bt = e−au · e−b(t−u) du
0

Downloaded From : www.EasyEngineering.net


Downloaded From : www.EasyEngineering.net

Laplace Transform 593

Zt Zt
= e −au
·e −bt+bu
du = e−au · e−bt · ebu du
0 0
Zt #t
e(b−a)u
"
−bt (b−a)u −bt
=e e du = e
b−a 0
0
e−bt  
= e(b−a)t − 1
b−a
1 h −bt+bt−at i
= e − e−bt
b−a

ww
1 h −at i
= e − e−bt .
b−a
1

w.E
h i
Example 5.151. Find L−1 using convolution theorem. [Jun 2008]
s(s2 − a2 )
1 1 1
Solution. = . 2
s(s − a ) s s − a2
2 2

Let F(s) =
1
s asy
and G(s) = 2
1
s − a2
1

En
h i h i h i h i
L−1 = L −1
F(s)G(s) = L −1
F(s) ∗ L −1
G(s)
s(s2 − a2 )

gin
h1i h 1 i 1
= L−1 ∗ L−1 2 2
= 1 ∗ sinh at
s s −a a
1 t
Z
1  cosh au t 1

ee

= sinh au.1du = = 2 cosh at − 1 .
a 0 a a 0 a

Example 5.152. Find L−1


"
2
1
#
using Convolution theorem.
rin
[Dec 2011]

g.n
s(s + 4)
Solution. " # " #
1 −1 1 1
L−1 = L ·
s(s2 + 4)

=L −1 1

Zt
s s2 + 4
" #

s
∗L −1
"
2
s +4
1
#
=1∗
sin 2t
2
et
1
= sin 2u · du
2
0
" #t
1 cos 2u
= −
2 2 0
1 1 − cos 2t
= − (cos 2t − 1) = .
4 4

Downloaded From : www.EasyEngineering.net


Downloaded From : www.EasyEngineering.net

594 Engineering Mathematics - II

h 1 i
Example 5.153. Using convolution theorem,evaluate L−1 2
.
(s + 1)(s + 1)
[Nov 2004]
−1
h 1 i
−1
h 1 1 i −1
h 1 i
−1
h 1 i
Solution. L =L . =L ∗L
(s + 1)(s2 + 1) s + 1 s2 + 1 s+1 s2 + 1
Z t
= e−t ∗ sin t = sin ue−(t−u) du
0
Z t Z t
= sin ue −t+u
du = sin ue−t eu du
0 0
Z t h eu it
−t u
=e e sin udu = e−t (sin u − cos u)
2 0

ww
0
e−t h i 1 h i
= et (sin t − cos t) + 1 = sin t − cos t + e−t .
2 2

w.E
Example 5.154. Find L−1 2
h
2
s 2

(s + a )(s + b )2 2
i
using convolution theorem.
[Dec 2015, Dec 2014, Jun 2014, Dec 2010, May 2003]
Solution. L−1 2
h
2
s 2

asy
2
h (ss + ai )(s +h b )s i
2
i
= L −1
h s
.
s2 + a2 s2 + b2
s i

En
−1 −1
=L 2 2
∗L = cos at ∗ cos bt
Z t s +a s2 + b2 Z
t
=

=
0Z
1 t
cos au cos b(t − u)du =

gin 0
cos au cos(bt − bu)du

{cos(au + bt − bu) + cos(au − bt + bu)}du

=
2 0

2 a−b
1 sin at sin bt sin at sin bt i
h
− +
+ ee
1 h sin(au + bt − bu) sin(au − bt + bu) it

+
a+b 0

rin
=
2 a−b a−b a+b a+b
1 a+b+a−b
2
h
a2 − b2
sin at +
a−b−a−b
a2 − b2
sin bt
i
g.n
1

et
h i a sin at − b sin bt
= 2a sin at − 2b sin bt = .
2(a2 − b2 ) a2 − b2
h s i
Example 5.155. Find L−1 2 2 2
using convolution theorem.
(s + a )
[Jun 2012, Jun 2010, May 2008]
s s 1
Solution. Let 2 = 2 . .
(s + a2 )2 s + a2 s2 + a2
s 1
∴ F(s) = 2 2
, G(s) = 2 .
s +a s + a2
h s i h i
L−1 2 2 2
= L−1 F(s)G(s) = L−1 F(s) ∗ L−1G(s)
(s + a )

Downloaded From : www.EasyEngineering.net


Downloaded From : www.EasyEngineering.net

Laplace Transform 595

h s i h 1 i sin at
= L−1 2 2
∗ 2 2
= cos at ∗
(s + a ) (s + a ) a
Z t
1
= cos au sin a(t − u)du
a 0
Z t
1
= 2 sin(at − au) cos audu
2a 0
Z tn
1 o
= sin(at − au + au) + sin(at − au − au) du
2a 0
1n t
Z Z t o
= sin atdu + sin(at − 2au)du
2a 0 0

ww =
1h
2a
1
t
sin at(u)0 −
1
 cos(at − 2au) t i
−2a 0

w.E
h i
= t sin at + (cos(−at) − cos at)
2a 2a
1h 1 i t sin at
= t sin at − 0 = .
2a 2a 2a

asy "
s2
#

En
−1
Example 5.156. Find L using convolution theorem. [Dec 2012]
(s2 + 4)2

Solution.L −1
"
s2
(s2 + 4)2
#
= L gin
−1
 s
·
s 
s2 + 4 s2 + 4
= L−1 2
ee
 s 
s +4
= cos 2t ∗ cos 2t
 s 
∗ L−1 2
s +4

rin
=
Zt
cos 2u · cos 2(t − u)du g.n
=
0
Zt
cos 2u cos(2t − 2u)du
et
0
Zt !
cos(2u + 2t − 2u) + cos(2u − 2t + 2u)
= du
2
0
Zt
1
= (cos 2t + cos[4u − 2t])du
2
0

Downloaded From : www.EasyEngineering.net


Downloaded From : www.EasyEngineering.net

596 Engineering Mathematics - II

Zt Zt
 
1 
 
= cos 2t du + cos(4u − 2t)du
2 
0 0
" " #t #
1 t sin(4u − 2t)
= cos 2t[u]0 +
2 4 0
" #
1 1
= t cos 2t + {sin 2t − sin(−2t)}
2 4
" #
1 1
= t cos 2t + 2 sin 2t
2 4
" #
1 sin 2t 1

ww Example 5.157. Find L−1


=
2
h
t cos 2t +

1
2
i
= (sin 2t + 2t cos 2t).
4

w.E
Solution. L−1
h 1
(s + 1)(s2 + 2s + 2)
i
−1
h
using convolution theorem.

1 i
−t −1
h
[May 2007]
1 i

asy
= L = e L
(s + 1)(s2 + 2s + 2) (s + 1)((s + 1)2 + 1) s(s2 + 1)
h  1 1 i h 1  1 i
= e−t L−1 . 2 = e−t L−1 ∗ L−1 2
s s +1 Z s s +1
−t
h
= e 1 ∗ sin t = e
i
−t
Ent
sin u.1du

gin
0
= e−t (− cos u)t0 = −e−t (cos t − 1) = e−t (1 − cos t).
h4 i
Example 5.158. Using convolution theorem, find L−1

h
Solution. L−1 2
4
(s + +2s + 5) 2
i
= L−1
h ee 4
2
(s + 1) + 4) 2
i
(s2 + 2s + 5)2

= e−t L−1 2
h 4
(s + 4) 2
i
.
[Jun 2013, May 2006]

rin (1)

Now, L−1 2
h 4
(s + 4)2
i
= L −1
h 2 2 i
s2 + 4 s2 + 4
= L −1
h 2 i
s2 + 4
∗ L −1
h 2 i
g.n
s2 + 4

= sin 2t ∗ sin 2t =

=
1 t
Z
Z t

0
sin 2u sin 2(t − u)du

cos{2u − 2(t − u)} − cos{2u + 2t − 2u} du



et
2 0
1h t
Z Z t i
= cos(4u − 2t)du − cos 2tdu
2 0 0
1 h sin(4u − 2t) t i
= − cos 2t(u)t0
2 4 0
1 h sin 2t sin 2t i 1 h sin 2t i
= + − t cos 2t = − t cos 2t .
2 4 4 2 2

Downloaded From : www.EasyEngineering.net


Downloaded From : www.EasyEngineering.net

Laplace Transform 597

h 1 i 1h i
From(1) L−1 = sin 2t − 2t cos 2t e−t .
(s2 + +2s + 5)2 4

Exercise 5 L

Find the Laplace inverse of the following using convolution theorem.

1
5. . [May 2008]

ww
1 s(s2 + 1)
1. 2 .
(s + 1)2
1
6. . [May 2008]

w.E 2. 2
s
(s + 4)2
. [May 2005]
7.
(s2

s3 (s2
+ 4)2
2
+ 5)
.

3.
s2
(s2 + a2 )2
.
asy [Dec 2006]
8.
(s2
1
.

En
+ 4)(s + 2)
s2 s2

gin
4. . 9. .
(s4 − a4 ) (s − 3)(s2 + 5)

residues. ee
Find the inverse Laplace transform of the following functions using the method of

rin
10.
2
(s + 1)(s2 + 1)
.
12.
1
(s − 1) (s2 + 1)
2
.
g.n
11.
2s
2s2 + 1
. 13.
(s +
1
1)3 (s − 2)2
. et
5.7 Solution of linear second order differential equations
✎ ☞
Worked Examples
✍ ✌
Example 5.159. Solve y′′ + 5y′ + 6y = 2 given y′ (0) = 0 and y(0) = 0 using Laplace
transform method. [Jun 2013]

Downloaded From : www.EasyEngineering.net


Downloaded From : www.EasyEngineering.net

598 Engineering Mathematics - II

Solution. y′′ + 5y′ + 6y = 2.


Taking Laplace transform both sides we get

L[y′′ ] + 5L[y′ ] + 6L[y] = L[2]

s2 L[y] − sy(0) − y′ (0)+5[sL[y] − y(0)] + 6L[y] = 2 · L[1]


2
L[y][s2 + 5s + 6] =
s
2
L[y] =

ww =
s(s2 + 5s + 6)
2
s(s + 2)(s + 3)

w.E
" #
−1 2
y= L
s(s + 2)(s + 3)
2 A B C
Let
asy
s(s + 2)(s + 3)
= +
s s+2 s+3
+
A(s + 2)(s + 3) + Bs(s + 3) + Cs(s + 2)

En
=
s(s + 2)(s + 3)
∴ 2 = A(s + 2)(s + 3) + Bs(s + 3) + Cs(s + 2).

gin 1
When s = 0, 6A = 2 ⇒ A = .
3

ee
When s = −2, −2B = 2 ⇒ B = −1.

When s = −3, 3C = 2 ⇒ C = .
2
3 rin

2 1 1
= · −
1
+ ·
s(s + 2)(s + 3) 3 s s + 2 3 s + 3
2 1
g.n
et
" #
−1 2
Now y = L
s(s + 2)(s + 3)
" #
−1 1 1 1 2 1
=L · − + ·
3 s s+2 3 s+3
" # " # " #
1 1 1 2 1
= L−1 − L−1 + L−1
3 s s+2 3 s+3
1 2
= × 1 − e−2t + e−3t
3 3
1 −2t 2 −3t
y= −e + e .
3 3

Downloaded From : www.EasyEngineering.net


Downloaded From : www.EasyEngineering.net

Laplace Transform 599

d2 x dx dx
Example 5.160. Solve 2
− 3 + 2x = 2 given x = 0 and = 5 for t = 0 using
dt dt dt
Laplace transform method. [Dec 2012, May 2011]
d2 x dx
Solution. 2 − 3 + 2x = 2.
dt dt
Taking Laplace transform both sides we get
" 2 # " #
d x dx
L 2
− 3L + 2L[x] = L[2]
dt dt
s2 L[x] − s✟ ✟ − x′ (0) − 3[sL[x] − x(0)] ✟ + 2L[x] = 2
x(0) ✟
s
2

ww s2 L[x] − 5 − 3sL[x] + 2L[x] =

L[x][s2 − 3s + 2] = + 5 =
2
s
5s +2

w.E L[x] =
5s + 2
=
s
5s + 2
s(s2 − 3s + 2) s(s − 1)(s − 2)
s

asy
" #
−1 5s + 2
x=L .
s(s − 1)(s − 2)

Let
En
5s + 2
s(s − 1)(s − 2)
A
= +
B
s s−1 s−2
+
C

gin =
A(s − 1)(s − 2) + Bs(s − 2) + Cs(s − 1)
s(s − 1)(s − 2)

ee
5s + 2 = A(s − 1)(s − 2) + Bs(s − 2) + Cs(s − 1).

When s = 0, 2A = 2 ⇒ A = 1.
rin
When s = 1, −B = 7 ⇒ B = −7.

When s = 2, 2C = 12 ⇒ C = 6. g.n

5s + 2 1
= −
7
s(s − 1)(s − 2) s s − 1 s − 2

Now, x = L −1
"
+

5s + 2
6
.
#
et
s(s − 1)(s − 2)
" #
−1 1 7 6
=L − +
s s−1 s−2
" # " # " #
−1 1 −1 1 −1 1
=L − 7L + 6L
s s−1 s−2
x = 1 − 7et + 6e2t .

Downloaded From : www.EasyEngineering.net


Downloaded From : www.EasyEngineering.net

600 Engineering Mathematics - II

d2 y dy
Example 5.161. Solve using Laplace transforms 2
− 2 − 8y = 0 given
dt dt
y(0) = 3, y′ (0) = 6. [May 2008]
d2 y dy
Solution. The given differential equation is 2
− 2 − 8y = 0.
dt dt
Taking Laplace transform on both sides, we get
s2 L[y] − sy(0) − y′ (0) − 2[sL(y) − y(0)] − 8L[y] = 0

L[y][s2 − 2s − 8] − 3s − 6 + 6 = 0

L[y](s − 4)(s + 2) = 3s

ww L[y] =

y = L−1
h
3s
(s − 4)(s − 2)
3s i

w.E
.
(s − 4)(s − 2)
3s A B
Let = +
(s − 4)(s − 2) s − 4 s − 2

asy
3s = A(s + 2) + B(s − 4).

En
When s = 4, 6A = 12 ⇒ A = 2.

When s = −2, −6B = 6 ⇒ B = −1.

y = L−1
h 2

s−4 s−2
1 i
gin
= 2e4t − e2t .

transforms.
ee
Example 5.162. Solve (D2 +5D+6)y = e−t given that y(0) = 0, y′ (0) = 0 using Laplace

rin [Nov 2003]


Solution. Given
d2 y
dt2
(D2
+ 5D + 6)y =
dy
+ 5 + 6y = e−t .
dt
e−t

g.n
Taking
L
" 2 #
d y
dt 2
Laplace
+ 5L
" # transform on both sides we get
dy
dt
+ 6L[y] = L[e−t ]
1
et
s2 L[y] − sy(0) − y′ (0) + 5[sL(y) − y(0)] + 6L[y] =
s+1
2 1
s L[y] + 5sL(y) + 6L[y] =
s+1
2 1
L[y](s + 5s + 6) =
s+1
1
L[y] =
(s + 1)(s2+ 5s + 6)

Downloaded From : www.EasyEngineering.net


Downloaded From : www.EasyEngineering.net

Laplace Transform 601

" # " #
−1 1 −1 1
y=L =L (1)
(s + 1)(s2 + 5s + 6) (s + 1)(s + 2)(s + 3)

1 A B C
Let = + + .
(s + 1)(s + 2)(s + 3) s + 1 s + 2 s + 3
1 = A(s + 2)(s + 3)+B(s + 1)(s + 3) + C(s + 1)(s + 2).

When s = −2 When s = −3
When s = −1,

ww 1 = A(1)(2)
1
1 = B(−1)(1)
B = −1.
1 = C(−2)(−1)
C= .
1
2

w.E A= .
2

1
= 2 −
1
1
+ 2
1

asy

(s + 1)(s + 2)(s + 3) s + 1 s + 2 s + 3
" # " # " # " #
−1 1 1 −1 1 −1 1 1 −1 1
L = L −L + L

En
(s + 1)(s + 2)(s + 3) 2
1
s+1
1
y = e−t − e−2t + e−3t .
s+2 2 s+3

gin 2 2

d2 y dy

y(0) = 1, y′ (0) = 0. ee
Example 5.163. Solve using Laplace transforms

Solution. The given differential equation is


d2 y
2
dy
dt2

− 4 + 3y = e−t .
− 4 + 3y = e−t given
dt

rin [May 2004]

g.n
dt dt
Taking Laplace on both sides, we get
 d2 y i h dy i
+ 3L[y] = L[e−t ]

et
L 2 − 4L
dt dt
1
s2 L[y] − sy(0) − y′ (0) − 4[sL(y) − y(0)] + 3L[y] =
s+1
1
L[y][s2 − 4s + 3] − s + 4 =
s+1
2 1
L[y][s − 4s + 3] = s − 4 +
s+1
(s − 4)(s + 1) + 1
L[y][(s − 3)(s − 1)] =
s+1
s2 − 3s − 4 + 1 s2 − 3s − 3
L[y] = = .
(s + 1)(s − 1)(s − 3) (s − 3)(s − 1)(s + 1)

Downloaded From : www.EasyEngineering.net


Downloaded From : www.EasyEngineering.net

602 Engineering Mathematics - II

h s2 − 3s − 3 i
y = L−1 .
(s − 3)(s − 1)(s + 1)
s2 − 3s − 3 A B C
Let = + +
(s − 3)(s − 1)(s + 1) s − 3 s − 1 s + 1
s2 − 3s − 3 = A(s − 1)(s + 1) + B(s − 3)(s + 1) + C(s − 3)(s − 1).
3
When s = 3, 8A = −3 ⇒ A = − .
8
5
When s = 1, −4B = −5 ⇒ B = .
4
1

ww
When s = −1, 8C = 1 ⇒ C = .
8
h −3 5 1 i
8 4 8 3 5 1
y=L −1
+ + = − e3t + et + e−t .

w.E s−3 s−1 s+1 8 4 8


d2 y dy
Example 5.164. Solve the differential equation 2 − 3 + 2y = e−t with y(0) = 1,
dt dt

d2 y
dt
dy
asy
and y′ (0) = 0 using Laplace transforms.
Solution. 2 − 3 + 2y = e−t .
dt
[Jun 2012]

" 2 # " #
En
Taking Laplace transform both sides we get

gin
d y dy
L 2
− 3L + 2L[y] = L[e−t ]
dt dt
✟✟ − 3[sL[y] − y(0)] + 2L[y] = 1
s2 L[y] − sy(0) − ✟y′ (0)

ee
s2 L[y] − s − 3sL[y] + 3 + 2L[y] =
1
s+1
s+1

rin
L[y][s2 − 3s + 2] =
s+1
1
+s−3
g.n
L[y](s − 1)(s − 2) =

=
1 + (s − 3)(s + 1)
s+1
1 + s2 − 2s − 3
s+1
et
s2 − 2s − 2
=
s+1
s2 − 2s − 2
L[y] = .
(s + 1)(s − 1)(s − 2)
s2 − 2s − 2
" #
−1
y= L .
(s + 1)(s − 1)(s − 2)

Downloaded From : www.EasyEngineering.net


Downloaded From : www.EasyEngineering.net

Laplace Transform 603

s2 − 2s − 2 A B C
Let = + +
(s + 1)(s − 1)(s − 2) s + 1 s − 1 s − 2
A(s − 1)(s − 2) + B(s + 1)(s − 2) + C(s + 1)(s − 1)
= .
(s + 1)(s − 1)(s − 2)
∴ s2 − 2s − 2 = A(s − 1)(s − 2) + B(s + 1)(s − 2) + C(s + 1)(s − 1).
3
When s = 1, −2B = −3 ⇒ B = .
2
1
When s = −1, 6A = 1 ⇒ A = .
6
2
When s = 2, 3C = −2 ⇒ C = − .

ww ∴
s2 − 2s − 2 1
= ·
1
3
+ ·
3 1
(s + 1)(s − 1)(s − 2) 6 s + 1 2 s − 1 3 s − 2
− ·
2 1

w.E Now, y = L −1
"
s2 − 2s − 2
(s + 1)(s − 1)(s − 2)
#

asy
" #
−1 1 1 3 1 2 1
=L · + · − ·
6 s+1 2 s−1 3 s−2

En
1 3 2
y = e−t + et − e2t .
6 2 3

gin
Example 5.165. Using Laplace transform, solve the differential equation
y′′ − 3y′ − 4y = 2e−t with y(0) = 1 = y′ (0). [Dec 2010, Jun 2010]
Solution. y′′ − 3y′ − 4y = 2e−t .
ee
Taking Laplace transform both sides we get
rin
L[y′′ ] − 3L[y′ ] − 4L[y] = 2L[e−t ]
2 g.n
s2 L[y] − sy(0) − y′ (0) − 3[sL[y] − y(0)] − 4L[y] =

s2 L[y] − s − 1 − 3[sL[y] − 1] − 4L[y] =


2
2
s+1
s+1
et
L[y][s2 − 3s − 4] − s − 1 + 3 =
s+1
2
L[y](s − 4)(s + 1) − s + 2 =
s+1
2
L[y](s − 4)(s + 1) = +s−2
s+1
2 + (s + 1)(s − 2) 2 + s2 − s − 2 s2 − s
= = =
s+1 s+1 s+1

Downloaded From : www.EasyEngineering.net


Downloaded From : www.EasyEngineering.net

604 Engineering Mathematics - II

s2 − s
L[y] =
(s + 1)(s − 4)(s + 1)
s2 − s
= .
(s − 4)(s + 1)2
s2 − s
" #
−1
y=L .
(s − 4)(s + 1)2
s2 − s A B C
Let 2
= + +
(s − 4)(s + 1) s − 4 s + 1 (s + 1)2
2
A(s + 1) + B(s − 4)(s + 1) + C(s − 4)
=
(s − 4)(s + 1)2

ww s2 − s = A(s + 1)2 + B(s − 4)(s + 1) + C(s − 4).

w.E When s = 4, 25A = 12 ⇒ A =

When s = −1, −5C = 2 ⇒ C = − .


12
25
2
.

A+B=1 ⇒
asy
+B=1⇒ B=1−
5
Equating the coefficient of s2 we get
12 12 13
= .
25
s2 − s En 25 25

gin
12 1 13 1 2 1
∴ 2
= · + · − ·
(s − 4)(s + 1) 25 s − 4 25 s + 1 5 (s + 1)2
s2 − s
" #

ee
−1
Now, y = L
(s − 4)(s + 1)2

rin
" #
−1 12 1 13 1 2 1
=L · + · − ·
25 s − 4 25 s + 1 5 (s + 1)2

g.n
" #
12 4t 13 −t 2 −t −1 1
= e + e − e L
25 25 5 s2
12 13
y = e4t + e−t − te−t .
25 25
2
5

Example 5.166. Solve using Laplace transforms,


d2 y dy
+ = t2 + 2t, given that
et
dt2 dt
y = 4, y′ = −2 when t = 0. [Dec 2013, May 2007]
d2 y dy 2
Solution. The given differential equation is 2 + = t + 2t.
dt dt
Taking Laplace transform on both sides, we get

2 2
s2 L[y] − sy(0) − y′ (0) + sL(y) − y(0) − y(0) = 3
+ 2
s s

Downloaded From : www.EasyEngineering.net


Downloaded From : www.EasyEngineering.net

Laplace Transform 605

2(s + 1)
(s2 + s)L[y] − 4s + 2 − 4 =
s3
2(s + 1) 2(s + 1)
s(s + 1)L(y) = 4s + 2 + 3
= 2s + 2 + 2s +
s s3
2(s + 1)
= 2(s + 1) + 2s +
s3
2 2 2
L(y) = + +
s s + 1 s4
h2 2 2i
y = L−1 + + 4
s s+1 s
2
= 2 + 2e + t3
−t

ww
3!
1
y = 2 + 2e + t3 .
−t
3

w.E
Laplace transforms.
d2 y
dt
dy
Example 5.167. Solve 2 + 4 + 4y = sin t, if
dt
dy
dt
= 0 and y = 2 when t = 0, using
[Dec 2011]
d2 y dy

asy
Solution. 2 + 4 + 4y = sin t.
dt dt
Taking Laplace transform both sides we get

L
" 2 #
d y
dt2
+ 4L
En
" #
dy
+ 4L[y] = L[sin t]

gin
dt
s2 L[y] − sy(0) − ✟ ✟✟ + 4[sL[y] − y(0)] + 4L[y] = 1
y′ (0)
s2 + 1

ee
s2 L[y] − 2s + 4[sL[y] − 2] + 4L[y] = 2

L[y][s2 + 4s + 4] − 2s − 8 = 2
1
1
s +1

rin
g.n
s +1
1
L[y](s + 2)2 = 2 + 2s + 8
s +1
=

=
1 + (2s + 8)(s2 + 1)

3
s2 + 1
1 + 2s + 2s + 8s2 + 8
et
s2 + 1
2s + 8s2 + 2s + 9
3
=
s2 + 1
2s3 + 8s2 + 2s + 9
L[y] =
(s + 2)2 (s2 + 1)
" 3 2
#
−1 2s + 8s + 2s + 9
y=L .
(s + 2)2 (s2 + 1)

Downloaded From : www.EasyEngineering.net


Downloaded From : www.EasyEngineering.net

606 Engineering Mathematics - II

2s3 + 8s2 + 2s + 9 A B Cs + D
Let 2 2
= + 2
+ 2
(s + 2) (s + 1) s + 2 (s + 2) s +1
A(s + 2)(s2 + 1) + B(s2 + 1) + (Cs + D)(s + 2)2
=
(s + 2)2 (s2 + 1)
∴ 2s3 + 8s2 + 2s + 9 = A(s + 2)(s2 + 1) + B(s2 + 1) + (Cs + D)(s + 2)2 .

21
When s = −2, 5B = 21 ⇒ B = .
5
Equating the coefficients of s3 , we get
A + C = 2. (1)

ww Equating the constants we get


2A + B + 4D = 9.

w.E 2A +
21
5
+ 4D = 9

2A + 4D = 9 −
21 24
= (2)
5
asy
5
Equating the coefficients of s2 we get

En
2A + B + 4C + D = 8.

gin
21
2A + + 4C + D = 8
5
21
2A + 4C + D = 8 −

2A + 4C + D =
19
5
ee
.
5

rin (3)

From (1), C = 2 − A.
19 g.n
(3) ⇒ 2A + 4(2 − A) + D =

2A + 8 − 4A + D =
5
19
5
19 21
et
−2A + D = −8=−
5 5
21
2A − D = . (4)
5
24 21 3
(2) − (4) ⇒ 5D = − = .
5 5 5
3
D= .
25

Downloaded From : www.EasyEngineering.net


Downloaded From : www.EasyEngineering.net

Laplace Transform 607

3 21
(4) ⇒ 2A − =
5 5
21 3 24
2A = + = .
5 5 5
12
A= .
5
12 2
C =2−A=2− =− .
5 5

2s3 + 8s2 + 2s + 9 12 1 21 1 2 s 3 1
∴ = + · − +
(s + 2)2 (s2 + 1) 5 s+2 5 (s + 2)2 5 s2 + 1 5 s2 + 1

ww
" 3 2
#
−1 2s + 8s + 2s + 9
Now, y = L
(s + 2)2 (s2 + 1)

w.E
" #
−1 12 1 21 1 2 s 3 1
=L + · − +
5 s+2 5 (s + 2)2 5 s2 + 1 5 s2 + 1
" # " # " #
12 −1 1 21 −1 1 2 −1  s  3 −1 1

asy
= L + L − L + L
5 s+2 5 (s + 2)2 5 s2 + 1 5 s2 + 1
" #
12 21 1 2 3
= e−2t + e−2t L−1 2 − cos t + sin t
5 5
12 −2t 21 −2t
En 2
= e + e t − cos t + sin t.
s 5
3
5

5 5
d2 y gin
5
dy
5
dy

ee
Example 5.168. Solve + 4 + 8y = cos 2t, y = 2 and = 1 where t = 0.
dt2 dt dt
[May 2002]
d2 y dy
Solution. The given differential equation is 2 + 4 + 8y = cos 2t.
dt dt rin
Taking Laplace transform on both sides, we get
" 2 #
d y
" #
dy g.n
L
dt2
+ 4L
dt
+ 8L[y] = L[cos 2t]

s2 L[y] − sy(0) − y′ (0) + 4[sL(y) − y(0)] + 8L[y] = 2


s
s
s +4
et
s2 L[y] − 2s − 1 + 4[sL(y) − 2] + 8L[y] = 2
s +4
2 s
L[y][s + 4s + 8] − 2s − 1 − 8 = 2
s +4
2 s
L[y][s + 4s + 8] = 2s + 9 + 2
s +4
2
(2s + 9)(s + 4) + s
=
s2 + 4

Downloaded From : www.EasyEngineering.net


Downloaded From : www.EasyEngineering.net

608 Engineering Mathematics - II

2s3 + 8s + 9s2 + 36 + s 2s3 + 9s2 + 9s + 36


= =
s2 + 4 s2 + 4
3 2
2s + 9s + 9s + 36
L[y] = 2
(s + 4)(s2 + 4s + 8)
h 2s3 + 9s2 + 9s + 36 i
y = L−1 2 .
(s + 4)(s2 + 4s + 8)
2s3 + 9s2 + 9s + 36 As + B Cs + D
Let 2 2
= 2 + 2
(s + 4)(s + 4s + 8) s +4 s + 4s + 8

2s3 + 9s2 + 9s + 36 = (As + B)(s2 + 4s + 8) + (Cs + D)(s2 + 4)

ww Equating the coefficients of s3 we get

A + C = 2. (1)

w.E Equating the coefficients of s2 we get

asy 4A + B + D = 9. (2)

En
Equating the coefficients of s we get

gin8A + 4B + 4C = 9. (3)

ee
Equating the constants we get, 8B + 4D = 36

2B + D = 9.
rin (4)

(3) =⇒ 8A + 4B + 4(2 − A) = 9 [using (1)]


g.n
8A + 4B + 8 − 4A = 9

4A + 4B = 1.
et (5)

(2) =⇒ 4A + B + 9 − 2B = 9 [from (4)]

4A − B = 0

B = 4A (6)
1
(5) =⇒ 4A + 4(4A) = 1 =⇒ 20A = 1 =⇒ A = .
20

Downloaded From : www.EasyEngineering.net


Downloaded From : www.EasyEngineering.net

Laplace Transform 609

1 1
(6) =⇒ B = 4 = .
20 5

1 39
(1) =⇒ C = 2 − A = 2 − = .
20 20

2 43
(4) =⇒ D = 9 − 2B = 9 − = .
5 5

ww ∴
2s3 + 9s2 + 9s + 36
=
1 1
20 s + 5
+
39 43
20 s + 5

w.E (s2 + 4)(s2 + 4s + 8) s2 + 4 s2 + 4s + 8

asy
 1
 s + 51 
 39
 20 s + 43
 
2s3 + 9s2 + 9s + 36
" #
−1  20 5
−1 −1

Now, y = L 2 2
= L  2  + L 
2

(s + 4)(s + 4s + 8) s +4 s + 4s + 8 

En
=
1 −1  s  1 −1
L + L
"
1
gin
#
+ L
39 −1  s  43
+ L −1
"
1
#

=
20
1
20
1
s2 + 4

cos 2t +
1
10
1
5

sin 2t + L
20
s2 + 4
39 −1
"
ee 20
s
(s + 2)2 + 4
"
39 −2t −1 s − 2
#
s2 + 4s + 8
#
43 −1
+ L
5
43 −2t −1
"
5
1
(s + 2)2 + 4
"
1
#
s2 + 4s + 8
#

rin
=
20
1
cos 2t +
10
1
sin 2t + e L
20
39
s2 + 4
39
+ e L
5
43
s2 + 4
g.n
=

=
20
1
20
cos 2t +

cos 2t +
10
1
10
sin 2t + e−2t cos 2t − e−2t sin 2t + e−2t sin 2t
20
39 −2t
20
20
47 −2t
sin 2t + e cos 2t + e sin 2t.
20
5
et
π
Example 5.169. Solve y′′ (t) + 9y(t) = 18t given that y(0) = 0, y = 0. [May 2005]
π 2
Solution. The given initial conditions are y(0) = 0, y = 0.
2
′ ′
Since y (0) is not given, let us assume that y (0) = k.
The given Differential equation is y′′ + 9y = 18t.

Downloaded From : www.EasyEngineering.net


Downloaded From : www.EasyEngineering.net

610 Engineering Mathematics - II

Taking Laplace transform both sides we get

L[y′′ ] + 9L[y] = 18L[t]


1
s2 L[y] − sy(0) − y′ (0) + 9L[y] = 18
s2
18
s2 L[y] − k + 9L[y] =
s2
18 ks2 + 18
L[y](s2 + 9) = k + =
s2 s2

ww L[y] =
ks2 + 18
s2 (s2 + 9)
" 2
−1 ks + 18
#

w.E y=L
s2 (s2 + 9)

ks2 + 18 A B
(1)

Let =

asy
+
s2 (s2 + 9) s2 s2 + 9
ks2 + 18 = A(s2 + 9) + Bs2 .
.

En Equating the Coeff. of s2 we get


When s = 0,
9A = 18
gin A+B=k
B = k − A = k − 2.
A = 2.


ks2 + 18
2 2
s (s + 9) " s
2
= 2+ 2
k−2 ee
s + #9
.
rin
2

g.n
" # " #
−1 ks + 18 −1 1 −1 1
Now, y = L = 2L + (k − 2)L .
s2 (s2 + 9) s2 s2 + 9
y = 2t +

Now y
k−2
 π 3
2
π k−2
=0
sin 3t.

3π k−2
et
∴0=2 + sin =π+ (−1).
2 3 2 3
k−2

3
k − 2 = 3π
k = 3π + 2.
3π + 2 − 2
∴ The solution is y = 2t + sin 3t.
3

Downloaded From : www.EasyEngineering.net


Downloaded From : www.EasyEngineering.net

Laplace Transform 611

y = 2t + π sin 3t.

Exercise 5 M

Solve the following differential equations using Laplace transforms

1. (D2 + 4D + 4)y = e−t given that y(0) = 0, y′ (0) = 0. [May 2006]


d2 y dy
2. + − 2y = 3 cos 3t − 11 sin 3t with y(0) = 0, y′ (0) = 6. [May 2008]
dt2 dt

ww π
3. (D2 + 9)y = cos 2t given that y(0) = 1, y( ) = −1.
2

w.E 4.
d2 x
dt2
dx
− 2 + x = et , given that x = 2,
dt
dx
dt
= −1 when t = 0.

5. y′′ + 5y′ + 6y = 2 given that y(0) = 1, y′ (0) = 0.


[Dec 2008]

asy
[May 2006]

6. y′′ − 3y′ + 2y = 4 given that y(0) = 2, y′ (0) = 3. [May 2005]

En
7. y′′ + 2y′ − 3y = 3, given y(0) = 4, y′ (0) = −7. [May 2007]

8.
d2 x
dt2
dx
dt gin
+ 2 + 5x = e−t sin t, given that x(0) = 0, x′ (0) = 1. [May 1998]

9.
d2 y
dt 2

d2 y
dy
dt
dy
ee
+ 4 + 4y = 10 sin t, given y = 0 =
dy
dt
when t = 0.

rin
[May 2004]

10.
dt 2

d2 y
+ 6 + 9y = 2e−3t , y(0) = 1, y′ (0) = −2.
dt
g.n [May 2002]

et
dy
11. 2 2
− 5 + 2y = 3 sin t, given that y(0) = y′ (0) = 0. [Apr 1996]
dt dt
12. y′′ − 3y′ + 2y = e−t , given y(0) = 1, y′ (0) = 0. [May 2000]

13. y′′ − 2y′ + y = et , given y(0) = 2, y′ (0) = 1. [May 2005]


d2 x dx dx
14. 2
+ 2 + x = te−t , given that x(0) = 0, = 2 for t = 0. [Apr 2005]
dt dt dt
d2 y dy dy
15. + 2 − 3y = sin t, given, y = 0 = when t = 0. [May 2005]
dt2 dt dt

Downloaded From : www.EasyEngineering.net


Anna University
Downloaded Solved Question Papers
From : www.EasyEngineering.net 229

1
Also | f (z)| = | | → 0 as |z| → ∞.
z2 + a2 Z
∴ By Jordon’s lemma, eimz f (z)dz → 0 as R → ∞.
s
ZR Z
π
Now (1)⇒ e−am = eimx f (x)dx + eimz f (z)dz.
a s
−R
Taking limit as R → ∞ we get

ZR Z
π −am
e = lim eimx f (x)dx + lim eimz f (z)dz
a R→∞ R→∞ s
−R
Z∞
ww π −am
a
e =
eimx
x2 + a2
dx + 0

w.E =
−∞
Z∞
cos mx + i sin mx
dx.
a syE −∞
x2 + a2

Equating the real parts we get

Z∞ ngi
cos mx
x2 + a2
dx =
πe−am
a nee
−∞
Z∞
πe−am rin
2
cos mx
2
x +a 2
dx =
a g.n
0
Z∞
emx
x2+a 2
dx =
πe−am
2a
.
et
0

2.5 Unit V Laplace Transforms.

2.5.1 May/June 2016 [R 2013]

Part A

5. State convolution theorem on laplace transforms.


Solution. Refer page 289 of the main text book.

Downloaded From : www.EasyEngineering.net


———-
230
Downloaded Supplement to Engineering Mathematics - II
From : www.EasyEngineering.net

s  
6. Find L−1 .
s2 + 4s + 5
 s   s 
Solution.L−1 2 = L−1 2
s + 4s + 5 s + 4s + 4 +! 1
−1 s+2−2
=L
(s + 2)2 + 1
! !
s+2 1
= L−1 − 2L −1
(s + 2)2 + 1 (s + 2)2 + 1
 s  !
1
= e−2t L−1 2 − 2e−2t L−1 2
s +1 s +1
= e−2t cos t − 2e−2t sin t

ww
Part B

w.E
13. (a) (i) Evaluate:
 

(2) L
a
(1) L t2 e−t cos t
−1 −2s
e
1
syE
(s2 + s + 1)2
!
.
Solution.
  d2 n h ngi
io
(1)L t2 e−t cos t = 2 L e−t cos t
ds nee
d2
= 2 {L [cos t] s→s+1 }
ds rin
d2  s 
= 2 2 g.n
=
ds s + 1 s→s+1
" 2
d s
#

ds2 s2 + 1 s→s+1
et
" #
d s2 + 1 − s(2s)
=
ds (s2 + 1)2 s→s+1
" #
d s2 + 1 − 2s2
=
ds (s2 + 1)2 s→s+1
" #
d 1 − s2
=
ds (s2 + 1)2 s→s+1
" 2 #
(s + 1)2 (−2s) − (1 − s2 )(2s)
=
(s2 + 1)4 s→s+1
" 2 2 2 2
#
(s + 1) (−2s) − (1 − s )2(s + 1)(2s)
=
(s2 + 1)4 s→s+1

Downloaded From : www.EasyEngineering.net


———-
Anna University
Downloaded Solved Question Papers
From : www.EasyEngineering.net 231

" #
(s2 + 1)(−2s) − (1 − s2 )2(2s)
=
(s2 + 1)3 s→s+1
" #
−2s3 − 2s − 4s + 4s3
=
(s2 + 1)3 s→s+1
" 3 #
(2s − 6s
=
(s2 + 1)3 s→s+1
" #
2s(s2 − 3)
=
(s2 + 1)3 s→s+1
" #
2(s + 1)(s2 + 2s − 2)
= .
(s2 + 2s + 2)3

ww (2)LetF(s) =
1

w.E = 
(s2
+ s + 1)2
1
2 2 = 
1
2 2

a s2 + 12 + 1 − 41



syE 


s2 + 21 + 34






−1 −1 
L [F(s)] = L  
1
 2 1 2 3 2  


ngi
= e− 2
t
L −1






s + 2 
1
√ 2

3 



nee
s +2 +4 2

 √  √   √ 
1   3   3   3 
=e − 2t

2 23 rin
 √ 3 sin  2 t −  2 t cos  2 t

− 2t 4 
 √  √ 
  3   3 
 √ 
 3  g.n
−1
L [F(s)] = e √ sin 
"
3
1
2
t − 
#
2
t cos 
h
2
i
t .
et
Now L−1 e−2s 2 2
= L−1 e−2s F(s)
(s + s + 1)
= L−1 [F(s)]t→t−2 (by second shifting theorem.)
  √  √   √ 


 − 2t 4   3   3   3 


=e √ sin  t −  t cos  t

 3 2 2 2 
t→t−2
 √  √  √ 
(t−2) 4    
  3(t − 2)   3(t − 2)    3(t − 2) 
= e− 2 √ sin  −
   cos 
  .
3 2 2 2

s
13. (a) (ii) Find the inverse Laplace transform of
(s2 + a2 )(s2 + b2 )
using convolution theorem.

Downloaded From : www.EasyEngineering.net


———-
232
Downloaded Supplement to Engineering Mathematics - II
From : www.EasyEngineering.net

Solution. Refer Example 3.154 on page 292 of the main text


book.





E if 0 < t < a/2
13. (b) (i) Find the Laplace transform of f (t) = 



−E if a/2 < t < a
where f (t + a) = f (t)
Solution. Refer Example 3.48 on page 235 of the main text
book.

13. (b) (ii) Using Laplace transform technique solve y′′ + y′ = t2 + 2t,

ww given y = 4, y′ = −2 when t = 0.

w.E
Solution. Refer Example 3.166 on page 302 of the main text
book.

a syE
2.5.2 Dec.2015/Jan. 2016 [R 2013]

Part A ngi
 t
Z



 F(s)
5. Prove that L  f (t) dt =
nee
  s
rin
, where L ( f (t)) = F(s).

g.n
0
Solution. Refer Example 3.3.1 on page 241 of the main text
book.

6. Find L−1 log
s 
.
et
s−a  s 
Solution. Let F(s) = log = log s − log(s − a).
s−a
1 1
F ′ (s) = − .
s s−a " #
  ′ 1 1 1 1
We know that L t f (t) = −F (s) = − − = − .
" # " # s s − a
" # s − a s
1 1 1 1
∴ t f (t) = L−1 − = L−1 − L−1 = eat − 1
s−a s s−a s
eat − 1
∴ f (t) = .
t
Part B

Downloaded From : www.EasyEngineering.net


———-
Anna University
Downloaded Solved Question Papers
From : www.EasyEngineering.net 233

  !
e−t − cos t
13. (a) (i) Find L e−t sin2 3t and L .
t
Solution. we have cos 2θ = 1 − 2 sin2 θ.
⇒ 2 sin2 θ = 1 − cos 2θ
1 − cos 2θ
⇒ sin2 θ =
2
2 1 − cos 6t
∴ sin 3t = .
2

  " #
−t 2 −t 1 − cos 6t
L e sin 3t = L e
2
1 h i
ww = L e−t − e−t cos 6t
2
1 h i 1 h i

w.E = L e−t − L e−t cos 6t


2"
1 1
# 2
1
a syE
=

=
2 s+1
1
"
1
#
− L [cos 6t] s→s+1


2
1 s 
2 s+1
1 ngi 1
2 s2 + 36 s→s+1
"
s+1
#
= −
nee
2(s + 1) 2 (s + 1)2 + 36
.

rin
13. (a) (ii) Solve x′′ + 2x′ + 5x = e−t sin t; x(0) = 0 and x′ (0) = 1 using
Laplace transform. g.n
Solution. Given, x′′ + 2x′ + 5x = e−t sin t, x(0) = 0 and x′ (0) = 1.
Taking Laplace transform on both sides we get
et
L[x′′ ] + 2L[x′ ] + 5L[x] = L[e−t sin t]
s2 L[x] − sx(0) − x′ (0) + 2 [sL[x] − x(0)] + 5L[x] = L[sin t] s→s+1
" #
2 1
s L[x] − s × 0 − 1 + 2sL[x] − 2 × 0 + 5L[x] = 2
s + 1 s→s+1
1
s2 L[x] − 1 + 2sL[x] + 5L[x] =
(s + 1)2 + 1

1
L[x](s2 + 2s + 5) = +1
s2 + 2s + 1 + 1

Downloaded From : www.EasyEngineering.net


———-
234
Downloaded Supplement to Engineering Mathematics - II
From : www.EasyEngineering.net

1
= +1
s2
+ 2s + 2
1 + s2 + 2s + 2 s2 + 2s + 3
= = 2 .
s2 + 2s + 2 s + 2s + 2
s2 + 2s + 3
L[x] = 2
(s + 2s + 2)(s2 + 2s + 5)
(s + 1)2 + 2
L[x] = .
((s + 1)2 + 1)((s + 1)2 + 4)
" #
−1 (s + 1)2 + 2
x=L
((s + 1)2 + 1)((s + 1)2 + 4)
" #
−t −1 s2 + 2
x=e L . (1)

ww (s2 + 1)(s2 + 4)

Let
w.E
2
s2 + 2
2
= 2
A
+ 2
B
(s + 1)(s + 4) (s + 1) (s + 4)
.

a ∴ s2 + 2 = A(s2 + 4) + B(s2 + 1)

Put s2 = −1
syE ⇒ 1 = 3A ⇒A=
1
.

Put s2 = −4 ngi
⇒ −2 = −3B ⇒B=
3
2
.

s2 + 2
1 2 nee 3

rin
3 3
∴ 2 = + .
(s + 1)(s2 + 4) (s2 + 1) (s2 + 4)
" # " # " #
L−1 2
s2 + 2
(s + 1)(s2 + 4)
=
1 −1
3
L
1
(s2 + 1)
+
1 −1
3
L
2
(s2 + 4)
. g.n
1 1
= sin t + sin 2t.
3 " 3 #
et
−t 1 1
(1) ⇒ x = e sin t + sin 2t
3 3
−t
e
= [sin t + sin 2t] .
3
!
−1 e−s
13. (b) (i) State second shifting theorem and also find L √ .
s+1
! !
e−s e−s−1+1
Solution.L−1 √ = L−1 √
s+1 s+1
−(s+1)+1
!
e
= L−1 √
s+1

Downloaded From : www.EasyEngineering.net


———-
Anna University
Downloaded Solved Question Papers
From : www.EasyEngineering.net 235

!
e−(s+1) e1
= L−1 √
s+1
−(s+1)
!
e
= eL−1 √
s+1
−s
!
−t −1 e
= ee L √
s
" # " " # √ #
1−t −1 1 1 π
=e L √ since, L √ = √
s t s
" # t→t−1
1
= e1−t √
πt t→t−1
" #
1
ww = e1−t √
π(t − 1)

w.E
13. (b) (ii) Find L−1
3s + 1
(s + 1)4
!
.

a
Solution.L syE
−1 3s + 1
(s + 1)4
!
=L −1 3(s + 1 − 1) + 1
(s + 1)4
!

!
=Lngi
−1 3(s + 1) − 3 + 1
(s + 1)4

=L nee
−1 3(s + 1) − 2
!

−1 3(s + 1)
rin
(s + 1)4
2
!
=L −
(s + 1)4 (s + 1)4
! g.n !
= 3L −1 1
(s + 1)3
!
− 2L −1 1
et
(s + 1)4
!
−t −1 1 −t −1 1
= 3e L − 2e L
s3 s4
−t
! −t
!
3e −1 2 2e −1 3!
= L − L
2! s3 3! s4
−t
3e 2 2e 3 3e 2 e−t 3
−t −t
= t − t = t − t
2 6 2 3
!
3 t e−t t2
= e−t t2 − = (9 − 2t).
2 3 6
πt
13. (b) (iii) Find the Laplace transform for f (t) = sin , such that
a
f (t + a) = f (t).

Downloaded From : www.EasyEngineering.net


———-
236
Downloaded Supplement to Engineering Mathematics - II
From : www.EasyEngineering.net

Solution. Refer Example 3.52 on page 237 of the main text


π
book. To get the correct solution replace ω by .
  πt  π
a
a πa
∴ L sin =  = 2 2  .
a 2
s2 + π2 (1 − e−asπ ) a s + π2 (1 − e−asπ )
a

2.5.3 April/May 2015 [R 2013]


Part A

5. State the sufficiency condition for the existence of Laplace


transform.

ww Solution. Let f (t) be defined for all t ≥ 0 such that (i) f (t) is

w.E
piecewise continuous in the interval [0, ∞) and (ii) f (t) is of
exponential order α > 0, then the Laplace transform of f (t)

a
exists for s > α.
syE
ngi
6. Evaluate using Laplace transform.
Z∞
Solution. By definition
nee
te−2t sin t dt = [L[t sin t]] s=2

rin
0
" #
d
= − {L[sin t]}
"
ds
d
"
1
##s=2 g.n
= −
"
ds s2 + 1 s=2
d h 2 i#
et
= − ·(s + 1)−1
ds s=2
h i
2 −2
= − (−1) · (s + 1) · 2s
s=2
" #
2s 2×2 4
= 2 2
= 2
= .
(s + 1) s=2 (4 + 1) 25

Part B

13. (a) (i) Find the Laplace transform of the triangular wave




t , in 0 < t ≤ c
function f (t) defined by f (t) = 
 and


2c − t , in c < t < 2c,

Downloaded From : www.EasyEngineering.net


———-
Anna University
Downloaded Solved Question Papers
From : www.EasyEngineering.net 237

f (t + 2c) = f (t) for all t.


Solution. Since f (t) is of period 2c,we have
Z 2c
1
L[ f (t)] = e−st f (t)dt
1 − e−2cs 0
"Z c Z 2c #
1 −st −st
= e f (t)dt + e f (t)dt
1 − e−2cs 0 c
1 hZ c Z 2c i
−st
= −2cs
e tdt + e−st (2c − t)dt
1−e 0 c
1 h Z c e−st Z 2c
e−st i
= td( ) + (2c − t)d( )
1 − e−2cs 0 −s c −s
h e−st c Z c e−st Z 2c −st
ww =
1−e
1
−2cs
t
−s 0

0 −s
dt +

(2c − t)
e−st 2c
−s c

c
e
−s
(−dt)
i

w.E =
1
1 − e−2cs −s
h e−cs 1  e−st c
c +
s −s 0
+0+ e −
c −cs 1  e−st 2c i
s s −s c

a=
1−e
1

1
−2cs
h

h syE
−c
e−cs
s
e−cs
1 −cs
− 2 (e − 1) + e + 2 (e
s
1 −cs 1
c −cs 1 −2cs
s
e−cs
s
1
− e−cs )
1
i

i
=
1−e −2cs
−c
s
ngi
− 2e + 2 + c
s
h e−2cs − e−cs − e−cs + 1 i
s s
+ 2 e−2cs − 2 e−cs
s s
1 h 1 − 2e−2cs + e−2cs i
=
1
1 − e−2cs s2 nee =
1 − e−cs s2
=
1
(1 − e−cs )(1 + e−cs )
.
(1 − e )
s2
−cs 2

rin
= 2
(1 − e )−cs

s .(1 + e−cs )

=  cs
cs
e 2 − e− 2
cs

 =
1
tan h
 cs 
. g.n
s2 e 2 + e− 2
(
s
cs

2
)
s2 2
et
13. (a) (ii) Find L−1   .
s2 + 1 s2 + 4
" #
−1 s2
Solution. L
(s2 + 12 )(s2 + 22 )
 s s 
= L−1 2 .
2 2 2
 s +s 1  s + 2  s 
= L−1 2 ∗ L−1 2
s + 12 s + 22
= cos t ∗ cos 2t
Z t
= cos u cos 2(t − u)du
0

Downloaded From : www.EasyEngineering.net


———-
238
Downloaded Supplement to Engineering Mathematics - II
From : www.EasyEngineering.net

Z t
= cos u cos(2t − 2u)du
0
Z
1 t
= {cos(u + 2t − 2u) + cos(u − 2t + 2u)}du
2 0
Z
1 t
= {cos(2t − u) + cos(3u − 2t)}du
2 0
" #t
1 sin(2t − u) sin(3u − 2t)
= +
2 −1 3 0
" #
1 sin t sin 2t sin t sin 2t
= − + +
2 −1 −1 3 3
" #
1 2 −4

ww =
2 −3
1
sin t +
−3
sin 2t

w.E =
2(−3)
[2 sin t − 4 sin 2t]
sin t − 2 sin 2t 2 sin 2t − sin t

a =
−3
syE
=
3
.

13. (b) (i) Solve the differential equation y′′ − 3y′ + 2y = 4t + e3t , where

ngi
y(0) = 1 and y′ (0) = −1 using Laplace transforms.
Solution. y − 3y + 2y = 4t + e3t .
′′ ′
nee
Taking Laplace transform both sides we get
rin
L[y′′ ] − 3L[y′ ] + 2L[y] = 4L[t] + L[e3t ]
4 1g.n
s2 L[y] − sy(0) − y′ (0) − 3[sL[y] − y(0)] + 2L[y] =

2
s L[y] − s + 1 − 3sL[y] + 3 + 2L[y] =
s
4(s − 3) + s2
2
+
s−3 et
s2 (s − 3)
2
4s − 12 + s
L[y][s2 − 3s + 2] − s + 4 =
s2 (s − 3)
4s − 12 + s2
L[y][s2 − 3s + 2] = +s−4
s2 (s − 3)
s2 + 4s − 12 s−4
L[y] = 2 +
s (s − 3)(s2 − 3s + 2) s2 − 3s + 2
(s + 6)✘ −✘
(s ✘ 2) s−4
L[y] = 2 ✘ +
s (s − 3)✘ ✘
(s − 2)(s − 1) (s − 2)(s − 1)
(s + 6) s−4
L[y] = 2 +
s (s − 3)(s − 1) (s − 1)(s − 2)

Downloaded From : www.EasyEngineering.net


———-
Anna University
Downloaded Solved Question Papers
From : www.EasyEngineering.net 239

(s + 6)(s − 2) + s2 (s − 3)(s − 4)
L[y] =
s2 (s − 1)(s − 2)(s − 3)
(s + 6)(s − 2) + s2 (s − 3)(s − 4)
L[y] = (1)
s2 (s − 1)(s − 2)(s − 3)
(s + 6)(s − 2) + s2 (s − 3)(s − 4) A B C D E
Let 2
= + 2+ + +
s (s − 1)(s − 2)(s − 3) s s s−1 s−2 s−3
(s + 6)(s − 2) + s2 (s − 3)(s − 4) = As(s − 1)(s − 2)(s − 3)
+ B(s − 1)(s − 2)(s − 3) + Cs2 (s − 2)(s − 3)
+ Ds2 (s − 1)(s − 3) + Es2 (s − 1)(s − 2).

ww Put s = 0, B(0 − 1)(0 − 2) = (0 + 6)(0 − 2) ⇒ 2B = −12 ⇒ B = −6.


Put s = 1,
w.E C · 1 · (0 − 1)(0 − 2) = 7(−1) + (−3)(0 − 2)
⇒ 2C = −7 + 6 = −1 ⇒ C =
−1
2
.

a
Put s = 2, (−2)4(−1) = D(4)(1)(−4) ⇒ 8 = −4D ⇒ D = −2.
syE
Put s = 3, 9 × 1 = E(9)(2)(1) ⇒ 2E = 1 ⇒ E = .
1
2
Equating the coefficient of s4 we get
ngi
1 = A+C +D+Enee
1
1= A− −2+
2
1
2 rin
1= A−2 ⇒A=3 g.n
−1 

 3 6 1
2
1 

et
y = L  − 2 − 2
− + 2 
s s s−1 s−2 s−3
" # " # " # " # " #
1 1 1 1 1 1 1
= 3L−1 − 6L−1 2 − L−1 − 2L−1 + L−1
s s 2 s−1 s−2 2 s−3
1 t 1
y = 3 · 1 − 6t − e − 2e2t + e3t .
2 2
( )
cos at − cos bt
13. (b) (ii) Find L .
t
" # Z ∞
cos at − cos bt
Solution. L = L[cos at − cos bt]ds
t s

Downloaded From : www.EasyEngineering.net


———-
240
Downloaded Supplement to Engineering Mathematics - II
From : www.EasyEngineering.net
Z 

s s 
= − ds
s s2 + a2 s2 + b2
" #∞
1 s2 + a2
= log 2
2 s + b2 s
" #
1 s2 + b2
= log 2
2 s + a2

2.5.4 November/December 2014 [R 2013]

Part A

ww
5. Find the Laplace transform of e−t sin 2t.

w.E
Solution. L[e−t sin 2t] = [L[sin 2t]] s=s+1
" #
a syE = 2
2
s + 4 s=s+1

=
ngi 2
(s + 1)2 + 4
= 2
2
nee = 2
2
s + 2s + 1 + 4 s + 2s + 5
.

rin
6. Find f (t) if the Laplace transform of f (t) is
s
(s + 1)2
.
g.n
Solution. f (t) = L−1
"
s
(s + 1)2
#
et
" #
−1 s + 1 − 1
=L
(s + 1)2
" #
−t −1 s − 1
=e L
s2
" #
−t −1 1 1
=e L −
s s2
( " # " #)
−t −1 1 −1 1
=e L −L = e−t [1 − t] .
s s2

Part B

Downloaded From : www.EasyEngineering.net


———-
Anna University
Downloaded Solved Question Papers
From : www.EasyEngineering.net 241

13. (a) (i) Find the Laplace transform of the following functions
1 − cos t
(1) e−t t cos t, (2) .
t

h i
Solution. L e−t t cos t = L [t cos t] s→s+1
( )
d
= − L[cos t]
ds
( )s→s+1
d  s 
= −
ds s2 + 1 s→s+1
 

 s2 + 1 − s × 2s 
ww = −


 s2 + 1
 2





s→s+1

w.E = −



 1−s 


2 

 s2 + 12 
 

a syE =
s→s+1
(s + 1)2 − 1
(s + 1)2 + 1 2


= ngi
s2 + 2s + 1 − 1
 =
s2 + 2s
 .
"
1 − cos t
# Z ∞
s2 + 2s + 1 + 1 2
nee s2 + 2s + 2 2

L
t
=
Z ∞
s
L [1 − cos t] ds
rin !
=
1
− 2
s s +1
s
ds
g.n
et
s
Z ∞ !
1 1 2s
= − ds
s s 2 s2 + 1
" #∞
1
= log s − log(s2 + 1)
2 s
" #∞
s
= log √
s2 + 1 s
 ∞
 
 s 
= log q 
 
s 1 + s12 
s
 ∞
 
 1 
= log q 
 1 + s12 

s

Downloaded From : www.EasyEngineering.net


———-
242
Downloaded Supplement to Engineering Mathematics - II
From : www.EasyEngineering.net

1
= log 1 − log q
1
1+ s2
 r 
 1 
= − log 1 − log 1 + 2 
s
r !
s2 + 1 1 s2 + 1
= log = log .
s2 2 s2
13. (a) (ii)( Apply convolution
) theorem to evaluate
2
−1 s
L .
(s2 + a2 )(s2 + b2 )
Solution.

ww L −1
"
s2
(s2 + a2 )(s2 + b2 )
#

w.E
= L−1 2
 s
.
2 2
s 
2
 s +s a  s + b  s 
a
= L−1 2
s + a2 syE
∗ L−1 2
s + b2
= cos at ∗ cos bt
Z t ngi
=
0
Z t
cos au cos b(t − u)du
nee
=
0
cos au cos(bt − bu)du
rin
=
1
Z t
{cos(au + bt − bu) + cos(au − bt + bu)}du g.n
=
2 0

2
"
1 sin(au + bt − bu) sin(au − bt + bu)
a−b
+
a+b
#t

0
et
" #
1 sin at sin bt sin at sin bt
= − + +
2 a−b a−b a+b a+b
" #
1 a+b+a−b a−b−a−b
= sin at + sin bt
2 a2 − b2 a2 − b2
1 a sin at − b sin bt
= [2a sin at − 2b sin bt] = .
2(a2 − b2 ) a2 − b2
13. (b) (i) Find
 the Laplace transform of




t , for 0 < t < a
f (t) = 
 f (t = 2a) = f (t).


2a − t , for a < t < 2a,

Downloaded From : www.EasyEngineering.net


———-
Anna University
Downloaded Solved Question Papers
From : www.EasyEngineering.net 243

Solution. Since f (t) is of period 2a,we have


Z 2a
1
L[ f (t)] = e−st f (t)dt
1 − e−2as 0
"Z a Z 2a #
1 −st −st
= e f (t)dt + e f (t)dt
1 − e−2as 0 a
1 hZ a Z 2a i
−st
= −2as
e tdt + e−st (2a − t)dt
1−e 0 a
h Z a Z 2a
1 e −st
e−st i
= −2as
td( )+ (2a − t)d( )
1−e 0 −s a −s
1 h e−st a Z a e−st  e−st 2a
Z 2a −st
e i

ww =
1−e
1
−2as
t
−s 0

h e−as 1  e−st a
0 −s
dt + (2a − t)
−s a
a −as 1  e−st 2a i

a −s
(−dt)

w.E =
1 − e−2as −s
1 h
a
e
+
−as
s −s 0
1
+0+ e −
s
a
s −s a
1
− 2 (e−as − 1) + e−as + 2 (e−2as − e−as )
i

a=

=
1−e
1
−2as

1 − e−2as
h
−a

syE
− a
s
e−as

s
1 −as 1
s2
e +
s2
+ a
s
e−as
+
s
1 −2as 1 −as i
s2
e − 2e

=
1
s
ngi
h e−2as − e−as − e−as + 1 i
=
s s
1 h 1 − 2e−2as + e−2as i

=
1 − e−2as
1
.
s2
(1 − e ) nee
−as 2
= 2
1 − e−as
−as
(1 − e )
s2

(1 − e−as )(1 + e−as )


as as
s2
 as  rin s .(1 + e−as )
e 2 − e− 2
=  as
s2 e 2 + e− 2
as
s
1
 = 2 tan h
2
.
g.n
13. (b) (ii) Use Laplace transform to solve (D2 − 3D + 2)y = e3t , with
et
y(0) = 1 and y′ (0) = 0.
d2 y dy
Solution. 2 − 3 + 2y = e3t .
dt dt
Taking Laplace transform both sides we get
" # " #
d2 y dy
L 2
− 3L + 2L[y] = L[e3t ]
dt dt
✟✟ − 3[sL[y] − y(0)] + 2L[y] = 1
s2 L[y] − sy(0) − ✟
y′ (0)
s−3
1
s2 L[y] − s − 3sL[y] + 3 + 2L[y] =
s−3

Downloaded From : www.EasyEngineering.net


———-
244
Downloaded Supplement to Engineering Mathematics - II
From : www.EasyEngineering.net

1
L[y][s2 − 3s + 2] = +s−3
s−3
1 + (s − 3)2 1 + s2 − 6s + 9 s2 − 6s + 10
L[y](s − 1)(s − 2) = = =
s−3 s−3 s−3
s2 − 6s + 10
L[y] = .
(s − 1)(s − 2)(s − 3)
" #
−1 s2 − 6s + 10
y=L .
(s − 1)(s − 2)(s − 3)

s2 − 6s + 10 A B C
Let = + +
(s − 1)(s − 2)(s − 3) s − 1 s − 2 s − 3

ww =
A(s − 2)(s − 3) + B(s − 1)(s − 3) + C(s − 1)(s − 2)
(s − 1)(s − 2)(s − 3)
.

w.E
∴ s2 − 6s + 10 = A(s − 2)(s − 3) + B(s − 1)(s − 3) + C(s − 1)(s − 2).
5
a
Puts = 1, 2A = 5 ⇒ A =
syE
Puts = 2, −B = 2 ⇒ B = −2.
2
.

Puts = 3, 2C = 1 ⇒ C =
1
ngi
.


s2 − 6s + 10
2
= ·
5 1 nee
+ −2 ·
1 1
+ ·
1
(s − 1)(s − 2)(s − 3) 2 s − 1
"
s2 − 6s + 10
#
rin s−2 2 s−3

Now, y = L −1

"
(s − 1)(s − 2)(s − 3)
# g.n
=L

5
−1 5
·
2 s−1
1

1
−2·
1
+ ·
s−2 2 s−3
1 1
et
y = et − 2e2t + e3t .
2 2

2.5.5 May/June 2014 [R 2013]


Part A
" #
sin t
5. Find L .
t
# Z∞
" Z∞ h i∞
sin t 1
Solution. L = L[sin t]ds = 2
]ds = tan−1 s
t s +1 s
s s

Downloaded From : www.EasyEngineering.net


———-
Anna University
Downloaded Solved Question Papers
From : www.EasyEngineering.net 245

π
= tan−1 (∞) − tan−1 (s) = − tan−1 (s) = cot−1 (s).
2
" #
−1 1
6. Evaluate L .
s2 + 6s + 13
" # " #
1 1
Solution. L−1 2 = L−1
s + 6s + 13 (s + 3)2 + 13 − 9
" #
1
= L−1
(s + 3)2 + 4
" #
1
= L−1
(s + 3)2 + 22
" #

ww = e−3t L−1 2
1
s +2 2
=
e−3t sin 2t
2
.

Part B w.E



a
13. (a) (i) Find
syE
 the Laplace transform of f (t), where
sin ωt , for 0 < t < ωπ

f (t) = 



0 π
, for ω < t < ω2π
ngi
and f (t + 2π
ω ) = f (t).

2π 2π nee
Solution. f (t) is defined in the interval (0, 2π ω)

ω ω
rin
f (t + ) = sin ω(t + ) = sin(ωt + 2π) = sin ωt = f (t)

∴ f (t) is periodic with period T =
ω
.
g.n
L[ f (t)] =
1
1 − e−sT 0
Z T

Z 2π
f (t)e−st dt et
1 ω
= s2π
e−st f (t)dt
− ω
1−e 0
Z π
1 ω
= e−st sin ωtdt
− s2π
1−e ω 0
" −st # ωπ
1 e
= s2π 2 2
[−s sin ωt − ω cos ωt]
1 − e− ω s + w 0
" −sπ #
1 eω −ω
= [−s.0 − ω(−1)] − 2
1 − e− ω s2 + w2 ω + s2
s2π

1 n −sπ o
=  ωe ω + ω
− s2π 2 2
1−e ω (s + ω )

Downloaded From : www.EasyEngineering.net


———-
246
Downloaded Supplement to Engineering Mathematics - II
From : www.EasyEngineering.net


ω(1 + e ω )
= −sπ −sπ
(s2 + ω2 )(1 − e ω )(1 + e ω )
ω
= −sπ .
(s2 + ω2 )(1 − e ω )

13. (a) (ii) Using convolution theorem find the inverse Laplace
s2
transform of 2 .
(s + a2 )(s2 + b2 )
Solution. Refer question 13. (a) (ii) of Nov./Dec.2014.

13. (b) (i) Find the Laplace transform of f (t) = te−3t cos 2t.
h i d
Solution. L te−3t cos 2t = − L[e−3t cos 2t]
ww ds
d  s 
!

w.E d
= − {L[cos 2t]} s→s+3 = −
ds
!
ds s2 + 4 s→s+3
=−
!
d s+3
ds (s + 3)2 + 4

=−
ad
2
s+3

syE
ds s + 6s + 9 + 4
=−
d
2
s+3
ds s + 6s + 13
(s2 + 6s + 13) · 1 − (s + 3)(2s + 6)
=−
(s2 + 6s + 13)2
ngi
s + 6s + 13 − (2s2 + 6s + 6s + 18)
nee
2
−s2 − 6s − 5
=− = −
(s2 + 6s + 13)2 (s2 + 6s + 13)2

= 2
2
s + 6s + 5
(s + 6s + 13)2
. rin
g.n
13. (b) (ii) Using Laplace transform, solve

y(0) = 3 and y (0) = 4.
d2 y
dt2
+ 4y = sin 2t, given et
d2 y
Solution. The given differential equation is 2 + 4y = sin 2t.
dt
Taking Laplace transform on both sides we get
" 2 #
d y
L + 4L[y] = L[sin 2t]
dt2
2
s2 L[y] − sy(0) − y′ (0) + 4L[y] = 2
s +4
2
s2 L[y] − 3s − 4 + 4L[y] = 2
s +4
2 2
L[y][s + 4] = 2 + 3s + 4
s +4

Downloaded From : www.EasyEngineering.net


———-
Anna University
Downloaded Solved Question Papers
From : www.EasyEngineering.net 247

2 + (3s + 4)(s2 + 4)
L[y] =
(s2 + 4)2
2 + 3s3 + 12s + 4s2 + 16
=
(s2 + 4)2
3s + 4s2 + 12s + 18
3
= .
(s2 + 4)2
" 3 #
3s + 4s2 + 12s + 18
y = L−1 .
(s2 + 4)2
3s3 + 4s2 + 12s + 18 As + B Cs + D
Let 2 2
= 2 + 2
(s + 4) s +4 (s + 4)2
∴ 3s3 + 4s2 + 12s + 18 = (As + B)(s2 + 4) + (Cs + D).
ww Equating the coefficients of s3 we get
A = 3.w.E
B = 4.
a
Equating the coefficients of s2 we get
syE
Equating the coefficients of s we get
ngi
4A + C = 12 ⇒ 12 + C = 12 ⇒ C = 0.
nee
Equating the constants we get
rin
4B + D = 18
3 2
⇒ 16 + D = 18 ⇒ D = 2.
g.n

3s + 4s + 12s + 18 3s + 4
(s2 + 4)2
"
−1 3s + 4
#
= 2

−1
+
s + 4 (s2 + 4)2
"
1
#
2
et
∴y=L + 2L
s2 + 4 (s2 + 4)2
 s  " # " #
−1 −1 1 −1 1
= 3L + 4L + 2L
s2 + 4 s2 + 4 (s2 + 4)2
sin 2t 1
= 3 cos 2t + 4 +2· [sin 2t − 2t cos 2t]
2 2 × 23
" " # #
−1 2a3
∵L = sin at − at cos at
(s2 + a2 )2
1
y = 3 cos 2t + 2 sin 2t + [sin 2t − 2t cos 2t].
8

Downloaded From : www.EasyEngineering.net


———-
248
Downloaded Supplement to Engineering Mathematics - II
From : www.EasyEngineering.net

2.5.6 November/December 2013[R 2008]

Part A

1 − et
5. Find the Laplace transform of the function f ( f ) = .
t
" # Z ∞ Z ∞ !
1 − et t 1 1
Solution. L = L[1 − e ]ds = − ds
t s s s s−1
   s ∞ 
= log s − log(s − 1) ∞ s = log
s−1 s
 
 1 ∞
ww = log   = log 1 − log(
1 − 1s s
s
s−1
)

w.E = log
s−1
s
!
.

a syE 


ngi
6. Find the Laplace transform of the function f (t) = 


1 t = 0


Solution. L[1] =
Z ∞
e−st × 1dt =
" −st #∞
e nee
−1
0 t , 0
1
[0 − 1] = .
0 −s 0
=
rin
s s

Part B g.n
13. (a) (i) Find L−1
"
3s2 + 16s + 26
s(s2 + 4s + 13)
#
.
et
3s2 + 16s + 26 A Bs + c
Solution. Let = + 2
s(s2 + 4s + 13) s s + 4s + 13
3s2 + 16s + 26 = A(s2 + 4s + 13) + (Bs + c)s.
When s = 0, 13A = 26 ⇒ A = 2.
Equating the coefficients of s2
A+B=3
2+B=3
B = 1.
Equating the coefficients of s

Downloaded From : www.EasyEngineering.net


———-
Anna University
Downloaded Solved Question Papers
From : www.EasyEngineering.net 249

4A + c = 16
8 + c = 16
c = 8.

3s2 + 16s + 26 2 s+8


∴ = + 2 .
s(s2 + 4s + 13) s s + 4s + 13
" 2 # " #
3s + 16s + 26 −1 2 s+8
L−1 = L +
s(s2 + 4s + 13) s s2 + 4s + 13
" # " #
1 s+8
= 2L−1 + L−1 2
s s + 4s + 13
" #

ww = 2 · 1 + L−1
"
s+2+6
(s + 2)2 + 13 − 4
#
w.E = 2 + L−1
s+2+6
(s + 2)2 + 9
" #
a syE
= 2 + e−2t L−1 2
"
s+6
s +9
 s  !#

ngi
= 2 + e−2t L−1 2
s +9
+ 2L−1 2
3
s +9

nee
= 2 + e−2t [cos 3t + 2 sin 3t].

−1
"
s+1
#
rin
13. (a) (ii) Find L log
s−1
.
s+1 g.n
Solution. Let F(s) = log

F ′ (S ) =
1

s+1 s−1
s−1
1
= log(s + 1) − log(s − 1)
et
1 1
L[t f (t)] = −F ′ (s) = − +
" s + 1 # s − 1 " # " #
1 1 1 1
∴ t f (t) = L −1
− =L −1
−L −1
= et − e−t
s−1 s+1 s−1 s+1
et − e−t
∴ f (t) = .
t
" # " #
s 1
13. (b) (i) Find L−1 2 and find L −1
and hence find
" (s +# a2 )2 (s2 + a2 )2
1
L−1 2 .
(s + 9s + 13)2

Downloaded From : www.EasyEngineering.net


———-
250
Downloaded Supplement to Engineering Mathematics - II
From : www.EasyEngineering.net

Solution.
" #
s t sin at
L−1 2 =
(s + a2 )2 2a
" # " #
1 −1 1 s
L−1 2 = L
(s + a2 )2 s (s2 + a2 )2
" #
F(s) s
= L−1 where F(s) = 2
s (s + a2 )2
Z t " #
s
= L−1 2 dt
0 (s + a2 )2
Z t Z t 
t sin at 1 − cos at 
= dt = td

ww =
0
"
2a
1 −t cos at  t
2a 0


Z t
a
− cos at 
dt
#

w.E =
2a
"
a
1 −t cos at 1 sin at
+
0 0 a
!t #
=
"
1 −t cos at 1
+ 2 sin at
#

a 2a
syE
1
a

= 3 [sin at − at cos at] .


2a
a a 0 2a a a

"
1
# "
ngi 1
#
L−1 2
nee
−1
= L
(s + 9s + 13)2 [(s + 3)2 + 13 − 9]2
" #
=e L−3t −1 1
(s2 + 4)2
, which is of the form 2rin 1
(s + a2 )2
with a = 2.

= e−3t
"
1
(sin 2t − 2t cos 2t)
#
=
e−3t g.n
[sin 2t − 2t cos 2t].
2 × 23 16

2
d y dy
et
13. (b) (ii) Solve using Laplace transforms, + = t2 + 2t, given
dt2 dt
that y = 4, y′ = −2 when t = 0.
d2 y dy
Solution. The given differential equation is + = t2 + 2t.
dt2 dt
Taking Laplace transform on both sides, we get
2 2
s2 L[y] − sy(0) − y′ (0) + sL(y) − y(0) − y(0) = +
s3 s2
2(s + 1)
(s2 + s)L[y] − 4s + 2 − 4 =
s3
2(s + 1) 2(s + 1)
s(s + 1)L(y) = 4s + 2 + = 2s + 2 + 2s +
s3 s3

Downloaded From : www.EasyEngineering.net


———-
Anna University
Downloaded Solved Question Papers
From : www.EasyEngineering.net 251

2(s + 1)
= 2(s + 1) + 2s +
s3
2 2 2
L(y) = + +
s" s + 1 s4 #
2 2 2 2
y = L−1 + + 4 = 2 + 2e−t + t3
s s+1 s 3!
−t 1 3
y = 2 + 2e + t .
3

2.5.7 May/June 2013 [R 2008]

Part A

ww t
5. Find the Laplace transform of t .
e
w.E t
Solution. L t = L[e−t t] = [L(t)] s→s+1 =
e
1
(s + 1)2
.

a syE
6. Verify the initial and final value theorems for the function
f (t) = ae−bt .
Solution. Let f (t) = ae−bt .
a
ngi
L[ f (t)] =
s+b
= F(s).
nee
lim f (t) = lim ae−bt = a.
t→0 t→0
rin
lim sF(s) = lim
s→∞ s→∞
as
s+b
= lim
s→∞
as
s(1 + bs )
= a.
g.n
∴ lim f (t) = lim sF(s).
t→0 s→∞
Hence, the initial value theorem is verified.
et
Part B

13. (a) (i) Find L[te−3t sin 2t].

−d h −3t i
Solution. L[te−3t sin 2t] = L[e sin 2t]
ds " #
−d −d 2
= [L[sin 2t] s→s+3 ] =
ds ds s2 + 4 s→s+3
" #
−d 2
=
ds (s + 3)2 + 4

Downloaded From : www.EasyEngineering.net


———-
252
Downloaded Supplement to Engineering Mathematics - II
From : www.EasyEngineering.net
" #
−d 2
=
ds s2 + 6s + 9 + 4
" #
−d 2
=
ds s2 + 6s + 13
d
= −2 (s2 + 6s + 13)−1
ds
h i
= −2 (−1)(s2 + 6s + 13)−2 (2s + 6)
" # " #
2s + 6 s+3
=2 2 = 4 .
(s + 6s + 13)2 (s2 + 6s + 13)2

ww
13. (a) (ii) Find the Laplace transform of the square-wave function
(or Meoander function) of period a defined as
w.E




1

f (t) = 

when 0 < t <
a
2


a a
−1 when < t < a.
2
syE
Solution. Since f (t) is a periodic function of period a, we have

1
Za ngi
L[ f (t)] =
1 − e−as
0
e−st f (t)dt
nee
1
 a
Z2

Za


 rin
=
1 − e−as
 e f (t)dt + e f (t)dt

−st −st
 g.n
et
0 a
2
 a 
Z2 Za 
1  −st −st


=  e dt + e (−1)dt
1 − e−as  
0 a
2
 !a ! 
 e−st 2 −st a 
1  e 
= − 
1 − e−as  −s 0 −s a 
2

1 h  a  i
−2s −as − a2 s
= − e − 1 + e − e
s(1 − e−as )
1 h a i
= −as
1 − 2e− 2 s + e−as
s(1 − e )
 2
1 − e− 2 s ✄
a

  
−✘
=
1 −✘e✘
s ✘
a a
2s 1 + e− 2 s

Downloaded From : www.EasyEngineering.net


———-
Anna University
Downloaded Solved Question Papers
From : www.EasyEngineering.net 253

a
1 − e− 2 s
as
1 e 4 − e− 4
as
1  as 
=  a
 = · as as = tan h .
s 1 + e− 2 s s e 4 + e− 4 s 4
" #
4
13. (b) (i) Using convolution theorem, find L−1 .
(s2 + 2s + 5)2
Solution.
" # " # " #
−1 4 −1 4 −t −1 4
L =L =e L (1)
(s2 + 2s + 5)2 (s + 1)2 + 4)2 (s2 + 4)2
" # " #
−1 4 −1 2 2
Now, L =L
(s2 + 4)2 s2 + 4 s2 + 4
" # " #
−1 2 −1 2
=L ∗L

ww s2 + 4

= sin 2t ∗ sin 2t =
Z t
s2 + 4

sin 2u sin 2(t − u)du


w.E Z t
0

a syE
=

=
"Z0 t
(cos{2u − 2(t − u)} − cos{2u + 2t − 2u}) du

cos(4u − 2t)du −
Z t
cos 2tdu
#

"
0
ngi
sin(4u − 2t)
!t
− cos 2t(u)t0
#
0

"
=

sin 2t sin 2t
4 0
nee
#
=
4
+
4
− t cos 2t
rin
=
"
sin 2t
#
− t cos 2t . g.n
"
(1) ⇒ L−1 2
1
2

(s + +2s + 5)2
#
1
= [sin 2t − 2t cos 2t] e−t .
2
et
13. (b) (ii) Solve y′′ + 5y′ + 6y = 2 given y′ (0) = 0 and y(0) = 0 using
Laplace transform method.
Solution. y′′ + 5y′ + 6y = 2.
Taking Laplace transform both sides we get

L[y′′ ] + 5L[y′ ] + 6L[y] = L[2]


s2 L[y] − sy(0) − y′ (0)+5[sL[y] − y(0)] + 6L[y] = 2 · L[1]
2
L[y][s2 + 5s + 6] =
s

Downloaded From : www.EasyEngineering.net


———-
254
Downloaded Supplement to Engineering Mathematics - II
From : www.EasyEngineering.net

2
L[y] =
s(s2 + 5s + 6)
2
=
s(s + 2)(s + 3)
" #
2
y= L−1
s(s + 2)(s + 3)
2 A B C
Let = + +
s(s + 2)(s + 3) s s+2 s+3
A(s + 2)(s + 3) + Bs(s + 3) + Cs(s + 2)
=
s(s + 2)(s + 3)
∴ 2 = A(s + 2)(s + 3) + Bs(s + 3) + Cs(s + 2).

ww When s = 0, 6A = 2 ⇒ A =
1
3
.

w.E
When s = −2, −2B = 2 ⇒ B = −1.
2
a
When s = −3, 3C = 2 ⇒ C =


2syE 1 1
= · −
3
1
.

+ ·
2 1

Now y = L−1
ngi
s(s + 2)(s + 3) 3 s s + 2 3 s + 3
"
2
#

"
1 1
nee
s(s + 2)(s + 3)
1 2 1
#
= L−1 · −
3 s s+2 3 s+3rin
+ ·

1
= L−1
" #
1
− L−1
"
1
#
2
+ L−1g.n
"
1
#

3
1
s

= × 1 − e−2t + e−3t
3
2
3
s+2 3 s+3
et
1 −2t 2 −3t
y= −e + e .
3 3

2.5.8 November/December 2012 [R 2008]

Part A
"#
1 − cos t
5. Is the linear property applicable to L ?
t
Solution. Linear property is applicable for continuous
1 − cos t
functions only. Here is not even defined at t = 0.
t

Downloaded From : www.EasyEngineering.net


———-
Anna University
Downloaded Solved Question Papers
From : www.EasyEngineering.net 255

Hence, for this function, linearity property is not applicable.


1
6. Find the inverse Laplace transform of .
" # (s + 1)(s + 2)
1
Solution. L−1
" (s + 1)(s + 2) #
s + 2 − (s + 1)
= L−1
" (s + 1)(s + 2) #
−1 s+2 s+1
=L −
" (s + 1)(s + 2) # (s + 1)(s + 2)
1 1
= L−1 −
" s + 1 # s + 2" #
1 1
ww =L −1
s+1
= e−t − e−2t .
−L −1
s+2

Part B w.E " #


a
13. (a) (i) Find L−1 2
s2

s
2
"(s + 24) # syE
using Convolution theorem.
 s s 
Solution. L−1 2
 s 
(s + 4)2
=
 s 
L −1

ngi
·
s2 + 4 s2 + 4

= L−1 2
s +4
∗ L−1 2
s +4 nee
= cos 2t ∗ cos 2t
rin
=
Zt
cos 2u · cos 2(t − u)du g.n
0
Zt
et
= cos 2u cos(2t − 2u)du
0
Zt !
cos(2u + 2t − 2u) + cos(2u − 2t + 2u)
= du
2
0
Zt
1
= (cos 2t + cos[4u − 2t])du
2
0
 
 Zt Zt 
1  
= cos 2t du + cos(4u − 2t)du
2 
0 0

Downloaded From : www.EasyEngineering.net


———-
256
Downloaded Supplement to Engineering Mathematics - II
From : www.EasyEngineering.net

" " #t #
1 sin(4u − 2t)
= cos 2t[u]t0 +
2 4 0
" #
1 1
= t cos 2t + {sin 2t − sin(−2t)}
2 4
" #
1 1
= t cos 2t + 2 sin 2t
2 4
" #
1 sin 2t
= t cos 2t +
2 2
1
= (sin 2t + 2t cos 2t).
4

ww
13. (a) (ii) Find the Laplace transform




 sin ωt
of the half-sine wave rectifier
0 < t < ωπ

w.E
function defined by f (t) = 


 0 π
< t < 2π .
ω ω
Solution. Refer question 13. (a) (i) on of May/June 2014.

a
13. (b) (i) Find L
"

" t syE
cos at − cos bt
#
.
# Z ∞
Solution. L
cos at − cos bt
t
= ngi L [cos at − cos bt] ds

nee
s
Z ∞
s s 
= − ds
s2 + a2 s2 + b2
1h
s

rin
log(s2 + a2 ) − log(s2 + b2 )
i∞
=
2
1
" 2
s +a
#
2 ∞
1 g.n s

s + b2
2

d x 2
dx
=
2
log 2
s +b s2

dx
= log 2
2 s + a2
.
et
13. (b) (ii) Solve − 3 + 2x = 2 given x = 0 and = 5 for t = 0
dt2 dt dt
using Laplace transform method.
d2 x dx
Solution. 2 − 3 + 2x = 2.
dt dt
Taking Laplace transform both sides we get
" 2 # " #
d x dx
L 2
− 3L + 2L[x] = L[2]
dt dt

s2 L[x] − s✟ ✟ − x′ (0) − 3[sL[x] − ✟


x(0) ✟ + 2L[x] = 2
x(0)]
s
2 2
s L[x] − 5 − 3sL[x] + 2L[x] =
s

Downloaded From : www.EasyEngineering.net


———-
Anna University
Downloaded Solved Question Papers
From : www.EasyEngineering.net 257

2 5s + 2
L[x][s2 − 3s + 2] = +5=
s s
5s + 2 5s + 2
L[x] = =
s(s2 − 3s + 2) s(s − 1)(s − 2)
" #
−1 5s + 2
x=L .
s(s − 1)(s − 2)

5s + 2 A B C
Let = + +
s(s − 1)(s − 2) s s−1 s−2
A(s − 1)(s − 2) + Bs(s − 2) + Cs(s − 1)
=
s(s − 1)(s − 2)

ww 5s + 2 = A(s − 1)(s − 2) + Bs(s − 2) + Cs(s − 1).

w.E
When s = 0, 2A = 2 ⇒ A = 1.
When s = 1, −B = 7 ⇒ B = −7.

a syE
When s = 2, 2C = 12 ⇒ C = 6.
5s + 2 1 7 6

s(s − 1)(s − 2)
= −
ngi
s s−1 s−2
"
+ .
#
Now, x = L −1

"
nee
5s + 2
s(s − 1)(s − 2)
#
=L −1 1

7
rin
+
6

−1 1
s s−1 s−2
" #
−1
"
1
#
g.n
−1
"
1
#
=L
s
− 7L

x = 1 − 7et + 6e2t .
s−1
+ 6L
s−2
et
2.5.9 May/June 2012 [R 2008]

Part A

5. State the first shifting theorem on Laplace transforms.


Statement: If L[ f (t)] = F(s), then L[e−at f (t)] = F(s + a).

6. Verify the initial value theorem for the function


f (t) = 1 + e−t (sin t + cos t).

Downloaded From : www.EasyEngineering.net


———-
258
Downloaded Supplement to Engineering Mathematics - II
From : www.EasyEngineering.net

Solution.

Let f (t) = 1 + e−t (sin t + cos t).


L[ f (t)] = L[1 + e−t (sin t + cos t)]
= L[1] + L[e−t (sin t + cos t)]
1
=+ L[sin t + cos t] s→s+1
s " #
1 1 s
= +L 2 +
s s + 1 s2 + 1 s→s+1
" #
1 s+1

ww = + 2
s
1
s + 1 s→s+1
s+2

w.E F(s) = +
s (s + 1)2 + 1
lim f (t) = lim[1 + e−t (sin t + cos t)]
a t→0

syE "
t→0

= 1 + 1 = 2.
#
lim sF(s) = lim 1 +
s→∞ s→∞ ngi
s(s + 2)
(s + 1)2 + 1

= 1 + lim nee
s2 (1 + 2s )
=1+
1
= 2.
s→∞ s2 (1 + 1 2
s)
rin
+ 1
s2
1

∴ lim f (t) = lim sF(s).


t→0 s→∞
g.n
Part B
" #
et
s
13. (a) (i) Find L−1 using convolution theorem.
(s2 + a2 )2
Solution.

s s 1
Let = 2 . .
(s2 + a2 )2 s + a2 s2 + a2
s 1
∴ F(s) = 2 2
, G(s) = 2 .
" # s +a s + a2
s
L−1 2 = L−1 [F(s)G(s)] = L−1 F(s) ∗ L−1G(s)
(s + a2 )2
" # " #
s 1 sin at
= L−1 2 ∗ = cos at ∗
(s + a2 ) (s2 + a2 ) a

Downloaded From : www.EasyEngineering.net


———-
Anna University
Downloaded Solved Question Papers
From : www.EasyEngineering.net 259

Z
1 t
= cos au sin a(t − u)du
a 0
Z t
1
= 2 sin(at − au) cos audu
2a 0
Z tn o
1
= sin(at − au + au) + sin(at − au − au) du
2a 0
(Z t Z t )
1
= sin atdu + sin(at − 2au)du
2a 0 0
" " #t #
1 cos(at − 2au)
= sin at(u)t0 −
2a −2a 0
" #
1 1

ww =
2a
1
"
t sin at + (cos(−at) − cos at)
2a
1
#
t sin at
w.E =
2a
t sin at − 0 =
2a 2a
.

a 

t

syE
13. (a) (ii) Find the Laplace transform of the triangular wave

0≤t≤π
function given by f (t) = 



2π − t π ≤ t ≤ 2π ngi
and f (t + 2π) = f (t).
Solution. Since f (t) is of period 2π,we have
nee
Z 2π rin
L[ f (t)] =
1
1 − e−2πs 0
e−st f (t)dt
g.n
=
1
1 − e−2πs 0
"Z π

"Z π
−st
e f (t)dt +
Z 2π
Z 2π

π
−st
e f (t)dt
#

#
et
1 −st −st
= e tdt + e (2π − t)dt
1 − e−2πs 0 π
"Z π −st Z 2π #
1 e e−st
= td( ) + (2π − t)d( )
1 − e−2πs 0 −s π −s
 !π Z π −st !2π Z 2π −st 
1  e −st
e e−st e 
= −2πs

 t − dt + (2π − t) − (−dt)
1−e −s 0 0 −s −s π π −s
 !π !2π 
1  e −πs
1 e −st
π 1 e −st 
= π + + 0 + e−πs − 
1−e −2πs −s s −s 0 s s −s π 
" #
1 e−πs 1 −πs π −πs 1 −2πs −πs
= −π − (e − 1) + e + (e − e )
1 − e−2πs s s2 s s2

Downloaded From : www.EasyEngineering.net


———-
260
Downloaded Supplement to Engineering Mathematics - II
From : www.EasyEngineering.net
" #
1 e−πs 1 −πs 1 e−πs 1 −2πs 1 −πs
= −π − e + + π + e − e
1 − e−2πs s s2 s2 s s2 s2
" −2πs # " #
1 e − e−πs − e−πs + 1 1 1 − 2e−2πs + e−2πs
= =
1 − e−2πs s2 1 − e−2πs s2
1 (1 − e−πs )2 (1 − e−πs )
= −πs −πs
. 2
= 2
(1 − e )(1 + e ) s s .(1 + e−πs )
πs
e 2 − e− 2
πs
1  πs 
=  πs  = 2 tan h .
2 − πs s 2
s e +e 2 2

eat − e−bt
13. (b) (i) Find the Laplace transform of .
t
ww "
eat − e−bt
# Z∞ h i Z∞ !

w.E
Solution. L
t
=
s
at
L e −e −bt
ds =
s
1

s−a s+b

1

s − a ∞
ds

a 
syE
= log(s − a) − log(s + b) ∞
s+b

s = log
s+b s
= log
s−a
.
ngi
13. (b) (ii) Find
R∞ nee
te−2t cos tdt using Laplace transform .
0
rin
Solution. Comparing the definition of the Laplace transform
and the given integral, we observe that g.n
Z∞
te−2t cos tdt = L[t cos t] s=2 = −
d
ds
[L(cos t)] s=2
et
0
" 2 #
d  s  s + 1 − s · 2s
=− = −
ds s2 + 1 s=2 (s2 + 1)2 s=2
" # " #
1 − s2 s2 − 1 4−1 3
=− 2 = = = .
(s + 1)2 s=2 (s2 + 1)2 s=2 (4 + 1)2 25

d2 y dy
13. (b) (iii) Solve the differential equation − 3 + 2y = e−t with
dt2 dt
y(0) = 1, and y′ (0) = 0 using Laplace transforms.
d2 y dy
Solution. 2 − 3 + 2y = e−t .
dt dt

Downloaded From : www.EasyEngineering.net


———-
Anna University
Downloaded Solved Question Papers
From : www.EasyEngineering.net 261

Taking Laplace transform both sides we get


" 2
# " #
d y dy
L 2
− 3L + 2L[y] = L[e−t ]
dt dt

s2 L[y] − sy(0) − ✟ ✟✟ − 3[sL[y] − y(0)] + 2L[y] = 1


y′ (0)
s+1
1
s2 L[y] − s − 3sL[y] + 3 + 2L[y] =
s+1
1
L[y][s2 − 3s + 2] = +s−3
s+1
1 + (s − 3)(s + 1)
L[y](s − 1)(s − 2) =
s+1

ww =
1 + s2 − 2s − 3 s2 − 2s − 2
s+1
=
s+1
w.E L[y] =
s2 − 2s − 2
(s + 1)(s − 1)(s − 2)
.

a y=
syE
L −1
"
s2 − 2s − 2
(s + 1)(s − 1)(s − 2)
#
.

Let
s2 − 2s − 2
=
A ngi
+
B
+
C

nee
(s + 1)(s − 1)(s − 2) s + 1 s − 1 s − 2
A(s − 1)(s − 2) + B(s + 1)(s − 2) + C(s + 1)(s − 1)
.
=
rin
(s + 1)(s − 1)(s − 2)

g.n
∴ s2 − 2s − 2 = A(s − 1)(s − 2) + B(s + 1)(s − 2) + C(s + 1)(s − 1).
3
When s = 1, −2B = −3 ⇒ B =

When s = −1, 6A = 1 ⇒ A = .
2
1
.
et
6
2
When s = 2, 3C = −2 ⇒ C = − .
3
s2 − 2s − 2 1 1 3 1 2 1
∴ = · + · − ·
(s + 1)(s − 1)(s − 2) 6 s + 1 2 s − 1 3 s − 2
" #
−1 s2 − 2s − 2
Now, y = L
(s + 1)(s − 1)(s − 2)
" #
1 1 3 1 2 1
= L−1 · + · − ·
6 s+1 2 s−1 3 s−2
1 −t 3 t 2 2t
y= e + e − e .
6 2 3

Downloaded From : www.EasyEngineering.net


———-
Downloaded From : www.EasyEngineering.net

ww
w.E
a syE
ngi
nee
rin
g.n
et

**Note: Other Websites/Blogs Owners Please do not Copy (or) Republish


this Materials, Students & Graduates if You Find the Same Materials with
EasyEngineering.net Watermarks or Logo, Kindly report us to
easyengineeringnet@gmail.com
Downloaded From : www.EasyEngineering.net

You might also like